You are on page 1of 561

Nursing Board Practice Test Compilation

FOUNDATION OF PROFESSIONAL NURSING PRACTICE188

Contents ANSWER KEY - FOUNDATION OF PROFESSIONAL


NURSING PRACTICE I: FOUNDATION OF NURSING NURSING PRACTICE.................................................199
PRACTICE.........................................................................4 COMMUNITY HEALTH NURSING AND CARE OF THE
NURSING PRACTICE II....................................................15 MOTHER AND CHILD....................................................200

NURSING PRACTICE III...................................................26 ANSWER KEY: COMMUNITY HEALTH NURSING AND


CARE OF THE MOTHER AND CHILD..........................211
NURSING PRACTICE IV...................................................36
Comprehensive Exam 1...............................................213
NURSING PRACTICE V....................................................46
CARE OF CLIENTS WITH PHYSIOLOGIC AND
TEST I - Foundation of Professional Nursing Practice....56
PSYCHOSOCIAL ALTERATIONS.....................................222
Answers and Rationale – Foundation of Professional
ANSWER KEY: CARE OF CLIENTS WITH PHYSIOLOGIC
Nursing Practice.........................................................66
AND PSYCHOSOCIAL ALTERATIONS.........................234
TEST II - Community Health Nursing and Care of the
Nursing Practice Test V................................................235
Mother and Child...........................................................74
Nursing Practice Test V................................................245
Answers and Rationale – Community Health Nursing
and Care of the Mother and Child.............................84 TEST I - Foundation of Professional Nursing Practice...255

TEST III - Care of Clients with Physiologic and Answers and Rationale – Foundation of Professional
Psychosocial Alterations................................................91 Nursing Practice.......................................................265

Answers and Rationale – Care of Clients with TEST II - Community Health Nursing and Care of the
Physiologic and Psychosocial Alterations.................102 Mother and Child.........................................................273

TEST IV - Care of Clients with Physiologic and Answers and Rationale – Community Health Nursing
Psychosocial Alterations..............................................111 and Care of the Mother and Child...........................283

Answers and Rationale – Care of Clients with TEST III - Care of Clients with Physiologic and
Physiologic and Psychosocial Alterations.................122 Psychosocial Alterations..............................................290

TEST V - Care of Clients with Physiologic and Psychosocial Answers and Rationale – Care of Clients with
Alterations...................................................................133 Physiologic and Psychosocial Alterations.................301

Answers and Rationale – Care of Clients with TEST IV - Care of Clients with Physiologic and
Physiologic and Psychosocial Alterations.................144 Psychosocial Alterations..............................................310

PART III PRACTICE TEST I FOUNDATION OF NURSING .153 Answers and Rationale – Care of Clients with
Physiologic and Psychosocial Alterations.................321
ANSWERS AND RATIONALE – FOUNDATION OF
NURSING..................................................................158 TEST V - Care of Clients with Physiologic and Psychosocial
Alterations...................................................................332
PRACTICE TEST II Maternal and Child Health...............162
Answers and Rationale – Care of Clients with
ANSWERS AND RATIONALE – MATERNAL AND CHILD
Physiologic and Psychosocial Alterations.................343
HEALTH....................................................................167
PART III........................................................................352
MEDICAL SURGICAL NURSING.....................................173
PRACTICE TEST I FOUNDATION OF NURSING...............352
ANSWERS AND RATIONALE – MEDICAL SURGICAL
NURSING..................................................................178 ANSWERS AND RATIONALE – FOUNDATION OF
NURSING..................................................................357
PSYCHIATRIC NURSING................................................180
PRACTICE TEST II Maternal and Child Health...............361
ANSWERS AND RATIONALE – PSYCHIATRIC NURSING
................................................................................. 185
ANSWERS AND RATIONALE – MATERNAL AND CHILD MEDICAL SURGICAL NURSING Part 1...........................475
HEALTH....................................................................366
ANSWERS and RATIONALES for MEDICAL SURGICAL
MEDICAL SURGICAL NURSING.....................................372 NURSING Part 1.......................................................479
ANSWERS AND RATIONALE – MEDICAL SURGICAL MEDICAL SURGICAL NURSING Part 2...........................481
NURSING..................................................................377
MEDICAL SURGICAL NURSING Part 2.......................485
PSYCHIATRIC NURSING................................................379
ANSWERS and RATIONALES for MEDICAL SURGICAL
ANSWERS AND RATIONALE – PSYCHIATRIC NURSING NURSING Part 2.......................................................489
................................................................................. 384
MEDICAL SURGICAL NURSING Part 3...........................491
FUNDAMENTALS OF NURSING PART 1........................387
ANSWERS and RATIONALES for MEDICAL SURGICAL
FUNDAMENTALS OF NURSING PART 2........................392 NURSING Part 3.......................................................495
ANSWERS and RATIONALES for FUNDAMENTALS OF PSYCHIATRIC NURSING Part 1......................................497
NURSING PART 2......................................................397
ANSWERS and RATIONALES for PSYCHIATRIC NURSING
FUNDAMENTALS OF NURSING PART 3........................401 Part 1.......................................................................502
ANSWERS and RATIONALES for FUNDAMENTALS OF PSYCHIATRIC NURSING Part 2......................................504
NURSING PART 3......................................................405
ANSWERS and RATIONALES for PSYCHIATRIC NURSING
MATERNITY NURSING Part 1.......................................409 Part 2.......................................................................509
ANSWERS and RATIONALES for MATERNITY NURSING PSYCHIATRIC NURSING Part 3......................................512
Part 1.......................................................................418
ANSWERS and RATIONALES for PSYCHIATRIC NURSING
MATERNITY NURSING Part 2.......................................428 Part 3.......................................................................516
Answer for maternity part 2....................................433 PROFESSIONAL ADJUSTMENT......................................519
PEDIATRIC NURSING....................................................434 LEADERSHIP and MANAGEMENT.................................522
ANSWERS and RATIONALES for PEDIATRIC NURSING NURSING RESEARCH Part 1..........................................532
................................................................................. 439
NURSING RESEARCH Part 2..........................................542
COMMUNITY HEALTH NURSING Part 1........................444
Nursing Research Suggested Answer Key................546
COMMUNITY HEALTH NURSING Part 2........................454

2
3
5. Benner’s “Proficient” nurse level is different
from the other levels in nursing expertise in the
NURSING PRACTICE I: FOUNDATION OF NURSING context of having:
PRACTICE a. the ability to organize and plan activities
b. having attained an advanced level of
SITUATION: Nursing is a profession. The nurse should education
have a background on the theories and foundation of c. a holistic understanding and perception
nursing as it influenced what is nursing today. of the client
d. intuitive and analytic ability in new
1. Nursing is the protection, promotion and situations
optimization of health and abilities, prevention
of illness and injury, alleviation of suffering SITUATION: The nurse has been asked to administer an
through the diagnosis and treatment of human injection via Z TRACK technique. Questions 6 to 10 refer
response and advocacy in the care of the to this.
individuals, families, communities and the
population. This is the most accepted definition 6. The nurse prepares an IM injection for an adult
of nursing as defined by the: client using the Z track technique. 4 ml of
a. PNA medication is to be administered to the client.
b. ANA Which of the following site will you choose?
c. Nightingale a. Deltoid
d. Henderson b. Rectus femoris
c. Ventrogluteal
2. Advancement in Nursing leads to the d. Vastus lateralis
development of the Expanded Career Roles.
Which of the following is NOT an expanded 7. In infants 1 year old and below, which of the
career role for nurses? following is the site of choice for intramuscular
a. Nurse practitioner Injection?
b. Nurse Researcher a. Deltoid
c. Clinical nurse specialist b. Rectus femoris
d. Nurse anaesthesiologist c. Ventrogluteal
d. Vastus lateralis
3. The Board of Nursing regulated the Nursing
profession in the Philippines and is responsible 8. In order to decrease discomfort in Z track
for the maintenance of the quality of nursing in administration, which of the following is
the country. Powers and duties of the board of applicable?
nursing are the following, EXCEPT: a. Pierce the skin quickly and smoothly at
a. Issue, suspend, revoke certificates of a 90 degree angle
registration b. Inject the medication steadily at around
b. Issue subpoena duces tecum, ad 10 minutes per millilitre
testificandum c. Pull back the plunger and aspirate for 1
c. Open and close colleges of nursing minute to make sure that the needle did
d. Supervise and regulate the practice of not hit a blood vessel
nursing d. Pierce the skin slowly and carefully at a
90 degree angle
4. A nursing student or a beginning staff nurse who
has not yet experienced enough real situations 9. After injection using the Z track technique, the
to make judgments about them is in what stage nurse should know that she needs to wait for a
of Nursing Expertise? few seconds before withdrawing the needle and
a. Novice this is to allow the medication to disperse into
b. Newbie the muscle tissue, thus decreasing the client’s
c. Advanced Beginner discomfort. How many seconds should the nurse
d. Competent wait before withdrawing the needle?
a. 2 seconds

4
5

b. 5 seconds that the patient smokes and drinks coffee. When


c. 10 seconds taking the blood pressure of a client who
d. 15 seconds recently smoked or drank coffee, how long
should the nurse wait before taking the client’s
10. The rationale in using the Z track technique in an blood pressure for accurate reading?
intramuscular injection is: a. 15 minutes
a. It decreases the leakage of b. 30 minutes
discolouring and irritating medication c. 1 hour
into the subcutaneous tissues d. 5 minutes
b. It will allow a faster absorption of the
medication 15. While the client has pulse oximeter on his
c. The Z track technique prevent irritation fingertip, you notice that the sunlight is shining
of the muscle on the area where the oximeter is. Your action
d. It is much more convenient for the nurse will be to:
a. Set and turn on the alarm of the
SITUATION: A Client was rushed to the emergency room oximeter
and you are his attending nurse. You are performing a b. Do nothing since there is no identified
vital sign assessment. problem
c. Cover the fingertip sensor with a towel
11. All of the following are correct methods in or bedsheet
assessment of the blood pressure EXCEPT: d. Change the location of the sensor every
a. Take the blood pressure reading on both four hours
arms for comparison
b. Listen to and identify the phases of 16. The nurse finds it necessary to recheck the blood
Korotkoff’s sound pressure reading. In case of such re assessment,
c. Pump the cuff to around 50 mmHg the nurse should wait for a period of:
above the point where the pulse is a. 15 seconds
obliterated b. 1 to 2 minutes
d. Observe procedures for infection control c. 30 minutes
d. 15 minutes
12. You attached a pulse oximeter to the client. You
know that the purpose is to: 17. If the arm is said to be elevated when taking the
a. Determine if the client’s hemoglobin blood pressure, it will create a:
level is low and if he needs blood a. False high reading
transfusion b. False low reading
b. Check level of client’s tissue perfusion c. True false reading
c. Measure the efficacy of the client’s anti- d. Indeterminate
hypertensive medications
d. Detect oxygen saturation of arterial 18. You are to assessed the temperature of the
blood before symptoms of hypoxemia client the next morning and found out that he
develops ate ice cream. How many minutes should you
wait before assessing the client’s oral
13. After a few hours in the Emergency Room, The temperature?
client is admitted to the ward with an order of a. 10 minutes
hourly monitoring of blood pressure. The nurse b. 20 minutes
finds that the cuff is too narrow and this will c. 30 minutes
cause the blood pressure reading to be: d. 15 minutes
a. inconsistent
b. low systolic and high diastolic 19. When auscultating the client’s blood pressure
c. higher than what the reading should be the nurse hears the following: From 150 mmHg
d. lower than what the reading should be to 130 mmHg: Silence, Then: a thumping sound
continuing down to 100 mmHg; muffled sound
14. Through the client’s health history, you gather continuing down to 80 mmHg and then silence.
What is the client’s blood pressure? to lungs. This can be avoided by:
a. 130/80 a. Cleaning teeth and mouth with cotton
b. 150/100 swabs soaked with mouthwash to avoid
c. 100/80 rinsing the buccal cavity
d. 150/100 b. swabbing the inside of the cheeks and
lips, tongue and gums with dry cotton
20. In a client with a previous blood pressure of swabs
130/80 4 hours ago, how long will it take to c. use fingers wrapped with wet cotton
release the blood pressure cuff to obtain an washcloth to rub inside the cheeks,
accurate reading? tongue, lips and ums
a. 10-20 seconds d. suctioning as needed while cleaning the
b. 30-45 seconds buccal cavity
c. 1-1.5 minutes
d. 3-3.5 minutes 25. Your client has difficulty of breathing and is
mouth breathing most of the time. This causes
Situation: Oral care is an important part of hygienic dryness of the mouth with unpleasant odor. Oral
practices and promoting client comfort. hygiene is recommended for the client and in
addition, you will keep the mouth moistened by
21. An elderly client, 84 years old, is unconscious. using:
Assessment of the mouth reveals excessive a. salt solution
dryness and presence of sores. Which of the b. petroleum jelly
following is BEST to use for oral care? c. water
a. lemon glycerine d. mentholated ointment
b. Mineral oil
c. hydrogen peroxide Situation – Ensuring safety before, during and after a
d. Normal saline solution diagnostic procedure is an important responsibility of
the nurse.
22. When performing oral care to an unconscious
client, which of the following is a special 26. To help Fernan better tolerate the
consideration to prevent aspiration of fluids into bronchoscopy, you should instruct him to
the lungs? practice which of the following prior to the
a. Put the client on a sidelying position procedure?
with head of bed lowered a. Clenching his fist every 2 minutes
b. Keep the client dry by placing towel b. Breathing in and out through the nose
under the chin with his mouth open
c. Wash hands and observes appropriate c. Tensing the shoulder muscles while lying
infection control on his back
d. Clean mouth with oral swabs in a careful d. Holding his breath periodically for 30
and an orderly progression seconds

23. The advantages of oral care for a client include 27. Following a bronchoscopy, which of the
all of the following, EXCEPT: following complains to Fernan should be noted
a. decreases bacteria in the mouth and as a possible complication:
teeth a. Nausea and vomiting
b. reduces need to use commercial b. Shortness of breath and laryngeal
mouthwash which irritate the buccal stridor
mucosa c. Blood tinged sputum and coughing
c. improves client’s appearance and self- d. Sore throat and hoarseness
confidence
d. improves appetite and taste of food 28. Immediately after bronchoscopy, you instructed
Fernan to:
24. A possible problem while providing oral care to a. Exercise the neck muscles
unconscious clients is the risk of fluid aspiration b. Refrain from coughing and talking

6
7

c. Breathe deeply d. Weber’s test


d. Clear his throat
34. A nurse is reviewing the arterial blood gas values
29. Thoracentesis may be performed for cytologic of a client and notes that the ph is 7.31, Pco2 is
study of pleural fluid. As a nurse your most 50 mmHg, and the bicarbonate is 27 mEq/L. The
important function during the procedure is to: nurse concludes that which acid base
a. Keep the sterile equipment from disturbance is present in this client?
contamination a. Respiratory acidosis
b. Assist the physician b. Metabolic acidosis
c. Open and close the three-way stopcock c. Respiratory alkalosis
d. Observe the patient’s vital signs d. Metabolic alkalosis

30. Right after thoracentesis, which of the following 35. Allen’s test checks the patency of the:
is most appropriate intervention? a. Ulnar artery
a. Instruct the patient not to cough or deep b. Carotid artery
breathe for two hours c. Radial artery
b. Observe for symptoms of tightness of d. Brachial artery
chest or bleeding
c. Place an ice pack to the puncture site Situation 6: Eileen, 45 years old is admitted to the
d. Remove the dressing to check for hospital with a diagnosis of renal calculi. She is
bleeding experiencing severe flank pain, nauseated and with a
temperature of 39 0C.
Situation: Knowledge of the acid-base disturbance and
the functions of the electrolytes is necessary to 36. Given the above assessment data, the most
determine appropriate intervention and nursing actions. immediate goal of the nurse would be which of
the following?
31. A client with diabetes milletus has a blood a. Prevent urinary complication
glucose level of 644 mg/dL. The nurse interprets b. maintains fluid and electrolytes
that this client is at most risk for the c. Alleviate pain
development of which type of acid-base d. Alleviating nausea
imbalance?
a. Respiratory acidosis 37. After IVP a renal stone was confirmed, a left
b. Respiratory alkalosis nephrectomy was done. Her post-operative
c. Metabolic acidosis order includes “daily urine specimen to be sent
d. Metabolic alkalosis to the laboratory”. Eileen has a foley catheter
attached to a urinary drainage system. How will
32. In a client in the health care clinic, arterial blood you collect the urine specimen?
gas analysis gives the following results: pH 7.48, a. remove urine from drainage tube with
PCO2 32 mmHg, PO2 94 mmHg, HCO3 24 mEq/L. sterile needle and syringe and empty
The nurse interprets that the client has which urine from the syringe into the
acid base disturbance? specimen container
a. Respiratory acidosis b. empty a sample urine from the
b. Metabolic acidosis collecting bag into the specimen
c. Respiratory alkalosis container
d. Metabolic alkalosis c. Disconnect the drainage tube from the
indwelling catheter and allow urine to
33. A client has an order for ABG analysis on radial flow from catheter into the specimen
artery specimens. The nurse ensures that which container.
of the following has been performed or tested d. Disconnect the drainage from the
before the ABG specimens are drawn? collecting bag and allow the urine to
a. Guthrie test flow from the catheter into the
b. Romberg’s test specimen container.
c. Allen’s test
38. Where would the nurse tape Eileen’s indwelling regulation is secreted in the:
catheter in order to reduce urethral irritation? a. Thyroid gland
a. to the patient’s inner thigh b. Parathyroid gland
b. to the patient’ buttocks c. Hypothalamus
c. to the patient’s lower thigh d. Anterior pituitary gland
d. to the patient lower abdomen
45. While Parathormone, a hormone that negates
39. Which of the following menu is appropriate for the effect of calcitonin is secreted by the:
one with low sodium diet? a. Thyroid gland
a. instant noodles, fresh fruits and ice tea b. Parathyroid gland
b. ham and cheese sandwich, fresh fruits c. Hypothalamus
and vegetables d. Anterior pituitary gland
c. white chicken sandwich, vegetable
salad and tea Situation: The staff nurse supervisor requests all the staff
d. canned soup, potato salad, and diet soda nurses to “brainstorm” and learn ways to instruct
diabetic clients on self-administration of insulin. She
40. How will you prevent ascending infection to wants to ensure that there are nurses available daily to
Eileen who has an indwelling catheter? do health education classes.
a. see to it that the drainage tubing
touches the level of the urine 46. The plan of the nurse supervisor is an example of
b. change he catheter every eight hours a. in service education process
c. see to it that the drainage tubing does b. efficient management of human
not touch the level of the urine resources
d. clean catheter may be used since c. increasing human resources
urethral meatus is not a sterile area d. primary prevention

Situation: Hormones are secreted by the various glands 47. When Mrs. Guevarra, a nurse, delegates aspects
in the body. Basic knowledge of the endocrine system is of the clients care to the nurse-aide who is an
necessary. unlicensed staff, Mrs. Guevarra
a. makes the assignment to teach the staff
41. Somatocrinin or the Growth hormone releasing member
hormone is secreted by the: b. is assigning the responsibility to the
a. Hypothalamus aide but not the accountability for
b. Posterior pituitary gland those tasks
c. Anterior pituitary gland c. does not have to supervise or evaluate
d. Thyroid gland the aide
d. most know how to perform task
42. All of the following are secreted by the anterior delegated
pituitary gland except:
a. Somatotropin/Growth hormone 48. Connie, the new nurse, appears tired and
b. Thyroid stimulating hormone sluggish and lacks the enthusiasm she had six
c. Follicle stimulating hormone weeks ago when she started the job. The
d. Gonadotropin hormone releasing nurse supervisor should
hormone a. empathize with the nurse and listen to
her
43. All of the following hormones are hormones b. tell her to take the day off
secreted by the Posterior pituitary gland except: c. discuss how she is adjusting to her new
a. Vasopressin job
b. Anti-diuretic hormone d. ask about her family life
c. Oxytocin
d. Growth hormone 49. Process of formal negotiations of working
conditions between a group of registered nurses
44. Calcitonin, a hormone necessary for calcium and employer is

8
9

a. grievance d. It should disclose previous diagnosis,


b. arbitration prognosis and alternative treatments
c. collective bargaining available for the client
d. strike
55. Delegation is the process of assigning tasks that
50. You are attending a certification on can be performed by a subordinate. The RN
cardiopulmonary resuscitation (CPR) offered and should always be accountable and should not
required by the hospital employing you. This is lose his accountability. Which of the following is
a. professional course towards credits a role included in delegation?
b. in-service education a. The RN must supervise all delegated
c. advance training tasks
d. continuing education b. After a task has been delegated, it is no
longer a responsibility of the RN
Situation: As a nurse, you are aware that proper c. The RN is responsible and accountable
documentation in the patient chart is your responsibility. for the delegated task in adjunct with
the delegate
51. Which of the following is not a legally binding d. Follow up with a delegated task is
document but nevertheless very important in necessary only if the assistive personnel
the care of all patients in any health care is not trustworthy
setting?
a. Bill of rights as provided in the Philippine Situation: When creating your lesson plan for
constitution cerebrovascular disease or STROKE. It is important to
b. Scope of nursing practice as defined by include the risk factors of stroke.
RA 9173
c. Board of nursing resolution adopting the 56. The most important risk factor is:
code of ethics a. Cigarette smoking
d. Patient’s bill of rights b. binge drinking
c. Hypertension
52. A nurse gives a wrong medication to the client. d. heredity
Another nurse employed by the same hospital as
a risk manager will expect to receive which of 57. Part of your lesson plan is to talk about etiology
the following communication? or cause of stroke. The types of stroke based on
a. Incident report cause are the following EXCEPT:
b. Nursing kardex a. Embolic stroke
c. Oral report b. diabetic stroke
d. Complain report c. Hemorrhagic stroke
d. thrombotic stroke
53. Performing a procedure on a client in the
absence of an informed consent can lead to 58. Hemmorhagic stroke occurs suddenly usually
which of the following charges? when the person is active. All are causes of
a. Fraud hemorrhage, EXCEPT:
b. Harassment a. phlebitis
c. Assault and battery b. damage to blood vessel
d. Breach of confidentiality c. trauma
d. aneurysm
54. Which of the following is the essence of
informed consent? 59. The nurse emphasizes that intravenous drug
a. It should have a durable power of abuse carries a high risk of stroke. Which drug is
attorney closely linked to this?
b. It should have coverage from an a. Amphetamines
insurance company b. shabu
c. It should respect the client’s freedom c. Cocaine
from coercion d. Demerol
d. Iron 75 mg/100 ml
60. A participant in the STROKE class asks what is a
risk factor of stroke. Your best response is: 65. Which of the following laboratory test result
a. “More red blood cells thicken indicate presence of an infectious process?
blood and make clots more a. Erythrocyte sedimentation rate (ESR) 12
possible.” mm/hr
b. “Increased RBC count is linked to high b. White blood cells (WBC) 18,000/mm3
cholesterol.” c. Iron 90 g/100ml
c. “More red blood cell increases d. Neutrophils 67%
hemoglobin content.”
d. “High RBC count increases blood Situation: Pleural effusion is the accumulation of fluid in
pressure.” the pleural space. Questions 66 to 70 refer to this.

Situation: Recognition of normal values is vital in 66. Which of the following is a finding that the nurse
assessment of clients with various disorders. will be able to assess in a client with Pleural
effusion?
61. A nurse is reviewing the laboratory test results a. Reduced or absent breath sound at the
for a client with a diagnosis of severe base of the lungs, dyspnea, tachpynea
dehydration. The nurse would expect the and shortness of breath
hematocrit level for this client to be which of the b. Hypoxemia, hypercapnea and
following? respiratory acidosis
a. 60% c. Noisy respiration, crackles, stridor and
b. 47% wheezing
c. 45% d. Tracheal deviation towards the affected
d. 32% side, increased fremitus and loud breath
sounds
62. A nurse is reviewing the electrolyte results of an
assigned client and notes that the potassium 67. Thoracentesis is performed to the client with
level is 5.6 mEq/L. Which of the following would effusion. The nurse knows that the removal of
the nurse expect to note on the ECG as a result fluid should be slow. Rapid removal of fluid in
of this laboratory value? thoracentesis might cause:
a. ST depression a. Pneumothorax
b. Prominent U wave b. Cardiovascular collapse
c. Inverted T wave c. Pleurisy or Pleuritis
d. Tall peaked T waves d. Hypertension

63. A nurse is reviewing the electrolyte results of an 68. 3 Days after thoracentesis, the client again
assigned client and notes that the potassium exhibited respiratory distress. The nurse will
level is 3.2 mEq/L. Which of the following would know that pleural effusion has reoccurred when
the nurse expect to note on the ECG as a result she noticed a sharp stabbing pain during
of this laboratory value? inspiration. The physician ordered a closed tube
a. U waves thoracotomy for the client. The nurse knows
b. Elevated T waves that the primary function of the chest tube is to:
c. Absent P waves a. Restore positive intrathoracic pressure
d. Elevated ST Segment b. Restore negative intrathoracic pressure
c. To visualize the intrathoracic content
64. Dorothy underwent diagnostic test and the d. As a method of air administration via
result of the blood examination are back. On ventilator
reviewing the result the nurse notices which of
the following as abnormal finding? 69. The chest tube is functioning properly if:
a. Neutrophils 60% a. There is an oscillation
b. White blood cells (WBC) 9000/mm b. There is no bubbling in the drainage
c. Erythrocyte sedimentation rate (ESR) is bottle
39 mm/hr
10
11

c. There is a continuous bubbling in the


waterseal 75. This form of Health Insurance provides
d. The suction control bottle has a comprehensive prepaid health services to
continuous bubbling enrollees for a fixed periodic payment.
a. Health Maintenance Organization
70. In a client with pleural effusion, the nurse is b. Medicare
instructing appropriate breathing technique. c. Philippine Health Insurance Act
Which of the following is included in the d. Hospital Maintenance Organization
teaching?
a. Breath normally Situation: Nursing ethics is an important part of the
b. Hold the breath after each inspiration nursing profession. As the ethical situation arises, so is
for 1 full minute the need to have an accurate and ethical decision
c. Practice abdominal breathing making.
d. Inhale slowly and hold the breath for 3
to 5 seconds after each inhalation 76. The purpose of having a nurses’ code of ethics is:
a. Delineate the scope and areas of nursing
SITUATION: Health care delivery system affects the practice
health status of every filipino. As a Nurse, Knowledge of b. identify nursing action recommended for
this system is expected to ensure quality of life. specific health care situations
c. To help the public understand
71. When should rehabilitation commence? professional conduct expected of
a. The day before discharge nurses
b. When the patient desires d. To define the roles and functions of the
c. Upon admission health care givers, nurses, clients
d. 24 hours after discharge
77. The principles that govern right and proper
72. What exemplified the preventive and promotive conduct of a person regarding life, biology and
programs in the hospital? the health professionals is referred to as:
a. Hospital as a center to prevent and a. Morality
control infection b. Religion
b. Program for smokers c. Values
c. Program for alcoholics and drug addicts d. Bioethics
d. Hospital Wellness Center
78. A subjective feeling about what is right or wrong
73. Which makes nursing dynamic? is said to be:
a. Every patient is a unique physical, a. Morality
emotional, social and spiritual being b. Religion
b. The patient participate in the overall c. Values
nursing care plan d. Bioethics
c. Nursing practice is expanding in the light
of modern developments that takes 79. Values are said to be the enduring believe about
place a worth of a person, ideas and belief. If Values
d. The health status of the patient is are going to be a part of a research, this is
constantly changing and the nurse must categorized under:
be cognizant and responsive to these a. Qualitative
changes b. Experimental
c. Quantitative
74. Prevention is an important responsibility of the d. Non Experimental
nurse in:
a. Hospitals 80. The most important nursing responsibility where
b. Community ethical situations emerge in patient care is to:
c. Workplace a. Act only when advised that the action is
d. All of the above ethically sound
b. Not takes sides, remain neutral and fair
c. Assume that ethical questions are the 85. Based on the Code of Ethics for Filipino Nurses,
responsibility of the health team what is regarded as the hallmark of nursing
d. Be accountable for his or her own responsibility and accountability?
actions a. Human rights of clients, regardless of
creed and gender
81. Why is there an ethical dilemma? b. The privilege of being a registered
a. the choices involved do not appear to be professional nurse
clearly right or wrong c. Health, being a fundamental right of
b. a client’s legal right co-exist with the every individual
nurse’s professional obligation d. Accurate documentation of actions and
c. decisions has to be made based on outcomes
societal norms.
d. decisions has to be mad quickly, often Situation: As a profession, nursing is dynamic and its
under stressful conditions practice is directed by various theoretical models. To
demonstrate caring behaviour, the nurse applies various
82. According to the code of ethics, which of the nursing models in providing quality nursing care.
following is the primary responsibility of the
nurse? 86. When you clean the bedside unit and regularly
a. Assist towards peaceful death attend to the personal hygiene of the patient as
b. Health is a fundamental right well as in washing your hands before and after a
c. Promotion of health, prevention of procedure and in between patients, you indent
illness, alleviation of suffering and to facilitate the body’s reparative processes.
restoration of health Which of the following nursing theory are you
d. Preservation of health at all cost applying in the above nursing action?
a. Hildegard Peplau
83. Which of the following is TRUE about the Code b. Dorothea Orem
of Ethics of Filipino Nurses, except: c. Virginia Henderson
a. The Philippine Nurses Association for d. Florence Nightingale
being the accredited professional
organization was given the privilege to 87. A communication skill is one of the important
formulate a Code of Ethics for Nurses competencies expected of a nurse. Interpersonal
which the Board of Nursing process is viewed as human to human
promulgated relationship. This statement is an application of
b. Code for Nurses was first formulated in whose nursing model?
1982 published in the Proceedings of the a. Joyce Travelbee
Third Annual Convention of the PNA b. Martha Rogers
House of Delegates c. Callista Roy
c. The present code utilized the Code of d. Imogene King
Good Governance for the Professions in
the Philippines 88. The statement “the health status of an individual
d. Certificates of Registration of registered is constantly changing and the nurse must be
nurses may be revoked or suspended for cognizant and responsive to these changes” best
violations of any provisions of the Code explains which of the following facts about
of Ethics. nursing?
a. Dynamic
84. Violation of the code of ethics might equate to b. Client centred
the revocation of the nursing license. Who c. Holistic
revokes the license? d. Art
a. PRC
b. PNA 89. Virginia Henderson professes that the goal of
c. DOH nursing is to work interdependently with other
d. BON health care working in assisting the patient to

12
13

gain independence as quickly as possible. Which include:


of the following nursing actions best a. Prescription of the doctor to the
demonstrates this theory in taking care of a 94 patient’s illness
year old client with dementia who is totally b. Plan of care for patient
immobile? c. Patient’s perception of one’s illness
a. Feeds the patient, brushes his teeth, d. Nursing problem and Nursing diagnosis
gives the sponge bath
b. Supervise the watcher in rendering 94. The medical records that are organized into
patient his morning care separate section from doctors or nurses has
c. Put the patient in semi fowler’s position, more disadvantages than advantages. This is
set the over bed table so the patient can classified as what type of recording?
eat by himself, brush his teeth and a. POMR
sponge himself b. Modified POMR
d. Assist the patient to turn to his sides and c. SOAPIE
allow him to brush and feed himself only d. SOMR
when he feels ready
95. Which of the following is the advantage of SOMR
90. In the self-care deficit theory by Dorothea Orem, or Traditional recording?
nursing care becomes necessary when a patient a. Increases efficiency in data gathering
is unable to fulfil his physiological, psychological b. Reinforces the use of the nursing
and social needs. A pregnant client needing process
prenatal check-up is classified as: c. The caregiver can easily locate proper
a. Wholly compensatory section for making charting entries
b. Supportive Educative d. Enhances effective communication
c. Partially compensatory among health care team members
d. Non compensatory
Situation: June is a 24 year old client with symptoms of
Situation: Documentation and reporting are just as dyspnea, absent breath sounds on the right lung and
important as providing patient care, As such, the nurse chest x ray revealed pleural effusion. The physician will
must be factual and accurate to ensure quality perform thoracentesis.
documentation and reporting.
96. Thoracentesis is useful in treating all of the
91. Health care reports have different purposes. The following pulmonary disorders except:
availability of patients’ record to all health team a. Hemothorax
members demonstrates which of the following b. Hydrothorax
purposes: c. Tuberculosis
a. Legal documentation d. Empyema
b. Research
c. Education 97. Which of the following psychological preparation
d. Vehicle for communication is not relevant for him?
a. Telling him that the gauge of the needle
92. When a nurse commits medication error, she and anesthesia to be used
should accurately document client’s response b. Telling him to keep still during the
and her corresponding action. This is very procedure to facilitate the insertion of
important for which of the following purposes: the needle in the correct place
a. Research c. Allow June to express his feelings and
b. Legal documentation concerns
c. Nursing Audit d. Physician’s explanation on the purpose
d. Vehicle for communication of the procedure and how it will be done

93. POMR has been widely used in many teaching 98. Before thoracentesis, the legal consideration you
hospitals. One of its unique features is SOAPIE must check is:
charting. The P in SOAPIE charting should a. Consent is signed by the client
b. Medicine preparation is correct
c. Position of the client is correct
d. Consent is signed by relative and
physician

99. As a nurse, you know that the position for June


before thoracentesis is:
a. Orthopneic
b. Low fowlers
c. Knee-chest
d. Sidelying position on the affected side

100. Which of the following anaesthetics drug is used


for thoracentesis?
a. Procaine 2%
b. Demerol 75 mg
c. Valium 250 mg
d. Phenobartbital 50 mg

14
15

D. Follicle stimulating hormone

NURSING PRACTICE II 5. The following month, Mariah suspects she is


pregnant. Her urine is positive for Human
Situation: Mariah is a 31 year old lawyer who has been chorionic gonadotrophin. Which structure
married for 6 months. She consults you for guidance in produces Hcg?
relation with her menstrual cycle and her desire to get A. Pituitary gland
pregnant. B. Trophoblastic cells of the embryo
C. Uterine deciduas
1. She wants to know the length of her menstrual D. Ovarian follicles
cycle. Her previous menstrual period is October
22 to 26. Her LMB is November 21. Which of the Situation: Mariah came back and she is now pregnant.
following number of days will be your correct
response? 6. At 5 month gestation, which of the following
A. 29 fetal development would probably be achieve?
B. 28 A. Fetal movement are felt by Mariah
C. 30 B. Vernix caseosa covers the entire body
D. 31 C. Viable if delivered within this period
D. Braxton hicks contractions are observed
2. You advised her to observe and record the signs
of Ovulation. Which of the following signs will 7. The nurse palpates the abdomen of Mariah.
she likely note down? Now At 5 month gestation, What level of the
1. A 1 degree Fahrenheit rise in basal body abdomen can the fundic height be
temperature palpated?
2. Cervical mucus becomes copious and A. Symphysis pubis
clear B. Midpoint between the umbilicus and the
3. One pound increase in weight xiphoid process
4. Mittelschmerz C. Midpoint between the symphysis pubis
A. 1, 2, 4 and the umbilicus
B. 1, 2, 3 D. Umbilicus
C. 2, 3, 4
D. 1, 3, 4 8. She worries about her small breasts, thinking
that she probably will not be able to breastfeed
3. You instruct Mariah to keep record of her basal her baby. Which of the following responses of
temperature every day, which of the following the nurse is correct?
instructions is incorrect? A. “The size of your breast will not affect
A. If coitus has occurred; this should be your lactation”
reflected in the chart B. “You can switch to bottle feeding”
B. It is best to have coitus on the evening C. “You can try to have exercise to increase
following a drop in BBT to become the size of your breast”
pregnant D. “Manual expression of milk is possible”
C. Temperature should be taken
immediately after waking and before 9. She tells the nurse that she does not take milk
getting out of bed regularly. She claims that she does not want to
D. BBT is lowest during the gain too much weight during her pregnancy.
secretory phase Which of the following nursing diagnosis is a
priority?
4. She reports an increase in BBT on December 16. A. Potential self-esteem disturbance
Which hormone brings about this change in her related to physiologic changes in
BBT? pregnancy
A. Estrogen B. Ineffective individual coping related to
B. Gonadotropine physiologic changes in pregnancy
C. Progesterone C. Fear related to the effects of pregnancy
D. Knowledge deficit regarding nutritional
requirements of pregnancies related to 15. While talking with Susan, 2 new patients arrived
lack of information sources and they are covered with large towels and the
nurse noticed that there are many cameraman
10. Which of the following interventions will likely and news people outside of the OPD. Upon
ensure compliance of Mariah? assessment the nurse noticed that both of them
A. Incorporate her food preferences that are still nude and the male client’s penis is still
are adequately nutritious in her meal inside the female client’s vagina and the male
plan client said that “I can’t pull it”. Vaginismus was
B. Consistently counsel toward optimum your first impression. You know that The
nutritional intake psychological cause of Vaginismus is related to:
C. Respect her right to reject dietary A. The male client inserted the penis too
information if she chooses deeply that it stimulates vaginal closure
D. Inform her of the adverse effects of B. The penis was too large that is why the
inadequate nutrition to her fetus vagina triggered its defense to attempt
to close it
Situation: Susan is a patient in the clinic where you work. C. The vagina does not want to be
She is inquiring about pregnancy. penetrated
D. It is due to learning patterns of the
11. Susan tells you she is worried because she female client where she views sex
develops breasts later than most of her friends. as bad or sinful
Breast development is termed as:
A. Adrenarche Situation: Overpopulation is one problem in the
B. Thelarche Philippines that causes economic drain. Most Filipinos
C. Mamarche are against in legalizing abortion. As a nurse, Mastery of
D. Menarche contraception is needed to contribute to the society and
economic growth.
12. Kevin, Susan’s husband tells you that he is
considering vasectomy After the birth of their 16. Supposed that Dana, 17 years old, tells you she
new child. Vasectomy involves the incision of wants to use fertility awareness method of
which organ? contraception. How will she determine her
A. The testes fertile days?
B. The epididymis A. She will notice that she feels hot, as if
C. The vas deferens she has an elevated temperature.
D. The scrotum B. She should assess whether her cervical
mucus is thin, copious, clear and
13. On examination, Susan has been found of having watery.
a cystocele. A cystocele is: C. She should monitor her emotions for
A. A sebaceous cyst arising from the vulvar sudden anger or crying
fold D. She should assess whether her breasts
B. Protrusion of intestines into the vagina feel sensitive to cool air
C. Prolapse of the uterus into the vagina
D. Herniation of the bladder into the 17. Dana chooses to use COC as her family planning
vaginal wall method. What is the danger sign of COC you
would ask her to report?
14. Susan typically has menstrual cycle of 34 days. A. A stuffy or runny nose
She told you she had coitus on days 8, 10, 15 and B. Slight weight gain
20 of her menstrual cycle. Which is the day on C. Arthritis like symptoms
which she is most likely to conceive? D. Migraine headache
A. 8th day
B. Day 15 18. Dana asks about subcutaneous implants and she
C. 10th day asks, how long will these implants be effective.
D. Day 20 Your best answer is:
A. One month

16
17

B. Five years
C. Twelve months 23. Another client named Lilia is diagnosed as having
D. 10 years endometriosis. This condition interferes with
fertility because:
19. Dana asks about female condoms. Which of the A. Endometrial implants can block the
following is true with regards to female fallopian tubes
condoms? B. The uterine cervix becomes inflamed
A. The hormone the condom releases and swollen
might cause mild weight gain C. The ovaries stop producing adequate
B. She should insert the condom before estrogen
any penile penetration D. Pressure on the pituitary leads to
C. She should coat the condom with decreased FSH levels
spermicide before use
D. Female condoms, unlike male condoms, 24. Lilia is scheduled to have a
are reusable hysterosalphingogram. Which of the following
instructions would you give her regarding this
20. Dana has asked about GIFT procedure. What procedure?
makes her a good candidate for GIFT? A. She will not be able to conceive for 3
A. She has patent fallopian tubes, so months after the procedure
fertilized ova can be implanted on them B. The sonogram of the uterus will reveal
B. She is RH negative, a necessary any tumors present
stipulation to rule out RH incompatibility C. Many women experience mild bleeding
C. She has normal uterus, so the sperm can as an after effect
be injected through the cervix into it D. She may feel some cramping when the
D. Her husband is taking sildenafil, so all dye is inserted
sperms will be motile
25. Lilia’s cousin on the other hand, knowing nurse
Situation: Nurse Lorena is a Family Planning and Lorena’s specialization asks what artificial
Infertility Nurse Specialist and currently attends to insemination by donor entails. Which would be
FAMILY PLANNING CLIENTS AND INFERTILE COUPLES. your best answer if you were Nurse Lorena?
The following conditions pertain to meeting the nursing A. Donor sperm are introduced vaginally
needs of this particular population group. into the uterus or cervix
B. Donor sperm are injected intra-
21. Dina, 17 years old, asks you how a tubal ligation abdominally into each ovary
prevents pregnancy. Which would be the best C. Artificial sperm are injected vaginally to
answer? test tubal patency
A. Prostaglandins released from the cut D. The husband’s sperm is administered
fallopian tubes can kill sperm intravenously weekly
B. Sperm cannot enter the uterus because
the cervical entrance is blocked. Situation: You are assigned to take care of a group of
C. Sperm can no longer reach the ova, patients across the lifespan.
because the fallopian tubes are blocked
D. The ovary no longer releases ova as 26. Pain in the elder persons requires careful
there is nowhere for them to go. assessment because they:
A. experienced reduce sensory perception
22. The Dators are a couple undergoing testing for B. have increased sensory perception
infertility. Infertility is said to exist when: C. are expected to experience chronic pain
A. A woman has no uterus D. have a decreased pain threshold
B. A woman has no children
C. A couple has been trying to conceive for 27. Administration of analgesics to the older persons
1 year requires careful patient assessment because
D. A couple has wanted a child for 6 older people:
months A. are more sensitive to drugs
B. have increased hepatic, renal and D. Chronic poverty
gastrointestinal function
C. have increased sensory perception 34. Which of the following signs and symptoms
D. mobilize drugs more rapidly would you most likely find when assessing and
infant with Arnold-Chiari malformation?
28. The elderly patient is at higher risk for urinary A. Weakness of the leg muscles, loss of
incontinence because of: sensation in the legs, and restlessness
A. increased glomerular filtration B. Difficulty swallowing, diminished or
B. decreased bladder capacity absent gag reflex, and respiratory
C. diuretic use distress
D. dilated urethra C. Difficulty sleeping, hypervigilant, and an
arching of the back
29. Which of the following is the MOST COMMON D. Paradoxical irritability, diarrhea, and
sign of infection among the elderly? vomiting.
A. decreased breath sounds with crackles
B. pain 35. A parent calls you and frantically reports that her
C. fever child has gotten into her famous ferrous sulfate
D. change in mental status pills and ingested a number of these pills. Her
child is now vomiting, has bloody diarrhea, and is
30. Priorities when caring for the elderly trauma complaining of abdominal pain. You will tell the
patient: mother to:
A. circulation, airway, breathing A. Call emergency medical services (EMS)
B. airway, breathing, disability (neurologic) and get the child to the emergency
C. disability (neurologic), airway, breathing room
D. airway, breathing, circulation B. Relax because these symptoms will pass
and the child will be fine
31. Preschoolers are able to see things from which C. Administer syrup of ipecac
of the following perspectives? D. Call the poison control center
A. Their peers
B. Their own and their mother’s 36. A client says she heard from a friend that you
C. Their own and their caregivers’ stop having periods once you are on the “pill”.
D. Only their own The most appropriate response would be:
A. “The pill prevents the uterus from
32. In conflict management, the win-win approach making such endometrial lining, that is
occurs when: why periods may often be scant or
A. There are two conflicts and the parties skipped occasionally.”
agree to each one B. “If your friend has missed her period,
B. Each party gives in on 50% of the she should stop taking the pills and get a
disagreements making up the conflict pregnancy test as soon as possible.”
C. Both parties involved are committed to C. “The pill should cause a normal
solving the conflict menstrual period every month. It
D. The conflict is settled out of court so the sounds like your friend has not
legal system and the parties win been taking the pills properly.”
D. “Missed period can be very dangerous
33. According to the social-interactional perspective and may lead to the formation of
of child abuse and neglect, four factors place the precancerous cells.”
family members at risk for abuse. These risk
factors are the family members at risk for abuse. 37. The nurse assessing newborn babies and infants
These risk factors are the family itself, the during their hospital stay after birth will notice
caregiver, the child, and which of the following symptoms as a primary
A. The presence of a family crisis manifestation of Hirschsprung’s disease?
B. The national emphasis on sex A. A fine rash over the trunk
C. Genetics B. Failure to pass meconium during the
first 24 to 48 hours after birth
18
19

C. The skin turns yellow and then brown release


over the first 48 hours of life B. a woman is less able to keep the
D. High-grade fever laceration clean because of her fatigue
C. healing is limited during pregnancy
38. A client is 7 months pregnant and has just been so these will not heal until after birth
diagnosed as having a partial placenta previa. D. increased bleeding can occur from
She is stable and has minimal spotting and is uterine pressure on leg veins
being sent home. Which of these instructions to
the client may indicate a need for further 43. In working with the caregivers of a client with an
teaching? acute or chronic illness, the nurse would:
A. Maintain bed rest with bathroom A. Teach care daily and let the caregivers
privileges do a return demonstration just before
B. Avoid intercourse for three days. discharge
C. Call if contractions occur. B. Difficulty swallowing, diminished or
D. Stay on left side as much as possible absent gag reflex, and respiratory
when lying down. distress.
C. Difficulty sleeping, hypervigilant, and an
39. A woman has been rushed to the hospital with arching of the back
ruptured membrane. Which of the following D. Paradoxical irritability, diarrhea, and
should the nurse check first? vomiting
A. Check for the presence of infection
B. Assess for Prolapse of the 44. Which of the following roles BEST exemplifies
umbilical cord the expanded role of the nurse?
C. Check the maternal heart rate A. Circulating nurse in surgery
D. Assess the color of the amniotic fluid B. Medication nurse
C. Obstetrical nurse
40. The nurse notes that the infant is wearing a D. Pediatric nurse practitioner
plastic-coated diaper. If a topical medication
were to be prescribed and it were to go on the 45. According to DeRosa and Kochura’s (2006)
stomachs or buttocks, the nurse would teach the article entitled “Implement Culturally Competent
caregivers to: Health Care in your work place,” cultures have
A. avoid covering the area of the topical different patterns of verbal and nonverbal
medication with the diaper communication. Which difference does?
B. avoid the use of clothing on top of the A. NOT necessarily belong?
diaper B. Personal behavior
C. put the diaper on as usual C. Subject matter
D. apply an icepack for 5 minutes to the D. Eye contact
outside of the diaper E. Conversational style

41. Which of the following factors is most important 46. You are the nurse assigned to work with a child
in determining the success of relationships used with acute glomerulonephritis. By following the
in delivering nursing care? prescribed treatment regimen, the child
A. Type of illness of the client experiences a remission. You are now checking
B. Transference and counter transference to make sure the child does not have a relapse.
C. Effective communication Which finding would most lead you to the
D. Personality of the participants conclusion that a relapse is happening?
A. Elevated temperature, cough, sore
42. Grace sustained a laceration on her leg from throat, changing complete blood count
automobile accident. Why are lacerations of (CBC) with diiferential
lower extremities potentially more serious B. A urine dipstick measurement of 2+
among pregnant women than other? proteinuria or more for 3 days, or the
A. lacerations can provoke allergic child found to have 3-4+ proteinutria
responses due to gonadotropic hormone plus edema.
C. The urine dipstick showing glucose in the
urine for 3 days, extreme thirst, increase 51. If a child with diarrhea registers two signs in the
in urine output, and a moon face. yellow row in the IMCI chart, we can classify the
D. A temperature of 37.8 degrees (100 patient as:
degrees F), flank pain, burning A. Moderate dehydration
frequency, urgency on voiding, and B. Severe dehydration
cloudy urine. C. Some dehydration
D. No dehydration
47. The nurse is working with an adolescent who
complains of being lonely and having a lack of 52. Celeste has had diarrhea for 8 days. There is no
fulfillment in her life. This adolescent shies away blood in the stool, he is irritable, his eyes are
from intimate relationships at times yet at other sunken, the nurse offers fluid to Celeste and he
times she appears promiscuous. The nurse will drinks eagerly. When the nurse pinched the
likely work with this adolescent in which of the abdomen it goes back slowly. How will you
following areas? classify Celeste’s illness?
A. Isolation A. Moderate dehydration
B. Lack of fulfillment B. Severe dehydration
C. Loneliness C. Some dehydration
D. Identity D. No dehydration

48. The use of interpersonal decision making, 53. A child who is 7 weeks has had diarrhea for 14
psychomotor skills, and application of days but has no sign of dehydration is classified
knowledge expected in the role of a licensed as:
health care professional in the context of public A. Persistent diarrhea
health welfare and safety is an example of: B. Dysentery
A. Delegation C. Severe dysentery
B. Responsibility D. Severe persistent diarrhea
C. Supervision
D. Competence 54. The child with no dehydration needs home
treatment. Which of the following is not
49. The painful phenomenon known as “back labor” included in the rules for home treatment in this
occurs in a client whose fetus in what position? case?
A. Brow position A. Forced fluids
B. Breech position B. When to return
C. Right Occipito-Anterior Position C. Give vitamin A supplement
D. Left Occipito-Posterior Position D. Feeding more

50. FOCUS methodology stands for: 55. Fever as used in IMCI includes:
A. Focus, Organize, Clarify, Understand A. Axillary temperature of 37.5 or higher
and Solution B. Rectal temperature of 38 or higher
B. Focus, Opportunity, Continuous, Utilize, C. Feeling hot to touch
Substantiate D. All of the above
C. Focus, Organize, Clarify, Understand, E. A and C only
Substantiate
D. Focus, Opportunity, Continuous Situation: Prevention of Dengue is an important nursing
(process), Understand, Solution responsibility and controlling it’s spread is a priority once
outbreak has been observed.
SITUATION: The infant and child mortality rate in the low
to middle income countries is ten times higher than 56. An important role of the community health
industrialized countries. In response to this, the WHO nurse in the prevention and control of Dengue
and UNICEF launched the protocol Integrated H-fever includes:
Management of Childhood Illnesses to reduce the A. Advising the elimination of vectors by
morbidity and mortality against childhood illnesses. keeping water containers covered

20
21

B. Conducting strong health education health worker should first:


drives/campaign directed towards A. Identify the myths and misconceptions
proper garbage disposal prevailing in the community
C. Explaining to the individuals, B. Identify the source of these myths and
families, groups and community the misconceptions
nature of the disease and its C. Explain how and why these myths came
causation about
D. Practicing residual spraying with D. Select the appropriate IEC strategies to
insecticides correct them

57. Community health nurses should be alert in 62. How many percent of measles are prevented by
observing a Dengue suspect. The following is immunization at 9 months of age?
NOT an indicator for hospitalization of H-fever A. 80%
suspects? B. 99%
A. Marked anorexia, abdominal pain and C. 90%
vomiting D. 95%
B. Increasing hematocrit count
C. Cough of 30 days 63. After TT3 vaccination a mother is said to be
D. Persistent headache protected to tetanus by around:
A. 80%
58. The community health nurses’ primary concern B. 99%
in the immediate control of hemorrhage among C. 85%
patients with dengue is: D. 90%
A. Advising low fiber and non-fat diet
B. Providing warmth through light weight 64. If ever convulsions occur after administering
covers DPT, what should the nurse best suggest to the
C. Observing closely the patient for vital mother?
signs leading to shock A. Do not continue DPT vaccination
D. Keeping the patient at rest anymore
B. Advise mother to comeback after 1 week
59. Which of these signs may NOT be REGARDED as C. Give DT instead of DPT
a truly positive signs indicative of Dengue H- D. Give pertussis of the DPT and remove DT
fever?
A. Prolonged bleeding time 65. These vaccines are given 3 doses at one month
B. Appearance of at least 20 petechiae intervals:
within 1cm square A. DPT, BCG, TT
C. Steadily increasing hematocrit count B. OPV, HEP. B, DPT
D. Fall in the platelet count C. DPT, TT, OPV
D. Measles, OPV, DPT
60. Which of the following is the most important
treatment of patients with Dengue H-fever? Situation – With the increasing documented cases of
A. Give aspirin for fever CANCER the best alternative to treatment still remains to
B. Replacement of body fluids be PREVENTION. The following conditions apply.
C. Avoid unnecessary movement of patient
D. Ice cap over the abdomen in case of 66. Which among the following is the primary focus
melena of prevention of cancer?
A. Elimination of conditions causing cancer
Situation: Health education and Health promotion is an B. Diagnosis and treatment
important part of nursing responsibility in the C. Treatment at early stage
community. Immunization is a form of health promotion D. Early detection
that aims at preventing the common childhood illnesses.
67. In the prevention and control of cancer, which of
61. In correcting misconceptions and myths about the following activities is the most important
certain diseases and their management, the
function of the community health nurse? C. 30 breaths per minute or more
A. Conduct community assemblies. D. 60 breaths per minute
B. Referral to cancer specialist those clients
with symptoms of cancer. 73. Nina, the 2nd child has diarrhea for 5 days.
C. Use the nine warning signs of cancer as There is no blood in the stool. She is irritable,
parameters in our process of detection, and her eyes are sunken. The nurse offered
control and treatment modalities. fluids and and the child drinks eagerly. How
D. Teach woman about proper/correct would you classify Nina’s illness?
nutrition. A. Some dehydration
B. Severe dehydration
68. Who among the following are recipients of the C. Dysentery
secondary level of care for cancer cases? D. No dehydration
A. Those under early case detection
B. Those under post case treatment 74. Nina’s treatment should include the following
C. Those scheduled for surgery EXCEPT:
D. Those undergoing treatment A. reassess the child and classify him for
dehydration
69. Who among the following are recipients of the B. for infants under 6 months old who are
tertiary level of care for cancer cases? not breastfed, give 100-200 ml clean
A. Those under early treatment water as well during this period
B. Those under early detection C. Give in the health center the
C. Those under supportive care recommended amount of ORS for 4
D. Those scheduled for surgery hours.
D. Do not give any other foods to the child
70. In Community Health Nursing, despite the for home treatment
availability and use of many equipment and
devices to facilitate the job of the community 75. While on treatment, Nina 18 months old
health nurse, the best tool any nurse should be weighed 18 kgs. and her temperature registered
wel be prepared to apply is a scientific approach. at 37 degrees C. Her mother says she developed
This approach ensures quality of care even at the cough 3 days ago. Nina has no general danger
community setting. This is nursing parlance is signs. She has 45 breaths/minute, no chest in-
nothing less than the: drawing, no stridor. How would you classify
A. nursing diagnosis Nina’s manifestation?
B. nursing research A. No pneumonia
C. nursing protocol B. Pneumonia
D. nursing process C. Severe pneumonia
D. Bronchopneumonia
Situation – Two children were brought to you. One with
chest indrawing and the other had diarrhea. The 76. Carol is 15 months old and weighs 5.5 kgs and it
following questions apply: is her initial visit. Her mother says that Carol is
not eating well and unable to breastfeed, he has
71. Using Integrated Management and Childhood no vomiting, has no convulsion and not
Illness (IMCI) approach, how would you classify abnormally sleepy or difficult to awaken. Her
the 1st child? temperature is 38.9 deg C. Using the integrated
A. Bronchopneumonia management of childhood illness or IMCI
B. Severe pneumonia strategy, if you were the nurse in charge of
C. No pneumonia : cough or cold Carol, how will you classify her illness?
D. Pneumonia A. a child at a general danger sign
B. severe pneumonia
72. The 1st child who is 13 months has fast C. very severe febrile disease
breathing using IMCI parameters he has: D. severe malnutrition
A. 40 breaths per minute or more
B. 50 breaths per minute 77. Why are small for gestational age newborns at

22
23
risk for difficulty maintaining body temperature? B. give the child more fluids
A. their skin is more susceptible to C. continue feeding the child
conduction of cold D. inform when to return to the health
B. they are preterm so are born relatively center
small in size
C. they do not have as many fat stored as 83. Ms. Jordan, RN, believes that a patient should be
other infants treated as individual. This ethical principle that
D. they are more active than usual so they the patient referred to:
throw off comes A. beneficence
B. respect for person
78. Oxytocin is administered to Rita to augment C. nonmaleficence
labor. What are the first symptoms of water D. autonomy
intoxication to observe for during this
procedure? 84. When patients cannot make decisions for
A. headache and vomiting themselves, the nurse advocate relies on the
B. a high choking voice ethical principle of:
C. a swollen tender tongue A. justice and beneficence
D. abdominal bleeding and pain B. beneficence and nonmaleficence
C. fidelity and nonmaleficence
79. Which of the following treatment should NOT be D. fidelity and justice
considered if the child has severe dengue
hemorrhagic fever? 85. Being a community health nurse, you have the
A. use plan C if there is bleeding from the responsibility of participating in protecting the
nose or gums health of people. Consider this situation:
B. give ORS if there is skin Petechiae, Vendors selling bread with their bare hands.
persistent vomiting, and positive They receive money with these hands. You do
tourniquet test not see them washing their hands. What should
C. give aspirin you say/do?
D. prevent low blood sugar A. “Miss, may I get the bread myself
because you have not washed your
80. In assessing the patient’s condition using the hands”
Integrated Management of Childhood Illness B. All of these
approach strategy, the first thing that a nurse C. “Miss, it is better to use a pick up
should do is to: forceps/ bread tong”
D. “Miss, your hands are dirty. Wash your
A. ask what are the child’s problem hands first before getting the bread”
B. check for the four main symptoms
C. check the patient’s level of Situation: The following questions refer to common
consciousness clinical encounters experienced by an entry level nurse.
D. check for the general danger signs
86. A female client asks the nurse about the use of a
81. A child with diarrhea is observed for the cervical cap. Which statement is correct
following EXCEPT: regarding the use of the cervical cap?
A. how long the child has diarrhea A. It may affect Pap smear results.
B. presence of blood in the stool B. It does not need to be fitted by the
C. skin Petechiae physician.
D. signs of dehydration C. It does not require the use of
spermicide.
82. The child with no dehydration needs home D. It must be removed within 24 hours.
treatment. Which of the following is NOT
included in the care for home management at 87. The major components of the communication
this case? process are:
A. give drugs every 4 hours A. Verbal, written and nonverbal
B. Speaker, listener and reply D. Parasites
C. Facial expression, tone of voice and
gestures 93. You are assigned in a private room of Mike.
D. Message, sender, channel, receiver and Which procedure should be of outmost
feedback importance;
A. Alcohol wash
88. The extent of burns in children are normally B. Washing Isolation
assessed and expressed in terms of: C. Universal precaution
A. The amount of body surface that is D. Gloving technique
unburned
B. Percentages of total body surface area 94. What primary health teaching would you give to
(TBSA) mike;
C. How deep the deepest burns are A. Daily exercise
D. The severity of the burns on a 1 to 5 B. reverse isolation
burn scale. C. Prevent infection
D. Proper nutrition
89. The school nurse notices a child who is wearing
old, dirty, poor-fitting clothes; is always hungry; 95. Exercise precaution must be taken to protect
has no lunch money; and is always tired. When health worker dealing with the AIDS patients .
the nurse asks the boy his tiredness, he talks of which among these must be done as priority:
playing outside until midnight. The nurse will A. Boil used syringe and needles
suspect that this child is: B. Use gloves when handling specimen
A. Being raised by a parent of low C. Label personal belonging
intelligence quotient (IQ) D. Avoid accidental wound
B. An orphan
C. A victim of child neglect Situation: Michelle is a 6 year old preschooler. She was
D. The victim of poverty reported by her sister to have measles but she is at
home because of fever, upper respiratory problem and
90. Which of the following indicates the type(s) of white sports in her mouth.
acute renal failure?
A. Four types: hemorrhagic with and 96. Rubeola is an Arabic term meaning Red, the rash
without clotting, and nonhemorrhagic appears on the skin in invasive stage prior to
with and without clottings eruption behind the ears. As a nurse, your
B. One type: acute physical examination must determine
C. Three types: prerenal, intrarenal and complication especially:
postrenal A. Otitis media
D. Two types: acute and subacute B. Inflammatory conjunctiva
C. Bronchial pneumonia
Situation: Mike 16 y/o has been diagnosed to have AIDS; D. Membranous laryngitis
he worked as entertainer in a cruise ship;
97. To render comfort measure is one of the
91. Which method of transmission is common to priorities, Which includes care of the skin, eyes,
contract AIDS? ears, mouth and nose. To clean the mouth, your
A. Syringe and needles antiseptic solution is in some form of which one
B. Sexual contact below?
C. Body fluids A. Water
D. Transfusion B. Alkaline
C. Sulfur
92. Causative organism in AIDS is one of the D. Salt
following;
A. Fungus 98. As a public health nurse, you teach mother and
B. retrovirus family members the prevention of complication
C. Bacteria of measles. Which of the following should be

2
2
closely watched?
A. Temperature fails to drop
B. Inflammation of the nasophraynx
C. Inflammation of the conjunctiva
D. Ulcerative stomatitis

99. Source of infection of measles is secretion of


nose and throat of infection person. Filterable
virus of measles is transmitted by:
A. Water supply
B. Food ingestion
C. Droplet
D. Sexual contact

100. Method of prevention is to avoid


exposure to an infection person. Nursing
responsibility for rehabilitation of patient
includes the provision of:
A. Terminal disinfection
B. Immunization
C. Injection of gamma globulin
D. Comfort measures
c. 50 days
d. 14 days
NURSING PRACTICE III
Situation: As a nurse researcher you must have a very
Situation: Leo lives in the squatter area. He goes to good understanding of the common terms of concept
nearby school. He helps his mother gather molasses used in research.
after school. One day, he was absent because of fever,
malaise, anorexia and abdominal discomfort. 6. The information that an investigator collects
from the subjects or participants in a research
1. Upon assessment, Leo was diagnosed to have study is usually called;
hepatitis A. Which mode of transmission has the a. Hypothesis
infection agent taken? b. Variable
a. Fecal-oral c. Data
b. Droplet d. Concept
c. Airborne
d. Sexual contact 7. Which of the following usually refers to the
independent variables in doing research
2. Which of the following is concurrent disinfection a. Result
in the case of Leo? b. output
a. Investigation of contact c. Cause
b. Sanitary disposal of faeces, urine and d. Effect
blood
c. Quarantine of the sick individual 8. The recipients of experimental treatment is an
d. removing all detachable objects in the experimental design or the individuals to be
room, cleaning lighting and air duct observed in a non experimental design are
surfaces in the ceiling, and cleaning called;
everything downward to the floor a. Setting
b. Treatment
3. Which of the following must be emphasized c. Subjects
during mother’s class to Leo’s mother? d. Sample
a. Administration of Immunoglobulin to
families 9. The device or techniques an investigator
b. Thorough hand washing before employs to collect data is called;
and after eating and toileting a. Sample
c. Use of attenuated vaccines b. hypothesis
d. Boiling of food especially meat c. Instrument
d. Concept
4. Disaster control should be undertaken when
there are 3 or more hepatitis A cases. Which of 10. The use of another person’s ideas or wordings
these measures is a priority? without giving appropriate credit results from
a. Eliminate faecal contamination from inaccurate or incomplete attribution of materials
foods to its sources. Which of the following is referred
b. Mass vaccination of uninfected to when another person’s idea is inappropriate
individuals credited as one’s own;
c. Health promotion and education to a. Plagiarism
families and communities about the b. assumption
disease it’s cause and transmission c. Quotation
d. Mass administration of Immunoglobulin d. Paraphrase

5. What is the average incubation period of Situation – Mrs. Pichay is admitted to your ward. The
Hepatitis A? MD ordered “Prepare for thoracentesis this pm to
a. 30 days remove excess air from the pleural cavity.”
b. 60 days

2
2
11. Which of the following nursing responsibilities is a. Ease the patient to the floor
essential in Mrs. Pichay who will undergo b. Lift the patient and put him on the bed
thoracentesis? c. Insert a padded tongue depressor
a. Support and reassure client during the between his jaws
procedure d. Restraint patient’s body movement
b. Ensure that informed consent has been
signed 17. Mr Santos is scheduled for CT SCAN for the next
c. Determine if client has allergic reaction day, noon time. Which of the following is the
to local anesthesia correct preparation as instructed by the nurse?
d. Ascertain if chest x-rays and other tests a. Shampoo hair thoroughly to remove oil
have been prescribed and completed and dirt
b. No special preparation is needed.
12. Mrs. Pichay who is for thoracentesis is assigned Instruct the patient to keep his head
by the nurse to which of the following positions? still and stead
a. Trendelenburg position c. Give a cleansing enema and give fluids
b. Supine position until 8 AM
c. Dorsal Recumbent position d. Shave scalp and securely attach
d. Orthopneic position electrodes to it

13. During thoracentesis, which of the following 18. Mr Santos is placed on seizure precaution.
nursing intervention will be most crucial? Which of the following would be
a. Place patient in a quiet and cool room contraindicated?
b. Maintain strict aseptic technique a. Obtain his oral temperature
c. Advice patient to sit perfectly still b. Encourage to perform his own
during needle insertion until it has been personal hygiene
withdrawn from the chest c. Allow him to wear his own clothing
d. Apply pressure over the puncture site as d. Encourage him to be out of bed
soon as the needle is withdrawn
19. Usually, how does the patient behave after his
14. To prevent leakage of fluid in the thoracic cavity, seizure has subsided?
how will you position the client after a. Most comfortable walking and moving
thoracentesis? about
a. Place flat in bed b. Becomes restless and agitated
b. Turn on the unaffected side c. Sleeps for a period of time
c. Turn on the affected side d. Say he is thirsty and hungry
d. On bed rest
20. Before, during and after seizure. The nurse
15. Chest x-ray was ordered after thoracentesis. knows that the patient is ALWAYS placed in what
When your client asks what is the reason for position?
another chest x-ray, you will explain: a. Low fowler’s
a. To rule out pneumothorax b. Side lying
b. To rule out any possible perforation c. Modified trendelenburg
c. To decongest d. Supine
d. To rule out any foreign body
Situation: Mrs. Damian an immediate post op
Situation: A computer analyst, Mr. Ricardo J. Santos, 25 cholecystectomy and choledocholithotomy patient,
was brought to the hospital for diagnostic workup after complained of severe pain at the wound site.
he had experienced seizure in his office.
21. Choledocholithotomy is:
16. Just as the nurse was entering the room, the a. The removal of the gallbladder
patient who was sitting on his chair begins to b. The removal of the stones in the
have a seizure. Which of the following must the gallbladder
nurse do first? c. The removal of the stones in the
common bile duct alleviate anxiety
d. The removal of the stones in the kidney c. Avoid overdosing to prevent
dependence/tolerance
22. The simplest pain relieving technique is: d. Monitor VS, more importantly RR
a. Distraction
b. Deep breathing exercise 28. The client complained of abdominal distention
c. Taking aspirin and pain. Your nursing intervention that can
d. Positioning alleviate pain is:
a. Instruct client to go to sleep and relax
23. Which of the following statement on pain is b. Advice the client to close the lips and
TRUE? avoid deep breathing and talking
a. Culture and pain are not associated c. Offer hot and clear soup
b. Pain accompanies acute illness d. Turn to sides frequently and avoid too
c. Patient’s reaction to pain Varies much talking
d. Pain produces the same reaction such as
groaning and moaning 29. Surgical pain might be minimized by which
nursing action in the O.R.
24. In pain assessment, which of the following a. Skill of surgical team and lesser
condition is a more reliable indicator? manipulation
a. Pain rating scale of 1 to 10 b. Appropriate preparation for the
b. Facial expression and gestures scheduled procedure
c. Physiological responses c. Use of modern technology in closing the
d. Patients description of the pain wound
sensation d. Proper positioning and draping of clients

25. When a client complains of pain, your initial 30. Inadequate anesthesia is said to be one of the
response is: common cause of pain both in intra and post op
a. Record the description of pain patients. If General anesthesia is desired, it will
b. Verbally acknowledge the pain involve loss of consciousness. Which of the
c. Refer the complaint to the doctor following are the 2 general types of GA?
d. Change to a more comfortable position a. Epidural and Spinal
b. Subarachnoid block and Intravenous
Situation: You are assigned at the surgical ward and c. Inhalation and Regional
clients have been complaining of post pain at varying d. Intravenous and Inhalation
degrees. Pain as you know, is very subjective.
Situation: Nurse’s attitudes toward the pain influence
26. A one-day postoperative abdominal surgery the way they perceive and interact with clients in pain.
client has been complaining of severe throbbing
abdominal pain described as 9 in a 1-10 pain 31. Nurses should be aware that older adults are at
rating. Your assessment reveals bowel sounds on risk of underrated pain. Nursing assessment and
all quadrants and the dressing is dry and intact. management of pain should address the
What nursing intervention would you take? following beliefs EXCEPT:
a. Medicate client as prescribed a. Older patients seldom tend to report
b. Encourage client to do imagery pain than the younger ones
c. Encourage deep breathing and turning b. Pain is a sign of weakness
d. Call surgeon stat c. Older patients do not believe in
analgesics, they are tolerant
27. Pentoxidone 5 mg IV every 8 hours was d. Complaining of pain will lead to
prescribed for post abdominal pain. Which will being labeled a ‘bad’ patient
be your priority nursing action?
a. Check abdominal dressing for possible 32. Nurses should understand that when a client
swelling responds favorably to a placebo, it is known as
b. Explain the proper use of PCA to the ‘placebo effect’. Placebos do not indicate

2
2
whether or not a client has: acting insulin first
a. Conscience
b. Disease 37. Janevi complains of nausea, vomiting,
c. Real pain diaphoresis and headache. Which of the
d. Drug tolerance following nursing intervention are you going to
carry out first?
33. You are the nurse in the pain clinic where you a. Withhold the client’s next insulin
have client who has difficulty specifying the injection
location of pain. How can you assist such client? b. Test the client’s blood glucose level
a. The pain is vague c. Administer Tylenol as ordered
b. By charting-it hurts all over d. Offer fruit juice, gelatine and chicken
c. Identify the absence and presence of bouillon
pain
d. As the client to point to the painful are 38. Janevi administered regular insulin at 7 A.M and
by just one finger the nurse should instruct Jane to avoid
exercising at around:
34. What symptom, more distressing than pain, a. 9 to 11 A.M
should the nurse monitor when giving opioids b. Between 8 A.M to 9 A.M
especially among elderly clients who are in pain? c. After 8 hours
a. Forgetfulness d. In the afternoon, after taking lunch
b. Drowsiness
c. Constipation 39. Janevi was brought at the emergency room after
d. Allergic reactions like pruritis four month because she fainted in her clinic. The
nurse should monitor which of the following test
35. Physical dependence occurs in anyone who to evaluate the overall therapeutic compliance
takes opiods over a period of time. What do you of a diabetic patient?
tell a mother of a ‘dependent’ when asked for a. Glycosylated hemoglobin
advice? b. Ketone levels
a. Start another drug and slowly lessen the c. Fasting blood glucose
opioid dosage d. Urine glucose level
b. Indulge in recreational outdoor activities
c. Isolate opioid dependent to a restful 40. Upon the assessment of Hba1c of Mrs. Segovia,
resort The nurse has been informed of a 9% Hba1c
d. Instruct slow tapering of the drug result. In this case, she will teach the patient to:
dosage and alleviate physical a. Avoid infection
withdrawal symptoms b. Prevent and recognize hyperglycaemia
c. Take adequate food and nutrition
Situation: The nurse is performing health education d. Prevent and recognize hypoglycaemia
activities for Janevi Segovia, a 30 year old Dentist with
Insulin dependent diabetes Miletus. 41. The nurse is teaching plan of care for Jane with
regards to proper foot care. Which of the
36. Janevi is preparing a mixed dose of insulin. The following should be included in the plan?
nurse is satisfied with her performance when a. Soak feet in hot water
she: b. Avoid using mild soap on the feet
a. Draw insulin from the vial of clear c. Apply a moisturizing lotion to dry feet
insulin first but not between the toes
b. Draw insulin from the vial of the d. Always have a podiatrist to cut your toe
intermediate acting insulin first nails; never cut them yourself
c. Fill both syringes with the prescribed
insulin dosage then shake the bottle 42. Another patient was brought to the emergency
vigorously room in an unresponsive state and a diagnosis of
d. Withdraw the intermediate acting hyperglycaemic hyperosmolar nonketotic
insulin first before withdrawing the short syndrome is made. The nurse immediately
prepares to initiate which of the following of which of the following physiologic changes
anticipated physician’s order? associated with aging.
a. Endotracheal intubation a. Ineffective airway clearance
b. 100 unites of NPH insulin b. Decreased alveolar surfaced area
c. Intravenous infusion of normal saline c. Decreased anterior-posterior chest
d. Intravenous infusion of sodium diameter
bicarbonate d. Hyperventilation

43. Jane eventually developed DKA and is being 47. The older patient is at higher risk for
treated in the emergency room. Which finding incontinence because of:
would the nurse expect to note as confirming a. Dilated urethra
this diagnosis? b. Increased glomerular filtration rate
a. Comatose state c. Diuretic use
b. Decreased urine output d. Decreased bladder capacity
c. Increased respiration and an increase in
pH 48. Merle, age 86, is complaining of dizziness when
d. Elevated blood glucose level and low she stands up. This may indicate:
plasma bicarbonate level a. Dementia
b. Functional decline
44. The nurse teaches Jane to know the difference c. A visual problem
between hypoglycaemia and ketoacidosis. Jane d. Drug toxicity
demonstrates understanding of the teaching by
stating that glucose will be taken if which of the 49. Cardiac ischemia in an older patient usually
following symptoms develops? produces:
a. Polyuria a. ST-T wave changes
b. Shakiness b. Chest pain radiating to the left arm
c. Blurred Vision c. Very high creatinine kinase level
d. Fruity breath odour d. Acute confusion

45. Jane has been scheduled to have a FBS taken in 50. The most dependable sign of infection in the
the morning. The nurse tells Jane not to eat or older patient is:
drink after midnight. Prior to taking the blood a. Change in mental status
specimen, the nurse noticed that Jane is holding b. Fever
a bottle of distilled water. The nurse asked Jane c. Pain
if she drink any, and she said “yes.” Which of the d. Decreased breath sounds with crackles
following is the best nursing action?
a. Administer syrup of ipecac to remove Situation – In the OR, there are safety protocols that
the distilled water from the stomach should be followed. The OR nurse should be well versed
b. Suction the stomach content using NGT with all these to safeguard the safety and quality of
prior to specimen collection patient delivery outcome.
c. Advice to physician to reschedule to
diagnostic examination next day 51. Which of the following should be given highest
d. Continue as usual and have the FBS priority when receiving patient in the OR?
analysis performed and specimen be a. Assess level of consciousness
taken b. Verify patient identification and
informed consent
Situation: Elderly clients usually produce unusual signs c. Assess vital signs
when it comes to different diseases. The ageing process d. Check for jewelry, gown, manicure, and
is a complicated process and the nurse should dentures
understand that it is an inevitable fact and she must be
prepared to care for the growing elderly population. 52. Surgeries like I and D (incision and drainage) and
debridement are relatively short procedures but
46. Hypoxia may occur in the older patients because considered ‘dirty cases’. When are these

3
3
procedures best scheduled? The nurse knows that the temperature and time
a. Last case is set to the optimum level to destroy not only
b. In between cases the microorganism, but also the spores. Which
c. According to availability of of the following is the ideal setting of the
anaesthesiologist autoclave machine?
d. According to the surgeon’s preference a. 10,000 degree Celsius for 1 hour
b. 5,000 degree Celsius for 30 minutes
53. OR nurses should be aware that maintaining the c. 37 degree Celsius for 15 minutes
client’s safety is the overall goal of nursing care d. 121 degree Celsius for 15 minutes
during the intraoperative phase. As the
circulating nurse, you make certain that 58. It is important that before a nurse prepares the
throughout the procedure… material to be sterilized, a chemical indicator
a. the surgeon greets his client before strip should be placed above the package,
induction of anesthesia preferably, Muslin sheet. What is the color of
b. the surgeon and anesthesiologist are in the striped produced after autoclaving?
tandem a. Black
c. strap made of strong non-abrasive b. Blue
materials are fastened securely around c. Gray
the joints of the knees and ankles and d. Purple
around the 2 hands around an arm
board. 59. Chemical indicators communicate that:
d. Client is monitored throughout the a. The items are sterile
surgery by the assistant anesthesiologist b. That the items had undergone
sterilization process but not necessarily
54. Another nursing check that should not be missed sterile
before the induction of general anesthesia is: c. The items are disinfected
a. check for presence underwear d. That the items had undergone
b. check for presence dentures disinfection process but not necessarily
c. check patient’s ID disinfected
d. check baseline vital signs
60. If a nurse will sterilize a heat and moisture labile
55. Some lifetime habits and hobbies affect instruments, It is according to AORN
postoperative respiratory function. If your client recommendation to use which of the following
smokes 3 packs of cigarettes a day for the past method of sterilization?
10 years, you will anticipate increased risk for: a. Ethylene oxide gas
a. perioperative anxiety and stress b. Autoclaving
b. delayed coagulation time c. Flash sterilizer
c. delayed wound healing d. Alcohol immersion
d. postoperative respiratory infection
Situation 5 – Nurses hold a variety of roles when
Situation: Sterilization is the process of removing ALL providing care to a perioperative patient.
living microorganism. To be free of ALL living
microorganism is sterility. 61. Which of the following role would be the
responsibility of the scrub nurse?
56. There are 3 general types of sterilization use in a. Assess the readiness of the client prior
the hospital, which one is not included? to surgery
a. Steam sterilization b. Ensure that the airway is adequate
b. Physical sterilization c. Account for the number of sponges,
c. Chemical sterilization needles, supplies, used during the
d. Sterilization by boiling surgical procedure.
d. Evaluate the type of anesthesia
57. Autoclave or steam under pressure is the most appropriate for the surgical client
common method of sterilization in the hospital.
62. As a perioperative nurse, how can you best meet patients will need surgical amputation but there
the safety need of the client after administering are no sterile surgical equipments. In this case,
preoperative narcotic? which of the following will the nurse expect?
a. Put side rails up and ask the client not a. Equipments needed for surgery need not
to get out of bed be sterilized if this is an emergency
b. Send the client to OR with the family necessitating life saving measures
c. Allow client to get up to go to the b. Forwarding the trauma client to the
comfort room nearest hospital that has available sterile
d. Obtain consent form equipments is appropriate
c. The nurse will need to sterilize the item
63. It is the responsibility of the pre-op nurse to do before using it to the client using the
skin prep for patients undergoing surgery. If hair regular sterilization setting at 121
at the operative site is not shaved, what should degree Celsius in 15 minutes
be done to make suturing easy and lessen d. In such cases, flash sterlizer will be use
chance of incision infection? at 132 degree Celsius in 3 minutes
a. Draped
b. Pulled 68. Tess, the PACU nurse, discovered that Malou,
c. Clipped who weighs 110 lbs prior to surgery, is in severe
d. Shampooed pain 3 hrs after cholecystectomy. Upon checking
the chart, Malou found out that she has an order
64. It is also the nurse’s function to determine when of Demerol 100 mg I.M. prn for pain. Tess should
infection is developing in the surgical incision. verify the order with:
The perioperative nurse should observe for what a. Nurse Supervisor
signs of impending infection? b. Surgeon
a. Localized heat and redness c. Anesthesiologist
b. Serosanguinous exudates and skin d. Intern on duty
blanching
c. Separation of the incision 69. Rosie, 57, who is diabetic is for debridement if
d. Blood clots and scar tissue are visible incision wound. When the circulating nurse
checked the present IV fluid, she found out that
65. Which of the following nursing interventions is there is no insulin incorporated as ordered.
done when examining the incision wound and What should the circulating nurse do?
changing the dressing? a. Double check the doctor’s order and
a. Observe the dressing and type and call the attending MD
odor of drainage if any b. Communicate with the ward nurse to
b. Get patient’s consent verify if insulin was incorporated or not
c. Wash hands c. Communicate with the client to verify if
d. Request the client to expose the incision insulin was incorporated
wound d. Incorporate insulin as ordered.

Situation – The preoperative nurse collaborates with the 70. The documentation of all nursing activities
client significant others, and healthcare providers. performed is legally and professionally vital.
Which of the following should NOT be included
66. To control environmental hazards in the OR, the in the patient’s chart?
nurse collaborates with the following a. Presence of prosthetoid devices such as
departments EXCEPT: dentures, artificial limbs hearing aid, etc.
a. Biomedical division b. Baseline physical, emotional, and
b. Infection control committee psychosocial data
c. Chaplaincy services c. Arguments between nurses and
d. Pathology department residents regarding treatments
d. Observed untoward signs and symptoms
67. An air crash occurred near the hospital leading and interventions including contaminant
to a surge of trauma patient. One of the last intervening factors

3
3

Situation: Basic knowledge on Intravenous solutions is


Situation – Team efforts is best demonstrated in the OR. necessary for care of clients with problems with fluids
and electrolytes.
71. If you are the nurse in charge for scheduling
surgical cases, what important information do 76. A client involved in a motor vehicle crash
you need to ask the surgeon? presents to the emergency department with
a. Who is your internist severe internal bleeding. The client is severely
b. Who is your assistant and hypotensive and unresponsive. The nurse
anaesthesiologist, and what is your anticipates which of the following intravenous
preferred time and type of surgery? solutions will most likely be prescribed to
c. Who are your anaesthesiologist, increase intravascular volume, replace
internist, and assistant immediate blood loss and increase blood
d. Who is your anaesthesiologist pressure?
a. 0.45% sodium chloride
72. In the OR, the nursing tandem for every surgery b. 0.33% sodium chloride
is: c. Normal saline solution
a. Instrument technician and circulating d. Lactated ringer’s solution
nurse
b. Nurse anaesthetist, nurse assistant, and 77. The physician orders the nurse to prepare an
instrument technician isotonic solution. Which of the following IV
c. Scrub nurse and nurse anaesthetist solution would the nurse expect the intern to
d. Scrub and circulating nurses prescribe?
a. 5% dextrose in water
73. While team effort is needed in the OR for b. 0.45% sodium chloride
efficient and quality patient care delivery, we c. 10% dextrose in water
should limit the number of people in the room d. 5% dextrose in 0.9% sodium chloride
for infection control. Who comprise this team?
a. Surgeon, anaesthesiologist, scrub nurse, 78. The nurse is making initial rounds on the nursing
radiologist, orderly unit to assess the condition of assigned clients.
b. Surgeon, assistants, scrub nurse, The nurse notes that the client’s IV Site is cool,
circulating nurse, anaesthesiologist pale and swollen and the solution is not infusing.
c. Surgeon, assistant surgeon, The nurse concludes that which of the following
anaesthesiologist, scrub nurse, complications has been experienced by the
pathologist client?
d. Surgeon, assistant surgeon, a. Infection
anaesthesiologist, intern, scrub nurse b. Phlebitis
c. Infiltration
74. Who usually act as an important part of the OR d. Thrombophelibitis
personnel by getting the wheelchair or stretcher,
and pushing/pulling them towards the operating 79. A nurse reviews the client’s electrolyte
room? laboratory report and notes that the potassium
a. Orderly/clerk level is 3.2 mEq/L. Which of the following would
b. Nurse Supervisor the nurse note on the electrocardiogram as a
c. Circulating Nurse result of the laboratory value?
d. Anaesthesiologist a. U waves
b. Absend P waves
75. The breakdown in teamwork is often times a c. Elevated T waves
failure in: d. Elevated ST segment
a. Electricity
b. Inadequate supply 80. One patient had a ‘runaway’ IV of 50% dextrose.
c. Leg work To prevent temporary excess of insulin or
d. Communication transient hyperinsulin reaction what solution
you prepare in anticipation of the doctor’s
order?
a. Any IV solution available to KVO 86. As an OR nurse, what are your foremost
b. Isotonic solution considerations for selecting chemical agents for
c. Hypertonic solution disinfection?
d. Hypotonic solution a. Material compatibility and efficiency
b. Odor and availability
81. An informed consent is required for: c. Cost and duration of disinfection process
a. closed reduction of a fracture d. Duration of disinfection and efficiency
b. irrigation of the external ear canal
c. insertion of intravenous catheter 87. Before you use a disinfected instrument it is
d. urethral catheterization essential that you:
a. Rinse with tap water followed by alcohol
82. Which of the following is not true with regards b. Wrap the instrument with sterile water
to the informed consent? c. Dry the instrument thoroughly
a. It should describe different treatment d. Rinse with sterile water
alternatives
b. It should contain a thorough and 88. You have a critical heat labile instrument to
detailed explanation of the procedure sterilize and are considering to use high level
to be done disinfectant. What should you do?
c. It should describe the client’s diagnosis a. Cover the soaking vessel to contain the
d. It should give an explanation of the vapor
client’s prognosis b. Double the amount of high
level disinfectant
83. You know that the hallmark of nursing c. Test the potency of the high level
accountability is the: disinfectant
a. accurate documentation and reporting d. Prolong the exposure time according to
b. admitting your mistakes manufacturer’s direction
c. filing an incidence report
d. reporting a medication error 89. To achieve sterilization using disinfectants,
which of the following is used?
84. A nurse is assigned to care for a group of clients. a. Low level disinfectants immersion in 24
On review of the client’s medical records, the hours
nurse determines that which client is at risk for b. Intermediate level disinfectants
excess fluid volume? immersion in 12 hours
a. The client taking diuretics c. High level disinfectants immersion in 1
b. The client with renal failure hour
c. The client with an ileostomy d. High level disinfectant immersion in 10
d. The client who requires gastrointestinal hours
suctioning
90. Bronchoscope, Thermometer, Endoscope, ET
85. A nurse is assigned to care for a group of clients. tube, Cytoscope are all BEST sterilized using
On review of the client’s medical records, the which of the following?
nurse determines that which client is at risk for a. Autoclaving at 121 degree Celsius in 15
deficient fluid volume? minutes
a. A client with colostomy b. Flash sterilizer at 132 degree Celsius in 3
b. A client with congestive heart failure minutes
c. A client with decreased kidney function c. Ethylene Oxide gas aeration for 20 hours
d. A client receiving frequent wound d. 2% Glutaraldehyde immersion for 10
irrigation hours

Situation: As a perioperative nurse, you are aware of the Situation: The OR is divided into three zones to control
correct processing methods for preparing instruments traffic flow and contamination
and other devices for patient use to prevent infection.

3
3
91. What OR attires are worn in the restricted area? process
a. Scrub suit, OR shoes, head cap
b. Head cap, scrub suit, mask, OR shoes 97. 2 organizations endorsed that sterility are
c. Mask, OR shoes, scrub suit affected by factors other than the time itself,
d. Cap, mask, gloves, shoes these are:
a. The PNA and the PRC
92. Nursing intervention for a patient on low dose IV b. AORN and JCAHO
insulin therapy includes the following, EXCEPT: c. ORNAP and MCNAP
a. Elevation of serum ketones to monitor d. MMDA and DILG
ketosis
b. Vital signs including BP 98. All of these factors affect the sterility of the OR
c. Estimate serum potassium equipments, these are the following except:
d. Elevation of blood glucose levels a. The material used for packaging
b. The handling of the materials as well as
93. The doctor ordered to incorporate 1000”u” its transport
insulin to the remaining on-going IV. The c. Storage
strength is 500 /ml. How much should you d. The chemical or process used in
incorporate into the IV solution? sterililzing the material
a. 10 ml
b. 0.5 ml 99. When you say sterile, it means:
c. 2 ml a. The material is clean
d. 5 ml b. The material as well as the equipments
are sterilized and had undergone a
94. Multiple vial-dose-insulin when in use should be rigorous sterilization process
a. Kept at room temperature c. There is a black stripe on the paper
b. Kept in narcotic cabinet indicator
c. Kept in the refrigerator d. The material has no microorganism nor
d. Store in the freezer spores present that might cause an
infection
95. Insulins using insulin syringe are given using how
many degrees of needle insertion? 100. In using liquid sterilizer versus autoclave
a. 45 machine, which of the following is true?
b. 180 a. Autoclave is better in sterilizing OR
c. 90 supplies versus liquid sterilizer
d. 15 b. They are both capable of sterilizing the
equipments, however, it is necessary to
Situation: Maintenance of sterility is an important soak supplies in the liquid sterilizer for
function a nurse should perform in any OR setting. a longer period of time
c. Sharps are sterilized using autoclave and
96. Which of the following is true with regards to not cidex
sterility? d. If liquid sterilizer is used, rinsing it
a. Sterility is time related, items are not before using is not necessary
considered sterile after a period of 30
days of being not use.
b. for 9 months, sterile items are
considered sterile as long as they are
covered with sterile muslin cover and
stored in a dust proof covers.
c. Sterility is event related, not time
related
d. For 3 weeks, items double covered with
muslin are considered sterile as long as
they have undergone the sterilization
d. CT Scan and Incidence report

NURSING PRACTICE IV Situation: An entry level nurse should be able to apply


theoretical knowledge in the performance of the basic
Situation: After an abdominal surgery, the circulating nursing skills.
and scrub nurses have critical responsibility about
sponge and instrument count. 6. A client has an indwelling urinary catheter and
she is suspected of having urinary infection. How
1. Counting is performed thrice: During the should you collect a urine specimen for culture
preincision phase, the operative phase and and sensitivity?
closing phase. Who counts the sponges, needles a. clamp tubing for 60 minutes and insert a
and instruments? sterile needle into the tubing above the
a. The scrub nurse only clamp to aspirate urine
b. The circulating nurse only b. drain urine from the drainage bag into
c. The surgeon and the assistant surgeon the sterile container
d. The scrub nurse and the circulating c. disconnect the tubing from the urinary
nurse catheter and let urine flow into a sterile
container
2. The layer of the abdomen is divided into 5. d. wipe the self-sealing aspiration port
Arrange the following from the first layer going with antiseptic solution and insert a
to the deepest layer: sterile needle into the self-sealing port
1. Fascia
2. Muscle 7. To obtain specimen for sputum culture and
3. Peritoneum sensitivity, which of the following instruction is
4. Subcutaneous/Fat best?
5. Skin a. Upon waking up, cough deeply and
a. 5,4,3,2,1 expectorate into container
b. 5,4,1,3,2 b. Cough after pursed lip breathing
c. 5,4,2,1,3 c. Save sputum for two days in covered
d. 5,4,1,2,3 container
d. After respiratory treatment, expectorate
3. When is the first sponge/instrument count into a container
reported?
a. Before closing the subcutaneous layer 8. The best time for collecting the sputum
b. Before peritoneum is closed specimen for culture and sensitivity is:
c. Before closing the skin a. Before retiring at night
d. Before the fascia is sutured b. Anytime of the day
c. Upon waking up in the morning
4. Like any nursing interventions, counts should be d. Before meals
documented. To whom does the scrub nurse
report any discrepancy of counts so that 9. When suctioning the endotracheal tube, the
immediate and appropriate action is instituted? nurse should:
a. Anaesthesiologists a. Explain procedure to patient; insert
b. Surgeon catheter gently applying suction.
c. OR nurse supervisor Withdrawn using twisting motion
d. Circulating nurse b. Insert catheter until resistance is met,
and then withdraw slightly, applying
5. Which of the following are 2 interventions of the suction intermittently as catheter is
surgical team when an instrument was withdrawn
confirmed missing? c. Hyperoxygenate client insert catheter
a. MRI and Incidence report using back and forth motion
b. CT Scan, MRI, Incidence report d. Insert suction catheter four inches into
c. X-RAY and Incidence report the tube, suction 30 seconds using

3
3
twirling motion as catheter is withdrawn Nursing intervention includes:
a. Bed rest
10. The purpose of NGT IMMEDIATELY after an b. Warm moist soak
operation is: c. Early ambulation
a. For feeding or gavage d. Hot sitz bath
b. For gastric decompression
c. For lavage, or the cleansing of the Situation – Mang Felix, a 79 year old man who is brought
stomach content to the Surgical Unit from PACU after a transurethral
d. For the rapid return of peristalsis resection. You are assigned to receive him. You noted
that he has a 3-way indwelling urinary catheter for
Situation - Mr. Santos, 50, is to undergo cystoscopy due continuous fast drip bladder irrigation which is
to multiple problems like scantly urination, hematuria connected to a straight drainage.
and dysuria.
16. Immediately after surgery, what would you
11. You are the nurse in charge in Mr. Santos. When expect his urine to be?
asked what are the organs to be examined a. Light yellow
during cystoscopy, you will enumerate as b. Bright red
follows: c. Amber
a. Urethra, kidney, bladder, urethra d. Pinkish to red
b. Urethra, bladder wall, trigone, ureteral
opening 17. The purpose of the continuous bladder irrigation
c. Bladder wall, uterine wall, and urethral is to:
opening a. Allow continuous monitoring of the fluid
d. Urethral opening, ureteral opening output status
bladder b. Provide continuous flushing of clots and
debris from the bladder
12. In the OR, you will position Mr. Santos who is c. Allow for proper exchange of
cystoscopy in: electrolytes and fluid
a. Supine d. Ensure accurate monitoring of intake
b. Lithotomy and output
c. Semi-fowler
d. Trendelenburg 18. Mang Felix informs you that he feels some
discomfort on the hypogastric area and he has
13. After cystoscopy, Mr. Santos asked you to to void. What will be your most appropriate
explain why there is no incision of any kind. action?
What do you tell him? a. Remove his catheter then allow him to
a. “Cystoscopy is direct visualization and void on his own
examination by urologist”. b. Irrigate his catheter
b. “Cystoscopy is done by x-ray c. Tell him to “Go ahead and void. You
visualization of the urinary tract”. have an indwelling catheter.”
c. “Cystoscopy is done by using lasers on d. Assess color and rate of outflow, if
the urinary tract”. there is changes refer to urologist for
d. “Cystoscopy is an endoscopic procedure possible irrigation.
of the urinary tract”.
19. You decided to check on Mang Felix’s IV fluid
14. Within 24-48 hours post cystoscopy, it is normal infusion. You noted a change in flow rate, pallor
to observe one the following: and coldness around the insertion site. What is
a. Pink-tinged urine your assessment finding?
b. Distended bladder a. Phlebitis
c. Signs of infection b. Infiltration to subcutaneous tissue
d. Prolonged hematuria c. Pyrogenic reaction
d. Air embolism
15. Leg cramps are NOT uncommon post cystoscopy.
20. Knowing that proper documentation of
assessment findings and interventions are d. Fever, Irritability and a large output
important responsibilities of the nurse during of diluted urine
first post-operative day, which of the following is
the LEAST relevant to document in the case of 25. What kind of renal failure will melamine
Mang Felix? poisoning cause?
a. Chest pain and vital signs a. Chronic, Prerenal
b. Intravenous infusion rate b. Chronic, Intrarenal
c. Amount, color, and consistency of c. Acute, Postrenal
bladder irrigation drainage d. Acute, Prerenal
d. Activities of daily living started
Situation: Leukemia is the most common type of
Situation: Melamine contamination in milk has brought childhood cancer. Acute Lymphoid Leukemia is the cause
worldwide crisis both in the milk production sector as of almost 1/3 of all cancer that occurs in children under
well as the health and economy. Being aware of the age 15.
current events is one quality that a nurse should possess
to prove that nursing is a dynamic profession that will 26. The survival rate for Acute Lymphoid Leukemia is
adapt depending on the patient’s needs. approximately:
a. 25%
21. Melamine is a synthetic resin used for b. 40%
whiteboards, hard plastics and jewellery box c. 75%
covers due to its fire retardant properties. Milk d. 95%
and food manufacturers add melamine in order
to: 27. Whereas acute nonlymphoid leukemia has a
a. It has a bacteriostatic property leading survival rate of:
to increase food and milk life as a way of a. 25%
preserving the foods b. 40%
b. Gives a glazy and more edible look on c. 75%
foods d. 95%
c. Make milks more tasty and creamy
d. Create an illusion of a high protein 28. The three main consequence of leukemia that
content on their products cause the most danger is:
a. Neutropenia causing infection, anemia
22. Most of the milks contaminated by Melamine causing impaired oxygenation and
came from which country? thrombocytopenia leading to bleeding
a. India tendencies
b. China b. Central nervous system infiltration,
c. Philippines anemia causing impaired oxygenation
d. Korea and thrombocytopenia leading to
bleeding tendencies
23. Which government agency is responsible for c. Splenomegaly, hepatomegaly, fractures
testing the melamine content of foods and food d. Invasion by the leukemic cells to the
products? bone causing severe bone pain
a. DOH
b. MMDA 29. Gold standard in the diagnosis of leukemia is by
c. NBI which of the following?
d. BFAD a. Blood culture and sensitivity
b. Bone marrow biopsy
24. Infants are the most vulnerable to melamine c. Blood biopsy
poisoning. Which of the following is NOT a sign d. CSF aspiration and examination
of melamine poisoning?
a. Irritability, Back ache, Urolithiasis 30. Adriamycin,Vincristine,Prednisone and L
b. High blood pressure, fever asparaginase are given to the client for long
c. Anuria, Oliguria or Hematuria term therapy. One common side effect,

3
3
especially of adriamycin is alopecia. The child sensitivity of the breast.
asks: “Will I get my hair back once again?” The
nurse best respond is by saying: 34. Carmen, who is asking the nurse the most
a. “Don’t be silly, ofcourse you will get your appropriate time of the month to do her self-
hair back” examination of the breast. The MOST
b. “We are not sure, let’s hope it’ll grow” appropriate reply by the nurse would be:
c. “This side effect is usually permanent, a. the 26th day of the menstrual cycle
But I will get the doctor to discuss it for b. 7 to 8 days after conclusion of the
you” menstrual period
d. “Your hair will regrow in 3 to 6 months c. during her menstruation
but of different color, usually darker d. the same day each month
and of different texture”
35. Carmen being treated with radiation therapy.
Situation: Breast Cancer is the 2nd most common type of What should be included in the plan of care to
cancer after lung cancer and 99% of which, occurs in minimize skin damage from the radiation
woman. Survival rate is 98% if this is detected early and therapy?
treated promptly. Carmen is a 53 year old patient in the a. Cover the areas with thick clothing
high risk group for breast cancer was recently diagnosed materials
with Breast cancer. b. Apply a heating pad to the site
c. Wash skin with water after the therapy
31. All of the following are factors that said to d. Avoid applying creams and powders to
contribute to the development of breast cancer the area
except:
a. Prolonged exposure to estrogen such as 36. Based on the DOH and World Health
an early menarche or late menopause, Organization (WHO) guidelines, the mainstay for
nulliparity and childbirth after age 30 early detection method for breast cancer that is
b. Genetics recommended for developing countries is:
c. Increasing Age a. a monthly breast self-examination (BSE)
d. Prolonged intake of Tamoxifen and an annual health worker breast
(Nolvadex) examination (HWBE)
b. an annual hormone receptor assay
32. Protective factors for the development of breast c. an annual mammogram
cancer includes which of the following except: d. a physician conduct a breast clinical
a. Exercise examination every 2 years
b. Breast feeding
c. Prophylactic Tamoxifen 37. The purpose of performing the breast self-
d. Alcohol intake examination (BSE) regularly is to discover:
a. fibrocystic masses
33. A patient diagnosed with breast cancer has been b. areas of thickness or fullness
offered the treatment choices of breast c. cancerous lumps
conservation surgery with radiation or a d. changes from previous BSE
modified radical mastectomy. When questioned
by the patient about these options, the nurse 38. If you are to instruct a postmenopausal woman
informs the patient that the lumpectomy with about BSE, when would you tell her to do BSE:
radiation: a. on the same day of each month
a. reduces the fear and anxiety that b. on the first day of her menstruation
accompany the diagnosis and treatment c. right after the menstrual period
of cancer d. on the last day of her menstruation
b. has about the same 10-year survival rate
as the modified radical mastectomy 39. During breast self-examination, the purpose of
c. provides a shorter treatment period with standing in front of the mirror it to observe the
a fewer long term complications breast for:
d. preserves the normal appearance and a. thickening of the tissue
b. lumps in the breast tissue d. Ineffective tissue perfusion, peripheral,
c. axillary lymphnodes cerebral, cardiovascular,
d. change in size and contour gastrointestinal, renal

40. When preparing to examine the left breast in a 45. What intervention should you include in your
reclining position, the purpose of placing a small care plan?
folded towel under the client’s left shoulder is a. Inspect his skin for petechiae, bruising,
to: GI bleeding regularly
a. bring the breast closer to the examiner’s b. Place Albert on strict isolation
right hand precaution
b. tense the pectoral muscle c. Provide rest in between activities
c. balance the breast tissue more evenly d. Administer antipyretics if his
on the chest wall temperature exceeds 38C
d. facilitate lateral positioning of the breast
Situation: Burn are cause by transfer of heat source to
Situation – Radiation therapy is another modality of the body. It can be thermal, electrical, radiation or
cancer management. With emphasis on multidisciplinary chemical.
management you have important responsibilities as
nurse. 46. A burn characterized by Pale white appearance,
charred or with fat exposed and painlessness is:
41. Albert is receiving external radiation therapy and a. Superficial partial thickness burn
he complains of fatigue and malaise. Which of b. Deep partial thickness burn
the following nursing interventions would be c. Full thickness burn
most helpful for Albert? d. Deep full thickness burn
a. Tell him that sometimes these feelings
can be psychogenic 47. Which of the following BEST describes superficial
b. Refer him to the physician partial thickness burn or first degree burn?
c. Reassure him that these feelings are a. Structures beneath the skin are damage
normal b. Dermis is partially damaged
d. Help him plan his activities c. Epidermis and dermis are both damaged
d. Epidermis is damaged
42. Immediately following the radiation teletherapy,
Albert is 48. A burn that is said to be “WEEPING” is classified
a. Considered radioactive for 24 hrs as:
b. Given a complete bath a. Superficial partial thickness burn
c. Placed on isolation for 6 hours b. Deep partial thickness burn
d. Free from radiation c. Full thickness burn
d. Deep full thickness burn
43. Albert is admitted with a radiation induced
thrombocytopenia. As a nurse you should 49. During the Acute phase of the burn injury, which
observe the following symptoms: of the following is a priority?
a. Petechiae, ecchymosis, epistaxis a. wound healing
b. Weakness, easy fatigability, pallor b. emotional support
c. Headache, dizziness, blurred vision c. reconstructive surgery
d. Severe sore throat, bacteremia, d. fluid resuscitation
hepatomegaly
50. While in the emergent phase, the nurse knows
44. What nursing diagnosis should be of highest that the priority is to:
priority? a. Prevent infection
a. Knowledge deficit regarding b. Prevent deformities and contractures
thrombocytopenia precautions c. Control pain
b. Activity intolerance d. Return the hemodynamic stability via
c. Impaired tissue integrity fluid resuscitation

4
4

the client is developing:


51. The MOST effective method of delivering pain a. Cerebral hypoxia
medication during the emergent phase is: b. metabolic acidosis
a. intramuscularly c. Hypervolemia
b. orally d. Renal failure
c. subcutaneously
d. intravenously 58. A 165 lbs trauma client was rushed to the
emergency room with full thickness burns on the
52. When a client accidentally splashes chemicals to whole face, right and left arm, and at the
his eyes, The initial priority care following the anterior upper chest sparing the abdominal area.
chemical burn is to: He also has superficial partial thickness burn at
a. irrigate with normal saline for 1 to 15 the posterior trunk and at the half upper portion
minutes of the left leg. He is at the emergent phase of
b. transport to a physician immediately burn. Using the parkland’s formula, you know
c. irrigate with water for 15 minutes or that during the first 8 hours of burn, the amount
longer of fluid will be given is:
d. cover the eyes with a sterile gauze a. 5,400 ml
b. 9, 450 ml
53. Which of the following can be a fatal c. 10,800 ml
complication of upper airway burns? d. 6,750 ml
a. stress ulcers
b. shock 59. The doctor incorporated insulin on the client’s
c. hemorrhage fluid during the emergent phase. The nurse
d. laryngeal spasms and swelling knows that insulin is given because:
a. Clients with burn also develops
54. When a client will rush towards you and he has a Metabolic acidosis
burning clothes on, It is your priority to do which b. Clients with burn also develops
of the following first? hyperglycemia
a. log roll on the grass/ground c. Insulin is needed for additional energy
b. slap the flames with his hands and glucose burning after the stressful
c. Try to remove the burning clothes incidence to hasten wound healing,
d. Splash the client with 1 bucket of cool regain of consciousness and rapid return
water of hemodynamic stability
d. For hyperkalemia
55. Once the flames are extinguished, it is most
important to: 60. The IV fluid of choice for burn as well as
a. cover clientwith a warm blanket dehydration is:
b. give him sips of water a. 0.45% NaCl
c. calculate the extent of his burns b. Sterile water
d. assess the Sergio’s breathing c. NSS
d. D5LR
56. During the first 24 hours after the thermal injury,
you should asses Sergio for: Situation: ENTEROSTOMAL THERAPY is now considered a
a. hypokalemia and hypernatremia specialty in nursing. You are participating in the OSTOMY
b. hypokalemia and hyponatremia CARE CLASS.
c. hyperkalemia and hyponatremia
d. hyperkalemia and hypernatremia 61. You plan to teach Fermin how to irrigate the
colostomy when:
57. A client who sustained deep partial thickness a. The perineal wound heals And Fermin
and full thickness burns of the face, whole can sit comfortably on the commode
anterior chest and both upper extremities two b. Fermin can lie on the side comfortably,
days ago begins to exhibit extreme restlessness. about the 3rd postoperative day
You recognize that this most likely indicates that c. The abdominal incision is closed and
contamination is no longer a danger it is important for nurses to gather as much information
d. The stools starts to become formed, to be able to address their needs for nursing care.
around the 7th postoperative day
66. Critically ill patients frequently complain about
62. When preparing to teach Fermin how to irrigate which of the following when hospitalized?
colostomy, you should plan to do the procedure: a. Hospital food
a. When Fermin would have normal bowel b. Lack of privacy
movement c. Lack of blankets
b. At least 2 hours before visiting hours d. Inadequate nursing staff
c. Prior to breakfast and morning care
d. After Fermin accepts alteration in body 67. Who of the following is at greatest risk of
image developing sensory problem?
a. Female patient
63. When observing a return demonstration of a b. Transplant patient
colostomy irrigation, you know that more c. Adoloscent
teaching is required if Fermin: d. Unresponsive patient
a. Lubricates the tip of the catheter prior to
inserting into the stoma 68. Which of the following factors may inhibit
b. Hangs the irrigating bag on the learning in critically ill patients?
bathroom door cloth hook during fluid a. Gender
insertion b. Educational level
c. Discontinues the insertion of fluid after c. Medication
500 ml of fluid has been instilled d. Previous knowledge of illness
d. Clamps of the flow of fluid when felling
uncomfortable 69. Which of the following statements does not
apply to critically ill patients?
64. You are aware that teaching about colostomy a. Majority need extensive rehabilitation
care is understood when Fermin states, “I will b. All have been hospitalized previously
contact my physician and report: c. Are physically unstable
a. If I have any difficulty inserting the d. Most have chronic illness
irrigating tub into the stoma.”
b. If I noticed a loss of sensation to touch in 70. Families of critically ill patients desire which of
the stoma tissue.” the following needs to be met first by the
c. The expulsion of flatus while the nurse?
irrigating fluid is running out.” a. Provision of comfortable space
d. When mucus is passed from the stoma b. Emotional support
between the irrigations.” c. Updated information on client’s status
d. Spiritual counselling
65. You would know after teaching Fermin that
dietary instruction for him is effective when he Situation: Johnny, sought consultation to the hospital
states, “It is important that I eat: because of fatigability, irritability, jittery and he has been
a. Soft food that is easily digested and experiencing this sign and symptoms for the past 5
absorbed by my large intestines.” months.
b. Bland food so that my intestines do not
become irritated.” 71. His diagnosis was hyperthyroidism, the following
c. Food low in fiber so that there are fewer are expected symptoms except:
stools.” a. Anorexia
d. Everything that I ate before the b. Fine tremors of the hand
operation, while avoiding foods that c. Palpitation
cause gas”. d. Hyper alertness

Situation: Based on studies of nurses working in special 72. She has to take drugs to treat her
units like the intensive care unit and coronary care unit, hyperthyroidism. Which of the following will you

4
4
NOT expect that the doctor will prescribe?
4
a. Colace (Docusate) to:
b. Tapazole (Methimazole) a. Decrease the vascularity and size of the
c. Cytomel (Liothyronine) thyroid gland
d. Synthroid (Levothyroxine) b. Decrease the size of the thyroid gland
only
73. The nurse knows that Tapazole has which of the c. Increase the vascularity and size of the
following side effect that will warrant immediate thyroid gland
withholding of the medication? d. Increase the size of the thyroid gland
a. Death only
b. Hyperthermia
c. Sore throat 79. Which of the following is a side effect of Lugol’s
d. Thrombocytosis solution?
a. Hypokalemia
74. You asked questions as soon as she regained b. Enlargement of the Thryoid gland
consciousness from thyroidectomy primarily to c. Nystagmus
assess the evidence of: d. Excessive salivation
a. Thyroid storm
b. Damage to the laryngeal nerve 80. In administering Lugol’s solution, the
c. Mediastinal shift precautionary measure should include:
d. Hypocalcaemia tetany a. Administer with glass only
b. Dilute with juice and administer with a
75. Should you check for haemorrhage, you will: straw
a. Slip your hand under the nape of c. Administer it with milk and drink it
her neck d. Follow it with milk of magnesia
b. Check for hypotension
c. Apply neck collar to prevent Situation: Pharmacological treatment was not effective
haemorrhage for Johnny’s hyperthyroidism and now, he is scheduled
d. Observe the dressing if it is soaked with for Thyroidectomy.
blood
81. Instruments in the surgical suite for surgery is
76. Basal Metabolic rate is assessed on Johnny to classified as either CRITICAL, SEMI CRITICAL and
determine his metabolic rate. In assessing the NON CRITICAL. If the instrument are introduced
BMR using the standard procedure, you need to directly into the blood stream or into any
tell Johnny that: normally sterile cavity or area of the body it is
a. Obstructing his vision classified as:
b. Restraining his upper and lower a. Critical
extremities b. Non Critical
c. Obstructing his hearing c. Semi Critical
d. Obstructing his nostrils with a clamp d. Ultra Critical

77. The BMR is based on the measurement that: 82. Instruments that do not touch the patient or
a. Rate of respiration under different have contact only to intact skin is classified as:
condition of activities and rest a. Critical
b. Amount of oxygen consumption under b. Non Critical
resting condition over a measured c. Semi Critical
period of time d. Ultra Critical
c. Amount of oxygen consumption under
stressed condition over a measured 83. If an instrument is classified as Semi Critical, an
period of time acceptable method of making the instrument
d. Ratio of respiration to pulse rate over a ready for surgery is through:
measured period of time a. Sterilization
b. Disinfection
78. Her physician ordered lugol’s solution in order c. Decontamination
d. Cleaning d. Tetany

84. While critical items and should be: 90. After surgery Johnny develops peripheral
a. Clean numbness, tingling and muscle twitching and
b. Sterilized spasm. What would you anticipate to
c. Decontaminated administer?
d. Disinfected a. Magnesium sulfate
b. Potassium iodide
85. As a nurse, you know that intact skin acts as an c. Calcium gluconate
effective barrier to most microorganisms. d. Potassium chloride
Therefore, items that come in contact with the
intact skin or mucus membranes should be: Situation: Budgeting is an important part of a nurse
a. Disinfected managerial activity. The correct allocation and
b. Clean distribution of resources is vital in the harmonious
c. Sterile operation of the financial balance of the agency.
d. Alcoholized
91. Which of the following best defines Budget?
86. You are caring for Johnny who is scheduled to a. Plan for the allocation of resources for
undergo total thyroidectomy because of a future use
diagnosis of thyroid cancer. Prior to total b. The process of allocating resources for
thyroidectomy, you should instruct Johnny to: future use
a. Perform range and motion exercise on c. Estimate cost of expenses
the head and neck d. Continuous process in seeing that the
b. Apply gentle pressure against the goals and objective of the agency is met
incision when swallowing
c. Cough and deep breathe every 2 hours 92. Which of the following best defines Capital
d. Support head with the hands when Budget?
changing position a. Budget to estimate the cost of direct
labour, number of staff to be hired and
87. As Johnny’s nurse, you plan to set up emergency necessary number of workers to meet
equipment at her bedside following the general patient needs
thyroidectomy. You should include: b. Includes the monthly and daily expenses
a. An airway and rebreathing tube and expected revenue and expenses
b. A tracheostomy set and oxygen c. These are related to long term planning
c. A crush cart with bed board and includes major replacement or
d. Two ampules of sodium bicarbonate expansion of the plant, major
equipment and inventories.
88. Which of the following nursing interventions is d. These are expenses that are not
appropriate after a total thyroidectomy? dependent on the level of production or
a. Place pillows under your patient’s sales. They tend to be time-related, such
shoulders. as salaries or rents being paid per month
b. Raise the knee-gatch to 30 degrees
c. Keep you patient in a high-fowler’s 93. Which of the following best described
position. Operational Budget?
d. Support the patient’s head and neck a. Budget to estimate the cost of direct
with pillows and sandbags. labour, number of staff to be hired and
necessary number of workers to meet
89. If there is an accidental injury to the parathyroid the general patient needs
gland during a thyroidectomy which of the b. Includes the monthly and daily
following might Leda develops postoperatively? expenses and expected revenue and
a. Cardiac arrest expenses
b. Respiratory failure c. These are related to long term planning
c. Dyspnea and includes major replacement or

4
4
expansion of the plant, major c. The Chinese Triad
equipments and inventories. d. Charcot’s Triad
d. These are expenses that are not
dependent on the level of production or 98. Which of the following is true with the Triad
sales. They tend to be time-related, such seen in head injuries?
as rent a. Narrowing of Pulse pressure, Cheyne
stokes respiration, Tachycardia
94. Which of the following accurately describes a b. Widening Pulse pressure, Irregular
Fixed Cost in budgeting? respiration, Bradycardia
a. These are usually the raw materials and c. Hypertension, Kussmaul’s respiration,
labour salaries that depend on the Tachycardia
production or sales d. Hypotension, Irregular respiration,
b. These are expenses that change in Bradycardia
proportion to the activity of a business
c. These are expenses that are not 99. In a client with a Cheyne stokes respiration,
dependent on the level of production or which of the following is the most
sales. They tend to be time-related, appropriate nursing diagnosis?
such as rent a. Ineffective airway clearance
d. This is the summation of the Variable b. Impaired gas exchange
Cost and the Fixed Cost c. Ineffective breathing pattern
d. Activity intolerance
95. Which of the following accurately describes
Variable Cost in budgeting? 100. You know the apnea is seen in client’s with
a. These are related to long term planning cheyne stokes respiration, APNEA is defined as:
and include major replacement or a. Inability to breathe in a supine position
expansion of the plant, major so the patient sits up in bed to breathe
equipments and inventories. b. The patient is dead, the breathing stops
b. These are expenses that change in c. There is an absence of breathing for a
proportion to the activity of a business period of time, usually 15 seconds or
c. These are expenses that are not more
dependent on the level of production or d. A period of hypercapnea and hypoxia
sales. They tend to be time-related, such due to the cessation of respiratory effort
as rent inspite of normal respiratory functioning
d. This is the summation of the Variable
Cost and the Fixed Cost

Situation – Andrea is admitted to the ER following an


assault where she was hit in the face and head. She was
brought to the ER by a police woman. Emergency
measures were started.

96. Andrea’s respiration is described as waxing and


waning. You know that this rhythm of respiration
is defined as:
a. Biot’s
b. Cheyne stokes
c. Kussmaul’s
d. Eupnea

97. What do you call the triad of sign and symptoms


seen in a client with increasing ICP?
a. Virchow’s Triad
b. Cushing’s Triad
5. A Client state, “I get down on myself when I
NURSING PRACTICE V make mistake.” Using Cognitive therapy
approach, the nurse should:
Situation: Understanding different models of care is a A. Teach the client relaxation exercise to
necessary part of the nurse patient relationship. diminish stress
B. Provide the client with Mastery
1. The focus of this therapy is to have a positive experience to boost self esteem
environmental manipulation, physical and social C. Explore the client’s past experiences that
to effect a positive change. causes the illness
A. Milieu D. Help client modify the belief that
B. Psychotherapy anything less than perfect is horrible
C. Behaviour
D. Group 6. The most advantageous therapy for a preschool
age child with a history of physical and sexual
2. The client asks the nurse about Milieu therapy. abuse would be:
The nurse responds knowing that the primary A. Play
focus of milieu therapy can be best described by B. Psychoanalysis
which of the following? C. Group
A. A form of behavior modification therapy D. Family
B. A cognitive approach of changing the
behaviour 7. An 18 year old client is admitted with the
C. A living, learning or working diagnosis of anorexia nervosa. A cognitive
environment behavioural approach is used as part of her
D. A behavioural approach to changing treatment plan. The nurse understands that the
behaviour purpose of this approach is to:
A. Help the client identify and examine
3. A nurse is caring for a client with phobia who is dysfunctional thoughts and beliefs
being treated for the condition. The client is B. Emphasize social interaction with clients
introduced to short periods of exposure to the who withdraw
phobic object while in relaxed state. The nurse C. Provide a supportive environment and a
understands that this form of behaviour therapeutic community
modification can be best described as: D. Examine intrapsychic conflicts and past
A. Systematic desensitization events in life
B. Self-control therapy
C. Aversion Therapy 8. The nurse is preparing to provide reminiscence
D. Operant conditioning therapy for a group of clients. Which of the
following clients will the nurse select for this
4. A client with major depression is considering group?
cognitive therapy. The client say to the nurse, A. A client who experiences profound
“How does this treatment works?” The nurse depression with moderate cognitive
responds by telling the client that: impairment
A. “This type of treatment helps you B. A catatonic, immobile client with
examine how your thoughts and moderate cognitive impairment
feelings contribute to your difficulties” C. An undifferentiated schizophrenic client
B. “This type of treatment helps you with moderate cognitive impairment
examine how your past life has D. A client with mild depression who
contributed to your problems.” exhibits who demonstrates normal
C. “This type of treatment helps you to cognition
confront your fears by exposing you to
the feared object abruptly. 9. Which intervention would be typical of a nurse
D. “This type of treatment will help you using cognitive-behavioral approach to a client
relax and develop new coping skills.” experiencing low self-esteem?

4
4
A. Use of unconditional positive regard C. refer the client to the psychiatrist
B. Analysis of free association D. refer the matter to the police
C. Classical conditioning
D. Examination of negative thought Situation: Rose seeks psychiatric consultation because of
patterns intense fear of flying in an airplane which has greatly
affected her chances of success in her job.
10. Which of the following therapies has been
strongly advocated for the treatment of post- 16. The most common defense mechanism used by
traumatic stress disorders? phobic clients is:
A. ECT A. Supression
B. Group Therapy B. Denial
C. Hypnotherapy C. Rationalization
D. Psychoanalysis D. Displacement

11. The nurse knows that in group therapy, the 17. The goal of the therapy in phobia is:
maximum number of members to include is: A. Change her lifestyle
A. 4 B. Ignore tension producing situation
B. 8 C. Change her reaction towards anxiety
C. 10 D. Eliminate fear producing situations
D. 16
18. The therapy most effective for client’s with
12. The nurse is providing information to a client phobia is:
with the use of disulfiram (antabuse) for the A. Hypnotherapy
treatment of alcohol abuse. The nurse B. Cognitive therapy
understands that this form of therapy works on C. Group therapy
what principle? D. Behavior therapy
A. Negative Reinforcement
B. Operant Conditioning 19. The fear and anxiety related to phobia is said to
C. Aversion Therapy be abruptly decreased when the patient is
D. Gestalt therapy exposed to what is feared through:
A. Guided Imagery
13. A biological or medical approach in treating B. Systematic desensitization
psychiatric patient is: C. Flooding
A. Million therapy D. Hypotherapy
B. Behavioral therapy
C. Somatic therapy 20. Based on the presence of symptom, the
D. Psychotherapy appropriate nursing diagnosis is:
A. Self-esteem disturbance
14. Which of these nursing actions belong to the B. Activity intolerance
secondary level of preventive intervention? C. Impaired adjustment
A. Providing mental health consultation to D. Ineffective individual coping
health care providers
B. Providing emergency psychiatric Situation: Mang Jose, 39 year old farmer, unmarried, had
services been confined in the National center for mental health
C. Being politically active in relation to for three years with a diagnosis of schizophrenia.
mental health issues
D. Providing mental health education to 21. The most common defense mechanism used by
members of the community a paranoid client is:
A. Displacement
15. When the nurse identifies a client who has B. Rationalization
attempted to commit suicide the nurse should: C. Suppression
A. call a priest D. Projection
B. counsel the client
22. When Mang Jose says to you: “The voices are is best described in one of the following
telling me bad things again!” The best response statements:
is: A. Unacceptable feelings or behavior are
A. “Whose voices are those?” kept out of awareness by developing the
B. “I doubt what the voices are telling you” opposite behavior or emotion
C. “I do not hear the voice you say you B. Consciously unacceptable instinctual
hear” drives are diverted into personally and
D. “Are you sure you hear these voices?” socially acceptable channels
C. Something unacceptable already done
23. A relevant nursing diagnosis for clients with is symbolically acted out in reverse
auditory hallucination is: D. Transfer of emotions associated with a
A. Sensory perceptual alteration particular person, object or situation to
B. Altered thought process another less threatening person, object
C. Impaired social interaction or situation
D. Impaired verbal communication
29. To be more effective, the nurse who cares for
24. During mealtime, Jose refused to eat telling that persons with obsessive compulsive disorder
the food was poisoned. The nurse should: must possess one of the following qualities:
A. Ignore his remark A. Compassion
B. Offer him food in his own container B. Patience
C. Show him how irrational his thinking is C. Consistency
D. Respect his refusal to eat D. Friendliness

25. When communicating with Jose, The nurse 30. Persons with OCD usually manifest:
considers the following except: A. Fear
A. Be warm and enthusiastic B. Apathy
B. Refrain from touching Jose C. Suspiciousness
C. Do not argue regarding his hallucination D. Anxiety
and delusion
D. Use simple, clear language Situation: The patient who is depressed will undergo
electroconvulsive therapy.
Situation: Gringo seeks psychiatric counselling for his
ritualistic behavior of counting his money as many as 10 31. Studies on biological depression support
times before leaving home. electroconvulsive therapy as a mode of
treatment. The rationale is:
26. An initial appropriate nursing diagnosis is: A. ECT produces massive brain damage
A. Impaired social interaction which destroys the specific area
B. Ineffective individual coping containing memories related to the
C. Impaired adjustment events surrounding the development of
D. Anxiety Moderate psychotic condition
B. The treatment serves as a symbolic
27. Obsessive compulsive disorder is BEST described punishment for the client who feels
by: guilty and worthless
A. Uncontrollable impulse to perform an C. ECT relieves depression psychologically
act or ritual repeatedly by increasing the norepinephrine level
B. Persistent thoughts D. ECT is seen as a life-threatening
C. Recurring unwanted and disturbing experience and depressed patients
thought alternating with a behavior mobilize all their bodily defences to deal
D. Pathological persistence of unwilled with this attack.
thought, feeling or impulse
32. The preparation of a patient for ECT ideally is
28. The defense mechanism used by persons with MOST similar to preparation for a patient for:
obsessive compulsive disorder is undoing and it A. electroencephalogram

4
4
B. general anesthesia A. Emotional crisis
C. X-ray B. Cholinergic crisis
D. electrocardiogram C. Menopausal crisis
D. Myasthenia crisis
33. Which of the following is a possible side effect
which you will discuss with the patient? 40. If you are not extra careful and by chance
A. hemorrhage within the brain you give over medication, this would lead to;
B. encephalitis A. Cholinergic crisis
C. robot-like body stiffness B. Menopausal crisis
D. confusion, disorientation and short C. Emotional crisis
term memory loss D. Myasthenia crisis

34. Informed consent is necessary for the treatment Situation: Rosanna 20 y/o unmarried patient believes
for involuntary clients. When this cannot be that the toilet for the female patient in contaminated
obtained, permission may be taken from the: with AIDS virus and refuses to use it unless she flushes it
A. social worker three times and wipes the seat same number of times
B. next of kin or guardian with antiseptic solution.
C. doctor
D. chief nurse 41. The fear of using “contaminated” toilet seat can
be attributed to Rosanna’s inability to;
35. After ECT, the nurse should do this action before A. Adjust to a strange environment
giving the client fluids, food or medication: B. Express her anxiety
A. assess the gag reflex C. Develop the sense of trust in other
B. next of kin or guardian person
C. assess the sensorium D. Control unacceptable impulses or
D. check O2 Sat with a pulse oximeter feelings

Situation: Mrs Ethel Agustin 50 y/o, teacher is afflicted 42. Assessment data upon admission help the nurse
with myasthenia gravis. to identify this appropriate nursing diagnosis
A. Ineffective denial
36. Looking at Mrs Agustin, your assessment would B. Impaired adjustment
include the following except; C. Ineffective individual coping
A. Nystagmus D. Impaired social interaction
B. Difficulty of hearing
C. Weakness of the levator palpebrae 43. An effective nursing intervention to help Rosana
D. Weakness of the ocular muscle is;
A. Convincing her to use the toilet after the
37. In an effort to combat complications which nurse has used it first
might occur relatives should he taught; B. Explaining to her that AIDS cannot be
A. Checking cardiac rate transmitted by using the toilet
B. Taking blood pressure reading C. Allowing her to flush and clear the
C. Techniques of oxygen inhalation toilet seat until she can manage
D. Administration of oxygen inhalation her anxiety
D. Explaining to her how AIDS is
38. The drug of choice for her condition is; transmitted
A. Prostigmine
B. Morphine 44. The goal for treatment for Rosana must be
C. Codeine directed toward helping her to;
D. Prednisone A. Walk freely about her past experience
B. Develop trusting relationship with others
39. As her nurse, you have to be cautious about C. Gain insight that her behaviour is
administration of medication, if she is under due to feeling of anxiety
medicated this can cause; D. Accept the environment unconditionally
B. Self-esteem disturbance
45. Psychotherapy which is prescribed for Rosana is C. Ineffective individual coping
described as; D. Defensive coping
A. Establishing an environment adapted to
an individual patient needs 50. Most appropriate nursing intervention for a
B. Sustained interaction between the client with suspicious behavior is one of the
therapist and client to help her develop following;
more functional behaviour A. Talk to the client constantly to reinforce
C. Using dramatic techniques to portray reality
interpersonal conflicts B. Involve him in competitive activities
D. Biologic treatment for mental disorder C. Use Non Judgmental and Consistent
approach
Situation: Dennis 40 y/o married man, an electrical D. Project cheerfulness in interacting with
engineer was admitted with the diagnosis of paranoid the patient
disorders. He has become suspicious and distrustful 2
months before admission. Upon admission, he kept Situation: Clients with Bipolar disorder receives a very
on saying, “my wife has been planning to kill me.” high nursing attention due to the increasing rate of
suicide related to the illness.
46. A paranoid individual who cannot accept the
guilt demonstrate one of the following defense 51. The nurse is assigned to care for a recently
mechanism; admitted client who has attempted suicide.
A. Denial What should the nurse do?
B. Projection A. Search the client's belongings and room
C. Rationalization carefully for items that could be used to
D. Displacement attempt suicide.
B. Express trust that the client won't cause
47. One morning, Dennis was seen tilting his head as self-harm while in the facility.
if he was listening to someone. An appropriate C. Respect the client's privacy by not
nursing intervention would be; searching any belongings.
A. Tell him to socialize with other patient to D. Remind all staff members to check on
divert his attention the client frequently.
B. Involve him in group activities
C. Address him by name to ask if he is 52. In planning activities for the depressed client,
hearing voices again especially during the early stages of
D. Request for an order of antipsychotic hospitalization, which of the following plan is
medicine best?
A. Provide an activity that is quiet and
48. When he says, “these voices are telling me my solitary to avoid increased fatigue such
wife is going to kill me.” A therapeutic as working on a puzzle and reading a
communication of the nurse is which one of the book.
following; B. Plan nothing until the client asks to
A. “i do not hear the voices you say you participate in the milieu
hear” C. Offer the client a menu of daily activities
B. “are you really sure you heard those and ask the client to participate in all of
voices?” them
C. “I do not think you heard those D. Provide a structured daily program of
voices?” activities and encourage the client to
D. “Whose voices are those?” participate

49. The nurse confirms that Dennis is manifesting 53. A client with a diagnosis of major depression,
auditory hallucination. The appropriate nursing recurrent with psychotic features is admitted to
diagnosis she identifiesis; the mental health unit. To create a safe
A. Sensory perceptual alteration environment for the client, the nurse most

5
5
importantly devises a plan of care that deals The nurse would initially:
specifically with the clients: A. Ask the client to leave the group session
A. Disturbed thought process B. Tell the client that she will not be
B. Imbalanced nutrition allowed to attend any more group
C. Self-Care Deficit sessions
D. Deficient Knowledge C. Tell the client that she needs to
allow other client in a group time to
54. The client is taking a Tricyclic anti-depressant, talk
which of the following is an example of TCA? D. Ask another nurse to escort the client
A. Paxil out of the group session
B. Nardil
C. Zoloft 59. A professional artist is admitted to the
D. Pamelor psychiatric unit for treatment of bipolar
disorder. During the last 2 weeks, the client has
55. A client visits the physician's office to seek created 154 paintings, slept only 2 to 3 hours
treatment for depression, feelings of every 2 days, and lost 18 lb (8.2 kg). Based on
hopelessness, poor appetite, insomnia, fatigue, Maslow's hierarchy of needs, what should the
low self-esteem, poor concentration, and nurse provide this client with first?
difficulty making decisions. The client states that A. The opportunity to explore family
these symptoms began at least 2 years ago. dynamics
Based on this report, the nurse suspects: B. Help with re-establishing a normal
A. cyclothymic disorder. sleep pattern
B. Bipolar disorder C. Experiences that build self-esteem
C. major depression. D. Art materials and equipment
D. dysthymic disorder.
60. The physician orders lithium carbonate
56. The nurse is planning activities for a client who (Lithonate) for a client who's in the manic phase
has bipolar disorder, which aggressive social of bipolar disorder. During lithium therapy, the
behaviour. Which of the following activities nurse should watch for which adverse reactions?
would be most appropriate for this client? A. Anxiety, restlessness, and sleep
A. Ping Pong disturbance
B. Linen delivery B. Nausea, diarrhea, tremor, and lethargy
C. Chess C. Constipation, lethargy, and ataxia
D. Basketball D. Weakness, tremor, and urine retention

57. The nurse assesses a client with admitted Situation – Annie has a morbid fear of heights. She asks
diagnosis of bipolar affective disorder, mania. the nurse what desensitization therapy is:
The symptom presented by the client that
requires the nurse’s immediate intervention is 61. The accurate information of the nurse of the
the client’s: goal of desensitization is:
A. Outlandish behaviour and inappropriate A. To help the clients relax and
dress progressively work up a list of anxiety
B. Grandiose delusion of being a royal provoking situations through imagery.
descendant of king arthut B. To provide corrective emotional
C. Nonstop physical activity and experiences through a one-to-one
poor nutritional intake intensive relationship.
D. Constant incessant talking that includes C. To help clients in a group therapy setting
sexual topics and teasing the staff to take on specific roles and reenact in
front of an audience, situations in which
58. A nurse is conducting a group therapy session interpersonal conflict is involved.
and during the session, A client with mania D. To help clients cope with their problems
consistently talks and dominates the group. The by learning behaviors that are more
behaviour is disrupting the group interaction. functional and be better equipped to
face reality and make decisions.
problem. After the identification of the research
62. It is essential in desensitization for the patient problem, which of the following should be done?
to: A. Methodology
A. Have rapport with the therapist B. Acknowledgement
B. Use deep breathing or another C. Review of related literature
relaxation technique D. Formulate hypothesis
C. Assess one’s self for the need of an
anxiolytic drug 68. Which of the following communicate the results
D. Work through unresolved unconscious of the research to the readers. They facilitate the
conflicts description of the data.
A. Hypothesis
63. In this level of anxiety, cognitive capacity B. Research problem
diminishes. Focus becomes limited and client C. Statistics
experiences tunnel vision. Physical signs of D. Tables and Graphs
anxiety become more pronounced.
A. Severe anxiety 69. In Quantitative date, which of the following is
B. Mild anxiety described as the distance in the scoring unites of
C. Panic the variable from the highest to the lower?
D. Moderate anxiety A. Frequency
B. Median
64. Antianxiety medications should be used with C. Mean
extreme caution because long term use can lead D. Range
to:
A. Parkinsonian like syndrome 70. This expresses the variability of the data in
B. Hepatic failure reference to the mean. It provides as with a
C. Hypertensive crisis numerical estimate of how far, on the average
D. Risk of addiction the separate observation are from the mean:
A. Mode
65. The nursing management of anxiety related with B. Median
post-traumatic stress disorder includes all of the C. Standard deviation
following EXCEPT: D. Frequency
A. Encourage participation in recreation or
sports activities Situation: Survey and Statistics are important part of
B. Reassure client’s safety while research that is necessary to explain the characteristics
touching client of the population.
C. Speak in a calm soothing voice
D. Remain with the client while fear level is 71. According to the WHO statistics on the Homeless
high population around the world, which of the
following groups of people in the world
SITUATION: You are fortunate to be chosen as part of disproportionately represents the homeless
the research team in the hospital. A review of the population?
following IMPORTANT nursing concepts was made. A. Hispanics
B. Asians
66. As a professional, a nurse can do research for C. African Americans
varied reason except: D. Caucasians
A. Professional advancement through
research participation 72. All but one of the following is not a measure
B. To validate results of new of Central Tendency:
nursing modalities A. Mode
C. For financial gains B. Standard Deviation
D. To improve nursing care C. Variance
D. Range
67. Each nurse participants was asked to identify a

5
5

73. In the value: 87, 85, 88, 92, 90; what is the A. There is a control group
mean? B. There is an experimental group
A. 88.2 C. Selection of subjects in the control group
B. 88.4 is randomized
C. 87 D. There is a careful selection of subjects
D. 90 in the experimental group

74. In the value: 80, 80, 80, 82, 82, 90, 90, 100; what 80. The researcher implemented a medication
is the mode? regimen using a new type of combination drugs
A. 80 to manic patients while another group of manic
B. 82 patient receives the routine drugs. The
C. 90 researcher however handpicked the
D. 85.5 experimental group for they are the clients with
75. In the value: 80, 80, 10, 10, 25, 65, 100, 200; multiple episodes of bipolar disorder. The
what is the median? researcher utilized which research design?
A. 71.25 A. Quasi-experimental
B. 22.5 B. Phenomenological
C. 10 and 25 C. Pure experimental
D. 72.5 D. Longitudinal

76. Draw Lots, Lottery, Table of random numbers or Situation 19: As a nurse, you are expected to participate
a sampling that ensures that each element of the in initiating or participating in the conduct of research
population has an equal and independent studies to improve nursing practice. You to be updated
chance of being chosen is called: on the latest trends and issues affected the profession
A. Cluster and the best practices arrived at by the profession.
B. Stratified
C. Simple 81. You are interested to study the effects of
D. Systematic mediation and relaxation on the pain
experienced by cancer patients. What type of
77. An investigator wants to determine some of the variable is pain?
problems that are experienced by diabetic A. Dependent
clients when using an insulin pump. The B. Independent
investigator went into a clinic where he C. Correlational
personally knows several diabetic clients having D. Demographic
problem with insulin pump. The type of sampling
done by the investigator is called: 82. You would like to compare the support
A. Probability system of patient with chronic illness to those
B. Snowball with acute illness. How will you best state
C. Purposive your problem?
D. Incidental A. A descriptive study to compare the
support system of patients with chronic
78. If the researcher implemented a new structured illness and those with acute illness in
counselling program with a randomized group of terms of demographic data and
subject and a routine counselling program with knowledge about intervention.
another randomized group of subject, the B. The effects of the types of support
research is utilizing which design? system of patients with chronic illness
A. Quasi experimental and those with acute illness.
B. Comparative C. A comparative analysis of the support
C. Experimental system of patients with chronic illness
D. Methodological and those with acute illness.
D. A study to compare the support system
79. Which of the following is not true about a Pure of patients with chronic illness and those
Experimental research? with acute illness.
E. What are the differences of the support 87. Which of the following studies is based on
system being received by patient with quantitative research?
chronic illness and patients with acute A. A study examining the bereavement
illness? process in spouse of clients with
terminal cancer
83. You would like to compare the support system B. A study exploring the factors influencing
of patients with chronic illness to those with weight control behaviour
acute illness. Considering that the hypothesis C. A Study measuring the effects of sleep
was: “Client’s with chronic illness have lesser deprivation on wound healing
support system than client’s with acute illness.” D. A study examining client’s feelings
What type of research is this? before, during and after bone marrow
A. Descriptive aspiration.
B. Correlational, Non experimental
C. Experimental 88. Which of the following studies is based on the
D. Quasi Experimental qualitative research?
A. A study examining clients’ reaction to
84. In any research study where individual persons stress after open heart surgery
are involved, it is important that an informed B. A study measuring nutrition and weight
consent of the study is obtained. The following loss/gain in clients with cancer
are essential information about the consent that C. A study examining oxygen levels after
you should disclose to the prospective subjects endotracheal suctioning
except: D. A study measuring differences in blood
A. Consent to incomplete disclosure pressure before, during and after
B. Description of benefits, risks and procedure
discomforts
C. Explanation of procedure 89. An 85 year old client in a nursing home tells a
D. Assurance of anonymity and nurse, “I signed the papers of that research
confidentiality study because the doctor was so insistent and I
want him to continue taking care for me” Which
85. In the Hypothesis: “The utilization of technology client right is being violated?
in teaching improves the retention and attention A. Right of self determination
of the nursing students.” Which is the B. Right to full disclosure
dependent variable? C. Right to privacy and confidentiality
A. Utilization of technology D. Right not to be harmed
B. Improvement in the retention and
attention 90. A supposition or system of ideas that is
C. Nursing students proposed to explain a given phenomenon best
D. Teaching defines:
A. A paradigm
Situation: You are actively practicing nurse who has just B. A theory
finished you graduate studies. You learned the value of C. A Concept
research and would like to utilize the knowledge and D. A conceptual framework
skills gained in the application of research to the nursing Situation: Mastery of research design determination is
service. The following questions apply to research. essential in passing the NLE.

86. Which type of research inquiry investigates the 91. Ana wants to know if the length of time she will
issues of human complexity (e.g understanding study for the board examination is proportional
the human expertise)? to her board rating. During the June 2008 board
A. Logical position examination, she studied for 6 months and
B. Positivism gained 68%, On the next board exam, she
C. Naturalistic inquiry studied for 6 months again for a total of 1 year
D. Quantitative research and gained 74%, On the third board exam, She
studied for 6 months for a total of 1 and a half

5
5
year and gained 82%. The research design she collected 100 random individuals and determine
used is: who is their favourite comedian actor. 50% said
A. Comparative Dolphy, 20% said Vic Sotto, while some
B. Experimental answered Joey de Leon, Allan K, Michael V.
C. Correlational Tonyo conducted what type of research study?
D. Qualitative A. Phenomenological
B. Non experimental
92. Anton was always eating high fat diet. You want C. Case Study
to determine if what will be the effect of high D. Survey
cholesterol food to Anton in the next 10 years.
You will use: 98. Jane visited a tribe located somewhere in China,
A. Comparative it is called the Shin Jea tribe. She studied the way
B. Historical of life, tradition and the societal structure of
C. Correlational these people. Jane will best use which research
D. Longitudinal design?
A. Historical
93. Community A was selected randomly as well as B. Phenomenological
community B, nurse Edna conducted teaching to C. Case Study
community A and assess if community A will D. Ethnographic
have a better status than community B. This is
an example of: 99. Anjoe researched on TB. Its transmission,
A. Comparative Causative agent and factors, treatment sign and
B. Experimental symptoms as well as medication and all other in
C. Correlational depth information about tuberculosis. This study
D. Qualitative is best suited for which research design?
A. Historical
94. Ana researched on the development of a new B. Phenomenological
way to measure intelligence by creating a 100 C. Case Study
item questionnaire that will assess the cognitive D. Ethnographic
skills of an individual. The design best suited for
this study is: 100. Diana is to conduct a study about the
A. Historical relationship of the number of family members in
B. Survey the household and the electricity bill. Which of
C. Methodological the following is the best research design suited
D. Case study for this study?
1. Descriptive
95. Gen is conducting a research study on how mark, 2. Exploratory
an AIDS client lives his life. A design suited for 3. Explanatory
this is: 4. Correlational
A. Historical 5. Comparative
B. Phenomenological 6. Experimental
C. Case Study A. 1,4
D. Ethnographic B. 2,5
C. 3,6
96. Marco is to perform a study about how nurses D. 1,5
perform surgical asepsis during World War II. A E. 2,4
design best for this study is:
A. Historical
B. Phenomenological
C. Case Study
D. Ethnographic

97. Tonyo conducts sampling at barangay 412. He


TEST I - Foundation of Professional Nursing 5. Nurse Betty is assigned to the following clients.
Practice The client that the nurse would see first after
endorsement?
1. The nurse In-charge in labor and delivery unit a. A 34 year-old post-operative
administered a dose of terbutaline to a client appendectomy client of five hours who
without checking the client’s pulse. The standard is complaining of pain.
that would be used to determine if the nurse b. A 44 year-old myocardial infarction (MI)
was negligent is: client who is complaining of nausea.
a. The physician’s orders. c. A 26 year-old client admitted for
b. The action of a clinical nurse specialist dehydration whose intravenous (IV) has
who is recognized expert in the field. infiltrated.
c. The statement in the drug literature d. A 63 year-old post operative’s
about administration of terbutaline. abdominal hysterectomy client of three
d. The actions of a reasonably prudent days whose incisional dressing is
nurse with similar education and saturated with serosanguinous fluid.
experience.
6. Nurse Gail places a client in a four-point restraint
2. Nurse Trish is caring for a female client with a following orders from the physician. The client
history of GI bleeding, sickle cell disease, and a care plan should include:
platelet count of 22,000/μl. The female client is a. Assess temperature frequently.
dehydrated and receiving dextrose 5% in half- b. Provide diversional activities.
normal saline solution at 150 ml/hr. The client c. Check circulation every 15-30 minutes.
complains of severe bone pain and is scheduled d. Socialize with other patients once a shift.
to receive a dose of morphine sulfate. In
administering the medication, Nurse Trish 7. A male client who has severe burns is receiving
should avoid which route? H2 receptor antagonist therapy. The nurse In-
a. I.V charge knows the purpose of this therapy is to:
b. I.M a. Prevent stress ulcer
c. Oral b. Block prostaglandin synthesis
d. S.C c. Facilitate protein synthesis.
d. Enhance gas exchange
3. Dr. Garcia writes the following order for the
client who has been recently admitted “Digoxin 8. The doctor orders hourly urine output
.125 mg P.O. once daily.” To prevent a dosage measurement for a postoperative male client.
error, how should the nurse document this order The nurse Trish records the following amounts of
onto the medication administration record? output for 2 consecutive hours: 8 a.m.: 50 ml; 9
a. “Digoxin .1250 mg P.O. once daily” a.m.: 60 ml. Based on these amounts, which
b. “Digoxin 0.1250 mg P.O. once daily” action should the nurse take?
c. “Digoxin 0.125 mg P.O. once daily” a. Increase the I.V. fluid infusion rate
d. “Digoxin .125 mg P.O. once daily” b. Irrigate the indwelling urinary catheter
c. Notify the physician
4. A newly admitted female client was diagnosed d. Continue to monitor and record hourly
with deep vein thrombosis. Which nursing urine output
diagnosis should receive the highest priority?
a. Ineffective peripheral tissue perfusion 9. Tony, a basketball player twist his right ankle
related to venous congestion. while playing on the court and seeks care for
b. Risk for injury related to edema. ankle pain and swelling. After the nurse applies
c. Excess fluid volume related to peripheral ice to the ankle for 30 minutes, which statement
vascular disease. by Tony suggests that ice application has been
d. Impaired gas exchange related to effective?
increased blood flow. a. “My ankle looks less swollen now”.
b. “My ankle feels warm”.
c. “My ankle appears redder now”.

5
5
d. “I need something stronger for pain d. Pulling the lobule down and forward
relief”
16. Which instruction should nurse Tom give to a
10. The physician prescribes a loop diuretic for a male client who is having external radiation
client. When administering this drug, the nurse therapy:
anticipates that the client may develop which a. Protect the irritated skin from sunlight.
electrolyte imbalance? b. Eat 3 to 4 hours before treatment.
a. Hypernatremia c. Wash the skin over regularly.
b. Hyperkalemia d. Apply lotion or oil to the radiated area
c. Hypokalemia when it is red or sore.
d. Hypervolemia
17. In assisting a female client for immediate
11. She finds out that some managers have surgery, the nurse In-charge is aware that she
benevolent-authoritative style of management. should:
Which of the following behaviors will she exhibit a. Encourage the client to void following
most likely? preoperative medication.
a. Have condescending trust and b. Explore the client’s fears and anxieties
confidence in their subordinates. about the surgery.
b. Gives economic and ego awards. c. Assist the client in removing dentures
c. Communicates downward to staffs. and nail polish.
d. Allows decision making among d. Encourage the client to drink water prior
subordinates. to surgery.

12. Nurse Amy is aware that the following is true 18. A male client is admitted and diagnosed with
about functional nursing acute pancreatitis after a holiday celebration of
a. Provides continuous, coordinated and excessive food and alcohol. Which assessment
comprehensive nursing services. finding reflects this diagnosis?
b. One-to-one nurse patient ratio. a. Blood pressure above normal range.
c. Emphasize the use of group b. Presence of crackles in both lung fields.
collaboration. c. Hyperactive bowel sounds
d. Concentrates on tasks and activities. d. Sudden onset of continuous epigastric
and back pain.
13. Which type of medication order might read
"Vitamin K 10 mg I.M. daily × 3 days?" 19. Which dietary guidelines are important for nurse
a. Single order Oliver to implement in caring for the client with
b. Standard written order burns?
c. Standing order a. Provide high-fiber, high-fat diet
d. Stat order b. Provide high-protein, high-carbohydrate
diet.
14. A female client with a fecal impaction frequently c. Monitor intake to prevent weight gain.
exhibits which clinical manifestation? d. Provide ice chips or water intake.
a. Increased appetite
b. Loss of urge to defecate 20. Nurse Hazel will administer a unit of whole
c. Hard, brown, formed stools blood, which priority information should the
d. Liquid or semi-liquid stools nurse have about the client?
a. Blood pressure and pulse rate.
15. Nurse Linda prepares to perform an otoscopic b. Height and weight.
examination on a female client. For proper c. Calcium and potassium levels
visualization, the nurse should position the d. Hgb and Hct levels.
client's ear by: 21. Nurse Michelle witnesses a female client sustain
a. Pulling the lobule down and back a fall and suspects that the leg may be broken.
b. Pulling the helix up and forward The nurse takes which priority action?
c. Pulling the helix up and back a. Takes a set of vital signs.
b. Call the radiology department for X-ray. c. 1.5 cc
c. Reassure the client that everything will d. 2.5 cc
be alright.
d. Immobilize the leg before moving the 27. A child of 10 years old is to receive 400 cc of IV
client. fluid in an 8 hour shift. The IV drip factor is 60.
The IV rate that will deliver this amount is:
22. A male client is being transferred to the nursing a. 50 cc/ hour
unit for admission after receiving a radium b. 55 cc/ hour
implant for bladder cancer. The nurse in-charge c. 24 cc/ hour
would take which priority action in the care of d. 66 cc/ hour
this client?
a. Place client on reverse isolation. 28. The nurse is aware that the most important
b. Admit the client into a private room. nursing action when a client returns from
c. Encourage the client to take frequent surgery is:
rest periods. a. Assess the IV for type of fluid and rate of
d. Encourage family and friends to visit. flow.
b. Assess the client for presence of pain.
23. A newly admitted female client was diagnosed c. Assess the Foley catheter for patency
with agranulocytosis. The nurse formulates and urine output
which priority nursing diagnosis? d. Assess the dressing for drainage.
a. Constipation
b. Diarrhea 29. Which of the following vital sign assessments
c. Risk for infection that may indicate cardiogenic shock after
d. Deficient knowledge myocardial infarction?
a. BP – 80/60, Pulse – 110 irregular
24. A male client is receiving total parenteral b. BP – 90/50, Pulse – 50 regular
nutrition suddenly demonstrates signs and c. BP – 130/80, Pulse – 100 regular
symptoms of an air embolism. What is the d. BP – 180/100, Pulse – 90 irregular
priority action by the nurse?
a. Notify the physician. 30. Which is the most appropriate nursing action in
b. Place the client on the left side in the obtaining a blood pressure measurement?
Trendelenburg position. a. Take the proper equipment, place the
c. Place the client in high-Fowlers position. client in a comfortable position, and
d. Stop the total parenteral nutrition. record the appropriate information in
the client’s chart.
25. Nurse May attends an educational conference b. Measure the client’s arm, if you are not
on leadership styles. The nurse is sitting with a sure of the size of cuff to use.
nurse employed at a large trauma center who c. Have the client recline or sit comfortably
states that the leadership style at the trauma in a chair with the forearm at the level of
center is task-oriented and directive. The nurse the heart.
determines that the leadership style used at the d. Document the measurement, which
trauma center is: extremity was used, and the position
a. Autocratic. that the client was in during the
b. Laissez-faire. measurement.
c. Democratic.
d. Situational 31. Asking the questions to determine if the person
26. The physician orders DS 500 cc with KCl 10 understands the health teaching provided by the
mEq/liter at 30 cc/hr. The nurse in-charge is nurse would be included during which step of
going to hang a 500 cc bag. KCl is supplied 20 the nursing process?
mEq/10 cc. How many cc’s of KCl will be added
to the IV solution? a. Assessment
a. .5 cc b. Evaluation
b. 5 cc c. Implementation

5
5
d. Planning and goals “Meperidine, 100 mg/ml.” How many milliliters
of meperidine should the client receive?
32. Which of the following item is considered the a. 0.75
single most important factor in assisting the b. 0.6
health professional in arriving at a diagnosis or c. 0.5
determining the person’s needs? d. 0.25
a. Diagnostic test results
b. Biographical date 38. A male client with diabetes mellitus is receiving
c. History of present illness insulin. Which statement correctly describes an
d. Physical examination insulin unit?
a. It’s a common measurement in the
33. In preventing the development of an external metric system.
rotation deformity of the hip in a client who b. It’s the basis for solids in the avoirdupois
must remain in bed for any period of time, the system.
most appropriate nursing action would be to c. It’s the smallest measurement in the
use: apothecary system.
a. Trochanter roll extending from the crest d. It’s a measure of effect, not a standard
of the ileum to the mid-thigh. measure of weight or quantity.
b. Pillows under the lower legs.
c. Footboard 39. Nurse Oliver measures a client’s temperature at
d. Hip-abductor pillow 102° F. What is the equivalent Centigrade
temperature?
34. Which stage of pressure ulcer development does a. 40.1 °C
the ulcer extend into the subcutaneous tissue? b. 38.9 °C
a. Stage I c. 48 °C
b. Stage II d. 38 °C
c. Stage III 40. The nurse is assessing a 48-year-old client who
d. Stage IV has come to the physician’s office for his annual
physical exam. One of the first physical signs of
35. When the method of wound healing is one in aging is:
which wound edges are not surgically a. Accepting limitations while developing
approximated and integumentary continuity is assets.
restored by granulations, the wound healing is b. Increasing loss of muscle tone.
termed c. Failing eyesight, especially close vision.
a. Second intention healing d. Having more frequent aches and pains.
b. Primary intention healing
c. Third intention healing 41. The physician inserts a chest tube into a female
d. First intention healing client to treat a pneumothorax. The tube is
connected to water-seal drainage. The nurse in-
36. An 80-year-old male client is admitted to the charge can prevent chest tube air leaks by:
hospital with a diagnosis of pneumonia. Nurse a. Checking and taping all connections.
Oliver learns that the client lives alone and b. Checking patency of the chest tube.
hasn’t been eating or drinking. When assessing c. Keeping the head of the bed slightly
him for dehydration, nurse Oliver would expect elevated.
to find: d. Keeping the chest drainage system
a. Hypothermia below the level of the chest.
b. Hypertension
c. Distended neck veins 42. Nurse Trish must verify the client’s identity
d. Tachycardia before administering medication. She is aware
that the safest way to verify identity is to:
37. The physician prescribes meperidine (Demerol), a. Check the client’s identification band.
75 mg I.M. every 4 hours as needed, to control a b. Ask the client to state his name.
client’s postoperative pain. The package insert is
c. State the client’s name out loud and c. Every 2 years
wait a client to repeat it. d. Once, to establish baseline
d. Check the room number and the client’s
name on the bed. 49. A male client has the following arterial blood gas
values: pH 7.30; Pao2 89 mmHg; Paco2 50
43. The physician orders dextrose 5 % in water, mmHg; and HCO3 26mEq/L. Based on these
1,000 ml to be infused over 8 hours. The I.V. values, Nurse Patricia should expect which
tubing delivers 15 drops/ml. Nurse John should condition?
run the I.V. infusion at a rate of: a. Respiratory acidosis
a. 30 drops/minute b. Respiratory alkalosis
b. 32 drops/minute c. Metabolic acidosis
c. 20 drops/minute d. Metabolic alkalosis
d. 18 drops/minute
50. Nurse Len refers a female client with terminal
44. If a central venous catheter becomes cancer to a local hospice. What is the goal of this
disconnected accidentally, what should the referral?
nurse in-charge do immediately? a. To help the client find appropriate
a. Clamp the catheter treatment options.
b. Call another nurse b. To provide support for the client and
c. Call the physician family in coping with terminal illness.
d. Apply a dry sterile dressing to the site. c. To ensure that the client gets counseling
regarding health care costs.
45. A female client was recently admitted. She has d. To teach the client and family about
fever, weight loss, and watery diarrhea is being cancer and its treatment.
admitted to the facility. While assessing the
client, Nurse Hazel inspects the client’s abdomen 51. When caring for a male client with a 3-cm stage I
and notice that it is slightly concave. Additional pressure ulcer on the coccyx, which of the
assessment should proceed in which order: following actions can the nurse institute
a. Palpation, auscultation, and percussion. independently?
b. Percussion, palpation, and auscultation. a. Massaging the area with an astringent
c. Palpation, percussion, and auscultation. every 2 hours.
d. Auscultation, percussion, and palpation. b. Applying an antibiotic cream to the area
three times per day.
46. Nurse Betty is assessing tactile fremitus in a c. Using normal saline solution to clean the
client with pneumonia. For this examination, ulcer and applying a protective dressing
nurse Betty should use the: as necessary.
a. Fingertips d. Using a povidone-iodine wash on the
b. Finger pads ulceration three times per day.
c. Dorsal surface of the hand 52. Nurse Oliver must apply an elastic bandage to a
d. Ulnar surface of the hand client’s ankle and calf. He should apply the
bandage beginning at the client’s:
47. Which type of evaluation occurs continuously a. Knee
throughout the teaching and learning process? b. Ankle
a. Summative c. Lower thigh
b. Informative d. Foot
c. Formative
d. Retrospective 53. A 10 year old child with type 1 diabetes develops
48. A 45 year old client, has no family history of diabetic ketoacidosis and receives a continuous
breast cancer or other risk factors for this insulin infusion. Which condition represents the
disease. Nurse John should instruct her to have greatest risk to this child?
mammogram how often? a. Hypernatremia
a. Twice per year b. Hypokalemia
b. Once per year c. Hyperphosphatemia

6
6
d. Hypercalcemia d. Obtaining the specimen from the urinary
drainage bag.
54. Nurse Len is administering sublingual nitrglycerin
(Nitrostat) to the newly admitted client. 59. Nurse Meredith is in the process of giving a
Immediately afterward, the client may client a bed bath. In the middle of the
experience: procedure, the unit secretary calls the nurse on
a. Throbbing headache or dizziness the intercom to tell the nurse that there is an
b. Nervousness or paresthesia. emergency phone call. The appropriate nursing
c. Drowsiness or blurred vision. action is to:
d. Tinnitus or diplopia. a. Immediately walk out of the client’s
room and answer the phone call.
b. Cover the client, place the call light
55. Nurse Michelle hears the alarm sound on the within reach, and answer the phone call.
telemetry monitor. The nurse quickly looks at c. Finish the bed bath before answering
the monitor and notes that a client is in a the phone call.
ventricular tachycardia. The nurse rushes to the d. Leave the client’s door open so the client
client’s room. Upon reaching the client’s can be monitored and the nurse can
bedside, the nurse would take which action answer the phone call.
first?
a. Prepare for cardioversion 60. Nurse Janah is collecting a sputum specimen for
b. Prepare to defibrillate the client culture and sensitivity testing from a client who
c. Call a code has a productive cough. Nurse Janah plans to
d. Check the client’s level of consciousness implement which intervention to obtain the
specimen?
56. Nurse Hazel is preparing to ambulate a female a. Ask the client to expectorate a small
client. The best and the safest position for the amount of sputum into the emesis basin.
nurse in assisting the client is to stand: b. Ask the client to obtain the specimen
a. On the unaffected side of the client. after breakfast.
b. On the affected side of the client. c. Use a sterile plastic container for
c. In front of the client. obtaining the specimen.
d. Behind the client. d. Provide tissues for expectoration and
obtaining the specimen.
57. Nurse Janah is monitoring the ongoing care
given to the potential organ donor who has been 61. Nurse Ron is observing a male client using a
diagnosed with brain death. The nurse walker. The nurse determines that the client is
determines that the standard of care had been using the walker correctly if the client:
maintained if which of the following data is a. Puts all the four points of the walker flat
observed? on the floor, puts weight on the hand
a. Urine output: 45 ml/hr pieces, and then walks into it.
b. Capillary refill: 5 seconds b. Puts weight on the hand pieces, moves
c. Serum pH: 7.32 the walker forward, and then walks into
d. Blood pressure: 90/48 mmHg it.
c. Puts weight on the hand pieces, slides
58. Nurse Amy has an order to obtain a urinalysis the walker forward, and then walks into
from a male client with an indwelling urinary it.
catheter. The nurse avoids which of the d. Walks into the walker, puts weight on
following, which contaminate the specimen? the hand pieces, and then puts all four
a. Wiping the port with an alcohol swab points of the walker flat on the floor.
before inserting the syringe.
b. Aspirating a sample from the port on the 62. Nurse Amy has documented an entry regarding
drainage bag. client care in the client’s medical record. When
c. Clamping the tubing of the drainage bag. checking the entry, the nurse realizes that
incorrect information was documented. How a. Prone with head turned toward the side
does the nurse correct this error? supported by a pillow.
a. Erases the error and writes in the correct b. Sims’ position with the head of the bed
information. flat.
b. Uses correction fluid to cover up the c. Right side-lying with the head of the bed
incorrect information and writes in the elevated 45 degrees.
correct information. d. Left side-lying with the head of the bed
c. Draws one line to cross out the incorrect elevated 45 degrees.
information and then initials the change.
d. Covers up the incorrect information 67. Nurse John develops methods for data
completely using a black pen and writes gathering. Which of the following criteria of a
in the correct information good instrument refers to the ability of the
instrument to yield the same results upon its
63. Nurse Ron is assisting with transferring a client repeated administration?
from the operating room table to a stretcher. To a. Validity
provide safety to the client, the nurse should: b. Specificity
a. Moves the client rapidly from the table c. Sensitivity
to the stretcher. d. Reliability
b. Uncovers the client completely before
transferring to the stretcher. 68. Harry knows that he has to protect the rights of
c. Secures the client safety belts after human research subjects. Which of the following
transferring to the stretcher. actions of Harry ensures anonymity?
d. Instructs the client to move self from the a. Keep the identities of the subject secret
table to the stretcher. b. Obtain informed consent
c. Provide equal treatment to all the
64. Nurse Myrna is providing instructions to a subjects of the study.
nursing assistant assigned to give a bed bath to a d. Release findings only to the participants
client who is on contact precautions. Nurse of the study
Myrna instructs the nursing assistant to use
which of the following protective items when 69. Patient’s refusal to divulge information is a
giving bed bath? limitation because it is beyond the control of
a. Gown and goggles Tifanny”. What type of research is appropriate
b. Gown and gloves for this study?
c. Gloves and shoe protectors a. Descriptive- correlational
d. Gloves and goggles b. Experiment
c. Quasi-experiment
65. Nurse Oliver is caring for a client with impaired d. Historical
mobility that occurred as a result of a stroke. The
client has right sided arm and leg weakness. The 70. Nurse Ronald is aware that the best tool for data
nurse would suggest that the client use which of gathering is?
the following assistive devices that would a. Interview schedule
provide the best stability for ambulating? b. Questionnaire
a. Crutches c. Use of laboratory data
b. Single straight-legged cane d. Observation
c. Quad cane
d. Walker 71. Monica is aware that there are times when only
manipulation of study variables is possible and
66. A male client with a right pleural effusion noted the elements of control or randomization are
on a chest X-ray is being prepared for not attendant. Which type of research is
thoracentesis. The client experiences severe referred to this?
dizziness when sitting upright. To provide a safe a. Field study
environment, the nurse assists the client to b. Quasi-experiment
which position for the procedure? c. Solomon-Four group design

6
6
d. Post-test only design d. Will remain unable to practice
professional nursing
72. Cherry notes down ideas that were derived from
the description of an investigation written by the 77. Ronald plans to conduct a research on the use of
person who conducted it. Which type of a new method of pain assessment scale. Which
reference source refers to this? of the following is the second step in the
a. Footnote conceptualizing phase of the research process?
b. Bibliography a. Formulating the research hypothesis
c. Primary source b. Review related literature
d. Endnotes c. Formulating and delimiting the research
problem
73. When Nurse Trish is providing care to his d. Design the theoretical and conceptual
patient, she must remember that her duty is framework
bound not to do doing any action that will cause
the patient harm. This is the meaning of the 78. The leader of the study knows that certain
bioethical principle: patients who are in a specialized research setting
a. Non-maleficence tend to respond psychologically to the
b. Beneficence conditions of the study. This referred to as :
c. Justice a. Cause and effect
d. Solidarity b. Hawthorne effect
c. Halo effect
74. When a nurse in-charge causes an injury to a d. Horns effect
female patient and the injury caused becomes
the proof of the negligent act, the presence of 79. Mary finally decides to use judgment sampling
the injury is said to exemplify the principle of: on her research. Which of the following actions
a. Force majeure of is correct?
b. Respondeat superior a. Plans to include whoever is there during
c. Res ipsa loquitor his study.
d. Holdover doctrine b. Determines the different nationality of
patients frequently admitted and
75. Nurse Myrna is aware that the Board of Nursing decides to get representations samples
has quasi-judicial power. An example of this from each.
power is: c. Assigns numbers for each of the
a. The Board can issue rules and patients, place these in a fishbowl and
regulations that will govern the practice draw 10 from it.
of nursing d. Decides to get 20 samples from the
b. The Board can investigate violations of admitted patients
the nursing law and code of ethics
c. The Board can visit a school applying for 80. The nursing theorist who developed
a permit in collaboration with CHED transcultural nursing theory is:
d. The Board prepares the board a. Florence Nightingale
examinations b. Madeleine Leininger
c. Albert Moore
76. When the license of nurse Krina is revoked, it d. Sr. Callista Roy
means that she:
a. Is no longer allowed to practice the 81. Marion is aware that the sampling method that
profession for the rest of her life gives equal chance to all units in the population
b. Will never have her/his license re-issued to get picked is:
since it has been revoked a. Random
c. May apply for re-issuance of his/her b. Accidental
license based on certain conditions c. Quota
stipulated in RA 9173 d. Judgment
82. John plans to use a Likert Scale to his study to
determine the: 89. Nurse Marian is preparing to administer a blood
a. Degree of agreement and disagreement transfusion. Which action should the nurse take
b. Compliance to expected standards first?
c. Level of satisfaction a. Arrange for typing and cross matching of
d. Degree of acceptance the client’s blood.
b. Compare the client’s identification
83. Which of the following theory addresses the four wristband with the tag on the unit of
modes of adaptation? blood.
a. Madeleine Leininger c. Start an I.V. infusion of normal saline
b. Sr. Callista Roy solution.
c. Florence Nightingale d. Measure the client’s vital signs.
d. Jean Watson
90. A 65 years old male client requests his
84. Ms. Garcia is responsible to the number of medication at 9 p.m. instead of 10 p.m. so that
personnel reporting to her. This principle refers he can go to sleep earlier. Which type of nursing
to: intervention is required?
a. Span of control a. Independent
b. Unity of command b. Dependent
c. Downward communication c. Interdependent
d. Leader d. Intradependent

85. Ensuring that there is an informed consent on 91. A female client is to be discharged from an acute
the part of the patient before a surgery is done, care facility after treatment for right leg
illustrates the bioethical principle of: thrombophlebitis. The Nurse Betty notes that
a. Beneficence the client's leg is pain-free, without redness or
b. Autonomy edema. The nurse's actions reflect which step of
c. Veracity the nursing process?
d. Non-maleficence a. Assessment
b. Diagnosis
86. Nurse Reese is teaching a female client with c. Implementation
peripheral vascular disease about foot care; d. Evaluation
Nurse Reese should include which instruction?
a. Avoid wearing cotton socks. 92. Nursing care for a female client includes
b. Avoid using a nail clipper to cut toenails. removing elastic stockings once per day. The
c. Avoid wearing canvas shoes. Nurse Betty is aware that the rationale for this
d. Avoid using cornstarch on feet. intervention?
a. To increase blood flow to the heart
87. A client is admitted with multiple pressure b. To observe the lower extremities
ulcers. When developing the client's diet plan, c. To allow the leg muscles to stretch and
the nurse should include: relax
a. Fresh orange slices d. To permit veins in the legs to fill with
b. Steamed broccoli blood.
c. Ice cream
d. Ground beef patties 93. Which nursing intervention takes highest priority
when caring for a newly admitted client who's
88. The nurse prepares to administer a cleansing receiving a blood transfusion?
enema. What is the most common client a. Instructing the client to report any
position used for this procedure? itching, swelling, or dyspnea.
a. Lithotomy b. Informing the client that the transfusion
b. Supine usually take 1 ½ to 2 hours.
c. Prone c. Documenting blood administration in
d. Sims’ left lateral the client care record.

6
6
d. Assessing the client’s vital signs when d. 30 minutes after administering the next
the transfusion ends. dose.

94. A male client complains of abdominal discomfort 99. Nurse May is aware that the main advantage of
and nausea while receiving tube feedings. Which using a floor stock system is:
intervention is most appropriate for this a. The nurse can implement medication
problem? orders quickly.
a. Give the feedings at room temperature. b. The nurse receives input from the
b. Decrease the rate of feedings and the pharmacist.
concentration of the formula. c. The system minimizes transcription
c. Place the client in semi-Fowler's position errors.
while feeding. d. The system reinforces accurate
d. Change the feeding container every 12 calculations.
hours.
100. Nurse Oliver is assessing a client's abdomen.
95. Nurse Patricia is reconstituting a powdered Which finding should the nurse report as
medication in a vial. After adding the solution to abnormal?
the powder, she nurse should: a. Dullness over the liver.
a. Do nothing. b. Bowel sounds occurring every 10
b. Invert the vial and let it stand for 3 to 5 seconds.
minutes. c. Shifting dullness over the abdomen.
c. Shake the vial vigorously. d. Vascular sounds heard over the renal
d. Roll the vial gently between the palms. arteries.

96. Which intervention should the nurse Trish use


when administering oxygen by face mask to a
female client?
a. Secure the elastic band tightly around
the client's head.
b. Assist the client to the semi-Fowler
position if possible.
c. Apply the face mask from the client's
chin up over the nose.
d. Loosen the connectors between the
oxygen equipment and humidifier.

97. The maximum transfusion time for a unit of


packed red blood cells (RBCs) is:
a. 6 hours
b. 4 hours
c. 3 hours
d. 2 hours

98. Nurse Monique is monitoring the effectiveness


of a client's drug therapy. When should the
nurse Monique obtain a blood sample to
measure the trough drug level?
a. 1 hour before administering the next
dose.
b. Immediately before administering the
next dose.
c. Immediately after administering the
next dose.
Rationale: Curling’s ulcer occurs as a
generalized stress response in burn patients.
Answers and Rationale – Foundation of This results in a decreased production of
Professional Nursing Practice mucus and increased secretion of gastric acid.
The best treatment for this prophylactic use of
antacids and H2 receptor blockers.
1. Answer: (D) The actions of a reasonably 8. Answer: (D) Continue to monitor and record
prudent nurse with similar education and hourly urine output
experience. Rationale: Normal urine output for an adult is
Rationale: The standard of care is determined approximately 1 ml/minute (60 ml/hour).
by the average degree of skill, care, and Therefore, this client's output is normal.
diligence by nurses in similar circumstances. Beyond continued evaluation, no nursing
2. Answer: (B) I.M action is warranted.
Rationale: With a platelet count of 22,000/μl, 9. Answer: (B) “My ankle feels warm”.
the clients tends to bleed easily. Therefore, Rationale: Ice application decreases pain and
the nurse should avoid using the I.M. route swelling. Continued or increased pain, redness,
because the area is a highly vascular and can and increased warmth are signs of
bleed readily when penetrated by a needle. inflammation that shouldn't occur after ice
The bleeding can be difficult to stop. application
3. Answer: (C) “Digoxin 0.125 mg P.O. once daily” 10. Answer: (B) Hyperkalemia
Rationale: The nurse should always place a Rationale: A loop diuretic removes water and,
zero before a decimal point so that no one along with it, sodium and potassium. This may
misreads the figure, which could result in a result in hypokalemia, hypovolemia, and
dosage error. The nurse should never insert a hyponatremia.
zero at the end of a dosage that includes a 11. Answer:(A) Have condescending trust and
decimal point because this could be misread, confidence in their subordinates
possibly leading to a tenfold increase in the Rationale: Benevolent-authoritative managers
dosage. pretentiously show their trust and confidence
4. Answer: (A) Ineffective peripheral tissue to their followers.
perfusion related to venous congestion. 12. Answer: (A) Provides continuous, coordinated
Rationale: Ineffective peripheral tissue and comprehensive nursing services.
perfusion related to venous congestion takes Rationale: Functional nursing is focused on
the highest priority because venous tasks and activities and not on the care of the
inflammation and clot formation impede blood patients.
flow in a client with deep vein thrombosis. 13. Answer: (B) Standard written order
5. Answer: (B) A 44 year-old myocardial Rationale: This is a standard written order.
infarction (MI) client who is complaining of Prescribers write a single order for
nausea. medications given only once. A stat order is
Rationale: Nausea is a symptom of impending written for medications given immediately for
myocardial infarction (MI) and should be an urgent client problem. A standing order,
assessed immediately so that treatment can also known as a protocol, establishes
be instituted and further damage to the heart guidelines for treating a particular disease or
is avoided. set of symptoms in special care areas such as
6. Answer: (C) Check circulation every 15-30 the coronary care unit. Facilities also may
minutes. institute medication protocols that specifically
Rationale: Restraints encircle the limbs, which designate drugs that a nurse may not give.
place the client at risk for circulation being 14. Answer: (D) Liquid or semi-liquid stools
restricted to the distal areas of the Rationale: Passage of liquid or semi-liquid
extremities. Checking the client’s circulation stools results from seepage of unformed
every 15-30 minutes will allow the nurse to bowel contents around the impacted stool in
adjust the restraints before injury from the rectum. Clients with fecal impaction don't
decreased blood flow occurs. pass hard, brown, formed stools because the
7. Answer: (A) Prevent stress ulcer feces can't move past the impaction. These

6
6
clients typically report the urge to defecate Rationale: The client who has a radiation
(although they can't pass stool) and a implant is placed in a private room and has a
decreased appetite. limited number of visitors. This reduces the
15. Answer: (C) Pulling the helix up and back exposure of others to the radiation.
Rationale: To perform an otoscopic 23. Answer: (C) Risk for infection
examination on an adult, the nurse grasps the Rationale: Agranulocytosis is characterized by
helix of the ear and pulls it up and back to a reduced number of leukocytes (leucopenia)
straighten the ear canal. For a child, the nurse and neutrophils (neutropenia) in the blood.
grasps the helix and pulls it down to straighten The client is at high risk for infection because
the ear canal. Pulling the lobule in any of the decreased body defenses against
direction wouldn't straighten the ear canal for microorganisms. Deficient knowledge related
visualization. to the nature of the disorder may be
16. Answer: (A) Protect the irritated skin from appropriate diagnosis but is not the priority.
sunlight. 24. Answer: (B) Place the client on the left side in
Rationale: Irradiated skin is very sensitive and the Trendelenburg position.
must be protected with clothing or sunblock. Rationale: Lying on the left side may prevent
The priority approach is the avoidance of air from flowing into the pulmonary veins. The
strong sunlight. Trendelenburg position increases intrathoracic
17. Answer: (C) Assist the client in removing pressure, which decreases the amount of
dentures and nail polish. blood pulled into the vena cava during
Rationale: Dentures, hairpins, and combs must aspiration.
be removed. Nail polish must be removed so 25. Answer: (A) Autocratic.
that cyanosis can be easily monitored by Rationale: The autocratic style of leadership is
observing the nail beds. a task-oriented and directive.
18. Answer: (D) Sudden onset of continuous 26. Answer: (D) 2.5 cc
epigastric and back pain. Rationale: 2.5 cc is to be added, because only a
Rationale: The autodigestion of tissue by the 500 cc bag of solution is being medicated
pancreatic enzymes results in pain from instead of a 1 liter.
inflammation, edema, and possible 27. Answer: (A) 50 cc/ hour
hemorrhage. Continuous, unrelieved epigastric Rationale: A rate of 50 cc/hr. The child is to
or back pain reflects the inflammatory process receive 400 cc over a period of 8 hours = 50
in the pancreas. cc/hr.
19. Answer: (B) Provide high-protein, high- 28. Answer: (B) Assess the client for presence of
carbohydrate diet. pain.
Rationale: A positive nitrogen balance is Rationale: Assessing the client for pain is a
important for meeting metabolic needs, tissue very important measure. Postoperative pain is
repair, and resistance to infection. Caloric an indication of complication. The nurse
goals may be as high as 5000 calories per day. should also assess the client for pain to
20. Answer: (A) Blood pressure and pulse rate. provide for the client’s comfort.
Rationale: The baseline must be established to 29. Answer: (A) BP – 80/60, Pulse – 110 irregular
recognize the signs of an anaphylactic or Rationale: The classic signs of cardiogenic
hemolytic reaction to the transfusion. shock are low blood pressure, rapid and weak
21. Answer: (D) Immobilize the leg before moving irregular pulse, cold, clammy skin, decreased
the client. urinary output, and cerebral hypoxia.
Rationale: If the nurse suspects a fracture, 30. Answer: (A) Take the proper equipment, place
splinting the area before moving the client is the client in a comfortable position, and
imperative. The nurse should call for record the appropriate information in the
emergency help if the client is not hospitalized client’s chart.
and call for a physician for the hospitalized Rationale: It is a general or comprehensive
client. statement about the correct procedure, and it
22. Answer: (B) Admit the client into a private includes the basic ideas which are found in the
room. other options
31. Answer: (B) Evaluation
Rationale: Evaluation includes observing the Rationale: Failing eyesight, especially close
person, asking questions, and comparing the vision, is one of the first signs of aging in
patient’s behavioral responses with the middle life (ages 46 to 64). More frequent
expected outcomes. aches and pains begin in the early late years
32. Answer: (C) History of present illness (ages 65 to 79). Increase in loss of muscle tone
Rationale: The history of present illness is the occurs in later years (age 80 and older).
single most important factor in assisting the 41. Answer: (A) Checking and taping all
health professional in arriving at a diagnosis or connections
determining the person’s needs. Rationale: Air leaks commonly occur if the
33. Answer: (A) Trochanter roll extending from the system isn’t secure. Checking all connections
crest of the ileum to the mid-thigh. and taping them will prevent air leaks. The
Rationale: A trochanter roll, properly placed, chest drainage system is kept lower to
provides resistance to the external rotation of promote drainage – not to prevent leaks.
the hip. 42. Answer: (A) Check the client’s identification
34. Answer: (C) Stage III band.
Rationale: Clinically, a deep crater or without Rationale: Checking the client’s identification
undermining of adjacent tissue is noted. band is the safest way to verify a client’s
35. Answer: (A) Second intention healing identity because the band is assigned on
Rationale: When wounds dehisce, they will admission and isn’t be removed at any time. (If
allowed to heal by secondary Intention it is removed, it must be replaced). Asking the
36. Answer: (D) Tachycardia client’s name or having the client repeated his
Rationale: With an extracellular fluid or plasma name would be appropriate only for a client
volume deficit, compensatory mechanisms who’s alert, oriented, and able to understand
stimulate the heart, causing an increase in what is being said, but isn’t the safe standard
heart rate. of practice. Names on bed aren’t always
37. Answer: (A) 0.75 reliable
Rationale: To determine the number of 43. Answer: (B) 32 drops/minute
milliliters the client should receive, the nurse Rationale: Giving 1,000 ml over 8 hours is the
uses the fraction method in the following same as giving 125 ml over 1 hour (60
equation. minutes). Find the number of milliliters per
75 mg/X ml = 100 mg/1 ml minute as follows:
To solve for X, cross-multiply: 125/60 minutes = X/1 minute
75 mg x 1 ml = X ml x 100 mg 60X = 125 = 2.1 ml/minute
75 = 100X To find the number of drops per minute:
75/100 = X 2.1 ml/X gtt = 1 ml/ 15 gtt
0.75 ml (or ¾ ml) = X X = 32 gtt/minute, or 32 drops/minute
38. Answer: (D) it’s a measure of effect, not a 44. Answer: (A) Clamp the catheter
standard measure of weight or quantity. Rationale: If a central venous catheter
Rationale: An insulin unit is a measure of becomes disconnected, the nurse should
effect, not a standard measure of weight or immediately apply a catheter clamp, if
quantity. Different drugs measured in units available. If a clamp isn’t available, the nurse
may have no relationship to one another in can place a sterile syringe or catheter plug in
quality or quantity. the catheter hub. After cleaning the hub with
39. Answer: (B) 38.9 °C alcohol or povidone-iodine solution, the nurse
Rationale: To convert Fahrenheit degreed to must replace the I.V. extension and restart the
Centigrade, use this formula infusion.
°C = (°F – 32) ÷ 1.8 45. Answer: (D) Auscultation, percussion, and
°C = (102 – 32) ÷ 1.8 palpation.
°C = 70 ÷ 1.8 Rationale: The correct order of assessment for
°C = 38.9 examining the abdomen is inspection,
40. Answer: (C) Failing eyesight, especially auscultation, percussion, and palpation. The
close vision. reason for this approach is that the less
intrusive techniques should be performed

6
6
before the more intrusive techniques. interventions and will protect the area. Using a
Percussion and palpation can alter natural povidone-iodine wash and an antibiotic cream
findings during auscultation. require a physician’s order. Massaging with an
46. Answer: (D) Ulnar surface of the hand astringent can further damage the skin.
Rationale: The nurse uses the ulnar surface, or 52. Answer: (D) Foot
ball, of the hand to assess tactile fremitus, Rationale: An elastic bandage should be
thrills, and vocal vibrations through the chest applied form the distal area to the proximal
wall. The fingertips and finger pads best area. This method promotes venous return. In
distinguish texture and shape. The dorsal this case, the nurse should begin applying the
surface best feels warmth. bandage at the client’s foot. Beginning at the
47. Answer: (C) Formative ankle, lower thigh, or knee does not promote
Rationale: Formative (or concurrent) venous return.
evaluation occurs continuously throughout the 53. Answer: (B) Hypokalemia
teaching and learning process. One benefit is Rationale: Insulin administration causes
that the nurse can adjust teaching strategies glucose and potassium to move into the cells,
as necessary to enhance learning. Summative, causing hypokalemia.
or retrospective, evaluation occurs at the 54. Answer: (A) Throbbing headache or dizziness
conclusion of the teaching and learning Rationale: Headache and dizziness often occur
session. Informative is not a type of when nitroglycerin is taken at the beginning of
evaluation. therapy. However, the client usually develops
48. Answer: (B) Once per year tolerance
Rationale: Yearly mammograms should begin 55. Answer: (D) Check the client’s level of
at age 40 and continue for as long as the consciousness
woman is in good health. If health risks, such Rationale: Determining unresponsiveness is
as family history, genetic tendency, or past the first step assessment action to take. When
breast cancer, exist, more frequent a client is in ventricular tachycardia, there is a
examinations may be necessary. significant decrease in cardiac output.
49. Answer: (A) Respiratory acidosis However, checking the unresponsiveness
Rationale: The client has a below-normal ensures whether the client is affected by the
(acidic) blood pH value and an above-normal decreased cardiac output.
partial pressure of arterial carbon dioxide 56. Answer: (B) On the affected side of the client.
(Paco2) value, indicating respiratory acidosis. Rationale: When walking with clients, the
In respiratory alkalosis, the pH value is above nurse should stand on the affected side and
normal and in the Paco2 value is below grasp the security belt in the midspine area of
normal. In metabolic acidosis, the pH and the small of the back. The nurse should
bicarbonate (Hco3) values are below normal. position the free hand at the shoulder area so
In metabolic alkalosis, the pH and Hco3 values that the client can be pulled toward the nurse
are above normal. in the event that there is a forward fall. The
50. Answer: (B) To provide support for the client client is instructed to look up and outward
and family in coping with terminal illness. rather than at his or her feet.
Rationale: Hospices provide supportive care 57. Answer: (A) Urine output: 45 ml/hr
for terminally ill clients and their families. Rationale: Adequate perfusion must be
Hospice care doesn’t focus on counseling maintained to all vital organs in order for the
regarding health care costs. Most client client to remain visible as an organ donor. A
referred to hospices have been treated for urine output of 45 ml per hour indicates
their disease without success and will receive adequate renal perfusion. Low blood pressure
only palliative care in the hospice. and delayed capillary refill time are circulatory
51. Answer: (C) Using normal saline solution to system indicators of inadequate perfusion. A
clean the ulcer and applying a protective serum pH of 7.32 is acidotic, which adversely
dressing as necessary. affects all body tissues.
Rationale: Washing the area with normal 58. Answer: (D ) Obtaining the specimen from the
saline solution and applying a protective urinary drainage bag.
dressing are within the nurse’s realm of
Rationale: A urine specimen is not taken from because of the risk for potential heat loss.
the urinary drainage bag. Urine undergoes Hurried movements and rapid changes in the
chemical changes while sitting in the bag and position should be avoided because these
does not necessarily reflect the current client predispose the client to hypotension. At the
status. In addition, it may become time of the transfer from the surgery table to
contaminated with bacteria from opening the the stretcher, the client is still affected by the
system. effects of the anesthesia; therefore, the client
59. Answer: (B) Cover the client, place the call should not move self. Safety belts can prevent
light within reach, and answer the phone call. the client from falling off the stretcher.
Rationale: Because telephone call is an 64. Answer: (B) Gown and gloves
emergency, the nurse may need to answer it. Rationale: Contact precautions require the use
The other appropriate action is to ask another of gloves and a gown if direct client contact is
nurse to accept the call. However, is not one of anticipated. Goggles are not necessary unless
the options. To maintain privacy and safety, the nurse anticipates the splashes of blood,
the nurse covers the client and places the call body fluids, secretions, or excretions may
light within the client’s reach. Additionally, the occur. Shoe protectors are not necessary.
client’s door should be closed or the room 65. Answer: (C) Quad cane
curtains pulled around the bathing area. Rationale: Crutches and a walker can be
60. Answer: (C) Use a sterile plastic container for difficult to maneuver for a client with
obtaining the specimen. weakness on one side. A cane is better suited
Rationale: Sputum specimens for culture and for client with weakness of the arm and leg on
sensitivity testing need to be obtained using one side. However, the quad cane would
sterile techniques because the test is done to provide the most stability because of the
determine the presence of organisms. If the structure of the cane and because a quad cane
procedure for obtaining the specimen is not has four legs.
sterile, then the specimen is not sterile, then 66. Answer: (D) Left side-lying with the head of
the specimen would be contaminated and the the bed elevated 45 degrees.
results of the test would be invalid. Rationale: To facilitate removal of fluid from
61. Answer: (A) Puts all the four points of the the chest wall, the client is positioned sitting at
walker flat on the floor, puts weight on the the edge of the bed leaning over the bedside
hand pieces, and then walks into it. table with the feet supported on a stool. If the
Rationale: When the client uses a walker, the client is unable to sit up, the client is
nurse stands adjacent to the affected side. The positioned lying in bed on the unaffected side
client is instructed to put all four points of the with the head of the bed elevated 30 to 45
walker 2 feet forward flat on the floor before degrees.
putting weight on hand pieces. This will ensure 67. Answer: (D) Reliability
client safety and prevent stress cracks in the Rationale: Reliability is consistency of the
walker. The client is then instructed to move research instrument. It refers to the
the walker forward and walk into it. repeatability of the instrument in extracting
62. Answer: (C) Draws one line to cross out the the same responses upon its repeated
incorrect information and then initials the administration.
change. 68. Answer: (A) Keep the identities of the subject
Rationale: To correct an error documented in a secret
medical record, the nurse draws one line Rationale: Keeping the identities of the
through the incorrect information and then research subject secret will ensure anonymity
initials the error. An error is never erased and because this will hinder providing link between
correction fluid is never used in the medical the information given to whoever is its source.
record. 69. Answer: (A) Descriptive- correlational
63. Answer: (C) Secures the client safety belts Rationale: Descriptive- correlational study is
after transferring to the stretcher. the most appropriate for this study because
Rationale: During the transfer of the client it studies the variables that could be the
after the surgical procedure is complete, the antecedents of the increased incidence of
nurse should avoid exposure of the client nosocomial infection.

7
7
70. Answer: (C) Use of laboratory data working conditions of the workers on their
Rationale: Incidence of nosocomial infection is productivity. It resulted to an increased
best collected through the use of productivity but not due to the intervention
biophysiologic measures, particularly in vitro but due to the psychological effects of being
measurements, hence laboratory data is observed. They performed differently because
essential. they were under observation.
71. Answer: (B) Quasi-experiment 79. Answer: (B) Determines the different
Rationale: Quasi-experiment is done when nationality of patients frequently admitted and
randomization and control of the variables are decides to get representations samples from
not possible. each.
72. Answer: (C) Primary source Rationale: Judgment sampling involves
Rationale: This refers to a primary source including samples according to the knowledge
which is a direct account of the investigation of the investigator about the participants in
done by the investigator. In contrast to this is a the study.
secondary source, which is written by 80. Answer: (B) Madeleine Leininger
someone other than the original researcher. Rationale: Madeleine Leininger developed the
73. Answer: (A) Non-maleficence theory on transcultural theory based on her
Rationale: Non-maleficence means do not observations on the behavior of selected
cause harm or do any action that will cause people within a culture.
any harm to the patient/client. To do good is 81. Answer: (A) Random
referred as beneficence. Rationale: Random sampling gives equal
74. Answer: (C) Res ipsa loquitor chance for all the elements in the population
Rationale: Res ipsa loquitor literally means the to be picked as part of the sample.
thing speaks for itself. This means in 82. Answer: (A) Degree of agreement and
operational terms that the injury caused is the disagreement
proof that there was a negligent act. Rationale: Likert scale is a 5-point summated
75. Answer: (B) The Board can investigate scale used to determine the degree of
violations of the nursing law and code of ethics agreement or disagreement of the
Rationale: Quasi-judicial power means that the respondents to a statement in a study
Board of Nursing has the authority to 83. Answer: (B) Sr. Callista Roy
investigate violations of the nursing law and Rationale: Sr. Callista Roy developed the
can issue summons, subpoena or subpoena Adaptation Model which involves the
duces tecum as needed. physiologic mode, self-concept mode, role
76. Answer: (C) May apply for re-issuance of function mode and dependence mode.
his/her license based on certain conditions 84. Answer: (A) Span of control
stipulated in RA 9173 Rationale: Span of control refers to the
Rationale: RA 9173 sec. 24 states that for number of workers who report directly to a
equity and justice, a revoked license maybe re- manager.
issued provided that the following conditions 85. Answer: (B) Autonomy
are met: a) the cause for revocation of license Rationale: Informed consent means that the
has already been corrected or removed; and, patient fully understands about the surgery,
b) at least four years has elapsed since the including the risks involved and the alternative
license has been revoked. solutions. In giving consent it is done with full
77. Answer: (B) Review related literature knowledge and is given freely. The action of
Rationale: After formulating and delimiting the allowing the patient to decide whether a
research problem, the researcher conducts a surgery is to be done or not exemplifies the
review of related literature to determine the bioethical principle of autonomy.
extent of what has been done on the study by 86. Answer: (C) Avoid wearing canvas shoes.
previous researchers. Rationale: The client should be instructed to
78. Answer: (B) Hawthorne effect avoid wearing canvas shoes. Canvas shoes
Rationale: Hawthorne effect is based on the cause the feet to perspire, which may, in turn,
study of Elton Mayo and company about the cause skin irritation and breakdown. Both
effect of an intervention done to improve the cotton and cornstarch absorb perspiration.
The client should be instructed to cut toenails nursing process where the nurse puts the plan
straight across with nail clippers. of care into action.
87. Answer: (D) Ground beef patties 92. Answer: (B) To observe the lower extremities
Rationale: Meat is an excellent source of Rationale: Elastic stockings are used to
complete protein, which this client needs to promote venous return. The nurse needs to
repair the tissue breakdown caused by remove them once per day to observe the
pressure ulcers. Oranges and broccoli supply condition of the skin underneath the stockings.
vitamin C but not protein. Ice cream supplies Applying the stockings increases blood flow to
only some incomplete protein, making it less the heart. When the stockings are in place, the
helpful in tissue repair. leg muscles can still stretch and relax, and the
88. Answer: (D) Sims’ left lateral veins can fill with blood.
Rationale: The Sims' left lateral position is the 93. Answer :(A) Instructing the client to report any
most common position used to administer a itching, swelling, or dyspnea.
cleansing enema because it allows gravity to Rationale: Because administration of blood or
aid the flow of fluid along the curve of the blood products may cause serious adverse
sigmoid colon. If the client can't assume this effects such as allergic reactions, the nurse
position nor has poor sphincter control, the must monitor the client for these effects. Signs
dorsal recumbent or right lateral position may and symptoms of life-threatening allergic
be used. The supine and prone positions are reactions include itching, swelling, and
inappropriate and uncomfortable for the dyspnea. Although the nurse should inform
client. the client of the duration of the transfusion
89. Answer: (A) Arrange for typing and cross and should document its administration, these
matching of the client’s blood. actions are less critical to the client's
Rationale: The nurse first arranges for typing immediate health. The nurse should assess
and cross matching of the client's blood to vital signs at least hourly during the
ensure compatibility with donor blood. The transfusion.
other options, although appropriate when 94. Answer: (B) Decrease the rate of feedings and
preparing to administer a blood transfusion, the concentration of the formula.
come later. Rationale: Complaints of abdominal
90. Answer: (A) Independent discomfort and nausea are common in clients
Rationale: Nursing interventions are classified receiving tube feedings. Decreasing the rate of
as independent, interdependent, or the feeding and the concentration of the
dependent. Altering the drug schedule to formula should decrease the client's
coincide with the client's daily routine discomfort. Feedings are normally given at
represents an independent intervention, room temperature to minimize abdominal
whereas consulting with the physician and cramping. To prevent aspiration during
pharmacist to change a client's medication feeding, the head of the client's bed should be
because of adverse reactions represents an elevated at least 30 degrees. Also, to prevent
interdependent intervention. Administering an bacterial growth, feeding containers should be
already-prescribed drug on time is a routinely changed every 8 to 12 hours.
dependent intervention. An intradependent 95. Answer: (D) Roll the vial gently between the
nursing intervention doesn't exist. palms.
91. Answer: (D) Evaluation Rationale: Rolling the vial gently between the
Rationale: The nursing actions described palms produces heat, which helps dissolve the
constitute evaluation of the expected medication. Doing nothing or inverting the vial
outcomes. The findings show that the wouldn't help dissolve the medication. Shaking
expected outcomes have been achieved. the vial vigorously could cause the medication
Assessment consists of the client's history, to break down, altering its action.
physical examination, and laboratory studies. 96. Answer: (B) Assist the client to the semi-
Analysis consists of considering assessment Fowler position if possible.
information to derive the appropriate nursing Rationale: By assisting the client to the semi-
diagnosis. Implementation is the phase of the Fowler position, the nurse promotes easier
chest expansion, breathing, and oxygen intake.

7
7
The nurse should secure the elastic band so
that the face mask fits comfortably and snugly
rather than tightly, which could lead to
irritation. The nurse should apply the face
mask from the client's nose down to the chin
— not vice versa. The nurse should check the
connectors between the oxygen equipment
and humidifier to ensure that they're airtight;
loosened connectors can cause loss of oxygen.
97. Answer: (B) 4 hours
Rationale: A unit of packed RBCs may be given
over a period of between 1 and 4 hours. It
shouldn't infuse for longer than 4 hours
because the risk of contamination and sepsis
increases after that time. Discard or return to
the blood bank any blood not given within this
time, according to facility policy.
98. Answer: (B) Immediately before administering
the next dose.
Rationale: Measuring the blood drug
concentration helps determine whether the
dosing has achieved the therapeutic goal. For
measurement of the trough, or lowest, blood
level of a drug, the nurse draws a blood
sample immediately before administering the
next dose. Depending on the drug's duration
of action and half-life, peak blood drug levels
typically are drawn after administering the
next dose.
99. Answer: (A) The nurse can implement
medication orders quickly.
Rationale: A floor stock system enables the
nurse to implement medication orders quickly.
It doesn't allow for pharmacist input, nor does
it minimize transcription errors or reinforce
accurate calculations.
100. Answer: (C) Shifting dullness over the
abdomen.
Rationale: Shifting dullness over the abdomen
indicates ascites, an abnormal finding. The
other options are normal abdominal findings.
TEST II - Community Health Nursing and Care of the a. Excessive fetal activity.
Mother and Child b. Larger than normal uterus for
gestational age.
1. May arrives at the health care clinic and tells the c. Vaginal bleeding
nurse that her last menstrual period was 9 d. Elevated levels of human chorionic
weeks ago. She also tells the nurse that a home gonadotropin.
pregnancy test was positive but she began to
have mild cramps and is now having moderate 6. A pregnant client is receiving magnesium
vaginal bleeding. During the physical sulfate for severe pregnancy induced
examination of the client, the nurse notes that hypertension (PIH). The clinical findings that
May has a dilated cervix. The nurse determines would warrant use of the antidote , calcium
that May is experiencing which type of abortion? gluconate is:
a. Inevitable a. Urinary output 90 cc in 2 hours.
b. Incomplete b. Absent patellar reflexes.
c. Threatened c. Rapid respiratory rate above 40/min.
d. Septic d. Rapid rise in blood pressure.

2. Nurse Reese is reviewing the record of a 7. During vaginal examination of Janah who is in
pregnant client for her first prenatal visit. Which labor, the presenting part is at station plus two.
of the following data, if noted on the client’s Nurse, correctly interprets it as:
record, would alert the nurse that the client is at a. Presenting part is 2 cm above the plane
risk for a spontaneous abortion? of the ischial spines.
a. Age 36 years b. Biparietal diameter is at the level of the
b. History of syphilis ischial spines.
c. History of genital herpes c. Presenting part in 2 cm below the plane
d. History of diabetes mellitus of the ischial spines.
d. Biparietal diameter is 2 cm above the
3. Nurse Hazel is preparing to care for a client who ischial spines.
is newly admitted to the hospital with a possible
diagnosis of ectopic pregnancy. Nurse Hazel 8. A pregnant client is receiving oxytocin (Pitocin)
develops a plan of care for the client and for induction of labor. A condition that warrant
determines that which of the following nursing the nurse in-charge to discontinue I.V. infusion
actions is the priority? of Pitocin is:
a. Monitoring weight a. Contractions every 1 ½ minutes lasting
b. Assessing for edema 70-80 seconds.
c. Monitoring apical pulse b. Maternal temperature 101.2
d. Monitoring temperature c. Early decelerations in the fetal heart
rate.
4. Nurse Oliver is teaching a diabetic pregnant d. Fetal heart rate baseline 140-160 bpm.
client about nutrition and insulin needs during
pregnancy. The nurse determines that the client 9. Calcium gluconate is being administered to a
understands dietary and insulin needs if the client with pregnancy induced hypertension
client states that the second half of pregnancy (PIH). A nursing action that must be initiated as
requires: the plan of care throughout injection of the drug
a. Decreased caloric intake is:
b. Increased caloric intake a. Ventilator assistance
c. Decreased Insulin b. CVP readings
d. Increase Insulin c. EKG tracings
d. Continuous CPR
5. Nurse Michelle is assessing a 24 year old client
with a diagnosis of hydatidiform mole. She is 10. A trial for vaginal delivery after an earlier
aware that one of the following is unassociated caesarean, would likely to be given to a gravida,
with this condition? who had:

7
7
a. First low transverse cesarean was for infant looks for it. The nurse is aware that
active herpes type 2 infections; vaginal estimated age of the infant would be:
culture at 39 weeks pregnancy was a. 6 months
positive. b. 4 months
b. First and second caesareans were for c. 8 months
cephalopelvic disproportion. d. 10 months
c. First caesarean through a classic incision
as a result of severe fetal distress. 16. Which of the following is the most prominent
d. First low transverse caesarean was for feature of public health nursing?
breech position. Fetus in this pregnancy a. It involves providing home care to sick
is in a vertex presentation. people who are not confined in the
hospital.
11. Nurse Ryan is aware that the best initial b. Services are provided free of charge to
approach when trying to take a crying toddler’s people within the catchments area.
temperature is: c. The public health nurse functions as part
a. Talk to the mother first and then to the of a team providing a public health
toddler. nursing services.
b. Bring extra help so it can be done d. Public health nursing focuses on
quickly. preventive, not curative, services.
c. Encourage the mother to hold the child.
d. Ignore the crying and screaming. 17. When the nurse determines whether resources
were maximized in implementing Ligtas Tigdas,
12. Baby Tina a 3 month old infant just had a cleft lip she is evaluating
and palate repair. What should the nurse do to a. Effectiveness
prevent trauma to operative site? b. Efficiency
a. Avoid touching the suture line, even c. Adequacy
when cleaning. d. Appropriateness
b. Place the baby in prone position.
c. Give the baby a pacifier. 18. Vangie is a new B.S.N. graduate. She wants to
d. Place the infant’s arms in soft elbow become a Public Health Nurse. Where should
restraints. she apply?
a. Department of Health
13. Which action should nurse Marian include in the b. Provincial Health Office
care plan for a 2 month old with heart failure? c. Regional Health Office
a. Feed the infant when he cries. d. Rural Health Unit
b. Allow the infant to rest before feeding.
c. Bathe the infant and administer 19. Tony is aware the Chairman of the Municipal
medications before feeding. Health Board is:
d. Weigh and bathe the infant before a. Mayor
feeding. b. Municipal Health Officer
c. Public Health Nurse
14. Nurse Hazel is teaching a mother who plans to d. Any qualified physician
discontinue breast feeding after 5 months. The
nurse should advise her to include which foods 20. Myra is the public health nurse in a municipality
in her infant’s diet? with a total population of about 20,000. There
a. Skim milk and baby food. are 3 rural health midwives among the RHU
b. Whole milk and baby food. personnel. How many more midwife items will
c. Iron-rich formula only. the RHU need?
d. Iron-rich formula and baby food. a. 1
b. 2
15. Mommy Linda is playing with her infant, who is c. 3
sitting securely alone on the floor of the clinic. d. The RHU does not need any more
The mother hides a toy behind her back and the midwife item.
26. The nurse is caring for a primigravid client in the
21. According to Freeman and Heinrich, community labor and delivery area. Which condition would
health nursing is a developmental service. Which place the client at risk for disseminated
of the following best illustrates this statement? intravascular coagulation (DIC)?
a. The community health nurse a. Intrauterine fetal death.
continuously develops himself b. Placenta accreta.
personally and professionally. c. Dysfunctional labor.
b. Health education and community d. Premature rupture of the membranes.
organizing are necessary in providing
community health services. 27. A fullterm client is in labor. Nurse Betty is aware
c. Community health nursing is intended that the fetal heart rate would be:
primarily for health promotion and a. 80 to 100 beats/minute
prevention and treatment of disease. b. 100 to 120 beats/minute
d. The goal of community health nursing is c. 120 to 160 beats/minute
to provide nursing services to people in d. 160 to 180 beats/minute
their own places of residence.
28. The skin in the diaper area of a 7 month old
22. Nurse Tina is aware that the disease declared infant is excoriated and red. Nurse Hazel should
through Presidential Proclamation No. 4 as a instruct the mother to:
target for eradication in the Philippines is? a. Change the diaper more often.
a. Poliomyelitis b. Apply talc powder with diaper changes.
b. Measles c. Wash the area vigorously with each
c. Rabies diaper change.
d. Neonatal tetanus d. Decrease the infant’s fluid intake to
decrease saturating diapers.
23. May knows that the step in community
organizing that involves training of potential 29. Nurse Carla knows that the common cardiac
leaders in the community is: anomalies in children with Down Syndrome (tri-
a. Integration somy 21) is:
b. Community organization a. Atrial septal defect
c. Community study b. Pulmonic stenosis
d. Core group formation c. Ventricular septal defect
d. Endocardial cushion defect
24. Beth a public health nurse takes an active role in
community participation. What is the primary 30. Malou was diagnosed with severe preeclampsia
goal of community organizing? is now receiving I.V. magnesium sulfate. The
a. To educate the people regarding adverse effects associated with magnesium
community health problems sulfate is:
b. To mobilize the people to resolve a. Anemia
community health problems b. Decreased urine output
c. To maximize the community’s resources c. Hyperreflexia
in dealing with health problems. d. Increased respiratory rate
d. To maximize the community’s resources
in dealing with health problems. 31. A 23 year old client is having her menstrual
period every 2 weeks that last for 1 week.
25. Tertiary prevention is needed in which stage of This type of menstrual pattern is bets defined
the natural history of disease? by:
a. Pre-pathogenesis a. Menorrhagia
b. Pathogenesis b. Metrorrhagia
c. Prodromal c. Dyspareunia
d. Terminal d. Amenorrhea

7
7
32. Jannah is admitted to the labor and delivery b. Dehydration and diarrhea
unit. The critical laboratory result for this client c. Bradycardia and hypotension
would be: d. Petechiae and hematuria
a. Oxygen saturation
b. Iron binding capacity 38. To evaluate a woman’s understanding about the
c. Blood typing use of diaphragm for family planning, Nurse
d. Serum Calcium Trish asks her to explain how she will use the
appliance. Which response indicates a need for
33. Nurse Gina is aware that the most common further health teaching?
condition found during the second-trimester of a. “I should check the diaphragm carefully
pregnancy is: for holes every time I use it”
a. Metabolic alkalosis b. “I may need a different size of
b. Respiratory acidosis diaphragm if I gain or lose weight more
c. Mastitis than 20 pounds”
d. Physiologic anemia c. “The diaphragm must be left in place for
atleast 6 hours after intercourse”
34. Nurse Lynette is working in the triage area of an d. “I really need to use the diaphragm and
emergency department. She sees that several jelly most during the middle of my
pediatric clients arrive simultaneously. The client menstrual cycle”.
who needs to be treated first is:
a. A crying 5 year old child with a 39. Hypoxia is a common complication of
laceration on his scalp. laryngotracheobronchitis. Nurse Oliver should
b. A 4 year old child with a barking coughs frequently assess a child with
and flushed appearance. laryngotracheobronchitis for:
c. A 3 year old child with Down syndrome a. Drooling
who is pale and asleep in his mother’s b. Muffled voice
arms. c. Restlessness
d. A 2 year old infant with stridorous d. Low-grade fever
breath sounds, sitting up in his mother’s
arms and drooling. 40. How should Nurse Michelle guide a child who is
blind to walk to the playroom?
35. Maureen in her third trimester arrives at the a. Without touching the child, talk
emergency room with painless vaginal bleeding. continuously as the child walks down the
Which of the following conditions is suspected? hall.
a. Placenta previa b. Walk one step ahead, with the child’s
b. Abruptio placentae hand on the nurse’s elbow.
c. Premature labor c. Walk slightly behind, gently guiding the
d. Sexually transmitted disease child forward.
d. Walk next to the child, holding the
36. A young child named Richard is suspected of child’s hand.
having pinworms. The community nurse collects
a stool specimen to confirm the diagnosis. The 41. When assessing a newborn diagnosed with
nurse should schedule the collection of this ductus arteriosus, Nurse Olivia should expect
specimen for: that the child most likely would have an:
a. Just before bedtime a. Loud, machinery-like murmur.
b. After the child has been bathe b. Bluish color to the lips.
c. Any time during the day c. Decreased BP reading in the upper
d. Early in the morning extremities
d. Increased BP reading in the upper
37. In doing a child’s admission assessment, Nurse extremities.
Betty should be alert to note which signs or
symptoms of chronic lead poisoning? 42. The reason nurse May keeps the neonate in a
a. Irritability and seizures neutral thermal environment is that when a
newborn becomes too cool, the neonate 47. Barangay Pinoy had an outbreak of German
requires: measles. To prevent congenital rubella, what is
a. Less oxygen, and the newborn’s the BEST advice that you can give to women in
metabolic rate increases. the first trimester of pregnancy in the barangay
b. More oxygen, and the newborn’s Pinoy?
metabolic rate decreases. a. Advise them on the signs of German
c. More oxygen, and the newborn’s measles.
metabolic rate increases. b. Avoid crowded places, such as markets
d. Less oxygen, and the newborn’s and movie houses.
metabolic rate decreases. c. Consult at the health center where
rubella vaccine may be given.
43. Before adding potassium to an infant’s I.V. line, d. Consult a physician who may give them
Nurse Ron must be sure to assess whether this rubella immunoglobulin.
infant has:
a. Stable blood pressure 48. Myrna a public health nurse knows that to
b. Patant fontanelles determine possible sources of sexually
c. Moro’s reflex transmitted infections, the BEST method that
d. Voided may be undertaken is:
a. Contact tracing
44. Nurse Carla should know that the most common b. Community survey
causative factor of dermatitis in infants and c. Mass screening tests
younger children is: d. Interview of suspects
a. Baby oil
b. Baby lotion 49. A 33-year old female client came for
c. Laundry detergent consultation at the health center with the chief
d. Powder with cornstarch complaint of fever for a week. Accompanying
symptoms were muscle pains and body malaise.
45. During tube feeding, how far above an infant’s A week after the start of fever, the client noted
stomach should the nurse hold the syringe with yellowish discoloration of his sclera. History
formula? showed that he waded in flood waters about 2
a. 6 inches weeks before the onset of symptoms. Based on
b. 12 inches her history, which disease condition will you
c. 18 inches suspect?
d. 24 inches a. Hepatitis A
b. Hepatitis B
46. In a mothers’ class, Nurse Lhynnete discussed c. Tetanus
childhood diseases such as chicken pox. Which d. Leptospirosis
of the following statements about chicken pox is
correct? 50. Mickey a 3-year old client was brought to the
a. The older one gets, the more susceptible health center with the chief complaint of severe
he becomes to the complications of diarrhea and the passage of “rice water” stools.
chicken pox. The client is most probably suffering from which
b. A single attack of chicken pox will condition?
prevent future episodes, including a. Giardiasis
conditions such as shingles. b. Cholera
c. To prevent an outbreak in the c. Amebiasis
community, quarantine may be imposed d. Dysentery
by health authorities.
d. Chicken pox vaccine is best given when 51. The most prevalent form of meningitis among
there is an impending outbreak in the children aged 2 months to 3 years is caused by
community. which microorganism?
a. Hemophilus influenzae
b. Morbillivirus

7
7
c. Steptococcus pneumoniae d. Use of protective footwear, such as
d. Neisseria meningitidis rubber boots

52. The student nurse is aware that the 58. Several clients is newly admitted and diagnosed
pathognomonic sign of measles is Koplik’s spot with leprosy. Which of the following clients
and you may see Koplik’s spot by inspecting the: should be classified as a case of multibacillary
a. Nasal mucosa leprosy?
b. Buccal mucosa a. 3 skin lesions, negative slit skin smear
c. Skin on the abdomen b. 3 skin lesions, positive slit skin smear
d. Skin on neck c. 5 skin lesions, negative slit skin smear
d. 5 skin lesions, positive slit skin smear
53. Angel was diagnosed as having Dengue fever.
You will say that there is slow capillary refill 59. Nurses are aware that diagnosis of leprosy is
when the color of the nailbed that you pressed highly dependent on recognition of symptoms.
does not return within how many seconds? Which of the following is an early sign of
a. 3 seconds leprosy?
b. 6 seconds a. Macular lesions
c. 9 seconds b. Inability to close eyelids
d. 10 seconds c. Thickened painful nerves
d. Sinking of the nosebridge
54. In Integrated Management of Childhood Illness,
the nurse is aware that the severe conditions 60. Marie brought her 10 month old infant for
generally require urgent referral to a hospital. consultation because of fever, started 4 days
Which of the following severe conditions DOES prior to consultation. In determining malaria
NOT always require urgent referral to a hospital? risk, what will you do?
a. Mastoiditis a. Perform a tourniquet test.
b. Severe dehydration b. Ask where the family resides.
c. Severe pneumonia c. Get a specimen for blood smear.
d. Severe febrile disease d. Ask if the fever is present every day.

55. Myrna a public health nurse will conduct 61. Susie brought her 4 years old daughter to the
outreach immunization in a barangay Masay RHU because of cough and colds. Following the
with a population of about 1500. The estimated IMCI assessment guide, which of the following is
number of infants in the barangay would be: a danger sign that indicates the need for urgent
a. 45 infants referral to a hospital?
b. 50 infants a. Inability to drink
c. 55 infants b. High grade fever
d. 65 infants c. Signs of severe dehydration
d. Cough for more than 30 days
56. The community nurse is aware that the
biological used in Expanded Program on 62. Jimmy a 2-year old child revealed “baggy pants”.
Immunization (EPI) should NOT be stored in the As a nurse, using the IMCI guidelines, how will
freezer? you manage Jimmy?
a. DPT a. Refer the child urgently to a hospital for
b. Oral polio vaccine confinement.
c. Measles vaccine b. Coordinate with the social worker to
d. MMR enroll the child in a feeding program.
c. Make a teaching plan for the mother,
57. It is the most effective way of controlling focusing on menu planning for her child.
schistosomiasis in an endemic area? d. Assess and treat the child for health
a. Use of molluscicides problems like infections and intestinal
b. Building of foot bridges parasitism.
c. Proper use of sanitary toilets
63. Gina is using Oresol in the management of 68. The nurse explains to a breastfeeding mother
diarrhea of her 3-year old child. She asked you that breast milk is sufficient for all of the baby’s
what to do if her child vomits. As a nurse you will nutrient needs only up to:
tell her to: a. 5 months
a. Bring the child to the nearest hospital b. 6 months
for further assessment. c. 1 year
b. Bring the child to the health center for d. 2 years
intravenous fluid therapy.
c. Bring the child to the health center for 69. Nurse Ron is aware that the gestational age of a
assessment by the physician. conceptus that is considered viable (able to live
d. Let the child rest for 10 minutes then outside the womb) is:
continue giving Oresol more slowly. a. 8 weeks
b. 12 weeks
64. Nikki a 5-month old infant was brought by his c. 24 weeks
mother to the health center because of diarrhea d. 32 weeks
for 4 to 5 times a day. Her skin goes back slowly
after a skin pinch and her eyes are sunken. Using 70. When teaching parents of a neonate the proper
the IMCI guidelines, you will classify this infant in position for the neonate’s sleep, the nurse
which category? Patricia stresses the importance of placing the
a. No signs of dehydration neonate on his back to reduce the risk of which
b. Some dehydration of the following?
c. Severe dehydration a. Aspiration
d. The data is insufficient. b. Sudden infant death syndrome (SIDS)
c. Suffocation
65. Chris a 4-month old infant was brought by her d. Gastroesophageal reflux (GER)
mother to the health center because of cough.
His respiratory rate is 42/minute. Using the 71. Which finding might be seen in baby James a
Integrated Management of Child Illness (IMCI) neonate suspected of having an infection?
guidelines of assessment, his breathing is a. Flushed cheeks
considered as: b. Increased temperature
a. Fast c. Decreased temperature
b. Slow d. Increased activity level
c. Normal
d. Insignificant 72. Baby Jenny who is small-for-gestation is at
increased risk during the transitional period for
66. Maylene had just received her 4th dose of which complication?
tetanus toxoid. She is aware that her baby will a. Anemia probably due to chronic fetal
have protection against tetanus for hyposia
a. 1 year b. Hyperthermia due to decreased
b. 3 years glycogen stores
c. 5 years c. Hyperglycemia due to decreased
d. Lifetime glycogen stores
d. Polycythemia probably due to chronic
67. Nurse Ron is aware that unused BCG should be fetal hypoxia
discarded after how many hours of
reconstitution? 73. Marjorie has just given birth at 42 weeks’
a. 2 hours gestation. When the nurse assessing the
b. 4 hours neonate, which physical finding is expected?
c. 8 hours a. A sleepy, lethargic baby
d. At the end of the day b. Lanugo covering the body
c. Desquamation of the epidermis
d. Vernix caseosa covering the body

8
8
74. After reviewing the Myrna’s maternal history of b. The parent’s expression of interest
magnesium sulfate during labor, which condition about the size of the new born.
would nurse Richard anticipate as a potential c. The parents’ indication that they want to
problem in the neonate? see the newborn.
a. Hypoglycemia d. The parents’ interactions with each
b. Jitteriness other.
c. Respiratory depression
d. Tachycardia 80. Following a precipitous delivery, examination of
the client's vagina reveals a fourth-degree
75. Which symptom would indicate the Baby laceration. Which of the following would be
Alexandra was adapting appropriately to extra- contraindicated when caring for this client?
uterine life without difficulty? a. Applying cold to limit edema during the
a. Nasal flaring first 12 to 24 hours.
b. Light audible grunting b. Instructing the client to use two or more
c. Respiratory rate 40 to 60 peripads to cushion the area.
breaths/minute c. Instructing the client on the use of sitz
d. Respiratory rate 60 to 80 baths if ordered.
breaths/minute d. Instructing the client about the
importance of perineal (kegel) exercises.
76. When teaching umbilical cord care for Jennifer a
new mother, the nurse Jenny would include 81. A pregnant woman accompanied by her
which information? husband, seeks admission to the labor and
a. Apply peroxide to the cord with each delivery area. She states that she's in labor and
diaper change says she attended the facility clinic for prenatal
b. Cover the cord with petroleum jelly after care. Which question should the nurse Oliver ask
bathing her first?
c. Keep the cord dry and open to air a. “Do you have any chronic illnesses?”
d. Wash the cord with soap and water each b. “Do you have any allergies?”
day during a tub bath. c. “What is your expected due date?”
d. “Who will be with you during labor?”
77. Nurse John is performing an assessment on a
neonate. Which of the following findings is 82. A neonate begins to gag and turns a dusky color.
considered common in the healthy neonate? What should the nurse do first?
a. Simian crease a. Calm the neonate.
b. Conjunctival hemorrhage b. Notify the physician.
c. Cystic hygroma c. Provide oxygen via face mask as ordered
d. Bulging fontanelle d. Aspirate the neonate’s nose and mouth
with a bulb syringe.
78. Dr. Esteves decides to artificially rupture the
membranes of a mother who is on labor. 83. When a client states that her "water broke,"
Following this procedure, the nurse Hazel checks which of the following actions would be
the fetal heart tones for which the following inappropriate for the nurse to do?
reasons? a. Observing the pooling of straw-colored
a. To determine fetal well-being. fluid.
b. To assess for prolapsed cord b. Checking vaginal discharge with nitrazine
c. To assess fetal position paper.
d. To prepare for an imminent delivery. c. Conducting a bedside ultrasound for an
79. Which of the following would be least likely to amniotic fluid index.
indicate anticipated bonding behaviors by new d. Observing for flakes of vernix in the
parents? vaginal discharge.
a. The parents’ willingness to touch and
hold the new born. 84. A baby girl is born 8 weeks premature. At birth,
she has no spontaneous respirations but is
successfully resuscitated. Within several hours c. Decreased inspiratory capacity
she develops respiratory grunting, cyanosis, d. Decreased oxygen consumption
tachypnea, nasal flaring, and retractions. She's
diagnosed with respiratory distress syndrome, 90. Emily has gestational diabetes and it is usually
intubated, and placed on a ventilator. Which managed by which of the following therapy?
nursing action should be included in the baby's a. Diet
plan of care to prevent retinopathy of b. Long-acting insulin
prematurity? c. Oral hypoglycemic
a. Cover his eyes while receiving oxygen. d. Oral hypoglycemic drug and insulin
b. Keep her body temperature low.
c. Monitor partial pressure of oxygen 91. Magnesium sulfate is given to Jemma with
(Pao2) levels. preeclampsia to prevent which of the following
d. Humidify the oxygen. condition?
a. Hemorrhage
85. Which of the following is normal newborn b. Hypertension
calorie intake? c. Hypomagnesemia
a. 110 to 130 calories per kg. d. Seizure
b. 30 to 40 calories per lb of body weight.
c. At least 2 ml per feeding 92. Cammile with sickle cell anemia has an increased
d. 90 to 100 calories per kg risk for having a sickle cell crisis during
pregnancy. Aggressive management of a sickle
86. Nurse John is knowledgeable that usually cell crisis includes which of the following
individual twins will grow appropriately and at measures?
the same rate as singletons until how many a. Antihypertensive agents
weeks? b. Diuretic agents
a. 16 to 18 weeks c. I.V. fluids
b. 18 to 22 weeks d. Acetaminophen (Tylenol) for pain
c. 30 to 32 weeks
d. 38 to 40 weeks 93. Which of the following drugs is the antidote for
magnesium toxicity?
87. Which of the following classifications applies to a. Calcium gluconate (Kalcinate)
monozygotic twins for whom the cleavage of the b. Hydralazine (Apresoline)
fertilized ovum occurs more than 13 days after c. Naloxone (Narcan)
fertilization? d. Rho (D) immune globulin (RhoGAM)
a. conjoined twins
b. diamniotic dichorionic twins 94. Marlyn is screened for tuberculosis during her
c. diamniotic monochorionic twin first prenatal visit. An intradermal injection of
d. monoamniotic monochorionic twins purified protein derivative (PPD) of the
tuberculin bacilli is given. She is considered to
88. Tyra experienced painless vaginal bleeding has have a positive test for which of the following
just been diagnosed as having a placenta previa. results?
Which of the following procedures is usually a. An indurated wheal under 10 mm in
performed to diagnose placenta previa? diameter appears in 6 to 12 hours.
a. Amniocentesis b. An indurated wheal over 10 mm in
b. Digital or speculum examination diameter appears in 48 to 72 hours.
c. External fetal monitoring c. A flat circumcised area under 10 mm in
d. Ultrasound diameter appears in 6 to 12 hours.
d. A flat circumcised area over 10 mm in
89. Nurse Arnold knows that the following changes diameter appears in 48 to 72 hours.
in respiratory functioning during pregnancy is
considered normal: 95. Dianne, 24 year-old is 27 weeks’ pregnant
a. Increased tidal volume arrives at her physician’s office with complaints
b. Increased expiratory volume of fever, nausea, vomiting, malaise, unilateral

8
8
flank pain, and costovertebral angle tenderness. a. Uterine inversion
Which of the following diagnoses is most likely? b. Uterine atony
a. Asymptomatic bacteriuria c. Uterine involution
b. Bacterial vaginosis d. Uterine discomfort
c. Pyelonephritis
d. Urinary tract infection (UTI)

96. Rh isoimmunization in a pregnant client


develops during which of the following
conditions?
a. Rh-positive maternal blood crosses into
fetal blood, stimulating fetal antibodies.
b. Rh-positive fetal blood crosses into
maternal blood, stimulating maternal
antibodies.
c. Rh-negative fetal blood crosses into
maternal blood, stimulating maternal
antibodies.
d. Rh-negative maternal blood crosses into
fetal blood, stimulating fetal antibodies.

97. To promote comfort during labor, the nurse John


advises a client to assume certain positions and
avoid others. Which position may cause
maternal hypotension and fetal hypoxia?
a. Lateral position
b. Squatting position
c. Supine position
d. Standing position

98. Celeste who used heroin during her pregnancy


delivers a neonate. When assessing the neonate,
the nurse Lhynnette expects to find:
a. Lethargy 2 days after birth.
b. Irritability and poor sucking.
c. A flattened nose, small eyes, and thin
lips.
d. Congenital defects such as limb
anomalies.

99. The uterus returns to the pelvic cavity in which


of the following time frames?
a. 7th to 9th day postpartum.
b. 2 weeks postpartum.
c. End of 6th week postpartum.
d. When the lochia changes to alba.

100. Maureen, a primigravida client, age 20, has


just completed a difficult, forceps-assisted
delivery of twins. Her labor was unusually
long and required oxytocin (Pitocin)
augmentation. The nurse who's caring for her
should stay alert for:
hyperstimulation of the uterus, which could
result in injury to the mother and the fetus if
Answers and Rationale – Community Health Pitocin is not discontinued.
Nursing and Care of the Mother and Child 9. Answer: (C) EKG tracings
Rationale: A potential side effect of calcium
1. Answer: (A) Inevitable gluconate administration is cardiac arrest.
Rationale: An inevitable abortion is termination Continuous monitoring of cardiac activity (EKG)
of pregnancy that cannot be prevented. throught administration of calcium gluconate is
Moderate to severe bleeding with mild an essential part of care.
cramping and cervical dilation would be noted 10. Answer: (D) First low transverse caesarean was
in this type of abortion. for breech position. Fetus in this pregnancy is in
2. Answer: (B) History of syphilis a vertex presentation.
Rationale: Maternal infections such as syphilis, Rationale: This type of client has no obstetrical
toxoplasmosis, and rubella are causes of indication for a caesarean section as she did
spontaneous abortion. with her first caesarean delivery.
3. Answer: (C) Monitoring apical pulse 11. Answer: (A) Talk to the mother first and then to
Rationale: Nursing care for the client with a the toddler.
possible ectopic pregnancy is focused on Rationale: When dealing with a crying toddler,
preventing or identifying hypovolemic shock the best approach is to talk to the mother and
and controlling pain. An elevated pulse rate is ignore the toddler first. This approach helps the
an indicator of shock. toddler get used to the nurse before she
4. Answer: (B) Increased caloric intake attempts any procedures. It also gives the
Rationale: Glucose crosses the placenta, but toddler an opportunity to see that the mother
insulin does not. High fetal demands for trusts the nurse.
glucose, combined with the insulin resistance 12. Answer: (D) Place the infant’s arms in soft
caused by hormonal changes in the last half of elbow restraints.
pregnancy can result in elevation of maternal Rationale: Soft restraints from the upper arm to
blood glucose levels. This increases the the wrist prevent the infant from touching her
mother’s demand for insulin and is referred to lip but allow him to hold a favorite item such as
as the diabetogenic effect of pregnancy. a blanket. Because they could damage the
5. Answer: (A) Excessive fetal activity. operative site, such as objects as pacifiers,
Rationale: The most common signs and suction catheters, and small spoons shouldn’t
symptoms of hydatidiform mole includes be placed in a baby’s mouth after cleft repair. A
elevated levels of human chorionic baby in a prone position may rub her face on
gonadotropin, vaginal bleeding, larger than the sheets and traumatize the operative site.
normal uterus for gestational age, failure to The suture line should be cleaned gently to
detect fetal heart activity even with sensitive prevent infection, which could interfere with
instruments, excessive nausea and vomiting, healing and damage the cosmetic appearance
and early development of pregnancy-induced of the repair.
hypertension. Fetal activity would not be noted. 13. Answer: (B) Allow the infant to rest before
6. Answer: (B) Absent patellar reflexes feeding.
Rationale: Absence of patellar reflexes is an Rationale: Because feeding requires so much
indicator of hypermagnesemia, which requires energy, an infant with heart failure should rest
administration of calcium gluconate. before feeding.
7. Answer: (C) Presenting part in 2 cm below the 14. Answer: (C) Iron-rich formula only.
plane of the ischial spines. Rationale: The infants at age 5 months should
Rationale: Fetus at station plus two indicates receive iron-rich formula and that they
that the presenting part is 2 cm below the shouldn’t receive solid food, even baby food
plane of the ischial spines. until age 6 months.
8. Answer: (A) Contractions every 1 ½ minutes 15. Answer: (D) 10 months
lasting 70-80 seconds. Rationale: A 10 month old infant can sit alone
Rationale: Contractions every 1 ½ minutes and understands object permanence, so he
lasting 70-80 seconds, is indicative of would look for the hidden toy. At age 4 to 6

8
8
months, infants can’t sit securely alone. At age Rationale: Intrauterine fetal death, abruptio
8 months, infants can sit securely alone but placentae, septic shock, and amniotic fluid
cannot understand the permanence of objects. embolism may trigger normal clotting
16. Answer: (D) Public health nursing focuses on mechanisms; if clotting factors are depleted,
preventive, not curative, services. DIC may occur. Placenta accreta, dysfunctional
Rationale: The catchments area in PHN consists labor, and premature rupture of the
of a residential community, many of whom are membranes aren't associated with DIC.
well individuals who have greater need for 27. Answer: (C) 120 to 160 beats/minute
preventive rather than curative services. Rationale: A rate of 120 to 160 beats/minute in
17. Answer: (B) Efficiency the fetal heart appropriate for filling the heart
Rationale: Efficiency is determining whether the with blood and pumping it out to the system.
goals were attained at the least possible cost. 28. Answer: (A) Change the diaper more often.
18. Answer: (D) Rural Health Unit Rationale: Decreasing the amount of time the
Rationale: R.A. 7160 devolved basic health skin comes contact with wet soiled diapers will
services to local government units (LGU’s ). The help heal the irritation.
public health nurse is an employee of the LGU. 29. Answer: (D) Endocardial cushion defect
19. Answer: (A) Mayor Rationale: Endocardial cushion defects are seen
Rationale: The local executive serves as the most in children with Down syndrome,
chairman of the Municipal Health Board. asplenia, or polysplenia.
20. Answer: (A) 1 30. Answer: (B) Decreased urine output
Rationale: Each rural health midwife is given a Rationale: Decreased urine output may occur in
population assignment of about 5,000. clients receiving I.V. magnesium and should be
21. Answer: (B) Health education and community monitored closely to keep urine output at
organizing are necessary in providing greater than 30 ml/hour, because magnesium is
community health services. Rationale: The excreted through the kidneys and can easily
community health nurse develops the health accumulate to toxic levels.
capability of people through health education 31. Answer: (A) Menorrhagia
and community organizing activities. Rationale: Menorrhagia is an excessive
22. Answer: (B) Measles menstrual period.
Rationale: Presidential Proclamation No. 4 is on 32. Answer: (C) Blood typing
the Ligtas Tigdas Program. Rationale: Blood type would be a critical value
23. Answer: (D) Core group formation to have because the risk of blood loss is always
Rationale: In core group formation, the nurse is a potential complication during the labor and
able to transfer the technology of community delivery process. Approximately 40% of a
organizing to the potential or informal woman’s cardiac output is delivered to the
community leaders through a training program. uterus, therefore, blood loss can occur quite
24. Answer: (D) To maximize the community’s rapidly in the event of uncontrolled bleeding.
resources in dealing with health problems. 33. Answer: (D) Physiologic anemia
Rationale: Community organizing is a Rationale: Hemoglobin values and hematocrit
developmental service, with the goal of decrease during pregnancy as the increase in
developing the people’s self-reliance in dealing plasma volume exceeds the increase in red
with community health problems. A, B and C blood cell production.
are objectives of contributory objectives to this 34. Answer: (D) A 2 year old infant with stridorous
goal. breath sounds, sitting up in his mother’s arms
25. Answer: (D) Terminal and drooling.
Rationale: Tertiary prevention involves Rationale: The infant with the airway
rehabilitation, prevention of permanent emergency should be treated first, because of
disability and disability limitations appropriate the risk of epiglottitis.
for convalescents, the disabled, complicated 35. Answer: (A) Placenta previa
cases and the terminally ill (those in the Rationale: Placenta previa with painless vaginal
terminal stage of a disease). bleeding.
26. Answer: (A) Intrauterine fetal death. 36. Answer: (D) Early in the morning
Rationale: Based on the nurse’s knowledge of nurse should withhold the potassium and notify
microbiology, the specimen should be collected the physician.
early in the morning. The rationale for this 44. Answer: (c) Laundry detergent
timing is that, because the female worm lays Rationale: Eczema or dermatitis is an allergic
eggs at night around the perineal area, the first skin reaction caused by an offending allergen.
bowel movement of the day will yield the best The topical allergen that is the most common
results. The specific type of stool specimen causative factor is laundry detergent.
used in the diagnosis of pinworms is called the 45. Answer: (A) 6 inches
tape test. Rationale: This distance allows for easy flow of
37. Answer: (A) Irritability and seizures the formula by gravity, but the flow will be slow
Rationale: Lead poisoning primarily affects the enough not to overload the stomach too
CNS, causing increased intracranial pressure. rapidly.
This condition results in irritability and changes 46. Answer: (A) The older one gets, the more
in level of consciousness, as well as seizure susceptible he becomes to the complications of
disorders, hyperactivity, and learning chicken pox.
disabilities. Rationale: Chicken pox is usually more severe in
38. Answer: (D) “I really need to use the adults than in children. Complications, such as
diaphragm and jelly most during the middle of pneumonia, are higher in incidence in adults.
my 47. Answer: (D) Consult a physician who may
menstrual cycle”. give them rubella immunoglobulin.
Rationale: The woman must understand that, Rationale: Rubella vaccine is made up of
although the “fertile” period is approximately attenuated German measles viruses. This is
mid-cycle, hormonal variations do occur and contraindicated in pregnancy. Immune globulin,
can result in early or late ovulation. To be a specific prophylactic against German measles,
effective, the diaphragm should be inserted may be given to pregnant women.
before every intercourse. 48. Answer: (A) Contact tracing
39. Answer: (C) Restlessness Rationale: Contact tracing is the most practical
Rationale: In a child, restlessness is the earliest and reliable method of finding possible sources
sign of hypoxia. Late signs of hypoxia in a child of person-to-person transmitted infections,
are associated with a change in color, such as such as sexually transmitted diseases.
pallor or cyanosis. 49. Answer: (D) Leptospirosis
40. Answer: (B) Walk one step ahead, with the Rationale: Leptospirosis is transmitted through
child’s hand on the nurse’s elbow. contact with the skin or mucous membrane
Rationale: This procedure is generally with water or moist soil contaminated with
recommended to follow in guiding a person urine of infected animals, like rats.
who is blind. 50. Answer: (B) Cholera
41. Answer: (A) Loud, machinery-like murmur. Rationale: Passage of profuse watery stools is
Rationale: A loud, machinery-like murmur is a the major symptom of cholera. Both amebic
characteristic finding associated with patent and bacillary dysentery are characterized by the
ductus arteriosus. presence of blood and/or mucus in the stools.
42. Answer: (C) More oxygen, and the newborn’s Giardiasis is characterized by fat malabsorption
metabolic rate increases. and, therefore, steatorrhea.
Rationale: When cold, the infant requires more 51. Answer: (A) Hemophilus influenzae
oxygen and there is an increase in metabolic Rationale: Hemophilus meningitis is unusual
rate. Non-shievering thermogenesis is a over the age of 5 years. In developing countries,
complex process that increases the metabolic the peak incidence is in children less than 6
rate and rate of oxygen consumption, months of age. Morbillivirus is the etiology of
therefore, the newborn increase heat measles. Streptococcus pneumonia and
production. Neisseria meningitidis may cause meningitis,
43. Answer: (D) Voided but age distribution is not specific in young
Rationale: Before administering potassium I.V. children.
to any client, the nurse must first check that the 52. Answer: (B) Buccal mucosa
client’s kidneys are functioning and that the
client is voiding. If the client is not voiding, the
8
8
Rationale: Koplik’s spot may be seen on the Rationale: A sick child aged 2 months to 5 years
mucosa of the mouth or the throat. must be referred urgently to a hospital if
53. Answer: (A) 3 seconds he/she has one or more of the following signs:
Rationale: Adequate blood supply to the area not able to feed or drink, vomits everything,
allows the return of the color of the nailbed convulsions, abnormally sleepy or difficult to
within 3 seconds. awaken.
54. Answer: (B) Severe dehydration 62. Answer: (A) Refer the child urgently to a
Rationale: The order of priority in the hospital for confinement.
management of severe dehydration is as Rationale: “Baggy pants” is a sign of severe
follows: intravenous fluid therapy, referral to a marasmus. The best management is urgent
facility where IV fluids can be initiated within 30 referral to a hospital.
minutes, Oresol or nasogastric tube. When the 63. Answer: (D) Let the child rest for 10 minutes
foregoing measures are not possible or then continue giving Oresol more slowly.
effective, then urgent referral to the hospital is Rationale: If the child vomits persistently, that
done. is, he vomits everything that he takes in, he has
55. Answer: (A) 45 infants to be referred urgently to a hospital. Otherwise,
Rationale: To estimate the number of infants, vomiting is managed by letting the child rest for
multiply total population by 3%. 10 minutes and then continuing with Oresol
56. Answer: (A) DPT administration. Teach the mother to give Oresol
Rationale: DPT is sensitive to freezing. The more slowly.
appropriate storage temperature of DPT is 2 to 64. Answer: (B) Some dehydration
8° C only. OPV and measles vaccine are highly Rationale: Using the assessment guidelines of
sensitive to heat and require freezing. MMR is IMCI, a child (2 months to 5 years old) with
not an immunization in the Expanded Program diarrhea is classified as having SOME
on Immunization. DEHYDRATION if he shows 2 or more of the
57. Answer: (C) Proper use of sanitary toilets following signs: restless or irritable, sunken
Rationale: The ova of the parasite get out of the eyes, the skin goes back slow after a skin pinch.
human body together with feces. Cutting the 65. Answer: (C) Normal
cycle at this stage is the most effective way of Rationale: In IMCI, a respiratory rate of
preventing the spread of the disease to 50/minute or more is fast breathing for an
susceptible hosts. infant aged 2 to 12 months.
58. Answer: (D) 5 skin lesions, positive slit skin 66. Answer: (A) 1 year
smear Rationale: The baby will have passive natural
Rationale: A multibacillary leprosy case is one immunity by placental transfer of antibodies.
who has a positive slit skin smear and at least 5 The mother will have active artificial immunity
skin lesions. lasting for about 10 years. 5 doses will give the
59. Answer: (C) Thickened painful nerves mother lifetime protection.
Rationale: The lesion of leprosy is not macular. 67. Answer: (B) 4 hours
It is characterized by a change in skin color Rationale: While the unused portion of other
(either reddish or whitish) and loss of sensation, biologicals in EPI may be given until the end of
sweating and hair growth over the lesion. the day, only BCG is discarded 4 hours after
Inability to close the eyelids (lagophthalmos) reconstitution. This is why BCG immunization is
and sinking of the nosebridge are late scheduled only in the morning.
symptoms. 68. Answer: (B) 6 months
60. Answer: (B) Ask where the family resides. Rationale: After 6 months, the baby’s nutrient
Rationale: Because malaria is endemic, the first needs, especially the baby’s iron requirement,
question to determine malaria risk is where the can no longer be provided by mother’s milk
client’s family resides. If the area of residence is alone.
not a known endemic area, ask if the child had 69. Answer: (C) 24 weeks
traveled within the past 6 months, where she Rationale: At approximately 23 to 24 weeks’
was brought and whether she stayed overnight gestation, the lungs are developed enough to
in that area. sometimes maintain extrauterine life. The lungs
61. Answer: (A) Inability to drink are the most immature system during the
gestation period. Medical care for premature Infants aren’t given tub bath but are sponged
labor begins much earlier (aggressively at 21 off until the cord falls off. Petroleum jelly
weeks’ gestation) prevents the cord from drying and encourages
70. Answer: (B) Sudden infant death syndrome infection. Peroxide could be painful and isn’t
(SIDS) recommended.
Rationale: Supine positioning is recommended 77. Answer: (B) Conjunctival hemorrhage
to reduce the risk of SIDS in infancy. The risk of Rationale: Conjunctival hemorrhages are
aspiration is slightly increased with the supine commonly seen in neonates secondary to the
position. Suffocation would be less likely with cranial pressure applied during the birth
an infant supine than prone and the position process. Bulging fontanelles are a sign of
for GER requires the head of the bed to be intracranial pressure. Simian creases are
elevated. present in 40% of the neonates with trisomy 21.
71. Answer: (C) Decreased temperature Cystic hygroma is a neck mass that can affect
Rationale: Temperature instability, especially the airway.
when it results in a low temperature in the 78. Answer: (B) To assess for prolapsed cord
neonate, may be a sign of infection. The Rationale: After a client has an amniotomy, the
neonate’s color often changes with an infection nurse should assure that the cord isn't
process but generally becomes ashen or prolapsed and that the baby tolerated the
mottled. The neonate with an infection will procedure well. The most effective way to do
usually show a decrease in activity level or this is to check the fetal heart rate. Fetal well-
lethargy. being is assessed via a nonstress test. Fetal
72. Answer: (D) Polycythemia probably due to position is determined by vaginal examination.
chronic fetal hypoxia Artificial rupture of membranes doesn't
Rationale: The small-for-gestation neonate is at indicate an imminent delivery.
risk for developing polycythemia during the 79. Answer: (D) The parents’ interactions with each
transitional period in an attempt to decrease other.
hypoxia. The neonates are also at increased risk Rationale: Parental interaction will provide the
for developing hypoglycemia and hypothermia nurse with a good assessment of the stability of
due to decreased glycogen stores. the family's home life but it has no indication
73. Answer: (C) Desquamation of the epidermis for parental bonding. Willingness to touch and
Rationale: Postdate fetuses lose the vernix hold the newborn, expressing interest about
caseosa, and the epidermis may become the newborn's size, and indicating a desire to
desquamated. These neonates are usually very see the newborn are behaviors indicating
alert. Lanugo is missing in the postdate parental bonding.
neonate. 80. Answer: (B) Instructing the client to use two or
74. Answer: (C) Respiratory depression more peripads to cushion the area
Rationale: Magnesium sulfate crosses the Rationale: Using two or more peripads would
placenta and adverse neonatal effects are do little to reduce the pain or promote perineal
respiratory depression, hypotonia, and healing. Cold applications, sitz baths, and Kegel
bradycardia. The serum blood sugar isn’t exercises are important measures when the
affected by magnesium sulfate. The neonate client has a fourth-degree laceration.
would be floppy, not jittery. 81. Answer: (C) “What is your expected due date?”
75. Answer: (C) Respiratory rate 40 to 60 Rationale: When obtaining the history of a
breaths/minute client who may be in labor, the nurse's highest
Rationale: A respiratory rate 40 to 60 priority is to determine her current status,
breaths/minute is normal for a neonate during particularly her due date, gravidity, and parity.
the transitional period. Nasal flaring, Gravidity and parity affect the duration of labor
respiratory rate more than 60 breaths/minute, and the potential for labor complications. Later,
and audible grunting are signs of respiratory the nurse should ask about chronic illnesses,
distress. allergies, and support persons.
76. Answer: (C) Keep the cord dry and open to air 82. Answer: (D) Aspirate the neonate’s nose and
Rationale: Keeping the cord dry and open to air mouth with a bulb syringe.
helps reduce infection and hastens drying.

8
8
Rationale: The nurse's first action should be to Rationale: The type of placenta that develops in
clear the neonate's airway with a bulb syringe. monozygotic twins depends on the time at
After the airway is clear and the neonate's color which cleavage of the ovum occurs. Cleavage in
improves, the nurse should comfort and calm conjoined twins occurs more than 13 days after
the neonate. If the problem recurs or the fertilization. Cleavage that occurs less than 3
neonate's color doesn't improve readily, the day after fertilization results in diamniotic
nurse should notify the physician. dicchorionic twins. Cleavage that occurs
Administering oxygen when the airway isn't between days 3 and 8 results in diamniotic
clear would be ineffective. monochorionic twins. Cleavage that occurs
83. Answer: (C) Conducting a bedside ultrasound between days 8 to 13 result in monoamniotic
for an amniotic fluid index. monochorionic twins.
Rationale: It isn't within a nurse's scope of 88. Answer: (D) Ultrasound
practice to perform and interpret a bedside Rationale: Once the mother and the fetus are
ultrasound under these conditions and without stabilized, ultrasound evaluation of the
specialized training. Observing for pooling of placenta should be done to determine the
straw-colored fluid, checking vaginal discharge cause of the bleeding. Amniocentesis is
with nitrazine paper, and observing for flakes of contraindicated in placenta previa. A digital or
vernix are appropriate assessments for speculum examination shouldn’t be done as
determining whether a client has ruptured this may lead to severe bleeding or
membranes. hemorrhage. External fetal monitoring won’t
84. Answer: (C) Monitor partial pressure of oxygen detect a placenta previa, although it will detect
(Pao2) levels. fetal distress, which may result from blood loss
Rationale: Monitoring PaO2 levels and reducing or placenta separation.
the oxygen concentration to keep PaO2 within 89. Answer: (A) Increased tidal volume
normal limits reduces the risk of retinopathy of Rationale: A pregnant client breathes deeper,
prematurity in a premature infant receiving which increases the tidal volume of gas moved
oxygen. Covering the infant's eyes and in and out of the respiratory tract with each
humidifying the oxygen don't reduce the risk of breath. The expiratory volume and residual
retinopathy of prematurity. Because cooling volume decrease as the pregnancy progresses.
increases the risk of acidosis, the infant should The inspiratory capacity increases during
be kept warm so that his respiratory distress pregnancy. The increased oxygen consumption
isn't aggravated. in the pregnant client is 15% to 20% greater
85. Answer: (A) 110 to 130 calories per kg. than in the nonpregnant state.
Rationale: Calories per kg is the accepted way 90. Answer: (A) Diet
of determined appropriate nutritional intake Rationale: Clients with gestational diabetes are
for a newborn. The recommended calorie usually managed by diet alone to control their
requirement is 110 to 130 calories per kg of glucose intolerance. Oral hypoglycemic drugs
newborn body weight. This level will maintain a are contraindicated in pregnancy. Long-acting
consistent blood glucose level and provide insulin usually isn’t needed for blood glucose
enough calories for continued growth and control in the client with gestational diabetes.
development. 91. Answer: (D) Seizure
86. Answer: (C) 30 to 32 weeks Rationale: The anticonvulsant mechanism of
Rationale: Individual twins usually grow at the magnesium is believes to depress seizure foci in
same rate as singletons until 30 to 32 weeks’ the brain and peripheral neuromuscular
gestation, then twins don’t’ gain weight as blockade. Hypomagnesemia isn’t a
rapidly as singletons of the same gestational complication of preeclampsia. Antihypertensive
age. The placenta can no longer keep pace with drug other than magnesium are preferred for
the nutritional requirements of both fetuses sustained hypertension. Magnesium doesn’t
after 32 weeks, so there’s some growth help prevent hemorrhage in preeclamptic
retardation in twins if they remain in utero at clients.
38 to 40 weeks. 92. Answer: (C) I.V. fluids
87. Answer: (A) conjoined twins Rationale: A sickle cell crisis during pregnancy is
usually managed by exchange transfusion
oxygen, and L.V. Fluids. The client usually needs position promotes comfort by taking advantage
a stronger analgesic than acetaminophen to of gravity. The standing position also takes
control the pain of a crisis. Antihypertensive advantage of gravity and aligns the fetus with
drugs usually aren’t necessary. Diuretic the pelvic angle.
wouldn’t be used unless fluid overload resulted. 98. Answer: (B) Irritability and poor sucking.
93. Answer: (A) Calcium gluconate (Kalcinate) Rationale: Neonates of heroin-addicted
Rationale: Calcium gluconate is the antidote for mothers are physically dependent on the drug
magnesium toxicity. Ten milliliters of 10% and experience withdrawal when the drug is no
calcium gluconate is given L.V. push over 3 to 5 longer supplied. Signs of heroin withdrawal
minutes. Hydralazine is given for sustained include irritability, poor sucking, and
elevated blood pressure in preeclamptic clients. restlessness. Lethargy isn't associated with
Rho (D) immune globulin is given to women neonatal heroin addiction. A flattened nose,
with Rh-negative blood to prevent antibody small eyes, and thin lips are seen in infants with
formation from RH-positive conceptions. fetal alcohol syndrome. Heroin use during
Naloxone is used to correct narcotic toxicity. pregnancy hasn't been linked to specific
94. Answer: (B) An indurated wheal over 10 mm in congenital anomalies.
diameter appears in 48 to 72 hours. 99. Answer: (A) 7th to 9th day postpartum
Rationale: A positive PPD result would be an Rationale: The normal involutional process
indurated wheal over 10 mm in diameter that returns the uterus to the pelvic cavity in 7 to 9
appears in 48 to 72 hours. The area must be a days. A significant involutional complication is
raised wheal, not a flat circumcised area to be the failure of the uterus to return to the pelvic
considered positive. cavity within the prescribed time period. This is
95. Answer: (C) Pyelonephritis known as subinvolution.
Rationale The symptoms indicate acute 100. Answer: (B) Uterine atony
pyelonephritis, a serious condition in a Rationale: Multiple fetuses, extended labor
pregnant client. UTI symptoms include dysuria, stimulation with oxytocin, and traumatic
urgency, frequency, and suprapubic delivery commonly are associated with uterine
tenderness. Asymptomatic bacteriuria doesn’t atony, which may lead to postpartum
cause symptoms. Bacterial vaginosis causes hemorrhage. Uterine inversion may precede or
milky white vaginal discharge but no systemic follow delivery and commonly results from
symptoms. apparent excessive traction on the umbilical
96. Answer: (B) Rh-positive fetal blood crosses into cord and attempts to deliver the placenta
maternal blood, stimulating maternal manually. Uterine involution and some uterine
antibodies. discomfort are normal after delivery.
Rationale: Rh isoimmunization occurs when Rh-
positive fetal blood cells cross into the maternal
circulation and stimulate maternal antibody
production. In subsequent pregnancies with Rh-
positive fetuses, maternal antibodies may cross
back into the fetal circulation and destroy the
fetal blood cells.
97. Answer: (C) Supine position
Rationale: The supine position causes
compression of the client's aorta and inferior
vena cava by the fetus. This, in turn, inhibits
maternal circulation, leading to maternal
hypotension and, ultimately, fetal hypoxia. The
other positions promote comfort and aid labor
progress. For instance, the lateral, or side-lying,
position improves maternal and fetal
circulation, enhances comfort, increases
maternal relaxation, reduces muscle tension,
and eliminates pressure points. The squatting

9
9
TEST III - Care of Clients with Physiologic and 6. Nurse Monett is caring for a client recovering
Psychosocial Alterations from gastro-intestinal bleeding. The nurse
should:
1. Nurse Michelle should know that the drainage is a. Plan care so the client can receive 8
normal 4 days after a sigmoid colostomy when hours of uninterrupted sleep each night.
the stool is: b. Monitor vital signs every 2 hours.
a. Green liquid c. Make sure that the client takes food and
b. Solid formed medications at prescribed intervals.
c. Loose, bloody d. Provide milk every 2 to 3 hours.
d. Semiformed
7. A male client was on warfarin (Coumadin) before
2. Where would nurse Kristine place the call light admission, and has been receiving heparin I.V.
for a male client with a right-sided brain attack for 2 days. The partial thromboplastin time (PTT)
and left homonymous hemianopsia? is 68 seconds. What should Nurse Carla do?
a. On the client’s right side a. Stop the I.V. infusion of heparin and
b. On the client’s left side notify the physician.
c. Directly in front of the client b. Continue treatment as ordered.
d. Where the client like c. Expect the warfarin to increase the PTT.
d. Increase the dosage, because the level is
3. A male client is admitted to the emergency lower than normal.
department following an accident. What are the
first nursing actions of the nurse? 8. A client undergone ileostomy, when should the
a. Check respiration, circulation, drainage appliance be applied to the stoma?
neurological response. a. 24 hours later, when edema has
b. Align the spine, check pupils, and check subsided.
for hemorrhage. b. In the operating room.
c. Check respirations, stabilize spine, and c. After the ileostomy begin to function.
check circulation. d. When the client is able to begin self-care
d. Assess level of consciousness and procedures.
circulation.
9. A client undergone spinal anesthetic, it will be
4. In evaluating the effect of nitroglycerin, Nurse important that the nurse immediately position
Arthur should know that it reduces preload and the client in:
relieves angina by: a. On the side, to prevent obstruction of
a. Increasing contractility and slowing airway by tongue.
heart rate. b. Flat on back.
b. Increasing AV conduction and heart rate. c. On the back, with knees flexed 15
c. Decreasing contractility and oxygen degrees.
consumption. d. Flat on the stomach, with the head
d. Decreasing venous return through turned to the side.
vasodilation.
10. While monitoring a male client several hours
5. Nurse Patricia finds a female client who is post- after a motor vehicle accident, which
myocardial infarction (MI) slumped on the side assessment data suggest increasing intracranial
rails of the bed and unresponsive to shaking or pressure?
shouting. Which is the nurse next action? a. Blood pressure is decreased from
a. Call for help and note the time. 160/90 to 110/70.
b. Clear the airway b. Pulse is increased from 87 to 95, with an
c. Give two sharp thumps to the occasional skipped beat.
precordium, and check the pulse. c. The client is oriented when aroused
d. Administer two quick blows. from sleep, and goes back to sleep
immediately.
d. The client refuses dinner because of 16. Nurse John is caring for a male client receiving
anorexia. lidocaine I.V. Which factor is the most relevant
to administration of this medication?
11. Mrs. Cruz, 80 years old is diagnosed with a. Decrease in arterial oxygen saturation
pneumonia. Which of the following symptoms (SaO2) when measured with a pulse
may appear first? oximeter.
a. Altered mental status and dehydration b. Increase in systemic blood pressure.
b. Fever and chills c. Presence of premature ventricular
c. Hemoptysis and Dyspnea contractions (PVCs) on a cardiac
d. Pleuritic chest pain and cough monitor.
d. Increase in intracranial pressure (ICP).
12. A male client has active tuberculosis (TB). Which
of the following symptoms will be exhibit? 17. Nurse Ron is caring for a male client taking an
a. Chest and lower back pain anticoagulant. The nurse should teach the client
b. Chills, fever, night sweats, and to:
hemoptysis a. Report incidents of diarrhea.
c. Fever of more than 104°F (40°C) and b. Avoid foods high in vitamin K
nausea c. Use a straight razor when shaving.
d. Headache and photophobia d. Take aspirin to pain relief.

13. Mark, a 7-year-old client is brought to the 18. Nurse Lhynnette is preparing a site for the
emergency department. He’s tachypneic and insertion of an I.V. catheter. The nurse should
afebrile and has a respiratory rate of 36 treat excess hair at the site by:
breaths/minute and has a nonproductive cough. a. Leaving the hair intact
He recently had a cold. Form this history; the b. Shaving the area
client may have which of the following c. Clipping the hair in the area
conditions? d. Removing the hair with a depilatory.
a. Acute asthma
b. Bronchial pneumonia 19. Nurse Michelle is caring for an elderly female
c. Chronic obstructive pulmonary disease with osteoporosis. When teaching the client, the
(COPD) nurse should include information about which
d. Emphysema major complication:
a. Bone fracture
14. Marichu was given morphine sulfate for pain. b. Loss of estrogen
She is sleeping and her respiratory rate is 4 c. Negative calcium balance
breaths/minute. If action isn’t taken quickly, she d. Dowager’s hump
might have which of the following reactions?
a. Asthma attack 20. Nurse Len is teaching a group of women to
b. Respiratory arrest perform BSE. The nurse should explain that
c. Seizure the purpose of performing the examination is
d. Wake up on his own to discover:
a. Cancerous lumps
15. A 77-year-old male client is admitted for elective b. Areas of thickness or fullness
knee surgery. Physical examination reveals c. Changes from previous examinations.
shallow respirations but no sign of respiratory d. Fibrocystic masses
distress. Which of the following is a normal
physiologic change related to aging? 21. When caring for a female client who is being
a. Increased elastic recoil of the lungs treated for hyperthyroidism, it is important to:
b. Increased number of functional a. Provide extra blankets and clothing to
capillaries in the alveoli keep the client warm.
c. Decreased residual volume b. Monitor the client for signs of
d. Decreased vital capacity restlessness, sweating, and excessive

9
9
weight loss during thyroid replacement During routine assessment, the nurse notices
therapy. Cheyne- Strokes respirations. Cheyne-strokes
c. Balance the client’s periods of activity respirations are:
and rest. a. A progressively deeper breaths followed
d. Encourage the client to be active to by shallower breaths with apneic
prevent constipation. periods.
b. Rapid, deep breathing with abrupt
22. Nurse Kris is teaching a client with history of pauses between each breath.
atherosclerosis. To decrease the risk of c. Rapid, deep breathing and irregular
atherosclerosis, the nurse should encourage the breathing without pauses.
client to: d. Shallow breathing with an increased
a. Avoid focusing on his weight. respiratory rate.
b. Increase his activity level.
c. Follow a regular diet. 28. Nurse Bea is assessing a male client with heart
d. Continue leading a high-stress lifestyle. failure. The breath sounds commonly
auscultated in clients with heart failure are:
23. Nurse Greta is working on a surgical floor. Nurse a. Tracheal
Greta must logroll a client following a: b. Fine crackles
a. Laminectomy c. Coarse crackles
b. Thoracotomy d. Friction rubs
c. Hemorrhoidectomy
d. Cystectomy. 29. The nurse is caring for Kenneth experiencing an
acute asthma attack. The client stops wheezing
24. A 55-year old client underwent cataract removal and breath sounds aren’t audible. The reason for
with intraocular lens implant. Nurse Oliver is this change is that:
giving the client discharge instructions. These a. The attack is over.
instructions should include which of the b. The airways are so swollen that no air
following? cannot get through.
a. Avoid lifting objects weighing more than c. The swelling has decreased.
5 lb (2.25 kg). d. Crackles have replaced wheezes.
b. Lie on your abdomen when in bed
c. Keep rooms brightly lit. 30. Mike with epilepsy is having a seizure. During
d. Avoiding straining during bowel the active seizure phase, the nurse should:
movement or bending at the waist. a. Place the client on his back remove
dangerous objects, and insert a bite
25. George should be taught about testicular block.
examinations during: b. Place the client on his side, remove
a. when sexual activity starts dangerous objects, and insert a bite
b. After age 69 block.
c. After age 40 c. Place the client o his back, remove
d. Before age 20. dangerous objects, and hold down his
26. A male client undergone a colon resection. While arms.
turning him, wound dehiscence with d. Place the client on his side, remove
evisceration occurs. Nurse Trish first response is dangerous objects, and protect his head.
to:
a. Call the physician 31. After insertion of a cheat tube for a
b. Place a saline-soaked sterile dressing on pneumothorax, a client becomes hypotensive
the wound. with neck vein distention, tracheal shift, absent
c. Take a blood pressure and pulse. breath sounds, and diaphoresis. Nurse Amanda
d. Pull the dehiscence closed. suspects a tension pneumothorax has occurred.
What cause of tension pneumothorax should the
27. Nurse Audrey is caring for a client who has nurse check for?
suffered a severe cerebrovascular accident. a. Infection of the lung.
b. Kinked or obstructed chest tube tuberculosis (TB). Which of the following clients
c. Excessive water in the water-seal entering the clinic today most likely to have TB?
chamber a. A 16-year-old female high school
d. Excessive chest tube drainage student
b. A 33-year-old day-care worker
32. Nurse Maureen is talking to a male client; the c. A 43-yesr-old homeless man with a
client begins choking on his lunch. He’s coughing history of alcoholism
forcefully. The nurse should: d. A 54-year-old businessman
a. Stand him up and perform the
abdominal thrust maneuver from 37. Virgie with a positive Mantoux test result will be
behind. sent for a chest X-ray. The nurse is aware that
b. Lay him down, straddle him, and which of the following reasons this is done?
perform the abdominal thrust a. To confirm the diagnosis
maneuver. b. To determine if a repeat skin test is
c. Leave him to get assistance needed
d. Stay with him but not intervene at this c. To determine the extent of lesions
time. d. To determine if this is a primary or
secondary infection
33. Nurse Ron is taking a health history of an 84 year
old client. Which information will be most useful 38. Kennedy with acute asthma showing inspiratory
to the nurse for planning care? and expiratory wheezes and a decreased forced
a. General health for the last 10 years. expiratory volume should be treated with which
b. Current health promotion activities. of the following classes of medication right
c. Family history of diseases. away?
d. Marital status. a. Beta-adrenergic blockers
b. Bronchodilators
34. When performing oral care on a comatose client, c. Inhaled steroids
Nurse Krina should: d. Oral steroids
a. Apply lemon glycerin to the client’s lips
at least every 2 hours. 39. Mr. Vasquez 56-year-old client with a 40-year
b. Brush the teeth with client lying supine. history of smoking one to two packs of cigarettes
c. Place the client in a side lying position, per day has a chronic cough producing thick
with the head of the bed lowered. sputum, peripheral edema and cyanotic nail
d. Clean the client’s mouth with hydrogen beds. Based on this information, he most likely
peroxide. has which of the following conditions?
a. Adult respiratory distress syndrome
35. A 77-year-old male client is admitted with a (ARDS)
diagnosis of dehydration and change in mental b. Asthma
status. He’s being hydrated with L.V. fluids. c. Chronic obstructive bronchitis
When the nurse takes his vital signs, she notes d. Emphysema
he has a fever of 103°F (39.4°C) a cough
producing yellow sputum and pleuritic chest Situation: Francis, age 46 is admitted to the hospital with
pain. The nurse suspects this client may have diagnosis of Chronic Lymphocytic Leukemia.
which of the following conditions?
a. Adult respiratory distress syndrome 40. The treatment for patients with leukemia is bone
(ARDS) marrow transplantation. Which statement about
b. Myocardial infarction (MI) bone marrow transplantation is not correct?
c. Pneumonia a. The patient is under local anesthesia
d. Tuberculosis during the procedure
b. The aspirated bone marrow is mixed
36. Nurse Oliver is working in an outpatient clinic. with heparin.
He has been alerted that there is an outbreak c. The aspiration site is the posterior or
of anterior iliac crest.

9
9
d. The recipient receives pressure of 126/76 mm Hg, and a
cyclophosphamide (Cytoxan) for 4 respiratory rate of 22 breaths/ minute.
consecutive days before the procedure. b. The 89-year-old client with end-stage
41. After several days of admission, Francis becomes right-sided heart failure, blood pressure
disoriented and complains of frequent of 78/50 mm Hg, and a “do not
headaches. The nurse in-charge first action resuscitate” order
would be: c. The 62-year-old client who was admitted
a. Call the physician 1 day ago with thrombophlebitis and is
b. Document the patient’s status in his receiving L.V. heparin
charts. d. The 75-year-old client who was admitted
c. Prepare oxygen treatment 1 hour ago with new-onset atrial
d. Raise the side rails fibrillation and is receiving L.V. dilitiazem
(Cardizem)
42. During routine care, Francis asks the nurse,
“How can I be anemic if this disease causes 46. Honey, a 23-year old client complains of
increased my white blood cell production?” The substernal chest pain and states that her heart
nurse in-charge best response would be that the feels like “it’s racing out of the chest”. She
increased number of white blood cells (WBC) is: reports no history of cardiac disorders. The
a. Crowd red blood cells nurse attaches her to a cardiac monitor and
b. Are not responsible for the anemia. notes sinus tachycardia with a rate of
c. Uses nutrients from other cells 136beats/minutes. Breath sounds are clear and
d. Have an abnormally short life span of the respiratory rate is 26 breaths/minutes.
cells. Which of the following drugs should the nurse
question the client about using?
43. Diagnostic assessment of Francis would probably a. Barbiturates
not reveal: b. Opioids
a. Predominance of lymhoblasts c. Cocaine
b. Leukocytosis d. Benzodiazepines
c. Abnormal blast cells in the bone marrow
d. Elevated thrombocyte counts 47. A 51-year-old female client tells the nurse in-
charge that she has found a painless lump in her
44. Robert, a 57-year-old client with acute arterial right breast during her monthly self-
occlusion of the left leg undergoes an examination. Which assessment finding would
emergency embolectomy. Six hours later, the strongly suggest that this client's lump is
nurse isn’t able to obtain pulses in his left foot cancerous?
using Doppler ultrasound. The nurse a. Eversion of the right nipple and mobile
immediately notifies the physician, and asks her mass
to prepare the client for surgery. As the nurse b. Nonmobile mass with irregular edges
enters the client’s room to prepare him, he c. Mobile mass that is soft and easily
states that he won’t have any more surgery. delineated
Which of the following is the best initial d. Nonpalpable right axillary lymph nodes
response by the nurse?
a. Explain the risks of not having 48. A 35-year-old client with vaginal cancer asks the
the surgery nurse, "What is the usual treatment for this type
b. Notifying the physician immediately of cancer?" Which treatment should the nurse
c. Notifying the nursing supervisor name?
d. Recording the client’s refusal in the a. Surgery
nurses’ notes b. Chemotherapy
c. Radiation
45. During the endorsement, which of the following d. Immunotherapy
clients should the on-duty nurse assess first?
a. The 58-year-old client who was admitted 49. Cristina undergoes a biopsy of a suspicious
2 days ago with heart failure, blood lesion. The biopsy report classifies the lesion
according to the TNM staging system as follows: a. prostate-specific antigen, which is used
TIS, N0, M0. What does this classification mean? to screen for prostate cancer.
a. No evidence of primary tumor, no b. protein serum antigen, which is used to
abnormal regional lymph nodes, and no determine protein levels.
evidence of distant metastasis c. pneumococcal strep antigen, which is a
b. Carcinoma in situ, no abnormal regional bacteria that causes pneumonia.
lymph nodes, and no evidence of distant d. Papanicolaou-specific antigen, which is
metastasis used to screen for cervical cancer.
c. Can't assess tumor or regional lymph
nodes and no evidence of metastasis 54. What is the most important postoperative
d. Carcinoma in situ, no demonstrable instruction that nurse Kate must give a client
metastasis of the regional lymph nodes, who has just returned from the operating room
and ascending degrees of distant after receiving a subarachnoid block?
metastasis a. "Avoid drinking liquids until the gag
reflex returns."
50. Lydia undergoes a laryngectomy to treat b. "Avoid eating milk products for 24
laryngeal cancer. When teaching the client how hours."
to care for the neck stoma, the nurse should c. "Notify a nurse if you experience blood
include which instruction? in your urine."
a. "Keep the stoma uncovered." d. "Remain supine for the time specified by
b. "Keep the stoma dry." the physician."
c. "Have a family member perform stoma
care initially until you get used to the 55. A male client suspected of having colorectal
procedure." cancer will require which diagnostic study to
d. "Keep the stoma moist." confirm the diagnosis?
a. Stool Hematest
51. A 37-year-old client with uterine cancer asks the b. Carcinoembryonic antigen (CEA)
nurse, "Which is the most common type of c. Sigmoidoscopy
cancer in women?" The nurse replies that it's d. Abdominal computed tomography (CT)
breast cancer. Which type of cancer causes the scan
most deaths in women?
a. Breast cancer 56. During a breast examination, which finding most
b. Lung cancer strongly suggests that the Luz has breast cancer?
c. Brain cancer a. Slight asymmetry of the breasts.
d. Colon and rectal cancer b. A fixed nodular mass with dimpling of
the overlying skin
52. Antonio with lung cancer develops Horner's c. Bloody discharge from the nipple
syndrome when the tumor invades the ribs and d. Multiple firm, round, freely movable
affects the sympathetic nerve ganglia. When masses that change with the menstrual
assessing for signs and symptoms of this cycle
syndrome, the nurse should note:
a. miosis, partial eyelid ptosis, and 57. A female client with cancer is being evaluated
anhidrosis on the affected side of the for possible metastasis. Which of the following is
face. one of the most common metastasis sites for
b. chest pain, dyspnea, cough, weight loss, cancer cells?
and fever. a. Liver
c. arm and shoulder pain and atrophy of b. Colon
arm and hand muscles, both on the c. Reproductive tract
affected side. d. White blood cells (WBCs)
d. hoarseness and dysphagia.
58. Nurse Mandy is preparing a client for magnetic
53. Vic asks the nurse what PSA is. The nurse should resonance imaging (MRI) to confirm or rule out a
reply that it stands for:

9
9
spinal cord lesion. During the MRI scan, which of 63. A 76-year-old male client had a thromboembolic
the following would pose a threat to the client? right stroke; his left arm is swollen. Which of the
a. The client lies still. following conditions may cause swelling after a
b. The client asks questions. stroke?
c. The client hears thumping sounds. a. Elbow contracture secondary to
d. The client wears a watch and wedding spasticity
band. b. Loss of muscle contraction decreasing
venous return
59. Nurse Cecile is teaching a female client about c. Deep vein thrombosis (DVT) due to
preventing osteoporosis. Which of the following immobility of the ipsilateral side
teaching points is correct? d. Hypoalbuminemia due to protein
a. Obtaining an X-ray of the bones every 3 escaping from an inflamed glomerulus
years is recommended to detect bone
loss. 64. Heberden’s nodes are a common sign of
b. To avoid fractures, the client should osteoarthritis. Which of the following statement
avoid strenuous exercise. is correct about this deformity?
c. The recommended daily allowance of a. It appears only in men
calcium may be found in a wide variety b. It appears on the distal interphalangeal
of foods. joint
d. Obtaining the recommended daily c. It appears on the proximal
allowance of calcium requires taking a interphalangeal joint
calcium supplement. d. It appears on the dorsolateral aspect of
the interphalangeal joint.
60. Before Jacob undergoes arthroscopy, the nurse
reviews the assessment findings for 65. Which of the following statements explains the
contraindications for this procedure. Which main difference between rheumatoid arthritis
finding is a contraindication? and osteoarthritis?
a. Joint pain a. Osteoarthritis is gender-specific,
b. Joint deformity rheumatoid arthritis isn’t
c. Joint flexion of less than 50% b. Osteoarthritis is a localized disease
d. Joint stiffness rheumatoid arthritis is systemic
c. Osteoarthritis is a systemic disease,
61. Mr. Rodriguez is admitted with severe pain in rheumatoid arthritis is localized
the knees. Which form of arthritis is d. Osteoarthritis has dislocations and
characterized by urate deposits and joint pain, subluxations, rheumatoid arthritis
usually in the feet and legs, and occurs primarily doesn’t
in men over age 30?
a. Septic arthritis 66. Mrs. Cruz uses a cane for assistance in walking.
b. Traumatic arthritis Which of the following statements is true about
c. Intermittent arthritis a cane or other assistive devices?
d. Gouty arthritis a. A walker is a better choice than a cane.
b. The cane should be used on the affected
62. A heparin infusion at 1,500 unit/hour is ordered side
for a 64-year-old client with stroke in evolution. c. The cane should be used on the
The infusion contains 25,000 units of heparin in unaffected side
500 ml of saline solution. How many milliliters d. A client with osteoarthritis should be
per hour should be given? encouraged to ambulate without the
a. 15 ml/hour cane
b. 30 ml/hour
c. 45 ml/hour 67. A male client with type 1 diabetes is scheduled
d. 50 ml/hour to receive 30 U of 70/30 insulin. There is no
70/30 insulin available. As a substitution, the
nurse may give the client:
a. 9 U regular insulin and 21 U neutral c. Restricting fluids
protamine Hagedorn (NPH). d. Administering glucose-containing I.V.
b. 21 U regular insulin and 9 U NPH. fluids as ordered
c. 10 U regular insulin and 20 U NPH.
d. 20 U regular insulin and 10 U NPH. 73. A female client tells nurse Nikki that she has
been working hard for the last 3 months to
68. Nurse Len should expect to administer which control her type 2 diabetes mellitus with diet
medication to a client with gout? and exercise. To determine the effectiveness of
a. aspirin the client's efforts, the nurse should check:
b. furosemide (Lasix) a. urine glucose level.
c. colchicines b. fasting blood glucose level.
d. calcium gluconate (Kalcinate) c. serum fructosamine level.
d. glycosylated hemoglobin level.
69. Mr. Domingo with a history of hypertension is
diagnosed with primary hyperaldosteronism. 74. Nurse Trinity administered neutral protamine
This diagnosis indicates that the client's Hagedorn (NPH) insulin to a diabetic client at 7
hypertension is caused by excessive hormone a.m. At what time would the nurse expect the
secretion from which of the following glands? client to be most at risk for a hypoglycemic
a. Adrenal cortex reaction?
b. Pancreas a. 10:00 am
c. Adrenal medulla b. Noon
d. Parathyroid c. 4:00 pm
d. 10:00 pm
70. For a diabetic male client with a foot ulcer, the
doctor orders bed rest, a wet-to-dry dressing 75. The adrenal cortex is responsible for producing
change every shift, and blood glucose which substances?
monitoring before meals and bedtime. Why are a. Glucocorticoids and androgens
wet-to-dry dressings used for this client? b. Catecholamines and epinephrine
a. They contain exudate and provide a c. Mineralocorticoids and catecholamines
moist wound environment. d. Norepinephrine and epinephrine
b. They protect the wound from
mechanical trauma and promote 76. On the third day after a partial thyroidectomy,
healing. Proserfina exhibits muscle twitching and
c. They debride the wound and promote hyperirritability of the nervous system. When
healing by secondary intention. questioned, the client reports numbness and
d. They prevent the entrance of tingling of the mouth and fingertips. Suspecting
microorganisms and minimize wound a life-threatening electrolyte disturbance, the
discomfort. nurse notifies the surgeon immediately. Which
electrolyte disturbance most commonly follows
71. Nurse Zeny is caring for a client in acute thyroid surgery?
addisonian crisis. Which laboratory data would a. Hypocalcemia
the nurse expect to find? b. Hyponatremia
a. Hyperkalemia c. Hyperkalemia
b. Reduced blood urea nitrogen (BUN) d. Hypermagnesemia
c. Hypernatremia
d. Hyperglycemia 77. Which laboratory test value is elevated in clients
who smoke and can't be used as a general
72. A client is admitted for treatment of the indicator of cancer?
syndrome of inappropriate antidiuretic hormone a. Acid phosphatase level
(SIADH). Which nursing intervention is b. Serum calcitonin level
appropriate? c. Alkaline phosphatase level
a. Infusing I.V. fluids rapidly as ordered d. Carcinoembryonic antigen level
b. Encouraging increased oral intake

9
9
78. Francis with anemia has been admitted to the c. Administer the antidote for penicillin, as
medical-surgical unit. Which assessment findings prescribed, and continue to monitor the
are characteristic of iron-deficiency anemia? client's vital signs.
a. Nights sweats, weight loss, and diarrhea d. Insert an indwelling urinary catheter and
b. Dyspnea, tachycardia, and pallor begin to infuse I.V. fluids as ordered.
c. Nausea, vomiting, and anorexia
d. Itching, rash, and jaundice 83. Mr. Marquez with rheumatoid arthritis is about
to begin aspirin therapy to reduce inflammation.
79. In teaching a female client who is HIV-positive When teaching the client about aspirin, the
about pregnancy, the nurse would know more nurse discusses adverse reactions to prolonged
teaching is necessary when the client says: aspirin therapy. These include:
a. The baby can get the virus from my a. weight gain.
placenta." b. fine motor tremors.
b. "I'm planning on starting on birth control c. respiratory acidosis.
pills." d. bilateral hearing loss.
c. "Not everyone who has the virus gives
birth to a baby who has the virus." 84. A 23-year-old client is diagnosed with human
d. "I'll need to have a C-section if I become immunodeficiency virus (HIV). After recovering
pregnant and have a baby." from the initial shock of the diagnosis, the client
expresses a desire to learn as much as possible
80. When preparing Judy with acquired about HIV and acquired immunodeficiency
immunodeficiency syndrome (AIDS) for syndrome (AIDS). When teaching the client
discharge to the home, the nurse should be sure about the immune system, the nurse states that
to include which instruction? adaptive immunity is provided by which type of
a. "Put on disposable gloves before white blood cell?
bathing." a. Neutrophil
b. "Sterilize all plates and utensils in boiling b. Basophil
water." c. Monocyte
c. "Avoid sharing such articles as d. Lymphocyte
toothbrushes and razors."
d. "Avoid eating foods from serving dishes 85. In an individual with Sjögren's syndrome, nursing
shared by other family members." care should focus on:
a. moisture replacement.
81. Nurse Marie is caring for a 32-year-old client b. electrolyte balance.
admitted with pernicious anemia. Which set of c. nutritional supplementation.
findings should the nurse expect when assessing d. arrhythmia management.
the client?
a. Pallor, bradycardia, and reduced pulse 86. During chemotherapy for lymphocytic leukemia,
pressure Mathew develops abdominal pain, fever, and
b. Pallor, tachycardia, and a sore tongue "horse barn" smelling diarrhea. It would be most
c. Sore tongue, dyspnea, and weight gain important for the nurse to advise the physician
d. Angina, double vision, and anorexia to order:
a. enzyme-linked immunosuppressant
82. After receiving a dose of penicillin, a client assay (ELISA) test.
develops dyspnea and hypotension. Nurse b. electrolyte panel and hemogram.
Celestina suspects the client is experiencing c. stool for Clostridium difficile test.
anaphylactic shock. What should the nurse do d. flat plate X-ray of the abdomen.
first?
a. Page an anesthesiologist immediately 87. A male client seeks medical evaluation for
and prepare to intubate the client. fatigue, night sweats, and a 20-lb weight loss in 6
b. Administer epinephrine, as prescribed, weeks. To confirm that the client has been
and prepare to intubate the client if infected with the human immunodeficiency virus
necessary. (HIV), the nurse expects the physician to order:
a. E-rosette immunofluorescence. d. A client with rheumatoid arthritis who
b. quantification of T-lymphocytes. states, “I am having trouble sleeping.”
c. enzyme-linked immunosorbent assay
(ELISA). 92. Nurse Sarah is caring for clients on the surgical
d. Western blot test with ELISA. floor and has just received report from the
previous shift. Which of the following clients
88. A complete blood count is commonly performed should the nurse see first?
before a Joe goes into surgery. What does this a. A 35-year-old admitted three hours ago
test seek to identify? with a gunshot wound; 1.5 cm area of
a. Potential hepatic dysfunction indicated dark drainage noted on the dressing.
by decreased blood urea nitrogen (BUN) b. A 43-year-old who had a mastectomy
and creatinine levels two days ago; 23 ml of
b. Low levels of urine constituents normally serosanguinous fluid noted in the
excreted in the urine Jackson-Pratt drain.
c. Abnormally low hematocrit (HCT) and c. A 59-year-old with a collapsed lung due
hemoglobin (Hb) levels to an accident; no drainage noted in the
d. Electrolyte imbalance that could affect previous eight hours.
the blood's ability to coagulate properly d. A 62-year-old who had an abdominal-
perineal resection three days ago; client
89. While monitoring a client for the development complaints of chills.
of disseminated intravascular coagulation (DIC),
the nurse should take note of what assessment 93. Nurse Eve is caring for a client who had a
parameters? thyroidectomy 12 hours ago for treatment of
a. Platelet count, prothrombin time, and Grave’s disease. The nurse would be most
partial thromboplastin time concerned if which of the following was
b. Platelet count, blood glucose levels, and observed?
white blood cell (WBC) count a. Blood pressure 138/82, respirations 16,
c. Thrombin time, calcium levels, and oral temperature 99 degrees Fahrenheit.
potassium levels b. The client supports his head and neck
d. Fibrinogen level, WBC, and platelet when turning his head to the right.
count c. The client spontaneously flexes his wrist
when the blood pressure is obtained.
90. When taking a dietary history from a newly d. The client is drowsy and complains of
admitted female client, Nurse Len should sore throat.
remember that which of the following foods is a
common allergen? 94. Julius is admitted with complaints of severe pain
a. Bread in the lower right quadrant of the abdomen. To
b. Carrots assist with pain relief, the nurse should take
c. Orange which of the following actions?
d. Strawberries a. Encourage the client to change positions
frequently in bed.
91. Nurse John is caring for clients in the outpatient b. Administer Demerol 50 mg IM q 4 hours
clinic. Which of the following phone calls should and PRN.
the nurse return first? c. Apply warmth to the abdomen with a
a. A client with hepatitis A who states, “My heating pad.
arms and legs are itching.” d. Use comfort measures and pillows to
b. A client with cast on the right leg who position the client.
states, “I have a funny feeling in my right
leg.” 95. Nurse Tina prepares a client for peritoneal
c. A client with osteomyelitis of the spine dialysis. Which of the following actions should
who states, “I am so nauseous that I the nurse take first?
can’t eat.” a. Assess for a bruit and a thrill.
b. Warm the dialysate solution.

10
1
c. Position the client on the left side.
10
d. Insert a Foley catheter takes small steps while balancing on the
walker.
96. Nurse Jannah teaches an elderly client with d. The client slides the walker 18 inches
right-sided weakness how to use cane. Which of forward, then takes small steps while
the following behaviors, if demonstrated by the holding onto the walker for balance.
client to the nurse, indicates that the teaching
was effective? 99. Nurse Deric is supervising a group of elderly
a. The client holds the cane with his right clients in a residential home setting. The nurse
hand, moves the can forward followed knows that the elderly are at greater risk of
by the right leg, and then moves the left developing sensory deprivation for what reason?
leg. a. Increased sensitivity to the side effects
b. The client holds the cane with his right of medications.
hand, moves the cane forward followed b. Decreased visual, auditory, and
by his left leg, and then moves the right gustatory abilities.
leg. c. Isolation from their families and familiar
c. The client holds the cane with his left surroundings.
hand, moves the cane forward followed d. Decrease musculoskeletal function and
by the right leg, and then moves the left mobility.
leg.
d. The client holds the cane with his left 100. A male client with emphysema becomes
hand, moves the cane forward followed restless and confused. What step should
by his left leg, and then moves the right nurse Jasmine take next?
leg. a. Encourage the client to perform pursed
lip breathing.
97. An elderly client is admitted to the nursing home b. Check the client’s temperature.
setting. The client is occasionally confused and c. Assess the client’s potassium level.
her gait is often unsteady. Which of the d. Increase the client’s oxygen flow rate.
following actions, if taken by the nurse, is most
appropriate?
a. Ask the woman’s family to provide
personal items such as photos or
mementos.
b. Select a room with a bed by the door so
the woman can look down the hall.
c. Suggest the woman eat her meals in the
room with her roommate.
d. Encourage the woman to ambulate in
the halls twice a day.

98. Nurse Evangeline teaches an elderly client how


to use a standard aluminum walker. Which of
the following behaviors, if demonstrated by the
client, indicates that the nurse’s teaching was
effective?
a. The client slowly pushes the walker
forward 12 inches, then takes small
steps forward while leaning on the
walker.
b. The client lifts the walker, moves it
forward 10 inches, and then takes
several small steps forward.
c. The client supports his weight on the
walker while advancing it forward, then
digestive enzymes and highly irritating to the
skin. Protection of the skin from the effects of
Answers and Rationale – Care of Clients with these enzymes is begun at once. Skin exposed
Physiologic and Psychosocial Alterations to these enzymes even for a short time
becomes reddened, painful, and excoriated.
1. Answer: (C) Loose, bloody 9. Answer: (B) Flat on back.
Rationale: Normal bowel function and soft- Rationale: To avoid the complication of a
formed stool usually do not occur until around painful spinal headache that can last for
the seventh day following surgery. The stool several days, the client is kept in flat in a
consistency is related to how much water is supine position for approximately 4 to 12
being absorbed. hours postoperatively. Headaches are
2. Answer: (A) On the client’s right side believed to be causes by the seepage of
Rationale: The client has left visual field cerebral spinal fluid from the puncture site. By
blindness. The client will see only from the keeping the client flat, cerebral spinal fluid
right side. pressures are equalized, which avoids trauma
3. Answer: (C) Check respirations, stabilize spine, to the neurons.
and check circulation 10. Answer: (C) The client is oriented when
Rationale: Checking the airway would be aroused from sleep, and goes back to sleep
priority, and a neck injury should be immediately.
suspected. Rationale: This finding suggest that the level
4. Answer: (D) Decreasing venous return through of consciousness is decreasing.
vasodilation. 11. Answer: (A) Altered mental status and
Rationale: The significant effect of dehydration
nitroglycerin is vasodilation and decreased Rationale: Fever, chills, hemortysis, dyspnea,
venous return, so the heart does not have to cough, and pleuritic chest pain are the
work hard. common symptoms of pneumonia, but elderly
5. Answer: (A) Call for help and note the time. clients may first appear with only an altered
Rationale: Having established, by stimulating lentil status and dehydration due to a blunted
the client, that the client is unconscious rather immune response.
than sleep, the nurse should immediately call 12. Answer: (B) Chills, fever, night sweats, and
for help. This may be done by dialing the hemoptysis
operator from the client’s phone and giving Rationale: Typical signs and symptoms are
the hospital code for cardiac arrest and the chills, fever, night sweats, and hemoptysis.
client’s room number to the operator, of if the Chest pain may be present from coughing, but
phone is not available, by pulling the isn’t usual. Clients with TB typically have low-
emergency call button. Noting the time is grade fevers, not higher than 102°F (38.9°C).
important baseline information for cardiac Nausea, headache, and photophobia aren’t
arrest procedure usual TB symptoms.
6. Answer: (C) Make sure that the client takes 13. Answer:(A) Acute asthma
food and medications at prescribed intervals. Rationale: Based on the client’s history and
Rationale: Food and drug therapy will prevent symptoms, acute asthma is the most likely
the accumulation of hydrochloric acid, or will diagnosis. He’s unlikely to have bronchial
neutralize and buffer the acid that does pneumonia without a productive cough and
accumulate. fever and he’s too young to have developed
7. Answer: (B) Continue treatment as ordered. (COPD) and emphysema.
Rationale: The effects of heparin are 14. Answer: (B) Respiratory arrest
monitored by the PTT is normally 30 to 45 Rationale: Narcotics can cause respiratory
seconds; the therapeutic level is 1.5 to 2 times arrest if given in large quantities. It’s unlikely
the normal level. the client will have asthma attack or a seizure
8. Answer: (B) In the operating room. or wake up on his own.
Rationale: The stoma drainage bag is applied 15. Answer: (D) Decreased vital capacity
in the operating room. Drainage from the Rationale: Reduction in vital capacity is a
ileostomy contains secretions that are rich in normal physiologic change includes decreased

10
10
elastic recoil of the lungs, fewer functional activity and rest. Many clients with
capillaries in the alveoli, and an increased in hyperthyroidism are hyperactive and complain
residual volume. of feeling very warm.
16. Answer: (C) Presence of premature ventricular 22. Answer: (B) Increase his activity level.
contractions (PVCs) on a cardiac monitor. Rationale: The client should be encouraged to
Rationale: Lidocaine drips are commonly used increase his activity level. aintaining an ideal
to treat clients whose arrhythmias haven’t weight; following a low-cholesterol, low
been controlled with oral medication and who sodium diet; and avoiding stress are all
are having PVCs that are visible on the cardiac important factors in decreasing the risk of
monitor. SaO2, blood pressure, and ICP are atherosclerosis.
important factors but aren’t as significant as 23. Answer: (A) Laminectomy
PVCs in the situation. Rationale: The client who has had spinal
17. Answer: (B) Avoid foods high in vitamin K surgery, such as laminectomy, must be log
Rationale: The client should avoid consuming rolled to keep the spinal column straight when
large amounts of vitamin K because vitamin K turning. Thoracotomy and cystectomy may
can interfere with anticoagulation. The client turn themselves or may be assisted into a
may need to report diarrhea, but isn’t effect comfortable position. Under normal
of taking an anticoagulant. An electric razor- circumstances, hemorrhoidectomy is an
not a straight razor-should be used to prevent outpatient procedure, and the client may
cuts that cause bleeding. Aspirin may increase resume normal activities immediately after
the risk of bleeding; acetaminophen should be surgery.
used to pain relief. 24. Answer: (D) Avoiding straining during bowel
18. Answer: (C) Clipping the hair in the area movement or bending at the waist.
Rationale: Hair can be a source of infection Rationale: The client should avoid straining,
and should be removed by clipping. Shaving lifting heavy objects, and coughing harshly
the area can cause skin abrasions and because these activities increase intraocular
depilatories can irritate the skin. pressure. Typically, the client is instructed to
19. Answer: (A) Bone fracture avoid lifting objects weighing more than 15 lb
Rationale: Bone fracture is a major (7kg) – not 5lb. instruct the client when lying
complication of osteoporosis that results in bed to lie on either the side or back. The
when loss of calcium and phosphate increased client should avoid bright light by wearing
the fragility of bones. Estrogen deficiencies sunglasses.
result from menopause-not osteoporosis. 25. Answer: (D) Before age 20.
Calcium and vitamin D supplements may be Rationale: Testicular cancer commonly occurs
used to support normal bone metabolism, But in men between ages 20 and 30. A male client
a negative calcium balance isn’t a should be taught how to perform testicular
complication of osteoporosis. Dowager’s self- examination before age 20, preferably
hump results from bone fractures. It develops when he enters his teens.
when repeated vertebral fractures increase 26. Answer: (B) Place a saline-soaked sterile
spinal curvature. dressing on the wound.
20. Answer: (C) Changes from previous Rationale: The nurse should first place saline-
examinations. soaked sterile dressings on the open wound to
Rationale: Women are instructed to examine prevent tissue drying and possible infection.
themselves to discover changes that have Then the nurse should call the physician and
occurred in the breast. Only a physician can take the client’s vital signs. The dehiscence
diagnose lumps that are cancerous, areas of needs to be surgically closed, so the nurse
thickness or fullness that signal the presence should never try to close it.
of a malignancy, or masses that are fibrocystic 27. Answer: (A) A progressively deeper breaths
as opposed to malignant. followed by shallower breaths with apneic
21. Answer: (C) Balance the client’s periods of periods.
activity and rest. Rationale: Cheyne-Strokes respirations are
Rationale: A client with hyperthyroidism breaths that become progressively deeper
needs to be encouraged to balance periods of fallowed by shallower respirations with
apneas periods. Biot’s respirations are rapid, 33. Answer: (B) Current health promotion
deep breathing with abrupt pauses between activities
each breath, and equal depth between each Rationale: Recognizing an individual’s positive
breath. Kussmaul’s respirationa are rapid, health measures is very useful. General health
deep breathing without pauses. Tachypnea is in the previous 10 years is important,
shallow breathing with increased respiratory however, the current activities of an 84 year
rate. old client are most significant in planning care.
28. Answer: (B) Fine crackles Family history of disease for a client in later
Rationale: Fine crackles are caused by fluid in years is of minor significance. Marital status
the alveoli and commonly occur in clients with information may be important for discharge
heart failure. Tracheal breath sounds are planning but is not as significant for
auscultated over the trachea. Coarse crackles addressing the immediate medical problem.
are caused by secretion accumulation in the 34. Answer: (C) Place the client in a side lying
airways. Friction rubs occur with pleural position, with the head of the bed lowered.
inflammation. Rationale: The client should be positioned in a
29. Answer: (B) The airways are so swollen that no side-lying position with the head of the bed
air cannot get through lowered to prevent aspiration. A small amount
Rationale: During an acute attack, wheezing of toothpaste should be used and the mouth
may stop and breath sounds become swabbed or suctioned to remove pooled
inaudible because the airways are so swollen secretions. Lemon glycerin can be drying if
that air can’t get through. If the attack is over used for extended periods. Brushing the teeth
and swelling has decreased, there would be with the client lying supine may lead to
no more wheezing and less emergent concern. aspiration. Hydrogen peroxide is caustic to
Crackles do not replace wheezes during an tissues and should not be used.
acute asthma attack. 35. Answer: (C) Pneumonia
30. Answer: (D) Place the client on his side, Rationale: Fever productive cough and
remove dangerous objects, and protect his pleuritic chest pain are common signs and
head. symptoms of pneumonia. The client with
Rationale: During the active seizure phase, ARDS has dyspnea and hypoxia with
initiate precautions by placing the client on his worsening hypoxia over time, if not treated
side, removing dangerous objects, and aggressively. Pleuritic chest pain varies with
protecting his head from injury. A bite block respiration, unlike the constant chest pain
should never be inserted during the active during an MI; so this client most likely isn’t
seizure phase. Insertion can break the teeth having an MI. the client with TB typically has a
and lead to aspiration. cough producing blood-tinged sputum. A
31. Answer: (B) Kinked or obstructed chest tube sputum culture should be obtained to confirm
Rationales: Kinking and blockage of the chest the nurse’s suspicions.
tube is a common cause of a tension 36. Answer: (C) A 43-yesr-old homeless man with
pneumothorax. Infection and excessive a history of alcoholism
drainage won’t cause a tension Rationale: Clients who are economically
pneumothorax. Excessive water won’t affect disadvantaged, malnourished, and have
the chest tube drainage. reduced immunity, such as a client with a
32. Answer: (D) Stay with him but not intervene at history of alcoholism, are at extremely high
this time. risk for developing TB. A high school student,
Rationale: If the client is coughing, he should day- care worker, and businessman probably
be able to dislodge the object or cause a have a much low risk of contracting TB.
complete obstruction. If complete obstruction 37. Answer: (C ) To determine the extent of
occurs, the nurse should perform the lesions
abdominal thrust maneuver with the client Rationale: If the lesions are large enough, the
standing. If the client is unconscious, she chest X-ray will show their presence in the
should lay him down. A nurse should never lungs. Sputum culture confirms the diagnosis.
leave a choking client alone. There can be false-positive and false-negative

10
10
skin test results. A chest X-ray can’t determine fibrillation and is receiving L.V. dilitiazem
if this is a primary or secondary infection. (Cardizem)
38. Answer: (B) Bronchodilators Rationale: The client with atrial fibrillation has
Rationale: Bronchodilators are the first line of the greatest potential to become unstable and
treatment for asthma because broncho- is on L.V. medication that requires close
constriction is the cause of reduced airflow. monitoring. After assessing this client, the
Beta- adrenergic blockers aren’t used to treat nurse should assess the client with
asthma and can cause broncho- constriction. thrombophlebitis who is receiving a heparin
Inhaled oral steroids may be given to reduce infusion, and then the 58- year-old client
the inflammation but aren’t used for admitted 2 days ago with heart failure (his
emergency relief. signs and symptoms are resolving and don’t
39. Answer: (C) Chronic obstructive bronchitis require immediate attention). The lowest
Rationale: Because of this extensive smoking priority is the 89-year-old with end-stage
history and symptoms the client most likely right-sided heart failure, who requires time-
has chronic obstructive bronchitis. Client with consuming supportive measures.
ARDS have acute symptoms of hypoxia and 46. Answer: (C) Cocaine
typically need large amounts of oxygen. Rationale: Because of the client’s age and
Clients with asthma and emphysema tend not negative medical history, the nurse should
to have chronic cough or peripheral edema. question her about cocaine use. Cocaine
40. Answer: (A) The patient is under local increases myocardial oxygen consumption and
anesthesia during the procedure Rationale: can cause coronary artery spasm, leading to
Before the procedure, the patient is tachycardia, ventricular fibrillation, myocardial
administered with drugs that would help to ischemia, and myocardial infarction.
prevent infection and rejection of the Barbiturate overdose may trigger respiratory
transplanted cells such as antibiotics, depression and slow pulse. Opioids can cause
cytotoxic, and corticosteroids. During the marked respiratory depression, while
transplant, the patient is placed under general benzodiazepines can cause drowsiness and
anesthesia. confusion.
41. Answer: (D) Raise the side rails 47. Answer: (B) Nonmobile mass with irregular
Rationale: A patient who is disoriented is at edges
risk of falling out of bed. The initial action of Rationale: Breast cancer tumors are fixed,
the nurse should be raising the side rails to hard, and poorly delineated with irregular
ensure patients safety. edges. A mobile mass that is soft and easily
42. Answer: (A) Crowd red blood cells delineated is most often a fluid-filled benign
Rationale: The excessive production of white cyst. Axillary lymph nodes may or may not be
blood cells crowd out red blood cells palpable on initial detection of a cancerous
production which causes anemia to occur. mass. Nipple retraction — not eversion —
43. Answer: (B) Leukocytosis may be a sign of cancer.
Rationale: Chronic Lymphocytic leukemia (CLL) 48. Answer: (C) Radiation
is characterized by increased production of Rationale: The usual treatment for vaginal
leukocytes and lymphocytes resulting in cancer is external or intravaginal radiation
leukocytosis, and proliferation of these cells therapy. Less often, surgery is performed.
within the bone marrow, spleen and liver. Chemotherapy typically is prescribed only if
44. Answer: (A) Explain the risks of not having the vaginal cancer is diagnosed in an early stage,
surgery which is rare. Immunotherapy isn't used to
Rationale: The best initial response is to treat vaginal cancer.
explain the risks of not having the surgery. If 49. Answer: (B) Carcinoma in situ, no abnormal
the client understands the risks but still regional lymph nodes, and no evidence of
refuses the nurse should notify the physician distant metastasis
and the nurse supervisor and then record the Rationale: TIS, N0, M0 denotes carcinoma in
client’s refusal in the nurses’ notes. situ, no abnormal regional lymph nodes, and
45. Answer: (D) The 75-year-old client who was no evidence of distant metastasis. No
admitted 1 hour ago with new-onset atrial evidence of primary tumor, no abnormal
regional lymph nodes, and no evidence of 54. Answer: (D) "Remain supine for the time
distant metastasis is classified as T0, N0, M0. If specified by the physician." Rationale: The
the tumor and regional lymph nodes can't be nurse should instruct the client to remain
assessed and no evidence of metastasis exists, supine for the time specified by the physician.
the lesion is classified as TX, NX, M0. A Local anesthetics used in a subarachnoid block
progressive increase in tumor size, no don't alter the gag reflex. No interactions
demonstrable metastasis of the regional between local anesthetics and food occur.
lymph nodes, and ascending degrees of Local anesthetics don't cause hematuria.
distant metastasis is classified as T1, T2, T3, or 55. Answer: (C) Sigmoidoscopy
T4; N0; and M1, M2, or M3. Rationale: Used to visualize the lower GI tract,
50. Answer: (D) "Keep the stoma moist." sigmoidoscopy and proctoscopy aid in the
Rationale: The nurse should instruct the client detection of two-thirds of all colorectal
to keep the stoma moist, such as by applying a cancers. Stool Hematest detects blood, which
thin layer of petroleum jelly around the edges, is a sign of colorectal cancer; however, the
because a dry stoma may become irritated. test doesn't confirm the diagnosis. CEA may
The nurse should recommend placing a stoma be elevated in colorectal cancer but isn't
bib over the stoma to filter and warm air considered a confirming test. An abdominal CT
before it enters the stoma. The client should scan is used to stage the presence of
begin performing stoma care without colorectal cancer.
assistance as soon as possible to gain 56. Answer: (B) A fixed nodular mass with
independence in self-care activities. dimpling of the overlying skin
51. Answer: (B) Lung cancer Rationale: A fixed nodular mass with dimpling
Rationale: Lung cancer is the most deadly type of the overlying skin is common during late
of cancer in both women and men. Breast stages of breast cancer. Many women have
cancer ranks second in women, followed (in slightly asymmetrical breasts. Bloody nipple
descending order) by colon and rectal cancer, discharge is a sign of intraductal papilloma, a
pancreatic cancer, ovarian cancer, uterine benign condition. Multiple firm, round, freely
cancer, lymphoma, leukemia, liver cancer, movable masses that change with the
brain cancer, stomach cancer, and multiple menstrual cycle indicate fibrocystic breasts, a
myeloma. benign condition.
52. Answer: (A) miosis, partial eyelid ptosis, and 57. Answer: (A) Liver
anhidrosis on the affected side of the face. Rationale: The liver is one of the five most
Rationale: Horner's syndrome, which occurs common cancer metastasis sites. The others
when a lung tumor invades the ribs and are the lymph nodes, lung, bone, and brain.
affects the sympathetic nerve ganglia, is The colon, reproductive tract, and WBCs are
characterized by miosis, partial eyelid ptosis, occasional metastasis sites.
and anhidrosis on the affected side of the 58. Answer: (D) The client wears a watch and
face. Chest pain, dyspnea, cough, weight loss, wedding band.
and fever are associated with pleural tumors. Rationale: During an MRI, the client should
Arm and shoulder pain and atrophy of the arm wear no metal objects, such as jewelry,
and hand muscles on the affected side suggest because the strong magnetic field can pull on
Pancoast's tumor, a lung tumor involving the them, causing injury to the client and (if they
first thoracic and eighth cervical nerves within fly off) to others. The client must lie still
the brachial plexus. Hoarseness in a client during the MRI but can talk to those
with lung cancer suggests that the tumor has performing the test by way of the microphone
extended to the recurrent laryngeal nerve; inside the scanner tunnel. The client should
dysphagia suggests that the lung tumor is hear thumping sounds, which are caused by
compressing the esophagus. the sound waves thumping on the magnetic
53. 53. Answer: (A) prostate-specific antigen, field.
which is used to screen for prostate cancer. 59. Answer: (C) The recommended daily
Rationale: PSA stands for prostate-specific allowance of calcium may be found in a wide
antigen, which is used to screen for prostate variety of foods.
cancer. The other answers are incorrect.

10
10
Rationale: Premenopausal women require more likely to occur in the lower extremities.
1,000 mg of calcium per day. Postmenopausal A stroke isn’t linked to protein loss.
women require 1,500 mg per day. It's often, 64. Answer: (B) It appears on the distal
though not always, possible to get the interphalangeal joint
recommended daily requirement in the foods Rationale: Heberden’s nodes appear on the
we eat. Supplements are available but not distal interphalageal joint on both men and
always necessary. Osteoporosis doesn't show women. Bouchard’s node appears on the
up on ordinary X-rays until 30% of the bone dorsolateral aspect of the proximal
loss has occurred. Bone densitometry can interphalangeal joint.
detect bone loss of 3% or less. This test is 65. Answer: (B) Osteoarthritis is a localized
sometimes recommended routinely for disease rheumatoid arthritis is systemic
women over 35 who are at risk. Strenuous Rationale: Osteoarthritis is a localized disease,
exercise won't cause fractures. rheumatoid arthritis is systemic. Osteoarthritis
60. Answer: (C) Joint flexion of less than 50% isn’t gender-specific, but rheumatoid arthritis
Rationale: Arthroscopy is contraindicated in is. Clients have dislocations and subluxations
clients with joint flexion of less than 50% in both disorders.
because of technical problems in inserting the 66. Answer: (C) The cane should be used on the
instrument into the joint to see it clearly. unaffected side
Other contraindications for this procedure Rationale: A cane should be used on the
include skin and wound infections. Joint pain unaffected side. A client with osteoarthritis
may be an indication, not a contraindication, should be encouraged to ambulate with a
for arthroscopy. Joint deformity and joint cane, walker, or other assistive device as
stiffness aren't contraindications for this needed; their use takes weight and stress off
procedure. joints.
61. Answer: (D) Gouty arthritis 67. Answer: (A) a. 9 U regular insulin and 21 U
Rationale: Gouty arthritis, a metabolic disease, neutral protamine Hagedorn (NPH).
is characterized by urate deposits and pain in Rationale: A 70/30 insulin preparation is 70%
the joints, especially those in the feet and NPH and 30% regular insulin. Therefore, a
legs. Urate deposits don't occur in septic or correct substitution requires mixing 21 U of
traumatic arthritis. Septic arthritis results from NPH and 9 U of regular insulin. The other
bacterial invasion of a joint and leads to choices are incorrect dosages for the
inflammation of the synovial lining. Traumatic prescribed insulin.
arthritis results from blunt trauma to a joint or 68. Answer: (C) colchicines
ligament. Intermittent arthritis is a rare, Rationale: A disease characterized by joint
benign condition marked by regular, recurrent inflammation (especially in the great toe),
joint effusions, especially in the knees. gout is caused by urate crystal deposits in the
62. Answer: (B) 30 ml/hou joints. The physician prescribes colchicine to
Rationale: An infusion prepared with 25,000 reduce these deposits and thus ease joint
units of heparin in 500 ml of saline solution inflammation. Although aspirin is used to
yields 50 units of heparin per milliliter of reduce joint inflammation and pain in clients
solution. The equation is set up as 50 units with osteoarthritis and rheumatoid arthritis, it
times X (the unknown quantity) equals 1,500 isn't indicated for gout because it has no
units/hour, X equals 30 ml/hour. effect on urate crystal formation. Furosemide,
63. Answer: (B) Loss of muscle contraction a diuretic, doesn't relieve gout. Calcium
decreasing venous return gluconate is used to reverse a negative
Rationale: In clients with hemiplegia or calcium balance and relieve muscle cramps,
hemiparesis loss of muscle contraction not to treat gout.
decreases venous return and may cause 69. Answer: (A) Adrenal cortex
swelling of the affected extremity. Rationale: Excessive secretion of aldosterone
Contractures, or bony calcifications may occur in the adrenal cortex is responsible for the
with a stroke, but don’t appear with swelling. client's hypertension. This hormone acts on
DVT may develop in clients with a stroke but is the renal tubule, where it promotes
reabsorption of sodium and excretion of
potassium and hydrogen ions. The pancreas is at greatest risk for hypoglycemia from 3
mainly secretes hormones involved in fuel p.m. to 7 p.m.
metabolism. The adrenal medulla secretes the 75. Answer: (A) Glucocorticoids and androgens
catecholamines — epinephrine and Rationale: The adrenal glands have two
norepinephrine. The parathyroids secrete divisions, the cortex and medulla. The cortex
parathyroid hormone. produces three types of hormones:
70. Answer: (C) They debride the wound and glucocorticoids, mineralocorticoids, and
promote healing by secondary intention androgens. The medulla produces
Rationale: For this client, wet-to-dry dressings catecholamines— epinephrine and
are most appropriate because they clean the norepinephrine.
foot ulcer by debriding exudate and necrotic 76. Answer: (A) Hypocalcemia
tissue, thus promoting healing by secondary Rationale: Hypocalcemia may follow thyroid
intention. Moist, transparent dressings surgery if the parathyroid glands were
contain exudate and provide a moist wound removed accidentally. Signs and symptoms of
environment. Hydrocolloid dressings prevent hypocalcemia may be delayed for up to 7 days
the entrance of microorganisms and minimize after surgery. Thyroid surgery doesn't directly
wound discomfort. Dry sterile dressings cause serum sodium, potassium, or
protect the wound from mechanical trauma magnesium abnormalities. Hyponatremia may
and promote healing. occur if the client inadvertently received too
71. Answer: (A) Hyperkalemia much fluid; however, this can happen to any
Rationale: In adrenal insufficiency, the client surgical client receiving I.V. fluid therapy, not
has hyperkalemia due to reduced aldosterone just one recovering from thyroid surgery.
secretion. BUN increases as the glomerular Hyperkalemia and hypermagnesemia usually
filtration rate is reduced. Hyponatremia is are associated with reduced renal excretion of
caused by reduced aldosterone secretion. potassium and magnesium, not thyroid
Reduced cortisol secretion leads to impaired surgery.
glyconeogenesis and a reduction of glycogen 77. Answer: (D) Carcinoembryonic antigen level
in the liver and muscle, causing hypoglycemia. Rationale: In clients who smoke, the level of
72. Answer: (C) Restricting fluids carcinoembryonic antigen is elevated.
Rationale: To reduce water retention in a Therefore, it can't be used as a general
client with the SIADH, the nurse should indicator of cancer. However, it is helpful in
restrict fluids. Administering fluids by any monitoring cancer treatment because the
route would further increase the client's level usually falls to normal within 1 month if
already heightened fluid load. treatment is successful. An elevated acid
73. Answer: (D) glycosylated hemoglobin level. phosphatase level may indicate prostate
Rationale: Because some of the glucose in the cancer. An elevated alkaline phosphatase level
bloodstream attaches to some of the may reflect bone metastasis. An elevated
hemoglobin and stays attached during the serum calcitonin level usually signals thyroid
120-day life span of red blood cells, cancer.
glycosylated hemoglobin levels provide 78. Answer: (B) Dyspnea, tachycardia, and pallor
information about blood glucose levels during Rationale: Signs of iron-deficiency anemia
the previous 3 months. Fasting blood glucose include dyspnea, tachycardia, and pallor as
and urine glucose levels only give information well as fatigue, listlessness, irritability, and
about glucose levels at the point in time when headache. Night sweats, weight loss, and
they were obtained. Serum fructosamine diarrhea may signal acquired
levels provide information about blood immunodeficiency syndrome (AIDS). Nausea,
glucose control over the past 2 to 3 weeks. vomiting, and anorexia may be signs of
74. Answer: (C) 4:00 pm hepatitis B. Itching, rash, and jaundice may
Rationale: NPH is an intermediate-acting result from an allergic or hemolytic reaction.
insulin that peaks 8 to 12 hours after 79. Answer: (D) "I'll need to have a C-section if I
administration. Because the nurse become pregnant and have a baby."
administered NPH insulin at 7 a.m., the client Rationale: The human immunodeficiency virus
(HIV) is transmitted from mother to child via

10
10
the transplacental route, but a Cesarean therapy is discontinued. Aspirin doesn't lead
section delivery isn't necessary when the to weight gain or fine motor tremors. Large or
mother is HIV-positive. The use of birth toxic salicylate doses may cause respiratory
control will prevent the conception of a child alkalosis, not respiratory acidosis.
who might have HIV. It's true that a mother 84. Answer: (D) Lymphocyte
who's HIV positive can give birth to a baby Rationale: The lymphocyte provides adaptive
who's HIV negative. immunity — recognition of a foreign antigen
80. Answer: (C) "Avoid sharing such articles as and formation of memory cells against the
toothbrushes and razors." antigen. Adaptive immunity is mediated by B
Rationale: The human immunodeficiency virus and T lymphocytes and can be acquired
(HIV), which causes AIDS, is most actively or passively. The neutrophil is crucial
concentrated in the blood. For this reason, the to phagocytosis. The basophil plays an
client shouldn't share personal articles that important role in the release of inflammatory
may be blood-contaminated, such as mediators. The monocyte functions in
toothbrushes and razors, with other family phagocytosis and monokine production.
members. HIV isn't transmitted by bathing or 85. Answer: (A) moisture replacement.
by eating from plates, utensils, or serving Rationale: Sjogren's syndrome is an
dishes used by a person with AIDS. autoimmune disorder leading to progressive
81. Answer: (B) Pallor, tachycardia, and a sore loss of lubrication of the skin, GI tract, ears,
tongue nose, and vagina. Moisture replacement is the
Rationale: Pallor, tachycardia, and a sore mainstay of therapy. Though malnutrition and
tongue are all characteristic findings in electrolyte imbalance may occur as a result of
pernicious anemia. Other clinical Sjogren's syndrome's effect on the GI tract, it
manifestations include anorexia; weight loss; a isn't the predominant problem. Arrhythmias
smooth, beefy red tongue; a wide pulse aren't a problem associated with Sjogren's
pressure; palpitations; angina; weakness; syndrome.
fatigue; and paresthesia of the hands and feet. 86. Answer: (C) stool for Clostridium difficile test.
Bradycardia, reduced pulse pressure, weight Rationale: Immunosuppressed clients — for
gain, and double vision aren't characteristic example, clients receiving chemotherapy, —
findings in pernicious anemia. are at risk for infection with C. difficile, which
82. Answer: (B) Administer epinephrine, as causes "horse barn" smelling diarrhea.
prescribed, and prepare to intubate the client Successful treatment begins with an accurate
if necessary. diagnosis, which includes a stool test. The
Rationale: To reverse anaphylactic shock, the ELISA test is diagnostic for human
nurse first should administer epinephrine, a immunodeficiency virus (HIV) and isn't
potent bronchodilator as prescribed. The indicated in this case. An electrolyte panel and
physician is likely to order additional hemogram may be useful in the overall
medications, such as antihistamines and evaluation of a client but aren't diagnostic for
corticosteroids; if these medications don't specific causes of diarrhea. A flat plate of the
relieve the respiratory compromise associated abdomen may provide useful information
with anaphylaxis, the nurse should prepare to about bowel function but isn't indicated in the
intubate the client. No antidote for penicillin case of "horse barn" smelling diarrhea.
exists; however, the nurse should continue to 87. Answer: (D) Western blot test with ELISA.
monitor the client's vital signs. A client who Rationale: HIV infection is detected by
remains hypotensive may need fluid analyzing blood for antibodies to HIV, which
resuscitation and fluid intake and output form approximately 2 to 12 weeks after
monitoring; however, administering exposure to HIV and denote infection. The
epinephrine is the first priority. Western blot test — electrophoresis of
83. Answer: (D) bilateral hearing loss. antibody proteins — is more than 98%
Rationale: Prolonged use of aspirin and other accurate in detecting HIV antibodies when
salicylates sometimes causes bilateral hearing used in conjunction with the ELISA. It isn't
loss of 30 to 40 decibels. Usually, this adverse specific when used alone. E-rosette
effect resolves within 2 weeks after the immunofluorescence is used to detect viruses
in general; it doesn't confirm HIV infection. Rationale: Using comfort measures and
Quantification of T-lymphocytes is a useful pillows to position the client is a non-
monitoring test but isn't diagnostic for HIV. pharmacological methods of pain relief.
The ELISA test detects HIV antibody particles 95. Answer: (B) Warm the dialysate solution.
but may yield inaccurate results; a positive Rationale: Cold dialysate increases discomfort.
ELISA result must be confirmed by the The solution should be warmed to body
Western blot test. temperature in warmer or heating pad; don’t
88. Answer: (C) Abnormally low hematocrit (HCT) use microwave oven.
and hemoglobin (Hb) levels 96. Answer: (C) The client holds the cane with his
Rationale: Low preoperative HCT and Hb left hand, moves the cane forward followed
levels indicate the client may require a blood by the right leg, and then moves the left leg.
transfusion before surgery. If the HCT and Hb Rationale: The cane acts as a support and aids
levels decrease during surgery because of in weight bearing for the weaker right leg.
blood loss, the potential need for a 97. Answer: (A) Ask the woman’s family to
transfusion increases. Possible renal failure is provide personal items such as photos or
indicated by elevated BUN or creatinine levels. mementos.
Urine constituents aren't found in the blood. Rationale: Photos and mementos provide
Coagulation is determined by the presence of visual stimulation to reduce sensory
appropriate clotting factors, not electrolytes. deprivation.
89. Answer: (A) Platelet count, prothrombin time, 98. Answer: (B) The client lifts the walker, moves
and partial thromboplastin time it forward 10 inches, and then takes several
Rationale: The diagnosis of DIC is based on the small steps forward.
results of laboratory studies of prothrombin Rationale: A walker needs to be picked up,
time, platelet count, thrombin time, partial placed down on all legs.
thromboplastin time, and fibrinogen level as 99. Answer: (C) Isolation from their families and
well as client history and other assessment familiar surroundings.
factors. Blood glucose levels, WBC count, Rationale: Gradual loss of sight, hearing, and
calcium levels, and potassium levels aren't taste interferes with normal functioning.
used to confirm a diagnosis of DIC. 100. Answer: (A) Encourage the client to perform
90. Answer: (D) Strawberries pursed lip breathing.
Rationale: Common food allergens include Rationale: Purse lip breathing prevents the
berries, peanuts, Brazil nuts, cashews, collapse of lung unit and helps client control
shellfish, and eggs. Bread, carrots, and rate and depth of breathing.
oranges rarely cause allergic reactions.
91. Answer: (B) A client with cast on the right leg
who states, “I have a funny feeling in my right
leg.”
Rationale: It may indicate neurovascular
compromise, requires immediate assessment.
92. Answer: (D) A 62-year-old who had an
abdominal-perineal resection three days ago;
client complaints of chills.
Rationale: The client is at risk for peritonitis;
should be assessed for further symptoms and
infection.
93. Answer: (C) The client spontaneously flexes
his wrist when the blood pressure is obtained.
Rationale: Carpal spasms indicate
hypocalcemia.
94. Answer: (D) Use comfort measures and
pillows to position the client.

11
11
TEST IV - Care of Clients with Physiologic and b. Decrease fluid intake at meal times.
Psychosocial Alterations c. Avoid foods that in the past caused
flatus.
1. Randy has undergone kidney transplant, what d. Adhere to a bland diet prior to social
assessment would prompt Nurse Katrina to events.
suspect organ rejection?
a. Sudden weight loss 7. Nurse Ron begins to teach a male client how to
b. Polyuria perform colostomy irrigations. The nurse would
c. Hypertension evaluate that the instructions were understood
d. Shock when the client states, “I should:
a. Lie on my left side while instilling the
2. The immediate objective of nursing care for an irrigating solution.”
overweight, mildly hypertensive male client with b. Keep the irrigating container less than
ureteral colic and hematuria is to decrease: 18 inches above the stoma.”
a. Pain c. Instill a minimum of 1200 ml of irrigating
b. Weight solution to stimulate evacuation of the
c. Hematuria bowel.”
d. Hypertension d. Insert the irrigating catheter deeper into
the stoma if cramping occurs during the
3. Matilda, with hyperthyroidism is to receive procedure.”
Lugol’s iodine solution before a subtotal
thyroidectomy is performed. The nurse is aware 8. Patrick is in the oliguric phase of acute tubular
that this medication is given to: necrosis and is experiencing fluid and electrolyte
a. Decrease the total basal metabolic rate. imbalances. The client is somewhat confused
b. Maintain the function of the parathyroid and complains of nausea and muscle weakness.
glands. As part of the prescribed therapy to correct this
c. Block the formation of thyroxine by the electrolyte imbalance, the nurse would expect
thyroid gland. to:
d. Decrease the size and vascularity of the a. Administer Kayexalate
thyroid gland. b. Restrict foods high in protein
c. Increase oral intake of cheese and milk.
4. Ricardo, was diagnosed with type I diabetes. The d. Administer large amounts of normal
nurse is aware that acute hypoglycemia also can saline via I.V.
develop in the client who is diagnosed with:
a. Liver disease 9. Mario has burn injury. After Forty48 hours, the
b. Hypertension physician orders for Mario 2 liters of IV fluid to
c. Type 2 diabetes be administered q12 h. The drop factor of the
d. Hyperthyroidism tubing is 10 gtt/ml. The nurse should set the
flow to provide:
5. Tracy is receiving combination chemotherapy for a. 18 gtt/min
treatment of metastatic carcinoma. Nurse Ruby b. 28 gtt/min
should monitor the client for the systemic side c. 32 gtt/min
effect of: d. 36 gtt/min
a. Ascites
b. Nystagmus 10. Terence suffered from burn injury. Using the rule
c. Leukopenia of nines, which has the largest percent of burns?
d. Polycythemia a. Face and neck
b. Right upper arm and penis
6. Norma, with recent colostomy expresses c. Right thigh and penis
concern about the inability to control the d. Upper trunk
passage of gas. Nurse Oliver should suggest that
the client plan to: 11. Herbert, a 45 year old construction engineer is
a. Eliminate foods high in cellulose. brought to the hospital unconscious after falling
from a 2-story building. When assessing the d. Only ice chips and cold liquids will be
client, the nurse would be most concerned if the allowed initially.
assessment revealed:
a. Reactive pupils 16. Nurse Tristan is caring for a male client in acute
b. A depressed fontanel renal failure. The nurse should expect hypertonic
c. Bleeding from ears glucose, insulin infusions, and sodium
d. An elevated temperature bicarbonate to be used to treat:
a. hypernatremia.
12. Nurse Sherry is teaching male client regarding b. hypokalemia.
his permanent artificial pacemaker. Which c. hyperkalemia.
information given by the nurse shows her d. hypercalcemia.
knowledge deficit about the artificial cardiac
pacemaker? 17. Ms. X has just been diagnosed with condylomata
a. take the pulse rate once a day, in the acuminata (genital warts). What information is
morning upon awakening appropriate to tell this client?
b. May be allowed to use electrical a. This condition puts her at a higher risk
appliances for cervical cancer; therefore, she should
c. Have regular follow up care have a Papanicolaou (Pap) smear
d. May engage in contact sports annually.
b. The most common treatment is
13. The nurse is ware that the most relevant metronidazole (Flagyl), which should
knowledge about oxygen administration to a eradicate the problem within 7 to 10
male client with COPD is days.
a. Oxygen at 1-2L/min is given to maintain c. The potential for transmission to her
the hypoxic stimulus for breathing. sexual partner will be eliminated if
b. Hypoxia stimulates the central condoms are used every time they have
chemoreceptors in the medulla that sexual intercourse.
makes the client breath. d. The human papillomavirus (HPV), which
c. Oxygen is administered best using a non- causes condylomata acuminata, can't be
rebreathing mask transmitted during oral sex.
d. Blood gases are monitored using a pulse
oximeter. 18. Maritess was recently diagnosed with a
genitourinary problem and is being examined in
14. Tonny has undergoes a left thoracotomy and a the emergency department. When palpating her
partial pneumonectomy. Chest tubes are kidneys, the nurse should keep which anatomical
inserted, and one-bottle water-seal drainage is fact in mind?
instituted in the operating room. In the a. The left kidney usually is slightly higher
postanesthesia care unit Tonny is placed in than the right one.
Fowler's position on either his right side or on b. The kidneys are situated just above the
his back. The nurse is aware that this position: adrenal glands.
a. Reduce incisional pain. c. The average kidney is approximately 5
b. Facilitate ventilation of the left lung. cm (2") long and 2 to 3 cm (¾" to 1-1/8")
c. Equalize pressure in the pleural space. wide.
d. Increase venous return d. The kidneys lie between the 10th and
12th thoracic vertebrae.
15. Kristine is scheduled for a bronchoscopy. When
teaching Kristine what to expect afterward, the 19. Jestoni with chronic renal failure (CRF) is
nurse's highest priority of information would be: admitted to the urology unit. The nurse is aware
a. Food and fluids will be withheld for at that the diagnostic test are consistent with CRF if
least 2 hours. the result is:
b. Warm saline gargles will be done q 2h. a. Increased pH with decreased hydrogen
c. Coughing and deep-breathing exercises ions.
will be done q2h.

11
11
b. Increased serum levels of potassium, b. Palpate the abdomen.
magnesium, and calcium. c. Change the client's position.
c. Blood urea nitrogen (BUN) 100 mg/dl d. Insert a rectal tube.
and serum creatinine 6.5 mg/ dl.
d. Uric acid analysis 3.5 mg/dl and 24. Wilfredo with a recent history of rectal bleeding
phenolsulfonphthalein (PSP) excretion is being prepared for a colonoscopy. How should
75%. the nurse Patricia position the client for this test
initially?
20. Katrina has an abnormal result on a a. Lying on the right side with legs straight
Papanicolaou test. After admitting that she read b. Lying on the left side with knees bent
her chart while the nurse was out of the room, c. Prone with the torso elevated
Katrina asks what dysplasia means. Which d. Bent over with hands touching the floor
definition should the nurse provide?
a. Presence of completely undifferentiated 25. A male client with inflammatory bowel disease
tumor cells that don't resemble cells of undergoes an ileostomy. On the first day after
the tissues of their origin. surgery, Nurse Oliver notes that the client's
b. Increase in the number of normal cells in stoma appears dusky. How should the nurse
a normal arrangement in a tissue or an interpret this finding?
organ. a. Blood supply to the stoma has been
c. Replacement of one type of fully interrupted.
differentiated cell by another in tissues b. This is a normal finding 1 day after
where the second type normally isn't surgery.
found. c. The ostomy bag should be adjusted.
d. Alteration in the size, shape, and d. An intestinal obstruction has occurred.
organization of differentiated cells.
26. Anthony suffers burns on the legs, which nursing
21. During a routine checkup, Nurse Mariane intervention helps prevent contractures?
assesses a male client with acquired a. Applying knee splints
immunodeficiency syndrome (AIDS) for signs and b. Elevating the foot of the bed
symptoms of cancer. What is the most common c. Hyperextending the client's palms
AIDS-related cancer? d. Performing shoulder range-of-motion
a. Squamous cell carcinoma exercises
b. Multiple myeloma
c. Leukemia 27. Nurse Ron is assessing a client admitted with
d. Kaposi's sarcoma second- and third-degree burns on the face,
arms, and chest. Which finding indicates a
22. Ricardo is scheduled for a prostatectomy, and potential problem?
the anesthesiologist plans to use a spinal a. Partial pressure of arterial oxygen
(subarachnoid) block during surgery. In the (PaO2) value of 80 mm Hg.
operating room, the nurse positions the client b. Urine output of 20 ml/hour.
according to the anesthesiologist's instructions. c. White pulmonary secretions.
Why does the client require special positioning d. Rectal temperature of 100.6° F (38° C).
for this type of anesthesia?
a. To prevent confusion 28. Mr. Mendoza who has suffered a
b. To prevent seizures cerebrovascular accident (CVA) is too weak to
c. To prevent cerebrospinal fluid (CSF) move on his own. To help the client avoid
leakage pressure ulcers, Nurse Celia should:
d. To prevent cardiac arrhythmias a. Turn him frequently.
b. Perform passive range-of-motion (ROM)
23. A male client had a nephrectomy 2 days ago and exercises.
is now complaining of abdominal pressure and c. Reduce the client's fluid intake.
nausea. The first nursing action should be to: d. Encourage the client to use a footboard.
a. Auscultate bowel sounds.
29. Nurse Maria plans to administer dexamethasone 34. A 37-year-old male client was admitted to the
cream to a female client who has dermatitis over coronary care unit (CCU) 2 days ago with an
the anterior chest. How should the nurse apply acute myocardial infarction. Which of the
this topical agent? following actions would breach the client
a. With a circular motion, to enhance confidentiality?
absorption. a. The CCU nurse gives a verbal report to
b. With an upward motion, to increase the nurse on the telemetry unit before
blood supply to the affected area transferring the client to that unit
c. In long, even, outward, and downward b. The CCU nurse notifies the on-call
strokes in the direction of hair growth physician about a change in the client’s
d. In long, even, outward, and upward condition
strokes in the direction opposite hair c. The emergency department nurse calls
growth up the latest electrocardiogram results
to check the client’s progress.
30. Nurse Kate is aware that one of the following d. At the client’s request, the CCU nurse
classes of medication protects the ischemic updates the client’s wife on his condition
myocardium by blocking catecholamines and
sympathetic nerve stimulation is: 35. A male client arriving in the emergency
a. Beta -adrenergic blockers department is receiving cardiopulmonary
b. Calcium channel blocker resuscitation from paramedics who are giving
c. Narcotics ventilations through an endotracheal (ET) tube
d. Nitrates that they placed in the client’s home. During a
pause in compressions, the cardiac monitor
31. A male client has jugular distention. On what shows narrow QRS complexes and a heart rate
position should the nurse place the head of the of beats/minute with a palpable pulse. Which of
bed to obtain the most accurate reading of the following actions should the nurse take first?
jugular vein distention? a. Start an L.V. line and administer
a. High Fowler’s amiodarone (Cardarone), 300 mg L.V.
b. Raised 10 degrees over 10 minutes.
c. Raised 30 degrees b. Check endotracheal tube placement.
d. Supine position c. Obtain an arterial blood gas (ABG)
sample.
32. The nurse is aware that one of the following d. Administer atropine, 1 mg L.V.
classes of medications maximizes cardiac
performance in clients with heart failure by 36. After cardiac surgery, a client’s blood pressure
increasing ventricular contractility? measures 126/80 mm Hg. Nurse Katrina
a. Beta-adrenergic blockers determines that mean arterial pressure (MAP) is
b. Calcium channel blocker which of the following?
c. Diuretics a. 46 mm Hg
d. Inotropic agents b. 80 mm Hg
c. 95 mm Hg
33. A male client has a reduced serum high-density d. 90 mm Hg
lipoprotein (HDL) level and an elevated low-
density lipoprotein (LDL) level. Which of the 37. A female client arrives at the emergency
following dietary modifications is not department with chest and stomach pain and a
appropriate for this client? report of black tarry stool for several months.
a. Fiber intake of 25 to 30 g daily Which of the following order should the nurse
b. Less than 30% of calories from fat Oliver anticipate?
c. Cholesterol intake of less than 300 mg a. Cardiac monitor, oxygen, creatine kinase
daily and lactate dehydrogenase levels
d. Less than 10% of calories from saturated b. Prothrombin time, partial
fat thromboplastin time, fibrinogen and
fibrin split product values.

11
11
c. Electrocardiogram, complete blood 43. The nurse is aware that the following symptom
count, testing for occult blood, is most commonly an early indication of stage 1
comprehensive serum metabolic panel. Hodgkin’s disease?
d. Electroencephalogram, alkaline a. Pericarditis
phosphatase and aspartate b. Night sweat
aminotransferase levels, basic serum c. Splenomegaly
metabolic panel d. Persistent hypothermia

38. Macario had coronary artery bypass graft (CABG) 44. Francis with leukemia has neutropenia. Which of
surgery 3 days ago. Which of the following the following functions must frequently
conditions is suspected by the nurse when a assessed?
decrease in platelet count from 230,000 ul to a. Blood pressure
5,000 ul is noted? b. Bowel sounds
a. Pancytopenia c. Heart sounds
b. Idiopathic thrombocytopemic purpura d. Breath sounds
(ITP)
c. Disseminated intravascular coagulation 45. The nurse knows that neurologic complications
(DIC) of multiple myeloma (MM) usually involve which
d. Heparin-associated thrombosis and of the following body system?
thrombocytopenia (HATT) a. Brain
b. Muscle spasm
39. Which of the following drugs would be ordered c. Renal dysfunction
by the physician to improve the platelet count in d. Myocardial irritability
a male client with idiopathic thrombocytopenic
purpura (ITP)? 46. Nurse Patricia is aware that the average length
a. Acetylsalicylic acid (ASA) of time from human immunodeficiency virus
b. Corticosteroids (HIV) infection to the development of acquired
c. Methotrezate immunodeficiency syndrome (AIDS)?
d. Vitamin K a. Less than 5 years
b. 5 to 7 years
40. A female client is scheduled to receive a heart c. 10 years
valve replacement with a porcine valve. Which d. More than 10 years
of the following types of transplant is this?
a. Allogeneic 47. An 18-year-old male client admitted with heat
b. Autologous stroke begins to show signs of disseminated
c. Syngeneic intravascular coagulation (DIC). Which of the
d. Xenogeneic following laboratory findings is most consistent
with DIC?
41. Marco falls off his bicycle and injuries his ankle. a. Low platelet count
Which of the following actions shows the initial b. Elevated fibrinogen levels
response to the injury in the extrinsic pathway? c. Low levels of fibrin degradation products
a. Release of Calcium d. Reduced prothrombin time
b. Release of tissue thromboplastin
c. Conversion of factors XII to factor XIIa 48. Mario comes to the clinic complaining of fever,
d. Conversion of factor VIII to factor VIIIa drenching night sweats, and unexplained weight
42. Instructions for a client with systemic lupus loss over the past 3 months. Physical
erythematosus (SLE) would include information examination reveals a single enlarged
about which of the following blood dyscrasias? supraclavicular lymph node. Which of the
a. Dressler’s syndrome following is the most probable diagnosis?
b. Polycythemia a. Influenza
c. Essential thrombocytopenia b. Sickle cell anemia
d. Von Willebrand’s disease c. Leukemia
d. Hodgkin’s disease
red and swollen, when the IV is touched Stacy
49. A male client with a gunshot wound requires an shouts in pain. The first nursing action to take is:
emergency blood transfusion. His blood type is a. Notify the physician
AB negative. Which blood type would be the b. Flush the IV line with saline solution
safest for him to receive? c. Immediately discontinue the infusion
a. AB Rh-positive d. Apply an ice pack to the site, followed by
b. A Rh-positive warm compress.
c. A Rh-negative
d. O Rh-positive 54. The term “blue bloater” refers to a male client
which of the following conditions?
Situation: Stacy is diagnosed with acute lymphoid a. Adult respiratory distress syndrome
leukemia (ALL) and beginning chemotherapy. (ARDS)
b. Asthma
50. Stacy is discharged from the hospital following c. Chronic obstructive bronchitis
her chemotherapy treatments. Which statement d. Emphysema
of Stacy’s mother indicated that she understands
when she will contact the physician? 55. The term “pink puffer” refers to the female
a. “I should contact the physician if Stacy client with which of the following conditions?
has difficulty in sleeping”. a. Adult respiratory distress syndrome
b. “I will call my doctor if Stacy has (ARDS)
persistent vomiting and diarrhea”. b. Asthma
c. “My physician should be called if Stacy is c. Chronic obstructive bronchitis
irritable and unhappy”. d. Emphysema
d. “Should Stacy have continued hair loss, I
need to call the doctor”. 56. Jose is in danger of respiratory arrest following
the administration of a narcotic analgesic. An
51. Stacy’s mother states to the nurse that it is hard arterial blood gas value is obtained. Nurse Oliver
to see Stacy with no hair. The best response for would expect the paco2 to be which of the
the nurse is: following values?
a. “Stacy looks very nice wearing a hat”. a. 15 mm Hg
b. “You should not worry about her hair, b. 30 mm Hg
just be glad that she is alive”. c. 40 mm Hg
c. “Yes it is upsetting. But try to cover up d. 80 mm Hg
your feelings when you are with her or
else she may be upset”. 57. Timothy’s arterial blood gas (ABG) results are as
d. “This is only temporary; Stacy will re- follows; pH 7.16; Paco2 80 mm Hg; Pao2 46 mm
grow new hair in 3-6 months, but may Hg; HCO3- 24mEq/L; Sao2 81%. This ABG result
be different in texture”. represents which of the following conditions?
a. Metabolic acidosis
52. Stacy has beginning stomatitis. To promote oral b. Metabolic alkalosis
hygiene and comfort, the nurse in-charge c. Respiratory acidosis
should: d. Respiratory alkalosis
a. Provide frequent mouthwash with
normal saline. 58. Norma has started a new drug for hypertension.
b. Apply viscous Lidocaine to oral ulcers as Thirty minutes after she takes the drug, she
needed. develops chest tightness and becomes short of
c. Use lemon glycerine swabs every 2 breath and tachypneic. She has a decreased level
hours. of consciousness. These signs indicate which of
d. Rinse mouth with Hydrogen Peroxide. the following conditions?
a. Asthma attack
53. During the administration of chemotherapy b. Pulmonary embolism
agents, Nurse Oliver observed that the IV site is c. Respiratory failure
d. Rheumatoid arthritis

11
11

decreased RBC count, decreased WBC


Situation: Mr. Gonzales was admitted to the hospital count.
with ascites and jaundice. To rule out cirrhosis of the d. Intermitted lower back pain, decreased
liver: blood pressure, decreased RBC count,
increased WBC count.
59. Which laboratory test indicates liver cirrhosis?
a. Decreased red blood cell count 64. After undergoing a cardiac catheterization, Tracy
b. Decreased serum acid phosphate level has a large puddle of blood under his buttocks.
c. Elevated white blood cell count Which of the following steps should the nurse
d. Elevated serum aminotransferase take first?
a. Call for help.
60. 60.The biopsy of Mr. Gonzales confirms the b. Obtain vital signs
diagnosis of cirrhosis. Mr. Gonzales is at c. Ask the client to “lift up”
increased risk for excessive bleeding primarily d. Apply gloves and assess the groin site
because of:
a. Impaired clotting mechanism 65. Which of the following treatment is a suitable
b. Varix formation surgical intervention for a client with unstable
c. Inadequate nutrition angina?
d. Trauma of invasive procedure a. Cardiac catheterization
b. Echocardiogram
61. Mr. Gonzales develops hepatic encephalopathy. c. Nitroglycerin
Which clinical manifestation is most common d. Percutaneous transluminal coronary
with this condition? angioplasty (PTCA)
a. Increased urine output
b. Altered level of consciousness 66. The nurse is aware that the following terms used
c. Decreased tendon reflex to describe reduced cardiac output and
d. Hypotension perfusion impairment due to ineffective
pumping of the heart is:
62. When Mr. Gonzales regained consciousness, the a. Anaphylactic shock
physician orders 50 ml of Lactose p.o. every 2 b. Cardiogenic shock
hours. Mr. Gozales develops diarrhea. The nurse c. Distributive shock
best action would be: d. Myocardial infarction (MI)
a. “I’ll see if your physician is in the
hospital”. 67. A client with hypertension asks the nurse which
b. “Maybe you’re reacting to the drug; I factors can cause blood pressure to drop to
will withhold the next dose”. normal levels?
c. “I’ll lower the dosage as ordered so the a. Kidneys’ excretion to sodium only.
drug causes only 2 to 4 stools a day”. b. Kidneys’ retention of sodium and water
d. “Frequently, bowel movements are c. Kidneys’ excretion of sodium and water
needed to reduce sodium level”. d. Kidneys’ retention of sodium and
excretion of water
63. Which of the following groups of symptoms
indicates a ruptured abdominal aortic 68. Nurse Rose is aware that the statement that
aneurysm? best explains why furosemide (Lasix) is
a. Lower back pain, increased blood administered to treat hypertension is:
pressure, decreased red blood cell (RBC) a. It dilates peripheral blood vessels.
count, increased white blood (WBC) b. It decreases sympathetic
count. cardioacceleration.
b. Severe lower back pain, decreased c. It inhibits the angiotensin-coverting
blood pressure, decreased RBC count, enzymes
increased WBC count. d. It inhibits reabsorption of sodium and
c. Severe lower back pain, decreased water in the loop of Henle.
blood pressure, decreased RBC count,
69. Nurse Nikki knows that laboratory results 73. JP has been diagnosed with gout and wants to
supports the diagnosis of systemic lupus know why colchicine is used in the treatment of
erythematosus (SLE) is: gout. Which of the following actions of
a. Elavated serum complement level colchicines explains why it’s effective for gout?
b. Thrombocytosis, elevated sedimentation a. Replaces estrogen
rate b. Decreases infection
c. Pancytopenia, elevated antinuclear c. Decreases inflammation
antibody (ANA) titer d. Decreases bone demineralization
d. Leukocysis, elevated blood urea nitrogen
(BUN) and creatinine levels 74. Norma asks for information about osteoarthritis.
Which of the following statements about
70. Arnold, a 19-year-old client with a mild osteoarthritis is correct?
concussion is discharged from the emergency a. Osteoarthritis is rarely debilitating
department. Before discharge, he complains of a b. Osteoarthritis is a rare form of arthritis
headache. When offered acetaminophen, his c. Osteoarthritis is the most common form
mother tells the nurse the headache is severe of arthritis
and she would like her son to have something d. Osteoarthritis afflicts people over 60
stronger. Which of the following responses by
the nurse is appropriate? 75. Ruby is receiving thyroid replacement therapy
a. “Your son had a mild concussion, develops the flu and forgets to take her thyroid
acetaminophen is strong enough.” replacement medicine. The nurse understands
b. “Aspirin is avoided because of the that skipping this medication will put the client
danger of Reye’s syndrome in children or at risk for developing which of the following life-
young adults.” threatening complications?
c. “Narcotics are avoided after a head a. Exophthalmos
injury because they may hide a b. Thyroid storm
worsening condition.” c. Myxedema coma
d. Stronger medications may lead to d. Tibial myxedema
vomiting, which increases the
intracarnial pressure (ICP).” 76. Nurse Sugar is assessing a client with Cushing's
71. When evaluating an arterial blood gas from a syndrome. Which observation should the nurse
male client with a subdural hematoma, the report to the physician immediately?
nurse notes the Paco2 is 30 mm Hg. Which of a. Pitting edema of the legs
the following responses best describes the b. An irregular apical pulse
result? c. Dry mucous membranes
a. Appropriate; lowering carbon dioxide d. Frequent urination
(CO2) reduces intracranial pressure (ICP)
b. Emergent; the client is poorly 77. Cyrill with severe head trauma sustained in a car
oxygenated accident is admitted to the intensive care unit.
c. Normal Thirty-six hours later, the client's urine output
d. Significant; the client has alveolar suddenly rises above 200 ml/hour, leading the
hypoventilation nurse to suspect diabetes insipidus. Which
laboratory findings support the nurse's suspicion
72. When prioritizing care, which of the following of diabetes insipidus?
clients should the nurse Olivia assess first? a. Above-normal urine and serum
a. A 17-year-old client’s 24-hours osmolality levels
postappendectomy b. Below-normal urine and serum
b. A 33-year-old client with a recent osmolality levels
diagnosis of Guillain-Barre syndrome c. Above-normal urine osmolality level,
c. A 50-year-old client 3 days below-normal serum osmolality level
postmyocardial infarction d. Below-normal urine osmolality level,
d. A 50-year-old client with diverticulitis above-normal serum osmolality level

11
11
78. Jomari is diagnosed with hyperosmolar d. Low corticotropin and low cortisol levels
hyperglycemic nonketotic syndrome (HHNS) is
stabilized and prepared for discharge. When 82. A male client is scheduled for a transsphenoidal
preparing the client for discharge and home hypophysectomy to remove a pituitary tumor.
management, which of the following statements Preoperatively, the nurse should assess for
indicates that the client understands her potential complications by doing which of the
condition and how to control it? following?
a. "I can avoid getting sick by not becoming a. Testing for ketones in the urine
dehydrated and by paying attention to b. Testing urine specific gravity
my need to urinate, drink, or eat more c. Checking temperature every 4 hours
than usual." d. Performing capillary glucose testing
b. "If I experience trembling, weakness, every 4 hours
and headache, I should drink a glass of
soda that contains sugar." 83. Capillary glucose monitoring is being performed
c. "I will have to monitor my blood glucose every 4 hours for a client diagnosed with
level closely and notify the physician if diabetic ketoacidosis. Insulin is administered
it's constantly elevated." using a scale of regular insulin according to
d. "If I begin to feel especially hungry and glucose results. At 2 p.m., the client has a
thirsty, I'll eat a snack high in capillary glucose level of 250 mg/dl for which he
carbohydrates." receives 8 U of regular insulin. Nurse Mariner
should expect the dose's:
79. A 66-year-old client has been complaining of a. onset to be at 2 p.m. and its peak to
sleeping more, increased urination, anorexia, be at 3 p.m.
weakness, irritability, depression, and bone pain b. onset to be at 2:15 p.m. and its peak to
that interferes with her going outdoors. Based be at 3 p.m.
on these assessment findings, the nurse would c. onset to be at 2:30 p.m. and its peak to
suspect which of the following disorders? be at 4 p.m.
a. Diabetes mellitus d. onset to be at 4 p.m. and its peak to
b. Diabetes insipidus be at 6 p.m.
c. Hypoparathyroidism
d. Hyperparathyroidism 84. The physician orders laboratory tests to confirm
hyperthyroidism in a female client with classic
80. Nurse Lourdes is teaching a client recovering signs and symptoms of this disorder. Which test
from addisonian crisis about the need to take result would confirm the diagnosis?
fludrocortisone acetate and hydrocortisone at a. No increase in the thyroid-stimulating
home. Which statement by the client indicates hormone (TSH) level after 30 minutes
an understanding of the instructions? during the TSH stimulation test
a. "I'll take my hydrocortisone in the late b. A decreased TSH level
afternoon, before dinner." c. An increase in the TSH level after 30
b. "I'll take all of my hydrocortisone in the minutes during the TSH stimulation test
morning, right after I wake up." d. Below-normal levels of serum
c. "I'll take two-thirds of the dose when I triiodothyronine (T3) and serum
wake up and one-third in the late thyroxine (T4) as detected by
afternoon." radioimmunoassay
d. "I'll take the entire dose at bedtime."
85. Rico with diabetes mellitus must learn how to
81. Which of the following laboratory test results self-administer insulin. The physician has
would suggest to the nurse Len that a client has prescribed 10 U of U-100 regular insulin and 35
a corticotropin-secreting pituitary adenoma? U of U-100 isophane insulin suspension (NPH) to
a. High corticotropin and low cortisol levels be taken before breakfast. When teaching the
b. Low corticotropin and high cortisol levels client how to select and rotate insulin injection
c. High corticotropin and high cortisol sites, the nurse should provide which
levels instruction?
a. "Inject insulin into healthy tissue with a. Adult respiratory distress syndrome
large blood vessels and nerves." (ARDS)
b. "Rotate injection sites within the same b. Atelectasis
anatomic region, not among different c. Bronchitis
regions." d. Pneumonia
c. "Administer insulin into areas of scar
tissue or hypotrophy whenever 91. A 67-year-old client develops acute shortness of
possible." breath and progressive hypoxia requiring right
d. "Administer insulin into sites above femur. The hypoxia was probably caused by
muscles that you plan to exercise heavily which of the following conditions?
later that day." a. Asthma attack
b. Atelectasis
86. Nurse Sarah expects to note an elevated serum c. Bronchitis
glucose level in a client with hyperosmolar d. Fat embolism
hyperglycemic nonketotic syndrome (HHNS).
Which other laboratory finding should the nurse 92. A client with shortness of breath has decreased
anticipate? to absent breath sounds on the right side, from
a. Elevated serum acetone level the apex to the base. Which of the following
b. Serum ketone bodies conditions would best explain this?
c. Serum alkalosis a. Acute asthma
d. Below-normal serum potassium level b. Chronic bronchitis
c. Pneumonia
87. For a client with Graves' disease, which nursing d. Spontaneous pneumothorax
intervention promotes comfort?
a. Restricting intake of oral fluids 93. A 62-year-old male client was in a motor vehicle
b. Placing extra blankets on the client's bed accident as an unrestrained driver. He’s now in
c. Limiting intake of high-carbohydrate the emergency department complaining of
foods difficulty of breathing and chest pain. On
d. Maintaining room temperature in the auscultation of his lung field, no breath sounds
low-normal range are present in the upper lobe. This client may
have which of the following conditions?
88. Patrick is treated in the emergency department a. Bronchitis
for a Colles' fracture sustained during a fall. b. Pneumonia
What is a Colles' fracture? c. Pneumothorax
a. Fracture of the distal radius d. Tuberculosis (TB)
b. Fracture of the olecranon
c. Fracture of the humerus 94. If a client requires a pneumonectomy, what fills
d. Fracture of the carpal scaphoid the area of the thoracic cavity?
a. The space remains filled with air only
89. Cleo is diagnosed with osteoporosis. Which b. The surgeon fills the space with a gel
electrolytes are involved in the development of c. Serous fluids fills the space and
this disorder? consolidates the region
a. Calcium and sodium d. The tissue from the other lung grows
b. Calcium and phosphorous over to the other side
c. Phosphorous and potassium
d. Potassium and sodium 95. Hemoptysis may be present in the client with a
pulmonary embolism because of which of the
90. Johnny a firefighter was involved in following reasons?
extinguishing a house fire and is being treated to a. Alveolar damage in the infracted area
smoke inhalation. He develops severe hypoxia b. Involvement of major blood vessels in
48 hours after the incident, requiring intubation the occluded area
and mechanical ventilation. He most likely has c. Loss of lung parenchyma
developed which of the following conditions? d. Loss of lung tissue

12
12

c. “Every four hours I should remove the


96. Aldo with a massive pulmonary embolism will stockings for a half hour.”
have an arterial blood gas analysis performed to d. “I should put on the stockings before
determine the extent of hypoxia. The acid-base getting out of bed in the morning.”
disorder that may be present is?
a. Metabolic acidosis
b. Metabolic alkalosis
c. Respiratory acidosis
d. Respiratory alkalosis

97. After a motor vehicle accident, Armand an 22-


year-old client is admitted with a pneumothorax.
The surgeon inserts a chest tube and attaches it
to a chest drainage system. Bubbling soon
appears in the water seal chamber. Which of the
following is the most likely cause of the
bubbling?
a. Air leak
b. Adequate suction
c. Inadequate suction
d. Kinked chest tube

98. Nurse Michelle calculates the IV flow rate for a


postoperative client. The client receives 3,000 ml
of Ringer’s lactate solution IV to run over 24
hours. The IV infusion set has a drop factor of 10
drops per milliliter. The nurse should regulate
the client’s IV to deliver how many drops per
minute?
a. 18
b. 21
c. 35
d. 40

99. Mickey, a 6-year-old child with a congenital


heart disorder is admitted with congestive heart
failure. Digoxin (lanoxin) 0.12 mg is ordered for
the child. The bottle of Lanoxin contains .05 mg
of Lanoxin in 1 ml of solution. What amount
should the nurse administer to the child?
a. 1.2 ml
b. 2.4 ml
c. 3.5 ml
d. 4.2 ml

100. Nurse Alexandra teaches a client about elastic


stockings. Which of the following statements,
if made by the client, indicates to the nurse
that the teaching was successful?
a. “I will wear the stockings until the
physician tells me to remove them.”
b. “I should wear the stockings even when I
am sleep.”
Answers and Rationale – Care of Clients with 10. Answer: (D) Upper trunk
Physiologic and Psychosocial Alterations Rationale: The percentage designated for
each burned part of the body using the
1. Answer: (C) Hypertension rule of nines: Head and neck 9%; Right
Rationale: Hypertension, along with fever, upper extremity 9%; Left upper extremity
and tenderness over the grafted kidney, 9%; Anterior trunk 18%; Posterior trunk
reflects acute rejection. 18%; Right lower extremity 18%; Left
2. Answer: (A) Pain lower extremity 18%; Perineum 1%.
Rationale: Sharp, severe pain (renal colic) 11. Answer: (C) Bleeding from ears
radiating toward the genitalia and thigh is Rationale: The nurse needs to perform a
caused by uretheral distention and thorough assessment that could indicate
smooth muscle spasm; relief form pain is alterations in cerebral function, increased
the priority. intracranial pressures, fractures and
3. Answer: (D) Decrease the size and bleeding. Bleeding from the ears occurs
vascularity of the thyroid gland. only with basal skull fractures that can
Rationale: Lugol’s solution provides easily contribute to increased intracranial
iodine, which aids in decreasing the pressure and brain herniation.
vascularity of the thyroid gland, which 12. Answer: (D) may engage in contact sports
limits the risk of hemorrhage when Rationale: The client should be advised by
surgery is performed. the nurse to avoid contact sports. This will
4. Answer: (A) Liver Disease prevent trauma to the area of the
Rationale: The client with liver disease has pacemaker generator.
a decreased ability to metabolize 13. Answer: (A) Oxygen at 1-2L/min is given to
carbohydrates because of a decreased maintain the hypoxic stimulus for
ability to form glycogen (glycogenesis) and breathing.
to form glucose from glycogen. Rationale: COPD causes a chronic CO2
5. Answer: (C) Leukopenia retention that renders the medulla
Rationale: Leukopenia, a reduction in insensitive to the CO2 stimulation for
WBCs, is a systemic effect of breathing. The hypoxic state of the client
chemotherapy as a result of then becomes the stimulus for breathing.
myelosuppression. Giving the client oxygen in low
6. Answer: (C) Avoid foods that in the past concentrations will maintain the client’s
caused flatus. hypoxic drive.
Rationale: Foods that bothered a person 14. Answer: (B) Facilitate ventilation of the
preoperatively will continue to do so after left lung.
a colostomy. Rationale: Since only a partial
7. Answer: (B) Keep the irrigating container pneumonectomy is done, there is a need
less than 18 inches above the stoma.” to promote expansion of this remaining
Rationale: This height permits the solution Left lung by positioning the client on the
to flow slowly with little force so that opposite unoperated side.
excessive peristalsis is not immediately 15. Answer: (A) Food and fluids will be
precipitated. withheld for at least 2 hours.
8. Answer: (A) Administer Kayexalate Rationale: Prior to bronchoscopy, the
Rationale: Kayexalate,a potassium doctors sprays the back of the throat with
exchange resin, permits sodium to be anesthetic to minimize the gag reflex and
exchanged for potassium in the intestine, thus facilitate the insertion of the
reducing the serum potassium level. bronchoscope. Giving the client food and
9. Answer:(B) 28 gtt/min drink after the procedure without
Rationale: This is the correct flow rate; checking on the return of the gag reflex
multiply the amount to be infused (2000 can cause the client to aspirate. The gag
ml) by the drop factor (10) and divide the reflex usually returns after two hours.
result by the amount of time in minutes 16. Answer: (C) hyperkalemia.
(12 hours x 60 minutes)

12
12
Rationale: Hyperkalemia is a common increases serum levels of potassium,
complication of acute renal failure. It's magnesium, and phosphorous, and
life-threatening if immediate action isn't decreases serum levels of calcium. A uric
taken to reverse it. The administration of acid analysis of 3.5 mg/dl falls within the
glucose and regular insulin, with sodium normal range of 2.7 to 7.7 mg/dl; PSP
bicarbonate if necessary, can temporarily excretion of 75% also falls with the normal
prevent cardiac arrest by moving range of 60% to 75%.
potassium into the cells and temporarily 20. Answer: (D) Alteration in the size, shape,
reducing serum potassium levels. and organization of differentiated cells
Hypernatremia, hypokalemia, and Rationale: Dysplasia refers to an alteration
hypercalcemia don't usually occur with in the size, shape, and organization of
acute renal failure and aren't treated with differentiated cells. The presence of
glucose, insulin, or sodium bicarbonate. completely undifferentiated tumor cells
17. Answer: (A) This condition puts her at a that don't resemble cells of the tissues of
higher risk for cervical cancer; therefore, their origin is called anaplasia. An increase
she should have a Papanicolaou (Pap) in the number of normal cells in a normal
smear annually. arrangement in a tissue or an organ is
Rationale: Women with condylomata called hyperplasia. Replacement of one
acuminata are at risk for cancer of the type of fully differentiated cell by another
cervix and vulva. Yearly Pap smears are in tissues where the second type normally
very important for early detection. isn't found is called metaplasia.
Because condylomata acuminata is a 21. Answer: (D) Kaposi's sarcoma
virus, there is no permanent cure. Rationale: Kaposi's sarcoma is the most
Because condylomata acuminata can common cancer associated with AIDS.
occur on the vulva, a condom won't Squamous cell carcinoma, multiple
protect sexual partners. HPV can be myeloma, and leukemia may occur in
transmitted to other parts of the body, anyone and aren't associated specifically
such as the mouth, oropharynx, and with AIDS.
larynx. 22. Answer: (C) To prevent cerebrospinal fluid
18. Answer: (A) The left kidney usually is (CSF) leakage
slightly higher than the right one. Rationale: The client receiving a
Rationale: The left kidney usually is subarachnoid block requires special
slightly higher than the right one. An positioning to prevent CSF leakage and
adrenal gland lies atop each kidney. The headache and to ensure proper anesthetic
average kidney measures approximately distribution. Proper positioning doesn't
11 cm (4-3/8") long, 5 to 5.8 cm (2" to help prevent confusion, seizures, or
2¼") wide, and 2.5 cm (1") thick. The cardiac arrhythmias.
kidneys are located retroperitoneally, in 23. Answer: (A) Auscultate bowel sounds.
the posterior aspect of the abdomen, on Rationale: If abdominal distention is
either side of the vertebral column. They accompanied by nausea, the nurse must
lie between the 12th thoracic and 3rd first auscultate bowel sounds. If bowel
lumbar vertebrae. sounds are absent, the nurse should
19. Answer: (C) Blood urea nitrogen (BUN) suspect gastric or small intestine dilation
100 mg/dl and serum creatinine 6.5mg/dl. and these findings must be reported to
Rationale: The normal BUN level ranges 8 the physician. Palpation should be
to 23 mg/dl; the normal serum creatinine avoided postoperatively with abdominal
level ranges from 0.7 to 1.5 mg/dl. The distention. If peristalsis is absent,
test results in option C are abnormally changing positions and inserting a rectal
elevated, reflecting CRF and the kidneys' tube won't relieve the client's discomfort.
decreased ability to remove nonprotein 24. Answer: (B) Lying on the left side with
nitrogen waste from the blood. CRF knees bent
causes decreased pH and increased Rationale: For a colonoscopy, the nurse
hydrogen ions — not vice versa. CRF also initially should position the client on the
left side with knees bent. Placing the capillaries become occluded, reducing
client on the right side with legs straight, circulation and oxygenation of the tissues
prone with the torso elevated, or bent and resulting in cell death and ulcer
over with hands touching the floor formation. During passive ROM exercises,
wouldn't allow proper visualization of the the nurse moves each joint through its
large intestine. range of movement, which improves joint
25. Answer: (A) Blood supply to the stoma has mobility and circulation to the affected
been interrupted area but doesn't prevent pressure ulcers.
Rationale: An ileostomy stoma forms as Adequate hydration is necessary to
the ileum is brought through the maintain healthy skin and ensure tissue
abdominal wall to the surface skin, repair. A footboard prevents plantar
creating an artificial opening for waste flexion and footdrop by maintaining the
elimination. The stoma should appear foot in a dorsiflexed position.
cherry red, indicating adequate arterial 29. Answer: (C) In long, even, outward, and
perfusion. A dusky stoma suggests downward strokes in the direction of hair
decreased perfusion, which may result growth
from interruption of the stoma's blood Rationale: When applying a topical agent,
supply and may lead to tissue damage or the nurse should begin at the midline and
necrosis. A dusky stoma isn't a normal use long, even, outward, and downward
finding. Adjusting the ostomy bag strokes in the direction of hair growth.
wouldn't affect stoma color, which This application pattern reduces the risk
depends on blood supply to the area. An of follicle irritation and skin inflammation.
intestinal obstruction also wouldn't 30. Answer: (A) Beta -adrenergic blockers
change stoma color. Rationale: Beta-adrenergic blockers work
26. Answer: (A) Applying knee splints by blocking beta receptors in the
Rationale: Applying knee splints prevents myocardium, reducing the response to
leg contractures by holding the joints in a catecholamines and sympathetic nerve
position of function. Elevating the foot of stimulation. They protect the
the bed can't prevent contractures myocardium, helping to reduce the risk of
because this action doesn't hold the joints another infraction by decreasing
in a position of function. Hyperextending a myocardial oxygen demand. Calcium
body part for an extended time is channel blockers reduce the workload of
inappropriate because it can cause the heart by decreasing the heart rate.
contractures. Performing shoulder range- Narcotics reduce myocardial oxygen
of-motion exercises can prevent demand, promote vasodilation, and
contractures in the shoulders, but not in decrease anxiety. Nitrates reduce
the legs. myocardial oxygen consumption bt
27. Answer: (B) Urine output of 20 ml/hour. decreasing left ventricular end diastolic
Rationale: A urine output of less than 40 pressure (preload) and systemic vascular
ml/hour in a client with burns indicates a resistance (afterload).
fluid volume deficit. This client's PaO2 31. Answer: (C) Raised 30 degrees
value falls within the normal range (80 to Rationale: Jugular venous pressure is
100 mm Hg). White pulmonary secretions measured with a centimeter ruler to
also are normal. The client's rectal obtain the vertical distance between the
temperature isn't significantly elevated sternal angle and the point of highest
and probably results from the fluid pulsation with the head of the bed
volume deficit. inclined between 15 to 30 degrees.
28. Answer: (A) Turn him frequently. Increased pressure can’t be seen when
Rationale: The most important the client is supine or when the head of
intervention to prevent pressure ulcers is the bed is raised 10 degrees because the
frequent position changes, which relieve point that marks the pressure level is
pressure on the skin and underlying above the jaw (therefore, not visible). In
tissues. If pressure isn't relieved,

12
12
high Fowler’s position, the veins would be ventricular fibrillation and atrial flutter –
barely discernible above the clavicle. not symptomatic bradycardia.
32. Answer: (D) Inotropic agents 36. Answer: (C) 95 mm Hg
Rationale: Inotropic agents are Rationale: Use the following formula to
administered to increase the force of the calculate MAP
heart’s contractions, thereby increasing MAP = systolic + 2 (diastolic)
ventricular contractility and ultimately 3
increasing cardiac output. Beta-adrenergic MAP=126 mm Hg + 2 (80 mm Hg)
blockers and calcium channel blockers 3
decrease the heart rate and ultimately MAP=286 mm HG
decreased the workload of the heart. 3
Diuretics are administered to decrease the MAP=95 mm Hg
overall vascular volume, also decreasing 37. Answer: (C) Electrocardiogram, complete
the workload of the heart. blood count, testing for occult blood,
33. Answer: (B) Less than 30% of calories from comprehensive serum metabolic panel.
fat Rationale: An electrocardiogram evaluates
Rationale: A client with low serum HDL the complaints of chest pain, laboratory
and high serum LDL levels should get less tests determines anemia, and the stool
than 30% of daily calories from fat. The test for occult blood determines blood in
other modifications are appropriate for the stool. Cardiac monitoring, oxygen, and
this client. creatine kinase and lactate
34. Answer: (C) The emergency department dehydrogenase levels are appropriate for
nurse calls up the latest electrocardiogram a cardiac primary problem. A basic
results to check the client’s progress metabolic panel and alkaline phosphatase
Rationale: The emergency department and aspartate aminotransferase levels
nurse is no longer directly involved with assess liver function. Prothrombin time,
the client’s care and thus has no legal partial thromboplastin time, fibrinogen
right to information about his present and fibrin split products are measured to
condition. Anyone directly involved in his verify bleeding dyscrasias; an
care (such as the telemetry nurse and the electroencephalogram evaluates brain
on-call physician) has the right to electrical activity.
information about his condition. Because 38. Answer: (D) Heparin-associated
the client requested that the nurse update thrombosis and thrombocytopenia (HATT)
his wife on his condition, doing so doesn’t Rationale: HATT may occur after CABG
breach confidentiality. surgery due to heparin use during surgery.
35. Answer: (B) Check endotracheal tube Although DIC and ITP cause platelet
placement. aggregation and bleeding, neither is
Rationale: ET tube placement should be common in a client after revascularization
confirmed as soon as the client arrives in surgery. Pancytopenia is a reduction in all
the emergency department. Once the blood cells.
airways is secured, oxygenation and 39. Answer: (B) Corticosteroids
ventilation should be confirmed using an Rationale: Corticosteroid therapy can
end-tidal carbon dioxide monitor and decrease antibody production and
pulse oximetry. Next, the nurse should phagocytosis of the antibody-coated
make sure L.V. access is established. If the platelets, retaining more functioning
client experiences symptomatic platelets. Methotrexate can cause
bradycardia, atropine is administered as thrombocytopenia. Vitamin K is used to
ordered 0.5 to 1 mg every 3 to 5 minutes treat an excessive anticoagulate state
to a total of 3 mg. Then the nurse should from warfarin overload, and ASA
try to find the cause of the client’s arrest decreases platelet aggregation.
by obtaining an ABG sample. Amiodarone 40. Answer: (D) Xenogeneic
is indicated for ventricular tachycardia, Rationale: An xenogeneic transplant is
between is between human and another
species. A syngeneic transplant is between options, which reflect parts of the nervous
identical twins, allogeneic transplant is system, aren’t usually affected by MM.
between two humans, and autologous is a 46. Answer: (C) 10 years
transplant from the same individual. Rationale: Epidermiologic studies show
41. Answer: (B) the average time from initial contact with
Rationale: Tissue thromboplastin is HIV to the development of AIDS is 10
released when damaged tissue comes in years.
contact with clotting factors. Calcium is 47. Answer: (A) Low platelet count
released to assist the conversion of Rationale: In DIC, platelets and clotting
factors X to Xa. Conversion of factors XII to factors are consumed, resulting in
XIIa and VIII to IIIa are part of the intrinsic microthrombi and excessive bleeding. As
pathway. clots form, fibrinogen levels decrease and
42. Answer: (C) Essential thrombocytopenia the prothrombin time increases. Fibrin
Rationale: Essential thrombocytopenia is degeneration products increase as
linked to immunologic disorders, such as fibrinolysis takes places.
SLE and human immunodeficiency virus. 48. Answer: (D) Hodgkin’s disease
The disorder known as von Willebrand’s Rationale: Hodgkin’s disease typically
disease is a type of hemophilia and isn’t causes fever night sweats, weight loss,
linked to SLE. Moderate to severe anemia and lymph mode enlargement. Influenza
is associated with SLE, not polycythemia. doesn’t last for months. Clients with sickle
Dressler’s syndrome is pericarditis that cell anemia manifest signs and symptoms
occurs after a myocardial infarction and of chronic anemia with pallor of the
isn’t linked to SLE. mucous membrane, fatigue, and
43. Answer: (B) Night sweat decreased tolerance for exercise; they
Rationale: In stage 1, symptoms include a don’t show fever, night sweats, weight
single enlarged lymph node (usually), loss or lymph node enlargement.
unexplained fever, night sweats, malaise, Leukemia doesn’t cause lymph node
and generalized pruritis. Although enlargement.
splenomegaly may be present in some 49. Answer: (C) A Rh-negative
clients, night sweats are generally more Rationale: Human blood can sometimes
prevalent. Pericarditis isn’t associated contain an inherited D antigen. Persons
with Hodgkin’s disease, nor is with the D antigen have Rh-positive blood
hypothermia. Moreover, splenomegaly type; those lacking the antigen have Rh-
and pericarditis aren’t symptoms. negative blood. It’s important that a
Persistent hypothermia is associated with person with Rh- negative blood receives
Hodgkin’s but isn’t an early sign of the Rh-negative blood. If Rh-positive blood is
disease. administered to an Rh-negative person,
44. Answer: (D) Breath sounds the recipient develops anti-Rh agglutinins,
Rationale: Pneumonia, both viral and and sub sequent transfusions with Rh-
fungal, is a common cause of death in positive blood may cause serious
clients with neutropenia, so frequent reactions with clumping and hemolysis of
assessment of respiratory rate and breath red blood cells.
sounds is required. Although assessing 50. Answer: (B) “I will call my doctor if Stacy
blood pressure, bowel sounds, and heart has persistent vomiting and diarrhea”.
sounds is important, it won’t help detect Rationale: Persistent (more than 24 hours)
pneumonia. vomiting, anorexia, and diarrhea are signs
45. Answer: (B) Muscle spasm of toxicity and the patient should stop the
Rationale: Back pain or paresthesia in the medication and notify the health care
lower extremities may indicate impending provider. The other manifestations are
spinal cord compression from a spinal expected side effects of chemotherapy.
tumor. This should be recognized and 51. Answer: (D) “This is only temporary; Stacy
treated promptly as progression of the will re-grow new hair in 3-6 months, but
tumor may result in paraplegia. The other may be different in texture”.

12
12
Rationale: This is the appropriate chronic obstructive bronchitis are bloated
response. The nurse should help the and cyanotic in appearance.
mother how to cope with her own feelings 56. Answer: D 80 mm Hg
regarding the child’s disease so as not to Rationale: A client about to go into
affect the child negatively. When the hair respiratory arrest will have inefficient
grows back, it is still of the same color and ventilation and will be retaining carbon
texture. dioxide. The value expected would be
52. Answer: (B) Apply viscous Lidocaine to around 80 mm Hg. All other values are
oral ulcers as needed. lower than expected.
Rationale: Stomatitis can cause pain and 57. Answer: (C) Respiratory acidosis
this can be relieved by applying topical Rationale: Because Paco2 is high at 80 mm
anesthetics such as lidocaine before Hg and the metabolic measure, HCO3- is
mouth care. When the patient is already normal, the client has respiratory acidosis.
comfortable, the nurse can proceed with The pH is less than 7.35, academic, which
providing the patient with oral rinses of eliminates metabolic and respiratory
saline solution mixed with equal part of alkalosis as possibilities. If the HCO3- was
water or hydrogen peroxide mixed water below 22 mEq/L the client would have
in 1:3 concentrations to promote oral metabolic acidosis.
hygiene. Every 2-4 hours. 58. Answer: (C) Respiratory failure
53. Answer: (C) Immediately discontinue the Rationale: The client was reacting to the
infusion drug with respiratory signs of impending
Rationale: Edema or swelling at the IV site anaphylaxis, which could lead to
is a sign that the needle has been eventually respiratory failure. Although
dislodged and the IV solution is leaking the signs are also related to an asthma
into the tissues causing the edema. The attack or a pulmonary embolism, consider
patient feels pain as the nerves are the new drug first. Rheumatoid arthritis
irritated by pressure and the IV solution. doesn’t manifest these signs.
The first action of the nurse would be to 59. Answer: (D) Elevated serum
discontinue the infusion right away to aminotransferase
prevent further edema and other Rationale: Hepatic cell death causes
complication. release of liver enzymes alanine
54. Answer: (C) Chronic obstructive bronchitis aminotransferase (ALT), aspartate
Rationale: Clients with chronic obstructive aminotransferase (AST) and lactate
bronchitis appear bloated; they have large dehydrogenase (LDH) into the circulation.
barrel chest and peripheral edema, Liver cirrhosis is a chronic and irreversible
cyanotic nail beds, and at times, disease of the liver characterized by
circumoral cyanosis. Clients with ARDS are generalized inflammation and fibrosis of
acutely short of breath and frequently the liver tissues.
need intubation for mechanical ventilation 60. Answer: (A) Impaired clotting mechanism
and large amount of oxygen. Clients with Rationale: Cirrhosis of the liver results in
asthma don’t exhibit characteristics of decreased Vitamin K absorption and
chronic disease, and clients with formation of clotting factors resulting in
emphysema appear pink and cachectic. impaired clotting mechanism.
55. Answer: (D) Emphysema 61. Answer: (B) Altered level of consciousness
Rationale: Because of the large amount of Rationale: Changes in behavior and level
energy it takes to breathe, clients with of consciousness are the first sins of
emphysema are usually cachectic. They’re hepatic encephalopathy. Hepatic
pink and usually breathe through pursed encephalopathy is caused by liver failure
lips, hence the term “puffer.” Clients with and develops when the liver is unable to
ARDS are usually acutely short of breath. convert protein metabolic product
Clients with asthma don’t have any ammonia to urea. This results in
particular characteristics, and clients with accumulation of ammonia and other toxic
in the blood that damages the cells.
62. Answer: (C) “I’ll lower the dosage as diagnosis test. Nitroglycerin is an oral
ordered so the drug causes only 2 to 4 sublingual medication. Cardiac
stools a day”. catheterization is a diagnostic tool – not a
Rationale: Lactulose is given to a patients treatment.
with hepatic encephalopathy to reduce 66. Answer: (B) Cardiogenic shock
absorption of ammonia in the intestines Rationale: Cardiogenic shock is shock
by binding with ammonia and promoting related to ineffective pumping of the
more frequent bowel movements. If the heart. Anaphylactic shock results from an
patient experience diarrhea, it indicates allergic reaction. Distributive shock results
over dosage and the nurse must reduce from changes in the intravascular volume
the amount of medication given to the distribution and is usually associated with
patient. The stool will be mashy or soft. increased cardiac output. MI isn’t a shock
Lactulose is also very sweet and may state, though a severe MI can lead to
cause cramping and bloating. shock.
63. Answer: (B) Severe lower back pain, 67. Answer: (C) Kidneys’ excretion of sodium
decreased blood pressure, decreased RBC and water
count, increased WBC count. Rationale: The kidneys respond to rise in
Rationale: Severe lower back pain blood pressure by excreting sodium and
indicates an aneurysm rupture, secondary excess water. This response ultimately
to pressure being applied within the affects sysmolic blood pressure by
abdominal cavity. When ruptured occurs, regulating blood volume. Sodium or water
the pain is constant because it can’t be retention would only further increase
alleviated until the aneurysm is repaired. blood pressure. Sodium and water travel
Blood pressure decreases due to the loss together across the membrane in the
of blood. After the aneurysm ruptures, the kidneys; one can’t travel without the
vasculature is interrupted and blood other.
volume is lost, so blood pressure wouldn’t 68. Answer: (D) It inhibits reabsorption of
increase. For the same reason, the RBC sodium and water in the loop of Henle.
count is decreased – not increased. The Rationale: Furosemide is a loop diuretic
WBC count increases as cell migrate to the that inhibits sodium and water
site of injury. reabsorption in the loop Henle, thereby
64. Answer: (D) Apply gloves and assess the causing a decrease in blood pressure.
groin site Vasodilators cause dilation of peripheral
Rationale: Observing standard precautions blood vessels, directly relaxing vascular
is the first priority when dealing with any smooth muscle and decreasing blood
blood fluid. Assessment of the groin site is pressure. Adrenergic blockers decrease
the second priority. This establishes where sympathetic cardioacceleration and
the blood is coming from and determines decrease blood pressure. Angiotensin-
how much blood has been lost. The goal in converting enzyme inhibitors decrease
this situation is to stop the bleeding. The blood pressure due to their action on
nurse would call for help if it were angiotensin.
warranted after the assessment of the 69. Answer: (C) Pancytopenia, elevated
situation. After determining the extent of antinuclear antibody (ANA) titer
the bleeding, vital signs assessment is Rationale: Laboratory findings for clients
important. The nurse should never move with SLE usually show pancytopenia,
the client, in case a clot has formed. elevated ANA titer, and decreased serum
Moving can disturb the clot and cause complement levels. Clients may have
rebleeding. elevated BUN and creatinine levels from
65. Answer: (D) Percutaneous transluminal nephritis, but the increase does not
coronary angioplasty (PTCA) indicate SLE.
Rationale: PTCA can alleviate the blockage 70. Answer: (C) Narcotics are avoided after a
and restore blood flow and oxygenation. head injury because they may hide a
An echocardiogram is a noninvasive worsening condition.

12
12
Rationale: Narcotics may mask changes in 75. Answer: (C) Myxedema coma
the level of consciousness that indicate Rationale: Myxedema coma, severe
increased ICP and shouldn’t hypothyroidism, is a life-threatening
acetaminophen is strong enough ignores condition that may develop if thyroid
the mother’s question and therefore isn’t replacement medication isn't taken.
appropriate. Aspirin is contraindicated in Exophthalmos, protrusion of the eyeballs,
conditions that may have bleeding, such is seen with hyperthyroidism. Thyroid
as trauma, and for children or young storm is life-threatening but is caused by
adults with viral illnesses due to the severe hyperthyroidism. Tibial myxedema,
danger of Reye’s syndrome. Stronger peripheral mucinous edema involving the
medications may not necessarily lead to lower leg, is associated with
vomiting but will sedate the client, hypothyroidism but isn't life-threatening.
thereby masking changes in his level of 76. Answer: (B) An irregular apical pulse
consciousness. Rationale: Because Cushing's syndrome
71. Answer: (A) Appropriate; lowering carbon causes aldosterone overproduction, which
dioxide (CO2) reduces intracranial increases urinary potassium loss, the
pressure (ICP) disorder may lead to hypokalemia.
Rationale: A normal Paco2 value is 35 to Therefore, the nurse should immediately
45 mm Hg CO2 has vasodilating report signs and symptoms of
properties; therefore, lowering Paco2 hypokalemia, such as an irregular apical
through hyperventilation will lower ICP pulse, to the physician. Edema is an
caused by dilated cerebral vessels. expected finding because aldosterone
Oxygenation is evaluated through Pao2 overproduction causes sodium and fluid
and oxygen saturation. Alveolar retention. Dry mucous membranes and
hypoventilation would be reflected in an frequent urination signal dehydration,
increased Paco2. which isn't associated with Cushing's
72. Answer: (B) A 33-year-old client with a syndrome.
recent diagnosis of Guillain-Barre 77. Answer: (D) Below-normal urine
syndrome osmolality level, above-normal serum
Rationale: Guillain-Barre syndrome is osmolality level
characterized by ascending paralysis and Rationale: In diabetes insipidus, excessive
potential respiratory failure. The order of polyuria causes dilute urine, resulting in a
client assessment should follow client below-normal urine osmolality level. At
priorities, with disorder of airways, the same time, polyuria depletes the body
breathing, and then circulation. There’s no of water, causing dehydration that leads
information to suggest the postmyocardial to an above-normal serum osmolality
infarction client has an arrhythmia or level. For the same reasons, diabetes
other complication. There’s no evidence insipidus doesn't cause above-normal
to suggest hemorrhage or perforation for urine osmolality or below-normal serum
the remaining clients as a priority of care. osmolality levels.
73. Answer: (C) Decreases inflammation 78. Answer: (A) "I can avoid getting sick by not
Rationale: Then action of colchicines is to becoming dehydrated and by paying
decrease inflammation by reducing the attention to my need to urinate, drink, or
migration of leukocytes to synovial fluid. eat more than usual."
Colchicine doesn’t replace estrogen, Rationale: Inadequate fluid intake during
decrease infection, or decrease bone hyperglycemic episodes often leads to
demineralization. HHNS. By recognizing the signs of
74. Answer: (C) Osteoarthritis is the most hyperglycemia (polyuria, polydipsia, and
common form of arthritis polyphagia) and increasing fluid intake,
Rationale: Osteoarthritis is the most the client may prevent HHNS. Drinking a
common form of arthritis and can be glass of nondiet soda would be
extremely debilitating. It can afflict people appropriate for hypoglycemia. A client
of any age, although most are elderly. whose diabetes is controlled with oral
antidiabetic agents usually doesn't need dangerously imbalanced. Temperature
to monitor blood glucose levels. A high- regulation may be affected by excess
carbohydrate diet would exacerbate the cortisol and isn't an accurate indicator of
client's condition, particularly if fluid infection.
intake is low. 83. Answer: (C) onset to be at 2:30 p.m. and
79. Answer: (D) Hyperparathyroidism its peak to be at 4 p.m.
Rationale: Hyperparathyroidism is most Rationale: Regular insulin, which is a
common in older women and is short-acting insulin, has an onset of 15 to
characterized by bone pain and weakness 30 minutes and a peak of 2 to 4 hours.
from excess parathyroid hormone (PTH). Because the nurse gave the insulin at 2
Clients also exhibit hypercaliuria-causing p.m., the expected onset would be from
polyuria. While clients with diabetes 2:15 p.m. to 2:30 p.m. and the peak from
mellitus and diabetes insipidus also have 4 p.m. to 6 p.m.
polyuria, they don't have bone pain and 84. Answer: (A) No increase in the thyroid-
increased sleeping. Hypoparathyroidism is stimulating hormone (TSH) level after 30
characterized by urinary frequency rather minutes during the TSH stimulation test
than polyuria. Rationale: In the TSH test, failure of the
80. Answer: (C) "I'll take two-thirds of the TSH level to rise after 30 minutes confirms
dose when I wake up and one-third in the hyperthyroidism. A decreased TSH level
late afternoon." indicates a pituitary deficiency of this
Rationale: Hydrocortisone, a hormone. Below-normal levels of T3 and
glucocorticoid, should be administered T4, as detected by radioimmunoassay,
according to a schedule that closely signal hypothyroidism. A below-normal T4
reflects the bodies own secretion of this level also occurs in malnutrition and liver
hormone; therefore, two-thirds of the disease and may result from
dose of hydrocortisone should be taken in administration of phenytoin and certain
the morning and one-third in the late other drugs.
afternoon. This dosage schedule reduces 85. Answer: (B) "Rotate injection sites within
adverse effects. the same anatomic region, not among
81. Answer: (C) High corticotropin and high different regions."
cortisol levels Rationale: The nurse should instruct the
Rationale: A corticotropin-secreting client to rotate injection sites within the
pituitary tumor would cause high same anatomic region. Rotating sites
corticotropin and high cortisol levels. A among different regions may cause
high corticotropin level with a low cortisol excessive day-to-day variations in the
level and a low corticotropin level with a blood glucose level; also, insulin
low cortisol level would be associated absorption differs from one region to the
with hypocortisolism. Low corticotropin next. Insulin should be injected only into
and high cortisol levels would be seen if healthy tissue lacking large blood vessels,
there was a primary defect in the adrenal nerves, or scar tissue or other deviations.
glands. Injecting insulin into areas of hypertrophy
82. Answer: (D) Performing capillary glucose may delay absorption. The client shouldn't
testing every 4 hours inject insulin into areas of lipodystrophy
Rationale: The nurse should perform (such as hypertrophy or atrophy); to
capillary glucose testing every 4 hours prevent lipodystrophy, the client should
because excess cortisol may cause insulin rotate injection sites systematically.
resistance, placing the client at risk for Exercise speeds drug absorption, so the
hyperglycemia. Urine ketone testing isn't client shouldn't inject insulin into sites
indicated because the client does secrete above muscles that will be exercised
insulin and, therefore, isn't at risk for heavily.
ketosis. Urine specific gravity isn't 86. Answer: (D) Below-normal serum
indicated because although fluid balance potassium level
can be compromised, it usually isn't

13
13
Rationale: A client with HHNS has an He could develop atelectasis but it
overall body deficit of potassium resulting typically doesn’t produce progressive
from diuresis, which occurs secondary to hypoxia.
the hyperosmolar, hyperglycemic state 92. Answer: (D) Spontaneous pneumothorax
caused by the relative insulin deficiency. Rationale: A spontaneous pneumothorax
An elevated serum acetone level and occurs when the client’s lung collapses,
serum ketone bodies are characteristic of causing an acute decreased in the amount
diabetic ketoacidosis. Metabolic acidosis, of functional lung used in oxygenation.
not serum alkalosis, may occur in HHNS. The sudden collapse was the cause of his
87. Answer: (D) Maintaining room chest pain and shortness of breath. An
temperature in the low-normal range asthma attack would show wheezing
Rationale: Graves' disease causes signs breath sounds, and bronchitis would have
and symptoms of hypermetabolism, such rhonchi. Pneumonia would have bronchial
as heat intolerance, diaphoresis, excessive breath sounds over the area of
thirst and appetite, and weight loss. To consolidation.
reduce heat intolerance and diaphoresis, 93. Answer: (C) Pneumothorax
the nurse should keep the client's room Rationale: From the trauma the client
temperature in the low-normal range. To experienced, it’s unlikely he has
replace fluids lost via diaphoresis, the bronchitis, pneumonia, or TB; rhonchi
nurse should encourage, not restrict, with bronchitis, bronchial breath sounds
intake of oral fluids. Placing extra blankets with TB would be heard.
on the bed of a client with heat 94. Answer: (C) Serous fluids fills the space
intolerance would cause discomfort. To and consolidates the region
provide needed energy and calories, the Rationale: Serous fluid fills the space and
nurse should encourage the client to eat eventually consolidates, preventing
high-carbohydrate foods. extensive mediastinal shift of the heart
88. Answer: (A) Fracture of the distal radius and remaining lung. Air can’t be left in the
Rationale: Colles' fracture is a fracture of space. There’s no gel that can be placed in
the distal radius, such as from a fall on an the pleural space. The tissue from the
outstretched hand. It's most common in other lung can’t cross the mediastinum,
women. Colles' fracture doesn't refer to a although a temporary mediastinal shift
fracture of the olecranon, humerus, or exits until the space is filled.
carpal scaphoid. 95. Answer: (A) Alveolar damage in the
89. Answer: (B) Calcium and phosphorous infracted area
Rationale: In osteoporosis, bones lose Rationale: The infracted area produces
calcium and phosphate salts, becoming alveolar damage that can lead to the
porous, brittle, and abnormally vulnerable production of bloody sputum, sometimes
to fracture. Sodium and potassium aren't in massive amounts. Clot formation
involved in the development of usually occurs in the legs. There’s a loss of
steoporosis. lung parenchyma and subsequent scar
90. Answer: (A) Adult respiratory distress tissue formation.
syndrome (ARDS) 96. Answer: (D) Respiratory alkalosis
Rationale: Severe hypoxia after smoke Rationale: A client with massive
inhalation is typically related to ARDS. The pulmonary embolism will have a large
other conditions listed aren’t typically region and blow off large amount of
associated with smoke inhalation and carbon dioxide, which crosses the
severe hypoxia. unaffected alveolar-capillary membrane
91. Answer: (D) Fat embolism more readily than does oxygen and results
Rationale: Long bone fractures are in respiratory alkalosis.
correlated with fat emboli, which cause 97. Answer: (A) Air leak
shortness of breath and hypoxia. It’s Rationale: Bubbling in the water seal
unlikely the client has developed asthma chamber of a chest drainage system stems
or bronchitis without a previous history. from an air leak. In pneumothorax an air
leak can occur as air is pulled from the
pleural space. Bubbling doesn’t normally
occur with either adequate or inadequate
suction or any preexisting bubbling in the
water seal chamber.
98. Answer: (B) 21
Rationale: 3000 x 10 divided by 24 x 60.
99. Answer: (B) 2.4 ml
Rationale: .05 mg/ 1 ml = .12mg/ x ml,
.05x = .12, x = 2.4 ml.
100. Answer: (D) “I should put on the stockings
before getting out of bed in the morning.
Rationale: Promote venous return by
applying external pressure on veins.

13
13
TEST V - Care of Clients with Physiologic and d. Suggest that the father and son work
Psychosocial Alterations things out.

1. Mr. Marquez reports of losing his job, not being 5. What is Nurse John likely to note in a male client
able to sleep at night, and feeling upset with his being admitted for alcohol withdrawal?
wife. Nurse John responds to the client, “You a. Perceptual disorders.
may want to talk about your employment b. Impending coma.
situation in group today.” The Nurse is using c. Recent alcohol intake.
which therapeutic technique? d. Depression with mutism.
a. Observations
b. Restating 6. Aira has taken amitriptyline HCL (Elavil) for 3
c. Exploring days, but now complains that it “doesn’t help”
d. Focusing and refuses to take it. What should the nurse say
or do?
2. Tony refuses his evening dose of Haloperidol a. Withhold the drug.
(Haldol), then becomes extremely agitated in the b. Record the client’s response.
dayroom while other clients are watching c. Encourage the client to tell the doctor.
television. He begins cursing and throwing d. Suggest that it takes a while before
furniture. Nurse Oliver first action is to: seeing the results.
a. Check the client’s medical record for an
order for an as-needed I.M. dose of 7. Dervid, an adolescent has a history of truancy
medication for agitation. from school, running away from home and
b. Place the client in full leather restraints. “barrowing” other people’s things without their
c. Call the attending physician and report permission. The adolescent denies stealing,
the behavior. rationalizing instead that as long as no one was
d. Remove all other clients from the using the items, it was all right to borrow them.
dayroom. It is important for the nurse to understand the
psychodynamically, this behavior may be largely
3. Tina who is manic, but not yet on medication, attributed to a developmental defect related to
comes to the drug treatment center. The nurse the:
would not let this client join the group session a. Id
because: b. Ego
a. The client is disruptive. c. Superego
b. The client is harmful to self. d. Oedipal complex
c. The client is harmful to others.
d. The client needs to be on medication 8. In preparing a female client for electroconvulsive
first. therapy (ECT), Nurse Michelle knows that
succinylcoline (Anectine) will be administered
4. Dervid, an adolescent boy was admitted for for which therapeutic effect?
substance abuse and hallucinations. The client’s a. Short-acting anesthesia
mother asks Nurse Armando to talk with his b. Decreased oral and respiratory
husband when he arrives at the hospital. The secretions.
mother says that she is afraid of what the father c. Skeletal muscle paralysis.
might say to the boy. The most appropriate d. Analgesia.
nursing intervention would be to:
a. Inform the mother that she and the 9. Nurse Gina is aware that the dietary implications
father can work through this problem for a client in manic phase of bipolar disorder is:
themselves. a. Serve the client a bowl of soup, buttered
b. Refer the mother to the hospital social French bread, and apple slices.
worker. b. Increase calories, decrease fat, and
c. Agree to talk with the mother and the decrease protein.
father together. c. Give the client pieces of cut-up steak,
carrots, and an apple.
d. Increase calories, carbohydrates, and deal with this conflict if you want to walk
protein. again."
b. "It must be awful not to be able to move
10. What parental behavior toward a child during an your legs. You may feel better if you
admission procedure should cause Nurse Ron to realize the problem is psychological, not
suspect child abuse? physical."
a. Flat affect c. "Your problem is real but there is no
b. Expressing guilt physical basis for it. We'll work on what
c. Acting overly solicitous toward the child. is going on in your life to find out why
d. Ignoring the child. it's happened."
d. "It isn't uncommon for someone with
11. Nurse Lynnette notices that a female client with your personality to develop a conversion
obsessive-compulsive disorder washes her hands disorder during times of stress."
for long periods each day. How should the nurse
respond to this compulsive behavior? 14. Nurse Krina knows that the following drugs have
a. By designating times during which the been known to be effective in treating
client can focus on the behavior. obsessive-compulsive disorder (OCD):
b. By urging the client to reduce the a. benztropine (Cogentin) and
frequency of the behavior as rapidly as diphenhydramine (Benadryl).
possible. b. chlordiazepoxide (Librium) and
c. By calling attention to or attempting to diazepam (Valium)
prevent the behavior. c. fluvoxamine (Luvox) and clomipramine
d. By discouraging the client from (Anafranil)
verbalizing anxieties. d. divalproex (Depakote) and lithium
(Lithobid)
12. After seeking help at an outpatient mental
health clinic, Ruby who was raped while walking 15. Alfred was newly diagnosed with anxiety
her dog is diagnosed with posttraumatic stress disorder. The physician prescribed buspirone
disorder (PTSD). Three months later, Ruby (BuSpar). The nurse is aware that the teaching
returns to the clinic, complaining of fear, loss of instructions for newly prescribed buspirone
control, and helpless feelings. Which nursing should include which of the following?
intervention is most appropriate for Ruby? a. A warning about the drugs delayed
a. Recommending a high-protein, low-fat therapeutic effect, which is from 14 to
diet. 30 days.
b. Giving sleep medication, as prescribed, b. A warning about the incidence of
to restore a normal sleep- wake cycle. neuroleptic malignant syndrome (NMS).
c. Allowing the client time to heal. c. A reminder of the need to schedule
d. Exploring the meaning of the traumatic blood work in 1 week to check blood
event with the client. levels of the drug.
d. A warning that immediate sedation can
13. Meryl, age 19, is highly dependent on her occur with a resultant drop in pulse.
parents and fears leaving home to go away to
college. Shortly before the semester starts, she 16. Richard with agoraphobia has been symptom-
complains that her legs are paralyzed and is free for 4 months. Classic signs and symptoms
rushed to the emergency department. When of phobias include:
physical examination rules out a physical cause a. Insomnia and an inability to concentrate.
for her paralysis, the physician admits her to the b. Severe anxiety and fear.
psychiatric unit where she is diagnosed with c. Depression and weight loss.
conversion disorder. Meryl asks the nurse, "Why d. Withdrawal and failure to distinguish
has this happened to me?" What is the nurse's reality from fantasy.
best response?
a. "You've developed this paralysis so you 17. Which medications have been found to help
can stay with your parents. You must reduce or eliminate panic attacks?

13
13
a. Antidepressants d. A low tolerance for frustration
b. Anticholinergics
c. Antipsychotics 22. Nurse Amy is providing care for a male client
d. Mood stabilizers undergoing opiate withdrawal. Opiate
withdrawal causes severe physical discomfort
18. A client seeks care because she feels depressed and can be life-threatening. To minimize these
and has gained weight. To treat her atypical effects, opiate users are commonly detoxified
depression, the physician prescribes with:
tranylcypromine sulfate (Parnate), 10 mg by a. Barbiturates
mouth twice per day. When this drug is used to b. Amphetamines
treat atypical depression, what is its onset of c. Methadone
action? d. Benzodiazepines
a. 1 to 2 days
b. 3 to 5 days 23. Nurse Cristina is caring for a client who
c. 6 to 8 days experiences false sensory perceptions with no
d. 10 to 14 days basis in reality. These perceptions are known as:
a. Delusions
19. A 65 years old client is in the first stage of b. Hallucinations
Alzheimer's disease. Nurse Patricia should plan c. Loose associations
to focus this client's care on: d. Neologisms
a. Offering nourishing finger foods to help
maintain the client's nutritional status. 24. Nurse Marco is developing a plan of care for a
b. Providing emotional support and client with anorexia nervosa. Which action
individual counseling. should the nurse include in the plan?
c. Monitoring the client to prevent minor a. Restricts visits with the family and
illnesses from turning into major friends until the client begins to eat.
problems. b. Provide privacy during meals.
d. Suggesting new activities for the client c. Set up a strict eating plan for the client.
and family to do together. d. Encourage the client to exercise, which
will reduce her anxiety.
20. The nurse is assessing a client who has just been
admitted to the emergency department. Which 25. Tim is admitted with a diagnosis of delusions of
signs would suggest an overdose of an grandeur. The nurse is aware that this diagnosis
antianxiety agent? reflects a belief that one is:
a. Combativeness, sweating, and confusion a. Highly important or famous.
b. Agitation, hyperactivity, and grandiose b. Being persecuted
ideation c. Connected to events unrelated to
c. Emotional lability, euphoria, and oneself
impaired memory d. Responsible for the evil in the world.
d. Suspiciousness, dilated pupils, and
increased blood pressure 26. Nurse Jen is caring for a male client with manic
depression. The plan of care for a client in a
21. The nurse is caring for a client diagnosed with manic state would include:
antisocial personality disorder. The client has a a. Offering a high-calorie meals and
history of fighting, cruelty to animals, and strongly encouraging the client to finish
stealing. Which of the following traits would the all food.
nurse be most likely to uncover during b. Insisting that the client remain active
assessment? through the day so that he’ll sleep at
a. History of gainful employment night.
b. Frequent expression of guilt regarding c. Allowing the client to exhibit
antisocial behavior hyperactive, demanding, manipulative
c. Demonstrated ability to maintain close, behavior without setting limits.
stable relationships
d. Listening attentively with a neutral 32. The nurse is aware that the side effect of
attitude and avoiding power struggles. electroconvulsive therapy that a client may
experience:
27. Ramon is admitted for detoxification after a a. Loss of appetite
cocaine overdose. The client tells the nurse that b. Postural hypotension
he frequently uses cocaine but that he can c. Confusion for a time after treatment
control his use if he chooses. Which coping d. Complete loss of memory for a time
mechanism is he using? 33. A dying male client gradually moves toward
a. Withdrawal resolution of feelings regarding impending
b. Logical thinking death. Basing care on the theory of Kubler-Ross,
c. Repression Nurse Trish plans to use nonverbal interventions
d. Denial when assessment reveals that the client is in the:
a. Anger stage
28. Richard is admitted with a diagnosis of b. Denial stage
schizotypal personality disorder. hich signs c. Bargaining stage
would this client exhibit during social situations? d. Acceptance stage
a. Aggressive behavior
b. Paranoid thoughts 34. The outcome that is unrelated to a crisis state is:
c. Emotional affect a. Learning more constructive coping skills
d. Independence needs b. Decompensation to a lower level of
functioning.
29. Nurse Mickey is caring for a client diagnosed c. Adaptation and a return to a prior level
with bulimia. The most appropriate initial goal of functioning.
for a client diagnosed with bulimia is to: d. A higher level of anxiety continuing for
a. Avoid shopping for large amounts of more than 3 months.
food.
b. Control eating impulses. 35. Miranda a psychiatric client is to be discharged
c. Identify anxiety-causing situations with orders for haloperidol (haldol) therapy.
d. Eat only three meals per day. When developing a teaching plan for discharge,
the nurse should include cautioning the client
30. Rudolf is admitted for an overdose of against:
amphetamines. When assessing the client, the a. Driving at night
nurse should expect to see: b. Staying in the sun
a. Tension and irritability c. Ingesting wines and cheeses
b. Slow pulse d. Taking medications containing aspirin
c. Hypotension
d. Constipation 36. Jen a nursing student is anxious about the
upcoming board examination but is able to study
31. Nicolas is experiencing hallucinations tells the intently and does not become distracted by a
nurse, “The voices are telling me I’m no good.” roommate’s talking and loud music. The
The client asks if the nurse hears the voices. The student’s ability to ignore distractions and to
most appropriate response by the nurse would focus on studying demonstrates:
be: a. Mild-level anxiety
a. “It is the voice of your conscience, which b. Panic-level anxiety
only you can control.” c. Severe-level anxiety
b. “No, I do not hear your voices, but I d. Moderate-level anxiety
believe you can hear them”.
c. “The voices are coming from within you 37. When assessing a premorbid personality
and only you can hear them.” characteristic of a client with a major
d. “Oh, the voices are a symptom of your depression, it would be unusual for the nurse to
illness; don’t pay any attention to them.” find that this client demonstrated:
a. Rigidity
b. Stubbornness

13
13
c. Diverse interest
d. Over meticulousness 43. When establishing an initial nurse-client
relationship, Nurse Hazel should explore with
38. Nurse Krina recognizes that the suicidal risk for the client the:
depressed client is greatest: a. Client’s perception of the presenting
a. As their depression begins to improve problem.
b. When their depression is most severe b. Occurrence of fantasies the client may
c. Before any type of treatment is started experience.
d. As they lose interest in the environment c. Details of any ritualistic acts carried out
by the client
39. Nurse Kate would expect that a client with d. Client’s feelings when external; controls
vascular dementis would experience: are instituted.
a. Loss of remote memory related
to anoxia 44. Tranylcypromine sulfate (Parnate) is prescribed
b. Loss of abstract thinking related to for a depressed client who has not responded to
emotional state the tricyclic antidepressants. After teaching the
c. Inability to concentrate related to client about the medication, Nurse Marian
decreased stimuli evaluates that learning has occurred when the
d. Disturbance in recalling recent events client states, “I will avoid:
related to cerebral hypoxia. a. Citrus fruit, tuna, and yellow
vegetables.”
40. Josefina is to be discharged on a regimen of b. Chocolate milk, aged cheese, and
lithium carbonate. In the teaching plan for yogurt’”
discharge the nurse should include: c. Green leafy vegetables, chicken, and
a. Advising the client to watch the diet milk.”
carefully d. Whole grains, red meats, and
b. Suggesting that the client take the pills carbonated soda.”
with milk
c. Reminding the client that a CBC must be 45. Nurse John is a aware that most crisis situations
done once a month. should resolve in about:
d. Encouraging the client to have blood a. 1 to 2 weeks
levels checked as ordered. b. 4 to 6 weeks
c. 4 to 6 months
41. The psychiatrist orders lithium carbonate 600 d. 6 to 12 months
mg p.o t.i.d for a female client. Nurse Katrina
would be aware that the teachings about the 46. Nurse Judy knows that statistics show that in
side effects of this drug were understood when adolescent suicide behavior:
the client state, “I will call my doctor a. Females use more dramatic methods
immediately if I notice any: than males
a. Sensitivity to bright light or sun b. Males account for more attempts than
b. Fine hand tremors or slurred speech do females
c. Sexual dysfunction or breast c. Females talk more about suicide before
enlargement attempting it
d. Inability to urinate or difficulty when d. Males are more likely to use lethal
urinating methods than are females

42. Nurse Mylene recognizes that the most 47. Dervid with paranoid schizophrenia repeatedly
important factor necessary for the establishment uses profanity during an activity therapy session.
of trust in a critical care area is: Which response by the nurse would be most
a. Privacy appropriate?
b. Respect a. "Your behavior won't be tolerated. Go to
c. Empathy your room immediately."
d. Presence
b. "You're just doing this to get back at me 52. Mr. Cruz visits the physician's office to seek
for making you come to therapy." treatment for depression, feelings of
c. "Your cursing is interrupting the activity. hopelessness, poor appetite, insomnia, fatigue,
Take time out in your room for 10 low self- esteem, poor concentration, and
minutes." difficulty making decisions. The client states that
d. "I'm disappointed in you. You can't these symptoms began at least 2 years ago.
control yourself even for a few minutes." Based on this report, the nurse Tyfany suspects:
a. Cyclothymic disorder.
48. Nurse Maureen knows that the nonantipsychotic b. Atypical affective disorder.
medication used to treat some clients with c. Major depression.
schizoaffective disorder is: d. Dysthymic disorder.
a. phenelzine (Nardil)
b. chlordiazepoxide (Librium) 53. After taking an overdose of phenobarbital
c. lithium carbonate (Lithane) (Barbita), Mario is admitted to the emergency
d. imipramine (Tofranil) department. Dr. Trinidad prescribes activated
charcoal (Charcocaps) to be administered by
49. Which information is most important for the mouth immediately. Before administering the
nurse Trinity to include in a teaching plan for a dose, the nurse verifies the dosage ordered.
male schizophrenic client taking clozapine What is the usual minimum dose of activated
(Clozaril)? charcoal?
a. Monthly blood tests will be necessary. a. 5 g mixed in 250 ml of water
b. Report a sore throat or fever to the b. 15 g mixed in 500 ml of water
physician immediately. c. 30 g mixed in 250 ml of water
c. Blood pressure must be monitored for d. 60 g mixed in 500 ml of water
hypertension.
d. Stop the medication when symptoms 54. What herbal medication for depression, widely
subside. used in Europe, is now being prescribed in the
United States?
50. Ricky with chronic schizophrenia takes a. Ginkgo biloba
neuroleptic medication is admitted to the b. Echinacea
psychiatric unit. Nursing assessment reveals c. St. John's wort
rigidity, fever, hypertension, and diaphoresis. d. Ephedra
These findings suggest which life- threatening
reaction: 55. Cely with manic episodes is taking lithium.
a. Tardive dyskinesia. Which electrolyte level should the nurse check
b. Dystonia. before administering this medication?
c. Neuroleptic malignant syndrome. a. Clcium
d. Akathisia. b. Sodium
c. Chloride
51. Which nursing intervention would be most d. Potassium
appropriate if a male client develop orthostatic
hypotension while taking amitriptyline (Elavil)? 56. Nurse Josefina is caring for a client who has been
a. Consulting with the physician about diagnosed with delirium. Which statement about
substituting a different type of delirium is true?
antidepressant. a. It's characterized by an acute onset and
b. Advising the client to sit up for 1 minute lasts about 1 month.
before getting out of bed. b. It's characterized by a slowly evolving
c. Instructing the client to double the onset and lasts about 1 week.
dosage until the problem resolves. c. It's characterized by a slowly evolving
d. Informing the client that this adverse onset and lasts about 1 month.
reaction should disappear within 1 d. It's characterized by an acute onset and
week. lasts hours to a number of days.

13
13
57. Edward, a 66 year old client with slight memory
impairment and poor concentration is diagnosed 61. Mr. Garcia, an attorney who throws books and
with primary degenerative dementia of the furniture around the office after losing a case is
Alzheimer's type. Early signs of this dementia referred to the psychiatric nurse in the law firm's
include subtle personality changes and employee assistance program. Nurse Beatriz
withdrawal from social interactions. To assess knows that the client's behavior most likely
for progression to the middle stage of represents the use of which defense
Alzheimer's disease, the nurse should observe mechanism?
the client for: a. Regression
a. Occasional irritable outbursts. b. Projection
b. Impaired communication. c. Reaction-formation
c. Lack of spontaneity. d. Intellectualization
d. Inability to perform self-care activities.
62. Nurse Anne is caring for a client who has been
58. Isabel with a diagnosis of depression is started treated long term with antipsychotic medication.
on imipramine (Tofranil), 75 mg by mouth at During the assessment, Nurse Anne checks the
bedtime. The nurse should tell the client that: client for tardive dyskinesia. If tardive dyskinesia
a. This medication may be habit forming is present, Nurse Anne would most likely
and will be discontinued as soon as the observe:
client feels better. a. Abnormal movements and involuntary
b. This medication has no serious adverse movements of the mouth, tongue, and
effects. face.
c. The client should avoid eating such b. Abnormal breathing through the nostrils
foods as aged cheeses, yogurt, and accompanied by a “thrill.”
chicken livers while taking the c. Severe headache, flushing, tremors, and
medication. ataxia.
d. This medication may initially cause d. Severe hypertension, migraine
tiredness, which should become less headache,
bothersome over time.
63. Dennis has a lithium level of 2.4 mEq/L. The
59. Kathleen is admitted to the psychiatric clinic for nurse immediately would assess the client for
treatment of anorexia nervosa. To promote the which of the following signs or symptoms?
client's physical health, the nurse should plan to: a. Weakness
a. Severely restrict the client's physical b. Diarrhea
activities. c. Blurred vision
b. Weigh the client daily, after the evening d. Fecal incontinence
meal.
c. Monitor vital signs, serum electrolyte 64. Nurse Jannah is monitoring a male client who
levels, and acid-base balance. has been placed inrestraints because of violent
d. Instruct the client to keep an accurate behavior. Nurse determines that it will be safe to
record of food and fluid intake. remove the restraints when:
a. The client verbalizes the reasons for the
60. Celia with a history of polysubstance abuse is violent behavior.
admitted to the facility. She complains of nausea b. The client apologizes and tells the nurse
and vomiting 24 hours after admission. The that it will never happen again.
nurse assesses the client and notes piloerection, c. No acts of aggression have been
pupillary dilation, and lacrimation. The nurse observed within 1 hour after the release
suspects that the client is going through which of of two of the extremity restraints.
the following withdrawals? d. The administered medication has taken
a. Alcohol withdrawal effect.
b. Cannibis withdrawal
c. Cocaine withdrawal
d. Opioid withdrawal
65. Nurse Irish is aware that Ritalin is the drug of a. Revealing personal information to the
choice for a child with ADHD. The side effects of client
the following may be noted by the nurse: b. Focusing on the feelings of the client.
a. Increased attention span and c. Confronting the client about
concentration discrepancies in verbal or non-verbal
b. Increase in appetite behavior
c. Sleepiness and lethargy d. The client feels angry towards the nurse
d. Bradycardia and diarrhea who resembles his mother.

66. Kitty, a 9 year old child has very limited 72. Tristan is on Lithium has suffered from diarrhea
vocabulary and interaction skills. She has an I.Q. and vomiting. What should the nurse in-charge
of 45. She is diagnosed to have Mental do first:
retardation of this classification: a. Recognize this as a drug interaction
a. Profound b. Give the client Cogentin
b. Mild c. Reassure the client that these are
c. Moderate common side effects of lithium therapy
d. Severe d. Hold the next dose and obtain an order
for a stat serum lithium level
67. The therapeutic approach in the care of Armand
an autistic child include the following EXCEPT: 73. Nurse Sarah ensures a therapeutic environment
a. Engage in diversionary activities when for all the client. Which of the following best
acting -out describes a therapeutic milieu?
b. Provide an atmosphere of acceptance a. A therapy that rewards adaptive
c. Provide safety measures behavior
d. Rearrange the environment to activate b. A cognitive approach to change behavior
the child c. A living, learning or working
environment.
68. Jeremy is brought to the emergency room by d. A permissive and congenial environment
friends who state that he took something an
hour ago. He is actively hallucinating, agitated, 74. Anthony is very hostile toward one of the staff
with irritated nasal septum. for no apparent reason. He is manifesting:
a. Heroin a. Splitting
b. Cocaine b. Transference
c. LSD c. Countertransference
d. Marijuana d. Resistance

69. Nurse Pauline is aware that Dementia unlike 75. Marielle, 17 years old was sexually attacked
delirium is characterized by: while on her way home from school. She is
a. Slurred speech brought to the hospital by her mother. Rape is
b. Insidious onset an example of which type of crisis:
c. Clouding of consciousness a. Situational
d. Sensory perceptual change b. Adventitious
c. Developmental
70. A 35 year old female has intense fear of riding an d. Internal
elevator. She claims “ As if I will die inside.” The
client is suffering from: 76. Nurse Greta is aware that the following is
a. Agoraphobia classified as an Axis I disorder by the Diagnosis
b. Social phobia and Statistical Manual of Mental Disorders, Text
c. Claustrophobia Revision (DSM-IV-TR) is:
d. Xenophobia a. Obesity
b. Borderline personality disorder
71. Nurse Myrna develops a counter-transference c. Major depression
reaction. This is evidenced by: d. Hypertension

14
14

d. It promotes emotional support or


77. Katrina, a newly admitted is extremely hostile attention for the client
toward a staff member she has just met, without
apparent reason. According to Freudian theory, 82. Dervid is diagnosed with panic disorder with
the nurse should suspect that the client is agoraphobia is talking with the nurse in-charge
experiencing which of the following about the progress made in treatment. Which of
phenomena? the following statements indicates a positive
a. Intellectualization client response?
b. Transference a. “I went to the mall with my friends last
c. Triangulation Saturday”
d. Splitting b. “I’m hyperventilating only when I have a
panic attack”
78. An 83year-old male client is in extended care c. “Today I decided that I can stop taking
facility is anxious most of the time and my medication”
frequently complains of a number of vague d. “Last night I decided to eat more than a
symptoms that interfere with his ability to eat. bowl of cereal”
These symptoms indicate which of the following
disorders? 83. The effectiveness of monoamine oxidase (MAO)
a. Conversion disorder inhibitor drug therapy in a client with
b. Hypochondriasis posttraumatic stress disorder can be
c. Severe anxiety demonstrated by which of the following client
d. Sublimation self –reports?
a. “I’m sleeping better and don’t have
79. Charina, a college student who frequently visited nightmares”
the health center during the past year with b. “I’m not losing my temper as much”
multiple vague complaints of GI symptoms c. “I’ve lost my craving for alcohol”
before course examinations. Although physical d. I’ve lost my phobia for water”
causes have been eliminated, the student
continues to express her belief that she has a 84. Mark, with a diagnosis of generalized anxiety
serious illness. These symptoms are typically of disorder wants to stop taking his lorazepam
which of the following disorders? (Ativan). Which of the following important facts
a. Conversion disorder should nurse Betty discuss with the client about
b. Depersonalization discontinuing the medication?
c. Hypochondriasis a. Stopping the drug may cause depression
d. Somatization disorder b. Stopping the drug increases cognitive
abilities
80. Nurse Daisy is aware that the following c. Stopping the drug decreases sleeping
pharmacologic agents are sedative- hypnotic difficulties
medication is used to induce sleep for a client d. Stopping the drug can cause withdrawal
experiencing a sleep disorder is: symptoms
a. Triazolam (Halcion)
b. Paroxetine (Paxil)\ 85. Jennifer, an adolescent who is depressed and
c. Fluoxetine (Prozac) reported by his parents as having difficulty in
d. Risperidone (Risperdal) school is brought to the community mental
health center to be evaluated. Which of the
81. Aldo, with a somatoform pain disorder may following other health problems would the nurse
obtain secondary gain. Which of the following suspect?
statement refers to a secondary gain? a. Anxiety disorder
a. It brings some stability to the family b. Behavioral difficulties
b. It decreases the preoccupation with the c. Cognitive impairment
physical illness d. Labile moods
c. It enables the client to avoid some
unpleasant activity
86. Ricardo, an outpatient in psychiatric facility is c. The client becomes anxious whenever
diagnosed with dysthymic disorder. Which of the the nurse leaves the bedside
following statement about dysthymic disorder is d. The client looks at the shadow on a wall
true? and tells the nurse she sees frightening
a. It involves a mood range from moderate faces on the wall.
depression to hypomania
b. It involves a single manic depression 91. During conversation of Nurse John with a client,
c. It’s a form of depression that occurs in he observes that the client shift from one topic
the fall and winter to the next on a regular basis. Which of the
d. It’s a mood disorder similar to major following terms describes this disorder?
depression but of mild to moderate a. Flight of ideas
severity b. Concrete thinking
c. Ideas of reference
87. The nurse is aware that the following ways in d. Loose association
vascular dementia different from Alzheimer’s
disease is: 92. Francis tells the nurse that her coworkers are
a. Vascular dementia has more abrupt sabotaging the computer. When the nurse asks
onset questions, the client becomes argumentative.
b. The duration of vascular dementia is This behavior shows personality traits associated
usually brief with which of the following personality disorder?
c. Personality change is common in a. Antisocial
vascular dementia b. Histrionic
d. The inability to perform motor activities c. Paranoid
occurs in vascular dementia d. Schizotypal

88. Loretta, a newly admitted client was diagnosed 93. Which of the following interventions is
with delirium and has history of hypertension important for a Cely experiencing with paranoid
and anxiety. She had been taking digoxin, personality disorder taking olanzapine
furosemide (Lasix), and diazepam (Valium) for (Zyprexa)?
anxiety. This client’s impairment may be related a. Explain effects of serotonin syndrome
to which of the following conditions? b. Teach the client to watch for
a. Infection extrapyramidal adverse reaction
b. Metabolic acidosis c. Explain that the drug is less affective if
c. Drug intoxication the client smokes
d. Hepatic encephalopathy d. Discuss the need to report paradoxical
effects such as euphoria
89. Nurse Ron enters a client’s room, the client says,
“They’re crawling on my sheets! Get them off 94. Nurse Alexandra notices other clients on the unit
my bed!” Which of the following assessment is avoiding a client diagnosed with antisocial
the most accurate? personality disorder. When discussing
a. The client is experiencing aphasia appropriate behavior in group therapy, which of
b. The client is experiencing dysarthria the following comments is expected about this
c. The client is experiencing a flight of ideas client by his peers?
d. The client is experiencing visual a. Lack of honesty
hallucination b. Belief in superstition
c. Show of temper tantrums
90. Which of the following descriptions of a client’s d. Constant need for attention
experience and behavior can be assessed as an
illusion? 95. Tommy, with dependent personality disorder is
a. The client tries to hit the nurse when working to increase his self- esteem. Which of
vital signs must be taken the following statements by the Tommy shows
b. The client says, “I keep hearing a voice teaching was successful?
telling me to run away”

14
14
a. “I’m not going to look just at the 100. Rocky has started taking haloperidol (Haldol).
negative things about myself” Which of the following instructions is most
b. “I’m most concerned about my level of appropriate for Ricky before taking
competence and progress” haloperidol?
c. “I’m not as envious of the things other a. Should report feelings of restlessness or
people have as I used to be” agitation at once
d. “I find I can’t stop myself from taking b. Use a sunscreen outdoors on a year-
over things other should be doing” round basis
c. Be aware you’ll feel increased energy
96. Norma, a 42-year-old client with a diagnosis of taking this drug
chronic undifferentiated schizophrenia lives in a d. This drug will indirectly control essential
rooming house that has a weekly nursing clinic. hypertension
She scratches while she tells the nurse she feels
creatures eating away at her skin. Which of the
following interventions should be done first?
a. Talk about his hallucinations and fears
b. Refer him for anticholinergic adverse
reactions
c. Assess for possible physical problems
such as rash
d. Call his physician to get his medication
increased to control his psychosis

97. Ivy, who is on the psychiatric unit is copying and


imitating the movements of her primary nurse.
During recovery, she says, “I thought the nurse
was my mirror. I felt connected only when I saw
my nurse.” This behavior is known by which of
the following terms?
a. Modeling
b. Echopraxia
c. Ego-syntonicity
d. Ritualism

98. Jun approaches the nurse and tells that he hears


a voice telling him that he’s evil and deserves to
die. Which of the following terms describes the
client’s perception?
a. Delusion
b. Disorganized speech
c. Hallucination
d. Idea of reference

99. Mike is admitted to a psychiatric unit with a


diagnosis of undifferentiated schizophrenia.
Which of the following defense mechanisms is
probably used by mike?
a. Projection
b. Rationalization
c. Regression
d. Repression
Answers and Rationale – Care of Clients with Rationale: This behavior is an example of
Physiologic and Psychosocial Alterations reaction formation, a coping mechanism.
11. Answer: (A) By designating times during which
1. Answer: (D) Focusing the client can focus on the behavior.
Rationale: The nurse is using focusing by Rationale: The nurse should designate times
suggesting that the client discuss a specific issue. during which the client can focus on the
The nurse didn’t restate the question, make compulsive behavior or obsessive thoughts. The
observation, or ask further question (exploring). nurse should urge the client to reduce the
2. Answer: (D) Remove all other clients from the frequency of the compulsive behavior gradually,
dayroom. not rapidly. She shouldn't call attention to or try
Rationale: The nurse’s first priority is to consider to prevent the behavior. Trying to prevent the
the safety of the clients in the therapeutic behavior may cause pain and terror in the client.
setting. The other actions are appropriate The nurse should encourage the client to
responses after ensuring the safety of other verbalize anxieties to help distract attention
clients. from the compulsive behavior.
3. Answer: (A) The client is disruptive. 12. Answer: (D) Exploring the meaning of the
Rationale: Group activity provides too much traumatic event with the client.
stimulation, which the client will not be able to Rationale: The client with PTSD needs
handle (harmful to self) and as a result will be encouragement to examine and understand the
disruptive to others. meaning of the traumatic event and consequent
4. Answer: (C) Agree to talk with the mother and losses. Otherwise, symptoms may worsen and
the father together. the client may become depressed or engage in
Rationale: By agreeing to talk with both parents, self-destructive behavior such as substance
the nurse can provide emotional support and abuse. The client must explore the meaning of
further assess and validate the family’s needs. the event and won't heal without this, no matter
5. Answer: (A) Perceptual disorders. how much time passes. Behavioral techniques,
Rationale: Frightening visual hallucinations are such as relaxation therapy, may help decrease
especially common in clients experiencing the client's anxiety and induce sleep. The
alcohol withdrawal. physician may prescribe antianxiety agents or
6. Answer: (D) Suggest that it takes a while before antidepressants cautiously to avoid dependence;
seeing the results. sleep medication is rarely appropriate. A special
Rationale: The client needs a specific response; diet isn't indicated unless the client also has an
that it takes 2 to 3 weeks (a delayed effect) until eating disorder or a nutritional problem.
the therapeutic blood level is reached. 13. Answer: (C) "Your problem is real but there is no
7. Answer: (C) Superego physical basis for it. We'll work on what is going
Rationale: This behavior shows a weak sense of on in your life to find out why it's happened."
moral consciousness. According to Freudian Rationale: The nurse must be honest with the
theory, personality disorders stem from a weak client by telling her that the paralysis has no
superego. physiologic cause while also conveying empathy
8. Answer: (C) Skeletal muscle paralysis. and acknowledging that her symptoms are real.
Rationale: Anectine is a depolarizing muscle The client will benefit from psychiatric
relaxant causing paralysis. It is used to reduce treatment, which will help her understand the
the intensity of muscle contractions during the underlying cause of her symptoms. After the
convulsive stage, thereby reducing the risk of psychological conflict is resolved, her symptoms
bone fractures or dislocation. will disappear. Saying that it must be awful not
9. Answer: (D) Increase calories, carbohydrates, to be able to move her legs wouldn't answer the
and protein. client's question; knowing that the cause is
Rationale: This client increased protein for tissue psychological wouldn't necessarily make her feel
building and increased calories to replace what is better. Telling her that she has developed
burned up (usually via carbohydrates). paralysis to avoid leaving her parents or that her
10. Answer: (C) Acting overly solicitous toward the personality caused her disorder wouldn't help
child. her understand and resolve the underlying
conflict.

14
14
14. Answer: (C) fluvoxamine (Luvox) and Rationale: Clients in the first stage of Alzheimer's
clomipramine (Anafranil) disease are aware that something is happening
Rationale: The antidepressants fluvoxamine and to them and may become overwhelmed and
clomipramine have been effective in the frightened. Therefore, nursing care typically
treatment of OCD. Librium and Valium may be focuses on providing emotional support and
helpful in treating anxiety related to OCD but individual counseling. The other options are
aren't drugs of choice to treat the illness. The appropriate during the second stage of
other medications mentioned aren't effective in Alzheimer's disease, when the client needs
the treatment of OCD. continuous monitoring to prevent minor
15. Answer: (A) A warning about the drugs delayed illnesses from progressing into major problems
therapeutic effect, which is from 14 to 30 days. and when maintaining adequate nutrition may
Rationale: The client should be informed that become a challenge. During this stage, offering
the drug's therapeutic effect might not be nourishing finger foods helps clients to feed
reached for 14 to 30 days. The client must be themselves and maintain adequate nutrition.
instructed to continue taking the drug as 20. Answer: (C) Emotional lability, euphoria, and
directed. Blood level checks aren't necessary. impaired memory
NMS hasn't been reported with this drug, but Rationale: Signs of antianxiety agent overdose
tachycardia is frequently reported. include emotional lability, euphoria, and
16. Answer: (B) Severe anxiety and fear. impaired memory. Phencyclidine overdose can
Rationale: Phobias cause severe anxiety (such as cause combativeness, sweating, and confusion.
a panic attack) that is out of proportion to the Amphetamine overdose can result in agitation,
threat of the feared object or situation. Physical hyperactivity, and grandiose ideation.
signs and symptoms of phobias include profuse Hallucinogen overdose can produce
sweating, poor motor control, tachycardia, and suspiciousness, dilated pupils, and increased
elevated blood pressure. Insomnia, an inability blood pressure.
to concentrate, and weight loss are common in 21. Answer: (D) A low tolerance for frustration
depression. Withdrawal and failure to Rationale: Clients with an antisocial personality
distinguish reality from fantasy occur in disorder exhibit a low tolerance for frustration,
schizophrenia. emotional immaturity, and a lack of impulse
17. Answer: (A) Antidepressants control. They commonly have a history of
Rationale: Tricyclic and monoamine oxidase unemployment, miss work repeatedly, and quit
(MAO) inhibitor antidepressants have been work without other plans for employment. They
found to be effective in treating clients with don't feel guilt about their behavior and
panic attacks. Why these drugs help control commonly perceive themselves as victims. They
panic attacks isn't clearly understood. also display a lack of responsibility for the
Anticholinergic agents, which are smooth- outcome of their actions. Because of a lack of
muscle relaxants, relieve physical symptoms of trust in others, clients with antisocial personality
anxiety but don't relieve the anxiety itself. disorder commonly have difficulty developing
Antipsychotic drugs are inappropriate because stable, close relationships.
clients who experience panic attacks aren't 22. Answer: (C) Methadone
psychotic. Mood stabilizers aren't indicated Rationale: Methadone is used to detoxify opiate
because panic attacks are rarely associated with users because it binds with opioid receptors at
mood changes. many sites in the central nervous system but
18. Answer: (B) 3 to 5 days doesn’t have the same deterious effects as other
Rationale: Monoamine oxidase inhibitors, such opiates, such as cocaine, heroin, and morphine.
as tranylcypromine, have an onset of action of Barbiturates, amphetamines, and
approximately 3 to 5 days. A full clinical benzodiazepines are highly addictive and would
response may be delayed for 3 to 4 weeks. The require detoxification treatment.
therapeutic effects may continue for 1 to 2 23. Answer: (B) Hallucinations
weeks after discontinuation. Rationale: Hallucinations are visual, auditory,
19. Answer: (B) Providing emotional support and gustatory, tactile, or olfactory perceptions that
individual counseling. have no basis in reality. Delusions are false
beliefs, rather than perceptions, that the client
accepts as real. Loose associations are rapid behavior is uncommon, although these clients
shifts among unrelated ideas. Neologisms are may experience agitation with anxiety. Their
bizarre words that have meaning only to the behavior is emotionally cold with a flattened
client. affect, regardless of the situation. These clients
24. Answer: (C) Set up a strict eating plan for the demonstrate a reduced capacity for close or
client. dependent relationships.
Rationale: Establishing a consistent eating plan 29. Answer: (C) Identify anxiety-causing situations
and monitoring the client’s weight are very Rationale: Bulimic behavior is generally a
important in this disorder. The family and friends maladaptive coping response to stress and
should be included in the client’s care. The client underlying issues. The client must identify
should be monitored during meals-not given anxiety-causing situations that stimulate the
privacy. Exercise must be limited and supervised. bulimic behavior and then learn new ways of
25. Answer: (A) Highly important or famous. coping with the anxiety.
Rationale: A delusion of grandeur is a false belief 30. Answer: (A) Tension and irritability
that one is highly important or famous. A Rationale: An amphetamine is a nervous system
delusion of persecution is a false belief that one stimulant that is subject to abuse because of its
is being persecuted. A delusion of reference is a ability to produce wakefulness and euphoria. An
false belief that one is connected to events overdose increases tension and irritability.
unrelated to oneself or a belief that one is Options B and C are incorrect because
responsible for the evil in the world. amphetamines stimulate norepinephrine, which
26. Answer: (D) Listening attentively with a neutral increase the heart rate and blood flow. Diarrhea
attitude and avoiding power struggles. is a common adverse effect so option D is
Rationale: The nurse should listen to the client’s incorrect.
requests, express willingness to seriously 31. Answer: (B) “No, I do not hear your voices, but I
consider the request, and respond later. The believe you can hear them”.
nurse should encourage the client to take short Rationale: The nurse, demonstrating knowledge
daytime naps because he expends so much and understanding, accepts the client’s
energy. The nurse shouldn’t try to restrain the perceptions even though they are hallucinatory.
client when he feels the need to move around as 32. Answer: (C) Confusion for a time after treatment
long as his activity isn’t harmful. High calorie Rationale: The electrical energy passing through
finger foods should be offered to supplement the cerebral cortex during ECT results in a
the client’s diet, if he can’t remain seated long temporary state of confusion after treatment.
enough to eat a complete meal. The nurse 33. Answer: (D) Acceptance stage
shouldn’t be forced to stay seated at the table to Rationale: Communication and intervention
finid=sh a meal. The nurse should set limits in a during this stage are mainly nonverbal, as when
calm, clear, and self-confident tone of voice. the client gestures to hold the nurse’s hand.
27. Answer: (D) Denial 34. Answer: (D) A higher level of anxiety continuing
Rationale: Denial is unconscious defense for more than 3 months.
mechanism in which emotional conflict and Rationale: This is not an expected outcome of a
anxiety is avoided by refusing to acknowledge crisis because by definition a crisis would be
feelings, desires, impulses, or external facts that resolved in 6 weeks.
are consciously intolerable. Withdrawal is a 35. Answer: (B) Staying in the sun
common response to stress, characterized by Rationale: Haldol causes photosensitivity. Severe
apathy. Logical thinking is the ability to think sunburn can occur on exposure to the sun.
rationally and make responsible decisions, which 36. Answer: (D) Moderate-level anxiety
would lead the client admitting the problem and Rationale: A moderately anxious person can
seeking help. Repression is suppressing past ignore peripheral events and focuses on central
events from the consciousness because of guilty concerns.
association. 37. Answer: (C) Diverse interest
28. Answer: (B) Paranoid thoughts Rationale: Before onset of depression, these
Rationale: Clients with schizotypal personality clients usually have very narrow, limited
disorder experience excessive social anxiety that interest.
can lead to paranoid thoughts. Aggressive

14
14
38. Answer: (A) As their depression begins to option A. Option B is incorrect because it implies
improve that the client’s actions reflect feelings toward
Rationale: At this point the client may have the staff instead of the client's own misery.
enough energy to plan and execute an attempt. Judgmental remarks, such as option D, may
39. Answer: (D) Disturbance in recalling recent decrease the client's self-esteem.
events related to cerebral hypoxia. 48. Answer: (C) lithium carbonate (Lithane)
Rationale: Cell damage seems to interfere with Rationale: Lithium carbonate, an antimania drug,
registering input stimuli, which affects the ability is used to treat clients with cyclical
to register and recall recent events; vascular schizoaffective disorder, a psychotic disorder
dementia is related to multiple vascular lesions once classified under schizophrenia that causes
of the cerebral cortex and subcortical structure. affective symptoms, including maniclike activity.
40. Answer: (D) Encouraging the client to have blood Lithium helps control the affective component of
levels checked as ordered. this disorder. Phenelzine is a monoamine
Rationale: Blood levels must be checked monthly oxidase inhibitor prescribed for clients who don't
or bimonthly when the client is on maintenance respond to other antidepressant drugs such as
therapy because there is only a small range imipramine. Chlordiazepoxide, an antianxiety
between therapeutic and toxic levels. agent, generally is contraindicated in psychotic
41. Answer: (B) Fine hand tremors or slurred speech clients. Imipramine, primarily considered an
Rationale: These are common side effects of antidepressant agent, is also used to treat clients
lithium carbonate. with agoraphobia and that undergoing cocaine
42. Answer: (D) Presence detoxification.
Rationale: The constant presence of a nurse 49. Answer: (B) Report a sore throat or fever to the
provides emotional support because the client physician immediately.
knows that someone is attentive and available in Rationale: A sore throat and fever are
case of an emergency. indications of an infection caused by
43. Answer: (A) Client’s perception of the presenting agranulocytosis, a potentially life-threatening
problem. complication of clozapine. Because of the risk of
Rationale: The nurse can be most therapeutic by agranulocytosis, white blood cell (WBC) counts
starting where the client is, because it is the are necessary weekly, not monthly. If the WBC
client’s concept of the problem that serves as count drops below 3,000/μl, the medication
the starting point of the relationship. must be stopped. Hypotension may occur in
44. Answer: (B) Chocolate milk, aged cheese, and clients taking this medication. Warn the client to
yogurt’” stand up slowly to avoid dizziness from
Rationale: These high-tyramine foods, when orthostatic hypotension. The medication should
ingested in the presence of an MAO inhibitor, be continued, even when symptoms have been
cause a severe hypertensive response. controlled. If the medication must be stopped, it
45. Answer: (B) 4 to 6 weeks should be slowly tapered over 1 to 2 weeks and
Rationale: Crisis is self-limiting and lasts from 4 only under the supervision of a physician.
to 6 weeks. 50. Answer: (C) Neuroleptic malignant syndrome.
46. Answer: (D) Males are more likely to use lethal Rationale: The client's signs and symptoms
methods than are females suggest neuroleptic malignant syndrome, a life-
Rationale: This finding is supported by research; threatening reaction to neuroleptic medication
females account for 90% of suicide attempts but that requires immediate treatment. Tardive
males are three times more successful because dyskinesia causes involuntary movements of the
of methods used. tongue, mouth, facial muscles, and arm and leg
47. Answer: (C) "Your cursing is interrupting the muscles. Dystonia is characterized by cramps
activity. Take time out in your room for 10 and rigidity of the tongue, face, neck, and back
minutes." muscles. Akathisia causes restlessness, anxiety,
Rationale: The nurse should set limits on client and jitteriness.
behavior to ensure a comfortable environment 51. Answer: (B) Advising the client to sit up for 1
for all clients. The nurse should accept hostile or minute before getting out of bed.
quarrelsome client outbursts within limits Rationale: To minimize the effects of
without becoming personally offended, as in amitriptyline-induced orthostatic hypotension,
the nurse should advise the client to sit up for 1 functions but sodium is most important to the
minute before getting out of bed. Orthostatic absorption of lithium.
hypotension commonly occurs with tricyclic 56. Answer: (D) It's characterized by an acute onset
antidepressant therapy. In these cases, the and lasts hours to a number of days
dosage may be reduced or the physician may Rationale: Delirium has an acute onset and
prescribe nortriptyline, another tricyclic typically can last from several hours to several
antidepressant. Orthostatic hypotension days.
disappears only when the drug is discontinued. 57. Answer: (B) Impaired communication.
52. Answer: (D) Dysthymic disorder. Rationale: Initially, memory impairment may be
Rationale: Dysthymic disorder is marked by the only cognitive deficit in a client with
feelings of depression lasting at least 2 years, Alzheimer's disease. During the early stage of
accompanied by at least two of the following this disease, subtle personality changes may also
symptoms: sleep disturbance, appetite be present. However, other than occasional
disturbance, low energy or fatigue, low self- irritable outbursts and lack of spontaneity, the
esteem, poor concentration, difficulty making client is usually cooperative and exhibits socially
decisions, and hopelessness. These symptoms appropriate behavior. Signs of advancement to
may be relatively continuous or separated by the middle stage of Alzheimer's disease include
intervening periods of normal mood that last a exacerbated cognitive impairment with obvious
few days to a few weeks. Cyclothymic disorder is personality changes and impaired
a chronic mood disturbance of at least 2 years' communication, such as inappropriate
duration marked by numerous periods of conversation, actions, and responses. During the
depression and hypomania. Atypical affective late stage, the client can't perform self-care
disorder is characterized by manic signs and activities and may become mute.
symptoms. Major depression is a recurring, 58. Answer: (D) This medication may initially cause
persistent sadness or loss of interest or pleasure tiredness, which should become less
in almost all activities, with signs and symptoms bothersome over time.
recurring for at least 2 weeks. Rationale: Sedation is a common early adverse
53. Answer: (C) 30 g mixed in 250 ml of water effect of imipramine, a tricyclic antidepressant,
Rationale: The usual adult dosage of activated and usually decreases as tolerance develops.
charcoal is 5 to 10 times the estimated weight of Antidepressants aren't habit forming and don't
the drug or chemical ingested, or a minimum cause physical or psychological dependence.
dose of 30 g, mixed in 250 ml of water. Doses However, after a long course of high-dose
less than this will be ineffective; doses greater therapy, the dosage should be decreased
than this can increase the risk of adverse gradually to avoid mild withdrawal symptoms.
reactions, although toxicity doesn't occur with Serious adverse effects, although rare, include
activated charcoal, even at the maximum dose. myocardial infarction, heart failure, and
54. Answer: (C) St. John's wort tachycardia. Dietary restrictions, such as
Rationale: St. John's wort has been found to avoiding aged cheeses, yogurt, and chicken
have serotonin-elevating properties, similar to livers, are necessary for a client taking a
prescription antidepressants. Ginkgo biloba is monoamine oxidase inhibitor, not a tricyclic
prescribed to enhance mental acuity. Echinacea antidepressant.
has immune-stimulating properties. Ephedra is a 59. Answer: (C) Monitor vital signs, serum
naturally occurring stimulant that is similar to electrolyte levels, and acid-base balance.
ephedrine. Rationale: An anorexic client who requires
55. Answer: (B) Sodium hospitalization is in poor physical condition from
Rationale: Lithium is chemically similar to starvation and may die as a result of
sodium. If sodium levels are reduced, such as arrhythmias, hypothermia, malnutrition,
from sweating or diuresis, lithium will be infection, or cardiac abnormalities secondary to
reabsorbed by the kidneys, increasing the risk of electrolyte imbalances. Therefore, monitoring
toxicity. Clients taking lithium shouldn't restrict the client's vital signs, serum electrolyte level,
their intake of sodium and should drink and acid base balance is crucial. Option A may
adequate amounts of fluid each day. The other worsen anxiety. Option B is incorrect because a
electrolytes are important for normal body weight obtained after breakfast is more accurate

14
14
than one obtained after the evening meal. 65. Answer: (A) increased attention span and
Option D would reward the client with attention concentration
for not eating and reinforce the control issues Rationale: The medication has a paradoxic effect
that are central to the underlying psychological that decreases hyperactivity and impulsivity
problem; also, the client may record food and among children with ADHD. B, C, D. Side effects
fluid intake inaccurately. of Ritalin include anorexia, insomnia, diarrhea
60. Answer: (D) Opioid withdrawal and irritability.
Rationale: The symptoms listed are specific to 66. Answer: (C) Moderate
opioid withdrawal. Alcohol withdrawal would Rationale: The child with moderate mental
show elevated vital signs. There is no real retardation has an I.Q. of 35- 50 Profound
withdrawal from cannibis. Symptoms of cocaine Mental retardation has an I.Q. of below 20; Mild
withdrawal include depression, anxiety, and mental retardation 50-70 and Severe mental
agitation. retardation has an I.Q. of 20-35.
61. Answer: (A) Regression 67. Answer: (D) Rearrange the environment to
Rationale: An adult who throws temper activate the child
tantrums, such as this one, is displaying Rationale: The child with autistic disorder does
regressive behavior, or behavior that is not want change. Maintaining a consistent
appropriate at a younger age. In projection, the environment is therapeutic. A. Angry outburst
client blames someone or something other than can be re-channeling through safe activities. B.
the source. In reaction formation, the client acts Acceptance enhances a trusting relationship. C.
in opposition to his feelings. In Ensure safety from self-destructive behaviors
intellectualization, the client overuses rational like head banging and hair pulling.
explanations or abstract thinking to decrease the 68. Answer: (B) cocaine
significance of a feeling or event. Rationale: The manifestations indicate
62. Answer: (A) Abnormal movements and intoxication with cocaine, a CNS stimulant. A.
involuntary movements of the mouth, tongue, Intoxication with heroine is manifested by
and face. euphoria then impairment in judgment,
Rationale: Tardive dyskinesia is a severe reaction attention and the presence of papillary
associated with long term use of antipsychotic constriction. C. Intoxication with hallucinogen
medication. The clinical manifestations include like LSD is manifested by grandiosity,
abnormal movements (dyskinesia) and hallucinations, synesthesia and increase in vital
involuntary movements of the mouth, tongue signs D. Intoxication with Marijuana, a
(fly catcher tongue), and face. cannabinoid is manifested by sensation of
63. Answer: (C) Blurred vision slowed time, conjunctival redness, social
Rationale: At lithium levels of 2 to 2.5 mEq/L the withdrawal, impaired judgment and
client will experienced blurred vision, muscle hallucinations.
twitching, severe hypotension, and persistent 69. Answer: (B) insidious onset
nausea and vomiting. With levels between 1.5 Rationale: Dementia has a gradual onset and
and 2 mEq/L the client experiencing vomiting, progressive deterioration. It causes pronounced
diarrhea, muscle weakness, ataxia, dizziness, memory and cognitive disturbances. A,C and D
slurred speech, and confusion. At lithium levels are all characteristics of delirium.
of 2.5 to 3 mEq/L or higher, urinary and fecal 70. Answer: (C) Claustrophobia
incontinence occurs, as well as seizures, cardiac Rationale: Claustrophobia is fear of closed space.
dysrythmias, peripheral vascular collapse, and A. Agoraphobia is fear of open space or being a
death. situation where escape is difficult. B. Social
64. Answer: (C) No acts of aggression have been phobia is fear of performing in the presence of
observed within 1 hour after the release of two others in a way that will be humiliating or
of the extremity restraints. embarrassing. D. Xenophobia is fear of
Rationale: The best indicator that the behavior is strangers.
controlled, if the client exhibits no signs of 71. Answer: (A) Revealing personal information to
aggression after partial release of restraints. the client
Options , B, and D do not ensure that the client Rationale: Counter-transference is an emotional
has controlled the behavior. reaction of the nurse on the client based on her
unconscious needs and conflicts. B and C. These Rationale: The DSM-IV-TR classifies major
are therapeutic approaches. D. This is depression as an Axis I disorder. Borderline
transference reaction where a client has an personality disorder as an Axis II; obesity and
emotional reaction towards the nurse based on hypertension, Axis III.
her past. 77. Answer: (B) Transference
72. Answer: (D) Hold the next dose and obtain an Rationale: Transference is the unconscious
order for a stat serum lithium level assignment of negative or positive feelings
Rationale: Diarrhea and vomiting are evoked by a significant person in the client’s past
manifestations of Lithium toxicity. The next dose to another person. Intellectualization is a
of lithium should be withheld and test is done to defense mechanism in which the client avoids
validate the observation. A. The manifestations dealing with emotions by focusing on facts.
are not due to drug interaction. B. Cogentin is Triangulation refers to conflicts involving three
used to manage the extra pyramidal symptom family members. Splitting is a defense
side effects of antipsychotics. C. The common mechanism commonly seen in clients with
side effects of Lithium are fine hand tremors, personality disorder in which the world is
nausea, polyuria and polydipsia. perceived as all good or all bad.
73. Answer: (C) A living, learning or working 78. Answer: (B) Hypochondriasis
environment. Rationale: Complains of vague physical
Rationale: A therapeutic milieu refers to a broad symptoms that have no apparent medical causes
conceptual approach in which all aspects of the are characteristic of clients with
environment are channeled to provide a hypochondriasis. In many cases, the GI system is
therapeutic environment for the client. The six affected. Conversion disorders are characterized
environmental elements include structure, by one or more neurologic symptoms. The
safety, norms; limit setting, balance and unit client’s symptoms don’t suggest severe anxiety.
modification. A. Behavioral approach in A client experiencing sublimation channels
psychiatric care is based on the premise that maladaptive feelings or impulses into socially
behavior can be learned or unlearned through acceptable behavior
the use of reward and punishment. B. Cognitive 79. Answer: (C) Hypochondriasis
approach to change behavior is done by Rationale: Hypochodriasis in this case is shown
correcting distorted perceptions and irrational by the client’s belief that she has a serious
beliefs to correct maladaptive behaviors. D. This illness, although pathologic causes have been
is not congruent with therapeutic milieu. eliminated. The disturbance usually lasts at least
74. Answer: (B) Transference 6 with identifiable life stressor such as, in this
Rationale: Transference is a positive or negative case, course examinations. Conversion disorders
feeling associated with a significant person in are characterized by one or more neurologic
the client’s past that are unconsciously assigned symptoms. Depersonalization refers to
to another A. Splitting is a defense mechanism persistent recurrent episodes of feeling
commonly seen in a client with personality detached from one’s self or body. Somatoform
disorder in which the world is perceived as all disorders generally have a chronic course with
good or all bad C. Countert-transference is a few remissions.
phenomenon where the nurse shifts feelings 80. Answer: (A) Triazolam (Halcion)
assigned to someone in her past to the patient Rationale: Triazolam is one of a group of
D. Resistance is the client’s refusal to submit sedative hypnotic medication that can be used
himself to the care of the nurse for a limited time because of the risk of
75. Answer: (B) Adventitious dependence. Paroxetine is a scrotonin-specific
Rationale: Adventitious crisis is a crisis involving reutake inhibitor used for treatment of
a traumatic event. It is not part of everyday life. depression panic disorder, and obsessive-
A. Situational crisis is from an external source compulsive disorder. Fluoxetine is a scrotonin-
that upset ones psychological equilibrium C and specific reuptake inhibitor used for depressive
D. are the same. They are transitional or disorders and obsessive-compulsive disorders.
developmental periods in life Risperidome is indicated for psychotic disorders.
76. Answer: (C) Major depression 81. Answer: (D) It promotes emotional support or
attention for the client

15
15
Rationale: Secondary gain refers to the benefits from moderate depression to hypomania.
of the illness that allow the client to receive Bipolar I disorder is characterized by a single
emotional support or attention. Primary gain manic episode with no past major depressive
enables the client to avoid some unpleasant episodes. Seasonal- affective disorder is a form
activity. A dysfunctional family may disregard of depression occurring in the fall and winter.
the real issue, although some conflict is relieved. 87. Answer: (A) Vascular dementia has more abrupt
Somatoform pain disorder is a preoccupation onset
with pain in the absence of physical disease. Rationale: Vascular dementia differs from
82. Answer: (A) “I went to the mall with my friends Alzheimer’s disease in that it has a more abrupt
last Saturday” onset and runs a highly variable course.
Rationale: Clients with panic disorder tent to be Personally change is common in Alzheimer’s
socially withdrawn. Going to the mall is a sign of disease. The duration of delirium is usually brief.
working on avoidance behaviors. The inability to carry out motor activities is
Hyperventilating is a key symptom of panic common in Alzheimer’s disease.
disorder. Teaching breathing control is a major 88. Answer: (C) Drug intoxication
intervention for clients with panic disorder. The Rationale: This client was taking several
client taking medications for panic disorder; such medications that have a propensity for
as tricylic antidepressants and benzodiazepines producing delirium; digoxin (a digitalis
must be weaned off these drugs. Most clients glycoxide), furosemide (a thiazide diuretic), and
with panic disorder with agoraphobia don’t have diazepam (a benzodiazepine). Sufficient
nutritional problems. supporting data don’t exist to suspect the other
83. Answer: (A) “I’m sleeping better and don’t have options as causes.
nightmares” 89. Answer: (D) The client is experiencing visual
Rationale: MAO inhibitors are used to treat sleep hallucination
problems, nightmares, and intrusive daytime Rationale: The presence of a sensory stimulus
thoughts in individual with posttraumatic stress correlates with the definition of a hallucination,
disorder. MAO inhibitors aren’t used to help which is a false sensory perception. Aphasia
control flashbacks or phobias or to decrease the refers to a communication problem. Dysarthria is
craving for alcohol. difficulty in speech production. Flight of ideas is
84. Answer: (D) Stopping the drug can cause rapid shifting from one topic to another.
withdrawal symptoms 90. Answer: (D) The client looks at the shadow on a
Rationale: Stopping antianxiety drugs such as wall and tells the nurse she sees frightening
benzodiazepines can cause the client to have faces on the wall.
withdrawal symptoms. Stopping a Rationale: Minor memory problems are
benzodiazepine doesn’t tend to cause distinguished from dementia by their minor
depression, increase cognitive abilities, or severity and their lack of significant interference
decrease sleeping difficulties. with the client’s social or occupational lifestyle.
85. Answer: (B) Behavioral difficulties Other options would be included in the history
Rationale: Adolescents tend to demonstrate data but don’t directly correlate with the client’s
severe irritability and behavioral problems lifestyle.
rather than simply a depressed mood. Anxiety 91. Answer: (D) Loose association
disorder is more commonly associated with Rationale: Loose associations are conversations
small children rather than with adolescents. that constantly shift in topic. Concrete thinking
Cognitive impairment is typically associated with implies highly definitive thought processes.
delirium or dementia. Labile mood is more Flight of ideas is characterized by conversation
characteristic of a client with cognitive that’s disorganized from the onset. Loose
impairment or bipolar disorder. associations don’t necessarily start in a cogently,
86. Answer: (D) It’s a mood disorder similar to major then becomes loose.
depression but of mild to moderate severity 92. Answer: (C) Paranoid
Rationale: Dysthymic disorder is a mood disorder Rationale: Because of their suspiciousness,
similar to major depression but it remains mild paranoid personalities ascribe malevolent
to moderate in severity. Cyclothymic disorder is activities to others and tent to be defensive,
a mood disorder characterized by a mood range becoming quarrelsome and argumentative.
Clients with antisocial personality disorder can 97. Answer: (B) Echopraxia
also be antagonistic and argumentative but are Rationale: Echopraxia is the copying of another’s
less suspicious than paranoid personalities. behaviors and is the result of the loss of ego
Clients with histrionic personality disorder are boundaries. Modeling is the conscious copying
dramatic, not suspicious and argumentative. of someone’s behaviors. Ego-syntonicity refers
Clients with schizoid personality disorder are to behaviors that correspond with the
usually detached from other and tend to have individual’s sense of self. Ritualism behaviors are
eccentric behavior. repetitive and compulsive.
93. Answer: (C) Explain that the drug is less affective 98. Answer: (C) Hallucination
if the client smokes Rationale: Hallucinations are sensory
Rationale: Olanzapine (Zyprexa) is less effective experiences that are misrepresentations of
for clients who smoke cigarettes. Serotonin reality or have no basis in reality. Delusions are
syndrome occurs with clients who take a beliefs not based in reality. Disorganized speech
combination of antidepressant medications. is characterized by jumping from one topic to
Olanzapine doesn’t cause euphoria, and the next or using unrelated words. An idea of
extrapyramidal adverse reactions aren’t a reference is a belief that an unrelated situation
problem. However, the client should be aware of holds special meaning for the client.
adverse effects such as tardive dyskinesia. 99. Answer: (C) Regression
94. Answer: (A) Lack of honesty Rationale: Regression, a return to earlier
Rationale: Clients with antisocial personality behavior to reduce anxiety, is the basic defense
disorder tent to engage in acts of dishonesty, mechanism in schizophrenia. Projection is a
shown by lying. Clients with schizotypal defense mechanism in which one blames others
personality disorder tend to be superstitious. and attempts to justify actions; it’s used
Clients with histrionic personality disorders tend primarily by people with paranoid schizophrenia
to overreact to frustrations and and delusional disorder. Rationalization is a
disappointments, have temper tantrums, and defense mechanism used to justify one’s action.
seek attention. Repression is the basic defense mechanism in
95. Answer: (A) “I’m not going to look just at the the neuroses; it’s an involuntary exclusion of
negative things about myself” painful thoughts, feelings, or experiences from
Rationale: As the client makes progress on awareness.
improving self-esteem, self- blame and negative 100. Answer: (A) Should report feelings of
self-evaluation will decrease. Clients with restlessness or agitation at once
dependent personality disorder tend to feel Rationale: Agitation and restlessness are adverse
fragile and inadequate and would be extremely effect of haloperidol and can be treated with
unlikely to discuss their level of competence and antocholinergic drugs. Haloperidol isn’t likely to
progress. These clients focus on self and aren’t cause photosensitivity or control essential
envious or jealous. Individuals with dependent hypertension. Although the client may
personality disorders don’t take over situations experience increased concentration and activity,
because they see themselves as inept and these effects are due to a decreased in
inadequate. symptoms, not the drug itself.
96. Answer: (C) Assess for possible physical
problems such as rash
Rationale: Clients with schizophrenia generally
have poor visceral recognition because they live
so fully in their fantasy world. They need to have
as in-depth assessment of physical complaints
that may spill over into their delusional
symptoms. Talking with the client won’t provide
as assessment of his itching, and itching isn’t as
adverse reaction of antipsychotic drugs, calling
the physician to get the client’s medication
increased doesn’t address his physical
complaints.

15
15
PART III PRACTICE TEST I FOUNDATION OF 7. Sterile technique is used whenever:
NURSING a. Strict isolation is required
b. Terminal disinfection is performed
1. Which element in the circular chain of infection c. Invasive procedures are performed
can be eliminated by preserving skin integrity? d. Protective isolation is necessary
a. Host
b. Reservoir 8. Which of the following constitutes a break in
c. Mode of transmission sterile technique while preparing a sterile field
d. Portal of entry for a dressing change?
a. Using sterile forceps, rather than sterile
2. Which of the following will probably result in a gloves, to handle a sterile item
break in sterile technique for respiratory b. Touching the outside wrapper of
isolation? sterilized material without sterile gloves
a. Opening the patient’s window to the c. Placing a sterile object on the edge of
outside environment the sterile field
b. Turning on the patient’s room ventilator d. Pouring out a small amount of solution
c. Opening the door of the patient’s room (15 to 30 ml) before pouring the solution
leading into the hospital corridor into a sterile container
d. Failing to wear gloves when 9. A natural body defense that plays an active role
administering a bed bath in preventing infection is:
a. Yawning
3. Which of the following patients is at greater risk b. Body hair
for contracting an infection? c. Hiccupping
a. A patient with leukopenia d. Rapid eye movements
b. A patient receiving broad-spectrum
antibiotics 10. All of the following statement are true about
c. A postoperative patient who has donning sterile gloves except:
undergone orthopedic surgery a. The first glove should be picked up by
d. A newly diagnosed diabetic patient grasping the inside of the cuff.
b. The second glove should be picked up by
4. Effective hand washing requires the use of: inserting the gloved fingers under the
a. Soap or detergent to promote cuff outside the glove.
emulsification c. The gloves should be adjusted by sliding
b. Hot water to destroy bacteria the gloved fingers under the sterile cuff
c. A disinfectant to increase surface and pulling the glove over the wrist
tension d. The inside of the glove is considered
d. All of the above sterile

5. After routine patient contact, hand washing 11. When removing a contaminated gown, the nurse
should last at least: should be careful that the first thing she touches
a. 30 seconds is the:
b. 1 minute a. Waist tie and neck tie at the back of the
c. 2 minute gown
d. 3 minutes b. Waist tie in front of the gown
c. Cuffs of the gown
6. Which of the following procedures always d. Inside of the gown
requires surgical asepsis?
a. Vaginal instillation of conjugated 12. Which of the following nursing interventions is
estrogen considered the most effective form or universal
b. Urinary catheterization precautions?
c. Nasogastric tube insertion a. Cap all used needles before removing
d. Colostomy irrigation them from their syringes
b. Discard all used uncapped needles and b. Before the procedure, the patient should
syringes in an impenetrable protective remove all jewelry, metallic objects, and
container buttons above the waist
c. Wear gloves when administering IM c. A signed consent is not required
injections d. Eating, drinking, and medications are
d. Follow enteric precautions allowed before this test

13. All of the following measures are recommended 19. The most appropriate time for the nurse to
to prevent pressure ulcers except: obtain a sputum specimen for culture is:
a. Massaging the reddened are with lotion a. Early in the morning
b. Using a water or air mattress b. After the patient eats a light breakfast
c. Adhering to a schedule for positioning c. After aerosol therapy
and turning d. After chest physiotherapy
d. Providing meticulous skin care
20. A patient with no known allergies is to receive
14. Which of the following blood tests should be penicillin every 6 hours.
performed before a blood transfusion? 21. When administering the medication, the nurse
a. Prothrombin and coagulation time observes a fine rash on the patient’s skin. The
b. Blood typing and cross-matching most appropriate nursing action would be to:
c. Bleeding and clotting time a. Withhold the moderation and notify the
d. Complete blood count (CBC) and physician
electrolyte levels. b. Administer the medication and notify
the physician
15. The primary purpose of a platelet count is to c. Administer the medication with an
evaluate the: antihistamine
a. Potential for clot formation d. Apply corn starch soaks to the rash
b. Potential for bleeding
c. Presence of an antigen-antibody 22. All of the following nursing interventions are
response correct when using the Z- track method of drug
d. Presence of cardiac enzymes injection except:
a. Prepare the injection site with alcohol
16. Which of the following white blood cell (WBC) b. Use a needle that’s a least 1” long
counts clearly indicates leukocytosis? c. Aspirate for blood before injection
a. 4,500/mm³ d. Rub the site vigorously after the
b. 7,000/mm³ injection to promote absorption
c. 10,000/mm³
d. 25,000/mm³ 23. The correct method for determining the
vastus lateralis site for I.M. injection is to:
17. After 5 days of diuretic therapy with 20mg of a. Locate the upper aspect of the upper
furosemide (Lasix) daily, a patient begins to outer quadrant of the buttock about 5 to
exhibit fatigue, muscle cramping and muscle 8 cm below the iliac crest
weakness. These symptoms probably indicate b. Palpate the lower edge of the acromion
that the patient is experiencing: process and the midpoint lateral aspect
a. Hypokalemia of the arm
b. Hyperkalemia c. Palpate a 1” circular area anterior to the
c. Anorexia umbilicus
d. Dysphagia d. Divide the area between the greater
femoral trochanter and the lateral
18. Which of the following statements about chest femoral condyle into thirds, and select
X-ray is false? the middle third on the anterior of the
a. No contradictions exist for this test thigh

15
15
24. The mid-deltoid injection site is seldom used for a. Fever
I.M. injections because it: b. Chronic Obstructive Pulmonary Disease
a. Can accommodate only 1 ml or less of c. Renal Failure
medication d. Dehydration
b. Bruises too easily
c. Can be used only when the patient is 32. All of the following are common signs and
lying down symptoms of phlebitis except:
d. Does not readily parenteral medication a. Pain or discomfort at the IV insertion site
b. Edema and warmth at the IV insertion
25. The appropriate needle size for insulin injection site
is: c. A red streak exiting the IV insertion site
a. 18G, 1 ½” long d. Frank bleeding at the insertion site
b. 22G, 1” long
c. 22G, 1 ½” long 33. The best way of determining whether a patient
d. 25G, 5/8” long has learned to instill ear medication properly is
for the nurse to:
26. The appropriate needle gauge for intradermal a. Ask the patient if he/she has used ear
injection is: drops before
a. 20G b. Have the patient repeat the nurse’s
b. 22G instructions using her own words
c. 25G c. Demonstrate the procedure to the
d. 26G patient and encourage to ask questions
d. Ask the patient to demonstrate the
27. Parenteral penicillin can be administered as an: procedure
a. IM injection or an IV solution
b. IV or an intradermal injection 34. Which of the following types of medications can
c. Intradermal or subcutaneous injection be administered via gastrostomy tube?
d. IM or a subcutaneous injection a. Any oral medications
b. Capsules whole contents are dissolve in
28. The physician orders gr 10 of aspirin for a water
patient. The equivalent dose in milligrams is: c. Enteric-coated tablets that are
a. 0.6 mg thoroughly dissolved in water
b. 10 mg d. Most tablets designed for oral use,
c. 60 mg except for extended-duration
d. 600 mg compounds

29. The physician orders an IV solution of dextrose 35. A patient who develops hives after receiving an
5% in water at 100ml/hour. What would the antibiotic is exhibiting drug:
flow rate be if the drop factor is 15 gtt = 1 ml? a. Tolerance
a. 5 gtt/minute b. Idiosyncrasy
b. 13 gtt/minute c. Synergism
c. 25 gtt/minute d. Allergy
d. 50 gtt/minute
36. A patient has returned to his room after femoral
30. Which of the following is a sign or symptom of a arteriography. All of the following are
hemolytic reaction to blood transfusion? appropriate nursing interventions except:
a. Hemoglobinuria a. Assess femoral, popliteal, and pedal
b. Chest pain pulses every 15 minutes for 2 hours
c. Urticaria b. Check the pressure dressing for
d. Distended neck veins sanguineous drainage
c. Assess vital signs every 15 minutes for 2
31. Which of the following conditions may require hours
fluid restriction?
d. Order a hemoglobin and hematocrit 43. All of the following are good sources of vitamin A
count 1 hour after the arteriography except:
a. White potatoes
37. The nurse explains to a patient that a cough: b. Carrots
a. Is a protective response to clear the c. Apricots
respiratory tract of irritants d. Egg yolks
b. Is primarily a voluntary action
c. Is induced by the administration of an 44. Which of the following is a primary nursing
antitussive drug intervention necessary for all patients with a
d. Can be inhibited by “splinting” the Foley Catheter in place?
abdomen a. Maintain the drainage tubing and
collection bag level with the patient’s
38. An infected patient has chills and begins bladder
shivering. The best nursing intervention is to: b. Irrigate the patient with 1% Neosporin
a. Apply iced alcohol sponges solution three times a daily
b. Provide increased cool liquids c. Clamp the catheter for 1 hour every 4
c. Provide additional bedclothes hours to maintain the bladder’s elasticity
d. Provide increased ventilation d. Maintain the drainage tubing and
collection bag below bladder level to
39. A clinical nurse specialist is a nurse who has: facilitate drainage by gravity
a. Been certified by the National League for
Nursing 45. The ELISA test is used to:
b. Received credentials from the Philippine a. Screen blood donors for antibodies to
Nurses’ Association human immunodeficiency virus (HIV)
c. Graduated from an associate degree b. Test blood to be used for transfusion for
program and is a registered professional HIV antibodies
nurse c. Aid in diagnosing a patient with AIDS
d. Completed a master’s degree in the d. All of the above
prescribed clinical area and is a
registered professional nurse. 46. The two blood vessels most commonly used for
TPN infusion are the:
40. The purpose of increasing urine acidity through a. Subclavian and jugular veins
dietary means is to: b. Brachial and subclavian veins
a. Decrease burning sensations c. Femoral and subclavian veins
b. Change the urine’s color d. Brachial and femoral veins
c. Change the urine’s concentration
d. Inhibit the growth of microorganisms 47. Effective skin disinfection before a surgical
procedure includes which of the following
41. Clay colored stools indicate: methods?
a. Upper GI bleeding a. Shaving the site on the day
b. Impending constipation before surgery
c. An effect of medication b. Applying a topical antiseptic to the skin
d. Bile obstruction on the evening before surgery
c. Having the patient take a tub bath on
42. In which step of the nursing process would the the morning of surgery
nurse ask a patient if the medication she d. Having the patient shower with an
administered relieved his pain? antiseptic soap on the evening v=before
a. Assessment and the morning of surgery
b. Analysis
c. Planning 48. When transferring a patient from a bed to a
d. Evaluation chair, the nurse should use which muscles to
avoid back injury?
a. Abdominal muscles

15
15
b. Back muscles
c. Leg muscles
d. Upper arm muscles

49. Thrombophlebitis typically develops in patients


with which of the following conditions?
a. Increases partial thromboplastin time
b. Acute pulsus paradoxus
c. An impaired or traumatized blood vessel
wall
d. Chronic Obstructive Pulmonary Disease
(COPD)

50. In a recumbent, immobilized patient, lung


ventilation can become altered, leading to such
respiratory complications as:
a. Respiratory acidosis, ateclectasis, and
hypostatic pneumonia
b. Appneustic breathing, atypical
pneumonia and respiratory alkalosis
c. Cheyne-Strokes respirations and
spontaneous pneumothorax
d. Kussmail’s respirations and
hypoventilation

51. Immobility impairs bladder elimination, resulting


in such disorders as
a. Increased urine acidity and relaxation of
the perineal muscles, causing
incontinence
b. Urine retention, bladder distention, and
infection
c. Diuresis, natriuresis, and decreased
urine specific gravity
d. Decreased calcium and phosphate levels
in the urine
ANSWERS AND RATIONALE – FOUNDATION OF to prepare them for reuse by another patient.
NURSING The purpose of protective (reverse) isolation is
to prevent a person with seriously impaired
1. D. In the circular chain of infection, pathogens resistance from coming into contact who
must be able to leave their reservoir and be potentially pathogenic organisms.
transmitted to a susceptible host through a 8. C. The edges of a sterile field are considered
portal of entry, such as broken skin. contaminated. When sterile items are allowed to
2. C. Respiratory isolation, like strict isolation, come in contact with the edges of the field, the
requires that the door to the door patient’s sterile items also become contaminated.
room remain closed. However, the patient’s 9. B. Hair on or within body areas, such as the
room should be well ventilated, so opening the nose, traps and holds particles that contain
window or turning on the ventricular is microorganisms. Yawning and hiccupping do not
desirable. The nurse does not need to wear prevent microorganisms from entering or
gloves for respiratory isolation, but good hand leaving the body. Rapid eye movement marks
washing is important for all types of isolation. the stage of sleep during which dreaming occurs.
3. A. Leukopenia is a decreased number of 10. D. The inside of the glove is always considered to
leukocytes (white blood cells), which are be clean, but not sterile.
important in resisting infection. None of the 11. A. The back of the gown is considered clean, the
other situations would put the patient at risk for front is contaminated. So, after removing gloves
contracting an infection; taking broad- spectrum and washing hands, the nurse should untie the
antibiotics might actually reduce the infection back of the gown; slowly move backward away
risk. from the gown, holding the inside of the gown
4. A. Soaps and detergents are used to help and keeping the edges off the floor; turn and
remove bacteria because of their ability to lower fold the gown inside out; discard it in a
the surface tension of water and act as contaminated linen container; then wash her
emulsifying agents. Hot water may lead to skin hands again.
irritation or burns. 12. B. According to the Centers for Disease Control
5. A. Depending on the degree of exposure to (CDC), blood-to-blood contact occurs most
pathogens, hand washing may last from 10 commonly when a health care worker attempts
seconds to 4 minutes. After routine patient to cap a used needle. Therefore, used needles
contact, hand washing for 30 seconds effectively should never be recapped; instead they should
minimizes the risk of pathogen transmission. be inserted in a specially designed puncture
6. B. The urinary system is normally free of resistant, labeled container. Wearing gloves is
microorganisms except at the urinary meatus. not always necessary when administering an I.M.
Any procedure that involves entering this system injection. Enteric precautions prevent the
must use surgically aseptic measures to maintain transfer of pathogens via feces.
a bacteria-free state. 13. A. Nurses and other health care professionals
7. C. All invasive procedures, including surgery, previously believed that massaging a reddened
catheter insertion, and administration of area with lotion would promote venous return
parenteral therapy, require sterile technique to and reduce edema to the area. However,
maintain a sterile environment. All equipment research has shown that massage only increases
must be sterile, and the nurse and the physician the likelihood of cellular ischemia and necrosis
must wear sterile gloves and maintain surgical to the area.
asepsis. In the operating room, the nurse and 14. B. Before a blood transfusion is performed, the
physician are required to wear sterile gowns, blood of the donor and recipient must be
gloves, masks, hair covers, and shoe covers for checked for compatibility. This is done by blood
all invasive procedures. Strict isolation requires typing (a test that determines a person’s blood
the use of clean gloves, masks, gowns and type) and cross-matching (a procedure that
equipment to prevent the transmission of highly determines the compatibility of the donor’s and
communicable diseases by contact or by recipient’s blood after the blood types has been
airborne routes. Terminal disinfection is the matched). If the blood specimens are
disinfection of all contaminated supplies and incompatible, hemolysis and antigen-antibody
equipment after a patient has been discharged reactions will occur.

15
15
15. A. Platelets are disk-shaped cells that are 21. D. The Z-track method is an I.M. injection
essential for blood coagulation. A platelet count technique in which the patient’s skin is pulled in
determines the number of thrombocytes in such a way that the needle track is sealed off
blood available for promoting hemostasis and after the injection. This procedure seals
assisting with blood coagulation after injury. It medication deep into the muscle, thereby
also is used to evaluate the patient’s potential minimizing skin staining and irritation. Rubbing
for bleeding; however, this is not its primary the injection site is contraindicated because it
purpose. The normal count ranges from 150,000 may cause the medication to extravasate into
to 350,000/mm3. A count of 100,000/mm3 or the skin.
less indicates a potential for bleeding; count of 22. D. The vastus lateralis, a long, thick muscle that
less than 20,000/mm3 is associated with extends the full length of the thigh, is viewed by
spontaneous bleeding. many clinicians as the site of choice for I.M.
16. D. Leukocytosis is any transient increase in the injections because it has relatively few major
number of white blood cells (leukocytes) in the nerves and blood vessels. The middle third of the
blood. Normal WBC counts range from 5,000 to muscle is recommended as the injection site.
100,000/mm3. Thus, a count of 25,000/mm3 The patient can be in a supine or sitting position
indicates leukocytosis. for an injection into this site.
17. A. Fatigue, muscle cramping, and muscle 23. A. The mid-deltoid injection site can
weaknesses are symptoms of hypokalemia (an accommodate only 1 ml or less of medication
inadequate potassium level), which is a potential because of its size and location (on the deltoid
side effect of diuretic therapy. The physician muscle of the arm, close to the brachial artery
usually orders supplemental potassium to and radial nerve).
prevent hypokalemia in patients receiving 24. D. A 25G, 5/8” needle is the recommended size
diuretics. Anorexia is another symptom of for insulin injection because insulin is
hypokalemia. Dysphagia means difficulty administered by the subcutaneous route. An
swallowing. 18G, 1 ½” needle is usually used for I.M.
18. A. Pregnancy or suspected pregnancy is the only injections in children, typically in the vastus
contraindication for a chest X-ray. However, if a lateralis. A 22G, 1 ½” needle is usually used for
chest X-ray is necessary, the patient can wear a adult I.M. injections, which are typically
lead apron to protect the pelvic region from administered in the vastus lateralis or
radiation. Jewelry, metallic objects, and buttons ventrogluteal site.
would interfere with the X-ray and thus should 25. D. Because an intradermal injection does not
not be worn above the waist. A signed consent is penetrate deeply into the skin, a small-bore 25G
not required because a chest X-ray is not an needle is recommended. This type of injection is
invasive examination. Eating, drinking and used primarily to administer antigens to
medications are allowed because the X-ray is of evaluate reactions for allergy or sensitivity
the chest, not the abdominal region. studies. A 20G needle is usually used for I.M.
19. A. Obtaining a sputum specimen early in this injections of oil- based medications; a 22G
morning ensures an adequate supply of bacteria needle for I.M. injections; and a 25G needle, for
for culturing and decreases the risk of I.M. injections; and a 25G needle, for
contamination from food or medication. subcutaneous insulin injections.
20. A. Initial sensitivity to penicillin is commonly 26. A. Parenteral penicillin can be administered I.M.
manifested by a skin rash, even in individuals or added to a solution and given I.V. It cannot be
who have not been allergic to it previously. administered subcutaneously or intradermally.
Because of the danger of anaphylactic shock, he 27. D. gr 10 x 60mg/gr 1 = 600 mg
nurse should withhold the drug and notify the 28. C. 100ml/60 min X 15 gtt/ 1 ml = 25 gtt/minute
physician, who may choose to substitute 29. A. Hemoglobinuria, the abnormal presence of
another drug. Administering an antihistamine is hemoglobin in the urine, indicates a hemolytic
a dependent nursing intervention that requires a reaction (incompatibility of the donor’s and
written physician’s order. Although applying recipient’s blood). In this reaction, antibodies in
corn starch to the rash may relieve discomfort, it the recipient’s plasma combine rapidly with
is not the nurse’s top priority in such a donor RBC’s; the cells are hemolyzed in either
potentially life-threatening situation. circulatory or reticuloendothelial system.
Hemolysis occurs more rapidly in ABO 36. A. Coughing, a protective response that clears
incompatibilities than in Rh incompatibilities. the respiratory tract of irritants, usually is
Chest pain and urticarial may be symptoms of involuntary; however it can be voluntary, as
impending anaphylaxis. Distended neck veins are when a patient is taught to perform coughing
an indication of hypervolemia. exercises. An antitussive drug inhibits coughing.
30. C. In real failure, the kidney loses their ability to Splinting the abdomen supports the abdominal
effectively eliminate wastes and fluids. Because muscles when a patient coughs.
of this, limiting the patient’s intake of oral and 37. C. In an infected patient, shivering results from
I.V. fluids may be necessary. Fever, chronic the body’s attempt to increase heat production
obstructive pulmonary disease, and dehydration and the production of neutrophils and
are conditions for which fluids should be phagocytotic action through increased skeletal
encouraged. muscle tension and contractions. Initial
31. D. Phlebitis, the inflammation of a vein, can be vasoconstriction may cause skin to feel cold to
caused by chemical irritants (I.V. solutions or the touch. Applying additional bed clothes helps
medications), mechanical irritants (the needle or to equalize the body temperature and stop the
catheter used during venipuncture or chills. Attempts to cool the body result in further
cannulation), or a localized allergic reaction to shivering, increased metabloism, and thus
the needle or catheter. Signs and symptoms of increased heat production.
phlebitis include pain or discomfort, edema and 38. D. A clinical nurse specialist must have
heat at the I.V. insertion site, and a red streak completed a master’s degree in a clinical
going up the arm or leg from the I.V. insertion specialty and be a registered professional nurse.
site. The National League of Nursing accredits
32. D. Return demonstration provides the most educational programs in nursing and provides a
certain evidence for evaluating the effectiveness testing service to evaluate student nursing
of patient teaching. competence but it does not certify nurses. The
33. D. Capsules, enteric-coated tablets, and most American Nurses Association identifies
extended duration or sustained release products requirements for certification and offers
should not be dissolved for use in a gastrostomy examinations for certification in many areas of
tube. They are pharmaceutically manufactured nursing, such as medical surgical nursing. These
in these forms for valid reasons, and altering certification (credentialing) demonstrates that
them destroys their purpose. The nurse should the nurse has the knowledge and the ability to
seek an alternate physician’s order when an provide high quality nursing care in the area of
ordered medication is inappropriate for delivery her certification. A graduate of an associate
by tube. degree program is not a clinical nurse specialist:
34. D. A drug-allergy is an adverse reaction resulting however, she is prepared to provide bed side
from an immunologic response following a nursing with a high degree of knowledge and
previous sensitizing exposure to the drug. The skill. She must successfully complete the
reaction can range from a rash or hives to licensing examination to become a registered
anaphylactic shock. Tolerance to a drug means professional nurse.
that the patient experiences a decreasing 39. D. Microorganisms usually do not grow in an
physiologic response to repeated administration acidic environment.
of the drug in the same dosage. Idiosyncrasy is 40. D. Bile colors the stool brown. Any inflammation
an individual’s unique hypersensitivity to a drug, or obstruction that impairs bile flow will affect
food, or other substance; it appears to be the stool pigment, yielding light, clay-colored
genetically determined. Synergism, is a drug stool. Upper GI bleeding results in black or tarry
interaction in which the sum of the drug’s stool. Constipation is characterized by small,
combined effects is greater than that of their hard masses. Many medications and foods will
separate effects. discolor stool – for example, drugs containing
35. D. A hemoglobin and hematocrit count would be iron turn stool black.; beets turn stool red.
ordered by the physician if bleeding were 41. D. In the evaluation step of the nursing process,
suspected. The other answers are appropriate the nurse must decide whether the patient has
nursing interventions for a patient who has achieved the expected outcome that was
undergone femoral arteriography. identified in the planning phase.

16
16
42. A. The main sources of vitamin A are yellow and not necessarily impede venous return of injure
green vegetables (such as carrots, sweet vessel walls.
potatoes, squash, spinach, collard greens, 49. A. Because of restricted respiratory movement, a
broccoli, and cabbage) and yellow fruits (such as recumbent, immobilize patient is at particular
apricots, and cantaloupe). Animal sources risk for respiratory acidosis from poor gas
include liver, kidneys, cream, butter, and egg exchange; atelectasis from reduced surfactant
yolks. and accumulated mucus in the bronchioles, and
43. D. Maintaing the drainage tubing and collection hypostatic pneumonia from bacterial growth
bag level with the patient’s bladder could result caused by stasis of mucus secretions.
in reflux of urine into the kidney. Irrigating the 50. B. The immobilized patient commonly suffers
bladder with Neosporin and clamping the from urine retention caused by decreased
catheter for 1 hour every 4 hours must be muscle tone in the perineum. This leads to
prescribed by a physician. bladder distention and urine stagnation, which
44. D. The ELISA test of venous blood is used to provide an excellent medium for bacterial
assess blood and potential blood donors to growth leading to infection. Immobility also
human immunodeficiency virus (HIV). A positive results in more alkaline urine with excessive
ELISA test combined with various signs and amounts of calcium, sodium and phosphate, a
symptoms helps to diagnose acquired gradual decrease in urine production, and an
immunodeficiency syndrome (AIDS) increased specific gravity.
45. D. Tachypnea (an abnormally rapid rate of
breathing) would indicate that the patient was
still hypoxic (deficient in oxygen).The partial
pressures of arterial oxygen and carbon dioxide
listed are within the normal range. Eupnea refers
to normal respiration.
46. D. Studies have shown that showering with an
antiseptic soap before surgery is the most
effective method of removing microorganisms
from the skin. Shaving the site of the intended
surgery might cause breaks in the skin, thereby
increasing the risk of infection; however, if
indicated, shaving, should be done immediately
before surgery, not the day before. A topical
antiseptic would not remove microorganisms
and would be beneficial only after proper
cleaning and rinsing. Tub bathing might transfer
organisms to another body site rather than rinse
them away.
47. C. The leg muscles are the strongest muscles in
the body and should bear the greatest stress
when lifting. Muscles of the abdomen, back, and
upper arms may be easily injured.
48. C. The factors, known as Virchow’s triad,
collectively predispose a patient to
thromboplebitis; impaired venous return to the
heart, blood hypercoagulability, and injury to a
blood vessel wall. Increased partial
thromboplastin time indicates a prolonged
bleeding time during fibrin clot formation,
commonly the result of anticoagulant (heparin)
therapy. Arterial blood disorders (such as pulsus
paradoxus) and lung diseases (such as COPD) do
PRACTICE TEST II Maternal and Child Health
7. The client tells the nurse that her last menstrual
1. For the client who is using oral contraceptives, period started on January 14 and ended on
the nurse informs the client about the need to January 20. Using Nagele’s rule, the nurse
take the pill at the same time each day to determines her EDD to be which of the
accomplish which of the following? following?
a. Decrease the incidence of nausea a. September 27
b. Maintain hormonal levels b. October 21
c. Reduce side effects c. November 7
d. Prevent drug interactions d. December 27

2. When teaching a client about contraception. 8. When taking an obstetrical history on a pregnant
Which of the following would the nurse include client who states, “I had a son born at 38 weeks
as the most effective method for preventing gestation, a daughter born at 30 weeks gestation
sexually transmitted infections? and I lost a baby at about 8 weeks,” the nurse
a. Spermicides should record her obstetrical history as which of
b. Diaphragm the following?
c. Condoms a. G2 T2 P0 A0 L2
d. Vasectomy b. G3 T1 P1 A0 L2
c. G3 T2 P0 A0 L2
3. When preparing a woman who is 2 days d. G4 T1 P1 A1 L2
postpartum for discharge, recommendations for
which of the following contraceptive methods 9. When preparing to listen to the fetal heart rate
would be avoided? at 12 weeks’ gestation, the nurse would use
a. Diaphragm which of the following?
b. Female condom a. Stethoscope placed midline at the
c. Oral contraceptives umbilicus
d. Rhythm method b. Doppler placed midline at the
suprapubic region
4. For which of the following clients would the c. Fetoscope placed midway between the
nurse expect that an intrauterine device would umbilicus and the xiphoid process
not be recommended? d. External electronic fetal monitor placed
a. Woman over age 35 at the umbilicus
b. Nulliparous woman
c. Promiscuous young adult 10. When developing a plan of care for a client
d. Postpartum client newly diagnosed with gestational diabetes,
which of the following instructions would be the
5. A client in her third trimester tells the nurse, priority?
“I’m constipated all the time!” Which of the a. Dietary intake
following should the nurse recommend? b. Medication
a. Daily enemas c. Exercise
b. Laxatives d. Glucose monitoring
c. Increased fiber intake
d. Decreased fluid intake 11. A client at 24 weeks gestation has gained 6
pounds in 4 weeks. Which of the following would
6. Which of the following would the nurse use as be the priority when assessing the client?
the basis for the teaching plan when caring for a a. Glucosuria
pregnant teenager concerned about gaining too b. Depression
much weight during pregnancy? c. Hand/face edema
a. 10 pounds per trimester d. Dietary intake
b. 1 pound per week for 40 weeks
c. ½ pound per week for 40 weeks 12. A client 12 weeks’ pregnant come to
d. A total gain of 25 to 30 pounds the emergency department with

16
16
abdominal
16
cramping and moderate vaginal bleeding. a. A dark red discharge on a 2-day
Speculum examination reveals 2 to 3 cms postpartum client
cervical dilation. The nurse would document b. A pink to brownish discharge on a client
these findings as which of the following? who is 5 days postpartum
a. Threatened abortion c. Almost colorless to creamy discharge on
b. Imminent abortion a client 2 weeks after delivery
c. Complete abortion d. A bright red discharge 5 days after
d. Missed abortion delivery

13. Which of the following would be the priority 18. A postpartum client has a temperature of
nursing diagnosis for a client with an ectopic 101.4ºF, with a uterus that is tender when
pregnancy? palpated, remains unusually large, and not
a. Risk for infection descending as normally expected. Which of the
b. Pain following should the nurse assess next?
c. Knowledge Deficit a. Lochia
d. Anticipatory Grieving b. Breasts
c. Incision
14. Before assessing the postpartum client’s uterus d. Urine
for firmness and position in relation to the
umbilicus and midline, which of the following 19. Which of the following is the priority focus of
should the nurse do first? nursing practice with the current early
a. Assess the vital signs postpartum discharge?
b. Administer analgesia a. Promoting comfort and restoration of
c. Ambulate her in the hall health
d. Assist her to urinate b. Exploring the emotional status of the
family
15. Which of the following should the nurse do c. Facilitating safe and effective self-and
when a primipara who is lactating tells the nurse newborn care
that she has sore nipples? d. Teaching about the importance of family
a. Tell her to breast feed more frequently planning
b. Administer a narcotic before breast
feeding 20. Which of the following actions would be least
c. Encourage her to wear a nursing effective in maintaining a neutral thermal
brassiere environment for the newborn?
d. Use soap and water to clean the nipples a. Placing infant under radiant warmer
after bathing
16. The nurse assesses the vital signs of a client, 4 b. Covering the scale with a warmed
hours’ postpartum that are as follows: BP 90/60; blanket prior to weighing
temperature 100.4ºF; pulse 100 weak, thready; c. Placing crib close to nursery window for
R 20 per minute. Which of the following should family viewing
the nurse do first? d. Covering the infant’s head with a knit
a. Report the temperature to the physician stockinette
b. Recheck the blood pressure with
another cuff 21. A newborn who has an asymmetrical Moro
c. Assess the uterus for firmness and reflex response should be further assessed for
position which of the following?
d. Determine the amount of lochia a. Talipes equinovarus
b. Fractured clavicle
17. The nurse assesses the postpartum vaginal c. Congenital hypothyroidism
discharge (lochia) on four clients. Which of the d. Increased intracranial pressure
following assessments would warrant
notification of the physician?
22. During the first 4 hours after a male b. 3 ounces
circumcision, assessing for which of the c. 4 ounces
following is the priority? d. 6 ounces
a. Infection
b. Hemorrhage 27. The postterm neonate with meconium-stained
c. Discomfort amniotic fluid needs care designed to especially
d. Dehydration monitor for which of the following?
a. Respiratory problems
23. The mother asks the nurse. “What’s wrong with b. Gastrointestinal problems
my son’s breasts? Why are they so enlarged?” c. Integumentary problems
Whish of the following would be the best d. Elimination problems
response by the nurse?
a. “The breast tissue is inflamed from the 28. When measuring a client’s fundal height, which
trauma experienced with birth” of the following techniques denotes the correct
b. “A decrease in material hormones method of measurement used by the nurse?
present before birth causes a. From the xiphoid process to the
enlargement,” umbilicus
c. “You should discuss this with your b. From the symphysis pubis to the xiphoid
doctor. It could be a malignancy” process
d. “The tissue has hypertrophied while the c. From the symphysis pubis to the fundus
baby was in the uterus” d. From the fundus to the umbilicus

24. Immediately after birth the nurse notes the 29. A client with severe preeclampsia is admitted
following on a male newborn: respirations 78; with of BP 160/110, proteinuria, and severe
apical hearth rate 160 BPM, nostril flaring; mild pitting edema. Which of the following would be
intercostal retractions; and grunting at the end most important to include in the client’s plan of
of expiration. Which of the following should the care?
nurse do? a. Daily weights
a. Call the assessment data to the b. Seizure precautions
physician’s attention c. Right lateral positioning
b. Start oxygen per nasal cannula at 2 d. Stress reduction
L/min.
c. Suction the infant’s mouth and nares 30. A postpartum primipara asks the nurse, “When
d. Recognize this as normal first period of can we have sexual intercourse again?” Which of
reactivity the following would be the nurse’s best
response?
25. The nurse hears a mother telling a friend on the a. “Anytime you both want to.”
telephone about umbilical cord care. Which of b. “As soon as choose a contraceptive
the following statements by the mother method.”
indicates effective teaching? c. “When the discharge has stopped and
a. “Daily soap and water cleansing is best” the incision is healed.”
b. ‘Alcohol helps it dry and kills germs” d. “After your 6 weeks examination.”
c. “An antibiotic ointment applied daily
prevents infection” 31. When preparing to administer the vitamin K
d. “He can have a tub bath each day” injection to a neonate, the nurse would select
which of the following sites as appropriate for
26. A newborn weighing 3000 grams and feeding the injection?
every 4 hours needs 120 calories/kg of body a. Deltoid muscle
weight every 24 hours for proper growth and b. Anterior femoris muscle
development. How many ounces of 20 cal/oz c. Vastus lateralis muscle
formula should this newborn receive at each d. Gluteus maximus muscle
feeding to meet nutritional needs?
a. 2 ounces

16
16
32. When performing a pelvic examination, the understanding that breathing techniques are
nurse observes a red swollen area on the right most important in achieving which of the
side of the vaginal orifice. The nurse would following?
document this as enlargement of which of the a. Eliminate pain and give the expectant
following? parents something to do
a. Clitoris b. Reduce the risk of fetal distress by
b. Parotid gland increasing uteroplacental perfusion
c. Skene’s gland c. Facilitate relaxation, possibly reducing
d. Bartholin’s gland the perception of pain
d. Eliminate pain so that less analgesia and
33. To differentiate as a female, the hormonal anesthesia are needed
stimulation of the embryo that must occur
involves which of the following? 38. After 4 hours of active labor, the nurse notes
a. Increase in maternal estrogen secretion that the contractions of a primigravida client are
b. Decrease in maternal androgen not strong enough to dilate the cervix. Which of
secretion the following would the nurse anticipate doing?
c. Secretion of androgen by the fetal gonad a. Obtaining an order to begin IV oxytocin
d. Secretion of estrogen by the fetal gonad infusion
b. Administering a light sedative to allow
34. A client at 8 weeks’ gestation calls complaining the patient to rest for several hour
of slight nausea in the morning hours. Which of c. Preparing for a cesarean section for
the following client interventions should the failure to progress
nurse question? d. Increasing the encouragement to the
a. Taking 1 teaspoon of bicarbonate of patient when pushing begins
soda in an 8-ounce glass of water
b. Eating a few low-sodium crackers before 39. A multigravida at 38 weeks’ gestation is
getting out of bed admitted with painless, bright red bleeding and
c. Avoiding the intake of liquids in the mild contractions every 7 to 10 minutes. Which
morning hours of the following assessments should be avoided?
d. Eating six small meals a day instead of a. Maternal vital sign
thee large meals b. Fetal heart rate
c. Contraction monitoring
35. The nurse documents positive ballottement in d. Cervical dilation
the client’s prenatal record. The nurse
understands that this indicates which of the 40. Which of the following would be the nurse’s
following? most appropriate response to a client who asks
a. Palpable contractions on the abdomen why she must have a cesarean delivery if she has
b. Passive movement of the unengaged a complete placenta previa?
fetus a. “You will have to ask your physician
c. Fetal kicking felt by the client when he returns.”
d. Enlargement and softening of the uterus b. “You need a cesarean to prevent
hemorrhage.”
36. During a pelvic exam the nurse notes a purple- c. “The placenta is covering most of your
blue tinge of the cervix. The nurse documents cervix.”
this as which of the following? d. “The placenta is covering the opening of
a. Braxton-Hicks sign the uterus and blocking your baby.”
b. Chadwick’s sign
c. Goodell’s sign 41. The nurse understands that the fetal head is in
d. McDonald’s sign which of the following positions with a face
presentation?
37. During a prenatal class, the nurse explains the a. Completely flexed
rationale for breathing techniques during b. Completely extended
preparation for labor based on the c. Partially extended
d. Partially flexed b. Nurse-midwifery
c. Clinical nurse specialist
42. With a fetus in the left-anterior breech d. Prepared childbirth
presentation, the nurse would expect the fetal
heart rate would be most audible in which of the 48. A client has a midpelvic contracture from a
following areas? previous pelvic injury due to a motor vehicle
a. Above the maternal umbilicus and to the accident as a teenager. The nurse is aware that
right of midline this could prevent a fetus from passing through
b. In the lower-left maternal abdominal or around which structure during childbirth?
quadrant a. Symphysis pubis
c. In the lower-right maternal abdominal b. Sacral promontory
quadrant c. Ischial spines
d. Above the maternal umbilicus and to the d. Pubic arch
left of midline
49. When teaching a group of adolescents about
43. The amniotic fluid of a client has a greenish tint. variations in the length of the menstrual cycle,
The nurse interprets this to be the result of the nurse understands that the underlying
which of the following? mechanism is due to variations in which of the
a. Lanugo following phases?
b. Hydramnio a. Menstrual phase
c. Meconium b. Proliferative phase
d. Vernix c. Secretory phase
d. Ischemic phase
44. A patient is in labor and has just been told she
has a breech presentation. The nurse should be 50. When teaching a group of adolescents about
particularly alert for which of the following? male hormone production, which of the
a. Quickening following would the nurse include as being
b. Ophthalmia neonatorum produced by the Leydig cells?
c. Pica a. Follicle-stimulating hormone
d. Prolapsed umbilical cord b. Testosterone
c. Leuteinizing hormone
45. When describing dizygotic twins to a couple, on d. Gonadotropin releasing hormone
which of the following would the nurse base the
explanation?
a. Two ova fertilized by separate sperm
b. Sharing of a common placenta
c. Each ova with the same genotype
d. Sharing of a common chorion

46. Which of the following refers to the single cell


that reproduces itself after conception?
a. Chromosome
b. Blastocyst
c. Zygote
d. Trophoblast

47. In the late 1950s, consumers and health care


professionals began challenging the routine use
of analgesics and anesthetics during childbirth.
Which of the following was an outgrowth of this
concept?
a. Labor, delivery, recovery, postpartum
(LDRP)

16
16
ANSWERS AND RATIONALE – MATERNAL AND 4. C. An IUD may increase the risk of pelvic
CHILD HEALTH inflammatory disease, especially in women with
more than one sexual partner, because of the
1. B. Regular timely ingestion of oral contraceptives increased risk of sexually transmitted infections.
is necessary to maintain hormonal levels of the An UID should not be used if the woman has an
drugs to suppress the action of the active or chronic pelvic infection, postpartum
hypothalamus and anterior pituitary leading to infection, endometrial hyperplasia or carcinoma,
inappropriate secretion of FSH and LH. or uterine abnormalities. Age is not a factor in
Therefore, follicles do not mature, ovulation is determining the risks associated with IUD use.
inhibited, and pregnancy is prevented. The Most IUD users are over the age of 30. Although
estrogen content of the oral site contraceptive there is a slightly higher risk for infertility in
may cause the nausea, regardless of when the women who have never been pregnant, the IUD
pill is taken. Side effects and drug interactions is an acceptable option as long as the risk-
may occur with oral contraceptives regardless of benefit ratio is discussed. IUDs may be inserted
the time the pill is taken. immediately after delivery, but this is not
2. C. Condoms, when used correctly and recommended because of the increased risk and
consistently, are the most effective rate of expulsion at this time.
contraceptive method or barrier against 5. C. During the third trimester, the enlarging
bacterial and viral sexually transmitted uterus places pressure on the intestines. This
infections. Although spermicides kill sperm, they coupled with the effect of hormones on smooth
do not provide reliable protection against the muscle relaxation causes decreased intestinal
spread of sexually transmitted infections, motility (peristalsis). Increasing fiber in the diet
especially intracellular organisms such as HIV. will help fecal matter pass more quickly through
Insertion and removal of the diaphragm along the intestinal tract, thus decreasing the amount
with the use of the spermicides may cause of water that is absorbed. As a result, stool is
vaginal irritations, which could place the client at softer and easier to pass. Enemas could
risk for infection transmission. Male sterilization precipitate preterm labor and/or electrolyte loss
eliminates spermatozoa from the ejaculate, but and should be avoided. Laxatives may cause
it does not eliminate bacterial and/or viral preterm labor by stimulating peristalsis and may
microorganisms that can cause sexually interfere with the absorption of nutrients. Use
transmitted infections. for more than 1 week can also lead to laxative
3. A. The diaphragm must be fitted individually to dependency. Liquid in the diet helps provide a
ensure effectiveness. Because of the changes to semisolid, soft consistency to the stool. Eight to
the reproductive structures during pregnancy ten glasses of fluid per day are essential to
and following delivery, the diaphragm must be maintain hydration and promote stool
refitted, usually at the 6 weeks’ examination evacuation.
following childbirth or after a weight loss of 15 6. D. To ensure adequate fetal growth and
lbs or more. In addition, for maximum development during the 40 weeks of a
effectiveness, spermicidal jelly should be placed pregnancy, a total weight gain 25 to 30 pounds is
in the dome and around the rim. However, recommended: 1.5 pounds in the first 10 weeks;
spermicidal jelly should not be inserted into the 9 pounds by 30 weeks; and 27.5 pounds by 40
vagina until involution is completed at weeks. The pregnant woman should gain less
approximately 6 weeks. Use of a female condom weight in the first and second trimester than in
protects the reproductive system from the the third. During the first trimester, the client
introduction of semen or spermicides into the should only gain 1.5 pounds in the first 10
vagina and may be used after childbirth. Oral weeks, not 1 pound per week. A weight gain of
contraceptives may be started within the first ½ pound per week would be 20 pounds for the
postpartum week to ensure suppression of total pregnancy, less than the recommended
ovulation. For the couple who has determined amount.
the female’s fertile period, using the rhythm 7. B. To calculate the EDD by Nagele’s rule, add 7
method, avoidance of intercourse during this days to the first day of the last menstrual period
period, is safe and effective. and count back 3 months, changing the year
appropriately. To obtain a date of September 27,
7 days have been added to the last day of the suspected, which may be caused by fluid
LMP (rather than the first day of the LMP), plus 4 retention manifested by edema, especially of the
months (instead of 3 months) were counted hands and face. The three classic signs of
back. To obtain the date of November 7, 7 days preeclampsia are hypertension, edema, and
have been subtracted (instead of added) from proteinuria. Although urine is checked for
the first day of LMP plus November indicates glucose at each clinic visit, this is not the priority.
counting back 2 months (instead of 3 months) Depression may cause either anorexia or
from January. To obtain the date of December excessive food intake, leading to excessive
27, 7 days were added to the last day of the LMP weight gain or loss. This is not, however, the
(rather than the first day of the LMP) and priority consideration at this time. Weight gain
December indicates counting back only 1 month thought to be caused by excessive food intake
(instead of 3 months) from January. would require a 24-hour diet recall. However,
8. D. The client has been pregnant four times, excessive intake would not be the primary
including current pregnancy (G). Birth at 38 consideration for this client at this time.
weeks’ gestation is considered full term (T), 12. B. Cramping and vaginal bleeding coupled with
while birth form 20 weeks to 38 weeks is cervical dilation signifies that termination of the
considered preterm (P). A spontaneous abortion pregnancy is inevitable and cannot be
occurred at 8 weeks (A). She has two living prevented. Thus, the nurse would document an
children (L). imminent abortion. In a threatened abortion,
9. B. At 12 weeks gestation, the uterus rises out of cramping and vaginal bleeding are present, but
the pelvis and is palpable above the symphysis there is no cervical dilation. The symptoms may
pubis. The Doppler intensifies the sound of the subside or progress to abortion. In a complete
fetal pulse rate so it is audible. The uterus has abortion all the products of conception are
merely risen out of the pelvis into the abdominal expelled. A missed abortion is early fetal
cavity and is not at the level of the umbilicus. intrauterine death without expulsion of the
The fetal heart rate at this age is not audible products of conception.
with a stethoscope. The uterus at 12 weeks is 13. B. For the client with an ectopic pregnancy,
just above the symphysis pubis in the abdominal lower abdominal pain, usually unilateral, is the
cavity, not midway between the umbilicus and primary symptom. Thus, pain is the priority.
the xiphoid process. At 12 weeks the FHR would Although the potential for infection is always
be difficult to auscultate with a fetoscope. present, the risk is low in ectopic pregnancy
Although the external electronic fetal monitor because pathogenic microorganisms have not
would project the FHR, the uterus has not risen been introduced from external sources. The
to the umbilicus at 12 weeks. client may have a limited knowledge of the
10. A. Although all of the choices are important in pathology and treatment of the condition and
the management of diabetes, diet therapy is the will most likely experience grieving, but this is
mainstay of the treatment plan and should not the priority at this time.
always be the priority. Women diagnosed with 14. D. Before uterine assessment is performed, it is
gestational diabetes generally need only diet essential that the woman empty her bladder. A
therapy without medication to control their full bladder will interfere with the accuracy of
blood sugar levels. Exercise, is important for all the assessment by elevating the uterus and
pregnant women and especially for diabetic displacing to the side of the midline. Vital sign
women, because it burns up glucose, thus assessment is not necessary unless an
decreasing blood sugar. However, dietary intake, abnormality in uterine assessment is identified.
not exercise, is the priority. All pregnant women Uterine assessment should not cause acute pain
with diabetes should have periodic monitoring that requires administration of analgesia.
of serum glucose. However, those with Ambulating the client is an essential component
gestational diabetes generally do not need daily of postpartum care, but is not necessary prior to
glucose monitoring. The standard of care assessment of the uterus.
recommends a fasting and 2- hour postprandial 15. A. Feeding more frequently, about every 2
blood sugar level every 2 weeks. hours, will decrease the infant’s frantic, vigorous
11. C. After 20 weeks’ gestation, when there is a sucking from hunger and will decrease breast
rapid weight gain, preeclampsia should be engorgement, soften the breast, and promote

16
16
ease of correct latching-on for feeding. Narcotics 18. A. The data suggests an infection of the
administered prior to breast feeding are passed endometrial lining of the uterus. The lochia may
through the breast milk to the infant, causing be decreased or copious, dark brown in
excessive sleepiness. Nipple soreness is not appearance, and foul smelling, providing further
severe enough to warrant narcotic analgesia. All evidence of a possible infection. All the client’s
postpartum clients, especially lactating mothers, data indicate a uterine problem, not a breast
should wear a supportive brassiere with wide problem. Typically, transient fever, usually
cotton straps. This does not, however, prevent 101ºF, may be present with breast
or reduce nipple soreness. Soaps are drying to engorgement. Symptoms of mastitis include
the skin of the nipples and should not be used influenza-like manifestations. Localized infection
on the breasts of lactating mothers. Dry nipple of an episiotomy or C-section incision rarely
skin predisposes to cracks and fissures, which causes systemic symptoms, and uterine
can become sore and painful. involution would not be affected. The client data
16. D. A weak, thready pulse elevated to 100 BPM do not include dysuria, frequency, or urgency,
may indicate impending hemorrhagic shock. An symptoms of urinary tract infections, which
increased pulse is a compensatory mechanism of would necessitate assessing the client’s urine.
the body in response to decreased fluid volume. 19. C. Because of early postpartum discharge and
Thus, the nurse should check the amount of limited time for teaching, the nurse’s priority is
lochia present. Temperatures up to 100.48F in to facilitate the safe and effective care of the
the first 24 hours after birth are related to the client and newborn. Although promoting
dehydrating effects of labor and are considered comfort and restoration of health, exploring the
normal. Although rechecking the blood pressure family’s emotional status, and teaching about
may be a correct choice of action, it is not the family planning are important in
first action that should be implemented in light postpartum/newborn nursing care, they are not
of the other data. The data indicate a potential the priority focus in the limited time presented
impending hemorrhage. Assessing the uterus for by early post-partum discharge.
firmness and position in relation to the umbilicus 20. C. Heat loss by radiation occurs when the
and midline is important, but the nurse should infant’s crib is placed too near cold walls or
check the extent of vaginal bleeding first. Then it windows. Thus placing the newborn’s crib close
would be appropriate to check the uterus, which to the viewing window would be least effective.
may be a possible cause of the hemorrhage. Body heat is lost through evaporation during
17. D. Any bright red vaginal discharge would be bathing. Placing the infant under the radiant
considered abnormal, but especially 5 days after warmer after bathing will assist the infant to be
delivery, when the lochia is typically pink to rewarmed. Covering the scale with a warmed
brownish. Lochia rubra, a dark red discharge, is blanket prior to weighing prevents heat loss
present for 2 to 3 days after delivery. Bright red through conduction. A knit cap prevents heat
vaginal bleeding at this time suggests late loss from the head a large head, a large body
postpartum hemorrhage, which occurs after the surface area of the newborn’s body.
first 24 hours following delivery and is generally 21. B. A fractured clavicle would prevent the normal
caused by retained placental fragments or Moro response of symmetrical sequential
bleeding disorders. Lochia rubra is the normal extension and abduction of the arms followed
dark red discharge occurring in the first 2 to 3 by flexion and adduction. In talipes equinovarus
days after delivery, containing epithelial cells, (clubfoot) the foot is turned medially, and in
erythrocyes, leukocytes and decidua. Lochia plantar flexion, with the heel elevated. The feet
serosa is a pink to brownish serosanguineous are not involved with the Moro reflex.
discharge occurring from 3 to 10 days after Hypothyroiddism has no effect on the primitive
delivery that contains decidua, erythrocytes, reflexes. Absence of the Moror reflex is the most
leukocytes, cervical mucus, and microorganisms. significant single indicator of central nervous
Lochia alba is an almost colorless to yellowish system status, but it is not a sign of increased
discharge occurring from 10 days to 3 weeks intracranial pressure.
after delivery and containing leukocytes, 22. B. Hemorrhage is a potential risk following any
decidua, epithelial cells, fat, cervical mucus, surgical procedure. Although the infant has been
cholesterol crystals, and bacteria. given vitamin K to facilitate clotting, the
prophylactic dose is often not sufficient to for gastrointestinal problems. Even though the
prevent bleeding. Although infection is a skin is stained with meconium, it is noninfectious
possibility, signs will not appear within 4 hours (sterile) and nonirritating. The postterm
after the surgical procedure. The primary meconium- stained infant is not at additional risk
discomfort of circumcision occurs during the for bowel or urinary problems.
surgical procedure, not afterward. Although 28. C. The nurse should use a nonelastic, flexible,
feedings are withheld prior to the circumcision, paper measuring tape, placing the zero point on
the chances of dehydration are minimal. the superior border of the symphysis pubis and
23. B. The presence of excessive estrogen and stretching the tape across the abdomen at the
progesterone in the maternal- fetal blood midline to the top of the fundus. The xiphoid and
followed by prompt withdrawal at birth umbilicus are not appropriate landmarks to use
precipitates breast engorgement, which will when measuring the height of the fundus
spontaneously resolve in 4 to 5 days after birth. (McDonald’s measurement).
The trauma of the birth process does not cause 29. B. Women hospitalized with severe
inflammation of the newborn’s breast tissue. preeclampsia need decreased CNS stimulation to
Newborns do not have breast malignancy. This prevent a seizure. Seizure precautions provide
reply by the nurse would cause the mother to environmental safety should a seizure occur.
have undue anxiety. Breast tissue does not Because of edema, daily weight is important but
hypertrophy in the fetus or newborns. not the priority. Preclampsia causes vasospasm
24. D. The first 15 minutes to 1 hour after birth is and therefore can reduce utero-placental
the first period of reactivity involving respiratory perfusion. The client should be placed on her left
and circulatory adaptation to extrauterine life. side to maximize blood flow, reduce blood
The data given reflect the normal changes during pressure, and promote diuresis. Interventions to
this time period. The infant’s assessment data reduce stress and anxiety are very important to
reflect normal adaptation. Thus, the physician facilitate coping and a sense of control, but
does not need to be notified and oxygen is not seizure precautions are the priority.
needed. The data do not indicate the presence 30. C. Cessation of the lochial discharge signifies
of choking, gagging or coughing, which are signs healing of the endometrium. Risk of hemorrhage
of excessive secretions. Suctioning is not and infection are minimal 3 weeks after a
necessary. normal vaginal delivery. Telling the client
25. B. Application of 70% isopropyl alcohol to the anytime is inappropriate because this response
cord minimizes microorganisms (germicidal) and does not provide the client with the specific
promotes drying. The cord should be kept dry information she is requesting. Choice of a
until it falls off and the stump has healed. contraceptive method is important, but not the
Antibiotic ointment should only be used to treat specific criteria for safe resumption of sexual
an infection, not as a prophylaxis. Infants should activity. Culturally, the 6- weeks’ examination
not be submerged in a tub of water until the has been used as the time frame for resuming
cord falls off and the stump has completely sexual activity, but it may be resumed earlier.
healed. 31. C. The middle third of the vastus lateralis is the
26. B. To determine the amount of formula needed, preferred injection site for vitamin K
do the following mathematical calculation. 3 kg x administration because it is free of blood vessels
120 cal/kg per day = 360 calories/day feeding q 4 and nerves and is large enough to absorb the
hours = 6 feedings per day = 60 calories per medication. The deltoid muscle of a newborn is
feeding: 60 calories per feeding; 60 calories per not large enough for a newborn IM injection.
feeding with formula 20 cal/oz = 3 ounces per Injections into this muscle in a small child might
feeding. Based on the calculation. 2, 4 or 6 cause damage to the radial nerve. The anterior
ounces are incorrect. femoris muscle is the next safest muscle to use
27. A. Intrauterine anoxia may cause relaxation of in a newborn but is not the safest. Because of
the anal sphincter and emptying of meconium the proximity of the sciatic nerve, the gluteus
into the amniotic fluid. At birth some of the maximus muscle should not be until the child
meconium fluid may be aspirated, causing has been walking 2 years.
mechanical obstruction or chemical 32. D. Bartholin’s glands are the glands on either
pneumonitis. The infant is not at increased risk side of the vaginal orifice. The clitoris is female

17
17
erectile tissue found in the perineal area above 39. D. The signs indicate placenta previa and vaginal
the urethra. The parotid glands are open into the exam to determine cervical dilation would not
mouth. Skene’s glands open into the posterior be done because it could cause hemorrhage.
wall of the female urinary meatus. Assessing maternal vital signs can help
33. D. The fetal gonad must secrete estrogen for the determine maternal physiologic status. Fetal
embryo to differentiate as a female. An increase heart rate is important to assess fetal well-being
in maternal estrogen secretion does not affect and should be done. Monitoring the contractions
differentiation of the embryo, and maternal will help evaluate the progress of labor.
estrogen secretion occurs in every pregnancy. 40. D. A complete placenta previa occurs when the
Maternal androgen secretion remains the same placenta covers the opening of the uterus, thus
as before pregnancy and does not affect blocking the passageway for the baby. This
differentiation. Secretion of androgen by the response explains what a complete previa is and
fetal gonad would produce a male fetus. the reason the baby cannot come out except by
34. A. Using bicarbonate would increase the amount cesarean delivery. Telling the client to ask the
of sodium ingested, which can cause physician is a poor response and would increase
complications. Eating low-sodium crackers the patient’s anxiety. Although a cesarean would
would be appropriate. Since liquids can increase help to prevent hemorrhage, the statement does
nausea avoiding them in the morning hours not explain why the hemorrhage could occur.
when nausea is usually the strongest is With a complete previa, the placenta is covering
appropriate. Eating six small meals a day would the entire cervix, not just most of it.
keep the stomach full, which often decrease 41. B. With a face presentation, the head is
nausea. completely extended. With a vertex
35. B. Ballottement indicates passive movement of presentation, the head is completely or partially
the unengaged fetus. Ballottement is not a flexed. With a brow (forehead) presentation, the
contraction. Fetal kicking felt by the client head would be partially extended.
represents quickening. Enlargement and 42. D. With this presentation, the fetal upper torso
softening of the uterus is known as Piskacek’s and back face the left upper maternal abdominal
sign. wall. The fetal heart rate would be most audible
36. B. Chadwick’s sign refers to the purple-blue tinge above the maternal umbilicus and to the left of
of the cervix. Braxton Hicks contractions are the middle. The other positions would be
painless contractions beginning around the 4 th incorrect.
month. Goodell’s sign indicates softening of the 43. C. The greenish tint is due to the presence of
cervix. Flexibility of the uterus against the cervix meconium. Lanugo is the soft, downy hair on the
is known as McDonald’s sign. shoulders and back of the fetus. Hydramnios
37. C. Breathing techniques can raise the pain represents excessive amniotic fluid. Vernix is the
threshold and reduce the perception of pain. white, cheesy substance covering the fetus.
They also promote relaxation. Breathing 44. D. In a breech position, because of the space
techniques do not eliminate pain, but they can between the presenting part and the cervix,
reduce it. Positioning, not breathing, increases prolapse of the umbilical cord is common.
uteroplacental perfusion. Quickening is the woman’s first perception of
38. A. The client’s labor is hypotonic. The nurse fetal movement. Ophthalmia neonatorum
should call the physical and obtain an order for usually results from maternal gonorrhea and is
an infusion of oxytocin, which will assist the conjunctivitis. Pica refers to the oral intake of
uterus to contact more forcefully in an attempt nonfood substances.
to dilate the cervix. Administering light sedative 45. A. Dizygotic (fraternal) twins involve two ova
would be done for hypertonic uterine fertilized by separate sperm. Monozygotic
contractions. Preparing for cesarean section is (identical) twins involve a common placenta,
unnecessary at this time. Oxytocin would same genotype, and common chorion.
increase the uterine contractions and hopefully 46. C. The zygote is the single cell that reproduces
progress labor before a cesarean would be itself after conception. The chromosome is the
necessary. It is too early to anticipate client material that makes up the cell and is gained
pushing with contractions. from each parent. Blastocyst and trophoblast are
later terms for the embryo after zygote.
47. D. Prepared childbirth was the direct result of
the 1950’s challenging of the routine use of
analgesic and anesthetics during childbirth. The
LDRP was a much later concept and was not a
direct result of the challenging of routine use of
analgesics and anesthetics during childbirth.
Roles for nurse midwives and clinical nurse
specialists did not develop from this challenge.
48. C. The ischial spines are located in the mid-pelvic
region and could be narrowed due to the
previous pelvic injury. The symphysis pubis,
sacral promontory, and pubic arch are not part
of the mid-pelvis.
49. B. Variations in the length of the menstrual cycle
are due to variations in the proliferative phase.
The menstrual, secretory and ischemic phases
do not contribute to this variation.
50. B. Testosterone is produced by the Leyding cells
in the seminiferous tubules. Follicle-stimulating
hormone and leuteinzing hormone are released
by the anterior pituitary gland. The
hypothalamus is responsible for releasing
gonadotropin-releasing hormone.

17
17
MEDICAL SURGICAL NURSING d. Urethral discharge
1. Marco who was diagnosed with brain tumor was
scheduled for craniotomy. In preventing the 7. A client has undergone with penile implant.
development of cerebral edema after surgery, After 24 hrs of surgery, the client’s scrotum was
the nurse should expect the use of: edematous and painful. The nurse should:
a. Diuretics a. Assist the client with sitz bath
b. Antihypertensive b. Apply war soaks in the scrotum
c. Steroids c. Elevate the scrotum using a soft support
d. Anticonvulsants d. Prepare for a possible incision and
drainage.
2. Halfway through the administration of blood,
the female client complains of lumbar pain. After 8. Nurse hazel receives emergency laboratory
stopping the infusion Nurse Hazel should: results for a client with chest pain and
a. Increase the flow of normal saline immediately informs the physician. An increased
b. Assess the pain further myoglobin level suggests which of the following?
c. Notify the blood bank a. Liver disease
d. Obtain vital signs. b. Myocardial damage
c. Hypertension
3. Nurse Maureen knows that the positive d. Cancer
diagnosis for HIV infection is made based on
which of the following: 9. Nurse Maureen would expect the client with
a. A history of high risk sexual behaviors. mitral stenosis would demonstrate symptoms
b. Positive ELISA and western blot tests associated with congestion in the:
c. Identification of an associated a. Right atrium
opportunistic infection b. Superior vena cava
d. Evidence of extreme weight loss and c. Aorta
high fever d. Pulmonary

4. Nurse Maureen is aware that a client who has 10. A client has been diagnosed with hypertension.
been diagnosed with chronic renal failure The nurse priority nursing diagnosis would be:
recognizes an adequate amount of high-biologic- a. Ineffective health maintenance
value protein when the food the client selected b. Impaired skin integrity
from the menu was: c. Deficient fluid volume
a. Raw carrots d. Pain
b. Apple juice
c. Whole wheat bread 11. Nurse Hazel teaches the client with angina about
d. Cottage cheese common expected side effects of nitroglycerin
including:
5. Kenneth who has diagnosed with uremic a. high blood pressure
syndrome has the potential to develop b. stomach cramps
complications. Which among the following c. headache
complications should the nurse anticipates: d. shortness of breath
a. Flapping hand tremors
b. An elevated hematocrit level 12. The following are lipid abnormalities. Which of
c. Hypotension the following is a risk factor for the development
d. Hypokalemia of atherosclerosis and PVD?
a. High levels of low density lipid (LDL)
6. A client is admitted to the hospital with benign cholesterol
prostatic hyperplasia, the nurse most relevant b. High levels of high density lipid (HDL)
assessment would be: cholesterol
a. Flank pain radiating in the groin c. Low concentration triglycerides
b. Distention of the lower abdomen d. Low levels of LDL cholesterol.
c. Perineal edema
13. Which of the following represents a significant c. Treating the underlying cause
risk immediately after surgery for repair of aortic d. Replacing depleted blood products
aneurysm?
a. Potential wound infection 20. Which of the following findings is the best
b. Potential ineffective coping indication that fluid replacement for the client
c. Potential electrolyte balance with hypovolemic shock is adequate?
d. Potential alteration in renal perfusion a. Urine output greater than 30ml/hr
b. Respiratory rate of 21 breaths/minute
14. Nurse Josie should instruct the client to eat c. Diastolic blood pressure greater than 90
which of the following foods to obtain the best mmhg
supply of Vitamin B12? d. Systolic blood pressure greater than 110
a. dairy products mmhg
b. vegetables
c. Grains 21. Which of the following signs and symptoms
d. Broccoli would Nurse Maureen include in teaching plan
as an early manifestation of laryngeal cancer?
15. Karen has been diagnosed with aplastic anemia. a. Stomatitis
The nurse monitors for changes in which of the b. Airway obstruction
following physiologic functions? c. Hoarseness
a. Bowel function d. Dysphagia
b. Peripheral sensation
c. Bleeding tendencies 22. Karina a client with myasthenia gravis is to
d. Intake and out put receive immunosuppressive therapy. The nurse
understands that this therapy is effective
16. Lydia is scheduled for elective splenectomy. because it:
Before the clients goes to surgery, the nurse in a. Promotes the removal of antibodies that
charge final assessment would be: impair the transmission of impulses
a. signed consent b. Stimulates the production of
b. vital signs acetylcholine at the neuromuscular
c. name band junction.
d. empty bladder c. Decreases the production of
autoantibodies that attack the
17. What is the peak age range in acquiring acute acetylcholine receptors.
lymphocytic leukemia (ALL)? d. Inhibits the breakdown of acetylcholine
a. 4 to 12 years. at the neuromuscular junction.
b. 20 to 30 years
c. 40 to 50 years 23. A female client is receiving IV Mannitol. An
d. 60 60 70 years assessment specific to safe administration of the
said drug is:
18. Marie with acute lymphocytic leukemia suffers a. Vital signs q4h
from nausea and headache. These clinical b. Weighing daily
manifestations may indicate all of the following c. Urine output hourly
except d. Level of consciousness q4h
a. effects of radiation
b. chemotherapy side effects 24. Patricia a 20 year old college student with
c. meningeal irritation diabetes mellitus requests additional
d. gastric distension information about the advantages of using a pen
like insulin delivery devices. The nurse explains
19. A client has been diagnosed with Disseminated that the advantages of these devices over
Intravascular Coagulation (DIC). Which of the syringes include:
following is contraindicated with the client? a. Accurate dose delivery
a. Administering Heparin b. Shorter injection time
b. Administering Coumadin

17
17
c. Lower cost with reusable insulin
cartridges 30. A male client has undergone spinal surgery, the
d. Use of smaller gauge needle. nurse should:
a. Observe the client’s bowel movement
25. A male client’s left tibia is fractures in an and voiding patterns
automobile accident, and a cast is applied. To b. Log-roll the client to prone position
assess for damage to major blood vessels from c. Assess the client’s feet for sensation and
the fracture tibia, the nurse in charge should circulation
monitor the client for: d. Encourage client to drink plenty of fluids
a. Swelling of the left thigh
b. Increased skin temperature of the foot 31. Marina with acute renal failure moves into the
c. Prolonged reperfusion of the toes after diuretic phase after one week of therapy. During
blanching this phase the client must be assessed for signs
d. Increased blood pressure of developing:
a. Hypovolemia
26. After a long leg cast is removed, the male client b. renal failure
should: c. metabolic acidosis
a. Cleanse the leg by scrubbing with a brisk d. hyperkalemia
motion
b. Put leg through full range of motion 32. Nurse Judith obtains a specimen of clear nasal
twice daily drainage from a client with a head injury. Which
c. Report any discomfort or stiffness to the of the following tests differentiates mucus from
physician cerebrospinal fluid (CSF)?
d. Elevate the leg when sitting for long a. Protein
periods of time. b. Specific gravity
c. Glucose
27. While performing a physical assessment of a d. Microorganism
male client with gout of the great toe,
NurseVivian should assess for additional tophi 33. A 22 year old client suffered from his first tonic-
(urate deposits) on the: clonic seizure. Upon awakening the client asks
a. Buttocks the nurse, “What caused me to have a seizure?
b. Ears Which of the following would the nurse include
c. Face in the primary cause of tonic-clonic seizures in
d. Abdomen adults more the 20 years?
a. Electrolyte imbalance
28. Nurse Katrina would recognize that the b. Head trauma
demonstration of crutch walking with tripod gait c. Epilepsy
was understood when the client places weight d. Congenital defect
on the:
a. Palms of the hands and axillary regions 34. What is the priority nursing assessment in the
b. Palms of the hand first 24 hours after admission of the client with
c. Axillary regions thrombotic CVA?
d. Feet, which are set apart a. Pupil size and papillary response
b. cholesterol level
29. Mang Jose with rheumatoid arthritis states, “the c. Echocardiogram
only time I am without pain is when I lie in bed d. Bowel sounds
perfectly still”. During the convalescent stage,
the nurse in charge with Mang Jose should 35. Nurse Linda is preparing a client with multiple
encourage: sclerosis for discharge from the hospital to
a. Active joint flexion and extension home. Which of the following instruction is most
b. Continued immobility until pain subsides appropriate?
c. Range of motion exercises twice daily
d. Flexion exercises three times daily
a. “Practice using the mechanical aids that d. A client with U.T.I
you will need when future disabilities
arise”. 42. Among the following clients, which among them
b. “Follow good health habits to change is high risk for potential hazards from the
the course of the disease”. surgical experience?
c. “Keep active, use stress reduction a. 67-year-old client
strategies, and avoid fatigue. b. 49-year-old client
d. “You will need to accept the necessity c. 33-year-old client
for a quiet and inactive lifestyle”. d. 15-year-old client

36. The nurse is aware the early indicator of hypoxia 43. Nurse Jon assesses vital signs on a client
in the unconscious client is: undergone epidural anesthesia.
a. Cyanosis 44. Which of the following would the nurse assess
b. Increased respirations next?
c. Hypertension a. Headache
d. Restlessness b. Bladder distension
c. Dizziness
37. A client is experiencing spinal shock. Nurse d. Ability to move legs
Myrna should expect the function of the bladder
to be which of the following? 45. Nurse Katrina should anticipate that all of the
a. Normal following drugs may be used in the attempt to
b. Atonic control the symptoms of Meniere's disease
c. Spastic except:
d. Uncontrolled a. Antiemetics
b. Diuretics
38. Which of the following stage the carcinogen is c. Antihistamines
irreversible? d. Glucocorticoids
a. Progression stage
b. Initiation stage 46. Which of the following complications associated
c. Regression stage with tracheostomy tube?
d. Promotion stage a. Increased cardiac output
b. Acute respiratory distress syndrome
39. Among the following components thorough pain (ARDS)
assessment, which is the most significant? c. Increased blood pressure
a. Effect d. Damage to laryngeal nerves
b. Cause
c. Causing factors 47. Nurse Faith should recognize that fluid shift in a
d. Intensity client with burn injury results from increase in
the:
40. A 65 year old female is experiencing flare up of a. Total volume of circulating whole blood
pruritus. Which of the client’s action could b. Total volume of intravascular plasma
aggravate the cause of flare ups? c. Permeability of capillary walls
a. Sleeping in cool and humidified d. Permeability of kidney tubules
environment
b. Daily baths with fragrant soap 48. An 83-year-old woman has several ecchymotic
c. Using clothes made from 100% cotton areas on her right arm. The bruises are probably
d. Increasing fluid intake caused by:
a. increased capillary fragility and
41. Atropine sulfate (Atropine) is contraindicated in permeability
all but one of the following client? b. increased blood supply to the skin
a. A client with high blood c. self-inflicted injury
b. A client with bowel obstruction d. elder abuse
c. A client with glaucoma

17
17
49. Nurse Anna is aware that early adaptation of
client with renal carcinoma is:
a. Nausea and vomiting
b. flank pain
c. weight gain
d. intermittent hematuria

50. A male client with tuberculosis asks Nurse Brian


how long the chemotherapy must be continued.
Nurse Brian’s accurate reply would be:
a. 1 to 3 weeks
b. 6 to 12 months
c. 3 to 5 months
d. 3 years and more

51. A client has undergone laryngectomy. The


immediate nursing priority would be:
a. Keep trachea free of secretions
b. Monitor for signs of infection
c. Provide emotional support
d. Promote means of communication
ANSWERS AND RATIONALE – MEDICAL SURGICAL 14. A. Good source of vitamin B12 are dairy
NURSING products and meats.
15. C. Aplastic anemia decreases the bone marrow
1. C. Glucocorticoids (steroids) are used for their production of RBC’s, white blood cells, and
anti-inflammatory action, which decreases the platelets. The client is at risk for bruising and
development of edema. bleeding tendencies.
2. A. The blood must be stopped at once, and then 16. B. An elective procedure is scheduled in advance
normal saline should be infused to keep the line so that all preparations can be completed ahead
patent and maintain blood volume. of time. The vital signs are the final check that
3. B. These tests confirm the presence of HIV must be completed before the client leaves the
antibodies that occur in response to the room so that continuity of care and assessment
presence of the human immunodeficiency virus is provided for.
(HIV). 17. A. The peak incidence of Acute Lymphocytic
4. D. One cup of cottage cheese contains Leukemia (ALL) is 4 years of age. It is uncommon
approximately 225 calories, 27g of protein, 9g of after 15 years of age.
fat, 30mg cholesterol, and 6g of carbohydrate. 18. D. Acute Lymphocytic Leukemia (ALL) does not
Proteins of high biologic value (HBV) contain cause gastric distention. It does invade the
optimal levels of amino acids essential for life. central nervous system, and clients experience
5. A. Elevation of uremic waste products causes headaches and vomiting from meningeal
irritation of the nerves, resulting in flapping irritation.
hand tremors. 19. B. Disseminated Intravascular Coagulation (DIC)
6. B. This indicates that the bladder is distended has not been found to respond to oral
with urine, therefore palpable. anticoagulants such as Coumadin.
7. C. Elevation increases lymphatic drainage, 20. A. Urine output provides the most sensitive
reducing edema and pain. indication of the client’s response to therapy for
8. B. Detection of myoglobin is a diagnostic tool to hypovolemic shock. Urine output should be
determine whether myocardial damage has consistently greater than 30 to 35 mL/hr.
occurred. 21. C. Early warning signs of laryngeal cancer can
9. D. When mitral stenosis is present, the left vary depending on tumor location. Hoarseness
atrium has difficulty emptying its contents into lasting 2 weeks should be evaluated because it is
the left ventricle because there is no valve to one of the most common warning signs.
prevent back ward flow into the pulmonary vein, 22. C. Steroids decrease the body’s immune
the pulmonary circulation is under pressure. response thus decreasing the production of
10. A. Managing hypertension is the priority for the antibodies that attack the acetylcholine
client with hypertension. Clients with receptors at the neuromuscular junction
hypertension frequently do not experience pain, 23. C. The osmotic diuretic mannitol is
deficient volume, or impaired skin integrity. It is contraindicated in the presence of inadequate
the asymptomatic nature of hypertension that renal function or heart failure because it
makes it so difficult to treat. increases the intravascular volume that must be
11. C. Because of its widespread vasodilating effects, filtered and excreted by the kidney.
nitroglycerin often produces side effects such as 24. A. These devices are more accurate because
headache, hypotension and dizziness. they are easily to used and have improved
12. A. An increased in LDL cholesterol concentration adherence in insulin regimens by young people
has been documented at risk factor for the because the medication can be administered
development of atherosclerosis. LDL cholesterol discreetly.
is not broken down into the liver but is 25. C. Damage to blood vessels may decrease the
deposited into the wall of the blood vessels. circulatory perfusion of the toes, this would
13. D. There is a potential alteration in renal indicate the lack of blood supply to the
perfusion manifested by decreased urine output. extremity.
The altered renal perfusion may be related to 26. D. Elevation will help control the edema that
renal artery embolism, prolonged hypotension, usually occurs.
or prolonged aortic cross-clamping during the 27. B. Uric acid has a low solubility, it tends to
surgery. precipitate and form deposits at various sites

17
17
where blood flow is least active, including 43. B. The last area to return sensation is in the
cartilaginous tissue such as the ears. perineal area, and the nurse in charge should
28. B. The palms should bear the client’s weight monitor the client for distended bladder.
to avoid damage to the nerves in the axilla. 44. D. Glucocorticoids play no significant role in
29. A. Active exercises, alternating extension, disease treatment.
flexion, abduction, and adduction, mobilize 45. D. Tracheostomy tube has several potential
exudates in the joints relieves stiffness and pain. complications including bleeding, infection and
30. C. Alteration in sensation and circulation laryngeal nerve damage.
indicates damage to the spinal cord, if these 46. C. In burn, the capillaries and small vessels
occurs notify physician immediately. dilate, and cell damage cause the release of a
31. A. In the diuretic phase fluid retained during the histamine-like substance. The substance causes
oliguric phase is excreted and may reach 3 to 5 the capillary walls to become more permeable
liters daily, hypovolemia may occur and fluids and significant quantities of fluid are lost.
should be replaced. 47. A. Aging process involves increased capillary
32. C. The constituents of CSF are similar to those of fragility and permeability. Older adults have a
blood plasma. An examination for glucose decreased amount of subcutaneous fat and
content is done to determine whether a body cause an increased incidence of bruise like
fluid is a mucus or a CSF. A CSF normally contains lesions caused by collection of extravascular
glucose. blood in loosely structured dermis.
33. B. Trauma is one of the primary causes of brain 48. D. Intermittent pain is the classic sign of renal
damage and seizure activity in adults. Other carcinoma. It is primarily due to capillary erosion
common causes of seizure activity in adults by the cancerous growth.
include neoplasms, withdrawal from drugs and 49. B. Tubercle bacillus is a drug resistant organism
alcohol, and vascular disease. and takes a long time to be eradicated. Usually a
34. A. It is crucial to monitor the pupil size and combination of three drugs is used for minimum
papillary response to indicate changes around of 6 months and at least six months beyond
the cranial nerves. culture conversion.
35. C. The nurse most positive approach is to 50. A. Patent airway is the most priority; therefore
encourage the client with multiple sclerosis to removal of secretions is necessary
stay active, use stress reduction techniques and
avoid fatigue because it is important to support
the immune system while remaining active.
36. D. Restlessness is an early indicator of hypoxia.
The nurse should suspect hypoxia in unconscious
client who suddenly becomes restless.
37. B. In spinal shock, the bladder becomes
completely atonic and will continue to fill unless
the client is catheterized.
38. A. Progression stage is the change of tumor from
the preneoplastic state or low degree of
malignancy to a fast growing tumor that cannot
be reversed.
39. D. Intensity is the major indicative of severity of
pain and it is important for the evaluation of the
treatment.
40. B. The use of fragrant soap is very drying to skin
hence causing the pruritus.
41. C. Atropine sulfate is contraindicated with
glaucoma patients because it increases
intraocular pressure.
42. A. A 67 year old client is greater risk because the
older adult client is more likely to have a less-
effective immune system.
PSYCHIATRIC NURSING 7. A 20 year old client was diagnosed with
dependent personality disorder. Which behavior
1. Marco approached Nurse Trish asking for advice is not likely to be evidence of ineffective
on how to deal with his alcohol addiction. Nurse individual coping?
Trish should tell the client that the only effective a. Recurrent self-destructive behavior
treatment for alcoholism is: b. Avoiding relationship
a. Psychotherapy c. Showing interest in solitary activities
b. Alcoholics anonymous (A.A.) d. Inability to make choices and decision
c. Total abstinence without advise
d. Aversion Therapy
8. A male client is diagnosed with schizotypal
2. Nurse Hazel is caring for a male client who personality disorder. Which signs would this
experience false sensory perceptions with no client exhibit during social situation?
basis in reality. This perception is known as: a. Paranoid thoughts
a. Hallucinations b. Emotional affect
b. Delusions c. Independence need
c. Loose associations d. Aggressive behavior
d. Neologisms
9. Nurse Claire is caring for a client diagnosed with
3. Nurse Monet is caring for a female client who bulimia. The most appropriate initial goal for a
has suicidal tendency. When accompanying the client diagnosed with bulimia is?
client to the restroom, Nurse Monet should… a. Encourage to avoid foods
a. Give her privacy b. Identify anxiety causing situations
b. Allow her to urinate c. Eat only three meals a day
c. Open the window and allow her to get d. Avoid shopping plenty of groceries
some fresh air
d. Observe her 10. Nurse Tony was caring for a 41 year old female
client. Which behavior by the client indicates
4. Nurse Maureen is developing a plan of care for a adult cognitive development?
female client with anorexia nervosa. Which a. Generates new levels of awareness
action should the nurse include in the plan? b. Assumes responsibility for her actions
a. Provide privacy during meals c. Has maximum ability to solve problems
b. Set-up a strict eating plan for the client and learn new skills
c. Encourage client to exercise to reduce d. Her perception are based on reality
anxiety
d. Restrict visits with the family 11. A neuromuscular blocking agent is administered
to a client before ECT therapy. The Nurse should
5. A client is experiencing anxiety attack. The most carefully observe the client for?
appropriate nursing intervention should include? a. Respiratory difficulties
a. Turning on the television b. Nausea and vomiting
b. Leaving the client alone c. Dizziness
c. Staying with the client and speaking in d. Seizures
short sentences
d. Ask the client to play with other clients 12. A 75 year old client is admitted to the hospital
with the diagnosis of dementia of the
6. A female client is admitted with a diagnosis of Alzheimer’s type and depression. The symptom
delusions of GRANDEUR. This diagnosis reflects a that is unrelated to depression would be?
belief that one is: a. Apathetic response to the environment
a. Being Killed b. “I don’t know” answer to questions
b. Highly famous and important c. Shallow of labile effect
c. Responsible for evil world d. Neglect of personal hygiene
d. Connected to client unrelated to oneself

18
18
13. Nurse Trish is working in a mental health facility; following actions by the nurse would be most
the nurse priority nursing intervention for a important?
newly admitted client with bulimia nervosa a. Ask a family member to stay with the
would be to? client at home temporarily
a. Teach client to measure I & O b. Discuss the meaning of the client’s
b. Involve client in planning daily meal statement with her
c. Observe client during meals c. Request an immediate extension for the
d. Monitor client continuously client
d. Ignore the clients statement because it’s
14. Nurse Patricia is aware that the major health a sign of manipulation
complication associated with intractable
anorexia nervosa would be? 19. Joey a client with antisocial personality disorder
a. Cardiac dysrhythmias resulting to belches loudly. A staff member asks Joey, “Do
cardiac arrest you know why people find you repulsive?” this
b. Glucose intolerance resulting in statement most likely would elicit which of the
protracted hypoglycemia following client reaction?
c. Endocrine imbalance causing cold a. Depensiveness
amenorrhea b. Embarrassment
d. Decreased metabolism causing cold c. Shame
intolerance d. Remorsefulness

15. Nurse Anna can minimize agitation in a 20. Which of the following approaches would be
disturbed client by? most appropriate to use with a client suffering
a. Increasing stimulation from narcissistic personality disorder when
b. limiting unnecessary interaction discrepancies exist between what the client
c. increasing appropriate sensory states and what actually exist?
perception a. Rationalization
d. ensuring constant client and staff b. Supportive confrontation
contact c. Limit setting
d. Consistency
16. A 39 year old mother with obsessive-compulsive
disorder has become immobilized by her 21. Cely is experiencing alcohol withdrawal exhibits
elaborate hand washing and walking rituals. tremors, diaphoresis and hyperactivity. Blood
Nurse Trish recognizes that the basis of O.C. pressure is 190/87 mmhg and pulse is 92 bpm.
disorder is often: Which of the medications would the nurse
a. Problems with being too conscientious expect to administer?
b. Problems with anger and remorse a. Naloxone (Narcan)
c. Feelings of guilt and inadequacy b. Benzlropine (Cogentin)
d. Feeling of unworthiness and c. Lorazepam (Ativan)
hopelessness d. Haloperidol (Haldol)

17. Mario is complaining to other clients about not 22. Which of the following foods would the nurse
being allowed by staff to keep food in his room. Trish eliminate from the diet of a client in
Which of the following interventions would be alcohol withdrawal?
most appropriate? a. Milk
a. Allowing a snack to be kept in his room b. Orange Juice
b. Reprimanding the client c. Soda
c. Ignoring the clients behavior d. Regular Coffee
d. Setting limits on the behavior
23. Which of the following would Nurse Hazel
18. Conney with borderline personality disorder who expect to assess for a client who is exhibiting
is to be discharge soon threatens to “do late signs of heroin withdrawal?
something” to herself if discharged. Which of the a. Yawning & diaphoresis
b. Restlessness & Irritability my best friend. The nurse recognizes that the
c. Constipation & steatorrhea client is using the defense mechanism known as?
d. Vomiting and Diarrhea a. Displacement
b. Projection
24. To establish open and trusting relationship with c. Sublimation
a female client who has been hospitalized with d. Denial
severe anxiety, the nurse in charge should?
a. Encourage the staff to have frequent 30. When working with a male client suffering
interaction with the client phobia about black cats, Nurse Trish should
b. Share an activity with the client anticipate that a problem for this client would
c. Give client feedback about behavior be?
d. Respect client’s need for personal space a. Anxiety when discussing phobia
b. Anger toward the feared object
25. Nurse Monette recognizes that the focus of c. Denying that the phobia exist
environmental (MILIEU) therapy is to: d. Distortion of reality when completing
a. Manipulate the environment to bring daily routines
about positive changes in behavior
b. Allow the client’s freedom to determine 31. Linda is pacing the floor and appears extremely
whether or not they will be involved in anxious. The duty nurse approaches in an
activities attempt to alleviate Linda’s anxiety. The most
c. Role play life events to meet individual therapeutic question by the nurse would be?
needs a. Would you like to watch TV?
d. Use natural remedies rather than drugs b. Would you like me to talk with you?
to control behavior c. Are you feeling upset now?
d. Ignore the client
26. Nurse Trish would expect a child with a diagnosis
of reactive attachment disorder to: 32. Nurse Penny is aware that the symptoms that
a. Have more positive relation with the distinguish post-traumatic stress disorder from
father than the mother other anxiety disorder would be:
b. Cling to mother & cry on separation a. Avoidance of situation & certain
c. Be able to develop only superficial activities that resemble the stress
relation with the others b. Depression and a blunted affect when
d. Have been physically abuse discussing the traumatic situation
c. Lack of interest in family & others
27. When teaching parents about childhood d. Re-experiencing the trauma in dreams or
depression Nurse Trina should say? flashback
a. It may appear acting out behavior
b. Does not respond to conventional 33. Nurse Benjie is communicating with a male client
treatment with substance-induced persisting dementia; the
c. Is short in duration & resolves easily client cannot remember facts and fills in the
d. Looks almost identical to adult gaps with imaginary information. Nurse Benjie is
depression aware that this is typical of?
a. Flight of ideas
28. Nurse Perry is aware that language development b. Associative looseness
in autistic child resembles: c. Confabulation
a. Scanning speech d. Concretism
b. Speech lag
c. Shuttering 34. Nurse Joey is aware that the signs & symptoms
d. Echolalia that would be most specific for diagnosis
anorexia are?
29. A 60 year old female client who lives alone tells a. Excessive weight loss, amenorrhea &
the nurse at the community health center “I abdominal distension
really don’t need anyone to talk to”. The TV is b. Slow pulse, 10% weight loss & alopecia

18
18
c. Compulsive behavior, excessive fears & detailed assessment, a diagnosis of
nausea schizophrenia is made. It is unlikely that the
d. Excessive activity, memory lapses & an client will demonstrate:
increased pulse a. Low self esteem
b. Concrete thinking
35. A characteristic that would suggest to Nurse c. Effective self-boundaries
Anne that an adolescent may have bulimia d. Weak ego
would be:
a. Frequent regurgitation & re-swallowing 41. A 23 year old client has been admitted with a
of food diagnosis of schizophrenia says to the nurse
b. Previous history of gastritis “Yes, its march, March is little woman”. That’s
c. Badly stained teeth literal you know”. These statement illustrate:
d. Positive body image a. Neologisms
b. Echolalia
36. Nurse Monette is aware that extremely c. Flight of ideas
depressed clients seem to do best in settings d. Loosening of association
where they have:
a. Multiple stimuli 42. A long term goal for a paranoid male client who
b. Routine Activities has unjustifiably accused his wife of having many
c. Minimal decision making extramarital affairs would be to help the client
d. Varied Activities develop:
a. Insight into his behavior
37. To further assess a client’s suicidal potential. b. Better self-control
Nurse Katrina should be especially alert to the c. Feeling of self-worth
client expression of: d. Faith in his wife
a. Frustration & fear of death
b. Anger & resentment 43. A male client who is experiencing disordered
c. Anxiety & loneliness thinking about food being poisoned is admitted
d. Helplessness & hopelessness to the mental health unit. The nurse uses which
communication technique to encourage the
38. A nursing care plan for a male client with bipolar client to eat dinner?
I disorder should include: a. Focusing on self-disclosure of own food
a. Providing a structured environment preference
b. Designing activities that will require the b. Using open ended question and silence
client to maintain contact with reality c. Offering opinion about the need to eat
c. Engaging the client in conversing about d. Verbalizing reasons that the client may
current affairs not choose to eat
d. Touching the client provide assurance
44. Nurse Nina is assigned to care for a client
39. When planning care for a female client using diagnosed with Catatonic Stupor. When Nurse
ritualistic behavior, Nurse Gina must recognize Nina enters the client’s room, the client is found
that the ritual: lying on the bed with a body pulled into a fetal
a. Helps the client focus on the inability to position. Nurse Nina should?
deal with reality a. Ask the client direct questions to
b. Helps the client control the anxiety encourage talking
c. Is under the client’s conscious control b. Rake the client into the dayroom to be
d. Is used by the client primarily for with other clients
secondary gains c. Sit beside the client in silence and
occasionally ask open-ended question
40. A 32 year old male graduate student, who has d. Leave the client alone and continue with
become increasingly withdrawn and neglectful providing care to the other clients
of his work and personal hygiene, is brought to
the psychiatric hospital by his parents. After
45. Nurse Tina is caring for a client with delirium and 49. Nurse Tina is caring for a client with depression
states that “look at the spiders on the wall”. who has not responded to antidepressant
What should the nurse respond to the client? medication. The nurse anticipates that what
a. “You’re having hallucination, there are treatment procedure may be prescribed.
no spiders in this room at all” a. Neuroleptic medication
b. “I can see the spiders on the wall, but b. Short term seclusion
they are not going to hurt you” c. Psychosurgery
c. “Would you like me to kill the spiders” d. Electroconvulsive therapy
d. “I know you are frightened, but I do not
see spiders on the wall” 50. Mario is admitted to the emergency room with
drug-included anxiety related to over ingestion
46. Nurse Jonel is providing information to a of prescribed antipsychotic medication. The
community group about violence in the family. most important piece of information the nurse
Which statement by a group member would in charge should obtain initially is the:
indicate a need to provide additional a. Length of time on the med.
information? b. Name of the ingested medication & the
a. “Abuse occurs more in low-income amount ingested
families” c. Reason for the suicide attempt
b. “Abuser Are often jealous or self- d. Name of the nearest relative & their
centered” phone number
c. “Abuser use fear and intimidation”
d. “Abuser usually have poor self-esteem”

47. During electroconvulsive therapy (ECT) the client


receives oxygen by mask via positive pressure
ventilation. The nurse assisting with this
procedure knows that positive pressure
ventilation is necessary because?
a. Anesthesia is administered during the
procedure
b. Decrease oxygen to the brain increases
confusion and disorientation
c. Grand mal seizure activity depresses
respirations
d. Muscle relaxations given to prevent
injury during seizure activity depress
respirations.

48. When planning the discharge of a client with


chronic anxiety, Nurse Chris evaluates
achievement of the discharge maintenance
goals. Which goal would be most appropriately
having been included in the plan of care
requiring evaluation?
a. The client eliminates all anxiety from
daily situations
b. The client ignores feelings of anxiety
c. The client identifies anxiety producing
situations
d. The client maintains contact with a crisis
counselor

18
18
ANSWERS AND RATIONALE – PSYCHIATRIC Rationale: With depression, there is little or no
NURSING emotional involvement therefore little alteration
in affect.
1. Answer: C 13. Answer: D
Rationale: Total abstinence is the only effective Rationale: These clients often hide food or force
treatment for alcoholism vomiting; therefore they must be carefully
2. Answer: A monitored.
Rationale: Hallucinations are visual, auditory, 14. Answer: A
gustatory, tactile or olfactory perceptions that Rationale: These clients have severely depleted
have no basis in reality. levels of sodium and potassium because of their
3. Answer: D starvation diet and energy expenditure, these
Rationale: The Nurse has a responsibility to electrolytes are necessary for cardiac
observe continuously the acutely suicidal client. functioning.
The Nurse should watch for clues, such as 15. Answer: B
communicating suicidal thoughts, and messages; Rationale: Limiting unnecessary interaction will
hoarding medications and talking about death. decrease stimulation and agitation.
4. Answer: B 16. Answer: C
Rationale: Establishing a consistent eating plan Rationale: Ritualistic behavior seen in this
and monitoring client’s weight are important to disorder is aimed at controlling guilt and
this disorder. inadequacy by maintaining an absolute set
5. Answer: C pattern of behavior.
Rationale: Appropriate nursing interventions for 17. Answer: D
an anxiety attack include using short sentences, Rationale: The nurse needs to set limits in the
staying with the client, decreasing stimuli, client’s manipulative behavior to help the client
remaining calm and medicating as needed. control dysfunctional behavior. A consistent
6. Answer:B approach by the staff is necessary to decrease
Rationale: Delusion of grandeur is a false belief manipulation.
that one is highly famous and important. 18. Answer: B
7. Answer: D Rationale: Any suicidal statement must be
Rationale: Individual with dependent personality assessed by the nurse. The nurse should discuss
disorder typically shows indecisiveness the client’s statement with her to determine its
submissiveness and clinging behavior so that meaning in terms of suicide.
others will make decisions with them. 19. Answer: A
8. Answer: A Rationale: When the staff member ask the client
Rationale: Clients with schizotypal personality if he wonders why others find him repulsive, the
disorder experience excessive social anxiety that client is likely to feel defensive because the
can lead to paranoid thoughts question is belittling. The natural tendency is to
9. Answer: B counterattack the threat to self-image.
Rationale: Bulimia disorder generally is a 20. Answer: B
maladaptive coping response to stress and Rationale: The nurse would specifically use
underlying issues. The client should identify supportive confrontation with the client to point
anxiety causing situation that stimulate the out discrepancies between what the client states
bulimic behavior and then learn new ways of and what actually exists to increase
coping with the anxiety. responsibility for self.
10. Answer: A 21. Answer: C
Rationale: An adult age 31 to 45 generates new Rationale: The nurse would most likely
level of awareness. administer benzodiazepine, such as lorazepan
11. Answer: A (ativan) to the client who is experiencing
Rationale: Neuromuscular Blocker, such as symptom: The client’s experiences symptoms of
SUCCINYLCHOLINE (Anectine) produces withdrawal because of the rebound
respiratory depression because it inhibits phenomenon when the sedation of the CNS
contractions of respiratory muscles. from alcohol begins to decrease.
12. Answer: C 22. Answer: D
Rationale: Regular coffee contains caffeine Rationale: These are the major signs of anorexia
which acts as psychomotor stimulants and leads nervosa. Weight loss is excessive (15% of
to feelings of anxiety and agitation. Serving expected weight)
coffee top the client may add to tremors or 35. Answer: C
wakefulness. Rationale: Dental enamel erosion occurs from
23. Answer: D repeated self-induced vomiting.
Rationale: Vomiting and diarrhea are usually the 36. Answer: B
late signs of heroin withdrawal, along with Rationale: Depression usually is both emotional
muscle spasm, fever, nausea, repetitive, & physical. A simple daily routine is the best,
abdominal cramps and backache. least stressful and least anxiety producing.
24. Answer: D 37. Answer: D
Rationale: Moving to a client’s personal space Rationale: The expression of these feeling may
increases the feeling of threat, which increases indicate that this client is unable to continue the
anxiety. struggle of life.
25. Answer: A 38. Answer: A
Rationale: Environmental (MILIEU) therapy aims Rationale: Structure tends to decrease agitation
at having everything in the client’s surrounding and anxiety and to increase the client’s feeling of
area toward helping the client. security.
26. Answer: C 39. Answer: B
Rationale: Children who have experienced Rationale: The rituals used by a client with
attachment difficulties with primary caregiver obsessive compulsive disorder help control the
are not able to trust others and therefore relate anxiety level by maintaining a set pattern of
superficially action.
27. Answer: A 40. Answer: C
Rationale: Children have difficulty verbally Rationale: A person with this disorder would not
expressing their feelings, acting out behavior, have adequate self-boundaries
such as temper tantrums, may indicate 41. Answer: D
underlying depression. Rationale: Loose associations are thoughts that
28. Answer: D are presented without the logical connections
Rationale: The autistic child repeats sounds or usually necessary for the listening to interpret
words spoken by others. the message.
29. Answer: D 42. Answer: C
Rationale: The client statement is an example of Rationale: Helping the client to develop feeling
the use of denial, a defense that blocks problem of self-worth would reduce the client’s need to
by unconscious refusing to admit they exist use pathologic defenses.
30. Answer: A 43. Answer: B
Rationale: Discussion of the feared object Rationale: Open ended questions and silence are
triggers an emotional response to the object. strategies used to encourage clients to discuss
31. Answer: B their problem in descriptive manner.
Rationale: The nurse presence may provide the 44. Answer: C
client with support & feeling of control. Rationale: Clients who are withdrawn may be
32. Answer: D immobile and mute, and require consistent,
Rationale: Experiencing the actual trauma in repeated interventions. Communication with
dreams or flashback is the major symptom that withdrawn clients requires much patience from
distinguishes post-traumatic stress disorder from the nurse. The nurse facilitates communication
other anxiety disorder. with the client by sitting in silence, asking open-
33. Answer: C ended question and pausing to provide
Rationale: Confabulation or the filling in of opportunities for the client to respond.
memory gaps with imaginary facts is a defense 45. Answer: D
mechanism used by people experiencing Rationale: When hallucination is present, the
memory deficits. nurse should reinforce reality with the client.
34. Answer: A 46. Answer: A

18
18
Rationale: Personal characteristics of abuser
include low self-esteem, immaturity,
dependence, insecurity and jealousy.
47. Answer: D
Rationale: A short acting skeletal muscle relaxant
such as succinylcholine (Anectine) is
administered during this procedure to prevent
injuries during seizure.
48. Answer: C
Rationale: Recognizing situations that produce
anxiety allows the client to prepare to cope with
anxiety or avoid specific stimulus.
49. Answer: D
Rationale: Electroconvulsive therapy is an
effective treatment for depression that has not
responded to medication
50. Answer: B
Rationale: In an emergency, lives saving facts are
obtained first. The name and the amount of
medication ingested are of outmost important in
treating this potentially life threatening
situation.
FOUNDATION OF PROFESSIONAL NURSING a. There are many alternative sites for subcutaneous
PRACTICE injection
b. Absorption time of the medicine is slower
Situation 1 - Mr. Ibarra is assigned to the triage area and c. There are less pain receptors in this area
while on duty, he assesses the condition of Mrs. Simon d. The medication can be injected while the client is in
who came in with asthma. She has difficulty breathing any position
and her respiratory rate is 40 per minute. Mr. Ibarra is
asked to inject the client epinephrine 0.3mg Situation 2 - The use of massage and meditation to help
subcutaneously decrease stress and pain have been strongly
recommended based on documented testimonials.
1. The indication for epinephrine injection for Mrs
Simon is to: 6. Martha wants to do a study on, this topic. "Effects of
massage and meditation on stress and pain." The type
a. Reduce anaphylaxis of research that best suits this topic is:
b. Relieve hypersensitivity to allergen
c. Relieve respirator distress due to bronchial spasm a. applied research
d. Restore client’s cardiac rhythm b. qualitative research
c. basic research
2. When preparing the epinephrine injection from an d. quantitative research
ampule, the nurse initially:
7. The type of research design that does not
a. Taps the ampule at the top to allow fluid to flow manipulate independent variable is:
to the base of the ampule
b. Checks expiration date of the medication ampule a. experimental design
c. Removes needle cap of syringe and pulls plunger to b. quasi-experimental design
expel air c. non-experimental design
d. Breaks the neck of the ampule with a gauze wrapped d. quantitative design
around it
8. This research topic has the potential to contribute
3. Mrs. Simon is obese. When administering a to nursing because it seeks to:
subcutaneous injection to an obese patient, it is best
for the nurse to: a. include new modalities of care
b. resolve a clinical problem
a Inject needle at a 15 degree angle' over the stretched c. clarify an ambiguous modality of care
skin of the client d. enhance client care
b. Pinch skin at the Injection site and use airlock
technique 9. Martha does review of related literature for
c. Pull skin of patient down to administer the drug in a Z the purpose of:
track
d. Spread skin or pinch at the injection site and inject a. determine statistical treatment of data research
needle at a 45-90 degree angle b. gathering data about what is already known or
unknown
4. When preparing for a subcutaneous injection, the c. to identify if problem can be replicated
proper size of syringe and needle would be: d. answering the research question

a. Syringe 3-5ml and needle gauge 21 to 23 10. Client’s rights should be protected when doing
b. Tuberculin syringe 1 mi with needle gauge 26 or 27 research using human subjects. Martha identifies these
c. Syringe 2ml and needle gauge 22 rights as follows EXCEPT:
d. Syringe 1-3ml and needle gauge 25 to 27
a. right of self-determination
5. The rationale for giving medications through b. right to compensation
the subcutaneous route is; c. right of privacy
d. right not to be harmed

18
18

c. Client has no signs of infection


Situation 3 - Richard has a nursing diagnosis of d. Time of fast food and fluid intake of the client
ineffective airway clearance related to excessive
secretions and is at risk for infection because of retained 15. The purpose of chest percussion and vibration is to
secretions. Part of Nurse Mario's nursing care plan is to loosen secretions in the lungs. The difference between
loosen and remove excessive secretions in the airway, the procedure is;

11. Mario listens to Richard's bilateral sounds and finds a. Percussion uses only one hand white vibration uses
that congestion is in the upper lobes of the lungs. The both hands
appropriate position to drain the anterior and posterior b. Percussion delivers cushioned blows to the chest with
apical segments of the lungs when Mario does cupped palms while gently shakes secretion loose on the
percussion would be: exhalation cycle
c. In both percussion and vibration the hands are on top
a. Client lying on his back then flat on his abdomen on of each other and hand action is in tune with client's
Trendelenburg position breath rhythm
b. Client seated upright in bed or on a chair then leaning d. Percussion slaps the chest to loosen secretions while
forward in sitting position then flat on his back and on vibration shakes the secretions along with the inhalation
his abdomen of air
c. Client lying flat on his back and then flat on his
abdomen Situation 4 - A 61 year old man, Mr. Regalado, is
d. Client lying on his right then left side on admitted to the private ward for observation; after
Trendelenburg position complaints of severe chest pain. You are assigned to take
care of the client.
12. When documenting outcome of Richard's
treatment Mario should include the following in his 16. When doing an initial assessment, the best way
recording EXCEPT: for you to identify the client’s priority problem is to:

a. Color, amount and consistent of sputum a. Interview the client for chief complaints and other
b. Character of breath sounds and respirator/rate before symptoms
and after procedure b. Talk to the relatives to gather data about history of
c. Amount of fluid intake of client before and after the illness
procedure c. Do auscultation to check for chest congestion
d. Significant changes in vital signs d. Do a physical examination white asking the client
relevant questions
13. When assessing Richard for chest percussion or
chest vibration and postural drainage Mario would 17. Upon establishing Mr. Regalado's nursing needs,
focus on the following EXCEPT: the next nursing approach would be to:

a. Amount of food and fluid taken during the last meal a. introduce the client to the ward staff to put the client
before treatment and family at ease
b. Respiratory rate, breath sounds and location of b. Give client and relatives a brief tour of the physical set
congestion up the unit
c. Teaching the client's relatives to perform 'the c. Take his vital signs for a baseline assessment
procedure d. Establish priority needs and implement appropriate
d. Doctor's order regarding position restriction and interventions
client's tolerance for lying flat
18. Mr. Regalado says he has "trouble going to
14. Mario prepares Richard for postural drainage sleep". In order to plan your nursing intervention you
and percussion. Which of the flowing is a special will.
consideration when doing the procedure?
a. Observe his sleeping patterns in the next few days
a. Respiratory rate of 16 to 20 per minute b. Ask him what he means by this statement
b. Client can tolerate sitting and lying position c. Check his physical environment to decrease noise level
d. Take his blood pressure before sleeping and upon
waking up 23. The nurse visits Nancy and prods her to eat her
food. Nancy replies "what's the use? My time is running
19. Mr. Regalado's lower extremities are swollen and out. The nurse's best response would be:
shiny. He has pitting pedal edema. When taking care of
Mr. Regalado, which of the following intervention a. "The doctor ordered full diet for you so that you will
would be the most appropriate immediate nursing be strong for surgery."
approach. b. "I understand how you fee! but you have 1o try for
your children's sake."
a. Moisturize lower extremities to prevent skin irritation c. "Have you told your, doctor how you feel? Are you
b. Measure fluid intake and output to decrease edema changing your mind) about surgery?"
c. Elevate lower extremities for postural drainage d. "You sound like you are giving up."
d. Provide the client a list of food low in sodium
24. The nurse feels sad about Nancy's illness and tells
20. Mr. Regalado will be discharged from your unit her head nurse during the end of shift endorsement
within the hour. Nursing actions when preparing a that "it's unfair for Nancy to have cancer when she is
client for discharge include all EXCEPT: still so young and with two kinds. The best response
of the head nurse would be:
a. Making a final physical assessment before client
leaves the hospital a. Advise the nurse to "be strong and learn to control her
b. Giving instructions about his medication regimen feelings"
c. Walking the client to the hospital exit to ensure his b. Assign the nurse to another client to avoid sympathy
safety for the client
d. Proper recording of pertinent data c. Reassure the nurse that the client has hope if she goes
through all statements prescribed for her
Situation 5 - Nancy, mother of 2 young kids. 36 years old, c. Ask the other nurses what they feel about the patient
had a mammogram and was told that she has breast to find out if they share the same feelings
cysts and that she may need surgery. This causes her
anxiety as shown by increase in her pulse and respiratory 25. Realizing that she feels angry about Nancy's
rate, sweating and feelings of tension. condition, the nurse Seams that being self-aware is a
conscious process that she should do in any situation
21. Considering her level of anxiety, the nurse can like this because:
best assist Nancy by:
a. This is a necessary part of the nurse -client
a. Giving her activities to divert her attention relationship process
b. Giving detailed explanations about the treatments she b. The nurse is a role model for the client and should be
will undergo strong
c. Preparing her and her family in case surgery is C. How the nurse thinks and feels affect her actions
not successful towards her client and her work
d. Giving her clear but brief information at the level of d. The nurse has to be therapeutic at all times and
her understanding should not be affected

22. Nancy blames God for her situation. She is easily Situation 6 – Mrs. Seva, 32 years old, asks you about
provoked to tears and wants to be left alone, refusing possible problems regarding her elimination now that
to eat or talk to her family. A religious person before, she is in the menopausal stage.
she now refuses to pray or go to church stating that
God has abandoned her. The nurse understands that 26. Instruction on health promotion regarding urinary
Nancy is grieving for her self and is in the stage of: elimination is important. Which would you include?

a. bargaining a. Hold urine, as long as she can before emptying the


b. denial bladder to strengthen her sphincters muscles
c. anger b. If burning sensation is experienced while voiding,
d. acceptance drink pineapple-juice
c. After urination, wipe from anal area up towards the

19
19
pubis
d. Jell client to empty the bladder at each voiding a. Carol with a tumor in the brain
b. Theresa with anemia
27. Mrs. Seva also tells the nurse that she is often c. Sonny Boy with a fracture in the femur
constipated. Because she is aging, what physical d. Brigette with diarrhea
changes predispose her to constipation?
32. You noted from the lab exams in the chart of
a. inhibition of the parasympathetic reflex Mr. Santos that he has reduced oxygen in the
b. weakness of sphincter muscles of the anus blood. This condition is called:
c. loss of tone of the smooth muscles of the color
d. decreased ability to absorb fluids in the lower a. Cyanosis
intestines b. Hypoxia
c. Hypoxemia
28. The nurse understands that one of these d. Anemia
factors contributes to constipation:
33. You will nasopharyngeal suctioning Mr. Abad.
a. excessive exercise Your guide for the length of insertion of the tubing for
b. high fiber diet an adult would be:
c. no regular tine for defecation daily
d. prolonged use of laxatives a. tip of the nose to the base of the .neck
b. the distance from the tip of the nose to the middle of
29. Mrs. Seva talks about rear of being incontinent due the cheek
to a prior experience of dribbling urine when laughing c. the distance from the tip of the nose to the tip of
or sneezing and when she has a full bladder. Your most the ear lobe
appropriate .instruction would be to: d. eight to ten inches

a. tell client to drink less fluids to avoid accidents 34. While doing nasopharyngeal suctioning on .Mr.
b. instruct client to start wearing thin adult diapers Abad, the nurse can avoid trauma to the area by:
c. ask the client to bring change of underwear "just in
case" a. Apply suction for at least 20-30 seconds each time to
d. teach client pelvic exercise to strengthen perineal ensure that all secretions are removed
muscles b. Using gloves to prevent introduction of pathogens to
the respiratory system
30. Mrs. Seva asked for instructions for skin care for c. Applying no suction while inserting the catheter
her mother who has urinary incontinence and is almost d. Rotating catheter as it is inserted with gentle suction
always in bed. Your instruction would focus on
prevention of skin irritation and breakdown by 35. Myrna has difficulty breathing when on her back
and must sit upright in bed to breath, effectively and
a. Using thick diapers to absorb urine well comfortably. The nurse documents this condition as:
b. Drying the skin with baby powder to prevent or mask
the smell of ammonia a. Apnea
c. Thorough washing, rising and during of skin area that b. Orthopnea
get wet with urine c. Dyspnea
d. Making sure that linen are smooth and dry at all times d. Tachypnea

Situation 7 - Using Maslow's need theory, Airway, Situation 8 - You are assigned to screen for
Breathing and Circulation are the physiological needs hypertension: Your task is to take blood pressure
vital to life. The nurse's knowledge and ability to identify readings and you are informed about avoiding the
and immediately intervene to meet these needs is common mistakes in BP taking that lead to 'false or
important to save lives. inaccurate blood pressure readings.

31. Which of these clients has a problem with the 36. When taking blood pressure reading the cuff should
transport of oxygen from the lungs to the tissues: be:
a. deflated fully then immediately start second reading a. Assess damage to property
for same client b. Assist in the police investigation since she is a witness
b deflated quickly after inflating up to 180 mmHg c. Report the incident immediately to the local police
c. large enough to wrap around upper arm of the adult authorities
client 1 cm above brachial artery d. Assess the extent of injuries incurred by the victims, of
d. inflated to 30 mmHg above the estimated systolic BP the accident
based on palpation of radial or bronchial artery
42. Priority attention should be given to which of these
37. Chronic Obstructive Pulmonary Disease (COPD) in clients?
one of the leading causes of death worldwide and is
a preventable disease. The primary cause of COPD is: a. Linda who shows severe anxiety due to trauma of
the accident
a. tobacco hack b. Ryan who has chest injury, is pate and with difficulty
b. bronchitis of breathing
c. asthma c. Noel who has lacerations on the arms with mild-
d. cigarette smoking bleeding
c. Andy whose left ankle swelled and has some abrasions
38. In your health education class for clients
with diabetes you teach, them the areas, for 43. In the emergency room, Nurse Rivera is assigned to
control . Diabetes which include all EXCEPT: attend to the client with .lacerations on the arms, while
assessing the extent of the wound the nurse observes
a. regular physical activity that the wound is now starting to bleed profusely. The
b. thorough knowledge of foot care most immediate nursing action would be to:
c. prevention nutrition
d. proper nutrition a. Apply antiseptic to prevent infection
b. Clean the wound vigorously of contaminants
39. You teach your clients the difference between, Type c. Control and. reduce bleeding of the wound
I (IDDM) and Type II (NDDM) Diabetes. Which of the d. Bandage the wound and elevate the arm
following is true?
44. The nurse applies pressure dressing on the bleeding
a. both types diabetes mellitus clients are all prone site. This intervention is done to:
to developing ketosis
b. Type II (NIDDM) is more common and is also a. Reduce the need to change dressing frequently
preventable compared to Type I (IDDM) diabetes which b. Allow the pus to surface faster
is genetic in etiology c. Protect the wound from micro organisms in the air
c. Type I (IDDM) is characterized by fasting d. Promote hemostasis
hyperglycemia
d. Type II (IDDM) is characterized by abnormal immune 45. After the treatment, the client is sent home and
response asked to come back for follow-up care. Your
responsibilities when the client is to be discharged
40. Lifestyle-related diseases in general share areas include the following EXCEPT:
common risk factors. These are the following
except a. Encouraging the client to go to the, outpatient clinic
a. physical activity for follow up care
b. smoking b. Accurate recording, of treatment done and
c. genetics instructions given to client
d. nutrition c. Instructing the client to see you after discharge for
further assistance
Situation 9 - Nurse Rivera witnesses a vehicular accident d. Providing instructions regarding wound care
near the hospital where she works. She decides to get
involved and help the victims of the accident. Situation 10 - While working in the clinic, a new client,
Geline, 35 years old, arrives for her doctor's
41. Her priority nursing action would be to:
19
19
appointment. As the clinic nurse, you are to assist the
client fiil up forms, gather data and make an assessment. a. Caffeine products affect the central nervous system
and may cause the mother to have a "nervous
46. The nurse purpose of your initial nursing breakdown"
interview is to: b. Malnutrition and its possible effects on growth and
development problems in the unborn fetus
a. Record pertinent information in the client chart for c. Caffeine causes a stimulant effect on both the mother
health team to read and the baby
b Assist the client find solutions to her health concerns d. Studies show conclusively that caffeine causes mental
c. Understand her lifestyle, health needs and possible retardation
problems to develop a plan of care
d. Make nursing diagnoses for identified health problems 50. Your health education plan for Geline stresses
proper diet for a pregnant woman and the prevention
47. While interviewing Geline, she starts to moan and of non-communicable diseases that are influenced by
doubles up in pain, She tells you that this pain occurs her lifestyle these include of the following EXCEPT:
about an hour after taking black coffee without
breakfast for a few weeks now. You will record this a. Cardiovascular diseases
as follows: b. Cancer
c. Diabetes Mellitus
a. Claims to have abdominal pains after intake of coffee d. Osteoporosis
unrelieved by analgesics
b. After drinking coffee, the client experienced severe Situation 11 - Management of nurse practitioners is
abdominal pain done by qualified nursing leaders who have had clinical
c. Client complained of intermittent abdominal pain an experience and management experience.
hour after drinking coffee
d. Client reported abdominal pain an hour after drinking 51. An example of a management function of a nurse is:
black coffee for three weeks now
a. Teaching patient do breathing and coughing exercises
48. Geline tells you that she drinks black coffee b. Preparing for a surprise party for a client
frequently within the day to "have energy and be wide c. Performing nursing procedures for clients
awake" and she eats nothing for breakfast and eats d. Directing and evaluating the staff nurses
strictly vegetable salads for lunch and dinner to lose
weight. She has lost weight during the past two 52. Your head nurse in the unit believes that the staff
weeks, in planning a healthy balanced diet with nurses are not capable of decision making so she
Geline, you will: makes the decisions for everyone without consulting
anybody. This type of leadership is:
a. Start her off with a cleansing diet to free her body of
toxins then change to a vegetarian, diet and drink plenty a. Laissez faire leadership
of fluids b. Democratic leadership
b. Plan a high protein, diet; low carbohydrate diet for her c. Autocratic leadership
considering her favorite food d. Managerial leadership
c. Instruct her to attend classes in nutrition to find food
rich in complex carbohydrates to maintain daily high 53. When the head nurse in your ward plots and
energy level approves your work schedules and directs your work,
d. Discuss with her the importance of eating a variety of she is demonstrating:
food from the major food groups with plenty of fluids
a. Responsibility
49. Geline tells you that she drinks 4-5 cups of black b. Delegation
coffee and diet cola drinks. She also smokes up to a c. Accountability
pack of cigarettes daily. She confesses that she is in her d. Authority
2nd month of pregnancy but she does not want to
become fat that is why she limits her food intake. You 54. The following tasks can be safely delegated' by a
warn or caution her about which of the following? nurse to a non-nurse health worker EXCEPT:
Mr. Dizon smokes and drinks coffee. When taking the
a. Transfer a client from bed to chair blood pressure of a client who recently smoked or
b. Change IV infusions drank coffee, how long should be the nurse wait before
c. Irrigation of a nasogastric tube taking the client’s blood pressure for accurate reading?
d. Take vital signs
a. 15 minutes
55. You made a mistake in giving the medicine to the b. 30 minutes
wrong client You notify the client’s doctor and write c. 1 hour
an incident report. You are demonstrating: d. 5 minutes

a. Responsibility 60. While the client has the pulse oximeter on his
b. Accountability fingertip, you notice that the sunlight is shining on .the
c. Authority area where the oximeter is. Your action will be to:
d. Autocracy
a. Set and turn on the alarm of the oximeter
Situation 12 – Mr. Dizon, 84 years old, is brought to the b. Do nothing since there is no identified problem
.Emergency Room for complaint of hypertension flushed c. Cover the fingertip sensor with a towel or bedsheet
face, severe headache, and nausea. You are doing the d. Change the location of the sensor every four hours
initial assessment of vital signs.
Situation 13 - The nurse's understanding of ethico-legal
56. You are to measure the client’s initial blood responsibilities will guide his/her nursing practice.
pressure reading by doing all of the following
EXCEPT: 61. The principles that .govern right and proper
conducts of a person regarding life, biology and
a. Take the blood pressure reading on both arms for the health professions is referred to as:
comparison
b. Listen to and identify the phases of Korotkoff’s sounds a. Morality
c. Pump the cuff up to around 50 mmHg above the point b. Religion
where the pulse is obliterated c. Values
d. Observe procedures for infection control d. Bioethics

57. A pulse oximeter is attached to Mr. Dizon’s finger 62. The purpose of having nurses’ code of ethics is:
to:
a. Delineate the scope and areas of nursing practice
a. Determine if the client’s hemoglobin level is low and if b. Identify nursing action recommended for specific
he needs blood transfusion healthcare situations
b. Check level of client’s tissue perfusion c. To help the public understand professional
c. Measure the efficacy of the client’s anti hypertensive conduct, expected of nurses
medications d. To define the roles and functions of the health care
d. Detect oxygen saturation of arterial blood before giver, nurses, clients
symptoms of hypoxemia develops
63. The most important nursing responsibility where
58. After a few hours in the Emergency Room, Mr. ethical situations emerge in patient care is to:
Dizon is admitted to the ward with an order of hourly
monitoring of blood pressure. The nurse finds that the a. Act only when advised that the action is ethically
cuff is too narrow and this will cause the blood pressure sound
reading to be: b. Not take sides remain neutral and fair
c. Assume that ethical questions are the responsibility: of
a. Inconsistent the health team
b. low systolic and high diastolic pressure d. Be accountable for his or her own actions
c. higher than what the reading should be
d. lower than what the reading should be 64. You inform the patient about his rights which
include the following EXCEPT:
59. Through the client’s health history, you gather that
19
19

a. Have the registered nurse, family spokesperson, nurse


a. Right to expect reasonable continuity of care supervisor and doctor sign
b. Right to consent to or decline to participate in b. Have two nurses validate the phone order, both
research studies or experiments nurses sign the order and the doctor should sign his
c. Right to obtain information about another patient order within 24 hours.
d. Right to expect that the records about his care will be c. Have the registered nurse, family and doctor sign
treated as confidential the order
d. Have 1 nurse take the order and sign it and have the
65. The principle states that a person has doctor sign it within 24 hours
unconditional worth and has the capacity to determine
his own destiny. 69. To ensure the client safety before starting
blood transfusion the following are needed before
a. Bioethics the procedure can be done EXCEPT:
b. Justice
c. Fidelity a. take baseline vital signs
d. Autonomy b. blood should be warmed to room temperature for 30
minutes before blood transfusion is administered
Situation 14 – Your director of nursing wants to improve c. have two nurses verify client identification, blood
the quality of health care offered in the hospital. As a type, unit number and expiration date of blood
staff nurse in that hospital you know that this entails d. get a consent signed for blood transfusion
quality assurance programs.
70. Part of standards of care has to do with the use
66. The following mechanisms can be utilized as part of of restraints. Which of the following statements is
the quality assessment program of your hospital NOT true?
EXCEPT:
a. Doctor’s order for restraints should be signed within
a. Patient satisfaction surveys provided 24 hours
b. Peer review clinical records of care of client b. Remove and reapply restraints every two hours
c. RO of the Nursing Intervention Classification c. Check client’s pulse, blood pressure and circulation
d. every four hours
d. Offer food and toileting every two hours
67. The nurse of the Standards of Nursing Practice is
important in the hospital. Which of the following Situation 15 – During the NUTRITION EDUCATION class
statements best describes what it is? discussion a 58 year old man, Mr. Bruno shows increased
interest.
a. These are statements that describe the maximum or
highest level of acceptable performance in nursing 71. Mr. Bruno asks what the "normal" allowable salt
practice. intake is. Your best response to Mr. Bruno is:
b. It refers to the scope of nursing as defined in Republic
Act 9173 a. 1 tsp of salt/day with iodine and sprinkle of MSG
c. It is a license issued by the Professional Regulation b. 5 gms per day or 1 tsp of table salt/day
Commission to protect the public from substandard c. 1 tbsp of salt/day with some patis and toyo
nursing practice. d. 1 tsp of salt/day but not patis or toyo
d. The Standards of care includes the various steps of the
nursing process and the standards of professional 72. Your instructions to reduce or limit salt
performance. intake include all the following EXCEPT:
a. eat natural food with little or no salt added
68. You are taking care of critically ill client and the b. limit use of table salt and use condiments instead
doctor in charge calls to order a DNR (do not c. use herbs and spices
resuscitate) for the client. Which of the following is d. limit intake of preserved or processed food
the appropriate action when getting DNR order over
the phone? 73. Teaching strategies and approaches when
giving nutrition education is influenced by age, sex
and
immediate concerns of the group. Your presentation nitroglycerin to your client. The following important
for a group of young mothers would be best if you guidelines to observe EXCEPT:
focus on:
a. Apply to hairlines clean are of the skin not subject to
a. diets limited in salt and fat much wrinkling
b. harmful effect on drugs and alcohol intake b. Patches may be applied to distal part of the
c. commercial preparation of dishes extremities like forearm
d. cooking demonstration and meal planning c. Change application and site regularly to prevent
irritation of the skin
74. Cancer cure is dependent on d. Wear gloves to avoid any medication of your hand

a. use of alternative methods of healing 79. You will be applying eye drops to Miss Romualdez.
b. watching out for warning signs of cancer After checking all the necessary information and
c. proficiency in doing breast self-examination cleaning the affected eyelid and eyelashes you
d. early detection and prompt treatment administer the ophthalmic drops by instilling the eye
drops.
75. The role of the health worker in health education is
to: a. directly onto the cornea
b. pressing on the lacrimal duct
a. report incidence of non-communicable disease to c. into the outer third of the lower conjunctival sac
community health center d. from the inner canthus going towards the side of the
b. educate as many people about warning signs of non- eye
communicable diseases
c. focus on smoking cessation projects 80. When applying eye ointment, the
d. monitor clients with hypertension following guidelines apply EXCEPT:

Situation 16 – You are assigned to take care of 10 a. squeeze about 2 cm of ointment and gently close
patients during the morning shift. The endorsement but not squeeze eye
includes the IV infusion and medications for these b. apply ointment from the inner canthus going outward
clients. of the affected eye
c. discard the first bead of the eye ointment before
76. Mr. Felipe, 36 years old is to be given 2700ml of application because the tube likely to expel more
D5RL to infuse for 18 hours starting at 8am. At what than desired amount of ointment
rate should the IV fluid be flowing hourly? d. hold the tube above the conjunctival sac do not let tip
touch the conjuctiva
a. 100 ml/hour
b. 210 ml/hour Situation 17 – The staff nurse supervisor request all the
c. 150 ml/hour staff nurses to “brainstorm” and learn ways to instruct
d. 90 ml/hour diabetic clients on self-administration of insulin. She
wants to ensure that there are nurses available daily to
77. Mr. Atienza is to receive 150mg/hour of D5W IV do health education classess.
infusion for 12 hours for a total of 1800ml. He is also
losing gastric fluid which must be replaced every 81. The plan of the nurse supervisor is an example of
two hours. Between 8am to 10am. Mr. Atienza has
lost 250ml of gastric fluid. How much fluid should he a. in service education process
receive at 11am? b. efficient management of human resources
c. increasing human resources
a. 350 ml/hour d. primary prevention
b. 275 ml/hour
c. 400 ml/hour 82. When Mrs. Guevarra, a nurse, delegates aspects
d. 200 ml/hour of the clients care to the nurse-aide who is an
unlicensed staff, Mrs. Guevarra.
78. You are to apply a transdermal patch of

19
19
a. makes the assignment to teach the staff member d. wellness center
b. is assigning the responsibility to the aide but not the
accountability for those tasks 88. Part of teaching client in health promotion is
c. does not have to supervise or evaluate the aide responsibility for one’s health. When Danica states
d. most know how to perform task delegated she need to improve her nutritional status this means:

83. Connie, the-new nurse, appears tired and sluggish a. Goals and interventions to be followed by client are
and lacks the enthusiasms she give six weeks ago based on nurse's priorities
when she started the job. The nurse supervisor should: b. Goals and intervention developed by nurse and client
should be approved by the doctor
a. empathize with the nurse and listen to her c. Nurse will decide goals and, interventions needed to
b. tell her to take the day off meet client goals
c. discuss how she is adjusting to her new job d. Client will decide the goals and interventions required
d. ask about her family life to meet her goals

84. Process of formal negotiations of working 89. Nurse Beatrice is providing tertiary prevention to
conditions between a group of registered nurses and Mrs. De Villa. An example of tertiary provestion is:
employer is:
a. Marriage counseling
a. grievance b. Self-examination for breast cancer
b. arbitration c. Identifying complication of diabetes
c. collective bargaining d. Poison, control
d. strike
90. Mrs. Ostrea has a schedule for Pap Smear. She has a
85. You are attending a certification program on strong family history of cervical cancer. This is an
cardiopulmonary resuscitation (CPR) offered and example of:
required by the hospital employing you. This is;
a. tertiary prevention
a. professional course towards credits b. secondary prevention
b. in-service education c. health screening
c. advance training d. primary prevention
d. continuing education
Situation: 19 - Ronnie has a vehicular accident where he
Situation 18 - There are various developments in health sustained injury to his left ankle. In the Emergency
education that the nurse should know about. Room, you notice how anxious he looks.

86. The provision of health information in the 91. You establish rapport with him and to reduce
rural areas nationwide through television and his anxiety you initially
radio programs and video conferencing is referred
to as: a. Take him to the radiology, section for X-ray of affected
extremity
a. Community health program b. Identify yourself and state your purpose in being with
b. Telehealth program the client
c. Wellness program c. Talk to the physician for an order of Valium
d. Red cross program d. Do inspection and palpation to check extent of his
injuries
87. A nearby community provides blood pressure
screening, height and weight measurement smoking 92. While doing your assessment, Ronnie asks you "Do
cessation classes and aerobics class services. This type I have a fracture? I don't want to have a cast.” The
of program is referred to as: most appropriate nursing response would be:

a. outreach program a. "You have to have an X-ray first to know if you have
b. hospital extension program a fracture."
c. barangay health center
b. "Why do you; sound so scared? It is just a cast and it's
not painful"
c. "You seem to be concerned about being in a cast."
d. "Based on my assessment, there doesn’t seem to be a
fracture."

19
19
ANSWER KEY - FOUNDATION OF PROFESSIONAL 51. D
NURSING PRACTICE 52. C
53. D
1. C 54. B
2. B 55. B
3. D 56. C
4. D 57. D
5. B 58. C
6. B 59. B
7. C 60. C
8. D 61. D
9. B 62. C
10. B 63. D
11. B 64. C
12. C 65. D
13. C 66. D
14. D 67. A
15. A 68. D
16. A 69. D
17. C 70. C
18. B 71. B
19. A 72. B
20. C 73. D
21. D 74. D
22. C 75. B
23. D 76. C
24. D 77. -
25. C 78. B
26. D 79. B
27. C 80. C
28. D 81. C
29. D 82. B
30. C 83. C
31. B 84. C
32. C 85. B
33. C 86. B
34. C 87. A
35. B 88. D
36. D 89. C
37. D 90. B
38. B 91. B
39. B 92. C
40. C
41. D
42. B
43. D
44. D
45. C
46. C
47. D
48. D
49. B
50. D
calorin intake for breast-feeding. By how much should a
lactating mother increase her caloric intake during the
COMMUNITY HEALTH NURSING AND CARE OF THE first 6 months after birth?
MOTHER AND CHILD
a. 350 kcal/day
Situation 1 - Nurse Minette is an independent Nurse b. 5CO kcal/day
Practitioner following-up referred clients in their c. 200 kcal/day
respective homes. Here she handles a case of d. 1,000 kcal/day
POSTPARTIAL MOTHER AND FAMILY focusing on
HOME CARE. Situation 2 - As the CPES is applicable for all professional
nurse, the professional growth and development of
1. Nurse Minette needs to schedule a first home visit to Nurses with specialties shall be addressed by a Specialty
OB client Leah. When is a first home-care visit typically Certification Council.
made? The following questions apply to these special groups of
nurses.
a. Within 4 days after discharge
b. Within 24 hours after discharge 6. Which of the following serves as the legal basis
c. Within 1 hour after discharge and statute authority for the Board of nursing to
d. Within 1 week of discharge promulgate measures to effect the creation of a
Specialty Certification Council and promulgate
2. Leah is developing constipation from being on bed professional development programs for this group of
rest. What measures would you suggest she take to nurse-professionals?
help prevent this?
a. R.A. 7610
a. Eat more frequent small meals instead of three large b. R.A. 223
one daily c. R.A. 9173
b. Walk for at least half an hour daily to stimulate d. R.A. 7164
peristalsis
c. Drink more milk, increased calcium intake prevents 7. By force of law, therefore, the PRC-Board of Nursing
constipation released Resolution No. 14 Series of the entitled:
d. Drink eight full glasses of fluid such as water daily "Adoption of a Nursing Specialty Certification Program
and Creation of Nursing Specialty Certification Council."
3. If you were Minette, which of the following actions, This rule-making power is called:
would alert you that a new mother is entering a
postpartial at taking-hold phase? a. Quasi-Judicial Power
b. Regulatory Power
a. She urges the baby to stay awake so that she can c. Quasi/Legislative Power
breast-feed him in her d. Executive/Promulgation Power
b. She tells you she was in a lot of pain all during labor
c. She says that she has not selected a name fir the 8. Under the PRC-Board of Nursing Resolution
baby as yet promulgating the adoption of a Nursing Specialty-
d. She sleeps as if exhausted from the effort of labor Certification Program and Council, which two (2) of the
following serves as the strongest for its enforcement?
4. At 6-week postpartum visit what should this (a) Advances made in science aid technology have
postpartial mother's fundic height be? provided the climate for specialization in almost all
aspects of human endeavor and
a. Inverted and palpable at the cervix (b) As necessary consequence, there has emerged a new
b. Six fingerbreadths below the umbilicus concept known as globalization which seeks to remove
c. No longer palpable on her abdomen barriers in trade, .industry and services imposed by the
d. One centimeter above the symphysis pubis national laws of countries all over the world; and
(c) Awareness of this development should impel the
5. This postpartal mother wants to loose the weight nursing sector to prepare our people in the services
she gained in pregnancy, so she is reluctant to increase sector to meet .the above challenges; and
her
20
20
(d) Current trends of specialization in nursing practice be acceptable TRUTHS applied to Community Health
recognized by; the International Council of Nurses (ICN) Nursing Practice.
of which the Philippines is a member for the benefit of
the Filipino in terms of deepening and refining nursing 11. Which of the following is the primary focus
practice and enhancing the quality of nursing care. of community health nursing practice?

a. b & c are strong justification a. Cure of illnesses


b. a & b are strong justification b. Prevention of illness
c. a & c are strong justification c. Rehabilitation back to health
d. a & d are strong justification d. Promotion of health

9. Which of the following is NOT a correct statement 12. In community health nursing, which of the
as regards Specialty Certification? following is our unit of service as nurses?

a. The Board of Nursing intended to create the Nursing a. The Community


Specialty Certification Program as a means of b. The Extended Members of every family
perpetuating the creation of an elite force of Filipino c. The individual members of the Barangay
Nurse Professionals d. The Family
b. The Board of Nursing shall oversee the administration
of the NSCP through the various Nursing Specialty 13. A very important part of the Community Health
Boards which will eventually, be created Nursing Assessment Process includes
c. The Board of Nursing at the time exercised their
powers under R.A. 7164 in order to adopt the creation of a. the application of professional judgment in estimating
the Nursing Specialty Certification /council and Program importance of facts to family and community
d. The Board of Nursing consulted nursing leaders of b. evaluation structures arid qualifications of health
national nursing associations and other concerned center team
nursing groups which later decided to ask a special group c. coordination with other sectors in relation to health
of nurses of .the program for nursing specialty concerns
certification d. carrying out nursing procedures as per plan of action

10. The NSCC was created for the purpose of 14. In community health nursing it is important to
implementing the Nursing Specialty policy under the take into account the family health with an equally
direct supervision and stewardship of the Board of important need to perform ocular inspection of the
Nursing. Who shall comprise the NSCC? areas activities which are powerful elements of:

a. A Chairperson who is the current President of the APO a. evaluation


a member from .the Academe, and the last member b. assessment
coming from the Regulatory Board c. implementation
b. The Chairperson and members of the Regulatory d. planning
Board ipso facto acts as the CPE Council
c. A Chairperson, chosen from among the Regulatory 15. The initial step in the PLANNING process in order
Board Members, a Vice Chairperson appointed by the to engage in any nursing project or parties at the
BON at-large; two other members also chosen at-large; community level involves:
and one representing the consumer group
d. A Chairperson who is the President of the Association a. goal-setting
from the Academe; a member from the Regulatory b. monitoring
Board, and the last member coming from the APO c. evaluation of data
d. provision of data
Situation 3 - Nurse Anna is a new BSEN graduate and has
just passed her Licensure Examination for Nurses in the Situation 4 - Please continue responding as a
Philippines. She has likewise been hired as a new professional nurse in these other health situations
Community Health Nurse in one of the Rural Health through the following questions.
Units in their City, which of the following conditions may
16. Transmission of HIV from an infected individual a. Prostaglandins released from the cut fallopian tubes
to another person occurs: can kill sperm
b. Sperm cannot enter the uterus, because the cervical
a. Most frequency in nurses with needlesticks entrance is blocked
b. Only if there is a large viral load in the blood c. Sperm can no longer reach the ova, because the
c. Most commonly as a result of sexual contact fallopian tubes are blocked
d. In all infants born to women with HIV infection d. The ovary no longer releases ova, as there is no where
for them to go
17. The medical record of a client reveals a condition
in which the fetus cannot pass through the maternal 22. The Dators are a couple undergoing testing for
pelvis. The nurse interprets this as: infertility. Infertility is said to exist when:

a. Contracted pelvis a. a woman has no uterus


b. Maternal disproportion b. a woman has no children
c. Cervical insufficiency c. a couple has been trying to conceive for 1 year
d. Fetopelvic disproportion d. a couple has wanted a child for 6 months

18. The nurse would anticipate a cesarean birth for a 23. Another client names Lilia is diagnosed as having
client who has which infection present at the onset of endometriosis. This condition interferes with the
labor? fertility because:

a. Herpes simplex virus a. endometrial implants can block the fallopian tubes
b. Human papilloma virus b. the uterine cervix becomes inflamed and swollen
c. Hepatitis c. ovaries stop producing adequate estrogen
d. Toxoplasmosia d. pressure on the pituitary leads to decreased FSH levels

19. After a vaginal examination, the nurse»e 24. Lilia is scheduled to have a hysterosalpingogram.
determines that the client's fetus is in an occiput Which of the following, instructions would you give her
posterior position. The nurse would anticipate that the regarding this procedure?
client will have:
a. She will not be able to conceive for 3 months after the
a. A precipitous birth procedure
b. Intense back pain b. The sonogram of the uterus will reveal any tumors
c. Frequent leg cramps present
d. Nausea and vomiting c. Many women experience mild bleeding as an after
effect
20. The rationales for using a prostaglandin gel for a d. She may feel some cramping when the dye is inserted
client prior to the induction of labor is to:
25. Lilia's cousin on the other hand, knowing nurse
a. Soften and efface the cervix Lorena's specialization asks what artificial insemination
b. Numb cervical' pain receptors by donor entails. Which would be your best answer if
c. Prevent cervical lacerations you were Nurse Lorena?
d. Stimulate uterine contractions
a. Donor sperm are introduced vaginally into the uterus
Situation 5 - Nurse Lorena is a Family Planning and or cervix
Infertility Nurse Specialist and currently attends to b. Donor sperm are injected intra-abdominally into each
FAMILY PANNING CLIENTS AND INFERTILE COUPLES. The ovary
following conditions pertain to meeting the nursing of c. Artificial sperm are injected vaginally to test tubal
this particular population group. patency
d. The husband's sperm is administered intravenously
21. Dina, 17 years old, asks you how a tubal ligation weekly
prevents pregnancy. Which would be the best answer?
Situation 6 - There are other important basic knowledge

20
20
in the performance of our task as Community Health pertalos to documentation/records management.
Nurse in relation to IMMUNIZATION these include:
31. This special form used when the patient is admitted
26. The correct temperature to store vaccines in a to the unit. The nurse completes, the information in
refrigerator is: this records particularly his/her .basic personal data,
current illness, previous health history, health history
a. between -4 deg C and +8 deg C of the family, emotional profile, environmental history
b. between 2 deg C and +8 deg C as well as physical assessment together with nursing
c. between -8 deg C and 0 deg C diagnosis on admission. What do you call this record?
d. between -8 deg C and +8 deg C
a. Nursing Kardex
27. Which of the following vaccines is not done b. Nursing Health History and Assessment Worksheet
by intramuscular (IM) injection? c. Medicine and Treatment Record
d. Discharge Summary
a. Measles vaccine
b. DPT 32. These, are sheets/forms which provide an efficient
c. Hepa B vaccines and time saving way to record information that must
d. DPT be obtained repeatedly at regular and/or short
intervals, of .time. This does not replace the progress
28. This vaccine content is derived from RNA notes; instead this record of information on vital signs,
recombinants: intake and output, treatment, postoperative care,
postpartum care, and diabetic regimen, etc., this is
a. Measles used whenever specific measurements or observations
b. Tetanus toxoids are needed to-be documented repeatedly. What is
c. Hepatitis B vaccines this?
d. DPT
a. Nursing Kardex
29. This is the vaccine needed before a child reaches b. Graphic Flow sheets
one (1) year in order for him/her to qualify as a "fully c. Discharge Summary
immunized child". d. Medicine and Treatment Record

a. DPT 33. These records show all medications and


b. Measles treatment provided on a repeated basis. What do you
c. Hepatitis B call this record?
d. BCG
a. Nursing Health History and Assessment Worksheet
30. Which of the following dose of tetanus toxoid b. Discharge Summary
is given to the mother to protect her .infant from c. Nursing Kardex
neonatal tetanus and likewise provide 10 years d. Medicine and Treatment Record
protection for the mother?
34. This flip-over card is usually kept in a portable file at
a. Tetanus toxoid 3 the Nurses Station. It has 2-parts: the activity and
b. Tetanus toxoid 2 treatment section and a nursing care plan section. This
c. Tetanus toxoid 1 carries information about basic demographic data,
d. Tetanus toxoid 4 primary medical diagnosis, current orders of the
physician to be carried out by the nurse, written
Situation 7 - Records contain those, comprehensive nursing care plan, nursing orders, scheduled tests and
descriptions of patient's health conditions and needs and procedures, safety precautions in-patient care and
at the same serve as evidences of every nurse's factors related to daily living activities/ this record is
accountability in the, care giving process. Nursing used in the charge-of-shift reports or during the beside
records normally differ from institution to, institution rounds or walking rounds. What record is this?
nonetheless they follow similar patterns of .meeting
needs for specifics, types of information. The following a. Discharge Summary
b. Medicine and Treatment Record a. A nurse withholding harmful information to the family
c. Nursing Health History and Assessment Worksheet members of a patient
d. Nursing Kardex b. A nurse declining commission sent by a doctor for her
referral
35. Most nurses regard this as conventional recording c. A nurse endorsing a person running for congress
of the date, time and mode by which the patient leaves d. Nurse Reviewers and/or nurse review center
a healthcare unit but this record includes importantly, managers who pays a considerable amount of cash for
directs of planning for discharge that starts soon after reviewees who would memorize items from the
the' person is admitted to a healthcare institution, it is Licensure exams and submit these to them after the
accepted that collaboration or multidisciplinary examination
involvement (of all members of the health team) in
discharge results in comprehensive care. What do you 39. A nurse should be cognizant that professional
call this? programs for specialty certification by the Board of
Nursing are accredited through the
a. Discharge Summary
b. Nursing Kardex a. Professional Regulation Commission
c. Medicine and Treatment Record b. Nursing Specialty Certification Council
d. Nursing Health History and Assessment Worksheet c. Association of Deans of Philippine Colleges of Nursing
d. Philippine Nurse Association
Situation 8 - As Filipino Professional Nurses we must be
knowledgeable, about the Code of Ethics for Filipino 40. Mr. Santos, R.N. works in a nursing home, and he
Nurses and practice these by heart. The next questions knows that one of his duties is to be an advocate for
pertain to this Code of Ethics. his patients. Mr. Santos knows a primary duty of an
advocate is to:
36. Which of the following is TRUE about the Code of
Ethics of Filipino Nurses? a. act as the patient's legal representative
b. complete all nursing responsibilities on time
a. The Philippine Nurses Association for being the c. safeguard the well being of every patient
accredited professional organization was given the d. maintain the patient's right to privacy
privilege to formulate a Code of Ethics which the Board
of Nurses promulgated Situation 9 - Nurse Joanna works as an OB-Gyne Nurse
b. Code of Nurses was first formulated in 1982 published and attends to several HIGH-RISK PREGNANCIES:
in the Proceedings of the Third Annual Convention of the Particularly women with preexisting of Newly Acquired
PNA House of Delegates illness. The following conditions apply.
c. The present code utilized the Code of Good
Governance for the Professions in the Philippines 41. Bernadette is a 22-year old woman. Which
d. Certificate of Registration of registered nurses; may be condition would make her more prone than others to
revoked or suspended for violations of any provisions of developing a Candida infection during pregnancy?
the Code of Ethics
a. Her husband plays gold 6 days a week
37. Based on the Code of Ethics for Filipino b. She was over 35 when she became pregnant
Nurses, what is regarded as the hallmark of c. She usually drinks tomato juice for breakfast
nursing responsibility and accountability? d. She has developed gestational diabetes

a. Human rights of clients, regardless of creed and 42. Bernadette develops a deep-vein thrombosis
gender following an auto accident and is prescribed heparin
b. The privilege of being a registered professional nurses sub-Q. What should Joanna educate her about in regard
c. Health, being a fundamental right of every individual to this?
d. Accurate documentation of actions and outcomes
a. Some infants will be born with allergic symptoms to
38. Which of the following nurses behavior is heparin
regarded as a violation of the Code of Ethics of Filipino b. Her infant will be born with scattered petechiae on his
Nurses? trunk

20
20
c. Heparin can cause darkened skin in newborns children with cough
d. Heparin does not cross the placenta and so does not c. Refer to the doctor
affect a fetus d. Teach the mother how to count her child's bearing

43. The cousin of Bernadette with sickle-cell anemia 47. In responding to the care concerns of children with
alerted Joanna that she may need further instruction severe disease, referral to the hospital of the essence
on prenatal care. Which statement signifies this fact? especially if the child manifests which of the following?

a. I've stopped jogging so I don't risk becoming a. Wheezing


dehydrated b. Stopped bleeding
b. I take an iron pull every day to help grown new red c. Fast breathing
blood cells d. Difficulty to awaken
c. I am careful to drink at least eight glasses of fluid
everyday 48. Which of the following is the most important
d. 1 understand why folic acid is important for red cell responsibility of a nurse in the prevention of
formation necessary deaths from pneumonia and other severe
diseases?
44. Bernadette routinely takes acetylsalicylic acid
(aspirin) for arthritis. Why should she limit or a. Giving of antibiotics
discontinue this toward the end of pregnancy? b. Taking of the temperature of the sick child
c. Provision of Careful Assessment
a. Aspirin can lead to deep vein thrombosis following d. Weighing of the sick child
birth
b. Newborns develop a red rash from salicylate toxicity 49. You were able to identify factors that lead to
c. Newborns develop withdrawal headaches from respiratory problems in the community where your
salicylates health facility serves. Your primary role therefore
d. Salicyates can lead to increased maternal bleeding at in order to reduce morbidity due to pneumonia is
childbirth to:

45. Bernadette received a laceration on her leg from a. Teach mothers how to recognize early signs and
her automotive accident. Why are lacerations of symptoms of pneumonia
lower extremities potentially more serious in b. Make home visits to sick children
pregnant women than others? c. Refer cases to hospitals
d. Seek assistance and mobilize the BHWs to have a
a. Lacerations can provoke allergic responses because meeting with mothers
of gonadothropic hormone
b. Increased bleeding can occur from uterine pressure on 50. Which of the following is the principal focus on the
leg veins CARI program of the Department of Health?
c. A woman is less able to keep the laceration clean
because o f her fatigue a. Enhancement of health team capabilities
d. Healing is limited during pregnancy, so these will not b. Teach mothers how to detect signs and where to refer
heal until after birth c. Mortality reduction through early detection
d. Teach other community health workers how to assess
Situation 10 - Still in your self-managed Child Health patients
Nursing Clinic, your encounter these cases pertaining to
the CARE OF CHILDREN WITH PULMONARY AFFECTIONS. Situation 11 - You are working as a Pediatric Nurse in
your own Child Health Nursing Clinic, the following cases
46. Josie brought her 3-rnonths old child to your clinic pertain to ASSESSMENT AND CARE OP THE NEWBORN
because of cough and colds. Which of the following is AT
your primary action? RISK conditions.

a. Give contrimoxazole tablet or syrup 51. Theresa, a mother with a 2 year old daughter asks,
b. Assess the patient using the chart on management of "at what are can I be able to take the blood pressure of
my daughter as a routine procedure since hypertension
is common in the family?" Your answer to this is:

20
a. At 2 years you may 57. Which of the following conditions is NOT true about
b. As early as 1 year old
c. When she's 3- years old
d. When she's 6 years old?

52. You typically gag children to inspect the back of


their throat. When is it important NOT to solicit a
gag reflex?

a. when a girl has a geographic tongue


b. when a boy has a possible inguinal hernia
c. when a child has symptoms of epiglottitis
d. when children are under 5 years of age

53. Baby John was given a drug at birth to reverse the


effects of a narcotic given to his mother in' labor. What
drug is commonly used for this?

a. Naloxone (Narcan)
b. Morphine Sulfate
c. Sodium Chloride
d. Penicillin G

54. Why are small-for-gestational-age newborns at risks


for difficulty maintaining body temperature?

a. They do not have as many fat stores as other infant’s


b. They are more active than usual so throw off covers
c. Their skin is more susceptible to conduction of cold
d. They are preterm so are born relatively small in size

55. Baby John develops hyperbilirubinemia. What is a


method used to treat hyperbilirubinemia in a
newborn?

a. Keeping infants in a warm arid dark environment


b. Administration of a cardiovascular stimulant
c. Gentle exercise to stop muscle breakdown
d. Early feeding to speed passage of meconium

Situation 12 - You are the nurse in the Out-Patient-


Department and during your shift you encountered
multiple children's condition. The following questions
apply.

56. You assessed a child with visible severe wasting,


he has:

a. edema
b. LBM
c. kwashiorkor
d. marasmus

20
20
contraindication to immunization? d. 28 cm, and fetal heart can be heard with a Doppler

a. do not give DPT2 or DPT3 to a child who


has convulsions within 3 days of DPT1
b. do not give BOG if the child has known hepatitis .
c. do not give OPT to a child who has
recurrent convulsion or active neurologic
disease
d. do not give BCG if the child has known AIDS

58. Which of the following statements


about immunization is NOT true:

a. A child with diarrhea who is due for OPV should


receive the OPV and make extra dose on the next
visit
b. There is no contraindication to immunization if
the child is well enough to go home
c. There is no contraindication to immunization if the
child is well enough to go home and a child should be
immunized in the health center before referrals are
both correct
d. A child should be immunized in the center
before referral

59. A child with visible severe wasting or severe


palmar pallor may be classified as:

a. moderate malnutrition/anemia
b. severe malnutrition/anemia
c. not very tow weight no anemia
d. anemia/very low weight

60. A child who has some palmar pallor can


be classified as:

a. moderate anemia/normal weight


b. severe malnutrition/anemia
c. anemia/very low weight
d. not very low eight to anemia

Situation 13 - Nette, a nurse palpates the abdomen of


Mrs. Medina, a primigravida. She is unsure of the date
of her last menstrual period. Leopold's Maneuver is
done. The obstetrician told mat she appears to be 20
weeks pregnant. .

61. Nette explains this because the fundus is:

a. At the level the umbilicus, and the fetal heart can


be heard with a fetoscope
b. 18 cm, and the baby is just about to move
c. is just over the symphysis, and fetal heart cannot
be heard
20

b. Nifedipine
62. In doing Leopold's maneuver palpation c. Butorphanol
which among the following is NOT considered a d. Diazepam
good preparation?
67. RhoGAM is given to Rh-negative women to
a. The woman should lie in a supine position wither prevent maternal sensitization from occurring. The
knees flexed slightly nurse is aware that in addition to pregnancy, Rh-
b. The hands of the nurse should be cold so that negative women would also receive this medication
abdominal muscles would contract and tighten after which of the following?
c. Be certain that your hands are warm (by washing them
in warm water first if necessary) a. Unsuccessful artificial insemination procedure
d. The woman empties her bladder before palpation b. Blood transfusion after hemorrhage
c. Therapeutic or spontaneous abortion
63. In her pregnancy, she experienced fatigue d. Head injury from a car accident
and drowsiness. This probably occurs because:
68. Which of the following would the nurse include
a. of high blood pressure when describing the pathophysiologv of gestational
b. she is expressing pressure diabetes?
c. the fetus utilizes her glucose stores and leaves her
with a Sow blood glucose a. Glucose levels decrease to accommodate fetal growth
d. of the rapid growth of the fetus b. Hypoinsulinemia develops early in the first trimester
c. Pregnancy fosters the development of carbohydrate
64. The nurse assesses the woman at 20 cravings
weeks gestation3 and expects the woman to d. There is progressive resistance to the effects of insulin
report:
69. When providing prenatal education to a pregnant
a. Spotting related to fetal implantation woman with asthma, which of the following would be
b. Symptoms of diabetes as human placental lactogen is important for the nurse to do?
released
c. Feeling fetal kicks a. Demonstrate how to assess her blood glucose
d. Nausea and vomiting related HCG production b. Teach correct administration of subcutaneous
bronchodilators
65. If Mrs. Medina comes to you for check-up on June 2, c. Ensure she seeks treatment for any acute
her EDO is June 11, what do you expect during exacerbation
assessment? d. Explain that she should avoid steroids during her
pregnancy
a. Fundic ht 2 fingers below xyphoid process, engaged
b. Cervix close, uneffaced, FH-midway between the 70. Which of the following conditions would cause an
umbilicus and symphysis pubis insulin-dependent diabetic client the most difficulty
c. Cervix open, fundic ht. 2 fingers below xyphoid during her pregnancy?
process, floating .
d. Fundic height at least at the level of the xyphoid a. Rh incompatibility
process, engaged b. Placenta previa
c. Hyperemesis gravidarum
Situation 14: - Please continue responding as a d. Abruption placentae
professional nurse in varied health situations through
the following questions. Situation 15 - One important toot a community health
nurse uses in the conduct of his/her activities is the CHN
66. Which of the following medications would the Bag. Which of the following BEST DESCRIBES the use of
nurse expect the physician to order for recurrent this vital facility for our practice?
convulsive seizures of a 10-year old child brought to
your clinic? 71. The Community/Public Health Bag is:

a. Phenobarbital
a. a requirement for home visits and management from the Barangay Level to the Local
b. an essential and indispensable equipment of the Government/Municipal City Level.
community health nurse
c. contains basic medications and articles used by the 76. The following statements can correctly be
community health nurse made about Organization and management?
d. a tool used by the Community health nurse is
rendering effective nursing procedure during a home A. An organization (or company) is people. Values make
visit people persons: values give vitality, meaning and
direction to a company. As the people of an organization
72. What is the rationale in the use of bag technique value, so the company becomes.
during home visit? B. Management is the process by which administration
achieves its mission, goals, and objectives
a. It helps render effective nursing care to clients or C. Management effectiveness can be measured in terms
other members of the family of accomplishment of the purpose of the organization
b. It saves time and effort of the nurse in the while management efficiency is measured in terms of
performance of nursing procedures the satisfaction of individual motives
c. It should minimize or prevent the spread of infection D. Management principles are universal therefore
from individuals to families one need not be concerned about people, culture,
d. It should not overshadow concerns for the patient values, traditions and human relations.

73. Which among the following is important in the use a. B and C only
of the bag technique during home visit? b. A, B and D only
c. A and D only
a. Arrangement of the bag's contents must be d. B, A, and C only
convenient to the nurse
b. The bag should contain all necessary supplies and 77. Management by Filipino values advocates the
equipment ready for use consideration of the Filipino goals trilogy according to
c. Be sure to thoroughly clean your bag especially when the Filipino priority-values which are:
exposed to communicable disease cases
d. Minimize if not totally prevent the spread of infection a. Family goals, national goals, organizational goals
b. Organizational goats, national goals, family goals
74. This is an important procedure of the nurse c. National goals, organizational goals, family goals
during home visits? d. Family goals, organizational goals, national goals

a. protection of the CHN bag 78. Since the advocacy for the utilization of Filipino
b. arrangement of the contents of the CHM bag value-system in management has been encouraged,
c. cleaning of the CHN bag the Nursing sector is no except, management needs to
d. proper handwashing examine Filipino values and discover its positive
potentials and harness them to achieve:
75. In consideration of the steps in applying the bag
technique, which side of the paper lining of the CHN a. Employee satisfaction
bag is considered clean to make a non-contaminated b. Organizational commits .ants, organizational
work area? objectives and employee satisfaction
c. Employee objectives/satisfaction, commitments and
a. The lower lip organizational objectives
b. The outer surface d. Organizational objectives, commitments and
c. The upper lip employee objective/satisfaction
d. The inside surface
79. The following statements can correctly be made
Situation 16 - As a Community Health Nurse relating with about an effective and efficient community or even
people in different communities, and in the agency managerial-leader.
implementation of health programs and projects you A. Considers the achievement and advancement of the
experience vividly as well the varying forms of leadership organization she/he represents as well as his people

20
20
B. Considers the recognition of individual efforts toward wound healing
the realization of organizational goals as well as the d. A study examining client's feelings before, during and
welfare of his people after a bone marrow aspiration
C. Considers the welfare of the organization above all
other consideration by higher administration 83. Which of the following studies is based
D. Considers its own recognition by higher on qualitative research?
administration for purposes of promotion and prestige
a. A study examining clients reactions to stress after
a. Only C and D are correct open heart surgery
b. A, C and D are correct b. A study measuring nutrition and weight, loss/gain in
c. B, C, and D are correct clients with cancer
d. Only A and B are correct c. A study examining oxygen levels after endotracheal
suctioning
80. Whether management at the community or d. A study measuring differences in blood pressure
agency level, there are 3 essential types of skills before during and after a procedure
managers must have, these are:
A. Human relation skills, technical skills, and cognitive 84. An 85 year old client in a nursing home tells a nurse,
skills "I signed the papers for that research study because
B. Conceptual skills, human relation/behavioral skills, the doctor was so insistent and I want: him to continue
and technical skills taking care of me." Which client right is being violated?
C. Technical skills, budget and accounting skills, skills
in fund-raising a. Right of self determination
D. Manipulative skills, technical skills, resource b. Right to privacy and confidentiality
management skills c. Right to full disclosure
d. Right not to be harmed
a. A and D are correct
b. B is correct 85. "A supposition or system of ideas that is
c. A is correct proposed to explain a given phenomenon," best
d. C and D are correct defines:

Situation 17 - You are actively practicing nurse who just a. a paradigm


finished your Graduate Studies. You earned the value of b. a concept
Research and would like to utilize the knowledge and c. a theory
skills gained in the application of research to Nursing d. a conceptual framework
service. The following questions apply to research.
Situation 18 - Nurse Michelle works with a Family
81. Which type of research Inquiry investigates Nursing Team in Calbayog Province specifically handling
the issue of human complexity (e.g. understanding a UNICEF Project for Children. The following conditions
the human expertise) pertain, to CARE OP THE FAMILIES PRESCHOOLERS.

a. Logical position 86. Ronnie asks constant questions. How many does
b. Naturalistic inquiry a typical 3-year-old ask in a day's time?
c. Positivism
d. Quantitative Research a. 1,200 or more
b. Less than 50
82. Which of the following studies is based c. 100-200
on quantitative research? d. 300-400

a. A study examining the bereavement process in 87. Ronnie will need to change to a new bed
spouses of clients with terminal cancer because his baby sister will need Ronnie's old crib.
b. A study exploring factors influencing weight control What measure would you suggest that his parents
behavior take to help decrease sibling rivalry between Ronnie
c. A study measuring the effects of sleep deprivation on and his new sister?
a. Move him to the new bed before the baby arrives 93. An ear discharge that has been present for
b. Explain that new sisters grow up to become best more than 14 days can be classified as:
friends
c. Tell him he will have to share with the new baby a. mastoditis
d. Ask him to get his crib ready for the new baby b. chronic ear infection
c. acute ear infection
88. Ronnie's parents want to know how to react to him d. complicated ear infection
when he begins to masturbate while watching
television. What would you suggest? 94. An ear discharge that has been present for jess
than 14 days can be classified as:
a. They refuse to allow him to watch television
b. They schedule a health check-up for sex-related a. chronic ear infection
disease b. mastoditis
c. They remind him that some activities are private c. acute ear infection
d. They give him "timeout" when this begins d. complicated ear infection

89. How many words does a typical 12-month-old 95. If the child has severe classification because of ear
infant use? problem, what would be the best thing that you as
the nurse can do?
a. About 12 words
b. Twenty or more words a. instruct mother when to return immediately
c. About 50 words b. refer urgently
d. Two, plus "mama" and "dada" c. give an antibiotic for 5 days
d. dry the ear by wicking
90. As a nurse. You reviewed infant safety
procedures with Bryan's mother. What are two of the Situation 20 - If a child with diarrhea registers one sign in
most common types of accidents among infants? the pink row and one in the yellow; row in the IMCI
Chart.
a. Aspiration and falls
b. Falls and auto accidents 96. We can classify the patient as:
c. Poisoning and burns
d. Drowning and homicide a. moderate dehydration
b. some dehydration
Situation 19 - Among common conditions found in c. no dehydration
children especially among poor communities are ear d. severe dehydration
infection/problems. The following questions apply.
97. The child with no dehydration needs home
91. A child with ear problem should be assessed for treatment Which of the following is not included the
the following EXCEPT: rules for home treatment in this case:

a is there any fever? a. continue feeding the child


b. ear discharge b. give oresol every 4 hours
c. if discharge is present for how long? c. know when to return to the health center
d. ear pain d. give the child extra fluids

92. If the child does not have ear problem, using 98. A child who has had diarrhea for 14 days but has
IMCI, what should you as the nurse do? no sign of dehydration is classified as:

a. Check for ear discharge a. severe persistent diarrhea


b. Check for tender swellings, behind the ear b. dysentery
c. Check for ear pain c. severe dysentery b. dysentery
d. Go to the next question, check for malnutrition d. persistent diarrhea

21
21
99. If the child has sunken eyes, drinking 28. C
eagerly, thirsty and skin pinch goes back slowly, 29. B
the classification would be: 30. D
31. B
a. no dehydration 32. B
b. moderate dehydration 33. D
c. some dehydration 34. D
d. severe dehydration 35. A
36. C
100. Carlo has had diarrhea for 5 days. There is no 37. C
blood in the stool, he is irritable. His eyes are 38. A
sunken the nurse offers fluid to Carlo and he drinks 39. B
eagerly. When the nurse pinched the abdomen, it 40. C
goes back slowly. How will you classify Carlo’s 41. D
illness? 42. D
43. B
a. severe dehydration 44. D
b. no dehydration 45. B
c. some dehydration 46. B
d. moderate dehydration 47. D
48. C
49. A
50. C
51. C
ANSWER KEY: COMMUNITY HEALTH NURSING
52. C
AND CARE OF THE MOTHER AND CHILD 53. A
54. A
1. A 55. D
2. B 56. D
3. A 57. B
4. C 58. A
5. B 59. B
6. D 60.
7. C 61. A
8. D 62. B
9. A 63. D
10. B 64. C
11. D 65. A
12. D 66. A
13. A 67. C
14. B 68. D
15. A 69. C
16. C 70. C
17. D 71. B
18. A 72. A
19. B 73. D
20. D 74. D
21. C 75. B
22. C 76. D
23. A 77. D
24. C 78. D
25. A 79. D
26. B 80. C
27. A
81. B
82. C
83. A
84. A
85. C
86. D
87. A
88. C
89. A
90. A
91. A
92. D
93. B
94. C
95. B
96. D
97. B
98. D
99. C
100. C

21
21
Comprehensive Exam 1
a. Security Division
Situation 1 - Concerted work efforts among members of b. Chaplaincy
the surgical team is essential to the success of the c. Social Service Section
surgical procedure. d. Pathology department

1. The sterile nurse or sterile personnel touch only Situation 2 - You are assigned in the Orthopedic Ward
sterile supplies and instruments. When there is a need where clients are complaining of pain in varying degrees
for sterile supply which is not in the sterile field, who upon movement of body parts.
hands out these items by opening its outer cover?
6. Troy is a one day post open reduction and internal
a. Circulating nurse fixation (ORIF) of the left hip and is in pain. Which of
b. Anesthesiologist the following observation would prompt you to call
c. Surgeon the doctor?
d. Nursing aide
a. Dressing is intact but partially soiled
2. The OR team performs distinct roles for one surgical b. Left foot is cold to touch and pedal pulse is absent
procedure to be accomplished within a prescribed time c. Left leg in limited functional anatomic position
frame and deliver a standard patient outcome. White d. BP 114/78, pulse of 82 beats/minute
the surgeon performs the surgical procedure, who
monitors the status of the client like urine output, 7. There is an order of Demerol 50 mg I.M. now and
blood loss? every 6 hours p r n. You injected Demerol at 5 pm. The
next dose of Demerol 50 mg I.M. is given:
a. Scrub nurse
b. Surgeon a. When the client asks for the next dose
c. Anesthesiologist b. When the patient is in severe pain
d. Circulating nurse c. At 11pm
d. At 12pm
3. Surgery schedules are communicated to the OR
usually a day prior to the procedure by the nurse of the 8. You continuously evaluate the client's adaptation to
floor or ward where the patient is confined. For pain. Which of the following behaviors-indicate
orthopedic cases, what department is usually informed appropriate adaptation?
to be present in the OR?
a. The client reports pain reduction and decreased
a. Rehabilitation department activity
b. Laboratory department b. The client denies existence of pain
c. Maintenance department c. The client can distract himself during pain episodes
d. Radiology department d. The client reports independence from watchers

4. Minimally invasive surgery is very much into 9. Pain in Ortho cases may not be mainly due to the
technology. Aside from the usual surgical team who surgery. There might be other factors such as cultural
else to be present when a client undergoes or psychological that influence pain. How can you alter
laparoscopic surgery? these factors as the nurse?

a. Information technician a. Explain all the possible interventions that may cause
b. Biomedical technician the client to worry.
c. Electrician b. Establish trusting relationship by giving his medication
d. Laboratory technicial on time
c. Stay with the client during pain episodes
5. In massive blood loss, prompt replacement of d. Promote client's sense of control and participation in
compatible blood is crucial. What department needs pain control by listening to his concerns
to be alerted to coordinate closely with the patient's
family for immediate blood component therapy? 10. In some hip surgeries, an epidural catheter for
Fentanyl epidural analgesia is given. What is your record, disposal. You know that your institution is
nursing priority care in such a case? covered by this policy it;

a. Instruct client to observe strict bed rest a. Your hospital is considered tertiary
b. Check for epidural catheter drainage b. Your hospital is in Metro Manila
c. Administer analgesia through epidural catheter as c. It obtained permit to operate from DOH
prescribed d. Your hospital is Philhealth accredited
d. Assess respiratory rate carefully
Situation 4 - In the OR, there are safety protocols that
Situation 3 - Records are vital tools in any institution and should be followed. The OR nurse should be well versed
should be properly maintained for specific use and time. with all these to safeguard the safety and quality to
patient delivery outcome.
11. The patient's medical record can work as a double-
edged swords. When can the medical record become 16. Which of the following should be given highest
the doctor's/nurse worst enemy? priority when receiving patient in the OR?

a. When the record is voluminous a. Assess level of consciousness


b. When a medical record is subpoenaed in court b. Verify patient identification and informed consent
c. When it is missing c. Assess vital signs
d. When the medical record is inaccurate, incomplete, d. Check for jewelry, gown, manicure and dentures
and inadequate
17. Surgeries like I and D (incision and drainage) and
12. Disposal of medical records in debribement are relatively short procedures but
government hospitals/institutions must be considered ‘dirty cases’. When are these; procedures
done in close coordination with what agency? best scheduled?

a. Department of Interior and Local Government (DILG) a. Last case


b. Metro Manila Development Authority (MMDA) b. In between cases
c. Records Management Archives Office (RMAO) c. According to availability of anesthesiologist
d. Depart of Health (DOH) d. According to the surgeon's preference

13. In the hospital, when you need-the medical record 18. OR nurses should be aware that maintaining the
of a discharged patient for research, you will request client's safety is the overall goal of nursing care
permission through: during the intraoperative phase. As the circulating
nurse, you make certain that throughout the
a. Doctor in charge procedure...
b. The hospital director
c. The nursing Service a. the surgeon greets his client before induction of
d. Medical records section anesthesia
b. the surgeon and anestheriologist are in tandem
14. You readmitted a client who was in another c. strap made of strong non-abrasive material are
department a month ago. Since you will need the fastened securely around the joints of the knees and
previous chart, from whom do you request the old ankles and around the 2 hands around an arm board
chart? d. client is monitored throughout the surgery by the
assistant anesthesiologist
a. Central supply section
b. Previous doctor's clinic 19. Another nursing check that should not be missed
c. Department where the patient was previously before the induction of general anesthesia is:
admitted
d. Medical records section a. check for presence underwear
b. check for presence dentures
15. Records Management and Archives Offices of the c. check patient's
DOH is responsible for implementing its policies on d. check baseline vital signs

21
21
20. Some different habits and hobbies affect 25. Which of the following nursing intervention is
postoperative respiratory function. If your client done when examining the incision wound and
smokes 3 packs of cigarettes a day for the part 10 years, changing the dressing?
you will anticipate increased risk for:
a. Observe the dressing and type and odor of drainage if
a. perioperative anxiety and stress any
b. delayed coagulation time b. Get patient's consent
c. delayed wound healing c. Wash hands
d. postoperative respiratory function d. Request the client to expose the incision wound

Situation 5 - Nurses hold a variety of roles when Situation 6 - Carlo, 16 years old, comes to the ER with
providing care to a perioperative patient. acute asthmatic attack. RR is 46/min and he appears to
be in acute respiratory distress.
21. Which of the following role would be
the responsibility of the scrub nurse? 26. Which of She following nursing actions should be
initiated first?
a. Assess the readiness of the client prior to surgery
b. Ensure that the airway is adequate a. Promote emotional support
c. Account for the number of sponges, needles, supplies, b. Administer oxygen at 6L/min
Used during the surgical procedure c. Suction the client every 30 min
d. Evaluate the type of anesthesia appropriate for the d. Administer bronchodilator by nebulizer
surgical client
27. Aminophylline was ordered for acute asthmatic
22. As a perioperative nurse, how can you best meet attack. The mother asked the nurse, what its indication
the safety need of the client after administering the nurse will say is:
preoperative narcotic?
a. Relax smooth muscles of the bronchial airway
a. Put side rails up and ask client not to get out of bed b. Promote expectoration
b. Send the client to ORD with the family c. Prevent thickening of secretions
c. Allow client to get up to go to the comfort room d. Suppress cough
d. Obtain consent form
28. You will give health instructions to Carlo, a case of
23. It is the responsibility of the pre-op, nurse to do bronchial asthma. The health instruction will include
skin prep for patients undergoing surgery. If hair at the following EXCEPT:
the operative site is not shaved, what should be done
to make suturing easy and lessen chance of incision a. Avoid emotional stress and extreme temperature
infection? b. Avoid pollution like smoking
c. Avoid pollens, dust seafood
a. Draped d. Practice respiratory isolation
b. Pulled
c. Clipped 29. The asthmatic client asked you what breathing
d. Shampooed technique he can best practice when asthmatic attack
starts. What will be the best position?
24. It is also the nurse's function to determine when
infection is developing in the surgical incision. The a. Sit in high-Fowler's position with extended legs
perioperative nurse should observe for what signs b. Sit-up with shoulders back
of impending infection? c. Push on abdomen during exhalation
d. Lean forward 30-40 degrees with each exhalation
a. Localized heat and redness
b. Serosanguinous exudates and skin blanching 30. As a nurse you are always alerted to monitor
c. Separation of the incision status asthmaticus who will likely and initially
d. Blood clots and scar tissue are visible manifest symptoms of:
a. metabolic alkalosis Incident Report (IR)
b. respiratory acidosis c. Allow client to walk with relative to the OF?
c. respiratory alkalosis d. Assess and periodically reassess individual client's risk
d. metabolic acidosis for falling

Situation 7 - Joint Commission on Accreditation of 35. As a nurse you know you can improve on
Hospital Organization (JCAHP) patient safety goals and accuracy of patient's identification by 2 patient
requirements include the care and efficient use of identifiers, EXCEPT:
technology in the OR arid elsewhere in the healthcare
facility. a. identify the client by his/her wrist tag and verify with
family members
31. As the head nurse in the OR, how can you b. identify client by his/her wrist tag and call his/her by
improve the effectiveness of clinical alarm systems? name
c. call the client by his/her case and bed number
a. limit suppliers to a few so that quality is maintained d. call the patient by his/her name and bed number
b. implement a regular inventory of supplies and
equipment Situation 8 - Team efforts is best demonstrated in the OR
c. Adherence to manufacturer's recommendation
d. Implement a regular maintenance and testing of alarm 36. If you are the nurse in charge for scheduling
systems surgical cases, what important information do you
need to ask the surgeon?
32. Over dosage of medication or anesthetic can
happen even with the aid of technology like infusion a. Who is your internist
pump, sphymomanometer, and similar b. Who is your assistant and anesthesiologist, and what
devices/machines. As a staff, how can you improve the is your preferred time and type of surgery?
safety of using infusion pumps? c. Who are your anesthesiologist, internist, and assistant
d. Who is your anesthesiologist.
a. Check the functionality of the pump before use
b. Select your brand of infusion pump like you do with 37. In the OR, the nursing tandem for every surgery is:
your cellphone
C. Allow the technician to set the; infusion pump before a. Instrument technician and circulating nurse
use b. Nurse anesthetist, nurse assistant, and instrument
d. Verify the flow rate against your computation technician
c. Scrub nurse and nurse anesthetist
33. JCAHOs universal protocol for surgical and invasive d. Scrub and circulating nurses
procedures to prevent wrong site, wrong person, and
wrong procedures/surgery includes the following 38. While team effort is needed in the OR for efficient
EXCEPT: and quality patient care delivery, we should limit the
number of people in the room for infection control.
a. Mark the operative site if possible Who comprise this team?
b. Conduct pre-procedure verification process
c. Take a video of the entire intra-operative procedure a. Surgeon, anesthesiologist, scrub nurse, radiologist,
d. Conduct time out immediately before starting the orderly
procedure b. Surgeon, assistants, scrub nurse, circulating nurse,
anesthesiologist
34. You identified a potential risk of pre and post c. Surgeon, assistant surgeon, anesthesiologist, scrub
operative clients. To reduce the risk of patient harm nurse, pathologist
resulting from fall, you can implement the following d. Surgeon, assistant surgeon, anesthesiologist, intern,
EXCEPT: scrub nurse

a. Assess potential risk of fail associated with the 39. When surgery is on-going, who coordinates the
patient's the following EXCEPT: medication regimen activities outside, including the family?
b. Take action to address any identified risks through

21
21
a. Orderly/clerk should be drained?
b. Nurse supervisor
c. Circulating nurse a. Sensation of taste
d. Anaesthesiologist b. Sensation of pressure
c. Sensation of smell
40. The breakdown in teamwork is often times a failure d. Urge to defecate
in:
Situation 10 - As a beginner in research, you are aware
a. Electricity that sampling is an essential element of the research
b. Inadequate supply process.
c. Leg work
d. Communication 46. What does a sample group represent?

Situation 9 - Colostomy is a surgically created anus- It a. Control group


can be temporary or permanent, depending on the b. Study subjects
disease condition. c. General population
d. Universe
41. Skin care around the stoma is critical. Which of
the following is not indicated as a skin care barriers? 47. What is the most important characteristics of
a. Apply liberal amount of mineral oil to the area a sample?
b. Use karaya paste and rings around the stoma
c. Clean the area daily with soap and water before a. Randomization
applying bag b. Appropriate location
d. Apply talcum powder twice a day c. Appropriate number
d. Representativeness
42. What health instruction will enhance regulation of a
colostomy (defecation) of clients? 48. Random sampling ensures that each subject has:

a. Irrigate after lunch everyday a. Been selected systematically


b. Eat fruits and vegetables in all three meals b. An equal change of selection
c. Eat balanced meals at regular intervals c. Been selected based on set criteria
d. Restrict exercise to walking only d. Characteristics that match other samples

43. After ileostomy, which of the following condition is 49. Which of the following sampling methods
NOT expected? allows the use of any group of research subject?

a. increased weight a. Purposive


b. Irritation of skin around the stoma b. Convenience
c. Liquid stool c. Snow-bail
d. Establishment of regular bowel movement d. Quota

44. The following are appropriate nursing 50. You decided to include 5 barangays in your
interventions during colostomy irrigation EXCEPT: municipality and chose a sampling method that would
get representative samples from each barangay. What
a. Increase the irrigating solution flow rate when should be the appropriate method for you to use in this
abdominal cramps is felt care?
b. Insert 2-4 inches of an adequately lubricated catheter
to the stoma a. Cluster sampling
c. Position client in semi-Fowler b. Random sampling
d. Hand the solution 18 inches above the stoma c. Stratifies sampling
d. Systematic sampling
45. What sensation is used as a gauge so that patients
with ileostomy can determine how often their pouch Situation 11 -After an abdominal surgery, the circulating
and scrub nurses have critical responsibility about confidence?
sponge and Instrument count.
a. Patient's advocate
51. When is the first sponge/instrument count b. Educator
reported? c. Patient's Liaison
d. Patient's arbiter
a. Before closing the subcutaneous layer
b. Before peritoneum is closed 57. As a nurse, you can help improve the effectiveness
c. Before dosing the skin of communication among healthcare givers
d. Before the fascia is sutured
a. Use of reminders of what to do
52. What major supportive layer of the abdominal b. Using standardized list of abbreviations, acronyms,
wall must be sutured with long tensile strength such and symbols
as cotton or nylon or silk suture? c. One-on-one oral endorsement
d. Text messaging and e-mail
a. Fascia
b. Muscle 58. As a nurse, your primary focus in the workplace
c. Peritoneum is the client's safety. However, personal safety is also
d. Skin a concern. You can communicate hazards to your co-
workers through the use of the following EXCEPT:
53. Like sutures, needles also vary in shape and uses. If
you are the scrub nurse for a patient who is prone to a. Formal training
keloid formation and has a low threshold of pain, b. Posters
what needle would you prepare? c. Posting IR in the bulletin board
d. Use of labels and signs
a. Round needle
b. A traumatic needle 59. As a nurse, what is one of the best way to reconcile
c. Reverse cutting needle medications across the continuum of care?
d. Tapered needle
a. Endorse on a case-to-case basis
54. Another alternative "suture" for skin closure is b. Communication a complete list of the patient's
the use of : medication to the next provider of service
c. Endorse in writing
a. Staple d. Endorse the routine and 'stat' medications every shift
b. Therapeutic glue
c. Absorbent dressing 60. As a nurse, you protect yourself and co-workers
d. invisible suture from misinformation and misrepresentations through
the following EXCEPT:
55. Like any nursing interventions, counts should be
documented. To whom does the scrub nurse report a. Provide information to clients about a variety of
any discrepancy of country so that immediate 'and services that can help alleviate the client's pain and
appropriate action in instituted? other conditions
b. Advising the client, by virtue of your expertise, that
a. Anesthesiologist which can contribute to the client's well-being
b. Surgeon c. Health education among clients and significant others
c. Or nurse supervisor regarding the use of chemical disinfectant
d. Circulating nurse d. Endorsement thru trimedia to advertise your favorite
disinfectant solution
Situation 12 - As a nurse, you should be aware and
prepared of the different roles you play. 61. A one-day postoperative abdominal surgery client
has been complaining of severe throbbing abdominal
56. What role do you play, when you hold all clients’ pain described as 9 in a 1-10 pain rating. Your
information entrusted to you in the strictest assessment reveals bowel sounds on all quadrants

21
21
and
21
the dressing is dry and intact. What nursing
intervention would you take? 67. As the nurse, you should anticipate to administer
which of the following medications to Zeny who is
a. Medicate client as prescribed diagnosed to be suffering from hypothyroidism?
b. Encourage client to do imagery
c. Encourage deep breathing and turning a. Levothyroxine
d. Call surgeon stat b. Lidocaine
c. Lipitor
62. Pentoxicodone 5 mg IV every 8 hours was d. Levophed
prescribed for post abdominal pain. Which will be your
priority nursing action? 68. Your appropriate nursing diagnosis for Zeny who is
suffering from hypothyroidism would probably include
a. Check abdominal dressing for possible swelling which of the following?
b. Explain the proper use of PCA to alleviate anxiety
c. Avoid overdosing to prevent dependence/tolerance a. Activity intolerance related to tiredness associated
d. Monitor VS, more importantly RR . with disorder
b. Risk to injury related to incomplete eyelid closure
63. The client complained of abdominal and pain. c. Imbalance nutrition related to hypermetabolism
Your nursing intervention that can alleviate pain is: d. Deficient fluid volume related to diarrhea

a. Instruct client to go to sleep and relax 69. Myxedema coma is a life threatening complication
b. Advice the client to close the lips and avoid deep of long standing and untreated hypothyroidism with
breathing and talking one of the following characteristics.
c. Offer hot and clear soup
d. Turn to sides frequently and avoid too much talking a. Hyperglycemia
b. hypothermia
64. Surgical pain might be minimized by which c. hyperthermia
nursing action in the OR: d. hypoglycemia

a. Skill of surgical team and lesser manipulation 70. As a nurse, you know that the most common
b. Appropriate preparation For the scheduled procedure type of goiter is related to a deficiency
c. Use of modem technology in closing the wound
d. Proper positioning and draping of clients a. thyroxine
b. thyrotropin
65. One very common cause of postoperative pain is: c. iron
d. iodine
a. Forceful traction during surgery
b. Prolonged surgery Situation 15 - Mrs. Pichay is admitted to your ward. The
c. Break in aseptic technique MD ordered "Prepared for thoracentesis this pm to
d. Inadequate anesthetic remove excess air from the pleural cavity."

Situation 14 - You were on duty at the medical ward 71. Which of the following nursing responsibility is
when Zeny came in for admission for tiredness, cold essential in Mrs. Pichay who will undergo
intolerance, constipation, and weight gain. Upon thoracentesis?
examination, the doctor's diagnosis was hypothyroidism.
a. Support, and reassure client during the procedure
66. Your independent nursing care for b. Ensure that informed consent has been signed
hypothyroidism includes: c. Determine if client has allergic reaction to local
anesthesia
a. administer sedative round the clock d. Ascertain if chest x-rays and other tests have been
b. administer thyroid hormone replacement prescribed and completed
c. providing a cool, quiet, and comfortable environment
d. encourage to drink 6-8 glasses of water 72. Mrs. Pichay who is for thoracentesis is assisted by
the nurse to any of the following positions, EXCEPT:
a. Gown
a. straddling a chair with arms and head resting on the b. Eyewear
back of the chair c. Face mask
b. lying on the unaffected side with the bed elevated 30- d. Gloves
40 degrees
c. lying prone with the head of the bed lowered 15-30 78. What will you do to ensure that Kyle, who is
degrees febrile, will have a liberal oral fluid intake?
d. sitting on the edge of the bed with her feet supported
and arms and head on a padded overhead table a. Provide a glass of fruit every meal
b. Regulate his IV to 30 drops per minute
73. During thoracentesis, which of the following nursing c. Provide a calibrated pitcher of drinking water and juice
intervention will be most crucial? at the bedside and monitor intake and output
d. Provide a writing pad to record his intake
a. Place patient in a quiet and cool room
b. Maintain strict aseptic technique 79. Before bedtime, you went to ensure Kyle's safety
c. Advice patient to sit perfectly still during needle in 'bed. You will do which of the following:
insertion until it has been withdrawn from the chest
d. Apply pressure over the puncture site as soon as the a. Put the lights on
needle is withdrawn b. Put the side rails up
c. Test the call system
74. To prevent leakage of fluid in the thoracic d. Lock the doors
cavity, how wilt you position the client after
thoracentesis? 80. Kyle's room is fully mechanized. What do you
teach the watcher and Kyle to alert the nurse for help?
a. Place flat in bed
b. Turn on the unaffected side a. How to lock side rails
c. Turn on the affected side b. Number of the telephone operator
d. On bed rest c. Call system
d. Remote control
75. Chest x-ray was ordered after thoracentesis. When
you client asks what is the reason for another chest x- Situation 17 - Tony, 11 years old, has 'kissing tonsils' and
ray, you will explain: is scheduled for tonsillectomy and adenoidectomy or T
and A.
a. to rule out pneumothorax
b. to rule out any possible perforation 81. You are the nurse of Tony who will undergo T and
c. to decongest A in the morning. His mother asked you if Tony will be
d. to rule out any foreign: body put to sleep. Your teaching will focus on:

Situation 16 - In the hospital, you are aware that we are a. spinal anesthesia
helped by the .use of a variety of equipment/devices to b. anesthesiologist’s preference
enhance quality patient care delivery; c. local anesthesia
d. general anesthesia
76. You are initiate an IV line to your patient, Kyle, 5,
who is febrile. What IV administration set will you 82. Mothers of children undergoing tonsillectomy
prepare? and adenoidectomy usually ask what food prepared
and give their children after surgery. You as the nurse
a. Blood transfusion set will say:
b. Macroset
c. Volumetric chamber a. balanced diet when fully awake
d. Microset b. hot soup when awake
c. ice cream when fully awake
77. Kyle is diagnosed to have measles. What will d. soft diet when fully awake
your protective personal attire include?
22
22

d. Bananas, cantaloupe, orange and other fresh fruits


83. The RR nurse should monitor for the most can be included in the diet
common postoperative complication of:
88. Rudy undergoes hemodialysis for the first time and
a. hemorrhage was scared of disequilibrium syndrome. He asked you
b. endotracheal tube perforation how this can be prevented. Your response is:
c. esopharyngeal edema
d. epiglottis a. maintain a conducive comfortable and cool
environment
84. The PACU nurse will maintain postoperative T and b. maintain fluid and electrolyte balance
A client in what position? c. initial hemodialysis shall be done for 30 minutes only
so as not to rapidly remove the waste from the blood
a. Supine with neck hyperextended and supported with than from the brain
pillow d. maintain aseptic technique throughout the
b. Prone with the head on pillow and tuned to the side hemodialysis
c. Semi-Fowler's with neck flexed
d. Reverse trendelenburg with extended neck 89. You are assisted by a nursing aide with the care of
the client with renal failure. Which delegated function
85. Tony is to be discharged in the afternoon of the to the aide would you particularly check?
same day after tonsillectomy and adenoidectomy. You
as the RN will make sure that the family knows to: a. Monitoring and recording I and O
b. Checking bowel movement
a. offer osteorized feeding c. Obtaining vital signs
b. offer soft foods for a week to minimize discomfort d. Monitoring diet
while swallowing
c. supplement his diet with vitamin C rich juices to 90. A renal failure patient was ordered for creatinine
enhance heating clearance. As the nurse you will collect
d. offer clear liquid for 3 days to prevent irritation
a. 48 jour urine specimen
Situation 18 - Rudy was diagnosed to have chronic renal b. first morning urine
failure. Hemodialysis is ordered that an A-V shunt was c. 24 hour urine specimen
surgically created. d. random urine specimen

86. Which of the following action would be of Situation 19 - Fe is experiencing left sharp pain and
highest priority with regards to the external shunt? occasional hematuria. She was advised to undergo IVP
by her physician.
a. Avoid taking BP or blood sample from the arm with
shunt 91. Fe was so anxious about the procedure and
b. Instruct the client not to exercise the arm with the particularly expressed her low pain threshold. Nursing
shunt health instruction will include:
c. Heparinize the shunt daily
d. Change dressing of the shunt daily a. assure the client that the pain is associated with the
warm sensation during the administration of the
87. Diet therapy for Rudy, who has acute renal Hypaque by IV
failure, is tow-protein, low potassium and sodium. b. assure the client that the procedure painless
The nutrition instruction should include: c. assure the client that contrast medium will be
given orally
a. Recommend protein of high biologic value like eggs, d. assure the client that x-ray procedure like IVP is only
poultry and lean meat done by experts
b. Encourage client to include raw cucumbers, carrot,
cabbage, and tomatoes 92. What will the nurse monitor and instruct the client
c. Allowing the client cheese, canned foods, and other and significant others, post IVP?
processed food
a. Report signs and symptoms for delayed allergic
reactions a. Administer by fast drip
b. Observe NPO for 6 hours b. Inject the drugs as close to the IV injection site
c. Increase fluid intake c. Incorporate to the IV solution
d. Monitor intake and output d. Use volumetric chamber

93. Post IVP, Fe should excrete the contrast medium. 98. One patient has a 'runaway' IV of 50% dextrose.
You instructed the family to include more vegetables in To prevent temporary excess of insulin transient
the diet and hyperinsulin reaction, what solution should you
prepare in anticipation of the doctors order?
a. increase fluid intake
b. barium enema a. Any IV solution available to KVO
c. cleansing enema b. Isotonic solution
d. gastric lavage c. Hypertonic solution
d. Hypotonic solution
94. The IVP reveals that Fe has small renal calculus
that can be passed out spontaneously. To increase the 99. How can nurse prevent drug interaction including
chance of passing the stones, you instructed her to absorption?
force fluids and do which of the following?
a. Always flush with NSS after IV administration
a. Balanced diet b. Administering drugs with more diluents
b. Ambulance more c. Improving on preparation techniques
c. Strain all urine d. Referring to manufacturer's guidelines
d. Bed rest
100. In insulin administration, it should be understood
95. The presence of calculi in the urinary tract is called: that our body normally releases insulin according to
our blood glucose level. When is insulin and glucose
a. Colelithiasis level highest?
b. Nephrolithiasis
c. Ureterolithiasis a. After excitement
d. Urolithiasis b. After a good night's rest
c. After an exercise
Situation 20 - At the medical-surgical ward, the nurse d. After ingestion of food
must also be concerned about drug interactions.

96. You have a client with TPN. You know that in TPN, CARE OF CLIENTS WITH PHYSIOLOGIC AND
like blood transfusion, there should be no drug PSYCHOSOCIAL ALTERATIONS
incorporation. However, the MD's order read;
incorporate insulin to present TPN. Will you follow Situation 1 - Because of the serious consequences of
the order? severe burns management requires a multi disciplinary
approach. You have important responsibilities as a
a. No, because insulin will induce hyperglycemia in nurse.
patients with TPN
b. Yes, because insulin is chemically stable with TPN and 1. While Sergio was lighting a barbecue grill with a
can enhance blood glucose level lighter fluid, his shirt burst into flames. The most
c. No, because insulin is not compatible with TPN effective way to extinguish the flames with as little
d. Yes, because it was ordered by the MD further damage as possible is to:

97. The RN should also know that some drugs have a. log roll on the grass/ground
increased absorption when infused in PVC b. slap the flames with his hands
container. How will you administer drugs such as c. remove the burning clothes
insulin, nitroglycerine hydralazine to promote better d. pour cold liquid over the flames
therapeutic drug effects?

22
22
2. Once the flames are extinguished, it is b. Call security officer and report the incident
most important to: c. Call your nurse supervisor and report the incident :
d. Call the physician on duty
a. cover Sergio with a warm blanket
b. give him sips of water 7. You are on morning duty in the medical ward. You
c. calculate the extent of his burns have 10 patients assigned to you. During your
d. assess the Sergio's breathing endorsement rounds, you found out that one of your
patients was not in bed. The patient next to him
3. Sergio is brought to the Emergency Room after the informed you that he went home without notifying
barbecue grill accident. Based on the assessment of the nurses. Which among the following will you do
the physician, Sergio sustained superficial partial first?
thickness bums on his trunk, right upper extremities ad
right lower extremities. His wife asks what that means. a. Make and incident report
Your most accurate response would be: b. Call security to report the incident
c. Wait for 2 hours before reporting
a. Structures beneath the skin are damaged d. Report the incident to your supervisor
b. Dermis is partially damaged
c. Epidermis and dermis are both damaged 8. You are on duty in the medical ward. You were
d. Epidermis is damaged asked to check the narcotics cabinet. You found out
that what is on record does not tally with the drugs
4. During the first 24 hours after thermal injury, you used. Which among the following will you do first?
should assess Sergio for
a. Write an incident report and refer the matter to the
a. hypokalemia and hypernatremia nursing director
b. hypokalemia and hyponatremia b. Keep your findings to yourself
c. hyperkalemia and hyponatremia c. Report the matter to your supervisor
d. hyperkalemia and hypernatremia d. Find out from the endorsement any patient who
might have been given narcotics
5. Teddy, who sustained deep partial thickness and full
thickness burns of the face, whole anterior chest and 9. You are on duty in the medical ward. The mother
both upper extremities two days ago, begins to exhibit of your patient who is also a nurse came running to
extreme restlessness. You recognize that this most the nurse station and informed you that Fiolo went
likely indicates that Teddy is developing: into cardiopulmonary arrest. Which among the
following will you do first?
a. Cerebral hypoxia
b. Hypervolemia a. Start basic life support measures
c. Metabolic acidosis b. Call for the Code
d. Renal failure . c. Bring the crush cart to the room
d. Go to see Fiolo and assess for airway patency and
Situation 2 - You are now working as a staff nurse in a breathing problems
general hospital. You have to be prepared to handle
situations with ethico-legal and moral implications. 10. You are admitting Jorge to the ward and you found
out that he is positive for HIV. Which among the
6. You are on night duty in the surgical ward. One of following will you do first?
our patients Martin is prisoner who sustained an
abdominal gunshot wound. He is being guarded by a. Take note of it and plan to endorse this to next shift
policemen from the local police unit. During your b. Keep this matter to your self
rounds you heard a commotion. You saw the policeman c. Write an incident report
trying to hit Martin. You asked why he was trying to d. Report the matter to your head nurse
hurt Martin. He denied the matter. Which among the
following activities will you do first? Situation 3 - Colorectal cancer can affect old and
younger people. Surgical procedures and other modes of
a. Write an incident report treatment are done to ensure quality of life. You are
assigned in the Cancer institute to care of patients with
this type of cancer. b. Fermin can lie on the side comfortably, about the
3rd postoperative day
11. Larry, 55 years old, who is suspected of having c. The abdominal incision is close and contamination is
colorectal cancer, is admitted to the CI. After taking the no longer a danger
history and vital signs the physician does which test as d. The stool starts to become formed, around the 7th
a screening test for colorectal cancer. postoperative day

a. Barium enema 17. When preparing to teach Fermin how to irrigate


b. Carcinoembryonig antigen his colostomy, you should plan to do the procedure:
c. Annual digital rectal examination
d. Proctosigmoidoscopy a. When Fermin would have normal bowel movement
b. At least 2 hours before visiting hours
12. To confirm his impression of colorectal cancer, c. Prior to breakfast and morning care
Larry will require which diagnostic study? d. After Fermin accepts alteration in body image

a. carcinoembryonic antigen 18. When observing a rectum demonstration of


b. proctosigmoidbscopy colostomy irrigation, you know that more teaching is
c. stool hematologic test required if Fermin:
d. abdominal computed tomography (CT) test
a. Lubricates the tip of the catheter prior to inserting
13. The following are risk factors for colorectal into the stoma
cancer, EXCEPT: b. Hands the irrigating bag on the bathroom door doth
hook during fluid insertion
a. inflammatory bowels c. Discontinues the insertion of fluid after only 500 ml of
b. high fat, high fiver diet fluid had been insertion
c. smoking d. Clamps off the flow of fluid when feeling
d. genetic factors-familial adenomatous polyposis uncomfortable

14. Symptoms associated with cancer of the colon 19. You are aware that teaching about colostomy
include: care is understood when Fermin states, "I will contact
my physician and report:
a. constipation, ascites and mucus in the stool
b. diarrhea, heartburn and eructation a. If I have any difficulty inserting the irrigating tub into
c. blood in the stools, anemia, and pencil-shaped, stools the stoma.”
d. anorexia, hematemesis, and increased peristalsis b. If I notice a loss of sensation to touch in the stoma
tissue."
15. Several days prior to bowel surgery, Larry may be c. The expulsion of flatus while the irrigating fluid is
given sulfasuxidine and neomycin primarily to: running out."
d. When mucus is passed from the stoma between
a. promote rest of the bowel by minimizing peristalsis irrigation."
b. reduce the bacterial content of the colon
c. empty the bowel of solid waste 20. You would know after teaching. Fermin that dietary
d. soften the stool by retaining water in the colon instruction for him is effective when he states, "It is
important that I eat:
Situation 4 - ENTEROSTOMAL THERAPY is now
considered especially in nursing. You are participating in a. Soft foods that are easily digested and absorbed by my
the OSTOMY CARE CLASS. large intestine."
b. Bland food so that my intestines do not become
16. You plan to teach Fermin how to irrigate the irritate."
colostomy when: c. Food low in fiber so that there is less stool."
d. Everything that I ate before the operation, while
a. The perineal wound heals and Fermin can sit avoiding foods that cause gas."
comfortably on the commode

22
22
Situation 5 - Ensuring safety is one of your most are favorable. Infection control is one important
important responsibilities. You will need to provide responsibility of the nurse to ensure quality of care.
instructions and information to your clients to prevent
complications. 26. Honrad, who has been complaining of anorexia
and feeling tired, develops jaundice. After a workup he
21. Randy has chest tubes attached to a pleural is diagnosed of having Hepatitis A. His wife asks you
drainage system. When caring for him you should: about gamma globulin for herself and her household
help. Your most appropriate response would be:
a. empty the drainage system at the end of the shift
b. clamp the chest tube when auctioning a. "Don't worry your husband's type of hepatitis is no
c. palpate the surrounding areas for crepitus longer communicable"
d. change the dressing daily using aseptic techniques b. "Gamma globulin provides passive immunity for
Hepatitis B"
22. Fanny came in from PACU after pelvic surgery. As c. "You should contact your physician immediately
Fanny's nurse you know that the sign that would be about getting gamma globulin."
indicative of a developing thrombophlebitis would be: d. "A vaccine has been developed for this type of
hepatitis"
a. a tender, painful area on the leg
b. a pitting edema of the ankle 27. Voltaire develops a nosocomial respiratory tract
c. a reddened area at the ankle infection. He asks you what that means.
d. pruritus on the calf and ankle
a. "You acquired the infection after you have been
23. To prevent recurrent attacks on Terry who has admitted to the hospital."
acute glumerulonephritis, you should instruct her to: b. "This is a highly contagious infection requiring
complete isolation."
a. seek early treatment for respiratory infections c. "The infection you had prior to hospitalization flared
b. take showers instead of tub bath up."
c. continue to take the same restrictions on fluid intake d. "As a result of medical treatment, you have acquired a
d. avoid situations that involve physical activity secondary infection.''

24. Herbert has a laryngectomy and he is now for 28. As a nurse you know that one of the
discharge. Re verbalized his concern regarding his complications that you have to watch out for when
laryngectomy tube being dislodged. What should you caring for Omar who is receiving total parenteral
teach him first? nutrition is:

a. Recognize that prompt closure of the tracheal a. stomatitis


opening may occur b. hepatitis
b. Keep calm because there is no immediate emergency c. dysrhythmia
c. Reinsert another tubing immediately d. infection
d. Notify the physician at once
29. A solution used to treat Pseudomonas would
25. When caring for Larry after an exploratory chest infection is:
surgery and pneumonectomy, your priority would be to
maintain: a. Dakin's solution
b. Half-strength hydrogen peroxide
a. supplementary oxygen b. Acetic acid
b. ventilation exchange d. Betadine
c. chest tube drainage
d. blood replacement 30. Which of the following is most reliable in
diagnosing a wound infection?
Situation 6 - Infection can cause debilitating
consequences when host resistance is compromised and a. Culture and sensitivity
virulence of microorganisms and environmental factors b. Purulent drainage from a wound
c. WBC count of 20,000/pL
d. Gram stain testing c. is permanently paralyzed
d. has received a significant brain injury
Situation 7 - As a nurse you need to anticipate the
occurrence of complications of stroke so that life Situation 8 - With the improvement in life expectancies
threatening situations can be prevented. and the emphasis in the quality of life it is important to
provide quality care to our older patients. There are
31. Wendy is admitted to the hospital with signs and frequently encountered situations and issues relevant to
symptoms of stroke. Her Glasgow Coma Scale is 6 on the older, patients.
admission. A central venous catheter was inserted
and an I.V. infusion was started. As a nurse assigned 36. Hypoxia may occur in the older patients because
to Wendy what will he your priority goal? of which of the following physiologic changer
associated with aging.
a. Prevent skin breakdown
b. Preserve muscle function a Ineffective airway clearance
c. Promote urinary elimination b. Decreased alveolar surface area
d. Maintain a patent airway c. Decreased anterior-posterior chest diameter
d. Hyperventilation
32. Knowing that for a comatose patient hearing is
the best last sense to be lost, as Judy's nurse, what 37. The older patient is at higher risk for
should you do? in inconvenience because of:

a. Tell her family that probably she can't hear them a. dilated urethra
b. Talk loudly so that Wendy can hear you b. increased glomerular filtration rate
c. Tell her family who are in the room not to talk c. diuretic use
d. Speak softly then hold her hands gently d. decreased bladder capacity

33. Which among the following interventions should 38. Merle, age 86, is complaining of dizziness when
you consider as the highest priority when caring for she stands up. This may indicate:
June who has hemiparersis secondary to stroke?
a. dementia
a. Place June on an upright lateral position b. a visual problem
b. Perform range of motion exercises c. functional decline
c. Apply antiembolic stocking d. drug toxicity
d. Use hand rolls or pillows for support
39. Cardiac ischemia in an older patient
34. Ivy, age 40, was admitted to the hospital with a usually produces:
severe headache, stiff neck and photophobia. She was
diagnosed with a subarachnoid hemorrhage secondary a. ST-T wave changes
to ruptured aneurysm. While waiting for surgery, you b. Very high creatinine kinase level
can provide a therapeutic by doing which of the c. chest pain radiating to the left arm
following? d. acute confusion

a. honoring her request for a television 40. The most dependable sign of infection in the
b. placing her bed near the window older patient is:
c. dimming the light in her room
d. allowing the family unrestricted visiting privileges a. change in mental status pain
b. fever
35. When performing a neurological assessment on c. pain
Walter, you find that his pupils are fixed and dilated. d. decreased breath sound with crackles
This indicated that he:
Situation 9 - A "disaster" is a large-scale emergency—
a. probably has meningitis even a small emergency left unmanaged may turn into a
b. is going to be blind because of trauma disaster. Disaster preparedness is crucial and is

22
22
everybody's business. There are agencies that are in d. Urgent
charge of ensuring prompt response. Comprehensive
Emergency Management (CEM) is an integrated 45. Which of the following terms refer to a process
approach to the management of emergency program by which the individual receives education about
and activities for all four emergency phases (mitigation, recognition of stress reactions and management
preparedness, response, and recovery), for all type of strategies for handling stress which may be instituted
emergencies and disasters (natural, man-made, and after a disaster?
attack) and for all levels of government and the private
sector. a. Critical incident stress management
b. Follow-up
41. Which of the four phases of emergency c. Defriefing
management is defined as "sustained action that d. Defusion
reduces or eliminates long-term risk to people and
properly from natural hazards and the effect"? Situation 10 - As a member of the health and nursing
team you have a crucial role to play in ensuring that all
a. Recovery the members participate actively is the various tasks
b. Mitigation agreed upon,
c. Response
d. Preparedness 46. While eating his meal, Matthew accidentally
dislodges his IV line and bleeds. Blood oozes on the
42. You are a community health nurse collaborating surface of the over-bed table. It is most appropriate
with the Red Cross and working with disaster relief that you instruct the housekeeper to clean the table
following a typhoon which flooded and devastated the with:
whole province. Finding safe housing for survivors,
organizing support for the family, organizing counseling a. Acetone
debriefing sessions and securing physical care are the b. Alcohol
services you are involved with. To which type of c. Ammonia
prevention are these activities included. d. Bleach

a. Tertiary prevention 47. You are a member of the infection control team, of
b. Primary prevention the hospital. Based on a feedback during the meeting
c. Aggregate care prevention of the committee there is an increased incidence of
d. Secondary prevention pseudomonas infection in the Burn Unit (3 out of 10
patients had positive blood and wound culture). What
43. During the disaster you see a victim with a green is your priority activity?
tag, you know that the person:
a. Establish policies for surveillance and monitoring
a. has injuries that are significant and require medical b. Do data gathering about the possible sources of
care but can wait hours will threat to life or limb infection (observation, chart review, interview)
b. has injuries that are life threatening but survival is c. Assign point persons who can implement policies
good with minimal intervention d. Meet with the nursing group working in the burn unit
c. indicates injuries that are extensive and chances of and discuss problem with them feel
survival are unlikely even with definitive care
d. has injuries that are minor and treatment can be 48. Part of your responsibility as a member of the
delayed from hours to days diabetes core group is to get referrals from the
various wards regarding diabetic patients needing
44. The term given to a category of triage that refers to diabetes education. Prior to discharge today 4
life threatening or potentially life threatening injury or patients are referred to you. How would you start
illness requiring immediate treatment: prioritizing your activities?

a. Immediate a. Bring your diabetes teaching kit and start your session
b. Emergent taking into consideration their distance from your office
c. Non-acute b. Contact the nurse-in-charge and find out from her the
reason for the referral help her by:
c. Determine their learning needs then prioritize
d. involve the whole family in the teaching class a. Coming back periodically and indicating your
availability if she would like you to sit with her
49. You have been designated as a member of the b. Insisting that Ruby should talk with you because it is
task force to plan activities for the Cancer not good to Keep everything inside
Consciousness Week. Your committee has 4 months to c. Leaving her atone because she is uncooperative and
plan and implement the plan. You are assigned to unpleasant to be with
contact the various cancer support groups in your d. Encouraging her to be physically active as possible
hospital. What will be your priority activity?
53. Leo who is terminally ill and recognizes that he is
a. Find out if there is a budget for this activity in the process of losing, everything and everybody he
b. Clarify objectives of the activity with the task force loves, is depressed. Which of the following would best
before contacting the support groups help him during depression?
c. Determine the VIPs and Celebrities who will be invited
d. Find out how many support groups there are in the a. Arrange for visitors who might cheer him
hospital and get the contact number of their president b. Sit down and talk with him for a while
c. Encourage him to look at the brighter side of things
50. You are invited to participate in the medical mission d. Sit silently with him
activity of your alumni association. In the planning
stage everybody is expected to identify what they can 54. Which of the following statements would best
do during the medical mission and what resources are indicate that Ruffy; who is dying has accepted this
needed. You though it is also your chance to share impending death?
what you can do for others. What will be your most
important role where you can demonstrate the impact a. "I'm ready to do."
of nursing health? b. "I have resigned myself to dying"
c. "What's the use"?
a. Conduct health education on healthy lifestyle d: "I'm giving up"
b. Be a triage nurse
c. Take the initial history and document findings 55. Maria, 90 years old has planned ahead for her-
d. Act as a coordinator death-philosophically, socially, financially and
emotionally. This is recognized as:
Situation 11 - One of the realities that we are confronted
with is'6w mortality. It is important for us nurses to be a. Acceptance that death is inevitable
aware of how we view suffering, pain, illness, and even b Avoidance of the true sedation
our death as well as its meaning. That way we can help c. Denial with planning for continued life
our patients cope with death and dying. d. Awareness that death will soon occur

51. Irma is terminally ill she speaks to you in Situation 12 - Brain tumor, whether malignant or benign,
confidence. You now feel that Irma's family could be has serious management implications nurse, you should
helpful if they knew what Irma has told you. What be able to understand the consequences of the disease
should you do first? and the treatment.

a. Tell the physician who in turn could tell the family 56. You are caring for Conrad who has a brain tumor
b. Obtain Irma's permission to share the information and increased intracranial Pressure (ICP). Which
with the family intervention should you include in your plan to
c. Tell Irma that she has to tell her family what she told reduce ICP?
you
d. Make an appointment to discuss the situation with a. Administer bowel! Softener
the family b. Position Conrad with his head turned toward the side
of the tumor
52. Ruby who has been told she has terminal cancer c. Provide sensory stimulation
turns away aha refuses to respond to you. You can d. Encourage coughing and deep breathing
best
22
22

c. Aris, who is newly admitted and is scheduled for an


57. Keeping Conrad's head and neck in executive check-up
alignment results in: d. Claire, who has cholelithiasis and is for operation on
call
a. increased intrathoracic pressure
b. increased venous outflow 63. Brenda, the Nursing Supervisor of the intensive
c. decreased venous outflow care unit (ICU) is not on duty when a staff nurse
d. increased intra abdominal pressure committed a serious medication error. Which
statement accurately reflects the accountability of the
58. Which of the following activities may increase nursing supervisor?
intracranial pressure (ICP)?
a. Brenda should be informed when she goes back on
a. Raising the head of the bed duty
b. Manual hyperventilation b. Although Brenda is not on duty, the nursing supervisor
c. Use of osmotic Diuretics on duty decides to call her if time permits
d. Valsava's maneuver c. The nursing supervisor on duty will notify Brenda at
home
59. After you assessed Conrad, you suspected increased d. Brenda is not duty therefore it is not necessary to
ICP! Your most appropriate respiratory goal is to: inform her

a. maintain partial pressure of arterial 02 (PaO2) above 64. Which barrier should you avoid, to manage your
80 mmHg time wisely?
b. lower arterial pH
c. prevent respiratory alkalosis a. Practical planning
d. promote CO2 elimination b. Procrastination
c. Setting limits
60. Conrad underwent craniotomy. As his nurse; you d. Realistic personal expectation
know that drainage on a craniotomy dressing must
be measured and marked. Which findings should you 65. You are caring for Vincent who has just been
report immediately to the surgeon? transferred to the private room. He is anxious
because he fears he won't be monitored as closely as
a. Foul-smelling drainage he was in the Coronary Care Unit. How can you allay
b. yellowish drainage his fear?
c. Greenish drainage
d. Bloody drainage a. Move his bed to a room far from nurse's station to
reduce
Situation 13 -As a Nurse, you have specific b. Assign the same nurse to him when possible
responsibilities as professional. You have to demonstrate c. Allow Vincent uninterrupted period of time
specific competencies. d. Limit Vincent's visitors to coincide with CCU policies

61. The essential components of professional Situation 14 - As a nurse in the Oncology Unit, you have
nursing practice are all the following EXCEPT: to be prepared to provide efficient and effective care to
your patients.
a. Culture
b. Care 66. Which one of the following nursing interventions
c. Cure would be most helpful in preparing the patient for
d. Coordination radiation therapy?

62. You are assigned to care for four (4) patients. a. Offer tranquilizers and antiemetics
Which of the following patients should you give first b. Instruct the patient of the possibility of radiation burn
priority? c. Emphasis on the therapeutic value of the treatment
d. Map out the precise course of treatment
a. Grace, who is terminally ill with breast cancer
b. Emy, who was previously lucid but is now unarousable 67. What side effects are most apt to occur to patient
during radiation therapy to the pelvis?

23
possible
a. Urinary retention b. make the most efficient use of available resources
b. Abnormal vaginal or perineal discharge
c. Paresthesia of the lower extremities
d. Nausea and vomiting and diarrhea

68. Which of the following can be used on the


irradiated skin during a course of radiation
therapy?

a. Adhesive tape
b. Mineral oil
c. Talcum powder
d. Zinc oxide ointment

69. Earliest sign of skin reaction to radiation therapy is:

a. desquamation
b. erythema
c. atrophy
d. pigmentation

70. What is the purpose of wearing a film badge


while caring for the patient who is radioactive?

a. Identify the nurse who is assigned to care for such a


patient
b. Prevent radiation-induced sterility
c. Protect the nurse from radiation effects
d. Measure the amount of exposure to radiation

Situation 15 - In a disaster there must be a chain of


command in place that defines the roles of each
member of the response team. Within the health care
group there are pre-assigned roles based on education,
experience and training on disaster.

71. As a nurse to which of the following groups are


you best prepared to join?

a. Treatment group
b. Triage group
c. Morgue management
d. Transport group

72. There are important principles that should guide


the triage team in disaster management that you have
to know if you were to volunteer as part of the triage
team. The following principles should be observed in
disaster triage, EXCEPT:

a. any disaster plan should have resource available to


triage at each facility and at the disaster site if

23
23
c. training on disaster is not important to the response
in the event of a real disaster because each disaster is
unique in itself
d. do the greatest good for the greatest number
of casualties

73. Which of the following categories of


conditions should be considered first priority in
a disaster?

a. Intracranial pressure and mental status


b. Lower gastrointestinal problems
c. Respiratory infection
d. Trauma

74. A guideline that is utilized in determining


priorities is to assess the status of the following,
EXCEPT?

a. perfusion
b. locomotion
c. respiration
d. mentation

75. The most important component of


neurologic assessment is:

a. pupil reactivity
b. vital sign assessment
c. cranial nerve assessment
d. level of consciousness/responsiveness

Situation 16 - You are going to participate in a Cancer


Consciousness Week. You are assigned to take charge
of the women to make them aware of cervical cancer.
You reviewed its manifestations and management.

76. The following are risk factors for cervical


Cancer EXCEPT:

a. immunisuppressive therapy
b. sex at an early age, multiple partners, exposure
to socially transmitted diseases, male partner's
sexual habits
c. viral agents like the Human Papilloma Virus
d. smoking

77. Late signs and symptoms of cervical cancer


include the following EXCEPT:

a. urinary/bowel changes
b. pain in pelvis, leg of flank
c. uterine bleeding
d. lymph edema of lower extremities
23
78. When a panhysterectomy is performed due to
cancer of the cervix, which of the following organs a. Place pillows under your patient's shoulders
are moved? b. Raise the knee-gatch to 30 degrees
c. Keep your patient in a high-fowler's position
a. the uterus, cervix, and one ovary d. Support the patient's head and neck with pillows and
b. the uterus, cervix, and two-thirds of the vagina sandbags
c. the uterus, cervix, tubes and ovaries
d. the uterus and cervix 84. If there is an accidental injury to the parathyroid
gland during a thyroidectomy which of the following
79. The primary modalities of treatment for Stage 1 might Leda develops postoperative?
and IIA cervical cancer include the following:
a. Cardiac arrest
a. surgery, radiation therapy and hormone therapy b. Dyspnea
b. surgery c. Respiratory failure
c. radiation therapy d. Tetany
d. surgery and radiation therapy
85. After surgery Leda develops peripheral numbness,
80. A common complication of hysterectomy is: tingling and muscle twitching and spasm. What
would you anticipate to administer?
a. thrombophlebitis of the pelvic and thigh vessels
b. diarrhea due to over stimulating a. Magnesium sulfate
c. atelectasis b. Calcium gluconate
d. wound dehiscence c. Potassium iodine
d. Potassium chloride
Situation 17 - The body has regulatory mechanism to
maintain the needed electrolytes. However there are Situation 18 - NURSES are involved in maintaining a safe
conditions/surgical interventions that could compromise and health environment. This is part of quality care
life. You have to understand how management of these management.
conditions are done.
86. The first step in decontamination is:
81. You are caring for Leda who is scheduled to
undergo total thyroidectomy because of a diagnosis of a. to immediately apply a chemical decontamination
thyroid cancer. Prior to total thyroidectomy, you foam to the area of contamination
should instruct Leda to: b. a thorough soap and water was and rinse of the
patient
a. Perform range and motion exercises on the head and c. to immediately apply personal protective equipment
neck d. removal of the patients clothing and jewelry and then
b. Apply gentle pressure against the incision when rinsing the patient with water
swallowing
c. Cough and deep breath every 2 hours 87. For a patient experiencing pruritus, you
d. Support head with the hands when changing position recommend which type of bath:

82. As Leda's nurse, you plan to set up an emergency a. Water


equipment at her beside following thyroidectomy. b. colloidal (oatmeal)
You should include: c. saline
d. sodium bicarbonate
a An airway and rebreathing tube
b. A tracheostomy set and oxygen 88. Induction of vomiting is indicated for the accidental
c. A crush cart .with bed board poisoning patient who has ingested.
d. Two ampules of sodium bicarbonate
a. rust remover
83. Which of the following nursing interventions b. gasoline
is appropriate after a total thyroidectomy? c. toilet bowl cleaner
d. aspirin 93. You would like to compare the support, system of
patients with chronic illness to those with acute illness.
89. Which of the following term most precisely refer What type of research it this?
to an infection acquired in the hospital that was not
present or incubating at the time of hospital a. Correlational
admission? b. Descriptive
c. Experimental
a. Secondary bloodstream infection d. Quasi-experimental
b. Nosocomial infection
c. Emerging infectious disease 94. You are shown a Likert Scale that will be used in
d. Primary bloodstream infection evaluating your performance in the clinical area. Which
of the following questions will you not use in critiquing
90. Which of the following guidelines is not the Likert Scale?
appropriate to helping family members cope with
sudden death? a. Are the techniques to complete and score the scale
provided?
a. Obtain orders for sedation of family members b. Are the reliability and validity information on the scale
b. Provide details of the factors attendant to the sudden described?
death c. If the Likert Scale is to be used for a study, was
c. Show acceptance of the body by touching it and giving the development process described?
the family permission to touch d. Is the instrument clearly described?
d. Inform the family that the patient has passed on
95. In any research study where individual persons are
Situation 19 - As a nurse you are expected to participate involves, it is important that an informed consent for
in initiating or participating in the conduct of research the Study is obtained. The following are essential
studies to improve nursing practice. You have to be information about the consent that you should disclose
updated on the latest trends and issues affecting to the prospective subjects EXCEPT:
profession and the best practices arrived at by the
profession a. Consent to incomplete disclosure
b. Description of benefits, risks and discomforts
91. You are interested to study the effects of c. Explanation of procedure
meditation and relaxation on the pain experienced by d. Assurance of anonymity and confidentiality,
cancer patients. What type of variable is pain?
Situation 20 - Because severe burn can affect the
a. Dependant person's totality it is important that you apply
b. Correlational interventions focusing on the various dimensions of
c. Independent man. You also have to understand the rationale of the
d. Demographic treatment.

92. You would like to compare the support system of 96. What type of debribement involves proteolytic
patients with chronic illness to those with acute enzymes?
illness. How will you best state your problem?
a. Interventional
a. A descriptive study to compare the support system of b. Mechanical
patients with chronic illness and those with acute illness c. Surgical
in terms of demographic data and knowledge about d Chemical
interventions
b. The effect of the Type of Support system of patients 97. Which topical antimicrobial is most frequently used
with chronic illness and those with acute illness in burn wound care?
c. A comparative analysis of the support: system of
patients with chronic illness and those with acute illness a. Neosporin
d. A study to compare the support system of patients b. Silver nitrate
with chronic illness and those with acute illness c. Silver sulfadiazine

23
23
d. Sulfamylon

98. Hypertrophic burns scars are caused by:

a. exaggerated contraction
b. random layering of collagen
c. wound ischemia
d. delayed epithelialization

99. The major disadvantage of whirlpool cleansing


of burn wounds is:

a. patient hypothermia
b. cross contamination of wound
c. patient discomfort
d. excessive manpower requirement

100. Oral analgecis are most frequently used to


control burn injury pain:

a. upon patient request


b. during the emergent phase
c. after hospital discharge
d. during the cute phase
ANSWER KEY: CARE OF CLIENTS WITH PHYSIOLOGIC 51. C
AND PSYCHOSOCIAL ALTERATIONS 52. A
53. D
1. A 54. A
2. D 55. D
3. D 56. A
4. B 57. B
5. D 58. B
6. A 59. D
7. B 60. A
8. A 61. A
9. D 62. B
10. A 63. A
11. B 64. B
12. B 65. B
13. B 66. C
14. C 67. A
15. B 68. D
16. C 69. B
17. C 70. C
18. C 71. B
19. A 72. C
20. C 73. D
21. C 74. B
22. A 75. D
23. A 76. A
24. D 77. B
25. A 78. C
26. D 79. D
27. A 80. A
28 D 81. C
29. C 82. B
30. D 83. C
31. D 84. D
32. D 85. B
33. B 86. C
34. C 87. B
35. D 88. D
36. B 89. B
37. D 90. A
38. B 91. A
39. C 92. C
40. C 93. A
41. B 94. A
42. C 95. A
43. D 96. D
44. D 97. B
45. A 98. A
46. D 99. A
47. A 100. C
48. C
49. B
50. A

23
23
Nursing Practice Test V 6. One of the reasons that persons with antisocial

Situation: The nurse is interviewing a handsome man. He


is intelligent and very charming. When asked about his
family, he states he has been married four times. He says
three of those marriages were "shotgun" weddings. He
states he never really loved any of his wives. He doesn't
know much about his three children. "I've lost track," he
states.

1. If a patient is very resistant in taking responsibility


of his action and asks, "Can you just give me some
medication?" the best response is:

a. "The medication has too many side effects."


b. You don't want to take medication, do you?"
c. Medication is given only as a East resort."
d. "There is no medication specific for your condition."

2. The patient asks the nurse, "What is this therapy


for anyway. I just don't understand it." the best reply
is:

a. "It keeps you from being put on medications."


b. "It helps you to change others in the family."
c. "The purpose of therapy is to help you change."
d. "No one but professionals can really understand

3. For patient in group therapy, the goal is:

a. Exchanging information and ideas


b. Developing insight by relating to others
c. Learning that everyone has problems
d. All of the above

4. In planning care for the patient with a personality


disorder, the nurse realizes that this patient will
most likely:

a. Not need long-term therapy


b. Not require medication
c. Require anti-anxiety medication
d. Resist any change in behavior

5. The person with an antisocial personality is


participating in therapy while a patient at a
psychiatric hospital. The nurse’s expectations are that
he will:

a. Make a complete recovery


b. Make significant changes
c. Begin the slow process of change
d. Make few changes, if any
personalities may marry repeatedly or get into trouble
with legal authorities is:

a. They usually just don't care


b. They are borderline mentally retarded
c. They are too psychotic to see what’s going on
d. They do not learn from past mistakes

7. The nurse recognizes that these are traits of:

a. Bipolar disorder
b. Alcoholic personality
c. Antisocial personality
d. Borderline personality

Situation: The patient with bipolar disorder is


pacing continuously and is skipping meals.

8. Blood levels are drawn on the patient who has


been taking Lithium for about six months. The
present level is 2.1 meq/L. The nurse evaluates this
level as:

a. Therapeutic
b. Below therapeutic
c. Potentially dangerous
d. Fatally toxic

9. The priority in working with patient a


thought disorder is:

a. Get him to understand what you're saying


b. Get him to do his ADLs
c. Reorient him to reality
d. Administer antipsychotic medications

10. The most recent Lithium level on bipolar patient


indicates a drop non-therapeutic level. What
associated behavior does the nurse assess?

a. Ataxia
b. Confusion
c. Hyperactivity
d. Lethargy

11. Adequate fluid intake for a patient on Lithium is:

a. 1,000 ml per day


b. 1,500 ml per day
c. 2,000 ml per day
d. 3,600 ml per day

12. The physician orders Lithium carbonate for


the bipolar patient. The nurse is aware that:

23
a. The patient should be put on a special diet 18. The composite picture of rape victim reveals that
b. The medication should be given only at night most victimized women are:
c. A salt-free should be provided for the patient
d. The drug level should be monitored regularly

13. The nursing plan should emphasize:

a. Offering him finger foods


b. Telling him he must sit down and eat
c. Serving food in his room and staying with him
d. Telling him to order fast food of he wants to eat

Situation: Anna, 25 years old was raped six months ago


states, "I just can't seem to get over this. My husband
and I don't even have sex anymore. What can I do?"

14. Supportive therapy to the rape victim is directed


at overwhelming feeling that the victim experiences
just after the rape has occurred?

a. Guilt
b. Rage
c. Damaged
d. Despair

15. Anna asks, "Why do I need to have pelvic exam?"


The nurse explains:

a. "To make sure you're not pregnant."


b. "To see if you got an infection."
c. "To make sure you were really raped."
d. "To gather legal evidence that is required."

16. In providing support therapy, the nurse explains


that rape has nothing to do with sexual desires or
heeds. The two most common elements in rape
are:

a. Guilt and shame


b. Shame and jealousy
c. Embarrassment and envy
d. Power and anger

17. The rape victim will not talk, is withdrawn and


depressed. The defensive mechanism being used
is:

a. Rationalization
b. Denial
c. Repression
d. Regression

23
23

a. Secretaries of anxiety
b. Elderly
c. Students
d. Professionals

19. The best intervention is:

a. Tell her it just takes a long time


b. Ask her if her husband is angry
c. Refer her and her husband to sex therapy
d. Tell her she is suffering PTSD

Situation: Obsessions are recurring thoughts that


become prevalent in the consciousness and may be
considered as senseless or repulsive white
compulsion are the repetitive acts that follow
obsessive thoughts.

20. To understand the meaning of the cleaning


rituals, the nurse must realize:

a. The patient cannot help herself


b. The patient cannot change
c. Rituals relieve intense anxiety
d. Medications cannot help

21. Upon admission to the hospital the patient


increases the ritual behavior at bedtime. She
cannot sleep. The treatment plan should include:

a. Recommending a sedative medication


b. Modifying the routine to diminish her bedtime
anxiety
c. Reminding her to perform rituals early in the evening
d. Limit the amount of time she spends washing
her hands

22. A patient has been diagnosed with a personality


disorder with .compulsive traits. Of the following
behavior's, which one would you expect the patient
to exhibit?

a. Inability to make decisions


b. Spontaneous playfulness
c. Inability to alter plans
d. Insistence that things be done his way

23. The patient will not be able to stop her


compulsive washing routines until she:

a. Acquires more superego


b. Recognizes the behavior is unrealistic
c. No longer needs them to manage her feelings
23
d. Regains contact with reality problem in this country.

24. A 48-year-old female patient is brought to the 29. The nurse is monitoring a drug abuser who states
hospital by her husband because her behavior is he was given cocaine and heroine that war cut with
blocking her ability to meet her family's needs. She cornstarch or some other kind of powder. He states, "It
has uncontrollable and constant desire to scrub her was really bad stuff." Which complication is most
hands, the walls, floors and sofa. She keeps threatening to this patient?
repeating," Everything is dirty." This is an example of:
a. Endocarditis
a. Compulsion b. Gangrene
b. Obsession c. Pulmonary abscess
c. Delusion d. Pulmonary embolism
d. Hallucination
30. The chronic drug abuser is suffering lymphedema in
25. The female patient is preoccupied with rules and all extremities, but particularly in the arm where the
regulations. She becomes upset if others do not drug was obviously injected. There is severe
follow her lead and adhere to the rules exactly. This is obstruction of veins and lymphatics. The nurse suspects
a characteristic of which of the following personality? the patient used:

a. Compulsive a. A dull, contaminated needle


b. Borderline b. A needle contaminated with AIDS
c. Antisocial c. Contaminated drugs
d. Schizoid d. Cocaine mixed with uncut heroin

26. In planning care focused on decreasing the 31. The nurse is assessing a heroin user who injected
patient's anxiety, what plan should the nurse have in the drug into an artery instead of a vein. Which
regards to the rituals? complication is the nurse most likely to expect?

a. Encourage the routines a. Infection


b. Ignore rituals b. Cardiac dysrhythmias
c. Work with her to develop limits of behavior c. Gangrene
d. Restrain her from the rituals d. Thrombophlebitis

27. After the patient entered the hospital she began to 32. The nurse is assessing a 16-year-old patient for drug
increase her ritualistic hand washing at bedtime and abuse. The patient is incoherent. Because she notes
could; not sleep. The nurse plans care around the fact irritation of eyes, nose and mouth, she suspects
that this patient needs: inhalants. Which sign is most indicative of inhalant
abuse?
a. A substitute activity to relieve anxiety
b. Medication for sleeping a. Vomiting
c. Anti-anxiety medication such as Xanax b. Bad breath
d. More scheduled activities during the day c. Bad trip
d. Sudden fear
28. The patient states, "I know all this scrubbing is silly
but I can’t help it:'', this statement indicates that the 33. An impaired nurse has been admitted for treatment
patient does not recognize: of Demerol addiction. She asks, "When will the
withdrawal begin?" the best response is:
a. What she is doing
b. Why she is cleaning a. "It varies, with each individual."
c. Her level of anxiety b. "There is no way to tell."
d. Need for medication c. "Withdrawal begins soon after the last dose."
d. "It depends upon how well the Demerol works."
Situation: Substance, abuse is a common, growing health
34. The patient has a blood pressure of 180/100, heart
rate of 120, associated with extreme restlessness. He is a. Rationalization
very suspicious of the hospital environment and actions b. Projection
of healthcare workers. The nurse should confront this c. Compensation
patient on abuse of; d. Substitution

a. Marijuana 40. An unattractive girl becomes a very good student.


b. Cocaine This is an example of:
c. Barbiturates
d. Tranquilizers a. displacement
b. Regression
35. The nursing interventions most effective in working c. Compensation
with substance dependent patients are: d. Projection

a. Firm and directive 41. A patient has been sharing a painful experience
b. Instillation of values of sexual abuse during his childhood. Suddenly he
c. Helpful and advisory stops and says, “l can't remember any more." The
d Subjective and non-judgmental nurse assesses his behavior as:

36. An adolescent patient has bloodshot eyes, a a. Stubbornness


voracious appetite (especially for junk foods), and a b. Forgetfulness
dry mouth. Which drug of abuse would the nurse most c. Blocking
likely suspect? d. Transference

a. Marijuana 42. The patient has a phobia about walking down in


b. Amphetamines dark halls. The nurse recognizes that the coping
c. Barbiturates mechanism usually associated with phobia is:
d. Anxiolytics
a. Compensation
Situation: Defense mechanisms are unconscious b. Denial
intrapsychic process implemented to cope with anxiety. c. Conversion
The use of some of these mechanisms is healthy, while d. Displacement
she use of others is unhealthy.
43. The patient is denying that he is an alcoholic He
37. A patient cries and curls in a fetal position states that his wife is an alcoholic. The defense
refusing to move or talk. This is an example of: mechanism he is utilizing is: v

a. Regression a. Sublimation
b. Suppression b. Projection
c. Conversion c. Suppression
d. Sublimation d. Displacement

38. A person who expands sexual energy in a Situation: Ms. Dwane, 17 years old, is admitted with
nonsexual, socially accepted way is using the coping anorexia nervosa. You have been assigned to sit with her
mechanism of. while she eats her dinner. Ms. Dwane says "My primary
nurse trusts me. I don't see why you don't."
a. Projection
b. Conversion 44. Which observation of the client with anorexia
c. Sublimation nervosa indicates the client is improving?
d. Compensation
a. The client eats meats in the dining room
39. "The reason I did not do well on the exam is that I b. The client gains one pound per week
was tired." This is an example of: c. The client attends group therapy sessions

23
23
d. The client has a more realistic self-concept
Situation: The nurse suspects a client is denying his
45. The nurse is caring for a client with anorexia feelings of anxiety
nervosa who is to be placed on behavioral
modification. Which is appropriate to include in (he 50. The nurse is monitoring a patient who is
nursing care plan? experiencing increasing anxiety related to recent
accident. She notes an increase in vital signs from
a. Remind the client frequently to eat all the food 130/70 to 160/30, pulse rate of 120, respiration 36.
served on the tray He is having difficulty communicating. His level of
b. Increased phone calls allowed for client by one per day anxiety is:
for each pound gained
c. Include the family of the client in therapy sessions two a. Mild
times per week b. Moderate
d. Weigh the client each day at 6:00 am in hospital gown c. Severe
and slippers after she voids d. Panic

46. A nursing intervention based on the behavior 51. The patient who suffers panic attacks is prescribed
modification model of treatment for anorexia nervosa a medication for short-term therapy. The nurse
would be: prepares to administer.

a. Role playing the client's interaction with her parents a. Elavil


b. Encouraging the client to vent her feelings through b. Librium
exercise c. Xanax
c. Providing a high-calorie, high protein diet d. Mellaril
with between meals snacks
d. Restricting the client's privileges until she gains three 52. In attempting to control a patient who is suffering
pounds panic attack, the nursing priority is:

47. While admitting Ms. Dwane, the nurse discovers a a. Provide safely
bottle of pills that Ms. Dwane calls antacids. She takes b. Hold the patient
them because her stomach hurts. The nurse's best c. Describe crisis in detail
initial response is: d. Demonstrate ADLs frequently

a. Tell me more about your stomach pain 53. Which assessment would the nurse most likely
b. These do not look like antacids. I need to get an order find in a person who is suffering increased anxiety?
for you to have them
c. Tell me more about you drug use a. Increasing BP, increasing heart rate and respirations
d. Some girls take pills to help them lose weight b. Decreasing BP, heart rate and respirations
c. Increased BP and decreased respirations
48. The primary objective in the treatment of the d. Increased respirations and decreased heart rate
hospitalized anorexic client is to:
54. A patient who suffers an acute anxiety disorder
a. Decrease the client's anxiety approaches the nurse and while clutching at his shirt
b. Increase the insight into the disorder states "I think I'm having a heart attack." The priority
c. Help the mother to gain control nursing action is:
d. Get the client to ea and gain weight
a. Reassure him he is OK
49. Your best response for Ms. Dwane is: b. Take vital signs stat
c. Administer Valium IM
a. I do trust you, but I was assigned to be with you d. Administer Xanax PO
b. It sounds as if you are manipulating me
c. Ok, when I return, you should have eaten everything 55. In teaching stress management, the goal of
d. Who is your primary nurse? therapy is to:
negativistic behavior. Her family states that Raul is in
a. Get rid of the major stressor good health. Raul asks you, "Where am I?"
b. Change lifestyle completely
c. Modify responses to stress 60. Another patient, Mr. Pat, has been brought to the
d. Learn new ways of thinking psychiatric unit and is pacing up and down the hall.
The nurse is to admit him to the hospital. To establish
56. Another client walks in to the mental health a nurse-client relationship, which approach should the
outpatient center and States, "I've had it. I can't go on nurse try first?
any longer. You've got to help me. "The nurse asks the
client to be seated in a private interview room. Which a. Assign someone to watch Mr. Pat until he is calm
action should the nurse take next? b. Ask Mr. Pat to sit down and orient him to the nurse's
name and the need for information
a. Reassure the client that someone will help him soon c. Check Mr. Pat's vital signs, ask him about allergies,
b. Assess the client's insurance coverage and call the physician for sedation
c. Find out more about what is happening to the client d. Explain the importance of accurate assessment data
d. Call the client's family to come and provide support to Mr. Pat .

57. Mr. Juan is admitted for panic attack. He frequently 61. If Raul will say "I'm so afraid! Where I am? Where
experiences shortness of breath, palpitations, nausea, is my family'?" How should the nurse respond?
diaphoresis, and terror. What should the nurse include
in the care plan for Mr. Juan? When he is shaving a a. "You are in the hospital and you're safe here. Your
panic attack? family will return at 10 o'clock, which is one hour from
now"
a. Calm reassurance, deep breathing and medications as b. "You know were you are. You were admitted here 2
ordered weeks ago. Don’t worry your family will be back soon."
b. Teach Mr. Juan problem solving in relation to his c. "I just told you that you're in the hospital and your
anxiety family will be here soon."
c. Explain the physiologic responses of anxiety d. "The name of the hospital is on the sigh over the door.
d. Explore alternate methods for dealing with the cause Let's go read it again."
of his anxiety
62. Raul has had difficulty sleeping since admission.
58. Ms. Wendy is pacing about the unit and wringing Which of the following would be the best intervention?
his hands. She is breathing rapidly and complains of
palpitations and nausea, and she has difficulty focusing a. Provide him with glass of warm milk
on what the nurse is saying. She says she is having a b. Ask the physician for a mild sedative
heart attack but refuses to rest. The nurse would c. Do not allow Raul to take naps during the day
interpret her level of anxiety as: d. Ask him family what they prefer

a. Mild 63. Which activity would you engage in Raul at the


b. Moderate nursing home?
c. Severe
d. Panic a. Reminiscence groups
b. Sing-along
59. When assessing this client, the nurse must d. Discussion groups
be particularly alert to: c. Exercise class

a. Restlessness 64. Which of the following would be an appropriate


b. Tapping of the feet strategy in reorienting a confused client to where her
c. Wringing of the hands room is?
d. His or her own anxiety level
a. Place pictures of her family on the bedside stand
Situation: Raul aged 70 was recently admitted to a b. Put her name in large letters on her forehead
nursing home because of confusion, disorientation, and c. Remind the client where her room is

24
24
d. Let the other residents know where the client’s room d. "What caused you to think you were God?"
is
70. The nurse is caring for a client who is experiencing
65. The best response for the nurse to make is: auditory hallucination. What would be most crucial for
the nurse to assess?
a. Don't worry, Raul. You're safe here
b. Where do you think you are? a. Possible hearing impairment
c. What did your family tell you? b. Family history of psychosis
d. You're at the community nursing home c. Content of the hallucination
d. Otitis media
Situation: The police bring a patient to the emergency
department. He has been locked in his apartment for the 71. A patient with schizophrenia reports that the
past 3 days, making frequent calls to the police and newscaster on the radio has a divine message
emergency services and stating that people are trying to especially for her. You would interpret this as
kill him. indicating.

66. A client on an inpatient psychiatric unit refuses a. Loose of associations


to eat and states that the staff is poisoning her food. b. Delusion of reference
Which action should the nurse include in the client's c. Paranoid speech
care plan? d. Flight of ideas

a. Explain to the client that the staff can be trusted 72. What type of delusions is the patient experiencing?
b. Show the client that others eat the food without harm
c. Offer the client factory-sealed foods and beverages a. Persecutory
d. Institute behavioral modification with privileges b. Grandiose
dependent on intake c. Jealous
d. Somatic
67. The client tells the nurse that he can't eat
because his food has been poisoned. This statement Situation: Helen, with a diagnosis of disorganized
is an indication of which of the following? schizophrenia is creating a disturbance in the day room.
She is yelling and pointing at another patient, accusing
a. Paranoia him to stealing her purse. Several patients are in the day
b. Delusion of persecution room when this incident starts.
c. Hallucination
d. Illusion 73. The nurse is preparing to care for a client
diagnosed with catatonic schizophrenia. In anticipation
68. The client on antipsychotic drugs begins to exhibit of this client's arrival, what should the nurse do?
signs and symptoms of which disorder?
a. Notify security
a. Akinesia b. Prepare a magnesium sulfate drip
b. Pseudoparkinsonism c. Place a specialty mattress overlay on the bed
c. Tardive dyskinesia d. Communicable the client's nothing-by-mouth status to
d. Oculogyric crisis the dietary department

69. During a patient history, a patient state that she 74. The nurse is caring for a client whom she suspects is
used to believe she was God. But she knows this paranoid. How would the nurse confirm this
isn't true. Which of the following would be your best assessment?
response?"
a. indirect questioning
a. "Does it bother you that you used to believe that b. Direct questioning
about yourself?" c. Les-ad-in-sentences
b. "Your thoughts are now more appropriate" d. Open-ended sentences
c. "Many people have these delusions."
75. Which of the following is an example of a c. Affect more women than men
negative symptom of schizophrenia? d. May be related to certain medical conditionsa

a. Delusions 80. A patient with schizophrenia (catatonic type) is


b. Disorganized speech mute and can't perform activities of daily living. The
c. Flat affect patient stares out the window for hours. What is
d. Catatonic behavior your first priority in this situation?

76. The patient tells you that a "voice" keeps a. Assist the patient with feeding
laughing at him and tells him he must crawl on his b. Assist the patient with showering and tasks for
hands and knees like a dog. Which of the following hygiene
would be the most appropriate response? c. Reassure the patient about safely, and try to orient
him to his surroundings
a. "They are imaginary voices and we're here to make d. Encourage, socialization with peers, and provide a
them go, away." stimulating environment
b. "If it makes you feel better, do what the voices tell
you." 81. Which of the following would you suspect in a
c. "The voices can't hurt you here in the hospital" patient receiving Chlorpromazine (Thorazine) who
d. "Even though I don't hear the voices, I understand that complains of a sore throat and has a fever?
you do."
a. An allergic reaction
77. A 23-year-old patient is receiving antipsychotic b. Jaundice
medication to treat his schizophrenia. He's c. Dyskinesia
experiencing some motor abnormalities called d. Agranulocytosis
extrapyramidal effects. Which of the following
extrapyramidal effects occurs most frequently in 82. While providing information for the family of a
younger make patients? patient with schizophrenia, you should be sure to
inform them about which of the following
a. Akathisia characteristics of the disorder?
b. Akinesia
c. Dystonia a. Relapse can be prevented if the patient takes
d. Pseudoparkinsonism medication
b. Support is available to help family members meet their
78. Which of the following should you do next? own needs
c. Improvement should occur if the patient's
a. Firmly redirect the patient to her room to discuss environment is carefully maintained
the incident d. Stressful situations in the family in the family can
b. Call the assistance and place the patient in locked precipitate a relapse in the patient
seclusion
c. Help the patient look for her purse 83. While caring for John, the nurse knows that
d. Don't intervene - the patients need a little bit of room John may have trouble with:
in which to work out differences
a. Staff who are cheerful
Situation: John is admitted with a diagnosis of paranoid b. Simple direct sentences
schizophrenia. c. Multiple commands
d. Violent behaviors
79. You're reaching a community group about
schizophrenia disorders. You explain the different types 84 Which nursing diagnosis is most likely to be
of schizophrenia and delusional disorders. You also associated with a person who has a medical diagnosis
explain that, unlike schizophrenia, delusional disorders: of schizophrenia, paranoid type?

a. Tend to begin in early childhood a. Fear of being along


b. Affect more men than women b. Perceptual disturbance related to delusion of

24
24
persecution hospital
c. Social isolation related to impaired ability to trust b. Provide nutritious food and a quite place to rest
d. Impaired social skills related to inadequate developed c. Protect the client and others from harm
superego d. Create a structured environment

85. Which of the following behaviors can the nurse Situation: Wendell, 24 year-old student with a primary
anticipate with this client? sleep disorder, is unable to initiate maintenance of
sleep. Primary sleep disorders may be categorized as
a. Negative cognitive distortions dyssomnias or parasomnias.
b. Impaired psychomotor development
c. Delusions of grandeur and hyperactivity 91. The nurse is caring for a client who complains; of
d. Alteration of appetite and sleep pattern fat?gue, inability to concentrate, and palpitations.
The client stales that she has been experiencing these
Situation: A client is admitted to the hospital. During the symptoms for the past 6 months. Which factor in the
assessment the nurse notes that the client has not slept client’s history has most likely contributed to.these
for a week. The client is talking rapidly, and throwing his symptoms?
arms around randomly.
a. History of recent fever
86. When writing an assessment of a client with b. Shift work
mood disorder, the nurse should specify: c. Hyperthyroidism
d. Fear
a. How flat the client's affect
b. How suicidal the client is 92. If Wendell complains of experiencing an
c. How grandiose the client is overwhelming urge to sleep and states that he's been
d. How the client is behaving falling asleep while studying and reports that these
episodes occur about 5 times daily Wendell is most
87. It is an apprehensive anticipation of an unknown likely experiencing which sleep disorder?
danger:
a. Breathing-related sleep disorder
a. Fear b. Narcolepsy
b. Anxiety c. Primary hypersomnia
c. Antisocial d. Circadian rhythm disorder
d. Schizoid
93. The nurse is preparing a teaching plan for a
88. It is an, emotional response to a client diagnosed with primary insomnia. Which of
consciously recognized threat. the following teaching topics should be included in
the plan?
a. Fear
b. Anxiety a. Eating unlimited spicy foods, and limiting caffeine and
c. Antisocial alcohol
d. Schizoid b. Exercising 1 hour before bedtime to promote sleep
c. Importance of steeping whenever the client tires
89. All but one is an example of situational crisis: d. Drinking warm milk before bed to induce sleep

a. Menstruation 94. Examples of dyssomnia includes:


b. Role changes
c. Rape a. Insomnia, hypersomnia, narcolepsy
d. Divorce b. Sleepwalking, nightmare
c. Snoring while sleeping
90. What would be the highest priority in formulating d. Non-rapid eye movement
a nursing care plan for this client?
Situation: The following questions refer to therapeutic
a. Isolate the client until he or she adjusts to 'the communication.
stimulated her anger by using a condescending tone of
95. When preparing to conduct group therapy, the voice. Which of the following responses by the nurse
nurse keeps in mind that the optimal number of would be the most therapeutic?
clients in a group would be:
a. "I feel angry when I hear that tone of voice"
a. 6 to 8 b. "You make me so angry when you talked to me that
b. 10 to 12 way."
c. 3 to 5 c. "Are you trying to make me angry?"
d. Unlimited d. "Why do you use that condescending tone of
voice with me?"
96. What occurs during the working phase of the-nurse-
client relationship? 100. A 35 year-old client tells the nurse that he never
disagrees with anyone and that he has loved everyone
a. The nurse assesses the client's needs and develops a he's ever known. What would be the nurse's best
plan of care response to this client?
b. The nurse and client together evaluate and modify the
goals of the relationship a. "How do you manage to do that?"
c. The nurse and client discuss their feelings about b. "That's hard to believe. Most people couldn't to that."
terminating the relationship c. "What do you do with your feelings of dissatisfaction
d. The nurse and client explore each other's expectations or anger?"
of-the relationship d. "How did you come to adopt such a way of life?"

97. A 42 year-old homemaker arrives at the emergency


department with uncomfortable crying and anxiety.
Her husband of 17 years has recently asked her for a
divorce. The patient is sitting in a chair, rocking back
and forth. Which is the best response for the nurse to
make?

a. "You must stop crying so that we can discuss your


feelings about the divorce."
b. "Once you find a job, you will feel much better and
more secure."
c. "I can see how upset you are. Let's sit in the office so
that we can talk about how you're feeling."
d. "Once you have a lawyer looking out for your
interests, you will feel better."

98. A client on the unit tells the nurse that his wife's
nagging really gets on his nerves. He asks the nurse
if she will talk with his wife about nagging during
their family session tomorrow afternoon. Which of
the following would be most therapeutic response
to client?

a. "Tell me more specifically about her complaints"


b. "Can you think why she might nag you so much?"
c. "I'll help you think about how to bring this up yourself
tomorrow."
d. "Why do you want me to initiate this discussion in
tomorrow's session rather than you?"

99. The nurse is working with a client who has just

24
24
Nursing Practice Test V 6. One of the reasons that persons with antisocial

Situation: The nurse is interviewing a handsome man. He


is intelligent and very charming. When asked about his
family, he states he has been married four times. He says
three of those marriages were "shotgun" weddings. He
states he never really loved any of his wives. He doesn't
know much about his three children. "I've lost track," he
states.

1. If a patient is very resistant in taking responsibility


of his action and asks, "Can you just give me some
medication?" the best response is:

a. "The medication has too many side effects."


b. You don't want to take medication, do you?"
c. Medication is given only as a East resort."
d. "There is no medication specific for your condition."

2. The patient asks the nurse, "What is this therapy


for anyway. I just don't understand it." the best reply
is:

a. "It keeps you from being put on medications."


b. "It helps you to change others in the family."
c. "The purpose of therapy is to help you change."
d. "No one but professionals can really understand

3. For patient in group therapy, the goal is:

a. Exchanging information and ideas


b. Developing insight by relating to others
c. Learning that everyone has problems
d. All of the above

4. In planning care for the patient with a personality


disorder, the nurse realizes that this patient will
most likely:

a. Not need long-term therapy


b. Not require medication
c. Require anti-anxiety medication
d. Resist any change in behavior

5. The person with an antisocial personality is


participating in therapy while a patient at a
psychiatric hospital. The nurse’s expectations are that
he will:

a. Make a complete recovery


b. Make significant changes
c. Begin the slow process of change
d. Make few changes, if any
personalities may marry repeatedly or get into trouble
with legal authorities is:

a. They usually just don't care


b. They are borderline mentally retarded
c. They are too psychotic to see what’s going on
d. They do not learn from past mistakes

7. The nurse recognizes that these are traits of:

a. Bipolar disorder
b. Alcoholic personality
c. Antisocial personality
d. Borderline personality

Situation: The patient with bipolar disorder is


pacing continuously and is skipping meals.

8. Blood levels are drawn on the patient who has


been taking Lithium for about six months. The
present level is 2.1 meq/L. The nurse evaluates this
level as:

a. Therapeutic
b. Below therapeutic
c. Potentially dangerous
d. Fatally toxic

9. The priority in working with patient a


thought disorder is:

a. Get him to understand what you're saying


b. Get him to do his ADLs
c. Reorient him to reality
d. Administer antipsychotic medications

10. The most recent Lithium level on bipolar patient


indicates a drop non-therapeutic level. What
associated behavior does the nurse assess?

a. Ataxia
b. Confusion
c. Hyperactivity
d. Lethargy

11. Adequate fluid intake for a patient on Lithium is:

a. 1,000 ml per day


b. 1,500 ml per day
c. 2,000 ml per day
d. 3,600 ml per day

12. The physician orders Lithium carbonate for


the bipolar patient. The nurse is aware that:

24
a. The patient should be put on a special diet most victimized women are:
b. The medication should be given only at night
c. A salt-free should be provided for the patient
d. The drug level should be monitored regularly

13. The nursing plan should emphasize:

a. Offering him finger foods


b. Telling him he must sit down and eat
c. Serving food in his room and staying with him
d. Telling him to order fast food of he wants to eat

Situation: Anna, 25 years old was raped six months ago


states, "I just can't seem to get over this. My husband
and I don't even have sex anymore. What can I do?"

14. Supportive therapy to the rape victim is directed


at overwhelming feeling that the victim experiences
just after the rape has occurred?

a. Guilt
b. Rage
c. Damaged
d. Despair

15. Anna asks, "Why do I need to have pelvic exam?"


The nurse explains:

a. "To make sure you're not pregnant."


b. "To see if you got an infection."
c. "To make sure you were really raped."
d. "To gather legal evidence that is required."

16. In providing support therapy, the nurse explains


that rape has nothing to do with sexual desires or
heeds. The two most common elements in rape
are:

a. Guilt and shame


b. Shame and jealousy
c. Embarrassment and envy
d. Power and anger

17. The rape victim will not talk, is withdrawn and


depressed. The defensive mechanism being used is:

a. Rationalization
b. Denial
c. Repression
d. Regression

18. The composite picture of rape victim reveals that

24
24

a. Secretaries of anxiety
b. Elderly
c. Students
d. Professionals

19. The best intervention is:

a. Tell her it just takes a long time


b. Ask her if her husband is angry
c. Refer her and her husband to sex therapy
d. Tell her she is suffering PTSD

Situation: Obsessions are recurring thoughts that


become prevalent in the consciousness and may be
considered as senseless or repulsive white
compulsion are the repetitive acts that follow
obsessive thoughts.

20. To understand the meaning of the cleaning


rituals, the nurse must realize:

a. The patient cannot help herself


b. The patient cannot change
c. Rituals relieve intense anxiety
d. Medications cannot help

21. Upon admission to the hospital the patient


increases the ritual behavior at bedtime. She
cannot sleep. The treatment plan should include:

a. Recommending a sedative medication


b. Modifying the routine to diminish her bedtime
anxiety
c. Reminding her to perform rituals early in the evening
d. Limit the amount of time she spends washing
her hands

22. A patient has been diagnosed with a personality


disorder with .compulsive traits. Of the following
behavior's, which one would you expect the patient
to exhibit?

a. Inability to make decisions


b. Spontaneous playfulness
c. Inability to alter plans
d. Insistence that things be done his way

23. The patient will not be able to stop her


compulsive washing routines until she:

a. Acquires more superego


b. Recognizes the behavior is unrealistic
c. No longer needs them to manage her feelings
24
d. Regains contact with reality problem in this country.

24. A 48-year-old female patient is brought to the 29. The nurse is monitoring a drug abuser who states
hospital by her husband because her behavior is he was given cocaine and heroine that war cut with
blocking her ability to meet her family's needs. She cornstarch or some other kind of powder. He states, "It
has uncontrollable and constant desire to scrub her was really bad stuff." Which complication is most
hands, the walls, floors and sofa. She keeps threatening to this patient?
repeating," Everything is dirty." This is an example of:
a. Endocarditis
a. Compulsion b. Gangrene
b. Obsession c. Pulmonary abscess
c. Delusion d. Pulmonary embolism
d. Hallucination
30. The chronic drug abuser is suffering lymphedema in
25. The female patient is preoccupied with rules and all extremities, but particularly in the arm where the
regulations. She becomes upset if others do not drug was obviously injected. There is severe
follow her lead and adhere to the rules exactly. This is obstruction of veins and lymphatics. The nurse suspects
a characteristic of which of the following personality? the patient used:

a. Compulsive a. A dull, contaminated needle


b. Borderline b. A needle contaminated with AIDS
c. Antisocial c. Contaminated drugs
d. Schizoid d. Cocaine mixed with uncut heroin

26. In planning care focused on decreasing the 31. The nurse is assessing a heroin user who injected
patient's anxiety, what plan should the nurse have in the drug into an artery instead of a vein. Which
regards to the rituals? complication is the nurse most likely to expect?

a. Encourage the routines a. Infection


b. Ignore rituals b. Cardiac dysrhythmias
c. Work with her to develop limits of behavior c. Gangrene
d. Restrain her from the rituals d. Thrombophlebitis

27. After the patient entered the hospital she began to 32. The nurse is assessing a 16-year-old patient for drug
increase her ritualistic hand washing at bedtime and abuse. The patient is incoherent. Because she notes
could; not sleep. The nurse plans care around the fact irritation of eyes, nose and mouth, she suspects
that this patient needs: inhalants. Which sign is most indicative of inhalant
abuse?
a. A substitute activity to relieve anxiety
b. Medication for sleeping a. Vomiting
c. Anti-anxiety medication such as Xanax b. Bad breath
d. More scheduled activities during the day c. Bad trip
d. Sudden fear
28. The patient states, "I know all this scrubbing is silly
but I can’t help it:'', this statement indicates that the 33. An impaired nurse has been admitted for treatment
patient does not recognize: of Demerol addiction. She asks, "When will the
withdrawal begin?" the best response is:
a. What she is doing
b. Why she is cleaning a. "It varies, with each individual."
c. Her level of anxiety b. "There is no way to tell."
d. Need for medication c. "Withdrawal begins soon after the last dose."
d. "It depends upon how well the Demerol works."
Situation: Substance, abuse is a common, growing health
34. The patient has a blood pressure of 180/100, heart
rate of 120, associated with extreme restlessness. He is a. Rationalization
very suspicious of the hospital environment and actions b. Projection
of healthcare workers. The nurse should confront this c. Compensation
patient on abuse of; d. Substitution

a. Marijuana 40. An unattractive girl becomes a very good student.


b. Cocaine This is an example of:
c. Barbiturates
d. Tranquilizers a. displacement
b. Regression
35. The nursing interventions most effective in working c. Compensation
with substance dependent patients are: d. Projection

a. Firm and directive 41. A patient has been sharing a painful experience
b. Instillation of values of sexual abuse during his childhood. Suddenly he
c. Helpful and advisory stops and says, “l can't remember any more." The
d Subjective and non-judgmental nurse assesses his behavior as:

36. An adolescent patient has bloodshot eyes, a a. Stubbornness


voracious appetite (especially for junk foods), and a b. Forgetfulness
dry mouth. Which drug of abuse would the nurse most c. Blocking
likely suspect? d. Transference

a. Marijuana 42. The patient has a phobia about walking down in


b. Amphetamines dark halls. The nurse recognizes that the coping
c. Barbiturates mechanism usually associated with phobia is:
d. Anxiolytics
a. Compensation
Situation: Defense mechanisms are unconscious b. Denial
intrapsychic process implemented to cope with anxiety. c. Conversion
The use of some of these mechanisms is healthy, while d. Displacement
she use of others is unhealthy.
43. The patient is denying that he is an alcoholic He
37. A patient cries and curls in a fetal position states that his wife is an alcoholic. The defense
refusing to move or talk. This is an example of: mechanism he is utilizing is: v

a. Regression a. Sublimation
b. Suppression b. Projection
c. Conversion c. Suppression
d. Sublimation d. Displacement

38. A person who expands sexual energy in a Situation: Ms. Dwane, 17 years old, is admitted with
nonsexual, socially accepted way is using the coping anorexia nervosa. You have been assigned to sit with her
mechanism of. while she eats her dinner. Ms. Dwane says "My primary
nurse trusts me. I don't see why you don't."
a. Projection
b. Conversion 44. Which observation of the client with anorexia
c. Sublimation nervosa indicates the client is improving?
d. Compensation
a. The client eats meats in the dining room
39. "The reason I did not do well on the exam is that I b. The client gains one pound per week
was tired." This is an example of: c. The client attends group therapy sessions

24
24
d. The client has a more realistic self-concept
Situation: The nurse suspects a client is denying his
45. The nurse is caring for a client with anorexia feelings of anxiety
nervosa who is to be placed on behavioral
modification. Which is appropriate to include in (he 50. The nurse is monitoring a patient who is
nursing care plan? experiencing increasing anxiety related to recent
accident. She notes an increase in vital signs from
a. Remind the client frequently to eat all the food 130/70 to 160/30, pulse rate of 120, respiration 36.
served on the tray He is having difficulty communicating. His level of
b. Increased phone calls allowed for client by one per day anxiety is:
for each pound gained
c. Include the family of the client in therapy sessions two a. Mild
times per week b. Moderate
d. Weigh the client each day at 6:00 am in hospital gown c. Severe
and slippers after she voids d. Panic

46. A nursing intervention based on the behavior 51. The patient who suffers panic attacks is prescribed
modification model of treatment for anorexia nervosa a medication for short-term therapy. The nurse
would be: prepares to administer.

a. Role playing the client's interaction with her parents a. Elavil


b. Encouraging the client to vent her feelings through b. Librium
exercise c. Xanax
c. Providing a high-calorie, high protein diet d. Mellaril
with between meals snacks
d. Restricting the client's privileges until she gains three 52. In attempting to control a patient who is suffering
pounds panic attack, the nursing priority is:

47. While admitting Ms. Dwane, the nurse discovers a a. Provide safely
bottle of pills that Ms. Dwane calls antacids. She takes b. Hold the patient
them because her stomach hurts. The nurse's best c. Describe crisis in detail
initial response is: d. Demonstrate ADLs frequently

a. Tell me more about your stomach pain 53. Which assessment would the nurse most likely
b. These do not look like antacids. I need to get an order find in a person who is suffering increased anxiety?
for you to have them
c. Tell me more about you drug use a. Increasing BP, increasing heart rate and respirations
d. Some girls take pills to help them lose weight b. Decreasing BP, heart rate and respirations
c. Increased BP and decreased respirations
48. The primary objective in the treatment of the d. Increased respirations and decreased heart rate
hospitalized anorexic client is to:
54. A patient who suffers an acute anxiety disorder
a. Decrease the client's anxiety approaches the nurse and while clutching at his shirt
b. Increase the insight into the disorder states "I think I'm having a heart attack." The priority
c. Help the mother to gain control nursing action is:
d. Get the client to ea and gain weight
a. Reassure him he is OK
49. Your best response for Ms. Dwane is: b. Take vital signs stat
c. Administer Valium IM
a. I do trust you, but I was assigned to be with you d. Administer Xanax PO
b. It sounds as if you are manipulating me
c. Ok, when I return, you should have eaten everything 55. In teaching stress management, the goal of therapy
d. Who is your primary nurse? is to:
negativistic behavior. Her family states that Raul is in
a. Get rid of the major stressor good health. Raul asks you, "Where am I?"
b. Change lifestyle completely
c. Modify responses to stress 60. Another patient, Mr. Pat, has been brought to the
d. Learn new ways of thinking psychiatric unit and is pacing up and down the hall.
The nurse is to admit him to the hospital. To establish
56. Another client walks in to the mental health a nurse-client relationship, which approach should the
outpatient center and States, "I've had it. I can't go on nurse try first?
any longer. You've got to help me. "The nurse asks
the client to be seated in a private interview room. a. Assign someone to watch Mr. Pat until he is calm
Which action should the nurse take next? b. Ask Mr. Pat to sit down and orient him to the nurse's
name and the need for information
a. Reassure the client that someone will help him soon c. Check Mr. Pat's vital signs, ask him about allergies,
b. Assess the client's insurance coverage and call the physician for sedation
c. Find out more about what is happening to the client d. Explain the importance of accurate assessment data
d. Call the client's family to come and provide support to Mr. Pat .

57. Mr. Juan is admitted for panic attack. He frequently 61. If Raul will say "I'm so afraid! Where I am? Where
experiences shortness of breath, palpitations, nausea, is my family'?" How should the nurse respond?
diaphoresis, and terror. What should the nurse include
in the care plan for Mr. Juan? When he is shaving a a. "You are in the hospital and you're safe here. Your
panic attack? family will return at 10 o'clock, which is one hour from
now"
a. Calm reassurance, deep breathing and medications as b. "You know were you are. You were admitted here 2
ordered weeks ago. Don’t worry your family will be back soon."
b. Teach Mr. Juan problem solving in relation to his c. "I just told you that you're in the hospital and your
anxiety family will be here soon."
c. Explain the physiologic responses of anxiety d. "The name of the hospital is on the sigh over the door.
d. Explore alternate methods for dealing with the cause Let's go read it again."
of his anxiety
62. Raul has had difficulty sleeping since admission.
58. Ms. Wendy is pacing about the unit and wringing Which of the following would be the best intervention?
his hands. She is breathing rapidly and complains of
palpitations and nausea, and she has difficulty focusing a. Provide him with glass of warm milk
on what the nurse is saying. She says she is having a b. Ask the physician for a mild sedative
heart attack but refuses to rest. The nurse would c. Do not allow Raul to take naps during the day
interpret her level of anxiety as: d. Ask him family what they prefer

a. Mild 63. Which activity would you engage in Raul at the


b. Moderate nursing home?
c. Severe
d. Panic a. Reminiscence groups
b. Sing-along
59. When assessing this client, the nurse must d. Discussion groups
be particularly alert to: c. Exercise class

a. Restlessness 64. Which of the following would be an appropriate


b. Tapping of the feet strategy in reorienting a confused client to where her
c. Wringing of the hands room is?
d. His or her own anxiety level
a. Place pictures of her family on the bedside stand
Situation: Raul aged 70 was recently admitted to a b. Put her name in large letters on her forehead
nursing home because of confusion, disorientation, and c. Remind the client where her room is

25
25
d. Let the other residents know where the client’s room d. "What caused you to think you were God?"
is
70. The nurse is caring for a client who is experiencing
65. The best response for the nurse to make is: auditory hallucination. What would be most crucial for
the nurse to assess?
a. Don't worry, Raul. You're safe here
b. Where do you think you are? a. Possible hearing impairment
c. What did your family tell you? b. Family history of psychosis
d. You're at the community nursing home c. Content of the hallucination
d. Otitis media
Situation: The police bring a patient to the emergency
department. He has been locked in his apartment for the 71. A patient with schizophrenia reports that the
past 3 days, making frequent calls to the police and newscaster on the radio has a divine message
emergency services and stating that people are trying to especially for her. You would interpret this as
kill him. indicating.

66. A client on an inpatient psychiatric unit refuses a. Loose of associations


to eat and states that the staff is poisoning her food. b. Delusion of reference
Which action should the nurse include in the client's c. Paranoid speech
care plan? d. Flight of ideas

a. Explain to the client that the staff can be trusted 72. What type of delusions is the patient experiencing?
b. Show the client that others eat the food without harm
c. Offer the client factory-sealed foods and beverages a. Persecutory
d. Institute behavioral modification with privileges b. Grandiose
dependent on intake c. Jealous
d. Somatic
67. The client tells the nurse that he can't eat
because his food has been poisoned. This statement Situation: Helen, with a diagnosis of disorganized
is an indication of which of the following? schizophrenia is creating a disturbance in the day room.
She is yelling and pointing at another patient, accusing
a. Paranoia him to stealing her purse. Several patients are in the day
b. Delusion of persecution room when this incident starts.
c. Hallucination
d. Illusion 73. The nurse is preparing to care for a client
diagnosed with catatonic schizophrenia. In anticipation
68. The client on antipsychotic drugs begins to exhibit of this client's arrival, what should the nurse do?
signs and symptoms of which disorder?
a. Notify security
a. Akinesia b. Prepare a magnesium sulfate drip
b. Pseudoparkinsonism c. Place a specialty mattress overlay on the bed
c. Tardive dyskinesia d. Communicable the client's nothing-by-mouth status to
d. Oculogyric crisis the dietary department

69. During a patient history, a patient state that she 74. The nurse is caring for a client whom she suspects is
used to believe she was God. But she knows this paranoid. How would the nurse confirm this
isn't true. Which of the following would be your best assessment?
response?"
a. indirect questioning
a. "Does it bother you that you used to believe that b. Direct questioning
about yourself?" c. Les-ad-in-sentences
b. "Your thoughts are now more appropriate" d. Open-ended sentences
c. "Many people have these delusions."
75. Which of the following is an example of a c. Affect more women than men
negative symptom of schizophrenia? d. May be related to certain medical conditionsa

a. Delusions 80. A patient with schizophrenia (catatonic type) is


b. Disorganized speech mute and can't perform activities of daily living. The
c. Flat affect patient stares out the window for hours. What is
d. Catatonic behavior your first priority in this situation?

76. The patient tells you that a "voice" keeps a. Assist the patient with feeding
laughing at him and tells him he must crawl on his b. Assist the patient with showering and tasks for
hands and knees like a dog. Which of the following hygiene
would be the most appropriate response? c. Reassure the patient about safely, and try to orient
him to his surroundings
a. "They are imaginary voices and we're here to make d. Encourage, socialization with peers, and provide a
them go, away." stimulating environment
b. "If it makes you feel better, do what the voices tell
you." 81. Which of the following would you suspect in a
c. "The voices can't hurt you here in the hospital" patient receiving Chlorpromazine (Thorazine) who
d. "Even though I don't hear the voices, I understand that complains of a sore throat and has a fever?
you do."
a. An allergic reaction
77. A 23-year-old patient is receiving antipsychotic b. Jaundice
medication to treat his schizophrenia. He's c. Dyskinesia
experiencing some motor abnormalities called d. Agranulocytosis
extrapyramidal effects. Which of the following
extrapyramidal effects occurs most frequently in 82. While providing information for the family of a
younger make patients? patient with schizophrenia, you should be sure to
inform them about which of the following
a. Akathisia characteristics of the disorder?
b. Akinesia
c. Dystonia a. Relapse can be prevented if the patient takes
d. Pseudoparkinsonism medication
b. Support is available to help family members meet
78. Which of the following should you do next? their own needs
c. Improvement should occur if the patient's
a. Firmly redirect the patient to her room to discuss environment is carefully maintained
the incident d. Stressful situations in the family in the family can
b. Call the assistance and place the patient in locked precipitate a relapse in the patient
seclusion
c. Help the patient look for her purse 83. While caring for John, the nurse knows that
d. Don't intervene - the patients need a little bit of room John may have trouble with:
in which to work out differences
a. Staff who are cheerful
Situation: John is admitted with a diagnosis of paranoid b. Simple direct sentences
schizophrenia. c. Multiple commands
d. Violent behaviors
79. You're reaching a community group about
schizophrenia disorders. You explain the different types 84 Which nursing diagnosis is most likely to be
of schizophrenia and delusional disorders. You also associated with a person who has a medical diagnosis
explain that, unlike schizophrenia, delusional disorders: of schizophrenia, paranoid type?

a. Tend to begin in early childhood a. Fear of being along


b. Affect more men than women b. Perceptual disturbance related to delusion of

25
25
persecution hospital
c. Social isolation related to impaired ability to trust b. Provide nutritious food and a quite place to rest
d. Impaired social skills related to inadequate developed c. Protect the client and others from harm
superego d. Create a structured environment

85. Which of the following behaviors can the nurse Situation: Wendell, 24 year-old student with a primary
anticipate with this client? sleep disorder, is unable to initiate maintenance of
sleep. Primary sleep disorders may be categorized as
a. Negative cognitive distortions dyssomnias or parasomnias.
b. Impaired psychomotor development
c. Delusions of grandeur and hyperactivity 91. The nurse is caring for a client who complains; of
d. Alteration of appetite and sleep pattern fat?gue, inability to concentrate, and palpitations.
The client stales that she has been experiencing these
Situation: A client is admitted to the hospital. During the symptoms for the past 6 months. Which factor in the
assessment the nurse notes that the client has not slept client’s history has most likely contributed to.these
for a week. The client is talking rapidly, and throwing his symptoms?
arms around randomly.
a. History of recent fever
86. When writing an assessment of a client with b. Shift work
mood disorder, the nurse should specify: c. Hyperthyroidism
d. Fear
a. How flat the client's affect
b. How suicidal the client is 92. If Wendell complains of experiencing an
c. How grandiose the client is overwhelming urge to sleep and states that he's
d. How the client is behaving been falling asleep while studying and reports that
these episodes occur about 5 times daily Wendell is
87. It is an apprehensive anticipation of an unknown most likely experiencing which sleep disorder?
danger:
a. Breathing-related sleep disorder
a. Fear b. Narcolepsy
b. Anxiety c. Primary hypersomnia
c. Antisocial d. Circadian rhythm disorder
d. Schizoid
93. The nurse is preparing a teaching plan for a
88. It is an, emotional response to a client diagnosed with primary insomnia. Which of
consciously recognized threat. the following teaching topics should be included in
the plan?
a. Fear
b. Anxiety a. Eating unlimited spicy foods, and limiting caffeine and
c. Antisocial alcohol
d. Schizoid b. Exercising 1 hour before bedtime to promote sleep
c. Importance of steeping whenever the client tires
89. All but one is an example of situational crisis: d. Drinking warm milk before bed to induce sleep

a. Menstruation 94. Examples of dyssomnia includes:


b. Role changes
c. Rape a. Insomnia, hypersomnia, narcolepsy
d. Divorce b. Sleepwalking, nightmare
c. Snoring while sleeping
90. What would be the highest priority in formulating d. Non-rapid eye movement
a nursing care plan for this client?
Situation: The following questions refer to therapeutic
a. Isolate the client until he or she adjusts to 'the communication.
stimulated her anger by using a condescending tone of
95. When preparing to conduct group therapy, the voice. Which of the following responses by the nurse
nurse keeps in mind that the optimal number of would be the most therapeutic?
clients in a group would be:
a. "I feel angry when I hear that tone of voice"
a. 6 to 8 b. "You make me so angry when you talked to me that
b. 10 to 12 way."
c. 3 to 5 c. "Are you trying to make me angry?"
d. Unlimited d. "Why do you use that condescending tone of
voice with me?"
96. What occurs during the working phase of the-nurse-
client relationship? 100. A 35 year-old client tells the nurse that he never
disagrees with anyone and that he has loved everyone
a. The nurse assesses the client's needs and develops a he's ever known. What would be the nurse's best
plan of care response to this client?
b. The nurse and client together evaluate and modify
the goals of the relationship a. "How do you manage to do that?"
c. The nurse and client discuss their feelings about b. "That's hard to believe. Most people couldn't to that."
terminating the relationship c. "What do you do with your feelings of dissatisfaction
d. The nurse and client explore each other's expectations or anger?"
of-the relationship d. "How did you come to adopt such a way of life?"

97. A 42 year-old homemaker arrives at the emergency


department with uncomfortable crying and anxiety.
Her husband of 17 years has recently asked her for a
divorce. The patient is sitting in a chair, rocking back
and forth. Which is the best response for the nurse to
make?

a. "You must stop crying so that we can discuss your


feelings about the divorce."
b. "Once you find a job, you will feel much better and
more secure."
c. "I can see how upset you are. Let's sit in the office so
that we can talk about how you're feeling."
d. "Once you have a lawyer looking out for your
interests, you will feel better."

98. A client on the unit tells the nurse that his wife's
nagging really gets on his nerves. He asks the nurse
if she will talk with his wife about nagging during
their family session tomorrow afternoon. Which of
the following would be most therapeutic response
to client?

a. "Tell me more specifically about her complaints"


b. "Can you think why she might nag you so much?"
c. "I'll help you think about how to bring this
up yourself tomorrow."
d. "Why do you want me to initiate this discussion in
tomorrow's session rather than you?"

99. The nurse is working with a client who has just

25
25
TEST I - Foundation of Professional Nursing 5. Nurse Betty is assigned to the following clients.
Practice The client that the nurse would see first after
endorsement?
1. The nurse In-charge in labor and delivery unit a. A 34 year-old post-operative
administered a dose of terbutaline to a client appendectomy client of five hours who
without checking the client’s pulse. The standard is complaining of pain.
that would be used to determine if the nurse b. A 44 year-old myocardial infarction (MI)
was negligent is: client who is complaining of nausea.
a. The physician’s orders. c. A 26 year-old client admitted for
b. The action of a clinical nurse specialist dehydration whose intravenous (IV) has
who is recognized expert in the field. infiltrated.
c. The statement in the drug literature d. A 63 year-old post operative’s
about administration of terbutaline. abdominal hysterectomy client of three
d. The actions of a reasonably prudent days whose incisional dressing is
nurse with similar education and saturated with serosanguinous fluid.
experience.
6. Nurse Gail places a client in a four-point restraint
2. Nurse Trish is caring for a female client with a following orders from the physician. The client
history of GI bleeding, sickle cell disease, and a care plan should include:
platelet count of 22,000/μl. The female client is a. Assess temperature frequently.
dehydrated and receiving dextrose 5% in half- b. Provide diversional activities.
normal saline solution at 150 ml/hr. The client c. Check circulation every 15-30 minutes.
complains of severe bone pain and is scheduled d. Socialize with other patients once a shift.
to receive a dose of morphine sulfate. In
administering the medication, Nurse Trish 7. A male client who has severe burns is receiving
should avoid which route? H2 receptor antagonist therapy. The nurse In-
a. I.V charge knows the purpose of this therapy is to:
b. I.M a. Prevent stress ulcer
c. Oral b. Block prostaglandin synthesis
d. S.C c. Facilitate protein synthesis.
d. Enhance gas exchange
3. Dr. Garcia writes the following order for the
client who has been recently admitted “Digoxin 8. The doctor orders hourly urine output
.125 mg P.O. once daily.” To prevent a dosage measurement for a postoperative male client.
error, how should the nurse document this order The nurse Trish records the following amounts of
onto the medication administration record? output for 2 consecutive hours: 8 a.m.: 50 ml; 9
a. “Digoxin .1250 mg P.O. once daily” a.m.: 60 ml. Based on these amounts, which
b. “Digoxin 0.1250 mg P.O. once daily” action should the nurse take?
c. “Digoxin 0.125 mg P.O. once daily” a. Increase the I.V. fluid infusion rate
d. “Digoxin .125 mg P.O. once daily” b. Irrigate the indwelling urinary catheter
c. Notify the physician
4. A newly admitted female client was diagnosed d. Continue to monitor and record hourly
with deep vein thrombosis. Which nursing urine output
diagnosis should receive the highest priority?
a. Ineffective peripheral tissue perfusion 9. Tony, a basketball player twist his right ankle
related to venous congestion. while playing on the court and seeks care for
b. Risk for injury related to edema. ankle pain and swelling. After the nurse applies
c. Excess fluid volume related to peripheral ice to the ankle for 30 minutes, which statement
vascular disease. by Tony suggests that ice application has been
d. Impaired gas exchange related to effective?
increased blood flow. a. “My ankle looks less swollen now”.
b. “My ankle feels warm”.
c. “My ankle appears redder now”.
d. “I need something stronger for pain d. Pulling the lobule down and forward
relief”
16. Which instruction should nurse Tom give to a
10. The physician prescribes a loop diuretic for a male client who is having external radiation
client. When administering this drug, the nurse therapy:
anticipates that the client may develop which a. Protect the irritated skin from sunlight.
electrolyte imbalance? b. Eat 3 to 4 hours before treatment.
a. Hypernatremia c. Wash the skin over regularly.
b. Hyperkalemia d. Apply lotion or oil to the radiated area
c. Hypokalemia when it is red or sore.
d. Hypervolemia
17. In assisting a female client for immediate
11. She finds out that some managers have surgery, the nurse In-charge is aware that she
benevolent-authoritative style of management. should:
Which of the following behaviors will she exhibit a. Encourage the client to void following
most likely? preoperative medication.
a. Have condescending trust and b. Explore the client’s fears and anxieties
confidence in their subordinates. about the surgery.
b. Gives economic and ego awards. c. Assist the client in removing dentures
c. Communicates downward to staffs. and nail polish.
d. Allows decision making among d. Encourage the client to drink water prior
subordinates. to surgery.

12. Nurse Amy is aware that the following is true 18. A male client is admitted and diagnosed with
about functional nursing acute pancreatitis after a holiday celebration of
a. Provides continuous, coordinated and excessive food and alcohol. Which assessment
comprehensive nursing services. finding reflects this diagnosis?
b. One-to-one nurse patient ratio. a. Blood pressure above normal range.
c. Emphasize the use of group b. Presence of crackles in both lung fields.
collaboration. c. Hyperactive bowel sounds
d. Concentrates on tasks and activities. d. Sudden onset of continuous epigastric
and back pain.
13. Which type of medication order might read
"Vitamin K 10 mg I.M. daily × 3 days?" 19. Which dietary guidelines are important for nurse
a. Single order Oliver to implement in caring for the client with
b. Standard written order burns?
c. Standing order a. Provide high-fiber, high-fat diet
d. Stat order b. Provide high-protein, high-carbohydrate
diet.
14. A female client with a fecal impaction frequently c. Monitor intake to prevent weight gain.
exhibits which clinical manifestation? d. Provide ice chips or water intake.
a. Increased appetite
b. Loss of urge to defecate 20. Nurse Hazel will administer a unit of whole
c. Hard, brown, formed stools blood, which priority information should the
d. Liquid or semi-liquid stools nurse have about the client?
a. Blood pressure and pulse rate.
15. Nurse Linda prepares to perform an otoscopic b. Height and weight.
examination on a female client. For proper c. Calcium and potassium levels
visualization, the nurse should position the d. Hgb and Hct levels.
client's ear by: 21. Nurse Michelle witnesses a female client sustain
a. Pulling the lobule down and back a fall and suspects that the leg may be broken.
b. Pulling the helix up and forward The nurse takes which priority action?
c. Pulling the helix up and back a. Takes a set of vital signs.

25
25
b. Call the radiology department for X-ray. c. 1.5 cc
c. Reassure the client that everything will d. 2.5 cc
be alright.
d. Immobilize the leg before moving the 27. A child of 10 years old is to receive 400 cc of IV
client. fluid in an 8 hour shift. The IV drip factor is 60.
The IV rate that will deliver this amount is:
22. A male client is being transferred to the nursing a. 50 cc/ hour
unit for admission after receiving a radium b. 55 cc/ hour
implant for bladder cancer. The nurse in-charge c. 24 cc/ hour
would take which priority action in the care of d. 66 cc/ hour
this client?
a. Place client on reverse isolation. 28. The nurse is aware that the most important
b. Admit the client into a private room. nursing action when a client returns from
c. Encourage the client to take frequent surgery is:
rest periods. a. Assess the IV for type of fluid and rate of
d. Encourage family and friends to visit. flow.
b. Assess the client for presence of pain.
23. A newly admitted female client was diagnosed c. Assess the Foley catheter for patency
with agranulocytosis. The nurse formulates and urine output
which priority nursing diagnosis? d. Assess the dressing for drainage.
a. Constipation
b. Diarrhea 29. Which of the following vital sign assessments
c. Risk for infection that may indicate cardiogenic shock after
d. Deficient knowledge myocardial infarction?
a. BP – 80/60, Pulse – 110 irregular
24. A male client is receiving total parenteral b. BP – 90/50, Pulse – 50 regular
nutrition suddenly demonstrates signs and c. BP – 130/80, Pulse – 100 regular
symptoms of an air embolism. What is the d. BP – 180/100, Pulse – 90 irregular
priority action by the nurse?
a. Notify the physician. 30. Which is the most appropriate nursing action in
b. Place the client on the left side in the obtaining a blood pressure measurement?
Trendelenburg position. a. Take the proper equipment, place the
c. Place the client in high-Fowlers position. client in a comfortable position, and
d. Stop the total parenteral nutrition. record the appropriate information in
the client’s chart.
25. Nurse May attends an educational conference b. Measure the client’s arm, if you are not
on leadership styles. The nurse is sitting with a sure of the size of cuff to use.
nurse employed at a large trauma center who c. Have the client recline or sit comfortably
states that the leadership style at the trauma in a chair with the forearm at the level of
center is task-oriented and directive. The nurse the heart.
determines that the leadership style used at the d. Document the measurement, which
trauma center is: extremity was used, and the position
a. Autocratic. that the client was in during the
b. Laissez-faire. measurement.
c. Democratic.
d. Situational 31. Asking the questions to determine if the person
26. The physician orders DS 500 cc with KCl 10 understands the health teaching provided by the
mEq/liter at 30 cc/hr. The nurse in-charge is nurse would be included during which step of
going to hang a 500 cc bag. KCl is supplied 20 the nursing process?
mEq/10 cc. How many cc’s of KCl will be added
to the IV solution? a. Assessment
a. .5 cc b. Evaluation
b. 5 cc c. Implementation
d. Planning and goals “Meperidine, 100 mg/ml.” How many milliliters
of meperidine should the client receive?
32. Which of the following item is considered the a. 0.75
single most important factor in assisting the b. 0.6
health professional in arriving at a diagnosis or c. 0.5
determining the person’s needs? d. 0.25
a. Diagnostic test results
b. Biographical date 38. A male client with diabetes mellitus is receiving
c. History of present illness insulin. Which statement correctly describes an
d. Physical examination insulin unit?
a. It’s a common measurement in the
33. In preventing the development of an external metric system.
rotation deformity of the hip in a client who b. It’s the basis for solids in the avoirdupois
must remain in bed for any period of time, the system.
most appropriate nursing action would be to c. It’s the smallest measurement in the
use: apothecary system.
a. Trochanter roll extending from the crest d. It’s a measure of effect, not a standard
of the ileum to the mid-thigh. measure of weight or quantity.
b. Pillows under the lower legs.
c. Footboard 39. Nurse Oliver measures a client’s temperature at
d. Hip-abductor pillow 102° F. What is the equivalent Centigrade
temperature?
34. Which stage of pressure ulcer development does a. 40.1 °C
the ulcer extend into the subcutaneous tissue? b. 38.9 °C
a. Stage I c. 48 °C
b. Stage II d. 38 °C
c. Stage III 40. The nurse is assessing a 48-year-old client who
d. Stage IV has come to the physician’s office for his annual
physical exam. One of the first physical signs of
35. When the method of wound healing is one in aging is:
which wound edges are not surgically a. Accepting limitations while developing
approximated and integumentary continuity is assets.
restored by granulations, the wound healing is b. Increasing loss of muscle tone.
termed c. Failing eyesight, especially close vision.
a. Second intention healing d. Having more frequent aches and pains.
b. Primary intention healing
c. Third intention healing 41. The physician inserts a chest tube into a female
d. First intention healing client to treat a pneumothorax. The tube is
connected to water-seal drainage. The nurse in-
36. An 80-year-old male client is admitted to the charge can prevent chest tube air leaks by:
hospital with a diagnosis of pneumonia. Nurse a. Checking and taping all connections.
Oliver learns that the client lives alone and b. Checking patency of the chest tube.
hasn’t been eating or drinking. When assessing c. Keeping the head of the bed slightly
him for dehydration, nurse Oliver would expect elevated.
to find: d. Keeping the chest drainage system
a. Hypothermia below the level of the chest.
b. Hypertension
c. Distended neck veins 42. Nurse Trish must verify the client’s identity
d. Tachycardia before administering medication. She is aware
that the safest way to verify identity is to:
37. The physician prescribes meperidine (Demerol), a. Check the client’s identification band.
75 mg I.M. every 4 hours as needed, to control a b. Ask the client to state his name.
client’s postoperative pain. The package insert is

25
25
c. State the client’s name out loud and c. Every 2 years
wait a client to repeat it. d. Once, to establish baseline
d. Check the room number and the client’s
name on the bed. 49. A male client has the following arterial blood gas
values: pH 7.30; Pao2 89 mmHg; Paco2 50
43. The physician orders dextrose 5 % in water, mmHg; and HCO3 26mEq/L. Based on these
1,000 ml to be infused over 8 hours. The I.V. values, Nurse Patricia should expect which
tubing delivers 15 drops/ml. Nurse John should condition?
run the I.V. infusion at a rate of: a. Respiratory acidosis
a. 30 drops/minute b. Respiratory alkalosis
b. 32 drops/minute c. Metabolic acidosis
c. 20 drops/minute d. Metabolic alkalosis
d. 18 drops/minute
50. Nurse Len refers a female client with terminal
44. If a central venous catheter becomes cancer to a local hospice. What is the goal of this
disconnected accidentally, what should the referral?
nurse in-charge do immediately? a. To help the client find appropriate
a. Clamp the catheter treatment options.
b. Call another nurse b. To provide support for the client and
c. Call the physician family in coping with terminal illness.
d. Apply a dry sterile dressing to the site. c. To ensure that the client gets counseling
regarding health care costs.
45. A female client was recently admitted. She has d. To teach the client and family about
fever, weight loss, and watery diarrhea is being cancer and its treatment.
admitted to the facility. While assessing the
client, Nurse Hazel inspects the client’s abdomen 51. When caring for a male client with a 3-cm stage I
and notice that it is slightly concave. Additional pressure ulcer on the coccyx, which of the
assessment should proceed in which order: following actions can the nurse institute
a. Palpation, auscultation, and percussion. independently?
b. Percussion, palpation, and auscultation. a. Massaging the area with an astringent
c. Palpation, percussion, and auscultation. every 2 hours.
d. Auscultation, percussion, and palpation. b. Applying an antibiotic cream to the area
three times per day.
46. Nurse Betty is assessing tactile fremitus in a c. Using normal saline solution to clean the
client with pneumonia. For this examination, ulcer and applying a protective dressing
nurse Betty should use the: as necessary.
a. Fingertips d. Using a povidone-iodine wash on the
b. Finger pads ulceration three times per day.
c. Dorsal surface of the hand 52. Nurse Oliver must apply an elastic bandage to a
d. Ulnar surface of the hand client’s ankle and calf. He should apply the
bandage beginning at the client’s:
47. Which type of evaluation occurs continuously a. Knee
throughout the teaching and learning process? b. Ankle
a. Summative c. Lower thigh
b. Informative d. Foot
c. Formative
d. Retrospective 53. A 10 year old child with type 1 diabetes develops
48. A 45 year old client, has no family history of diabetic ketoacidosis and receives a continuous
breast cancer or other risk factors for this insulin infusion. Which condition represents the
disease. Nurse John should instruct her to have greatest risk to this child?
mammogram how often? a. Hypernatremia
a. Twice per year b. Hypokalemia
b. Once per year c. Hyperphosphatemia
d. Hypercalcemia d. Obtaining the specimen from the urinary
drainage bag.
54. Nurse Len is administering sublingual nitrglycerin
(Nitrostat) to the newly admitted client. 59. Nurse Meredith is in the process of giving a
Immediately afterward, the client may client a bed bath. In the middle of the
experience: procedure, the unit secretary calls the nurse on
a. Throbbing headache or dizziness the intercom to tell the nurse that there is an
b. Nervousness or paresthesia. emergency phone call. The appropriate nursing
c. Drowsiness or blurred vision. action is to:
d. Tinnitus or diplopia. a. Immediately walk out of the client’s
room and answer the phone call.
b. Cover the client, place the call light
55. Nurse Michelle hears the alarm sound on the within reach, and answer the phone call.
telemetry monitor. The nurse quickly looks at c. Finish the bed bath before answering
the monitor and notes that a client is in a the phone call.
ventricular tachycardia. The nurse rushes to the d. Leave the client’s door open so the client
client’s room. Upon reaching the client’s can be monitored and the nurse can
bedside, the nurse would take which action answer the phone call.
first?
a. Prepare for cardioversion 60. Nurse Janah is collecting a sputum specimen for
b. Prepare to defibrillate the client culture and sensitivity testing from a client who
c. Call a code has a productive cough. Nurse Janah plans to
d. Check the client’s level of consciousness implement which intervention to obtain the
specimen?
56. Nurse Hazel is preparing to ambulate a female a. Ask the client to expectorate a small
client. The best and the safest position for the amount of sputum into the emesis basin.
nurse in assisting the client is to stand: b. Ask the client to obtain the specimen
a. On the unaffected side of the client. after breakfast.
b. On the affected side of the client. c. Use a sterile plastic container for
c. In front of the client. obtaining the specimen.
d. Behind the client. d. Provide tissues for expectoration and
obtaining the specimen.
57. Nurse Janah is monitoring the ongoing care
given to the potential organ donor who has been 61. Nurse Ron is observing a male client using a
diagnosed with brain death. The nurse walker. The nurse determines that the client is
determines that the standard of care had been using the walker correctly if the client:
maintained if which of the following data is a. Puts all the four points of the walker flat
observed? on the floor, puts weight on the hand
a. Urine output: 45 ml/hr pieces, and then walks into it.
b. Capillary refill: 5 seconds b. Puts weight on the hand pieces, moves
c. Serum pH: 7.32 the walker forward, and then walks into
d. Blood pressure: 90/48 mmHg it.
c. Puts weight on the hand pieces, slides
58. Nurse Amy has an order to obtain a urinalysis the walker forward, and then walks into
from a male client with an indwelling urinary it.
catheter. The nurse avoids which of the d. Walks into the walker, puts weight on
following, which contaminate the specimen? the hand pieces, and then puts all four
a. Wiping the port with an alcohol swab points of the walker flat on the floor.
before inserting the syringe.
b. Aspirating a sample from the port on the 62. Nurse Amy has documented an entry regarding
drainage bag. client care in the client’s medical record. When
c. Clamping the tubing of the drainage bag. checking the entry, the nurse realizes that

26
26
incorrect information was documented. How a. Prone with head turned toward the side
does the nurse correct this error? supported by a pillow.
a. Erases the error and writes in the correct b. Sims’ position with the head of the bed
information. flat.
b. Uses correction fluid to cover up the c. Right side-lying with the head of the bed
incorrect information and writes in the elevated 45 degrees.
correct information. d. Left side-lying with the head of the bed
c. Draws one line to cross out the incorrect elevated 45 degrees.
information and then initials the change.
d. Covers up the incorrect information 67. Nurse John develops methods for data
completely using a black pen and writes gathering. Which of the following criteria of a
in the correct information good instrument refers to the ability of the
instrument to yield the same results upon its
63. Nurse Ron is assisting with transferring a client repeated administration?
from the operating room table to a stretcher. To a. Validity
provide safety to the client, the nurse should: b. Specificity
a. Moves the client rapidly from the table c. Sensitivity
to the stretcher. d. Reliability
b. Uncovers the client completely before
transferring to the stretcher. 68. Harry knows that he has to protect the rights of
c. Secures the client safety belts after human research subjects. Which of the following
transferring to the stretcher. actions of Harry ensures anonymity?
d. Instructs the client to move self from the a. Keep the identities of the subject secret
table to the stretcher. b. Obtain informed consent
c. Provide equal treatment to all the
64. Nurse Myrna is providing instructions to a subjects of the study.
nursing assistant assigned to give a bed bath to a d. Release findings only to the participants
client who is on contact precautions. Nurse of the study
Myrna instructs the nursing assistant to use
which of the following protective items when 69. Patient’s refusal to divulge information is a
giving bed bath? limitation because it is beyond the control of
a. Gown and goggles Tifanny”. What type of research is appropriate
b. Gown and gloves for this study?
c. Gloves and shoe protectors a. Descriptive- correlational
d. Gloves and goggles b. Experiment
c. Quasi-experiment
65. Nurse Oliver is caring for a client with impaired d. Historical
mobility that occurred as a result of a stroke. The
client has right sided arm and leg weakness. The 70. Nurse Ronald is aware that the best tool for data
nurse would suggest that the client use which of gathering is?
the following assistive devices that would a. Interview schedule
provide the best stability for ambulating? b. Questionnaire
a. Crutches c. Use of laboratory data
b. Single straight-legged cane d. Observation
c. Quad cane
d. Walker 71. Monica is aware that there are times when only
manipulation of study variables is possible and
66. A male client with a right pleural effusion noted the elements of control or randomization are
on a chest X-ray is being prepared for not attendant. Which type of research is
thoracentesis. The client experiences severe referred to this?
dizziness when sitting upright. To provide a safe a. Field study
environment, the nurse assists the client to b. Quasi-experiment
which position for the procedure? c. Solomon-Four group design
d. Post-test only design d. Will remain unable to practice
professional nursing
72. Cherry notes down ideas that were derived from
the description of an investigation written by the 77. Ronald plans to conduct a research on the use of
person who conducted it. Which type of a new method of pain assessment scale. Which
reference source refers to this? of the following is the second step in the
a. Footnote conceptualizing phase of the research process?
b. Bibliography a. Formulating the research hypothesis
c. Primary source b. Review related literature
d. Endnotes c. Formulating and delimiting the research
problem
73. When Nurse Trish is providing care to his d. Design the theoretical and conceptual
patient, she must remember that her duty is framework
bound not to do doing any action that will cause
the patient harm. This is the meaning of the 78. The leader of the study knows that certain
bioethical principle: patients who are in a specialized research setting
a. Non-maleficence tend to respond psychologically to the
b. Beneficence conditions of the study. This referred to as :
c. Justice a. Cause and effect
d. Solidarity b. Hawthorne effect
c. Halo effect
74. When a nurse in-charge causes an injury to a d. Horns effect
female patient and the injury caused becomes
the proof of the negligent act, the presence of 79. Mary finally decides to use judgment sampling
the injury is said to exemplify the principle of: on her research. Which of the following actions
a. Force majeure of is correct?
b. Respondeat superior a. Plans to include whoever is there during
c. Res ipsa loquitor his study.
d. Holdover doctrine b. Determines the different nationality of
patients frequently admitted and
75. Nurse Myrna is aware that the Board of Nursing decides to get representations samples
has quasi-judicial power. An example of this from each.
power is: c. Assigns numbers for each of the
a. The Board can issue rules and patients, place these in a fishbowl and
regulations that will govern the practice draw 10 from it.
of nursing d. Decides to get 20 samples from the
b. The Board can investigate violations of admitted patients
the nursing law and code of ethics
c. The Board can visit a school applying for 80. The nursing theorist who developed
a permit in collaboration with CHED transcultural nursing theory is:
d. The Board prepares the board a. Florence Nightingale
examinations b. Madeleine Leininger
c. Albert Moore
76. When the license of nurse Krina is revoked, it d. Sr. Callista Roy
means that she:
a. Is no longer allowed to practice the 81. Marion is aware that the sampling method that
profession for the rest of her life gives equal chance to all units in the population
b. Will never have her/his license re-issued to get picked is:
since it has been revoked a. Random
c. May apply for re-issuance of his/her b. Accidental
license based on certain conditions c. Quota
stipulated in RA 9173 d. Judgment

26
26
82. John plans to use a Likert Scale to his study to
determine the: 89. Nurse Marian is preparing to administer a blood
a. Degree of agreement and disagreement transfusion. Which action should the nurse take
b. Compliance to expected standards first?
c. Level of satisfaction a. Arrange for typing and cross matching of
d. Degree of acceptance the client’s blood.
b. Compare the client’s identification
83. Which of the following theory addresses the four wristband with the tag on the unit of
modes of adaptation? blood.
a. Madeleine Leininger c. Start an I.V. infusion of normal saline
b. Sr. Callista Roy solution.
c. Florence Nightingale d. Measure the client’s vital signs.
d. Jean Watson
90. A 65 years old male client requests his
84. Ms. Garcia is responsible to the number of medication at 9 p.m. instead of 10 p.m. so that
personnel reporting to her. This principle refers he can go to sleep earlier. Which type of nursing
to: intervention is required?
a. Span of control a. Independent
b. Unity of command b. Dependent
c. Downward communication c. Interdependent
d. Leader d. Intradependent

85. Ensuring that there is an informed consent on 91. A female client is to be discharged from an acute
the part of the patient before a surgery is done, care facility after treatment for right leg
illustrates the bioethical principle of: thrombophlebitis. The Nurse Betty notes that
a. Beneficence the client's leg is pain-free, without redness or
b. Autonomy edema. The nurse's actions reflect which step of
c. Veracity the nursing process?
d. Non-maleficence a. Assessment
b. Diagnosis
86. Nurse Reese is teaching a female client with c. Implementation
peripheral vascular disease about foot care; d. Evaluation
Nurse Reese should include which instruction?
a. Avoid wearing cotton socks. 92. Nursing care for a female client includes
b. Avoid using a nail clipper to cut toenails. removing elastic stockings once per day. The
c. Avoid wearing canvas shoes. Nurse Betty is aware that the rationale for this
d. Avoid using cornstarch on feet. intervention?
a. To increase blood flow to the heart
87. A client is admitted with multiple pressure b. To observe the lower extremities
ulcers. When developing the client's diet plan, c. To allow the leg muscles to stretch and
the nurse should include: relax
a. Fresh orange slices d. To permit veins in the legs to fill with
b. Steamed broccoli blood.
c. Ice cream
d. Ground beef patties 93. Which nursing intervention takes highest priority
when caring for a newly admitted client who's
88. The nurse prepares to administer a cleansing receiving a blood transfusion?
enema. What is the most common client a. Instructing the client to report any
position used for this procedure? itching, swelling, or dyspnea.
a. Lithotomy b. Informing the client that the transfusion
b. Supine usually take 1 ½ to 2 hours.
c. Prone c. Documenting blood administration in
d. Sims’ left lateral the client care record.
d. Assessing the client’s vital signs when d. 30 minutes after administering the next
the transfusion ends. dose.

94. A male client complains of abdominal discomfort 99. Nurse May is aware that the main advantage of
and nausea while receiving tube feedings. Which using a floor stock system is:
intervention is most appropriate for this a. The nurse can implement medication
problem? orders quickly.
a. Give the feedings at room temperature. b. The nurse receives input from the
b. Decrease the rate of feedings and the pharmacist.
concentration of the formula. c. The system minimizes transcription
c. Place the client in semi-Fowler's position errors.
while feeding. d. The system reinforces accurate
d. Change the feeding container every 12 calculations.
hours.
100. Nurse Oliver is assessing a client's abdomen.
95. Nurse Patricia is reconstituting a powdered Which finding should the nurse report as
medication in a vial. After adding the solution to abnormal?
the powder, she nurse should: a. Dullness over the liver.
a. Do nothing. b. Bowel sounds occurring every 10
b. Invert the vial and let it stand for 3 to 5 seconds.
minutes. c. Shifting dullness over the abdomen.
c. Shake the vial vigorously. d. Vascular sounds heard over the renal
d. Roll the vial gently between the palms. arteries.

96. Which intervention should the nurse Trish use


when administering oxygen by face mask to a
female client?
a. Secure the elastic band tightly around
the client's head.
b. Assist the client to the semi-Fowler
position if possible.
c. Apply the face mask from the client's
chin up over the nose.
d. Loosen the connectors between the
oxygen equipment and humidifier.

97. The maximum transfusion time for a unit of


packed red blood cells (RBCs) is:
a. 6 hours
b. 4 hours
c. 3 hours
d. 2 hours

98. Nurse Monique is monitoring the effectiveness


of a client's drug therapy. When should the
nurse Monique obtain a blood sample to
measure the trough drug level?
a. 1 hour before administering the next
dose.
b. Immediately before administering the
next dose.
c. Immediately after administering the
next dose.

26
26
Answers and Rationale – Foundation of The best treatment for this prophylactic use of
Professional Nursing Practice antacids and H2 receptor blockers.
8. Answer: (D) Continue to monitor and record
1. Answer: (D) The actions of a reasonably prudent hourly urine output
nurse with similar education and experience. Rationale: Normal urine output for an adult is
Rationale: The standard of care is determined approximately 1 ml/minute (60 ml/hour).
by the average degree of skill, care, and Therefore, this client's output is normal.
diligence by nurses in similar circumstances. Beyond continued evaluation, no nursing
2. Answer: (B) I.M action is warranted.
Rationale: With a platelet count of 22,000/μl, 9. Answer: (B) “My ankle feels warm”.
the clients tends to bleed easily. Therefore, Rationale: Ice application decreases pain and
the nurse should avoid using the I.M. route swelling. Continued or increased pain, redness,
because the area is a highly vascular and can and increased warmth are signs of
bleed readily when penetrated by a needle. inflammation that shouldn't occur after ice
The bleeding can be difficult to stop. application
3. Answer: (C) “Digoxin 0.125 mg P.O. once daily” 10. Answer: (B) Hyperkalemia
Rationale: The nurse should always place a Rationale: A loop diuretic removes water and,
zero before a decimal point so that no one along with it, sodium and potassium. This may
misreads the figure, which could result in a result in hypokalemia, hypovolemia, and
dosage error. The nurse should never insert a hyponatremia.
zero at the end of a dosage that includes a 11. Answer:(A) Have condescending trust and
decimal point because this could be misread, confidence in their subordinates
possibly leading to a tenfold increase in the Rationale: Benevolent-authoritative managers
dosage. pretentiously show their trust and confidence
4. Answer: (A) Ineffective peripheral tissue to their followers.
perfusion related to venous congestion. 12. Answer: (A) Provides continuous, coordinated
Rationale: Ineffective peripheral tissue and comprehensive nursing services.
perfusion related to venous congestion takes Rationale: Functional nursing is focused on
the highest priority because venous tasks and activities and not on the care of the
inflammation and clot formation impede blood patients.
flow in a client with deep vein thrombosis. 13. Answer: (B) Standard written order
5. Answer: (B) A 44 year-old myocardial Rationale: This is a standard written order.
infarction (MI) client who is complaining of Prescribers write a single order for
nausea. medications given only once. A stat order is
Rationale: Nausea is a symptom of impending written for medications given immediately for
myocardial infarction (MI) and should be an urgent client problem. A standing order,
assessed immediately so that treatment can also known as a protocol, establishes
be instituted and further damage to the heart guidelines for treating a particular disease or
is avoided. set of symptoms in special care areas such as
6. Answer: (C) Check circulation every 15-30 the coronary care unit. Facilities also may
minutes. institute medication protocols that specifically
Rationale: Restraints encircle the limbs, which designate drugs that a nurse may not give.
place the client at risk for circulation being 14. Answer: (D) Liquid or semi-liquid stools
restricted to the distal areas of the Rationale: Passage of liquid or semi-liquid
extremities. Checking the client’s circulation stools results from seepage of unformed
every 15-30 minutes will allow the nurse to bowel contents around the impacted stool in
adjust the restraints before injury from the rectum. Clients with fecal impaction don't
decreased blood flow occurs. pass hard, brown, formed stools because the
7. Answer: (A) Prevent stress ulcer feces can't move past the impaction. These
Rationale: Curling’s ulcer occurs as a clients typically report the urge to defecate
generalized stress response in burn patients. (although they can't pass stool) and a
This results in a decreased production of decreased appetite.
mucus and increased secretion of gastric acid. 15. Answer: (C) Pulling the helix up and back
Rationale: To perform an otoscopic Rationale: Agranulocytosis is characterized by
examination on an adult, the nurse grasps the a reduced number of leukocytes (leucopenia)
helix of the ear and pulls it up and back to and neutrophils (neutropenia) in the blood.
straighten the ear canal. For a child, the nurse The client is at high risk for infection because
grasps the helix and pulls it down to straighten of the decreased body defenses against
the ear canal. Pulling the lobule in any microorganisms. Deficient knowledge related
direction wouldn't straighten the ear canal for to the nature of the disorder may be
visualization. appropriate diagnosis but is not the priority.
16. Answer: (A) Protect the irritated skin from 24. Answer: (B) Place the client on the left side in
sunlight. the Trendelenburg position.
Rationale: Irradiated skin is very sensitive and Rationale: Lying on the left side may prevent
must be protected with clothing or sunblock. air from flowing into the pulmonary veins. The
The priority approach is the avoidance of Trendelenburg position increases intrathoracic
strong sunlight. pressure, which decreases the amount of
17. Answer: (C) Assist the client in removing blood pulled into the vena cava during
dentures and nail polish. aspiration.
Rationale: Dentures, hairpins, and combs must 25. Answer: (A) Autocratic.
be removed. Nail polish must be removed so Rationale: The autocratic style of leadership is
that cyanosis can be easily monitored by a task-oriented and directive.
observing the nail beds. 26. Answer: (D) 2.5 cc
18. Answer: (D) Sudden onset of continuous Rationale: 2.5 cc is to be added, because only a
epigastric and back pain. 500 cc bag of solution is being medicated
Rationale: The autodigestion of tissue by the instead of a 1 liter.
pancreatic enzymes results in pain from 27. Answer: (A) 50 cc/ hour
inflammation, edema, and possible Rationale: A rate of 50 cc/hr. The child is to
hemorrhage. Continuous, unrelieved epigastric receive 400 cc over a period of 8 hours = 50
or back pain reflects the inflammatory process cc/hr.
in the pancreas. 28. Answer: (B) Assess the client for presence of
19. Answer: (B) Provide high-protein, high- pain.
carbohydrate diet. Rationale: Assessing the client for pain is a
Rationale: A positive nitrogen balance is very important measure. Postoperative pain is
important for meeting metabolic needs, tissue an indication of complication. The nurse
repair, and resistance to infection. Caloric should also assess the client for pain to
goals may be as high as 5000 calories per day. provide for the client’s comfort.
20. Answer: (A) Blood pressure and pulse rate. 29. Answer: (A) BP – 80/60, Pulse – 110 irregular
Rationale: The baseline must be established to Rationale: The classic signs of cardiogenic
recognize the signs of an anaphylactic or shock are low blood pressure, rapid and weak
hemolytic reaction to the transfusion. irregular pulse, cold, clammy skin, decreased
21. Answer: (D) Immobilize the leg before moving urinary output, and cerebral hypoxia.
the client. 30. Answer: (A) Take the proper equipment, place
Rationale: If the nurse suspects a fracture, the client in a comfortable position, and
splinting the area before moving the client is record the appropriate information in the
imperative. The nurse should call for client’s chart.
emergency help if the client is not hospitalized Rationale: It is a general or comprehensive
and call for a physician for the hospitalized statement about the correct procedure, and it
client. includes the basic ideas which are found in the
22. Answer: (B) Admit the client into a private other options
room. 31. Answer: (B) Evaluation
Rationale: The client who has a radiation Rationale: Evaluation includes observing the
implant is placed in a private room and has a person, asking questions, and comparing the
limited number of visitors. This reduces the patient’s behavioral responses with the
exposure of others to the radiation. expected outcomes.
23. Answer: (C) Risk for infection 32. Answer: (C) History of present illness

26
26
Rationale: The history of present illness is the 41. Answer: (A) Checking and taping all
single most important factor in assisting the connections
health professional in arriving at a diagnosis or Rationale: Air leaks commonly occur if the
determining the person’s needs. system isn’t secure. Checking all connections
33. Answer: (A) Trochanter roll extending from the and taping them will prevent air leaks. The
crest of the ileum to the mid-thigh. chest drainage system is kept lower to
Rationale: A trochanter roll, properly placed, promote drainage – not to prevent leaks.
provides resistance to the external rotation of 42. Answer: (A) Check the client’s identification
the hip. band.
34. Answer: (C) Stage III Rationale: Checking the client’s identification
Rationale: Clinically, a deep crater or without band is the safest way to verify a client’s
undermining of adjacent tissue is noted. identity because the band is assigned on
35. Answer: (A) Second intention healing admission and isn’t be removed at any time. (If
Rationale: When wounds dehisce, they will it is removed, it must be replaced). Asking the
allowed to heal by secondary Intention client’s name or having the client repeated his
36. Answer: (D) Tachycardia name would be appropriate only for a client
Rationale: With an extracellular fluid or plasma who’s alert, oriented, and able to understand
volume deficit, compensatory mechanisms what is being said, but isn’t the safe standard
stimulate the heart, causing an increase in of practice. Names on bed aren’t always
heart rate. reliable
37. Answer: (A) 0.75 43. Answer: (B) 32 drops/minute
Rationale: To determine the number of Rationale: Giving 1,000 ml over 8 hours is the
milliliters the client should receive, the nurse same as giving 125 ml over 1 hour (60
uses the fraction method in the following minutes). Find the number of milliliters per
equation. minute as follows:
75 mg/X ml = 100 mg/1 ml 125/60 minutes = X/1 minute
To solve for X, cross-multiply: 60X = 125 = 2.1 ml/minute
75 mg x 1 ml = X ml x 100 mg To find the number of drops per minute:
75 = 100X 2.1 ml/X gtt = 1 ml/ 15 gtt
75/100 = X X = 32 gtt/minute, or 32 drops/minute
0.75 ml (or ¾ ml) = X 44. Answer: (A) Clamp the catheter
38. Answer: (D) it’s a measure of effect, not a Rationale: If a central venous catheter
standard measure of weight or quantity. becomes disconnected, the nurse should
Rationale: An insulin unit is a measure of immediately apply a catheter clamp, if
effect, not a standard measure of weight or available. If a clamp isn’t available, the nurse
quantity. Different drugs measured in units can place a sterile syringe or catheter plug in
may have no relationship to one another in the catheter hub. After cleaning the hub with
quality or quantity. alcohol or povidone-iodine solution, the nurse
39. Answer: (B) 38.9 °C must replace the I.V. extension and restart the
Rationale: To convert Fahrenheit degreed to infusion.
Centigrade, use this formula 45. Answer: (D) Auscultation, percussion, and
°C = (°F – 32) ÷ 1.8 palpation.
°C = (102 – 32) ÷ 1.8 Rationale: The correct order of assessment for
°C = 70 ÷ 1.8 examining the abdomen is inspection,
°C = 38.9 auscultation, percussion, and palpation. The
40. Answer: (C) Failing eyesight, especially close reason for this approach is that the less
vision. intrusive techniques should be performed
Rationale: Failing eyesight, especially close before the more intrusive techniques.
vision, is one of the first signs of aging in Percussion and palpation can alter natural
middle life (ages 46 to 64). More frequent findings during auscultation.
aches and pains begin in the early late years 46. Answer: (D) Ulnar surface of the hand
(ages 65 to 79). Increase in loss of muscle tone Rationale: The nurse uses the ulnar surface, or
occurs in later years (age 80 and older). ball, of the hand to assess tactile fremitus,
thrills, and vocal vibrations through the chest Rationale: An elastic bandage should be
wall. The fingertips and finger pads best applied form the distal area to the proximal
distinguish texture and shape. The dorsal area. This method promotes venous return. In
surface best feels warmth. this case, the nurse should begin applying the
47. Answer: (C) Formative bandage at the client’s foot. Beginning at the
Rationale: Formative (or concurrent) ankle, lower thigh, or knee does not promote
evaluation occurs continuously throughout the venous return.
teaching and learning process. One benefit is 53. Answer: (B) Hypokalemia
that the nurse can adjust teaching strategies Rationale: Insulin administration causes
as necessary to enhance learning. Summative, glucose and potassium to move into the cells,
or retrospective, evaluation occurs at the causing hypokalemia.
conclusion of the teaching and learning 54. Answer: (A) Throbbing headache or dizziness
session. Informative is not a type of Rationale: Headache and dizziness often occur
evaluation. when nitroglycerin is taken at the beginning of
48. Answer: (B) Once per year therapy. However, the client usually develops
Rationale: Yearly mammograms should begin tolerance
at age 40 and continue for as long as the 55. Answer: (D) Check the client’s level of
woman is in good health. If health risks, such consciousness
as family history, genetic tendency, or past Rationale: Determining unresponsiveness is
breast cancer, exist, more frequent the first step assessment action to take. When
examinations may be necessary. a client is in ventricular tachycardia, there is a
49. Answer: (A) Respiratory acidosis significant decrease in cardiac output.
Rationale: The client has a below-normal However, checking the unresponsiveness
(acidic) blood pH value and an above-normal ensures whether the client is affected by the
partial pressure of arterial carbon dioxide decreased cardiac output.
(Paco2) value, indicating respiratory acidosis. 56. Answer: (B) On the affected side of the client.
In respiratory alkalosis, the pH value is above Rationale: When walking with clients, the
normal and in the Paco2 value is below nurse should stand on the affected side and
normal. In metabolic acidosis, the pH and grasp the security belt in the midspine area of
bicarbonate (Hco3) values are below normal. the small of the back. The nurse should
In metabolic alkalosis, the pH and Hco3 values position the free hand at the shoulder area so
are above normal. that the client can be pulled toward the nurse
50. Answer: (B) To provide support for the client in the event that there is a forward fall. The
and family in coping with terminal illness. client is instructed to look up and outward
Rationale: Hospices provide supportive care rather than at his or her feet.
for terminally ill clients and their families. 57. Answer: (A) Urine output: 45 ml/hr
Hospice care doesn’t focus on counseling Rationale: Adequate perfusion must be
regarding health care costs. Most client maintained to all vital organs in order for the
referred to hospices have been treated for client to remain visible as an organ donor. A
their disease without success and will receive urine output of 45 ml per hour indicates
only palliative care in the hospice. adequate renal perfusion. Low blood pressure
51. Answer: (C) Using normal saline solution to and delayed capillary refill time are circulatory
clean the ulcer and applying a protective system indicators of inadequate perfusion. A
dressing as necessary. serum pH of 7.32 is acidotic, which adversely
Rationale: Washing the area with normal affects all body tissues.
saline solution and applying a protective 58. Answer: (D ) Obtaining the specimen from the
dressing are within the nurse’s realm of urinary drainage bag.
interventions and will protect the area. Using a Rationale: A urine specimen is not taken from
povidone-iodine wash and an antibiotic cream the urinary drainage bag. Urine undergoes
require a physician’s order. Massaging with an chemical changes while sitting in the bag and
astringent can further damage the skin. does not necessarily reflect the current client
52. Answer: (D) Foot status. In addition, it may become

26
26
contaminated with bacteria from opening the the stretcher, the client is still affected by the
system. effects of the anesthesia; therefore, the client
59. Answer: (B) Cover the client, place the call should not move self. Safety belts can prevent
light within reach, and answer the phone call. the client from falling off the stretcher.
Rationale: Because telephone call is an 64. Answer: (B) Gown and gloves
emergency, the nurse may need to answer it. Rationale: Contact precautions require the use
The other appropriate action is to ask another of gloves and a gown if direct client contact is
nurse to accept the call. However, is not one of anticipated. Goggles are not necessary unless
the options. To maintain privacy and safety, the nurse anticipates the splashes of blood,
the nurse covers the client and places the call body fluids, secretions, or excretions may
light within the client’s reach. Additionally, the occur. Shoe protectors are not necessary.
client’s door should be closed or the room 65. Answer: (C) Quad cane
curtains pulled around the bathing area. Rationale: Crutches and a walker can be
60. Answer: (C) Use a sterile plastic container for difficult to maneuver for a client with
obtaining the specimen. weakness on one side. A cane is better suited
Rationale: Sputum specimens for culture and for client with weakness of the arm and leg on
sensitivity testing need to be obtained using one side. However, the quad cane would
sterile techniques because the test is done to provide the most stability because of the
determine the presence of organisms. If the structure of the cane and because a quad cane
procedure for obtaining the specimen is not has four legs.
sterile, then the specimen is not sterile, then 66. Answer: (D) Left side-lying with the head of
the specimen would be contaminated and the the bed elevated 45 degrees.
results of the test would be invalid. Rationale: To facilitate removal of fluid from
61. Answer: (A) Puts all the four points of the the chest wall, the client is positioned sitting at
walker flat on the floor, puts weight on the the edge of the bed leaning over the bedside
hand pieces, and then walks into it. table with the feet supported on a stool. If the
Rationale: When the client uses a walker, the client is unable to sit up, the client is
nurse stands adjacent to the affected side. The positioned lying in bed on the unaffected side
client is instructed to put all four points of the with the head of the bed elevated 30 to 45
walker 2 feet forward flat on the floor before degrees.
putting weight on hand pieces. This will ensure 67. Answer: (D) Reliability
client safety and prevent stress cracks in the Rationale: Reliability is consistency of the
walker. The client is then instructed to move research instrument. It refers to the
the walker forward and walk into it. repeatability of the instrument in extracting
62. Answer: (C) Draws one line to cross out the the same responses upon its repeated
incorrect information and then initials the administration.
change. 68. Answer: (A) Keep the identities of the subject
Rationale: To correct an error documented in a secret
medical record, the nurse draws one line Rationale: Keeping the identities of the
through the incorrect information and then research subject secret will ensure anonymity
initials the error. An error is never erased and because this will hinder providing link between
correction fluid is never used in the medical the information given to whoever is its source.
record. 69. Answer: (A) Descriptive- correlational
63. Answer: (C) Secures the client safety belts Rationale: Descriptive- correlational study is
after transferring to the stretcher. the most appropriate for this study because
Rationale: During the transfer of the client it studies the variables that could be the
after the surgical procedure is complete, the antecedents of the increased incidence of
nurse should avoid exposure of the client nosocomial infection.
because of the risk for potential heat loss. 70. Answer: (C) Use of laboratory data
Hurried movements and rapid changes in the Rationale: Incidence of nosocomial infection is
position should be avoided because these best collected through the use of
predispose the client to hypotension. At the biophysiologic measures, particularly in vitro
time of the transfer from the surgery table to
measurements, hence laboratory data is observed. They performed differently because
essential. they were under observation.
71. Answer: (B) Quasi-experiment 79. Answer: (B) Determines the different
Rationale: Quasi-experiment is done when nationality of patients frequently admitted and
randomization and control of the variables are decides to get representations samples from
not possible. each.
72. Answer: (C) Primary source Rationale: Judgment sampling involves
Rationale: This refers to a primary source including samples according to the knowledge
which is a direct account of the investigation of the investigator about the participants in
done by the investigator. In contrast to this is a the study.
secondary source, which is written by 80. Answer: (B) Madeleine Leininger
someone other than the original researcher. Rationale: Madeleine Leininger developed the
73. Answer: (A) Non-maleficence theory on transcultural theory based on her
Rationale: Non-maleficence means do not observations on the behavior of selected
cause harm or do any action that will cause people within a culture.
any harm to the patient/client. To do good is 81. Answer: (A) Random
referred as beneficence. Rationale: Random sampling gives equal
74. Answer: (C) Res ipsa loquitor chance for all the elements in the population
Rationale: Res ipsa loquitor literally means the to be picked as part of the sample.
thing speaks for itself. This means in 82. Answer: (A) Degree of agreement and
operational terms that the injury caused is the disagreement
proof that there was a negligent act. Rationale: Likert scale is a 5-point summated
75. Answer: (B) The Board can investigate scale used to determine the degree of
violations of the nursing law and code of ethics agreement or disagreement of the
Rationale: Quasi-judicial power means that the respondents to a statement in a study
Board of Nursing has the authority to 83. Answer: (B) Sr. Callista Roy
investigate violations of the nursing law and Rationale: Sr. Callista Roy developed the
can issue summons, subpoena or subpoena Adaptation Model which involves the
duces tecum as needed. physiologic mode, self-concept mode, role
76. Answer: (C) May apply for re-issuance of function mode and dependence mode.
his/her license based on certain conditions 84. Answer: (A) Span of control
stipulated in RA 9173 Rationale: Span of control refers to the
Rationale: RA 9173 sec. 24 states that for number of workers who report directly to a
equity and justice, a revoked license maybe re- manager.
issued provided that the following conditions 85. Answer: (B) Autonomy
are met: a) the cause for revocation of license Rationale: Informed consent means that the
has already been corrected or removed; and, patient fully understands about the surgery,
b) at least four years has elapsed since the including the risks involved and the alternative
license has been revoked. solutions. In giving consent it is done with full
77. Answer: (B) Review related literature knowledge and is given freely. The action of
Rationale: After formulating and delimiting the allowing the patient to decide whether a
research problem, the researcher conducts a surgery is to be done or not exemplifies the
review of related literature to determine the bioethical principle of autonomy.
extent of what has been done on the study by 86. Answer: (C) Avoid wearing canvas shoes.
previous researchers. Rationale: The client should be instructed to
78. Answer: (B) Hawthorne effect avoid wearing canvas shoes. Canvas shoes
Rationale: Hawthorne effect is based on the cause the feet to perspire, which may, in turn,
study of Elton Mayo and company about the cause skin irritation and breakdown. Both
effect of an intervention done to improve the cotton and cornstarch absorb perspiration.
working conditions of the workers on their The client should be instructed to cut toenails
productivity. It resulted to an increased straight across with nail clippers.
productivity but not due to the intervention 87. Answer: (D) Ground beef patties
but due to the psychological effects of being

27
27
Rationale: Meat is an excellent source of Rationale: Elastic stockings are used to
complete protein, which this client needs to promote venous return. The nurse needs to
repair the tissue breakdown caused by remove them once per day to observe the
pressure ulcers. Oranges and broccoli supply condition of the skin underneath the stockings.
vitamin C but not protein. Ice cream supplies Applying the stockings increases blood flow to
only some incomplete protein, making it less the heart. When the stockings are in place, the
helpful in tissue repair. leg muscles can still stretch and relax, and the
88. Answer: (D) Sims’ left lateral veins can fill with blood.
Rationale: The Sims' left lateral position is the 93. Answer :(A) Instructing the client to report any
most common position used to administer a itching, swelling, or dyspnea.
cleansing enema because it allows gravity to Rationale: Because administration of blood or
aid the flow of fluid along the curve of the blood products may cause serious adverse
sigmoid colon. If the client can't assume this effects such as allergic reactions, the nurse
position nor has poor sphincter control, the must monitor the client for these effects. Signs
dorsal recumbent or right lateral position may and symptoms of life-threatening allergic
be used. The supine and prone positions are reactions include itching, swelling, and
inappropriate and uncomfortable for the dyspnea. Although the nurse should inform
client. the client of the duration of the transfusion
89. Answer: (A) Arrange for typing and cross and should document its administration, these
matching of the client’s blood. actions are less critical to the client's
Rationale: The nurse first arranges for typing immediate health. The nurse should assess
and cross matching of the client's blood to vital signs at least hourly during the
ensure compatibility with donor blood. The transfusion.
other options, although appropriate when 94. Answer: (B) Decrease the rate of feedings and
preparing to administer a blood transfusion, the concentration of the formula.
come later. Rationale: Complaints of abdominal
90. Answer: (A) Independent discomfort and nausea are common in clients
Rationale: Nursing interventions are classified receiving tube feedings. Decreasing the rate of
as independent, interdependent, or the feeding and the concentration of the
dependent. Altering the drug schedule to formula should decrease the client's
coincide with the client's daily routine discomfort. Feedings are normally given at
represents an independent intervention, room temperature to minimize abdominal
whereas consulting with the physician and cramping. To prevent aspiration during
pharmacist to change a client's medication feeding, the head of the client's bed should be
because of adverse reactions represents an elevated at least 30 degrees. Also, to prevent
interdependent intervention. Administering an bacterial growth, feeding containers should be
already-prescribed drug on time is a routinely changed every 8 to 12 hours.
dependent intervention. An intradependent 95. Answer: (D) Roll the vial gently between the
nursing intervention doesn't exist. palms.
91. Answer: (D) Evaluation Rationale: Rolling the vial gently between the
Rationale: The nursing actions described palms produces heat, which helps dissolve the
constitute evaluation of the expected medication. Doing nothing or inverting the vial
outcomes. The findings show that the wouldn't help dissolve the medication. Shaking
expected outcomes have been achieved. the vial vigorously could cause the medication
Assessment consists of the client's history, to break down, altering its action.
physical examination, and laboratory studies. 96. Answer: (B) Assist the client to the semi-
Analysis consists of considering assessment Fowler position if possible.
information to derive the appropriate nursing Rationale: By assisting the client to the semi-
diagnosis. Implementation is the phase of the Fowler position, the nurse promotes easier
nursing process where the nurse puts the plan chest expansion, breathing, and oxygen intake.
of care into action. The nurse should secure the elastic band so
92. Answer: (B) To observe the lower extremities that the face mask fits comfortably and snugly
rather than tightly, which could lead to
irritation. The nurse should apply the face
mask from the client's nose down to the chin
— not vice versa. The nurse should check the
connectors between the oxygen equipment
and humidifier to ensure that they're airtight;
loosened connectors can cause loss of oxygen.
97. Answer: (B) 4 hours
Rationale: A unit of packed RBCs may be given
over a period of between 1 and 4 hours. It
shouldn't infuse for longer than 4 hours
because the risk of contamination and sepsis
increases after that time. Discard or return to
the blood bank any blood not given within this
time, according to facility policy.
98. Answer: (B) Immediately before administering
the next dose.
Rationale: Measuring the blood drug
concentration helps determine whether the
dosing has achieved the therapeutic goal. For
measurement of the trough, or lowest, blood
level of a drug, the nurse draws a blood
sample immediately before administering the
next dose. Depending on the drug's duration
of action and half-life, peak blood drug levels
typically are drawn after administering the
next dose.
99. Answer: (A) The nurse can implement
medication orders quickly.
Rationale: A floor stock system enables the
nurse to implement medication orders quickly.
It doesn't allow for pharmacist input, nor does
it minimize transcription errors or reinforce
accurate calculations.
100. Answer: (C) Shifting dullness over the
abdomen.
Rationale: Shifting dullness over the abdomen
indicates ascites, an abnormal finding. The
other options are normal abdominal findings.

27
27
TEST II - Community Health Nursing and Care of a. Excessive fetal activity.
the Mother and Child b. Larger than normal uterus for
gestational age.
1. May arrives at the health care clinic and tells the c. Vaginal bleeding
nurse that her last menstrual period was 9 d. Elevated levels of human chorionic
weeks ago. She also tells the nurse that a home gonadotropin.
pregnancy test was positive but she began to
have mild cramps and is now having moderate 6. A pregnant client is receiving magnesium sulfate
vaginal bleeding. During the physical for severe pregnancy induced hypertension
examination of the client, the nurse notes that (PIH). The clinical findings that would warrant
May has a dilated cervix. The nurse determines use of the antidote , calcium gluconate is:
that May is experiencing which type of abortion? a. Urinary output 90 cc in 2 hours.
a. Inevitable b. Absent patellar reflexes.
b. Incomplete c. Rapid respiratory rate above 40/min.
c. Threatened d. Rapid rise in blood pressure.
d. Septic
7. During vaginal examination of Janah who is in
2. Nurse Reese is reviewing the record of a labor, the presenting part is at station plus two.
pregnant client for her first prenatal visit. Which Nurse, correctly interprets it as:
of the following data, if noted on the client’s a. Presenting part is 2 cm above the plane
record, would alert the nurse that the client is at of the ischial spines.
risk for a spontaneous abortion? b. Biparietal diameter is at the level of the
a. Age 36 years ischial spines.
b. History of syphilis c. Presenting part in 2 cm below the plane
c. History of genital herpes of the ischial spines.
d. History of diabetes mellitus d. Biparietal diameter is 2 cm above the
ischial spines.
3. Nurse Hazel is preparing to care for a client who
is newly admitted to the hospital with a possible 8. A pregnant client is receiving oxytocin (Pitocin)
diagnosis of ectopic pregnancy. Nurse Hazel for induction of labor. A condition that warrant
develops a plan of care for the client and the nurse in-charge to discontinue I.V. infusion
determines that which of the following nursing of Pitocin is:
actions is the priority? a. Contractions every 1 ½ minutes lasting
a. Monitoring weight 70-80 seconds.
b. Assessing for edema b. Maternal temperature 101.2
c. Monitoring apical pulse c. Early decelerations in the fetal heart
d. Monitoring temperature rate.
d. Fetal heart rate baseline 140-160 bpm.
4. Nurse Oliver is teaching a diabetic pregnant
client about nutrition and insulin needs during 9. Calcium gluconate is being administered to a
pregnancy. The nurse determines that the client client with pregnancy induced hypertension
understands dietary and insulin needs if the (PIH). A nursing action that must be initiated as
client states that the second half of pregnancy the plan of care throughout injection of the drug
requires: is:
a. Decreased caloric intake a. Ventilator assistance
b. Increased caloric intake b. CVP readings
c. Decreased Insulin c. EKG tracings
d. Increase Insulin d. Continuous CPR

5. Nurse Michelle is assessing a 24 year old client 10. A trial for vaginal delivery after an earlier
with a diagnosis of hydatidiform mole. She is caesarean, would likely to be given to a gravida,
aware that one of the following is unassociated who had:
with this condition?
a. First low transverse cesarean was for infant looks for it. The nurse is aware that
active herpes type 2 infections; vaginal estimated age of the infant would be:
culture at 39 weeks pregnancy was a. 6 months
positive. b. 4 months
b. First and second caesareans were for c. 8 months
cephalopelvic disproportion. d. 10 months
c. First caesarean through a classic incision
as a result of severe fetal distress. 16. Which of the following is the most prominent
d. First low transverse caesarean was for feature of public health nursing?
breech position. Fetus in this pregnancy a. It involves providing home care to sick
is in a vertex presentation. people who are not confined in the
hospital.
11. Nurse Ryan is aware that the best initial b. Services are provided free of charge to
approach when trying to take a crying toddler’s people within the catchments area.
temperature is: c. The public health nurse functions as part
a. Talk to the mother first and then to the of a team providing a public health
toddler. nursing services.
b. Bring extra help so it can be done d. Public health nursing focuses on
quickly. preventive, not curative, services.
c. Encourage the mother to hold the child.
d. Ignore the crying and screaming. 17. When the nurse determines whether resources
were maximized in implementing Ligtas Tigdas,
12. Baby Tina a 3 month old infant just had a cleft lip she is evaluating
and palate repair. What should the nurse do to a. Effectiveness
prevent trauma to operative site? b. Efficiency
a. Avoid touching the suture line, even c. Adequacy
when cleaning. d. Appropriateness
b. Place the baby in prone position.
c. Give the baby a pacifier. 18. Vangie is a new B.S.N. graduate. She wants to
d. Place the infant’s arms in soft elbow become a Public Health Nurse. Where should
restraints. she apply?
a. Department of Health
13. Which action should nurse Marian include in the b. Provincial Health Office
care plan for a 2 month old with heart failure? c. Regional Health Office
a. Feed the infant when he cries. d. Rural Health Unit
b. Allow the infant to rest before feeding.
c. Bathe the infant and administer 19. Tony is aware the Chairman of the Municipal
medications before feeding. Health Board is:
d. Weigh and bathe the infant before a. Mayor
feeding. b. Municipal Health Officer
c. Public Health Nurse
14. Nurse Hazel is teaching a mother who plans to d. Any qualified physician
discontinue breast feeding after 5 months. The
nurse should advise her to include which foods 20. Myra is the public health nurse in a municipality
in her infant’s diet? with a total population of about 20,000. There
a. Skim milk and baby food. are 3 rural health midwives among the RHU
b. Whole milk and baby food. personnel. How many more midwife items will
c. Iron-rich formula only. the RHU need?
d. Iron-rich formula and baby food. a. 1
b. 2
15. Mommy Linda is playing with her infant, who is c. 3
sitting securely alone on the floor of the clinic. d. The RHU does not need any more
The mother hides a toy behind her back and midwife item.
the
27
27

26. The nurse is caring for a primigravid client in the


21. According to Freeman and Heinrich, community labor and delivery area. Which condition would
health nursing is a developmental service. Which place the client at risk for disseminated
of the following best illustrates this statement? intravascular coagulation (DIC)?
a. The community health nurse a. Intrauterine fetal death.
continuously develops himself b. Placenta accreta.
personally and professionally. c. Dysfunctional labor.
b. Health education and community d. Premature rupture of the membranes.
organizing are necessary in providing
community health services. 27. A fullterm client is in labor. Nurse Betty is aware
c. Community health nursing is intended that the fetal heart rate would be:
primarily for health promotion and a. 80 to 100 beats/minute
prevention and treatment of disease. b. 100 to 120 beats/minute
d. The goal of community health nursing is c. 120 to 160 beats/minute
to provide nursing services to people in d. 160 to 180 beats/minute
their own places of residence.
28. The skin in the diaper area of a 7 month old
22. Nurse Tina is aware that the disease declared infant is excoriated and red. Nurse Hazel should
through Presidential Proclamation No. 4 as a instruct the mother to:
target for eradication in the Philippines is? a. Change the diaper more often.
a. Poliomyelitis b. Apply talc powder with diaper changes.
b. Measles c. Wash the area vigorously with each
c. Rabies diaper change.
d. Neonatal tetanus d. Decrease the infant’s fluid intake to
decrease saturating diapers.
23. May knows that the step in community
organizing that involves training of potential 29. Nurse Carla knows that the common cardiac
leaders in the community is: anomalies in children with Down Syndrome (tri-
a. Integration somy 21) is:
b. Community organization a. Atrial septal defect
c. Community study b. Pulmonic stenosis
d. Core group formation c. Ventricular septal defect
d. Endocardial cushion defect
24. Beth a public health nurse takes an active role in
community participation. What is the primary 30. Malou was diagnosed with severe preeclampsia
goal of community organizing? is now receiving I.V. magnesium sulfate. The
a. To educate the people regarding adverse effects associated with magnesium
community health problems sulfate is:
b. To mobilize the people to resolve a. Anemia
community health problems b. Decreased urine output
c. To maximize the community’s resources c. Hyperreflexia
in dealing with health problems. d. Increased respiratory rate
d. To maximize the community’s resources
in dealing with health problems. 31. A 23 year old client is having her menstrual
period every 2 weeks that last for 1 week. This
25. Tertiary prevention is needed in which stage of type of menstrual pattern is bets defined by:
the natural history of disease? a. Menorrhagia
a. Pre-pathogenesis b. Metrorrhagia
b. Pathogenesis c. Dyspareunia
c. Prodromal d. Amenorrhea
d. Terminal
32. Jannah is admitted to the labor and delivery b. Dehydration and diarrhea
unit. The critical laboratory result for this client c. Bradycardia and hypotension
would be: d. Petechiae and hematuria
a. Oxygen saturation
b. Iron binding capacity 38. To evaluate a woman’s understanding about the
c. Blood typing use of diaphragm for family planning, Nurse
d. Serum Calcium Trish asks her to explain how she will use the
appliance. Which response indicates a need for
33. Nurse Gina is aware that the most common further health teaching?
condition found during the second-trimester of a. “I should check the diaphragm carefully
pregnancy is: for holes every time I use it”
a. Metabolic alkalosis b. “I may need a different size of
b. Respiratory acidosis diaphragm if I gain or lose weight more
c. Mastitis than 20 pounds”
d. Physiologic anemia c. “The diaphragm must be left in place for
atleast 6 hours after intercourse”
34. Nurse Lynette is working in the triage area of an d. “I really need to use the diaphragm and
emergency department. She sees that several jelly most during the middle of my
pediatric clients arrive simultaneously. The client menstrual cycle”.
who needs to be treated first is:
a. A crying 5 year old child with a 39. Hypoxia is a common complication of
laceration on his scalp. laryngotracheobronchitis. Nurse Oliver should
b. A 4 year old child with a barking coughs frequently assess a child with
and flushed appearance. laryngotracheobronchitis for:
c. A 3 year old child with Down syndrome a. Drooling
who is pale and asleep in his mother’s b. Muffled voice
arms. c. Restlessness
d. A 2 year old infant with stridorous d. Low-grade fever
breath sounds, sitting up in his mother’s
arms and drooling. 40. How should Nurse Michelle guide a child who is
blind to walk to the playroom?
35. Maureen in her third trimester arrives at the a. Without touching the child, talk
emergency room with painless vaginal bleeding. continuously as the child walks down the
Which of the following conditions is suspected? hall.
a. Placenta previa b. Walk one step ahead, with the child’s
b. Abruptio placentae hand on the nurse’s elbow.
c. Premature labor c. Walk slightly behind, gently guiding the
d. Sexually transmitted disease child forward.
d. Walk next to the child, holding the
36. A young child named Richard is suspected of child’s hand.
having pinworms. The community nurse collects
a stool specimen to confirm the diagnosis. The 41. When assessing a newborn diagnosed with
nurse should schedule the collection of this ductus arteriosus, Nurse Olivia should expect
specimen for: that the child most likely would have an:
a. Just before bedtime a. Loud, machinery-like murmur.
b. After the child has been bathe b. Bluish color to the lips.
c. Any time during the day c. Decreased BP reading in the upper
d. Early in the morning extremities
d. Increased BP reading in the upper
37. In doing a child’s admission assessment, Nurse extremities.
Betty should be alert to note which signs or
symptoms of chronic lead poisoning? 42. The reason nurse May keeps the neonate in a
a. Irritability and seizures neutral thermal environment is that when a

27
27
newborn becomes too cool, the neonate 47. Barangay Pinoy had an outbreak of German
requires: measles. To prevent congenital rubella, what is
a. Less oxygen, and the newborn’s the BEST advice that you can give to women in
metabolic rate increases. the first trimester of pregnancy in the barangay
b. More oxygen, and the newborn’s Pinoy?
metabolic rate decreases. a. Advise them on the signs of German
c. More oxygen, and the newborn’s measles.
metabolic rate increases. b. Avoid crowded places, such as markets
d. Less oxygen, and the newborn’s and movie houses.
metabolic rate decreases. c. Consult at the health center where
rubella vaccine may be given.
43. Before adding potassium to an infant’s I.V. line, d. Consult a physician who may give them
Nurse Ron must be sure to assess whether this rubella immunoglobulin.
infant has:
a. Stable blood pressure 48. Myrna a public health nurse knows that to
b. Patant fontanelles determine possible sources of sexually
c. Moro’s reflex transmitted infections, the BEST method that
d. Voided may be undertaken is:
a. Contact tracing
44. Nurse Carla should know that the most common b. Community survey
causative factor of dermatitis in infants and c. Mass screening tests
younger children is: d. Interview of suspects
a. Baby oil
b. Baby lotion 49. A 33-year old female client came for
c. Laundry detergent consultation at the health center with the chief
d. Powder with cornstarch complaint of fever for a week. Accompanying
symptoms were muscle pains and body malaise.
45. During tube feeding, how far above an infant’s A week after the start of fever, the client noted
stomach should the nurse hold the syringe with yellowish discoloration of his sclera. History
formula? showed that he waded in flood waters about 2
a. 6 inches weeks before the onset of symptoms. Based on
b. 12 inches her history, which disease condition will you
c. 18 inches suspect?
d. 24 inches a. Hepatitis A
b. Hepatitis B
46. In a mothers’ class, Nurse Lhynnete discussed c. Tetanus
childhood diseases such as chicken pox. Which d. Leptospirosis
of the following statements about chicken pox is
correct? 50. Mickey a 3-year old client was brought to the
a. The older one gets, the more susceptible health center with the chief complaint of severe
he becomes to the complications of diarrhea and the passage of “rice water” stools.
chicken pox. The client is most probably suffering from which
b. A single attack of chicken pox will condition?
prevent future episodes, including a. Giardiasis
conditions such as shingles. b. Cholera
c. To prevent an outbreak in the c. Amebiasis
community, quarantine may be imposed d. Dysentery
by health authorities.
d. Chicken pox vaccine is best given when 51. The most prevalent form of meningitis among
there is an impending outbreak in the children aged 2 months to 3 years is caused by
community. which microorganism?
a. Hemophilus influenzae
b. Morbillivirus
c. Steptococcus pneumoniae d. Use of protective footwear, such as
d. Neisseria meningitidis rubber boots

52. The student nurse is aware that the 58. Several clients is newly admitted and diagnosed
pathognomonic sign of measles is Koplik’s spot with leprosy. Which of the following clients
and you may see Koplik’s spot by inspecting the: should be classified as a case of multibacillary
a. Nasal mucosa leprosy?
b. Buccal mucosa a. 3 skin lesions, negative slit skin smear
c. Skin on the abdomen b. 3 skin lesions, positive slit skin smear
d. Skin on neck c. 5 skin lesions, negative slit skin smear
d. 5 skin lesions, positive slit skin smear
53. Angel was diagnosed as having Dengue fever.
You will say that there is slow capillary refill 59. Nurses are aware that diagnosis of leprosy is
when the color of the nailbed that you pressed highly dependent on recognition of symptoms.
does not return within how many seconds? Which of the following is an early sign of
a. 3 seconds leprosy?
b. 6 seconds a. Macular lesions
c. 9 seconds b. Inability to close eyelids
d. 10 seconds c. Thickened painful nerves
d. Sinking of the nosebridge
54. In Integrated Management of Childhood Illness,
the nurse is aware that the severe conditions 60. Marie brought her 10 month old infant for
generally require urgent referral to a hospital. consultation because of fever, started 4 days
Which of the following severe conditions DOES prior to consultation. In determining malaria
NOT always require urgent referral to a hospital? risk, what will you do?
a. Mastoiditis a. Perform a tourniquet test.
b. Severe dehydration b. Ask where the family resides.
c. Severe pneumonia c. Get a specimen for blood smear.
d. Severe febrile disease d. Ask if the fever is present every day.

55. Myrna a public health nurse will conduct 61. Susie brought her 4 years old daughter to the
outreach immunization in a barangay Masay RHU because of cough and colds. Following the
with a population of about 1500. The estimated IMCI assessment guide, which of the following is
number of infants in the barangay would be: a danger sign that indicates the need for urgent
a. 45 infants referral to a hospital?
b. 50 infants a. Inability to drink
c. 55 infants b. High grade fever
d. 65 infants c. Signs of severe dehydration
d. Cough for more than 30 days
56. The community nurse is aware that the
biological used in Expanded Program on 62. Jimmy a 2-year old child revealed “baggy pants”.
Immunization (EPI) should NOT be stored in the As a nurse, using the IMCI guidelines, how will
freezer? you manage Jimmy?
a. DPT a. Refer the child urgently to a hospital for
b. Oral polio vaccine confinement.
c. Measles vaccine b. Coordinate with the social worker to
d. MMR enroll the child in a feeding program.
c. Make a teaching plan for the mother,
57. It is the most effective way of controlling focusing on menu planning for her child.
schistosomiasis in an endemic area? d. Assess and treat the child for health
a. Use of molluscicides problems like infections and intestinal
b. Building of foot bridges parasitism.
c. Proper use of sanitary toilets

27
27
63. Gina is using Oresol in the management of 68. The nurse explains to a breastfeeding mother
diarrhea of her 3-year old child. She asked you that breast milk is sufficient for all of the baby’s
what to do if her child vomits. As a nurse you will nutrient needs only up to:
tell her to: a. 5 months
a. Bring the child to the nearest hospital b. 6 months
for further assessment. c. 1 year
b. Bring the child to the health center for d. 2 years
intravenous fluid therapy.
c. Bring the child to the health center for 69. Nurse Ron is aware that the gestational age of a
assessment by the physician. conceptus that is considered viable (able to live
d. Let the child rest for 10 minutes then outside the womb) is:
continue giving Oresol more slowly. a. 8 weeks
b. 12 weeks
64. Nikki a 5-month old infant was brought by his c. 24 weeks
mother to the health center because of diarrhea d. 32 weeks
for 4 to 5 times a day. Her skin goes back slowly
after a skin pinch and her eyes are sunken. Using 70. When teaching parents of a neonate the proper
the IMCI guidelines, you will classify this infant in position for the neonate’s sleep, the nurse
which category? Patricia stresses the importance of placing the
a. No signs of dehydration neonate on his back to reduce the risk of which
b. Some dehydration of the following?
c. Severe dehydration a. Aspiration
d. The data is insufficient. b. Sudden infant death syndrome (SIDS)
c. Suffocation
65. Chris a 4-month old infant was brought by her d. Gastroesophageal reflux (GER)
mother to the health center because of cough.
His respiratory rate is 42/minute. Using the 71. Which finding might be seen in baby James a
Integrated Management of Child Illness (IMCI) neonate suspected of having an infection?
guidelines of assessment, his breathing is a. Flushed cheeks
considered as: b. Increased temperature
a. Fast c. Decreased temperature
b. Slow d. Increased activity level
c. Normal
d. Insignificant 72. Baby Jenny who is small-for-gestation is at
increased risk during the transitional period for
66. Maylene had just received her 4th dose of which complication?
tetanus toxoid. She is aware that her baby will a. Anemia probably due to chronic fetal
have protection against tetanus for hyposia
a. 1 year b. Hyperthermia due to decreased
b. 3 years glycogen stores
c. 5 years c. Hyperglycemia due to decreased
d. Lifetime glycogen stores
d. Polycythemia probably due to chronic
67. Nurse Ron is aware that unused BCG should be fetal hypoxia
discarded after how many hours of
reconstitution? 73. Marjorie has just given birth at 42 weeks’
a. 2 hours gestation. When the nurse assessing the
b. 4 hours neonate, which physical finding is expected?
c. 8 hours a. A sleepy, lethargic baby
d. At the end of the day b. Lanugo covering the body
c. Desquamation of the epidermis
d. Vernix caseosa covering the body
74. After reviewing the Myrna’s maternal history of b. The parent’s expression of interest
magnesium sulfate during labor, which condition about the size of the new born.
would nurse Richard anticipate as a potential c. The parents’ indication that they want to
problem in the neonate? see the newborn.
a. Hypoglycemia d. The parents’ interactions with each
b. Jitteriness other.
c. Respiratory depression
d. Tachycardia 80. Following a precipitous delivery, examination of
the client's vagina reveals a fourth-degree
75. Which symptom would indicate the Baby laceration. Which of the following would be
Alexandra was adapting appropriately to extra- contraindicated when caring for this client?
uterine life without difficulty? a. Applying cold to limit edema during the
a. Nasal flaring first 12 to 24 hours.
b. Light audible grunting b. Instructing the client to use two or more
c. Respiratory rate 40 to 60 peripads to cushion the area.
breaths/minute c. Instructing the client on the use of sitz
d. Respiratory rate 60 to 80 baths if ordered.
breaths/minute d. Instructing the client about the
importance of perineal (kegel) exercises.
76. When teaching umbilical cord care for Jennifer a
new mother, the nurse Jenny would include 81. A pregnant woman accompanied by her
which information? husband, seeks admission to the labor and
a. Apply peroxide to the cord with each delivery area. She states that she's in labor and
diaper change says she attended the facility clinic for prenatal
b. Cover the cord with petroleum jelly after care. Which question should the nurse Oliver ask
bathing her first?
c. Keep the cord dry and open to air a. “Do you have any chronic illnesses?”
d. Wash the cord with soap and water each b. “Do you have any allergies?”
day during a tub bath. c. “What is your expected due date?”
d. “Who will be with you during labor?”
77. Nurse John is performing an assessment on a
neonate. Which of the following findings is 82. A neonate begins to gag and turns a dusky color.
considered common in the healthy neonate? What should the nurse do first?
a. Simian crease a. Calm the neonate.
b. Conjunctival hemorrhage b. Notify the physician.
c. Cystic hygroma c. Provide oxygen via face mask as ordered
d. Bulging fontanelle d. Aspirate the neonate’s nose and mouth
with a bulb syringe.
78. Dr. Esteves decides to artificially rupture the
membranes of a mother who is on labor. 83. When a client states that her "water broke,"
Following this procedure, the nurse Hazel checks which of the following actions would be
the fetal heart tones for which the following inappropriate for the nurse to do?
reasons? a. Observing the pooling of straw-colored
a. To determine fetal well-being. fluid.
b. To assess for prolapsed cord b. Checking vaginal discharge with nitrazine
c. To assess fetal position paper.
d. To prepare for an imminent delivery. c. Conducting a bedside ultrasound for an
79. Which of the following would be least likely to amniotic fluid index.
indicate anticipated bonding behaviors by new d. Observing for flakes of vernix in the
parents? vaginal discharge.
a. The parents’ willingness to touch and
hold the new born. 84. A baby girl is born 8 weeks premature. At birth,
she has no spontaneous respirations but is

28
28
successfully resuscitated. Within several hours c. Decreased inspiratory capacity
she develops respiratory grunting, cyanosis, d. Decreased oxygen consumption
tachypnea, nasal flaring, and retractions. She's
diagnosed with respiratory distress syndrome, 90. Emily has gestational diabetes and it is usually
intubated, and placed on a ventilator. Which managed by which of the following therapy?
nursing action should be included in the baby's a. Diet
plan of care to prevent retinopathy of b. Long-acting insulin
prematurity? c. Oral hypoglycemic
a. Cover his eyes while receiving oxygen. d. Oral hypoglycemic drug and insulin
b. Keep her body temperature low.
c. Monitor partial pressure of oxygen 91. Magnesium sulfate is given to Jemma with
(Pao2) levels. preeclampsia to prevent which of the following
d. Humidify the oxygen. condition?
a. Hemorrhage
85. Which of the following is normal newborn b. Hypertension
calorie intake? c. Hypomagnesemia
a. 110 to 130 calories per kg. d. Seizure
b. 30 to 40 calories per lb of body weight.
c. At least 2 ml per feeding 92. Cammile with sickle cell anemia has an increased
d. 90 to 100 calories per kg risk for having a sickle cell crisis during
pregnancy. Aggressive management of a sickle
86. Nurse John is knowledgeable that usually cell crisis includes which of the following
individual twins will grow appropriately and at measures?
the same rate as singletons until how many a. Antihypertensive agents
weeks? b. Diuretic agents
a. 16 to 18 weeks c. I.V. fluids
b. 18 to 22 weeks d. Acetaminophen (Tylenol) for pain
c. 30 to 32 weeks
d. 38 to 40 weeks 93. Which of the following drugs is the antidote for
magnesium toxicity?
87. Which of the following classifications applies to a. Calcium gluconate (Kalcinate)
monozygotic twins for whom the cleavage of the b. Hydralazine (Apresoline)
fertilized ovum occurs more than 13 days after c. Naloxone (Narcan)
fertilization? d. Rho (D) immune globulin (RhoGAM)
a. conjoined twins
b. diamniotic dichorionic twins 94. Marlyn is screened for tuberculosis during her
c. diamniotic monochorionic twin first prenatal visit. An intradermal injection of
d. monoamniotic monochorionic twins purified protein derivative (PPD) of the
tuberculin bacilli is given. She is considered to
88. Tyra experienced painless vaginal bleeding has have a positive test for which of the following
just been diagnosed as having a placenta previa. results?
Which of the following procedures is usually a. An indurated wheal under 10 mm in
performed to diagnose placenta previa? diameter appears in 6 to 12 hours.
a. Amniocentesis b. An indurated wheal over 10 mm in
b. Digital or speculum examination diameter appears in 48 to 72 hours.
c. External fetal monitoring c. A flat circumcised area under 10 mm in
d. Ultrasound diameter appears in 6 to 12 hours.
d. A flat circumcised area over 10 mm in
89. Nurse Arnold knows that the following changes diameter appears in 48 to 72 hours.
in respiratory functioning during pregnancy is
considered normal: 95. Dianne, 24 year-old is 27 weeks’ pregnant
a. Increased tidal volume arrives at her physician’s office with complaints
b. Increased expiratory volume of fever, nausea, vomiting, malaise, unilateral
flank pain, and costovertebral angle tenderness. a. Uterine inversion
Which of the following diagnoses is most likely? b. Uterine atony
a. Asymptomatic bacteriuria c. Uterine involution
b. Bacterial vaginosis d. Uterine discomfort
c. Pyelonephritis
d. Urinary tract infection (UTI)

96. Rh isoimmunization in a pregnant client


develops during which of the following
conditions?
a. Rh-positive maternal blood crosses into
fetal blood, stimulating fetal antibodies.
b. Rh-positive fetal blood crosses into
maternal blood, stimulating maternal
antibodies.
c. Rh-negative fetal blood crosses into
maternal blood, stimulating maternal
antibodies.
d. Rh-negative maternal blood crosses into
fetal blood, stimulating fetal antibodies.

97. To promote comfort during labor, the nurse John


advises a client to assume certain positions and
avoid others. Which position may cause
maternal hypotension and fetal hypoxia?
a. Lateral position
b. Squatting position
c. Supine position
d. Standing position

98. Celeste who used heroin during her pregnancy


delivers a neonate. When assessing the neonate,
the nurse Lhynnette expects to find:
a. Lethargy 2 days after birth.
b. Irritability and poor sucking.
c. A flattened nose, small eyes, and thin
lips.
d. Congenital defects such as limb
anomalies.

99. The uterus returns to the pelvic cavity in which


of the following time frames?
a. 7th to 9th day postpartum.
b. 2 weeks postpartum.
c. End of 6th week postpartum.
d. When the lochia changes to alba.

100. Maureen, a primigravida client, age 20, has


just completed a difficult, forceps-assisted
delivery of twins. Her labor was unusually
long and required oxytocin (Pitocin)
augmentation. The nurse who's caring for her
should stay alert for:

28
28
Answers and Rationale – Community Health result in injury to the mother and the fetus if
Nursing and Care of the Mother and Child Pitocin is not discontinued.
9. Answer: (C) EKG tracings
1. Answer: (A) Inevitable Rationale: A potential side effect of calcium
Rationale: An inevitable abortion is termination gluconate administration is cardiac arrest.
of pregnancy that cannot be prevented. Continuous monitoring of cardiac activity (EKG)
Moderate to severe bleeding with mild throught administration of calcium gluconate is
cramping and cervical dilation would be noted an essential part of care.
in this type of abortion. 10. Answer: (D) First low transverse caesarean was
2. Answer: (B) History of syphilis for breech position. Fetus in this pregnancy is in
Rationale: Maternal infections such as syphilis, a vertex presentation.
toxoplasmosis, and rubella are causes of Rationale: This type of client has no obstetrical
spontaneous abortion. indication for a caesarean section as she did
3. Answer: (C) Monitoring apical pulse with her first caesarean delivery.
Rationale: Nursing care for the client with a 11. Answer: (A) Talk to the mother first and then to
possible ectopic pregnancy is focused on the toddler.
preventing or identifying hypovolemic shock Rationale: When dealing with a crying toddler,
and controlling pain. An elevated pulse rate is the best approach is to talk to the mother and
an indicator of shock. ignore the toddler first. This approach helps the
4. Answer: (B) Increased caloric intake toddler get used to the nurse before she
Rationale: Glucose crosses the placenta, but attempts any procedures. It also gives the
insulin does not. High fetal demands for toddler an opportunity to see that the mother
glucose, combined with the insulin resistance trusts the nurse.
caused by hormonal changes in the last half of 12. Answer: (D) Place the infant’s arms in soft
pregnancy can result in elevation of maternal elbow restraints.
blood glucose levels. This increases the Rationale: Soft restraints from the upper arm to
mother’s demand for insulin and is referred to the wrist prevent the infant from touching her
as the diabetogenic effect of pregnancy. lip but allow him to hold a favorite item such as
5. Answer: (A) Excessive fetal activity. a blanket. Because they could damage the
Rationale: The most common signs and operative site, such as objects as pacifiers,
symptoms of hydatidiform mole includes suction catheters, and small spoons shouldn’t
elevated levels of human chorionic be placed in a baby’s mouth after cleft repair. A
gonadotropin, vaginal bleeding, larger than baby in a prone position may rub her face on
normal uterus for gestational age, failure to the sheets and traumatize the operative site.
detect fetal heart activity even with sensitive The suture line should be cleaned gently to
instruments, excessive nausea and vomiting, prevent infection, which could interfere with
and early development of pregnancy-induced healing and damage the cosmetic appearance
hypertension. Fetal activity would not be noted. of the repair.
6. Answer: (B) Absent patellar reflexes 13. Answer: (B) Allow the infant to rest before
Rationale: Absence of patellar reflexes is an feeding.
indicator of hypermagnesemia, which requires Rationale: Because feeding requires so much
administration of calcium gluconate. energy, an infant with heart failure should rest
7. Answer: (C) Presenting part in 2 cm below the before feeding.
plane of the ischial spines. 14. Answer: (C) Iron-rich formula only.
Rationale: Fetus at station plus two indicates Rationale: The infants at age 5 months should
that the presenting part is 2 cm below the receive iron-rich formula and that they
plane of the ischial spines. shouldn’t receive solid food, even baby food
8. Answer: (A) Contractions every 1 ½ minutes until age 6 months.
lasting 70-80 seconds. 15. Answer: (D) 10 months
Rationale: Contractions every 1 ½ minutes Rationale: A 10 month old infant can sit alone
lasting 70-80 seconds, is indicative of and understands object permanence, so he
hyperstimulation of the uterus, which could would look for the hidden toy. At age 4 to 6
months, infants can’t sit securely alone. At age
8 months, infants can sit securely alone but embolism may trigger normal clotting
cannot understand the permanence of objects. mechanisms; if clotting factors are depleted,
16. Answer: (D) Public health nursing focuses on DIC may occur. Placenta accreta, dysfunctional
preventive, not curative, services. labor, and premature rupture of the
Rationale: The catchments area in PHN consists membranes aren't associated with DIC.
of a residential community, many of whom are 27. Answer: (C) 120 to 160 beats/minute
well individuals who have greater need for Rationale: A rate of 120 to 160 beats/minute in
preventive rather than curative services. the fetal heart appropriate for filling the heart
17. Answer: (B) Efficiency with blood and pumping it out to the system.
Rationale: Efficiency is determining whether the 28. Answer: (A) Change the diaper more often.
goals were attained at the least possible cost. Rationale: Decreasing the amount of time the
18. Answer: (D) Rural Health Unit skin comes contact with wet soiled diapers will
Rationale: R.A. 7160 devolved basic health help heal the irritation.
services to local government units (LGU’s ). The 29. Answer: (D) Endocardial cushion defect
public health nurse is an employee of the LGU. Rationale: Endocardial cushion defects are seen
19. Answer: (A) Mayor most in children with Down syndrome,
Rationale: The local executive serves as the asplenia, or polysplenia.
chairman of the Municipal Health Board. 30. Answer: (B) Decreased urine output
20. Answer: (A) 1 Rationale: Decreased urine output may occur in
Rationale: Each rural health midwife is given a clients receiving I.V. magnesium and should be
population assignment of about 5,000. monitored closely to keep urine output at
21. Answer: (B) Health education and community greater than 30 ml/hour, because magnesium is
organizing are necessary in providing excreted through the kidneys and can easily
community health services. Rationale: The accumulate to toxic levels.
community health nurse develops the health 31. Answer: (A) Menorrhagia
capability of people through health education Rationale: Menorrhagia is an excessive
and community organizing activities. menstrual period.
22. Answer: (B) Measles 32. Answer: (C) Blood typing
Rationale: Presidential Proclamation No. 4 is on Rationale: Blood type would be a critical value
the Ligtas Tigdas Program. to have because the risk of blood loss is always
23. Answer: (D) Core group formation a potential complication during the labor and
Rationale: In core group formation, the nurse is delivery process. Approximately 40% of a
able to transfer the technology of community woman’s cardiac output is delivered to the
organizing to the potential or informal uterus, therefore, blood loss can occur quite
community leaders through a training program. rapidly in the event of uncontrolled bleeding.
24. Answer: (D) To maximize the community’s 33. Answer: (D) Physiologic anemia
resources in dealing with health problems. Rationale: Hemoglobin values and hematocrit
Rationale: Community organizing is a decrease during pregnancy as the increase in
developmental service, with the goal of plasma volume exceeds the increase in red
developing the people’s self-reliance in dealing blood cell production.
with community health problems. A, B and C 34. Answer: (D) A 2 year old infant with stridorous
are objectives of contributory objectives to this breath sounds, sitting up in his mother’s arms
goal. and drooling.
25. Answer: (D) Terminal Rationale: The infant with the airway
Rationale: Tertiary prevention involves emergency should be treated first, because of
rehabilitation, prevention of permanent the risk of epiglottitis.
disability and disability limitations appropriate 35. Answer: (A) Placenta previa
for convalescents, the disabled, complicated Rationale: Placenta previa with painless vaginal
cases and the terminally ill (those in the bleeding.
terminal stage of a disease). 36. Answer: (D) Early in the morning
26. Answer: (A) Intrauterine fetal death. Rationale: Based on the nurse’s knowledge of
Rationale: Intrauterine fetal death, abruptio microbiology, the specimen should be collected
placentae, septic shock, and amniotic fluid early in the morning. The rationale for this

28
28
timing is that, because the female worm lays Rationale: Eczema or dermatitis is an allergic
eggs at night around the perineal area, the first skin reaction caused by an offending allergen.
bowel movement of the day will yield the best The topical allergen that is the most common
results. The specific type of stool specimen causative factor is laundry detergent.
used in the diagnosis of pinworms is called the 45. Answer: (A) 6 inches
tape test. Rationale: This distance allows for easy flow of
37. Answer: (A) Irritability and seizures the formula by gravity, but the flow will be slow
Rationale: Lead poisoning primarily affects the enough not to overload the stomach too
CNS, causing increased intracranial pressure. rapidly.
This condition results in irritability and changes 46. Answer: (A) The older one gets, the more
in level of consciousness, as well as seizure susceptible he becomes to the complications of
disorders, hyperactivity, and learning chicken pox.
disabilities. Rationale: Chicken pox is usually more severe in
38. Answer: (D) “I really need to use the adults than in children. Complications, such as
diaphragm and jelly most during the middle of pneumonia, are higher in incidence in adults.
my 47. Answer: (D) Consult a physician who may
menstrual cycle”. give them rubella immunoglobulin.
Rationale: The woman must understand that, Rationale: Rubella vaccine is made up of
although the “fertile” period is approximately attenuated German measles viruses. This is
mid-cycle, hormonal variations do occur and contraindicated in pregnancy. Immune globulin,
can result in early or late ovulation. To be a specific prophylactic against German measles,
effective, the diaphragm should be inserted may be given to pregnant women.
before every intercourse. 48. Answer: (A) Contact tracing
39. Answer: (C) Restlessness Rationale: Contact tracing is the most practical
Rationale: In a child, restlessness is the earliest and reliable method of finding possible sources
sign of hypoxia. Late signs of hypoxia in a child of person-to-person transmitted infections,
are associated with a change in color, such as such as sexually transmitted diseases.
pallor or cyanosis. 49. Answer: (D) Leptospirosis
40. Answer: (B) Walk one step ahead, with the Rationale: Leptospirosis is transmitted through
child’s hand on the nurse’s elbow. contact with the skin or mucous membrane
Rationale: This procedure is generally with water or moist soil contaminated with
recommended to follow in guiding a person urine of infected animals, like rats.
who is blind. 50. Answer: (B) Cholera
41. Answer: (A) Loud, machinery-like murmur. Rationale: Passage of profuse watery stools is
Rationale: A loud, machinery-like murmur is a the major symptom of cholera. Both amebic
characteristic finding associated with patent and bacillary dysentery are characterized by the
ductus arteriosus. presence of blood and/or mucus in the stools.
42. Answer: (C) More oxygen, and the newborn’s Giardiasis is characterized by fat malabsorption
metabolic rate increases. and, therefore, steatorrhea.
Rationale: When cold, the infant requires more 51. Answer: (A) Hemophilus influenzae
oxygen and there is an increase in metabolic Rationale: Hemophilus meningitis is unusual
rate. Non-shievering thermogenesis is a over the age of 5 years. In developing countries,
complex process that increases the metabolic the peak incidence is in children less than 6
rate and rate of oxygen consumption, months of age. Morbillivirus is the etiology of
therefore, the newborn increase heat measles. Streptococcus pneumonia and
production. Neisseria meningitidis may cause meningitis,
43. Answer: (D) Voided but age distribution is not specific in young
Rationale: Before administering potassium I.V. children.
to any client, the nurse must first check that the 52. Answer: (B) Buccal mucosa
client’s kidneys are functioning and that the Rationale: Koplik’s spot may be seen on the
client is voiding. If the client is not voiding, the mucosa of the mouth or the throat.
nurse should withhold the potassium and notify 53. Answer: (A) 3 seconds
the physician.
44. Answer: (c) Laundry detergent
Rationale: Adequate blood supply to the area not able to feed or drink, vomits everything,
allows the return of the color of the nailbed convulsions, abnormally sleepy or difficult to
within 3 seconds. awaken.
54. Answer: (B) Severe dehydration 62. Answer: (A) Refer the child urgently to a
Rationale: The order of priority in the hospital for confinement.
management of severe dehydration is as Rationale: “Baggy pants” is a sign of severe
follows: intravenous fluid therapy, referral to a marasmus. The best management is urgent
facility where IV fluids can be initiated within 30 referral to a hospital.
minutes, Oresol or nasogastric tube. When the 63. Answer: (D) Let the child rest for 10 minutes
foregoing measures are not possible or then continue giving Oresol more slowly.
effective, then urgent referral to the hospital is Rationale: If the child vomits persistently, that
done. is, he vomits everything that he takes in, he has
55. Answer: (A) 45 infants to be referred urgently to a hospital. Otherwise,
Rationale: To estimate the number of infants, vomiting is managed by letting the child rest for
multiply total population by 3%. 10 minutes and then continuing with Oresol
56. Answer: (A) DPT administration. Teach the mother to give Oresol
Rationale: DPT is sensitive to freezing. The more slowly.
appropriate storage temperature of DPT is 2 to 64. Answer: (B) Some dehydration
8° C only. OPV and measles vaccine are highly Rationale: Using the assessment guidelines of
sensitive to heat and require freezing. MMR is IMCI, a child (2 months to 5 years old) with
not an immunization in the Expanded Program diarrhea is classified as having SOME
on Immunization. DEHYDRATION if he shows 2 or more of the
57. Answer: (C) Proper use of sanitary toilets following signs: restless or irritable, sunken
Rationale: The ova of the parasite get out of the eyes, the skin goes back slow after a skin pinch.
human body together with feces. Cutting the 65. Answer: (C) Normal
cycle at this stage is the most effective way of Rationale: In IMCI, a respiratory rate of
preventing the spread of the disease to 50/minute or more is fast breathing for an
susceptible hosts. infant aged 2 to 12 months.
58. Answer: (D) 5 skin lesions, positive slit skin 66. Answer: (A) 1 year
smear Rationale: The baby will have passive natural
Rationale: A multibacillary leprosy case is one immunity by placental transfer of antibodies.
who has a positive slit skin smear and at least 5 The mother will have active artificial immunity
skin lesions. lasting for about 10 years. 5 doses will give the
59. Answer: (C) Thickened painful nerves mother lifetime protection.
Rationale: The lesion of leprosy is not macular. 67. Answer: (B) 4 hours
It is characterized by a change in skin color Rationale: While the unused portion of other
(either reddish or whitish) and loss of sensation, biologicals in EPI may be given until the end of
sweating and hair growth over the lesion. the day, only BCG is discarded 4 hours after
Inability to close the eyelids (lagophthalmos) reconstitution. This is why BCG immunization is
and sinking of the nosebridge are late scheduled only in the morning.
symptoms. 68. Answer: (B) 6 months
60. Answer: (B) Ask where the family resides. Rationale: After 6 months, the baby’s nutrient
Rationale: Because malaria is endemic, the first needs, especially the baby’s iron requirement,
question to determine malaria risk is where the can no longer be provided by mother’s milk
client’s family resides. If the area of residence is alone.
not a known endemic area, ask if the child had 69. Answer: (C) 24 weeks
traveled within the past 6 months, where she Rationale: At approximately 23 to 24 weeks’
was brought and whether she stayed overnight gestation, the lungs are developed enough to
in that area. sometimes maintain extrauterine life. The lungs
61. Answer: (A) Inability to drink are the most immature system during the
Rationale: A sick child aged 2 months to 5 years gestation period. Medical care for premature
must be referred urgently to a hospital if labor begins much earlier (aggressively at 21
he/she has one or more of the following signs: weeks’ gestation)

28
28
70. Answer: (B) Sudden infant death syndrome infection. Peroxide could be painful and isn’t
(SIDS) recommended.
Rationale: Supine positioning is recommended 77. Answer: (B) Conjunctival hemorrhage
to reduce the risk of SIDS in infancy. The risk of Rationale: Conjunctival hemorrhages are
aspiration is slightly increased with the supine commonly seen in neonates secondary to the
position. Suffocation would be less likely with cranial pressure applied during the birth
an infant supine than prone and the position process. Bulging fontanelles are a sign of
for GER requires the head of the bed to be intracranial pressure. Simian creases are
elevated. present in 40% of the neonates with trisomy 21.
71. Answer: (C) Decreased temperature Cystic hygroma is a neck mass that can affect
Rationale: Temperature instability, especially the airway.
when it results in a low temperature in the 78. Answer: (B) To assess for prolapsed cord
neonate, may be a sign of infection. The Rationale: After a client has an amniotomy, the
neonate’s color often changes with an infection nurse should assure that the cord isn't
process but generally becomes ashen or prolapsed and that the baby tolerated the
mottled. The neonate with an infection will procedure well. The most effective way to do
usually show a decrease in activity level or this is to check the fetal heart rate. Fetal well-
lethargy. being is assessed via a nonstress test. Fetal
72. Answer: (D) Polycythemia probably due to position is determined by vaginal examination.
chronic fetal hypoxia Artificial rupture of membranes doesn't
Rationale: The small-for-gestation neonate is at indicate an imminent delivery.
risk for developing polycythemia during the 79. Answer: (D) The parents’ interactions with each
transitional period in an attempt to decrease other.
hypoxia. The neonates are also at increased risk Rationale: Parental interaction will provide the
for developing hypoglycemia and hypothermia nurse with a good assessment of the stability of
due to decreased glycogen stores. the family's home life but it has no indication
73. Answer: (C) Desquamation of the epidermis for parental bonding. Willingness to touch and
Rationale: Postdate fetuses lose the vernix hold the newborn, expressing interest about
caseosa, and the epidermis may become the newborn's size, and indicating a desire to
desquamated. These neonates are usually very see the newborn are behaviors indicating
alert. Lanugo is missing in the postdate parental bonding.
neonate. 80. Answer: (B) Instructing the client to use two or
74. Answer: (C) Respiratory depression more peripads to cushion the area
Rationale: Magnesium sulfate crosses the Rationale: Using two or more peripads would
placenta and adverse neonatal effects are do little to reduce the pain or promote perineal
respiratory depression, hypotonia, and healing. Cold applications, sitz baths, and Kegel
bradycardia. The serum blood sugar isn’t exercises are important measures when the
affected by magnesium sulfate. The neonate client has a fourth-degree laceration.
would be floppy, not jittery. 81. Answer: (C) “What is your expected due date?”
75. Answer: (C) Respiratory rate 40 to 60 Rationale: When obtaining the history of a
breaths/minute client who may be in labor, the nurse's highest
Rationale: A respiratory rate 40 to 60 priority is to determine her current status,
breaths/minute is normal for a neonate during particularly her due date, gravidity, and parity.
the transitional period. Nasal flaring, Gravidity and parity affect the duration of labor
respiratory rate more than 60 breaths/minute, and the potential for labor complications. Later,
and audible grunting are signs of respiratory the nurse should ask about chronic illnesses,
distress. allergies, and support persons.
76. Answer: (C) Keep the cord dry and open to air 82. Answer: (D) Aspirate the neonate’s nose and
Rationale: Keeping the cord dry and open to air mouth with a bulb syringe.
helps reduce infection and hastens drying. Rationale: The nurse's first action should be to
Infants aren’t given tub bath but are sponged clear the neonate's airway with a bulb syringe.
off until the cord falls off. Petroleum jelly After the airway is clear and the neonate's color
prevents the cord from drying and encourages improves, the nurse should comfort and calm
the neonate. If the problem recurs or the day after fertilization results in diamniotic
neonate's color doesn't improve readily, the dicchorionic twins. Cleavage that occurs
nurse should notify the physician. between days 3 and 8 results in diamniotic
Administering oxygen when the airway isn't monochorionic twins. Cleavage that occurs
clear would be ineffective. between days 8 to 13 result in monoamniotic
83. Answer: (C) Conducting a bedside ultrasound monochorionic twins.
for an amniotic fluid index. 88. Answer: (D) Ultrasound
Rationale: It isn't within a nurse's scope of Rationale: Once the mother and the fetus are
practice to perform and interpret a bedside stabilized, ultrasound evaluation of the
ultrasound under these conditions and without placenta should be done to determine the
specialized training. Observing for pooling of cause of the bleeding. Amniocentesis is
straw-colored fluid, checking vaginal discharge contraindicated in placenta previa. A digital or
with nitrazine paper, and observing for flakes of speculum examination shouldn’t be done as
vernix are appropriate assessments for this may lead to severe bleeding or
determining whether a client has ruptured hemorrhage. External fetal monitoring won’t
membranes. detect a placenta previa, although it will detect
84. Answer: (C) Monitor partial pressure of oxygen fetal distress, which may result from blood loss
(Pao2) levels. or placenta separation.
Rationale: Monitoring PaO2 levels and reducing 89. Answer: (A) Increased tidal volume
the oxygen concentration to keep PaO2 within Rationale: A pregnant client breathes deeper,
normal limits reduces the risk of retinopathy of which increases the tidal volume of gas moved
prematurity in a premature infant receiving in and out of the respiratory tract with each
oxygen. Covering the infant's eyes and breath. The expiratory volume and residual
humidifying the oxygen don't reduce the risk of volume decrease as the pregnancy progresses.
retinopathy of prematurity. Because cooling The inspiratory capacity increases during
increases the risk of acidosis, the infant should pregnancy. The increased oxygen consumption
be kept warm so that his respiratory distress in the pregnant client is 15% to 20% greater
isn't aggravated. than in the nonpregnant state.
85. Answer: (A) 110 to 130 calories per kg. 90. Answer: (A) Diet
Rationale: Calories per kg is the accepted way Rationale: Clients with gestational diabetes are
of determined appropriate nutritional intake usually managed by diet alone to control their
for a newborn. The recommended calorie glucose intolerance. Oral hypoglycemic drugs
requirement is 110 to 130 calories per kg of are contraindicated in pregnancy. Long-acting
newborn body weight. This level will maintain a insulin usually isn’t needed for blood glucose
consistent blood glucose level and provide control in the client with gestational diabetes.
enough calories for continued growth and 91. Answer: (D) Seizure
development. Rationale: The anticonvulsant mechanism of
86. Answer: (C) 30 to 32 weeks magnesium is believes to depress seizure foci in
Rationale: Individual twins usually grow at the the brain and peripheral neuromuscular
same rate as singletons until 30 to 32 weeks’ blockade. Hypomagnesemia isn’t a
gestation, then twins don’t’ gain weight as complication of preeclampsia. Antihypertensive
rapidly as singletons of the same gestational drug other than magnesium are preferred for
age. The placenta can no longer keep pace with sustained hypertension. Magnesium doesn’t
the nutritional requirements of both fetuses help prevent hemorrhage in preeclamptic
after 32 weeks, so there’s some growth clients.
retardation in twins if they remain in utero at 92. Answer: (C) I.V. fluids
38 to 40 weeks. Rationale: A sickle cell crisis during pregnancy is
87. Answer: (A) conjoined twins usually managed by exchange transfusion
Rationale: The type of placenta that develops in oxygen, and L.V. Fluids. The client usually needs
monozygotic twins depends on the time at a stronger analgesic than acetaminophen to
which cleavage of the ovum occurs. Cleavage in control the pain of a crisis. Antihypertensive
conjoined twins occurs more than 13 days after drugs usually aren’t necessary. Diuretic
fertilization. Cleavage that occurs less than 3 wouldn’t be used unless fluid overload resulted.

28
28
93. Answer: (A) Calcium gluconate (Kalcinate) Rationale: Neonates of heroin-addicted
Rationale: Calcium gluconate is the antidote for mothers are physically dependent on the drug
magnesium toxicity. Ten milliliters of 10% and experience withdrawal when the drug is no
calcium gluconate is given L.V. push over 3 to 5 longer supplied. Signs of heroin withdrawal
minutes. Hydralazine is given for sustained include irritability, poor sucking, and
elevated blood pressure in preeclamptic clients. restlessness. Lethargy isn't associated with
Rho (D) immune globulin is given to women neonatal heroin addiction. A flattened nose,
with Rh-negative blood to prevent antibody small eyes, and thin lips are seen in infants with
formation from RH-positive conceptions. fetal alcohol syndrome. Heroin use during
Naloxone is used to correct narcotic toxicity. pregnancy hasn't been linked to specific
94. Answer: (B) An indurated wheal over 10 mm in congenital anomalies.
diameter appears in 48 to 72 hours. 99. Answer: (A) 7th to 9th day postpartum
Rationale: A positive PPD result would be an Rationale: The normal involutional process
indurated wheal over 10 mm in diameter that returns the uterus to the pelvic cavity in 7 to 9
appears in 48 to 72 hours. The area must be a days. A significant involutional complication is
raised wheal, not a flat circumcised area to be the failure of the uterus to return to the pelvic
considered positive. cavity within the prescribed time period. This is
95. Answer: (C) Pyelonephritis known as subinvolution.
Rationale The symptoms indicate acute 100. Answer: (B) Uterine atony
pyelonephritis, a serious condition in a Rationale: Multiple fetuses, extended labor
pregnant client. UTI symptoms include dysuria, stimulation with oxytocin, and traumatic
urgency, frequency, and suprapubic delivery commonly are associated with uterine
tenderness. Asymptomatic bacteriuria doesn’t atony, which may lead to postpartum
cause symptoms. Bacterial vaginosis causes hemorrhage. Uterine inversion may precede or
milky white vaginal discharge but no systemic follow delivery and commonly results from
symptoms. apparent excessive traction on the umbilical
96. Answer: (B) Rh-positive fetal blood crosses into cord and attempts to deliver the placenta
maternal blood, stimulating maternal manually. Uterine involution and some uterine
antibodies. discomfort are normal after delivery.
Rationale: Rh isoimmunization occurs when Rh-
positive fetal blood cells cross into the maternal
circulation and stimulate maternal antibody
production. In subsequent pregnancies with Rh-
positive fetuses, maternal antibodies may cross
back into the fetal circulation and destroy the
fetal blood cells.
97. Answer: (C) Supine position
Rationale: The supine position causes
compression of the client's aorta and inferior
vena cava by the fetus. This, in turn, inhibits
maternal circulation, leading to maternal
hypotension and, ultimately, fetal hypoxia. The
other positions promote comfort and aid labor
progress. For instance, the lateral, or side-lying,
position improves maternal and fetal
circulation, enhances comfort, increases
maternal relaxation, reduces muscle tension,
and eliminates pressure points. The squatting
position promotes comfort by taking advantage
of gravity. The standing position also takes
advantage of gravity and aligns the fetus with
the pelvic angle.
98. Answer: (B) Irritability and poor sucking.
TEST III - Care of Clients with Physiologic and 6. Nurse Monett is caring for a client recovering
Psychosocial Alterations from gastro-intestinal bleeding. The nurse
should:
1. Nurse Michelle should know that the drainage is a. Plan care so the client can receive 8
normal 4 days after a sigmoid colostomy when hours of uninterrupted sleep each night.
the stool is: b. Monitor vital signs every 2 hours.
a. Green liquid c. Make sure that the client takes food and
b. Solid formed medications at prescribed intervals.
c. Loose, bloody d. Provide milk every 2 to 3 hours.
d. Semiformed
7. A male client was on warfarin (Coumadin) before
2. Where would nurse Kristine place the call light admission, and has been receiving heparin I.V.
for a male client with a right-sided brain attack for 2 days. The partial thromboplastin time (PTT)
and left homonymous hemianopsia? is 68 seconds. What should Nurse Carla do?
a. On the client’s right side a. Stop the I.V. infusion of heparin and
b. On the client’s left side notify the physician.
c. Directly in front of the client b. Continue treatment as ordered.
d. Where the client like c. Expect the warfarin to increase the PTT.
d. Increase the dosage, because the level is
3. A male client is admitted to the emergency lower than normal.
department following an accident. What are the
first nursing actions of the nurse? 8. A client undergone ileostomy, when should the
a. Check respiration, circulation, drainage appliance be applied to the stoma?
neurological response. a. 24 hours later, when edema has
b. Align the spine, check pupils, and check subsided.
for hemorrhage. b. In the operating room.
c. Check respirations, stabilize spine, and c. After the ileostomy begin to function.
check circulation. d. When the client is able to begin self-care
d. Assess level of consciousness and procedures.
circulation.
9. A client undergone spinal anesthetic, it will be
4. In evaluating the effect of nitroglycerin, Nurse important that the nurse immediately position
Arthur should know that it reduces preload and the client in:
relieves angina by: a. On the side, to prevent obstruction of
a. Increasing contractility and slowing airway by tongue.
heart rate. b. Flat on back.
b. Increasing AV conduction and heart rate. c. On the back, with knees flexed 15
c. Decreasing contractility and oxygen degrees.
consumption. d. Flat on the stomach, with the head
d. Decreasing venous return through turned to the side.
vasodilation.
10. While monitoring a male client several hours
5. Nurse Patricia finds a female client who is post- after a motor vehicle accident, which
myocardial infarction (MI) slumped on the side assessment data suggest increasing intracranial
rails of the bed and unresponsive to shaking or pressure?
shouting. Which is the nurse next action? a. Blood pressure is decreased from
a. Call for help and note the time. 160/90 to 110/70.
b. Clear the airway b. Pulse is increased from 87 to 95, with an
c. Give two sharp thumps to the occasional skipped beat.
precordium, and check the pulse. c. The client is oriented when aroused
d. Administer two quick blows. from sleep, and goes back to sleep
immediately.

29
29
d. The client refuses dinner because of 16. Nurse John is caring for a male client receiving
anorexia. lidocaine I.V. Which factor is the most relevant
to administration of this medication?
11. Mrs. Cruz, 80 years old is diagnosed with a. Decrease in arterial oxygen saturation
pneumonia. Which of the following symptoms (SaO2) when measured with a pulse
may appear first? oximeter.
a. Altered mental status and dehydration b. Increase in systemic blood pressure.
b. Fever and chills c. Presence of premature ventricular
c. Hemoptysis and Dyspnea contractions (PVCs) on a cardiac
d. Pleuritic chest pain and cough monitor.
d. Increase in intracranial pressure (ICP).
12. A male client has active tuberculosis (TB). Which
of the following symptoms will be exhibit? 17. Nurse Ron is caring for a male client taking an
a. Chest and lower back pain anticoagulant. The nurse should teach the client
b. Chills, fever, night sweats, and to:
hemoptysis a. Report incidents of diarrhea.
c. Fever of more than 104°F (40°C) and b. Avoid foods high in vitamin K
nausea c. Use a straight razor when shaving.
d. Headache and photophobia d. Take aspirin to pain relief.

13. Mark, a 7-year-old client is brought to the 18. Nurse Lhynnette is preparing a site for the
emergency department. He’s tachypneic and insertion of an I.V. catheter. The nurse should
afebrile and has a respiratory rate of 36 treat excess hair at the site by:
breaths/minute and has a nonproductive cough. a. Leaving the hair intact
He recently had a cold. Form this history; the b. Shaving the area
client may have which of the following c. Clipping the hair in the area
conditions? d. Removing the hair with a depilatory.
a. Acute asthma
b. Bronchial pneumonia 19. Nurse Michelle is caring for an elderly female
c. Chronic obstructive pulmonary disease with osteoporosis. When teaching the client, the
(COPD) nurse should include information about which
d. Emphysema major complication:
a. Bone fracture
14. Marichu was given morphine sulfate for pain. b. Loss of estrogen
She is sleeping and her respiratory rate is 4 c. Negative calcium balance
breaths/minute. If action isn’t taken quickly, she d. Dowager’s hump
might have which of the following reactions?
a. Asthma attack 20. Nurse Len is teaching a group of women to
b. Respiratory arrest perform BSE. The nurse should explain that
c. Seizure the purpose of performing the examination is
d. Wake up on his own to discover:
a. Cancerous lumps
15. A 77-year-old male client is admitted for elective b. Areas of thickness or fullness
knee surgery. Physical examination reveals c. Changes from previous examinations.
shallow respirations but no sign of respiratory d. Fibrocystic masses
distress. Which of the following is a normal
physiologic change related to aging? 21. When caring for a female client who is being
a. Increased elastic recoil of the lungs treated for hyperthyroidism, it is important to:
b. Increased number of functional a. Provide extra blankets and clothing to
capillaries in the alveoli keep the client warm.
c. Decreased residual volume b. Monitor the client for signs of
d. Decreased vital capacity restlessness, sweating, and excessive
weight loss during thyroid replacement During routine assessment, the nurse notices
therapy. Cheyne- Strokes respirations. Cheyne-strokes
c. Balance the client’s periods of activity respirations are:
and rest. a. A progressively deeper breaths followed
d. Encourage the client to be active to by shallower breaths with apneic
prevent constipation. periods.
b. Rapid, deep breathing with abrupt
22. Nurse Kris is teaching a client with history of pauses between each breath.
atherosclerosis. To decrease the risk of c. Rapid, deep breathing and irregular
atherosclerosis, the nurse should encourage the breathing without pauses.
client to: d. Shallow breathing with an increased
a. Avoid focusing on his weight. respiratory rate.
b. Increase his activity level.
c. Follow a regular diet. 28. Nurse Bea is assessing a male client with heart
d. Continue leading a high-stress lifestyle. failure. The breath sounds commonly
auscultated in clients with heart failure are:
23. Nurse Greta is working on a surgical floor. Nurse a. Tracheal
Greta must logroll a client following a: b. Fine crackles
a. Laminectomy c. Coarse crackles
b. Thoracotomy d. Friction rubs
c. Hemorrhoidectomy
d. Cystectomy. 29. The nurse is caring for Kenneth experiencing an
acute asthma attack. The client stops wheezing
24. A 55-year old client underwent cataract removal and breath sounds aren’t audible. The reason for
with intraocular lens implant. Nurse Oliver is this change is that:
giving the client discharge instructions. These a. The attack is over.
instructions should include which of the b. The airways are so swollen that no air
following? cannot get through.
a. Avoid lifting objects weighing more than c. The swelling has decreased.
5 lb (2.25 kg). d. Crackles have replaced wheezes.
b. Lie on your abdomen when in bed
c. Keep rooms brightly lit. 30. Mike with epilepsy is having a seizure. During
d. Avoiding straining during bowel the active seizure phase, the nurse should:
movement or bending at the waist. a. Place the client on his back remove
dangerous objects, and insert a bite
25. George should be taught about testicular block.
examinations during: b. Place the client on his side, remove
a. when sexual activity starts dangerous objects, and insert a bite
b. After age 69 block.
c. After age 40 c. Place the client o his back, remove
d. Before age 20. dangerous objects, and hold down his
26. A male client undergone a colon resection. While arms.
turning him, wound dehiscence with d. Place the client on his side, remove
evisceration occurs. Nurse Trish first response is dangerous objects, and protect his head.
to:
a. Call the physician 31. After insertion of a cheat tube for a
b. Place a saline-soaked sterile dressing on pneumothorax, a client becomes hypotensive
the wound. with neck vein distention, tracheal shift, absent
c. Take a blood pressure and pulse. breath sounds, and diaphoresis. Nurse Amanda
d. Pull the dehiscence closed. suspects a tension pneumothorax has occurred.
What cause of tension pneumothorax should the
27. Nurse Audrey is caring for a client who has nurse check for?
suffered a severe cerebrovascular accident. a. Infection of the lung.

29
29
b. Kinked or obstructed chest tube tuberculosis (TB). Which of the following clients
c. Excessive water in the water-seal entering the clinic today most likely to have TB?
chamber a. A 16-year-old female high school
d. Excessive chest tube drainage student
b. A 33-year-old day-care worker
32. Nurse Maureen is talking to a male client; the c. A 43-yesr-old homeless man with a
client begins choking on his lunch. He’s coughing history of alcoholism
forcefully. The nurse should: d. A 54-year-old businessman
a. Stand him up and perform the
abdominal thrust maneuver from 37. Virgie with a positive Mantoux test result will be
behind. sent for a chest X-ray. The nurse is aware that
b. Lay him down, straddle him, and which of the following reasons this is done?
perform the abdominal thrust a. To confirm the diagnosis
maneuver. b. To determine if a repeat skin test is
c. Leave him to get assistance needed
d. Stay with him but not intervene at this c. To determine the extent of lesions
time. d. To determine if this is a primary or
secondary infection
33. Nurse Ron is taking a health history of an 84 year
old client. Which information will be most useful 38. Kennedy with acute asthma showing inspiratory
to the nurse for planning care? and expiratory wheezes and a decreased forced
a. General health for the last 10 years. expiratory volume should be treated with which
b. Current health promotion activities. of the following classes of medication right
c. Family history of diseases. away?
d. Marital status. a. Beta-adrenergic blockers
b. Bronchodilators
34. When performing oral care on a comatose client, c. Inhaled steroids
Nurse Krina should: d. Oral steroids
a. Apply lemon glycerin to the client’s lips
at least every 2 hours. 39. Mr. Vasquez 56-year-old client with a 40-year
b. Brush the teeth with client lying supine. history of smoking one to two packs of cigarettes
c. Place the client in a side lying position, per day has a chronic cough producing thick
with the head of the bed lowered. sputum, peripheral edema and cyanotic nail
d. Clean the client’s mouth with hydrogen beds. Based on this information, he most likely
peroxide. has which of the following conditions?
a. Adult respiratory distress syndrome
35. A 77-year-old male client is admitted with a (ARDS)
diagnosis of dehydration and change in mental b. Asthma
status. He’s being hydrated with L.V. fluids. c. Chronic obstructive bronchitis
When the nurse takes his vital signs, she notes d. Emphysema
he has a fever of 103°F (39.4°C) a cough
producing yellow sputum and pleuritic chest Situation: Francis, age 46 is admitted to the hospital with
pain. The nurse suspects this client may have diagnosis of Chronic Lymphocytic Leukemia.
which of the following conditions?
a. Adult respiratory distress syndrome 40. The treatment for patients with leukemia is bone
(ARDS) marrow transplantation. Which statement about
b. Myocardial infarction (MI) bone marrow transplantation is not correct?
c. Pneumonia a. The patient is under local anesthesia
d. Tuberculosis during the procedure
b. The aspirated bone marrow is mixed
36. Nurse Oliver is working in an outpatient clinic. with heparin.
He has been alerted that there is an outbreak c. The aspiration site is the posterior or
of anterior iliac crest.
d. The recipient receives pressure of 126/76 mm Hg, and a
cyclophosphamide (Cytoxan) for 4 respiratory rate of 22 breaths/ minute.
consecutive days before the procedure. b. The 89-year-old client with end-stage
41. After several days of admission, Francis becomes right-sided heart failure, blood pressure
disoriented and complains of frequent of 78/50 mm Hg, and a “do not
headaches. The nurse in-charge first action resuscitate” order
would be: c. The 62-year-old client who was admitted
a. Call the physician 1 day ago with thrombophlebitis and is
b. Document the patient’s status in his receiving L.V. heparin
charts. d. The 75-year-old client who was admitted
c. Prepare oxygen treatment 1 hour ago with new-onset atrial
d. Raise the side rails fibrillation and is receiving L.V. dilitiazem
(Cardizem)
42. During routine care, Francis asks the nurse,
“How can I be anemic if this disease causes 46. Honey, a 23-year old client complains of
increased my white blood cell production?” The substernal chest pain and states that her heart
nurse in-charge best response would be that the feels like “it’s racing out of the chest”. She
increased number of white blood cells (WBC) is: reports no history of cardiac disorders. The
a. Crowd red blood cells nurse attaches her to a cardiac monitor and
b. Are not responsible for the anemia. notes sinus tachycardia with a rate of
c. Uses nutrients from other cells 136beats/minutes. Breath sounds are clear and
d. Have an abnormally short life span of the respiratory rate is 26 breaths/minutes.
cells. Which of the following drugs should the nurse
question the client about using?
43. Diagnostic assessment of Francis would probably a. Barbiturates
not reveal: b. Opioids
a. Predominance of lymhoblasts c. Cocaine
b. Leukocytosis d. Benzodiazepines
c. Abnormal blast cells in the bone marrow
d. Elevated thrombocyte counts 47. A 51-year-old female client tells the nurse in-
charge that she has found a painless lump in her
44. Robert, a 57-year-old client with acute arterial right breast during her monthly self-
occlusion of the left leg undergoes an examination. Which assessment finding would
emergency embolectomy. Six hours later, the strongly suggest that this client's lump is
nurse isn’t able to obtain pulses in his left foot cancerous?
using Doppler ultrasound. The nurse a. Eversion of the right nipple and mobile
immediately notifies the physician, and asks her mass
to prepare the client for surgery. As the nurse b. Nonmobile mass with irregular edges
enters the client’s room to prepare him, he c. Mobile mass that is soft and easily
states that he won’t have any more surgery. delineated
Which of the following is the best initial d. Nonpalpable right axillary lymph nodes
response by the nurse?
a. Explain the risks of not having 48. A 35-year-old client with vaginal cancer asks the
the surgery nurse, "What is the usual treatment for this type
b. Notifying the physician immediately of cancer?" Which treatment should the nurse
c. Notifying the nursing supervisor name?
d. Recording the client’s refusal in the a. Surgery
nurses’ notes b. Chemotherapy
c. Radiation
45. During the endorsement, which of the following d. Immunotherapy
clients should the on-duty nurse assess first?
a. The 58-year-old client who was admitted 49. Cristina undergoes a biopsy of a suspicious
2 days ago with heart failure, blood lesion. The biopsy report classifies the lesion

29
29
according to the TNM staging system as follows: a. prostate-specific antigen, which is used
TIS, N0, M0. What does this classification mean? to screen for prostate cancer.
a. No evidence of primary tumor, no b. protein serum antigen, which is used to
abnormal regional lymph nodes, and no determine protein levels.
evidence of distant metastasis c. pneumococcal strep antigen, which is a
b. Carcinoma in situ, no abnormal regional bacteria that causes pneumonia.
lymph nodes, and no evidence of distant d. Papanicolaou-specific antigen, which is
metastasis used to screen for cervical cancer.
c. Can't assess tumor or regional lymph
nodes and no evidence of metastasis 54. What is the most important postoperative
d. Carcinoma in situ, no demonstrable instruction that nurse Kate must give a client
metastasis of the regional lymph nodes, who has just returned from the operating room
and ascending degrees of distant after receiving a subarachnoid block?
metastasis a. "Avoid drinking liquids until the gag
reflex returns."
50. Lydia undergoes a laryngectomy to treat b. "Avoid eating milk products for 24
laryngeal cancer. When teaching the client how hours."
to care for the neck stoma, the nurse should c. "Notify a nurse if you experience blood
include which instruction? in your urine."
a. "Keep the stoma uncovered." d. "Remain supine for the time specified by
b. "Keep the stoma dry." the physician."
c. "Have a family member perform stoma
care initially until you get used to the 55. A male client suspected of having colorectal
procedure." cancer will require which diagnostic study to
d. "Keep the stoma moist." confirm the diagnosis?
a. Stool Hematest
51. A 37-year-old client with uterine cancer asks the b. Carcinoembryonic antigen (CEA)
nurse, "Which is the most common type of c. Sigmoidoscopy
cancer in women?" The nurse replies that it's d. Abdominal computed tomography (CT)
breast cancer. Which type of cancer causes the scan
most deaths in women?
a. Breast cancer 56. During a breast examination, which finding most
b. Lung cancer strongly suggests that the Luz has breast cancer?
c. Brain cancer a. Slight asymmetry of the breasts.
d. Colon and rectal cancer b. A fixed nodular mass with dimpling of
the overlying skin
52. Antonio with lung cancer develops Horner's c. Bloody discharge from the nipple
syndrome when the tumor invades the ribs and d. Multiple firm, round, freely movable
affects the sympathetic nerve ganglia. When masses that change with the menstrual
assessing for signs and symptoms of this cycle
syndrome, the nurse should note:
a. miosis, partial eyelid ptosis, and 57. A female client with cancer is being evaluated
anhidrosis on the affected side of the for possible metastasis. Which of the following is
face. one of the most common metastasis sites for
b. chest pain, dyspnea, cough, weight loss, cancer cells?
and fever. a. Liver
c. arm and shoulder pain and atrophy of b. Colon
arm and hand muscles, both on the c. Reproductive tract
affected side. d. White blood cells (WBCs)
d. hoarseness and dysphagia.
58. Nurse Mandy is preparing a client for magnetic
53. Vic asks the nurse what PSA is. The nurse should resonance imaging (MRI) to confirm or rule out a
reply that it stands for:
spinal cord lesion. During the MRI scan, which of 63. A 76-year-old male client had a thromboembolic
the following would pose a threat to the client? right stroke; his left arm is swollen. Which of the
a. The client lies still. following conditions may cause swelling after a
b. The client asks questions. stroke?
c. The client hears thumping sounds. a. Elbow contracture secondary to
d. The client wears a watch and wedding spasticity
band. b. Loss of muscle contraction decreasing
venous return
59. Nurse Cecile is teaching a female client about c. Deep vein thrombosis (DVT) due to
preventing osteoporosis. Which of the following immobility of the ipsilateral side
teaching points is correct? d. Hypoalbuminemia due to protein
a. Obtaining an X-ray of the bones every 3 escaping from an inflamed glomerulus
years is recommended to detect bone
loss. 64. Heberden’s nodes are a common sign of
b. To avoid fractures, the client should osteoarthritis. Which of the following statement
avoid strenuous exercise. is correct about this deformity?
c. The recommended daily allowance of a. It appears only in men
calcium may be found in a wide variety b. It appears on the distal interphalangeal
of foods. joint
d. Obtaining the recommended daily c. It appears on the proximal
allowance of calcium requires taking a interphalangeal joint
calcium supplement. d. It appears on the dorsolateral aspect of
the interphalangeal joint.
60. Before Jacob undergoes arthroscopy, the nurse
reviews the assessment findings for 65. Which of the following statements explains the
contraindications for this procedure. Which main difference between rheumatoid arthritis
finding is a contraindication? and osteoarthritis?
a. Joint pain a. Osteoarthritis is gender-specific,
b. Joint deformity rheumatoid arthritis isn’t
c. Joint flexion of less than 50% b. Osteoarthritis is a localized disease
d. Joint stiffness rheumatoid arthritis is systemic
c. Osteoarthritis is a systemic disease,
61. Mr. Rodriguez is admitted with severe pain in rheumatoid arthritis is localized
the knees. Which form of arthritis is d. Osteoarthritis has dislocations and
characterized by urate deposits and joint pain, subluxations, rheumatoid arthritis
usually in the feet and legs, and occurs primarily doesn’t
in men over age 30?
a. Septic arthritis 66. Mrs. Cruz uses a cane for assistance in walking.
b. Traumatic arthritis Which of the following statements is true about
c. Intermittent arthritis a cane or other assistive devices?
d. Gouty arthritis a. A walker is a better choice than a cane.
b. The cane should be used on the affected
62. A heparin infusion at 1,500 unit/hour is ordered side
for a 64-year-old client with stroke in evolution. c. The cane should be used on the
The infusion contains 25,000 units of heparin in unaffected side
500 ml of saline solution. How many milliliters d. A client with osteoarthritis should be
per hour should be given? encouraged to ambulate without the
a. 15 ml/hour cane
b. 30 ml/hour
c. 45 ml/hour 67. A male client with type 1 diabetes is scheduled
d. 50 ml/hour to receive 30 U of 70/30 insulin. There is no
70/30 insulin available. As a substitution, the
nurse may give the client:

29
29
a. 9 U regular insulin and 21 U neutral c. Restricting fluids
protamine Hagedorn (NPH). d. Administering glucose-containing I.V.
b. 21 U regular insulin and 9 U NPH. fluids as ordered
c. 10 U regular insulin and 20 U NPH.
d. 20 U regular insulin and 10 U NPH. 73. A female client tells nurse Nikki that she has
been working hard for the last 3 months to
68. Nurse Len should expect to administer which control her type 2 diabetes mellitus with diet
medication to a client with gout? and exercise. To determine the effectiveness of
a. aspirin the client's efforts, the nurse should check:
b. furosemide (Lasix) a. urine glucose level.
c. colchicines b. fasting blood glucose level.
d. calcium gluconate (Kalcinate) c. serum fructosamine level.
d. glycosylated hemoglobin level.
69. Mr. Domingo with a history of hypertension is
diagnosed with primary hyperaldosteronism. 74. Nurse Trinity administered neutral protamine
This diagnosis indicates that the client's Hagedorn (NPH) insulin to a diabetic client at 7
hypertension is caused by excessive hormone a.m. At what time would the nurse expect the
secretion from which of the following glands? client to be most at risk for a hypoglycemic
a. Adrenal cortex reaction?
b. Pancreas a. 10:00 am
c. Adrenal medulla b. Noon
d. Parathyroid c. 4:00 pm
d. 10:00 pm
70. For a diabetic male client with a foot ulcer, the
doctor orders bed rest, a wet-to-dry dressing 75. The adrenal cortex is responsible for producing
change every shift, and blood glucose which substances?
monitoring before meals and bedtime. Why are a. Glucocorticoids and androgens
wet-to-dry dressings used for this client? b. Catecholamines and epinephrine
a. They contain exudate and provide a c. Mineralocorticoids and catecholamines
moist wound environment. d. Norepinephrine and epinephrine
b. They protect the wound from
mechanical trauma and promote 76. On the third day after a partial thyroidectomy,
healing. Proserfina exhibits muscle twitching and
c. They debride the wound and promote hyperirritability of the nervous system. When
healing by secondary intention. questioned, the client reports numbness and
d. They prevent the entrance of tingling of the mouth and fingertips. Suspecting
microorganisms and minimize wound a life-threatening electrolyte disturbance, the
discomfort. nurse notifies the surgeon immediately. Which
electrolyte disturbance most commonly follows
71. Nurse Zeny is caring for a client in acute thyroid surgery?
addisonian crisis. Which laboratory data would a. Hypocalcemia
the nurse expect to find? b. Hyponatremia
a. Hyperkalemia c. Hyperkalemia
b. Reduced blood urea nitrogen (BUN) d. Hypermagnesemia
c. Hypernatremia
d. Hyperglycemia 77. Which laboratory test value is elevated in clients
who smoke and can't be used as a general
72. A client is admitted for treatment of the indicator of cancer?
syndrome of inappropriate antidiuretic hormone a. Acid phosphatase level
(SIADH). Which nursing intervention is b. Serum calcitonin level
appropriate? c. Alkaline phosphatase level
a. Infusing I.V. fluids rapidly as ordered d. Carcinoembryonic antigen level
b. Encouraging increased oral intake
78. Francis with anemia has been admitted to the c. Administer the antidote for penicillin, as
medical-surgical unit. Which assessment findings prescribed, and continue to monitor the
are characteristic of iron-deficiency anemia? client's vital signs.
a. Nights sweats, weight loss, and diarrhea d. Insert an indwelling urinary catheter and
b. Dyspnea, tachycardia, and pallor begin to infuse I.V. fluids as ordered.
c. Nausea, vomiting, and anorexia
d. Itching, rash, and jaundice 83. Mr. Marquez with rheumatoid arthritis is about
to begin aspirin therapy to reduce inflammation.
79. In teaching a female client who is HIV-positive When teaching the client about aspirin, the
about pregnancy, the nurse would know more nurse discusses adverse reactions to prolonged
teaching is necessary when the client says: aspirin therapy. These include:
a. The baby can get the virus from my a. weight gain.
placenta." b. fine motor tremors.
b. "I'm planning on starting on birth control c. respiratory acidosis.
pills." d. bilateral hearing loss.
c. "Not everyone who has the virus gives
birth to a baby who has the virus." 84. A 23-year-old client is diagnosed with human
d. "I'll need to have a C-section if I become immunodeficiency virus (HIV). After recovering
pregnant and have a baby." from the initial shock of the diagnosis, the client
expresses a desire to learn as much as possible
80. When preparing Judy with acquired about HIV and acquired immunodeficiency
immunodeficiency syndrome (AIDS) for syndrome (AIDS). When teaching the client
discharge to the home, the nurse should be sure about the immune system, the nurse states that
to include which instruction? adaptive immunity is provided by which type of
a. "Put on disposable gloves before white blood cell?
bathing." a. Neutrophil
b. "Sterilize all plates and utensils in boiling b. Basophil
water." c. Monocyte
c. "Avoid sharing such articles as d. Lymphocyte
toothbrushes and razors."
d. "Avoid eating foods from serving dishes 85. In an individual with Sjögren's syndrome, nursing
shared by other family members." care should focus on:
a. moisture replacement.
81. Nurse Marie is caring for a 32-year-old client b. electrolyte balance.
admitted with pernicious anemia. Which set of c. nutritional supplementation.
findings should the nurse expect when assessing d. arrhythmia management.
the client?
a. Pallor, bradycardia, and reduced pulse 86. During chemotherapy for lymphocytic leukemia,
pressure Mathew develops abdominal pain, fever, and
b. Pallor, tachycardia, and a sore tongue "horse barn" smelling diarrhea. It would be most
c. Sore tongue, dyspnea, and weight gain important for the nurse to advise the physician
d. Angina, double vision, and anorexia to order:
a. enzyme-linked immunosuppressant
82. After receiving a dose of penicillin, a client assay (ELISA) test.
develops dyspnea and hypotension. Nurse b. electrolyte panel and hemogram.
Celestina suspects the client is experiencing c. stool for Clostridium difficile test.
anaphylactic shock. What should the nurse do d. flat plate X-ray of the abdomen.
first?
a. Page an anesthesiologist immediately 87. A male client seeks medical evaluation for
and prepare to intubate the client. fatigue, night sweats, and a 20-lb weight loss in 6
b. Administer epinephrine, as prescribed, weeks. To confirm that the client has been
and prepare to intubate the client if infected with the human immunodeficiency virus
necessary. (HIV), the nurse expects the physician to order:

29
29
a. E-rosette immunofluorescence. d. A client with rheumatoid arthritis who
b. quantification of T-lymphocytes. states, “I am having trouble sleeping.”
c. enzyme-linked immunosorbent assay
(ELISA). 92. Nurse Sarah is caring for clients on the surgical
d. Western blot test with ELISA. floor and has just received report from the
previous shift. Which of the following clients
88. A complete blood count is commonly performed should the nurse see first?
before a Joe goes into surgery. What does this a. A 35-year-old admitted three hours ago
test seek to identify? with a gunshot wound; 1.5 cm area of
a. Potential hepatic dysfunction indicated dark drainage noted on the dressing.
by decreased blood urea nitrogen (BUN) b. A 43-year-old who had a mastectomy
and creatinine levels two days ago; 23 ml of serosanguinous
b. Low levels of urine constituents normally fluid noted in the Jackson-Pratt drain.
excreted in the urine c. A 59-year-old with a collapsed lung due
c. Abnormally low hematocrit (HCT) and to an accident; no drainage noted in the
hemoglobin (Hb) levels previous eight hours.
d. Electrolyte imbalance that could affect d. A 62-year-old who had an abdominal-
the blood's ability to coagulate properly perineal resection three days ago; client
complaints of chills.
89. While monitoring a client for the development
of disseminated intravascular coagulation (DIC), 93. Nurse Eve is caring for a client who had a
the nurse should take note of what assessment thyroidectomy 12 hours ago for treatment of
parameters? Grave’s disease. The nurse would be most
a. Platelet count, prothrombin time, and concerned if which of the following was
partial thromboplastin time observed?
b. Platelet count, blood glucose levels, and a. Blood pressure 138/82, respirations 16,
white blood cell (WBC) count oral temperature 99 degrees Fahrenheit.
c. Thrombin time, calcium levels, and b. The client supports his head and neck
potassium levels when turning his head to the right.
d. Fibrinogen level, WBC, and platelet c. The client spontaneously flexes his wrist
count when the blood pressure is obtained.
d. The client is drowsy and complains of
90. When taking a dietary history from a newly sore throat.
admitted female client, Nurse Len should
remember that which of the following foods is a 94. Julius is admitted with complaints of severe pain
common allergen? in the lower right quadrant of the abdomen. To
a. Bread assist with pain relief, the nurse should take
b. Carrots which of the following actions?
c. Orange a. Encourage the client to change positions
d. Strawberries frequently in bed.
b. Administer Demerol 50 mg IM q 4 hours
91. Nurse John is caring for clients in the outpatient and PRN.
clinic. Which of the following phone calls should c. Apply warmth to the abdomen with a
the nurse return first? heating pad.
a. A client with hepatitis A who states, “My d. Use comfort measures and pillows to
arms and legs are itching.” position the client.
b. A client with cast on the right leg who
states, “I have a funny feeling in my right 95. Nurse Tina prepares a client for peritoneal
leg.” dialysis. Which of the following actions should
c. A client with osteomyelitis of the spine the nurse take first?
who states, “I am so nauseous that I a. Assess for a bruit and a thrill.
can’t eat.” b. Warm the dialysate solution.
c. Position the client on the left side.
d. Insert a Foley catheter takes small steps while balancing on the
walker.
96. Nurse Jannah teaches an elderly client with d. The client slides the walker 18 inches
right-sided weakness how to use cane. Which of forward, then takes small steps while
the following behaviors, if demonstrated by the holding onto the walker for balance.
client to the nurse, indicates that the teaching
was effective? 99. Nurse Deric is supervising a group of elderly
a. The client holds the cane with his right clients in a residential home setting. The nurse
hand, moves the can forward followed knows that the elderly are at greater risk of
by the right leg, and then moves the left developing sensory deprivation for what reason?
leg. a. Increased sensitivity to the side effects
b. The client holds the cane with his right of medications.
hand, moves the cane forward followed b. Decreased visual, auditory, and
by his left leg, and then moves the right gustatory abilities.
leg. c. Isolation from their families and familiar
c. The client holds the cane with his left surroundings.
hand, moves the cane forward followed d. Decrease musculoskeletal function and
by the right leg, and then moves the left mobility.
leg.
d. The client holds the cane with his left 100. A male client with emphysema becomes
hand, moves the cane forward followed restless and confused. What step should
by his left leg, and then moves the right nurse Jasmine take next?
leg. a. Encourage the client to perform pursed
lip breathing.
97. An elderly client is admitted to the nursing home b. Check the client’s temperature.
setting. The client is occasionally confused and c. Assess the client’s potassium level.
her gait is often unsteady. Which of the d. Increase the client’s oxygen flow rate.
following actions, if taken by the nurse, is most
appropriate?
a. Ask the woman’s family to provide
personal items such as photos or
mementos.
b. Select a room with a bed by the door so
the woman can look down the hall.
c. Suggest the woman eat her meals in the
room with her roommate.
d. Encourage the woman to ambulate in
the halls twice a day.

98. Nurse Evangeline teaches an elderly client how


to use a standard aluminum walker. Which of
the following behaviors, if demonstrated by the
client, indicates that the nurse’s teaching was
effective?
a. The client slowly pushes the walker
forward 12 inches, then takes small
steps forward while leaning on the
walker.
b. The client lifts the walker, moves it
forward 10 inches, and then takes
several small steps forward.
c. The client supports his weight on the
walker while advancing it forward, then

30
30
Answers and Rationale – Care of Clients with these enzymes is begun at once. Skin exposed
Physiologic and Psychosocial Alterations to these enzymes even for a short time
becomes reddened, painful, and excoriated.
1. Answer: (C) Loose, bloody 9. Answer: (B) Flat on back.
Rationale: Normal bowel function and soft- Rationale: To avoid the complication of a
formed stool usually do not occur until around painful spinal headache that can last for
the seventh day following surgery. The stool several days, the client is kept in flat in a
consistency is related to how much water is supine position for approximately 4 to 12
being absorbed. hours postoperatively. Headaches are
2. Answer: (A) On the client’s right side believed to be causes by the seepage of
Rationale: The client has left visual field cerebral spinal fluid from the puncture site. By
blindness. The client will see only from the keeping the client flat, cerebral spinal fluid
right side. pressures are equalized, which avoids trauma
3. Answer: (C) Check respirations, stabilize spine, to the neurons.
and check circulation 10. Answer: (C) The client is oriented when
Rationale: Checking the airway would be aroused from sleep, and goes back to sleep
priority, and a neck injury should be immediately.
suspected. Rationale: This finding suggest that the level
4. Answer: (D) Decreasing venous return through of consciousness is decreasing.
vasodilation. 11. Answer: (A) Altered mental status and
Rationale: The significant effect of dehydration
nitroglycerin is vasodilation and decreased Rationale: Fever, chills, hemortysis, dyspnea,
venous return, so the heart does not have to cough, and pleuritic chest pain are the
work hard. common symptoms of pneumonia, but elderly
5. Answer: (A) Call for help and note the time. clients may first appear with only an altered
Rationale: Having established, by stimulating lentil status and dehydration due to a blunted
the client, that the client is unconscious rather immune response.
than sleep, the nurse should immediately call 12. Answer: (B) Chills, fever, night sweats, and
for help. This may be done by dialing the hemoptysis
operator from the client’s phone and giving Rationale: Typical signs and symptoms are
the hospital code for cardiac arrest and the chills, fever, night sweats, and hemoptysis.
client’s room number to the operator, of if the Chest pain may be present from coughing, but
phone is not available, by pulling the isn’t usual. Clients with TB typically have low-
emergency call button. Noting the time is grade fevers, not higher than 102°F (38.9°C).
important baseline information for cardiac Nausea, headache, and photophobia aren’t
arrest procedure usual TB symptoms.
6. Answer: (C) Make sure that the client takes 13. Answer:(A) Acute asthma
food and medications at prescribed intervals. Rationale: Based on the client’s history and
Rationale: Food and drug therapy will prevent symptoms, acute asthma is the most likely
the accumulation of hydrochloric acid, or will diagnosis. He’s unlikely to have bronchial
neutralize and buffer the acid that does pneumonia without a productive cough and
accumulate. fever and he’s too young to have developed
7. Answer: (B) Continue treatment as ordered. (COPD) and emphysema.
Rationale: The effects of heparin are 14. Answer: (B) Respiratory arrest
monitored by the PTT is normally 30 to 45 Rationale: Narcotics can cause respiratory
seconds; the therapeutic level is 1.5 to 2 times arrest if given in large quantities. It’s unlikely
the normal level. the client will have asthma attack or a seizure
8. Answer: (B) In the operating room. or wake up on his own.
Rationale: The stoma drainage bag is applied 15. Answer: (D) Decreased vital capacity
in the operating room. Drainage from the Rationale: Reduction in vital capacity is a
ileostomy contains secretions that are rich in normal physiologic change includes decreased
digestive enzymes and highly irritating to the elastic recoil of the lungs, fewer functional
skin. Protection of the skin from the effects of
capillaries in the alveoli, and an increased in hyperthyroidism are hyperactive and complain
residual volume. of feeling very warm.
16. Answer: (C) Presence of premature ventricular 22. Answer: (B) Increase his activity level.
contractions (PVCs) on a cardiac monitor. Rationale: The client should be encouraged to
Rationale: Lidocaine drips are commonly used increase his activity level. aintaining an ideal
to treat clients whose arrhythmias haven’t weight; following a low-cholesterol, low
been controlled with oral medication and who sodium diet; and avoiding stress are all
are having PVCs that are visible on the cardiac important factors in decreasing the risk of
monitor. SaO2, blood pressure, and ICP are atherosclerosis.
important factors but aren’t as significant as 23. Answer: (A) Laminectomy
PVCs in the situation. Rationale: The client who has had spinal
17. Answer: (B) Avoid foods high in vitamin K surgery, such as laminectomy, must be log
Rationale: The client should avoid consuming rolled to keep the spinal column straight when
large amounts of vitamin K because vitamin K turning. Thoracotomy and cystectomy may
can interfere with anticoagulation. The client turn themselves or may be assisted into a
may need to report diarrhea, but isn’t effect comfortable position. Under normal
of taking an anticoagulant. An electric razor- circumstances, hemorrhoidectomy is an
not a straight razor-should be used to prevent outpatient procedure, and the client may
cuts that cause bleeding. Aspirin may increase resume normal activities immediately after
the risk of bleeding; acetaminophen should be surgery.
used to pain relief. 24. Answer: (D) Avoiding straining during bowel
18. Answer: (C) Clipping the hair in the area movement or bending at the waist.
Rationale: Hair can be a source of infection Rationale: The client should avoid straining,
and should be removed by clipping. Shaving lifting heavy objects, and coughing harshly
the area can cause skin abrasions and because these activities increase intraocular
depilatories can irritate the skin. pressure. Typically, the client is instructed to
19. Answer: (A) Bone fracture avoid lifting objects weighing more than 15 lb
Rationale: Bone fracture is a major (7kg) – not 5lb. instruct the client when lying
complication of osteoporosis that results in bed to lie on either the side or back. The
when loss of calcium and phosphate increased client should avoid bright light by wearing
the fragility of bones. Estrogen deficiencies sunglasses.
result from menopause-not osteoporosis. 25. Answer: (D) Before age 20.
Calcium and vitamin D supplements may be Rationale: Testicular cancer commonly occurs
used to support normal bone metabolism, But in men between ages 20 and 30. A male client
a negative calcium balance isn’t a should be taught how to perform testicular
complication of osteoporosis. Dowager’s self- examination before age 20, preferably
hump results from bone fractures. It develops when he enters his teens.
when repeated vertebral fractures increase 26. Answer: (B) Place a saline-soaked sterile
spinal curvature. dressing on the wound.
20. Answer: (C) Changes from previous Rationale: The nurse should first place saline-
examinations. soaked sterile dressings on the open wound to
Rationale: Women are instructed to examine prevent tissue drying and possible infection.
themselves to discover changes that have Then the nurse should call the physician and
occurred in the breast. Only a physician can take the client’s vital signs. The dehiscence
diagnose lumps that are cancerous, areas of needs to be surgically closed, so the nurse
thickness or fullness that signal the presence should never try to close it.
of a malignancy, or masses that are fibrocystic 27. Answer: (A) A progressively deeper breaths
as opposed to malignant. followed by shallower breaths with apneic
21. Answer: (C) Balance the client’s periods of periods.
activity and rest. Rationale: Cheyne-Strokes respirations are
Rationale: A client with hyperthyroidism breaths that become progressively deeper
needs to be encouraged to balance periods of fallowed by shallower respirations with
activity and rest. Many clients with apneas periods. Biot’s respirations are rapid,

30
30
deep breathing with abrupt pauses between Rationale: Recognizing an individual’s positive
each breath, and equal depth between each health measures is very useful. General health
breath. Kussmaul’s respirationa are rapid, in the previous 10 years is important,
deep breathing without pauses. Tachypnea is however, the current activities of an 84 year
shallow breathing with increased respiratory old client are most significant in planning care.
rate. Family history of disease for a client in later
28. Answer: (B) Fine crackles years is of minor significance. Marital status
Rationale: Fine crackles are caused by fluid in information may be important for discharge
the alveoli and commonly occur in clients with planning but is not as significant for
heart failure. Tracheal breath sounds are addressing the immediate medical problem.
auscultated over the trachea. Coarse crackles 34. Answer: (C) Place the client in a side lying
are caused by secretion accumulation in the position, with the head of the bed lowered.
airways. Friction rubs occur with pleural Rationale: The client should be positioned in a
inflammation. side-lying position with the head of the bed
29. Answer: (B) The airways are so swollen that no lowered to prevent aspiration. A small amount
air cannot get through of toothpaste should be used and the mouth
Rationale: During an acute attack, wheezing swabbed or suctioned to remove pooled
may stop and breath sounds become secretions. Lemon glycerin can be drying if
inaudible because the airways are so swollen used for extended periods. Brushing the teeth
that air can’t get through. If the attack is over with the client lying supine may lead to
and swelling has decreased, there would be aspiration. Hydrogen peroxide is caustic to
no more wheezing and less emergent concern. tissues and should not be used.
Crackles do not replace wheezes during an 35. Answer: (C) Pneumonia
acute asthma attack. Rationale: Fever productive cough and
30. Answer: (D) Place the client on his side, pleuritic chest pain are common signs and
remove dangerous objects, and protect his symptoms of pneumonia. The client with
head. ARDS has dyspnea and hypoxia with
Rationale: During the active seizure phase, worsening hypoxia over time, if not treated
initiate precautions by placing the client on his aggressively. Pleuritic chest pain varies with
side, removing dangerous objects, and respiration, unlike the constant chest pain
protecting his head from injury. A bite block during an MI; so this client most likely isn’t
should never be inserted during the active having an MI. the client with TB typically has a
seizure phase. Insertion can break the teeth cough producing blood-tinged sputum. A
and lead to aspiration. sputum culture should be obtained to confirm
31. Answer: (B) Kinked or obstructed chest tube the nurse’s suspicions.
Rationales: Kinking and blockage of the chest 36. Answer: (C) A 43-yesr-old homeless man with
tube is a common cause of a tension a history of alcoholism
pneumothorax. Infection and excessive Rationale: Clients who are economically
drainage won’t cause a tension disadvantaged, malnourished, and have
pneumothorax. Excessive water won’t affect reduced immunity, such as a client with a
the chest tube drainage. history of alcoholism, are at extremely high
32. Answer: (D) Stay with him but not intervene at risk for developing TB. A high school student,
this time. day- care worker, and businessman probably
Rationale: If the client is coughing, he should have a much low risk of contracting TB.
be able to dislodge the object or cause a 37. Answer: (C ) To determine the extent of
complete obstruction. If complete obstruction lesions
occurs, the nurse should perform the Rationale: If the lesions are large enough, the
abdominal thrust maneuver with the client chest X-ray will show their presence in the
standing. If the client is unconscious, she lungs. Sputum culture confirms the diagnosis.
should lay him down. A nurse should never There can be false-positive and false-negative
leave a choking client alone. skin test results. A chest X-ray can’t determine
33. Answer: (B) Current health promotion if this is a primary or secondary infection.
activities 38. Answer: (B) Bronchodilators
Rationale: Bronchodilators are the first line of Rationale: The client with atrial fibrillation has
treatment for asthma because broncho- the greatest potential to become unstable and
constriction is the cause of reduced airflow. is on L.V. medication that requires close
Beta- adrenergic blockers aren’t used to treat monitoring. After assessing this client, the
asthma and can cause broncho- constriction. nurse should assess the client with
Inhaled oral steroids may be given to reduce thrombophlebitis who is receiving a heparin
the inflammation but aren’t used for infusion, and then the 58- year-old client
emergency relief. admitted 2 days ago with heart failure (his
39. Answer: (C) Chronic obstructive bronchitis signs and symptoms are resolving and don’t
Rationale: Because of this extensive smoking require immediate attention). The lowest
history and symptoms the client most likely priority is the 89-year-old with end-stage
has chronic obstructive bronchitis. Client with right-sided heart failure, who requires time-
ARDS have acute symptoms of hypoxia and consuming supportive measures.
typically need large amounts of oxygen. 46. Answer: (C) Cocaine
Clients with asthma and emphysema tend not Rationale: Because of the client’s age and
to have chronic cough or peripheral edema. negative medical history, the nurse should
40. Answer: (A) The patient is under local question her about cocaine use. Cocaine
anesthesia during the procedure Rationale: increases myocardial oxygen consumption and
Before the procedure, the patient is can cause coronary artery spasm, leading to
administered with drugs that would help to tachycardia, ventricular fibrillation, myocardial
prevent infection and rejection of the ischemia, and myocardial infarction.
transplanted cells such as antibiotics, Barbiturate overdose may trigger respiratory
cytotoxic, and corticosteroids. During the depression and slow pulse. Opioids can cause
transplant, the patient is placed under general marked respiratory depression, while
anesthesia. benzodiazepines can cause drowsiness and
41. Answer: (D) Raise the side rails confusion.
Rationale: A patient who is disoriented is at 47. Answer: (B) Nonmobile mass with irregular
risk of falling out of bed. The initial action of edges
the nurse should be raising the side rails to Rationale: Breast cancer tumors are fixed,
ensure patients safety. hard, and poorly delineated with irregular
42. Answer: (A) Crowd red blood cells edges. A mobile mass that is soft and easily
Rationale: The excessive production of white delineated is most often a fluid-filled benign
blood cells crowd out red blood cells cyst. Axillary lymph nodes may or may not be
production which causes anemia to occur. palpable on initial detection of a cancerous
43. Answer: (B) Leukocytosis mass. Nipple retraction — not eversion —
Rationale: Chronic Lymphocytic leukemia (CLL) may be a sign of cancer.
is characterized by increased production of 48. Answer: (C) Radiation
leukocytes and lymphocytes resulting in Rationale: The usual treatment for vaginal
leukocytosis, and proliferation of these cells cancer is external or intravaginal radiation
within the bone marrow, spleen and liver. therapy. Less often, surgery is performed.
44. Answer: (A) Explain the risks of not having the Chemotherapy typically is prescribed only if
surgery vaginal cancer is diagnosed in an early stage,
Rationale: The best initial response is to which is rare. Immunotherapy isn't used to
explain the risks of not having the surgery. If treat vaginal cancer.
the client understands the risks but still 49. Answer: (B) Carcinoma in situ, no abnormal
refuses the nurse should notify the physician regional lymph nodes, and no evidence of
and the nurse supervisor and then record the distant metastasis
client’s refusal in the nurses’ notes. Rationale: TIS, N0, M0 denotes carcinoma in
45. Answer: (D) The 75-year-old client who was situ, no abnormal regional lymph nodes, and
admitted 1 hour ago with new-onset atrial no evidence of distant metastasis. No
fibrillation and is receiving L.V. dilitiazem evidence of primary tumor, no abnormal
(Cardizem) regional lymph nodes, and no evidence of
distant metastasis is classified as T0, N0, M0. If

30
30
the tumor and regional lymph nodes can't be nurse should instruct the client to remain
assessed and no evidence of metastasis exists, supine for the time specified by the physician.
the lesion is classified as TX, NX, M0. A Local anesthetics used in a subarachnoid block
progressive increase in tumor size, no don't alter the gag reflex. No interactions
demonstrable metastasis of the regional between local anesthetics and food occur.
lymph nodes, and ascending degrees of Local anesthetics don't cause hematuria.
distant metastasis is classified as T1, T2, T3, or 55. Answer: (C) Sigmoidoscopy
T4; N0; and M1, M2, or M3. Rationale: Used to visualize the lower GI tract,
50. Answer: (D) "Keep the stoma moist." sigmoidoscopy and proctoscopy aid in the
Rationale: The nurse should instruct the client detection of two-thirds of all colorectal
to keep the stoma moist, such as by applying a cancers. Stool Hematest detects blood, which
thin layer of petroleum jelly around the edges, is a sign of colorectal cancer; however, the
because a dry stoma may become irritated. test doesn't confirm the diagnosis. CEA may
The nurse should recommend placing a stoma be elevated in colorectal cancer but isn't
bib over the stoma to filter and warm air considered a confirming test. An abdominal CT
before it enters the stoma. The client should scan is used to stage the presence of
begin performing stoma care without colorectal cancer.
assistance as soon as possible to gain 56. Answer: (B) A fixed nodular mass with
independence in self-care activities. dimpling of the overlying skin
51. Answer: (B) Lung cancer Rationale: A fixed nodular mass with dimpling
Rationale: Lung cancer is the most deadly type of the overlying skin is common during late
of cancer in both women and men. Breast stages of breast cancer. Many women have
cancer ranks second in women, followed (in slightly asymmetrical breasts. Bloody nipple
descending order) by colon and rectal cancer, discharge is a sign of intraductal papilloma, a
pancreatic cancer, ovarian cancer, uterine benign condition. Multiple firm, round, freely
cancer, lymphoma, leukemia, liver cancer, movable masses that change with the
brain cancer, stomach cancer, and multiple menstrual cycle indicate fibrocystic breasts, a
myeloma. benign condition.
52. Answer: (A) miosis, partial eyelid ptosis, and 57. Answer: (A) Liver
anhidrosis on the affected side of the face. Rationale: The liver is one of the five most
Rationale: Horner's syndrome, which occurs common cancer metastasis sites. The others
when a lung tumor invades the ribs and are the lymph nodes, lung, bone, and brain.
affects the sympathetic nerve ganglia, is The colon, reproductive tract, and WBCs are
characterized by miosis, partial eyelid ptosis, occasional metastasis sites.
and anhidrosis on the affected side of the 58. Answer: (D) The client wears a watch and
face. Chest pain, dyspnea, cough, weight loss, wedding band.
and fever are associated with pleural tumors. Rationale: During an MRI, the client should
Arm and shoulder pain and atrophy of the arm wear no metal objects, such as jewelry,
and hand muscles on the affected side suggest because the strong magnetic field can pull on
Pancoast's tumor, a lung tumor involving the them, causing injury to the client and (if they
first thoracic and eighth cervical nerves within fly off) to others. The client must lie still
the brachial plexus. Hoarseness in a client during the MRI but can talk to those
with lung cancer suggests that the tumor has performing the test by way of the microphone
extended to the recurrent laryngeal nerve; inside the scanner tunnel. The client should
dysphagia suggests that the lung tumor is hear thumping sounds, which are caused by
compressing the esophagus. the sound waves thumping on the magnetic
53. 53. Answer: (A) prostate-specific antigen, field.
which is used to screen for prostate cancer. 59. Answer: (C) The recommended daily
Rationale: PSA stands for prostate-specific allowance of calcium may be found in a wide
antigen, which is used to screen for prostate variety of foods.
cancer. The other answers are incorrect. Rationale: Premenopausal women require
54. Answer: (D) "Remain supine for the time 1,000 mg of calcium per day. Postmenopausal
specified by the physician." Rationale: The women require 1,500 mg per day. It's often,
though not always, possible to get the Rationale: Heberden’s nodes appear on the
recommended daily requirement in the foods distal interphalageal joint on both men and
we eat. Supplements are available but not women. Bouchard’s node appears on the
always necessary. Osteoporosis doesn't show dorsolateral aspect of the proximal
up on ordinary X-rays until 30% of the bone interphalangeal joint.
loss has occurred. Bone densitometry can 65. Answer: (B) Osteoarthritis is a localized
detect bone loss of 3% or less. This test is disease rheumatoid arthritis is systemic
sometimes recommended routinely for Rationale: Osteoarthritis is a localized disease,
women over 35 who are at risk. Strenuous rheumatoid arthritis is systemic. Osteoarthritis
exercise won't cause fractures. isn’t gender-specific, but rheumatoid arthritis
60. Answer: (C) Joint flexion of less than 50% is. Clients have dislocations and subluxations
Rationale: Arthroscopy is contraindicated in in both disorders.
clients with joint flexion of less than 50% 66. Answer: (C) The cane should be used on the
because of technical problems in inserting the unaffected side
instrument into the joint to see it clearly. Rationale: A cane should be used on the
Other contraindications for this procedure unaffected side. A client with osteoarthritis
include skin and wound infections. Joint pain should be encouraged to ambulate with a
may be an indication, not a contraindication, cane, walker, or other assistive device as
for arthroscopy. Joint deformity and joint needed; their use takes weight and stress off
stiffness aren't contraindications for this joints.
procedure. 67. Answer: (A) a. 9 U regular insulin and 21 U
61. Answer: (D) Gouty arthritis neutral protamine Hagedorn (NPH).
Rationale: Gouty arthritis, a metabolic disease, Rationale: A 70/30 insulin preparation is 70%
is characterized by urate deposits and pain in NPH and 30% regular insulin. Therefore, a
the joints, especially those in the feet and correct substitution requires mixing 21 U of
legs. Urate deposits don't occur in septic or NPH and 9 U of regular insulin. The other
traumatic arthritis. Septic arthritis results from choices are incorrect dosages for the
bacterial invasion of a joint and leads to prescribed insulin.
inflammation of the synovial lining. Traumatic 68. Answer: (C) colchicines
arthritis results from blunt trauma to a joint or Rationale: A disease characterized by joint
ligament. Intermittent arthritis is a rare, inflammation (especially in the great toe),
benign condition marked by regular, recurrent gout is caused by urate crystal deposits in the
joint effusions, especially in the knees. joints. The physician prescribes colchicine to
62. Answer: (B) 30 ml/hou reduce these deposits and thus ease joint
Rationale: An infusion prepared with 25,000 inflammation. Although aspirin is used to
units of heparin in 500 ml of saline solution reduce joint inflammation and pain in clients
yields 50 units of heparin per milliliter of with osteoarthritis and rheumatoid arthritis, it
solution. The equation is set up as 50 units isn't indicated for gout because it has no
times X (the unknown quantity) equals 1,500 effect on urate crystal formation. Furosemide,
units/hour, X equals 30 ml/hour. a diuretic, doesn't relieve gout. Calcium
63. Answer: (B) Loss of muscle contraction gluconate is used to reverse a negative
decreasing venous return calcium balance and relieve muscle cramps,
Rationale: In clients with hemiplegia or not to treat gout.
hemiparesis loss of muscle contraction 69. Answer: (A) Adrenal cortex
decreases venous return and may cause Rationale: Excessive secretion of aldosterone
swelling of the affected extremity. in the adrenal cortex is responsible for the
Contractures, or bony calcifications may occur client's hypertension. This hormone acts on
with a stroke, but don’t appear with swelling. the renal tubule, where it promotes
DVT may develop in clients with a stroke but is reabsorption of sodium and excretion of
more likely to occur in the lower extremities. potassium and hydrogen ions. The pancreas
A stroke isn’t linked to protein loss. mainly secretes hormones involved in fuel
64. Answer: (B) It appears on the distal metabolism. The adrenal medulla secretes the
interphalangeal joint catecholamines — epinephrine and

30
30
norepinephrine. The parathyroids secrete produces three types of hormones:
parathyroid hormone. glucocorticoids, mineralocorticoids, and
70. Answer: (C) They debride the wound and androgens. The medulla produces
promote healing by secondary intention catecholamines— epinephrine and
Rationale: For this client, wet-to-dry dressings norepinephrine.
are most appropriate because they clean the 76. Answer: (A) Hypocalcemia
foot ulcer by debriding exudate and necrotic Rationale: Hypocalcemia may follow thyroid
tissue, thus promoting healing by secondary surgery if the parathyroid glands were
intention. Moist, transparent dressings removed accidentally. Signs and symptoms of
contain exudate and provide a moist wound hypocalcemia may be delayed for up to 7 days
environment. Hydrocolloid dressings prevent after surgery. Thyroid surgery doesn't directly
the entrance of microorganisms and minimize cause serum sodium, potassium, or
wound discomfort. Dry sterile dressings magnesium abnormalities. Hyponatremia may
protect the wound from mechanical trauma occur if the client inadvertently received too
and promote healing. much fluid; however, this can happen to any
71. Answer: (A) Hyperkalemia surgical client receiving I.V. fluid therapy, not
Rationale: In adrenal insufficiency, the client just one recovering from thyroid surgery.
has hyperkalemia due to reduced aldosterone Hyperkalemia and hypermagnesemia usually
secretion. BUN increases as the glomerular are associated with reduced renal excretion of
filtration rate is reduced. Hyponatremia is potassium and magnesium, not thyroid
caused by reduced aldosterone secretion. surgery.
Reduced cortisol secretion leads to impaired 77. Answer: (D) Carcinoembryonic antigen level
glyconeogenesis and a reduction of glycogen Rationale: In clients who smoke, the level of
in the liver and muscle, causing hypoglycemia. carcinoembryonic antigen is elevated.
72. Answer: (C) Restricting fluids Therefore, it can't be used as a general
Rationale: To reduce water retention in a indicator of cancer. However, it is helpful in
client with the SIADH, the nurse should monitoring cancer treatment because the
restrict fluids. Administering fluids by any level usually falls to normal within 1 month if
route would further increase the client's treatment is successful. An elevated acid
already heightened fluid load. phosphatase level may indicate prostate
73. Answer: (D) glycosylated hemoglobin level. cancer. An elevated alkaline phosphatase level
Rationale: Because some of the glucose in the may reflect bone metastasis. An elevated
bloodstream attaches to some of the serum calcitonin level usually signals thyroid
hemoglobin and stays attached during the cancer.
120-day life span of red blood cells, 78. Answer: (B) Dyspnea, tachycardia, and pallor
glycosylated hemoglobin levels provide Rationale: Signs of iron-deficiency anemia
information about blood glucose levels during include dyspnea, tachycardia, and pallor as
the previous 3 months. Fasting blood glucose well as fatigue, listlessness, irritability, and
and urine glucose levels only give information headache. Night sweats, weight loss, and
about glucose levels at the point in time when diarrhea may signal acquired
they were obtained. Serum fructosamine immunodeficiency syndrome (AIDS). Nausea,
levels provide information about blood vomiting, and anorexia may be signs of
glucose control over the past 2 to 3 weeks. hepatitis B. Itching, rash, and jaundice may
74. Answer: (C) 4:00 pm result from an allergic or hemolytic reaction.
Rationale: NPH is an intermediate-acting 79. Answer: (D) "I'll need to have a C-section if I
insulin that peaks 8 to 12 hours after become pregnant and have a baby."
administration. Because the nurse Rationale: The human immunodeficiency virus
administered NPH insulin at 7 a.m., the client (HIV) is transmitted from mother to child via
is at greatest risk for hypoglycemia from 3 the transplacental route, but a Cesarean
p.m. to 7 p.m. section delivery isn't necessary when the
75. Answer: (A) Glucocorticoids and androgens mother is HIV-positive. The use of birth
Rationale: The adrenal glands have two control will prevent the conception of a child
divisions, the cortex and medulla. The cortex who might have HIV. It's true that a mother
who's HIV positive can give birth to a baby Rationale: The lymphocyte provides adaptive
who's HIV negative. immunity — recognition of a foreign antigen
80. Answer: (C) "Avoid sharing such articles as and formation of memory cells against the
toothbrushes and razors." antigen. Adaptive immunity is mediated by B
Rationale: The human immunodeficiency virus and T lymphocytes and can be acquired
(HIV), which causes AIDS, is most actively or passively. The neutrophil is crucial
concentrated in the blood. For this reason, the to phagocytosis. The basophil plays an
client shouldn't share personal articles that important role in the release of inflammatory
may be blood-contaminated, such as mediators. The monocyte functions in
toothbrushes and razors, with other family phagocytosis and monokine production.
members. HIV isn't transmitted by bathing or 85. Answer: (A) moisture replacement.
by eating from plates, utensils, or serving Rationale: Sjogren's syndrome is an
dishes used by a person with AIDS. autoimmune disorder leading to progressive
81. Answer: (B) Pallor, tachycardia, and a sore loss of lubrication of the skin, GI tract, ears,
tongue nose, and vagina. Moisture replacement is the
Rationale: Pallor, tachycardia, and a sore mainstay of therapy. Though malnutrition and
tongue are all characteristic findings in electrolyte imbalance may occur as a result of
pernicious anemia. Other clinical Sjogren's syndrome's effect on the GI tract, it
manifestations include anorexia; weight loss; a isn't the predominant problem. Arrhythmias
smooth, beefy red tongue; a wide pulse aren't a problem associated with Sjogren's
pressure; palpitations; angina; weakness; syndrome.
fatigue; and paresthesia of the hands and feet. 86. Answer: (C) stool for Clostridium difficile test.
Bradycardia, reduced pulse pressure, weight Rationale: Immunosuppressed clients — for
gain, and double vision aren't characteristic example, clients receiving chemotherapy, —
findings in pernicious anemia. are at risk for infection with C. difficile, which
82. Answer: (B) Administer epinephrine, as causes "horse barn" smelling diarrhea.
prescribed, and prepare to intubate the client Successful treatment begins with an accurate
if necessary. diagnosis, which includes a stool test. The
Rationale: To reverse anaphylactic shock, the ELISA test is diagnostic for human
nurse first should administer epinephrine, a immunodeficiency virus (HIV) and isn't
potent bronchodilator as prescribed. The indicated in this case. An electrolyte panel and
physician is likely to order additional hemogram may be useful in the overall
medications, such as antihistamines and evaluation of a client but aren't diagnostic for
corticosteroids; if these medications don't specific causes of diarrhea. A flat plate of the
relieve the respiratory compromise associated abdomen may provide useful information
with anaphylaxis, the nurse should prepare to about bowel function but isn't indicated in the
intubate the client. No antidote for penicillin case of "horse barn" smelling diarrhea.
exists; however, the nurse should continue to 87. Answer: (D) Western blot test with ELISA.
monitor the client's vital signs. A client who Rationale: HIV infection is detected by
remains hypotensive may need fluid analyzing blood for antibodies to HIV, which
resuscitation and fluid intake and output form approximately 2 to 12 weeks after
monitoring; however, administering exposure to HIV and denote infection. The
epinephrine is the first priority. Western blot test — electrophoresis of
83. Answer: (D) bilateral hearing loss. antibody proteins — is more than 98%
Rationale: Prolonged use of aspirin and other accurate in detecting HIV antibodies when
salicylates sometimes causes bilateral hearing used in conjunction with the ELISA. It isn't
loss of 30 to 40 decibels. Usually, this adverse specific when used alone. E-rosette
effect resolves within 2 weeks after the immunofluorescence is used to detect viruses
therapy is discontinued. Aspirin doesn't lead in general; it doesn't confirm HIV infection.
to weight gain or fine motor tremors. Large or Quantification of T-lymphocytes is a useful
toxic salicylate doses may cause respiratory monitoring test but isn't diagnostic for HIV.
alkalosis, not respiratory acidosis. The ELISA test detects HIV antibody particles
84. Answer: (D) Lymphocyte but may yield inaccurate results; a positive

30
30
ELISA result must be confirmed by the temperature in warmer or heating pad; don’t
Western blot test. use microwave oven.
88. Answer: (C) Abnormally low hematocrit (HCT) 96. Answer: (C) The client holds the cane with his
and hemoglobin (Hb) levels left hand, moves the cane forward followed
Rationale: Low preoperative HCT and Hb by the right leg, and then moves the left leg.
levels indicate the client may require a blood Rationale: The cane acts as a support and aids
transfusion before surgery. If the HCT and Hb in weight bearing for the weaker right leg.
levels decrease during surgery because of 97. Answer: (A) Ask the woman’s family to
blood loss, the potential need for a provide personal items such as photos or
transfusion increases. Possible renal failure is mementos.
indicated by elevated BUN or creatinine levels. Rationale: Photos and mementos provide
Urine constituents aren't found in the blood. visual stimulation to reduce sensory
Coagulation is determined by the presence of deprivation.
appropriate clotting factors, not electrolytes. 98. Answer: (B) The client lifts the walker, moves
89. Answer: (A) Platelet count, prothrombin time, it forward 10 inches, and then takes several
and partial thromboplastin time small steps forward.
Rationale: The diagnosis of DIC is based on the Rationale: A walker needs to be picked up,
results of laboratory studies of prothrombin placed down on all legs.
time, platelet count, thrombin time, partial 99. Answer: (C) Isolation from their families and
thromboplastin time, and fibrinogen level as familiar surroundings.
well as client history and other assessment Rationale: Gradual loss of sight, hearing, and
factors. Blood glucose levels, WBC count, taste interferes with normal functioning.
calcium levels, and potassium levels aren't 100. Answer: (A) Encourage the client to perform
used to confirm a diagnosis of DIC. pursed lip breathing.
90. Answer: (D) Strawberries Rationale: Purse lip breathing prevents the
Rationale: Common food allergens include collapse of lung unit and helps client control
berries, peanuts, Brazil nuts, cashews, rate and depth of breathing.
shellfish, and eggs. Bread, carrots, and
oranges rarely cause allergic reactions.
91. Answer: (B) A client with cast on the right leg
who states, “I have a funny feeling in my right
leg.”
Rationale: It may indicate neurovascular
compromise, requires immediate assessment.
92. Answer: (D) A 62-year-old who had an
abdominal-perineal resection three days ago;
client complaints of chills.
Rationale: The client is at risk for peritonitis;
should be assessed for further symptoms and
infection.
93. Answer: (C) The client spontaneously flexes
his wrist when the blood pressure is obtained.
Rationale: Carpal spasms indicate
hypocalcemia.
94. Answer: (D) Use comfort measures and
pillows to position the client.
Rationale: Using comfort measures and
pillows to position the client is a non-
pharmacological methods of pain relief.
95. Answer: (B) Warm the dialysate solution.
Rationale: Cold dialysate increases discomfort.
The solution should be warmed to body
TEST IV - Care of Clients with Physiologic and b. Decrease fluid intake at meal times.
Psychosocial Alterations c. Avoid foods that in the past caused
flatus.
1. Randy has undergone kidney transplant, what d. Adhere to a bland diet prior to social
assessment would prompt Nurse Katrina to events.
suspect organ rejection?
a. Sudden weight loss 7. Nurse Ron begins to teach a male client how to
b. Polyuria perform colostomy irrigations. The nurse would
c. Hypertension evaluate that the instructions were understood
d. Shock when the client states, “I should:
a. Lie on my left side while instilling the
2. The immediate objective of nursing care for an irrigating solution.”
overweight, mildly hypertensive male client with b. Keep the irrigating container less than
ureteral colic and hematuria is to decrease: 18 inches above the stoma.”
a. Pain c. Instill a minimum of 1200 ml of irrigating
b. Weight solution to stimulate evacuation of the
c. Hematuria bowel.”
d. Hypertension d. Insert the irrigating catheter deeper into
the stoma if cramping occurs during the
3. Matilda, with hyperthyroidism is to receive procedure.”
Lugol’s iodine solution before a subtotal
thyroidectomy is performed. The nurse is aware 8. Patrick is in the oliguric phase of acute tubular
that this medication is given to: necrosis and is experiencing fluid and electrolyte
a. Decrease the total basal metabolic rate. imbalances. The client is somewhat confused
b. Maintain the function of the parathyroid and complains of nausea and muscle weakness.
glands. As part of the prescribed therapy to correct this
c. Block the formation of thyroxine by the electrolyte imbalance, the nurse would expect
thyroid gland. to:
d. Decrease the size and vascularity of the a. Administer Kayexalate
thyroid gland. b. Restrict foods high in protein
c. Increase oral intake of cheese and milk.
4. Ricardo, was diagnosed with type I diabetes. The d. Administer large amounts of normal
nurse is aware that acute hypoglycemia also can saline via I.V.
develop in the client who is diagnosed with:
a. Liver disease 9. Mario has burn injury. After Forty48 hours, the
b. Hypertension physician orders for Mario 2 liters of IV fluid to
c. Type 2 diabetes be administered q12 h. The drop factor of the
d. Hyperthyroidism tubing is 10 gtt/ml. The nurse should set the
flow to provide:
5. Tracy is receiving combination chemotherapy for a. 18 gtt/min
treatment of metastatic carcinoma. Nurse Ruby b. 28 gtt/min
should monitor the client for the systemic side c. 32 gtt/min
effect of: d. 36 gtt/min
a. Ascites
b. Nystagmus 10. Terence suffered from burn injury. Using the rule
c. Leukopenia of nines, which has the largest percent of burns?
d. Polycythemia a. Face and neck
b. Right upper arm and penis
6. Norma, with recent colostomy expresses c. Right thigh and penis
concern about the inability to control the d. Upper trunk
passage of gas. Nurse Oliver should suggest that
the client plan to: 11. Herbert, a 45 year old construction engineer is
a. Eliminate foods high in cellulose. brought to the hospital unconscious after falling

31
31
from a 2-story building. When assessing the d. Only ice chips and cold liquids will be
client, the nurse would be most concerned if the allowed initially.
assessment revealed:
a. Reactive pupils 16. Nurse Tristan is caring for a male client in acute
b. A depressed fontanel renal failure. The nurse should expect hypertonic
c. Bleeding from ears glucose, insulin infusions, and sodium
d. An elevated temperature bicarbonate to be used to treat:
a. hypernatremia.
12. Nurse Sherry is teaching male client regarding b. hypokalemia.
his permanent artificial pacemaker. Which c. hyperkalemia.
information given by the nurse shows her d. hypercalcemia.
knowledge deficit about the artificial cardiac
pacemaker? 17. Ms. X has just been diagnosed with condylomata
a. take the pulse rate once a day, in the acuminata (genital warts). What information is
morning upon awakening appropriate to tell this client?
b. May be allowed to use electrical a. This condition puts her at a higher risk
appliances for cervical cancer; therefore, she should
c. Have regular follow up care have a Papanicolaou (Pap) smear
d. May engage in contact sports annually.
b. The most common treatment is
13. The nurse is ware that the most relevant metronidazole (Flagyl), which should
knowledge about oxygen administration to a eradicate the problem within 7 to 10
male client with COPD is days.
a. Oxygen at 1-2L/min is given to maintain c. The potential for transmission to her
the hypoxic stimulus for breathing. sexual partner will be eliminated if
b. Hypoxia stimulates the central condoms are used every time they have
chemoreceptors in the medulla that sexual intercourse.
makes the client breath. d. The human papillomavirus (HPV), which
c. Oxygen is administered best using a non- causes condylomata acuminata, can't be
rebreathing mask transmitted during oral sex.
d. Blood gases are monitored using a pulse
oximeter. 18. Maritess was recently diagnosed with a
genitourinary problem and is being examined in
14. Tonny has undergoes a left thoracotomy and a the emergency department. When palpating her
partial pneumonectomy. Chest tubes are kidneys, the nurse should keep which anatomical
inserted, and one-bottle water-seal drainage is fact in mind?
instituted in the operating room. In the a. The left kidney usually is slightly higher
postanesthesia care unit Tonny is placed in than the right one.
Fowler's position on either his right side or on b. The kidneys are situated just above the
his back. The nurse is aware that this position: adrenal glands.
a. Reduce incisional pain. c. The average kidney is approximately 5
b. Facilitate ventilation of the left lung. cm (2") long and 2 to 3 cm (¾" to 1-1/8")
c. Equalize pressure in the pleural space. wide.
d. Increase venous return d. The kidneys lie between the 10th and
12th thoracic vertebrae.
15. Kristine is scheduled for a bronchoscopy. When
teaching Kristine what to expect afterward, the 19. Jestoni with chronic renal failure (CRF) is
nurse's highest priority of information would be: admitted to the urology unit. The nurse is aware
a. Food and fluids will be withheld for at that the diagnostic test are consistent with CRF if
least 2 hours. the result is:
b. Warm saline gargles will be done q 2h. a. Increased pH with decreased hydrogen
c. Coughing and deep-breathing exercises ions.
will be done q2h.
b. Increased serum levels of potassium, b. Palpate the abdomen.
magnesium, and calcium. c. Change the client's position.
c. Blood urea nitrogen (BUN) 100 mg/dl d. Insert a rectal tube.
and serum creatinine 6.5 mg/ dl.
d. Uric acid analysis 3.5 mg/dl and 24. Wilfredo with a recent history of rectal bleeding
phenolsulfonphthalein (PSP) excretion is being prepared for a colonoscopy. How should
75%. the nurse Patricia position the client for this test
initially?
20. Katrina has an abnormal result on a a. Lying on the right side with legs straight
Papanicolaou test. After admitting that she read b. Lying on the left side with knees bent
her chart while the nurse was out of the room, c. Prone with the torso elevated
Katrina asks what dysplasia means. Which d. Bent over with hands touching the floor
definition should the nurse provide?
a. Presence of completely undifferentiated 25. A male client with inflammatory bowel disease
tumor cells that don't resemble cells of undergoes an ileostomy. On the first day after
the tissues of their origin. surgery, Nurse Oliver notes that the client's
b. Increase in the number of normal cells in stoma appears dusky. How should the nurse
a normal arrangement in a tissue or an interpret this finding?
organ. a. Blood supply to the stoma has been
c. Replacement of one type of fully interrupted.
differentiated cell by another in tissues b. This is a normal finding 1 day after
where the second type normally isn't surgery.
found. c. The ostomy bag should be adjusted.
d. Alteration in the size, shape, and d. An intestinal obstruction has occurred.
organization of differentiated cells.
26. Anthony suffers burns on the legs, which nursing
21. During a routine checkup, Nurse Mariane intervention helps prevent contractures?
assesses a male client with acquired a. Applying knee splints
immunodeficiency syndrome (AIDS) for signs and b. Elevating the foot of the bed
symptoms of cancer. What is the most common c. Hyperextending the client's palms
AIDS-related cancer? d. Performing shoulder range-of-motion
a. Squamous cell carcinoma exercises
b. Multiple myeloma
c. Leukemia 27. Nurse Ron is assessing a client admitted with
d. Kaposi's sarcoma second- and third-degree burns on the face,
arms, and chest. Which finding indicates a
22. Ricardo is scheduled for a prostatectomy, and potential problem?
the anesthesiologist plans to use a spinal a. Partial pressure of arterial oxygen
(subarachnoid) block during surgery. In the (PaO2) value of 80 mm Hg.
operating room, the nurse positions the client b. Urine output of 20 ml/hour.
according to the anesthesiologist's instructions. c. White pulmonary secretions.
Why does the client require special positioning d. Rectal temperature of 100.6° F (38° C).
for this type of anesthesia?
a. To prevent confusion 28. Mr. Mendoza who has suffered a
b. To prevent seizures cerebrovascular accident (CVA) is too weak to
c. To prevent cerebrospinal fluid (CSF) move on his own. To help the client avoid
leakage pressure ulcers, Nurse Celia should:
d. To prevent cardiac arrhythmias a. Turn him frequently.
b. Perform passive range-of-motion (ROM)
23. A male client had a nephrectomy 2 days ago and exercises.
is now complaining of abdominal pressure and c. Reduce the client's fluid intake.
nausea. The first nursing action should be to: d. Encourage the client to use a footboard.
a. Auscultate bowel sounds.

31
31
29. Nurse Maria plans to administer dexamethasone 34. A 37-year-old male client was admitted to the
cream to a female client who has dermatitis over coronary care unit (CCU) 2 days ago with an
the anterior chest. How should the nurse apply acute myocardial infarction. Which of the
this topical agent? following actions would breach the client
a. With a circular motion, to enhance confidentiality?
absorption. a. The CCU nurse gives a verbal report to
b. With an upward motion, to increase the nurse on the telemetry unit before
blood supply to the affected area transferring the client to that unit
c. In long, even, outward, and downward b. The CCU nurse notifies the on-call
strokes in the direction of hair growth physician about a change in the client’s
d. In long, even, outward, and upward condition
strokes in the direction opposite hair c. The emergency department nurse calls
growth up the latest electrocardiogram results
to check the client’s progress.
30. Nurse Kate is aware that one of the following d. At the client’s request, the CCU nurse
classes of medication protects the ischemic updates the client’s wife on his condition
myocardium by blocking catecholamines and
sympathetic nerve stimulation is: 35. A male client arriving in the emergency
a. Beta -adrenergic blockers department is receiving cardiopulmonary
b. Calcium channel blocker resuscitation from paramedics who are giving
c. Narcotics ventilations through an endotracheal (ET) tube
d. Nitrates that they placed in the client’s home. During a
pause in compressions, the cardiac monitor
31. A male client has jugular distention. On what shows narrow QRS complexes and a heart rate
position should the nurse place the head of the of beats/minute with a palpable pulse. Which of
bed to obtain the most accurate reading of the following actions should the nurse take first?
jugular vein distention? a. Start an L.V. line and administer
a. High Fowler’s amiodarone (Cardarone), 300 mg L.V.
b. Raised 10 degrees over 10 minutes.
c. Raised 30 degrees b. Check endotracheal tube placement.
d. Supine position c. Obtain an arterial blood gas (ABG)
sample.
32. The nurse is aware that one of the following d. Administer atropine, 1 mg L.V.
classes of medications maximizes cardiac
performance in clients with heart failure by 36. After cardiac surgery, a client’s blood pressure
increasing ventricular contractility? measures 126/80 mm Hg. Nurse Katrina
a. Beta-adrenergic blockers determines that mean arterial pressure (MAP) is
b. Calcium channel blocker which of the following?
c. Diuretics a. 46 mm Hg
d. Inotropic agents b. 80 mm Hg
c. 95 mm Hg
33. A male client has a reduced serum high-density d. 90 mm Hg
lipoprotein (HDL) level and an elevated low-
density lipoprotein (LDL) level. Which of the 37. A female client arrives at the emergency
following dietary modifications is not department with chest and stomach pain and a
appropriate for this client? report of black tarry stool for several months.
a. Fiber intake of 25 to 30 g daily Which of the following order should the nurse
b. Less than 30% of calories from fat Oliver anticipate?
c. Cholesterol intake of less than 300 mg a. Cardiac monitor, oxygen, creatine kinase
daily and lactate dehydrogenase levels
d. Less than 10% of calories from saturated b. Prothrombin time, partial
fat thromboplastin time, fibrinogen and
fibrin split product values.
c. Electrocardiogram, complete blood 43. The nurse is aware that the following symptom
count, testing for occult blood, is most commonly an early indication of stage 1
comprehensive serum metabolic panel. Hodgkin’s disease?
d. Electroencephalogram, alkaline a. Pericarditis
phosphatase and aspartate b. Night sweat
aminotransferase levels, basic serum c. Splenomegaly
metabolic panel d. Persistent hypothermia

38. Macario had coronary artery bypass graft (CABG) 44. Francis with leukemia has neutropenia. Which of
surgery 3 days ago. Which of the following the following functions must frequently
conditions is suspected by the nurse when a assessed?
decrease in platelet count from 230,000 ul to a. Blood pressure
5,000 ul is noted? b. Bowel sounds
a. Pancytopenia c. Heart sounds
b. Idiopathic thrombocytopemic purpura d. Breath sounds
(ITP)
c. Disseminated intravascular coagulation 45. The nurse knows that neurologic complications
(DIC) of multiple myeloma (MM) usually involve which
d. Heparin-associated thrombosis and of the following body system?
thrombocytopenia (HATT) a. Brain
b. Muscle spasm
39. Which of the following drugs would be ordered c. Renal dysfunction
by the physician to improve the platelet count in d. Myocardial irritability
a male client with idiopathic thrombocytopenic
purpura (ITP)? 46. Nurse Patricia is aware that the average length
a. Acetylsalicylic acid (ASA) of time from human immunodeficiency virus
b. Corticosteroids (HIV) infection to the development of acquired
c. Methotrezate immunodeficiency syndrome (AIDS)?
d. Vitamin K a. Less than 5 years
b. 5 to 7 years
40. A female client is scheduled to receive a heart c. 10 years
valve replacement with a porcine valve. Which d. More than 10 years
of the following types of transplant is this?
a. Allogeneic 47. An 18-year-old male client admitted with heat
b. Autologous stroke begins to show signs of disseminated
c. Syngeneic intravascular coagulation (DIC). Which of the
d. Xenogeneic following laboratory findings is most consistent
with DIC?
41. Marco falls off his bicycle and injuries his ankle. a. Low platelet count
Which of the following actions shows the initial b. Elevated fibrinogen levels
response to the injury in the extrinsic pathway? c. Low levels of fibrin degradation products
a. Release of Calcium d. Reduced prothrombin time
b. Release of tissue thromboplastin
c. Conversion of factors XII to factor XIIa 48. Mario comes to the clinic complaining of fever,
d. Conversion of factor VIII to factor VIIIa drenching night sweats, and unexplained weight
42. Instructions for a client with systemic lupus loss over the past 3 months. Physical
erythematosus (SLE) would include information examination reveals a single enlarged
about which of the following blood dyscrasias? supraclavicular lymph node. Which of the
a. Dressler’s syndrome following is the most probable diagnosis?
b. Polycythemia a. Influenza
c. Essential thrombocytopenia b. Sickle cell anemia
d. Von Willebrand’s disease c. Leukemia
d. Hodgkin’s disease

31
31

red and swollen, when the IV is touched Stacy


49. A male client with a gunshot wound requires an shouts in pain. The first nursing action to take is:
emergency blood transfusion. His blood type is a. Notify the physician
AB negative. Which blood type would be the b. Flush the IV line with saline solution
safest for him to receive? c. Immediately discontinue the infusion
a. AB Rh-positive d. Apply an ice pack to the site, followed by
b. A Rh-positive warm compress.
c. A Rh-negative
d. O Rh-positive 54. The term “blue bloater” refers to a male client
which of the following conditions?
Situation: Stacy is diagnosed with acute lymphoid a. Adult respiratory distress syndrome
leukemia (ALL) and beginning chemotherapy. (ARDS)
b. Asthma
50. Stacy is discharged from the hospital following c. Chronic obstructive bronchitis
her chemotherapy treatments. Which statement d. Emphysema
of Stacy’s mother indicated that she understands
when she will contact the physician? 55. The term “pink puffer” refers to the female
a. “I should contact the physician if Stacy client with which of the following conditions?
has difficulty in sleeping”. a. Adult respiratory distress syndrome
b. “I will call my doctor if Stacy has (ARDS)
persistent vomiting and diarrhea”. b. Asthma
c. “My physician should be called if Stacy is c. Chronic obstructive bronchitis
irritable and unhappy”. d. Emphysema
d. “Should Stacy have continued hair loss, I
need to call the doctor”. 56. Jose is in danger of respiratory arrest following
the administration of a narcotic analgesic. An
51. Stacy’s mother states to the nurse that it is hard arterial blood gas value is obtained. Nurse Oliver
to see Stacy with no hair. The best response for would expect the paco2 to be which of the
the nurse is: following values?
a. “Stacy looks very nice wearing a hat”. a. 15 mm Hg
b. “You should not worry about her hair, b. 30 mm Hg
just be glad that she is alive”. c. 40 mm Hg
c. “Yes it is upsetting. But try to cover up d. 80 mm Hg
your feelings when you are with her or
else she may be upset”. 57. Timothy’s arterial blood gas (ABG) results are as
d. “This is only temporary; Stacy will re- follows; pH 7.16; Paco2 80 mm Hg; Pao2 46 mm
grow new hair in 3-6 months, but may Hg; HCO3- 24mEq/L; Sao2 81%. This ABG result
be different in texture”. represents which of the following conditions?
a. Metabolic acidosis
52. Stacy has beginning stomatitis. To promote oral b. Metabolic alkalosis
hygiene and comfort, the nurse in-charge c. Respiratory acidosis
should: d. Respiratory alkalosis
a. Provide frequent mouthwash with
normal saline. 58. Norma has started a new drug for hypertension.
b. Apply viscous Lidocaine to oral ulcers as Thirty minutes after she takes the drug, she
needed. develops chest tightness and becomes short of
c. Use lemon glycerine swabs every 2 breath and tachypneic. She has a decreased level
hours. of consciousness. These signs indicate which of
d. Rinse mouth with Hydrogen Peroxide. the following conditions?
a. Asthma attack
53. During the administration of chemotherapy b. Pulmonary embolism
agents, Nurse Oliver observed that the IV site is c. Respiratory failure
d. Rheumatoid arthritis
decreased RBC count, decreased WBC
Situation: Mr. Gonzales was admitted to the hospital count.
with ascites and jaundice. To rule out cirrhosis of the d. Intermitted lower back pain, decreased
liver: blood pressure, decreased RBC count,
increased WBC count.
59. Which laboratory test indicates liver cirrhosis?
a. Decreased red blood cell count 64. After undergoing a cardiac catheterization, Tracy
b. Decreased serum acid phosphate level has a large puddle of blood under his buttocks.
c. Elevated white blood cell count Which of the following steps should the nurse
d. Elevated serum aminotransferase take first?
a. Call for help.
60. 60.The biopsy of Mr. Gonzales confirms the b. Obtain vital signs
diagnosis of cirrhosis. Mr. Gonzales is at c. Ask the client to “lift up”
increased risk for excessive bleeding primarily d. Apply gloves and assess the groin site
because of:
a. Impaired clotting mechanism 65. Which of the following treatment is a suitable
b. Varix formation surgical intervention for a client with unstable
c. Inadequate nutrition angina?
d. Trauma of invasive procedure a. Cardiac catheterization
b. Echocardiogram
61. Mr. Gonzales develops hepatic encephalopathy. c. Nitroglycerin
Which clinical manifestation is most common d. Percutaneous transluminal coronary
with this condition? angioplasty (PTCA)
a. Increased urine output
b. Altered level of consciousness 66. The nurse is aware that the following terms used
c. Decreased tendon reflex to describe reduced cardiac output and
d. Hypotension perfusion impairment due to ineffective
pumping of the heart is:
62. When Mr. Gonzales regained consciousness, the a. Anaphylactic shock
physician orders 50 ml of Lactose p.o. every 2 b. Cardiogenic shock
hours. Mr. Gozales develops diarrhea. The nurse c. Distributive shock
best action would be: d. Myocardial infarction (MI)
a. “I’ll see if your physician is in the
hospital”. 67. A client with hypertension asks the nurse which
b. “Maybe you’re reacting to the drug; I factors can cause blood pressure to drop to
will withhold the next dose”. normal levels?
c. “I’ll lower the dosage as ordered so the a. Kidneys’ excretion to sodium only.
drug causes only 2 to 4 stools a day”. b. Kidneys’ retention of sodium and water
d. “Frequently, bowel movements are c. Kidneys’ excretion of sodium and water
needed to reduce sodium level”. d. Kidneys’ retention of sodium and
excretion of water
63. Which of the following groups of symptoms
indicates a ruptured abdominal aortic 68. Nurse Rose is aware that the statement that
aneurysm? best explains why furosemide (Lasix) is
a. Lower back pain, increased blood administered to treat hypertension is:
pressure, decreased red blood cell (RBC) a. It dilates peripheral blood vessels.
count, increased white blood (WBC) b. It decreases sympathetic
count. cardioacceleration.
b. Severe lower back pain, decreased c. It inhibits the angiotensin-coverting
blood pressure, decreased RBC count, enzymes
increased WBC count. d. It inhibits reabsorption of sodium and
c. Severe lower back pain, decreased water in the loop of Henle.
blood pressure, decreased RBC count,

31
31
69. Nurse Nikki knows that laboratory results 73. JP has been diagnosed with gout and wants to
supports the diagnosis of systemic lupus know why colchicine is used in the treatment of
erythematosus (SLE) is: gout. Which of the following actions of
a. Elavated serum complement level colchicines explains why it’s effective for gout?
b. Thrombocytosis, elevated sedimentation a. Replaces estrogen
rate b. Decreases infection
c. Pancytopenia, elevated antinuclear c. Decreases inflammation
antibody (ANA) titer d. Decreases bone demineralization
d. Leukocysis, elevated blood urea nitrogen
(BUN) and creatinine levels 74. Norma asks for information about osteoarthritis.
Which of the following statements about
70. Arnold, a 19-year-old client with a mild osteoarthritis is correct?
concussion is discharged from the emergency a. Osteoarthritis is rarely debilitating
department. Before discharge, he complains of a b. Osteoarthritis is a rare form of arthritis
headache. When offered acetaminophen, his c. Osteoarthritis is the most common form
mother tells the nurse the headache is severe of arthritis
and she would like her son to have something d. Osteoarthritis afflicts people over 60
stronger. Which of the following responses by
the nurse is appropriate? 75. Ruby is receiving thyroid replacement therapy
a. “Your son had a mild concussion, develops the flu and forgets to take her thyroid
acetaminophen is strong enough.” replacement medicine. The nurse understands
b. “Aspirin is avoided because of the that skipping this medication will put the client
danger of Reye’s syndrome in children or at risk for developing which of the following life-
young adults.” threatening complications?
c. “Narcotics are avoided after a head a. Exophthalmos
injury because they may hide a b. Thyroid storm
worsening condition.” c. Myxedema coma
d. Stronger medications may lead to d. Tibial myxedema
vomiting, which increases the
intracarnial pressure (ICP).” 76. Nurse Sugar is assessing a client with Cushing's
71. When evaluating an arterial blood gas from a syndrome. Which observation should the nurse
male client with a subdural hematoma, the report to the physician immediately?
nurse notes the Paco2 is 30 mm Hg. Which of a. Pitting edema of the legs
the following responses best describes the b. An irregular apical pulse
result? c. Dry mucous membranes
a. Appropriate; lowering carbon dioxide d. Frequent urination
(CO2) reduces intracranial pressure (ICP)
b. Emergent; the client is poorly 77. Cyrill with severe head trauma sustained in a car
oxygenated accident is admitted to the intensive care unit.
c. Normal Thirty-six hours later, the client's urine output
d. Significant; the client has alveolar suddenly rises above 200 ml/hour, leading the
hypoventilation nurse to suspect diabetes insipidus. Which
laboratory findings support the nurse's suspicion
72. When prioritizing care, which of the following of diabetes insipidus?
clients should the nurse Olivia assess first? a. Above-normal urine and serum
a. A 17-year-old client’s 24-hours osmolality levels
postappendectomy b. Below-normal urine and serum
b. A 33-year-old client with a recent osmolality levels
diagnosis of Guillain-Barre syndrome c. Above-normal urine osmolality level,
c. A 50-year-old client 3 days below-normal serum osmolality level
postmyocardial infarction d. Below-normal urine osmolality level,
d. A 50-year-old client with diverticulitis above-normal serum osmolality level
78. Jomari is diagnosed with hyperosmolar d. Low corticotropin and low cortisol levels
hyperglycemic nonketotic syndrome (HHNS) is
stabilized and prepared for discharge. When 82. A male client is scheduled for a transsphenoidal
preparing the client for discharge and home hypophysectomy to remove a pituitary tumor.
management, which of the following statements Preoperatively, the nurse should assess for
indicates that the client understands her potential complications by doing which of the
condition and how to control it? following?
a. "I can avoid getting sick by not becoming a. Testing for ketones in the urine
dehydrated and by paying attention to b. Testing urine specific gravity
my need to urinate, drink, or eat more c. Checking temperature every 4 hours
than usual." d. Performing capillary glucose testing
b. "If I experience trembling, weakness, every 4 hours
and headache, I should drink a glass of
soda that contains sugar." 83. Capillary glucose monitoring is being performed
c. "I will have to monitor my blood glucose every 4 hours for a client diagnosed with
level closely and notify the physician if diabetic ketoacidosis. Insulin is administered
it's constantly elevated." using a scale of regular insulin according to
d. "If I begin to feel especially hungry and glucose results. At 2 p.m., the client has a
thirsty, I'll eat a snack high in capillary glucose level of 250 mg/dl for which he
carbohydrates." receives 8 U of regular insulin. Nurse Mariner
should expect the dose's:
79. A 66-year-old client has been complaining of a. onset to be at 2 p.m. and its peak to
sleeping more, increased urination, anorexia, be at 3 p.m.
weakness, irritability, depression, and bone pain b. onset to be at 2:15 p.m. and its peak to
that interferes with her going outdoors. Based be at 3 p.m.
on these assessment findings, the nurse would c. onset to be at 2:30 p.m. and its peak to
suspect which of the following disorders? be at 4 p.m.
a. Diabetes mellitus d. onset to be at 4 p.m. and its peak to
b. Diabetes insipidus be at 6 p.m.
c. Hypoparathyroidism
d. Hyperparathyroidism 84. The physician orders laboratory tests to confirm
hyperthyroidism in a female client with classic
80. Nurse Lourdes is teaching a client recovering signs and symptoms of this disorder. Which test
from addisonian crisis about the need to take result would confirm the diagnosis?
fludrocortisone acetate and hydrocortisone at a. No increase in the thyroid-stimulating
home. Which statement by the client indicates hormone (TSH) level after 30 minutes
an understanding of the instructions? during the TSH stimulation test
a. "I'll take my hydrocortisone in the late b. A decreased TSH level
afternoon, before dinner." c. An increase in the TSH level after 30
b. "I'll take all of my hydrocortisone in the minutes during the TSH stimulation test
morning, right after I wake up." d. Below-normal levels of serum
c. "I'll take two-thirds of the dose when I triiodothyronine (T3) and serum
wake up and one-third in the late thyroxine (T4) as detected by
afternoon." radioimmunoassay
d. "I'll take the entire dose at bedtime."
85. Rico with diabetes mellitus must learn how to
81. Which of the following laboratory test results self-administer insulin. The physician has
would suggest to the nurse Len that a client has prescribed 10 U of U-100 regular insulin and 35
a corticotropin-secreting pituitary adenoma? U of U-100 isophane insulin suspension (NPH) to
a. High corticotropin and low cortisol levels be taken before breakfast. When teaching the
b. Low corticotropin and high cortisol levels client how to select and rotate insulin injection
c. High corticotropin and high cortisol sites, the nurse should provide which
levels instruction?

31
31
a. "Inject insulin into healthy tissue with a. Adult respiratory distress syndrome
large blood vessels and nerves." (ARDS)
b. "Rotate injection sites within the same b. Atelectasis
anatomic region, not among different c. Bronchitis
regions." d. Pneumonia
c. "Administer insulin into areas of scar
tissue or hypotrophy whenever 91. A 67-year-old client develops acute shortness of
possible." breath and progressive hypoxia requiring right
d. "Administer insulin into sites above femur. The hypoxia was probably caused by
muscles that you plan to exercise heavily which of the following conditions?
later that day." a. Asthma attack
b. Atelectasis
86. Nurse Sarah expects to note an elevated serum c. Bronchitis
glucose level in a client with hyperosmolar d. Fat embolism
hyperglycemic nonketotic syndrome (HHNS).
Which other laboratory finding should the nurse 92. A client with shortness of breath has decreased
anticipate? to absent breath sounds on the right side, from
a. Elevated serum acetone level the apex to the base. Which of the following
b. Serum ketone bodies conditions would best explain this?
c. Serum alkalosis a. Acute asthma
d. Below-normal serum potassium level b. Chronic bronchitis
c. Pneumonia
87. For a client with Graves' disease, which nursing d. Spontaneous pneumothorax
intervention promotes comfort?
a. Restricting intake of oral fluids 93. A 62-year-old male client was in a motor vehicle
b. Placing extra blankets on the client's bed accident as an unrestrained driver. He’s now in
c. Limiting intake of high-carbohydrate the emergency department complaining of
foods difficulty of breathing and chest pain. On
d. Maintaining room temperature in the auscultation of his lung field, no breath sounds
low-normal range are present in the upper lobe. This client may
have which of the following conditions?
88. Patrick is treated in the emergency department a. Bronchitis
for a Colles' fracture sustained during a fall. b. Pneumonia
What is a Colles' fracture? c. Pneumothorax
a. Fracture of the distal radius d. Tuberculosis (TB)
b. Fracture of the olecranon
c. Fracture of the humerus 94. If a client requires a pneumonectomy, what fills
d. Fracture of the carpal scaphoid the area of the thoracic cavity?
a. The space remains filled with air only
89. Cleo is diagnosed with osteoporosis. Which b. The surgeon fills the space with a gel
electrolytes are involved in the development of c. Serous fluids fills the space and
this disorder? consolidates the region
a. Calcium and sodium d. The tissue from the other lung grows
b. Calcium and phosphorous over to the other side
c. Phosphorous and potassium
d. Potassium and sodium 95. Hemoptysis may be present in the client with a
pulmonary embolism because of which of the
90. Johnny a firefighter was involved in following reasons?
extinguishing a house fire and is being treated to a. Alveolar damage in the infracted area
smoke inhalation. He develops severe hypoxia b. Involvement of major blood vessels in
48 hours after the incident, requiring intubation the occluded area
and mechanical ventilation. He most likely has c. Loss of lung parenchyma
developed which of the following conditions? d. Loss of lung tissue
c. “Every four hours I should remove the
96. Aldo with a massive pulmonary embolism will stockings for a half hour.”
have an arterial blood gas analysis performed to d. “I should put on the stockings before
determine the extent of hypoxia. The acid-base getting out of bed in the morning.”
disorder that may be present is?
a. Metabolic acidosis
b. Metabolic alkalosis
c. Respiratory acidosis
d. Respiratory alkalosis

97. After a motor vehicle accident, Armand an 22-


year-old client is admitted with a pneumothorax.
The surgeon inserts a chest tube and attaches it
to a chest drainage system. Bubbling soon
appears in the water seal chamber. Which of the
following is the most likely cause of the
bubbling?
a. Air leak
b. Adequate suction
c. Inadequate suction
d. Kinked chest tube

98. Nurse Michelle calculates the IV flow rate for a


postoperative client. The client receives 3,000 ml
of Ringer’s lactate solution IV to run over 24
hours. The IV infusion set has a drop factor of 10
drops per milliliter. The nurse should regulate
the client’s IV to deliver how many drops per
minute?
a. 18
b. 21
c. 35
d. 40

99. Mickey, a 6-year-old child with a congenital


heart disorder is admitted with congestive heart
failure. Digoxin (lanoxin) 0.12 mg is ordered for
the child. The bottle of Lanoxin contains .05 mg
of Lanoxin in 1 ml of solution. What amount
should the nurse administer to the child?
a. 1.2 ml
b. 2.4 ml
c. 3.5 ml
d. 4.2 ml

100. Nurse Alexandra teaches a client about elastic


stockings. Which of the following statements,
if made by the client, indicates to the nurse
that the teaching was successful?
a. “I will wear the stockings until the
physician tells me to remove them.”
b. “I should wear the stockings even when I
am sleep.”

32
32
Answers and Rationale – Care of Clients with 10. Answer: (D) Upper trunk
Physiologic and Psychosocial Alterations Rationale: The percentage designated for
each burned part of the body using the
1. Answer: (C) Hypertension rule of nines: Head and neck 9%; Right
Rationale: Hypertension, along with fever, upper extremity 9%; Left upper extremity
and tenderness over the grafted kidney, 9%; Anterior trunk 18%; Posterior trunk
reflects acute rejection. 18%; Right lower extremity 18%; Left
2. Answer: (A) Pain lower extremity 18%; Perineum 1%.
Rationale: Sharp, severe pain (renal colic) 11. Answer: (C) Bleeding from ears
radiating toward the genitalia and thigh is Rationale: The nurse needs to perform a
caused by uretheral distention and thorough assessment that could indicate
smooth muscle spasm; relief form pain is alterations in cerebral function, increased
the priority. intracranial pressures, fractures and
3. Answer: (D) Decrease the size and bleeding. Bleeding from the ears occurs
vascularity of the thyroid gland. only with basal skull fractures that can
Rationale: Lugol’s solution provides easily contribute to increased intracranial
iodine, which aids in decreasing the pressure and brain herniation.
vascularity of the thyroid gland, which 12. Answer: (D) may engage in contact sports
limits the risk of hemorrhage when Rationale: The client should be advised by
surgery is performed. the nurse to avoid contact sports. This will
4. Answer: (A) Liver Disease prevent trauma to the area of the
Rationale: The client with liver disease has pacemaker generator.
a decreased ability to metabolize 13. Answer: (A) Oxygen at 1-2L/min is given to
carbohydrates because of a decreased maintain the hypoxic stimulus for
ability to form glycogen (glycogenesis) and breathing.
to form glucose from glycogen. Rationale: COPD causes a chronic CO2
5. Answer: (C) Leukopenia retention that renders the medulla
Rationale: Leukopenia, a reduction in insensitive to the CO2 stimulation for
WBCs, is a systemic effect of breathing. The hypoxic state of the client
chemotherapy as a result of then becomes the stimulus for breathing.
myelosuppression. Giving the client oxygen in low
6. Answer: (C) Avoid foods that in the past concentrations will maintain the client’s
caused flatus. hypoxic drive.
Rationale: Foods that bothered a person 14. Answer: (B) Facilitate ventilation of the
preoperatively will continue to do so after left lung.
a colostomy. Rationale: Since only a partial
7. Answer: (B) Keep the irrigating container pneumonectomy is done, there is a need
less than 18 inches above the stoma.” to promote expansion of this remaining
Rationale: This height permits the solution Left lung by positioning the client on the
to flow slowly with little force so that opposite unoperated side.
excessive peristalsis is not immediately 15. Answer: (A) Food and fluids will be
precipitated. withheld for at least 2 hours.
8. Answer: (A) Administer Kayexalate Rationale: Prior to bronchoscopy, the
Rationale: Kayexalate,a potassium doctors sprays the back of the throat with
exchange resin, permits sodium to be anesthetic to minimize the gag reflex and
exchanged for potassium in the intestine, thus facilitate the insertion of the
reducing the serum potassium level. bronchoscope. Giving the client food and
9. Answer:(B) 28 gtt/min drink after the procedure without
Rationale: This is the correct flow rate; checking on the return of the gag reflex
multiply the amount to be infused (2000 can cause the client to aspirate. The gag
ml) by the drop factor (10) and divide the reflex usually returns after two hours.
result by the amount of time in minutes 16. Answer: (C) hyperkalemia.
(12 hours x 60 minutes)
Rationale: Hyperkalemia is a common increases serum levels of potassium,
complication of acute renal failure. It's magnesium, and phosphorous, and
life-threatening if immediate action isn't decreases serum levels of calcium. A uric
taken to reverse it. The administration of acid analysis of 3.5 mg/dl falls within the
glucose and regular insulin, with sodium normal range of 2.7 to 7.7 mg/dl; PSP
bicarbonate if necessary, can temporarily excretion of 75% also falls with the normal
prevent cardiac arrest by moving range of 60% to 75%.
potassium into the cells and temporarily 20. Answer: (D) Alteration in the size, shape,
reducing serum potassium levels. and organization of differentiated cells
Hypernatremia, hypokalemia, and Rationale: Dysplasia refers to an alteration
hypercalcemia don't usually occur with in the size, shape, and organization of
acute renal failure and aren't treated with differentiated cells. The presence of
glucose, insulin, or sodium bicarbonate. completely undifferentiated tumor cells
17. Answer: (A) This condition puts her at a that don't resemble cells of the tissues of
higher risk for cervical cancer; therefore, their origin is called anaplasia. An increase
she should have a Papanicolaou (Pap) in the number of normal cells in a normal
smear annually. arrangement in a tissue or an organ is
Rationale: Women with condylomata called hyperplasia. Replacement of one
acuminata are at risk for cancer of the type of fully differentiated cell by another
cervix and vulva. Yearly Pap smears are in tissues where the second type normally
very important for early detection. isn't found is called metaplasia.
Because condylomata acuminata is a 21. Answer: (D) Kaposi's sarcoma
virus, there is no permanent cure. Rationale: Kaposi's sarcoma is the most
Because condylomata acuminata can common cancer associated with AIDS.
occur on the vulva, a condom won't Squamous cell carcinoma, multiple
protect sexual partners. HPV can be myeloma, and leukemia may occur in
transmitted to other parts of the body, anyone and aren't associated specifically
such as the mouth, oropharynx, and with AIDS.
larynx. 22. Answer: (C) To prevent cerebrospinal fluid
18. Answer: (A) The left kidney usually is (CSF) leakage
slightly higher than the right one. Rationale: The client receiving a
Rationale: The left kidney usually is subarachnoid block requires special
slightly higher than the right one. An positioning to prevent CSF leakage and
adrenal gland lies atop each kidney. The headache and to ensure proper anesthetic
average kidney measures approximately distribution. Proper positioning doesn't
11 cm (4-3/8") long, 5 to 5.8 cm (2" to help prevent confusion, seizures, or
2¼") wide, and 2.5 cm (1") thick. The cardiac arrhythmias.
kidneys are located retroperitoneally, in 23. Answer: (A) Auscultate bowel sounds.
the posterior aspect of the abdomen, on Rationale: If abdominal distention is
either side of the vertebral column. They accompanied by nausea, the nurse must
lie between the 12th thoracic and 3rd first auscultate bowel sounds. If bowel
lumbar vertebrae. sounds are absent, the nurse should
19. Answer: (C) Blood urea nitrogen (BUN) suspect gastric or small intestine dilation
100 mg/dl and serum creatinine 6.5mg/dl. and these findings must be reported to
Rationale: The normal BUN level ranges 8 the physician. Palpation should be
to 23 mg/dl; the normal serum creatinine avoided postoperatively with abdominal
level ranges from 0.7 to 1.5 mg/dl. The distention. If peristalsis is absent,
test results in option C are abnormally changing positions and inserting a rectal
elevated, reflecting CRF and the kidneys' tube won't relieve the client's discomfort.
decreased ability to remove nonprotein 24. Answer: (B) Lying on the left side with
nitrogen waste from the blood. CRF knees bent
causes decreased pH and increased Rationale: For a colonoscopy, the nurse
hydrogen ions — not vice versa. CRF also initially should position the client on the

32
32
left side with knees bent. Placing the capillaries become occluded, reducing
client on the right side with legs straight, circulation and oxygenation of the tissues
prone with the torso elevated, or bent and resulting in cell death and ulcer
over with hands touching the floor formation. During passive ROM exercises,
wouldn't allow proper visualization of the the nurse moves each joint through its
large intestine. range of movement, which improves joint
25. Answer: (A) Blood supply to the stoma has mobility and circulation to the affected
been interrupted area but doesn't prevent pressure ulcers.
Rationale: An ileostomy stoma forms as Adequate hydration is necessary to
the ileum is brought through the maintain healthy skin and ensure tissue
abdominal wall to the surface skin, repair. A footboard prevents plantar
creating an artificial opening for waste flexion and footdrop by maintaining the
elimination. The stoma should appear foot in a dorsiflexed position.
cherry red, indicating adequate arterial 29. Answer: (C) In long, even, outward, and
perfusion. A dusky stoma suggests downward strokes in the direction of hair
decreased perfusion, which may result growth
from interruption of the stoma's blood Rationale: When applying a topical agent,
supply and may lead to tissue damage or the nurse should begin at the midline and
necrosis. A dusky stoma isn't a normal use long, even, outward, and downward
finding. Adjusting the ostomy bag strokes in the direction of hair growth.
wouldn't affect stoma color, which This application pattern reduces the risk
depends on blood supply to the area. An of follicle irritation and skin inflammation.
intestinal obstruction also wouldn't 30. Answer: (A) Beta -adrenergic blockers
change stoma color. Rationale: Beta-adrenergic blockers work
26. Answer: (A) Applying knee splints by blocking beta receptors in the
Rationale: Applying knee splints prevents myocardium, reducing the response to
leg contractures by holding the joints in a catecholamines and sympathetic nerve
position of function. Elevating the foot of stimulation. They protect the
the bed can't prevent contractures myocardium, helping to reduce the risk of
because this action doesn't hold the joints another infraction by decreasing
in a position of function. Hyperextending a myocardial oxygen demand. Calcium
body part for an extended time is channel blockers reduce the workload of
inappropriate because it can cause the heart by decreasing the heart rate.
contractures. Performing shoulder range- Narcotics reduce myocardial oxygen
of-motion exercises can prevent demand, promote vasodilation, and
contractures in the shoulders, but not in decrease anxiety. Nitrates reduce
the legs. myocardial oxygen consumption bt
27. Answer: (B) Urine output of 20 ml/hour. decreasing left ventricular end diastolic
Rationale: A urine output of less than 40 pressure (preload) and systemic vascular
ml/hour in a client with burns indicates a resistance (afterload).
fluid volume deficit. This client's PaO2 31. Answer: (C) Raised 30 degrees
value falls within the normal range (80 to Rationale: Jugular venous pressure is
100 mm Hg). White pulmonary secretions measured with a centimeter ruler to
also are normal. The client's rectal obtain the vertical distance between the
temperature isn't significantly elevated sternal angle and the point of highest
and probably results from the fluid pulsation with the head of the bed
volume deficit. inclined between 15 to 30 degrees.
28. Answer: (A) Turn him frequently. Increased pressure can’t be seen when
Rationale: The most important the client is supine or when the head of
intervention to prevent pressure ulcers is the bed is raised 10 degrees because the
frequent position changes, which relieve point that marks the pressure level is
pressure on the skin and underlying above the jaw (therefore, not visible). In
tissues. If pressure isn't relieved,
high Fowler’s position, the veins would be ventricular fibrillation and atrial flutter –
barely discernible above the clavicle. not symptomatic bradycardia.
32. Answer: (D) Inotropic agents 36. Answer: (C) 95 mm Hg
Rationale: Inotropic agents are Rationale: Use the following formula to
administered to increase the force of the calculate MAP
heart’s contractions, thereby increasing MAP = systolic + 2 (diastolic)
ventricular contractility and ultimately 3
increasing cardiac output. Beta-adrenergic MAP=126 mm Hg + 2 (80 mm Hg)
blockers and calcium channel blockers 3
decrease the heart rate and ultimately MAP=286 mm HG
decreased the workload of the heart. 3
Diuretics are administered to decrease the MAP=95 mm Hg
overall vascular volume, also decreasing 37. Answer: (C) Electrocardiogram, complete
the workload of the heart. blood count, testing for occult blood,
33. Answer: (B) Less than 30% of calories from comprehensive serum metabolic panel.
fat Rationale: An electrocardiogram evaluates
Rationale: A client with low serum HDL the complaints of chest pain, laboratory
and high serum LDL levels should get less tests determines anemia, and the stool
than 30% of daily calories from fat. The test for occult blood determines blood in
other modifications are appropriate for the stool. Cardiac monitoring, oxygen, and
this client. creatine kinase and lactate
34. Answer: (C) The emergency department dehydrogenase levels are appropriate for
nurse calls up the latest electrocardiogram a cardiac primary problem. A basic
results to check the client’s progress metabolic panel and alkaline phosphatase
Rationale: The emergency department and aspartate aminotransferase levels
nurse is no longer directly involved with assess liver function. Prothrombin time,
the client’s care and thus has no legal partial thromboplastin time, fibrinogen
right to information about his present and fibrin split products are measured to
condition. Anyone directly involved in his verify bleeding dyscrasias; an
care (such as the telemetry nurse and the electroencephalogram evaluates brain
on-call physician) has the right to electrical activity.
information about his condition. Because 38. Answer: (D) Heparin-associated
the client requested that the nurse update thrombosis and thrombocytopenia (HATT)
his wife on his condition, doing so doesn’t Rationale: HATT may occur after CABG
breach confidentiality. surgery due to heparin use during surgery.
35. Answer: (B) Check endotracheal tube Although DIC and ITP cause platelet
placement. aggregation and bleeding, neither is
Rationale: ET tube placement should be common in a client after revascularization
confirmed as soon as the client arrives in surgery. Pancytopenia is a reduction in all
the emergency department. Once the blood cells.
airways is secured, oxygenation and 39. Answer: (B) Corticosteroids
ventilation should be confirmed using an Rationale: Corticosteroid therapy can
end-tidal carbon dioxide monitor and decrease antibody production and
pulse oximetry. Next, the nurse should phagocytosis of the antibody-coated
make sure L.V. access is established. If the platelets, retaining more functioning
client experiences symptomatic platelets. Methotrexate can cause
bradycardia, atropine is administered as thrombocytopenia. Vitamin K is used to
ordered 0.5 to 1 mg every 3 to 5 minutes treat an excessive anticoagulate state
to a total of 3 mg. Then the nurse should from warfarin overload, and ASA
try to find the cause of the client’s arrest decreases platelet aggregation.
by obtaining an ABG sample. Amiodarone 40. Answer: (D) Xenogeneic
is indicated for ventricular tachycardia, Rationale: An xenogeneic transplant is
between is between human and another

32
32
species. A syngeneic transplant is between options, which reflect parts of the nervous
identical twins, allogeneic transplant is system, aren’t usually affected by MM.
between two humans, and autologous is a 46. Answer: (C) 10 years
transplant from the same individual. Rationale: Epidermiologic studies show
41. Answer: (B) the average time from initial contact with
Rationale: Tissue thromboplastin is HIV to the development of AIDS is 10
released when damaged tissue comes in years.
contact with clotting factors. Calcium is 47. Answer: (A) Low platelet count
released to assist the conversion of Rationale: In DIC, platelets and clotting
factors X to Xa. Conversion of factors XII to factors are consumed, resulting in
XIIa and VIII to IIIa are part of the intrinsic microthrombi and excessive bleeding. As
pathway. clots form, fibrinogen levels decrease and
42. Answer: (C) Essential thrombocytopenia the prothrombin time increases. Fibrin
Rationale: Essential thrombocytopenia is degeneration products increase as
linked to immunologic disorders, such as fibrinolysis takes places.
SLE and human immunodeficiency virus. 48. Answer: (D) Hodgkin’s disease
The disorder known as von Willebrand’s Rationale: Hodgkin’s disease typically
disease is a type of hemophilia and isn’t causes fever night sweats, weight loss,
linked to SLE. Moderate to severe anemia and lymph mode enlargement. Influenza
is associated with SLE, not polycythemia. doesn’t last for months. Clients with sickle
Dressler’s syndrome is pericarditis that cell anemia manifest signs and symptoms
occurs after a myocardial infarction and of chronic anemia with pallor of the
isn’t linked to SLE. mucous membrane, fatigue, and
43. Answer: (B) Night sweat decreased tolerance for exercise; they
Rationale: In stage 1, symptoms include a don’t show fever, night sweats, weight
single enlarged lymph node (usually), loss or lymph node enlargement.
unexplained fever, night sweats, malaise, Leukemia doesn’t cause lymph node
and generalized pruritis. Although enlargement.
splenomegaly may be present in some 49. Answer: (C) A Rh-negative
clients, night sweats are generally more Rationale: Human blood can sometimes
prevalent. Pericarditis isn’t associated contain an inherited D antigen. Persons
with Hodgkin’s disease, nor is with the D antigen have Rh-positive blood
hypothermia. Moreover, splenomegaly type; those lacking the antigen have Rh-
and pericarditis aren’t symptoms. negative blood. It’s important that a
Persistent hypothermia is associated with person with Rh- negative blood receives
Hodgkin’s but isn’t an early sign of the Rh-negative blood. If Rh-positive blood is
disease. administered to an Rh-negative person,
44. Answer: (D) Breath sounds the recipient develops anti-Rh agglutinins,
Rationale: Pneumonia, both viral and and sub sequent transfusions with Rh-
fungal, is a common cause of death in positive blood may cause serious
clients with neutropenia, so frequent reactions with clumping and hemolysis of
assessment of respiratory rate and breath red blood cells.
sounds is required. Although assessing 50. Answer: (B) “I will call my doctor if Stacy
blood pressure, bowel sounds, and heart has persistent vomiting and diarrhea”.
sounds is important, it won’t help detect Rationale: Persistent (more than 24 hours)
pneumonia. vomiting, anorexia, and diarrhea are signs
45. Answer: (B) Muscle spasm of toxicity and the patient should stop the
Rationale: Back pain or paresthesia in the medication and notify the health care
lower extremities may indicate impending provider. The other manifestations are
spinal cord compression from a spinal expected side effects of chemotherapy.
tumor. This should be recognized and 51. Answer: (D) “This is only temporary; Stacy
treated promptly as progression of the will re-grow new hair in 3-6 months, but
tumor may result in paraplegia. The other may be different in texture”.
Rationale: This is the appropriate chronic obstructive bronchitis are bloated
response. The nurse should help the and cyanotic in appearance.
mother how to cope with her own feelings 56. Answer: D 80 mm Hg
regarding the child’s disease so as not to Rationale: A client about to go into
affect the child negatively. When the hair respiratory arrest will have inefficient
grows back, it is still of the same color and ventilation and will be retaining carbon
texture. dioxide. The value expected would be
52. Answer: (B) Apply viscous Lidocaine to around 80 mm Hg. All other values are
oral ulcers as needed. lower than expected.
Rationale: Stomatitis can cause pain and 57. Answer: (C) Respiratory acidosis
this can be relieved by applying topical Rationale: Because Paco2 is high at 80 mm
anesthetics such as lidocaine before Hg and the metabolic measure, HCO3- is
mouth care. When the patient is already normal, the client has respiratory acidosis.
comfortable, the nurse can proceed with The pH is less than 7.35, academic, which
providing the patient with oral rinses of eliminates metabolic and respiratory
saline solution mixed with equal part of alkalosis as possibilities. If the HCO3- was
water or hydrogen peroxide mixed water below 22 mEq/L the client would have
in 1:3 concentrations to promote oral metabolic acidosis.
hygiene. Every 2-4 hours. 58. Answer: (C) Respiratory failure
53. Answer: (C) Immediately discontinue the Rationale: The client was reacting to the
infusion drug with respiratory signs of impending
Rationale: Edema or swelling at the IV site anaphylaxis, which could lead to
is a sign that the needle has been eventually respiratory failure. Although
dislodged and the IV solution is leaking the signs are also related to an asthma
into the tissues causing the edema. The attack or a pulmonary embolism, consider
patient feels pain as the nerves are the new drug first. Rheumatoid arthritis
irritated by pressure and the IV solution. doesn’t manifest these signs.
The first action of the nurse would be to 59. Answer: (D) Elevated serum
discontinue the infusion right away to aminotransferase
prevent further edema and other Rationale: Hepatic cell death causes
complication. release of liver enzymes alanine
54. Answer: (C) Chronic obstructive bronchitis aminotransferase (ALT), aspartate
Rationale: Clients with chronic obstructive aminotransferase (AST) and lactate
bronchitis appear bloated; they have large dehydrogenase (LDH) into the circulation.
barrel chest and peripheral edema, Liver cirrhosis is a chronic and irreversible
cyanotic nail beds, and at times, disease of the liver characterized by
circumoral cyanosis. Clients with ARDS are generalized inflammation and fibrosis of
acutely short of breath and frequently the liver tissues.
need intubation for mechanical ventilation 60. Answer: (A) Impaired clotting mechanism
and large amount of oxygen. Clients with Rationale: Cirrhosis of the liver results in
asthma don’t exhibit characteristics of decreased Vitamin K absorption and
chronic disease, and clients with formation of clotting factors resulting in
emphysema appear pink and cachectic. impaired clotting mechanism.
55. Answer: (D) Emphysema 61. Answer: (B) Altered level of consciousness
Rationale: Because of the large amount of Rationale: Changes in behavior and level
energy it takes to breathe, clients with of consciousness are the first sins of
emphysema are usually cachectic. They’re hepatic encephalopathy. Hepatic
pink and usually breathe through pursed encephalopathy is caused by liver failure
lips, hence the term “puffer.” Clients with and develops when the liver is unable to
ARDS are usually acutely short of breath. convert protein metabolic product
Clients with asthma don’t have any ammonia to urea. This results in
particular characteristics, and clients with accumulation of ammonia and other toxic
in the blood that damages the cells.

32
32
62. Answer: (C) “I’ll lower the dosage as diagnosis test. Nitroglycerin is an oral
ordered so the drug causes only 2 to 4 sublingual medication. Cardiac
stools a day”. catheterization is a diagnostic tool – not a
Rationale: Lactulose is given to a patients treatment.
with hepatic encephalopathy to reduce 66. Answer: (B) Cardiogenic shock
absorption of ammonia in the intestines Rationale: Cardiogenic shock is shock
by binding with ammonia and promoting related to ineffective pumping of the
more frequent bowel movements. If the heart. Anaphylactic shock results from an
patient experience diarrhea, it indicates allergic reaction. Distributive shock results
over dosage and the nurse must reduce from changes in the intravascular volume
the amount of medication given to the distribution and is usually associated with
patient. The stool will be mashy or soft. increased cardiac output. MI isn’t a shock
Lactulose is also very sweet and may state, though a severe MI can lead to
cause cramping and bloating. shock.
63. Answer: (B) Severe lower back pain, 67. Answer: (C) Kidneys’ excretion of sodium
decreased blood pressure, decreased RBC and water
count, increased WBC count. Rationale: The kidneys respond to rise in
Rationale: Severe lower back pain blood pressure by excreting sodium and
indicates an aneurysm rupture, secondary excess water. This response ultimately
to pressure being applied within the affects sysmolic blood pressure by
abdominal cavity. When ruptured occurs, regulating blood volume. Sodium or water
the pain is constant because it can’t be retention would only further increase
alleviated until the aneurysm is repaired. blood pressure. Sodium and water travel
Blood pressure decreases due to the loss together across the membrane in the
of blood. After the aneurysm ruptures, the kidneys; one can’t travel without the
vasculature is interrupted and blood other.
volume is lost, so blood pressure wouldn’t 68. Answer: (D) It inhibits reabsorption of
increase. For the same reason, the RBC sodium and water in the loop of Henle.
count is decreased – not increased. The Rationale: Furosemide is a loop diuretic
WBC count increases as cell migrate to the that inhibits sodium and water
site of injury. reabsorption in the loop Henle, thereby
64. Answer: (D) Apply gloves and assess the causing a decrease in blood pressure.
groin site Vasodilators cause dilation of peripheral
Rationale: Observing standard precautions blood vessels, directly relaxing vascular
is the first priority when dealing with any smooth muscle and decreasing blood
blood fluid. Assessment of the groin site is pressure. Adrenergic blockers decrease
the second priority. This establishes where sympathetic cardioacceleration and
the blood is coming from and determines decrease blood pressure. Angiotensin-
how much blood has been lost. The goal in converting enzyme inhibitors decrease
this situation is to stop the bleeding. The blood pressure due to their action on
nurse would call for help if it were angiotensin.
warranted after the assessment of the 69. Answer: (C) Pancytopenia, elevated
situation. After determining the extent of antinuclear antibody (ANA) titer
the bleeding, vital signs assessment is Rationale: Laboratory findings for clients
important. The nurse should never move with SLE usually show pancytopenia,
the client, in case a clot has formed. elevated ANA titer, and decreased serum
Moving can disturb the clot and cause complement levels. Clients may have
rebleeding. elevated BUN and creatinine levels from
65. Answer: (D) Percutaneous transluminal nephritis, but the increase does not
coronary angioplasty (PTCA) indicate SLE.
Rationale: PTCA can alleviate the blockage 70. Answer: (C) Narcotics are avoided after a
and restore blood flow and oxygenation. head injury because they may hide a
An echocardiogram is a noninvasive worsening condition.
Rationale: Narcotics may mask changes in 75. Answer: (C) Myxedema coma
the level of consciousness that indicate Rationale: Myxedema coma, severe
increased ICP and shouldn’t hypothyroidism, is a life-threatening
acetaminophen is strong enough ignores condition that may develop if thyroid
the mother’s question and therefore isn’t replacement medication isn't taken.
appropriate. Aspirin is contraindicated in Exophthalmos, protrusion of the eyeballs,
conditions that may have bleeding, such is seen with hyperthyroidism. Thyroid
as trauma, and for children or young storm is life-threatening but is caused by
adults with viral illnesses due to the severe hyperthyroidism. Tibial myxedema,
danger of Reye’s syndrome. Stronger peripheral mucinous edema involving the
medications may not necessarily lead to lower leg, is associated with
vomiting but will sedate the client, hypothyroidism but isn't life-threatening.
thereby masking changes in his level of 76. Answer: (B) An irregular apical pulse
consciousness. Rationale: Because Cushing's syndrome
71. Answer: (A) Appropriate; lowering carbon causes aldosterone overproduction, which
dioxide (CO2) reduces intracranial increases urinary potassium loss, the
pressure (ICP) disorder may lead to hypokalemia.
Rationale: A normal Paco2 value is 35 to Therefore, the nurse should immediately
45 mm Hg CO2 has vasodilating report signs and symptoms of
properties; therefore, lowering Paco2 hypokalemia, such as an irregular apical
through hyperventilation will lower ICP pulse, to the physician. Edema is an
caused by dilated cerebral vessels. expected finding because aldosterone
Oxygenation is evaluated through Pao2 overproduction causes sodium and fluid
and oxygen saturation. Alveolar retention. Dry mucous membranes and
hypoventilation would be reflected in an frequent urination signal dehydration,
increased Paco2. which isn't associated with Cushing's
72. Answer: (B) A 33-year-old client with a syndrome.
recent diagnosis of Guillain-Barre 77. Answer: (D) Below-normal urine
syndrome osmolality level, above-normal serum
Rationale: Guillain-Barre syndrome is osmolality level
characterized by ascending paralysis and Rationale: In diabetes insipidus, excessive
potential respiratory failure. The order of polyuria causes dilute urine, resulting in a
client assessment should follow client below-normal urine osmolality level. At
priorities, with disorder of airways, the same time, polyuria depletes the body
breathing, and then circulation. There’s no of water, causing dehydration that leads
information to suggest the postmyocardial to an above-normal serum osmolality
infarction client has an arrhythmia or level. For the same reasons, diabetes
other complication. There’s no evidence insipidus doesn't cause above-normal
to suggest hemorrhage or perforation for urine osmolality or below-normal serum
the remaining clients as a priority of care. osmolality levels.
73. Answer: (C) Decreases inflammation 78. Answer: (A) "I can avoid getting sick by not
Rationale: Then action of colchicines is to becoming dehydrated and by paying
decrease inflammation by reducing the attention to my need to urinate, drink, or
migration of leukocytes to synovial fluid. eat more than usual."
Colchicine doesn’t replace estrogen, Rationale: Inadequate fluid intake during
decrease infection, or decrease bone hyperglycemic episodes often leads to
demineralization. HHNS. By recognizing the signs of
74. Answer: (C) Osteoarthritis is the most hyperglycemia (polyuria, polydipsia, and
common form of arthritis polyphagia) and increasing fluid intake,
Rationale: Osteoarthritis is the most the client may prevent HHNS. Drinking a
common form of arthritis and can be glass of nondiet soda would be
extremely debilitating. It can afflict people appropriate for hypoglycemia. A client
of any age, although most are elderly. whose diabetes is controlled with oral

32
32
antidiabetic agents usually doesn't need dangerously imbalanced. Temperature
to monitor blood glucose levels. A high- regulation may be affected by excess
carbohydrate diet would exacerbate the cortisol and isn't an accurate indicator of
client's condition, particularly if fluid infection.
intake is low. 83. Answer: (C) onset to be at 2:30 p.m. and
79. Answer: (D) Hyperparathyroidism its peak to be at 4 p.m.
Rationale: Hyperparathyroidism is most Rationale: Regular insulin, which is a
common in older women and is short-acting insulin, has an onset of 15 to
characterized by bone pain and weakness 30 minutes and a peak of 2 to 4 hours.
from excess parathyroid hormone (PTH). Because the nurse gave the insulin at 2
Clients also exhibit hypercaliuria-causing p.m., the expected onset would be from
polyuria. While clients with diabetes 2:15 p.m. to 2:30 p.m. and the peak from
mellitus and diabetes insipidus also have 4 p.m. to 6 p.m.
polyuria, they don't have bone pain and 84. Answer: (A) No increase in the thyroid-
increased sleeping. Hypoparathyroidism is stimulating hormone (TSH) level after 30
characterized by urinary frequency rather minutes during the TSH stimulation test
than polyuria. Rationale: In the TSH test, failure of the
80. Answer: (C) "I'll take two-thirds of the TSH level to rise after 30 minutes confirms
dose when I wake up and one-third in the hyperthyroidism. A decreased TSH level
late afternoon." indicates a pituitary deficiency of this
Rationale: Hydrocortisone, a hormone. Below-normal levels of T3 and
glucocorticoid, should be administered T4, as detected by radioimmunoassay,
according to a schedule that closely signal hypothyroidism. A below-normal T4
reflects the bodies own secretion of this level also occurs in malnutrition and liver
hormone; therefore, two-thirds of the disease and may result from
dose of hydrocortisone should be taken in administration of phenytoin and certain
the morning and one-third in the late other drugs.
afternoon. This dosage schedule reduces 85. Answer: (B) "Rotate injection sites within
adverse effects. the same anatomic region, not among
81. Answer: (C) High corticotropin and high different regions."
cortisol levels Rationale: The nurse should instruct the
Rationale: A corticotropin-secreting client to rotate injection sites within the
pituitary tumor would cause high same anatomic region. Rotating sites
corticotropin and high cortisol levels. A among different regions may cause
high corticotropin level with a low cortisol excessive day-to-day variations in the
level and a low corticotropin level with a blood glucose level; also, insulin
low cortisol level would be associated absorption differs from one region to the
with hypocortisolism. Low corticotropin next. Insulin should be injected only into
and high cortisol levels would be seen if healthy tissue lacking large blood vessels,
there was a primary defect in the adrenal nerves, or scar tissue or other deviations.
glands. Injecting insulin into areas of hypertrophy
82. Answer: (D) Performing capillary glucose may delay absorption. The client shouldn't
testing every 4 hours inject insulin into areas of lipodystrophy
Rationale: The nurse should perform (such as hypertrophy or atrophy); to
capillary glucose testing every 4 hours prevent lipodystrophy, the client should
because excess cortisol may cause insulin rotate injection sites systematically.
resistance, placing the client at risk for Exercise speeds drug absorption, so the
hyperglycemia. Urine ketone testing isn't client shouldn't inject insulin into sites
indicated because the client does secrete above muscles that will be exercised
insulin and, therefore, isn't at risk for heavily.
ketosis. Urine specific gravity isn't 86. Answer: (D) Below-normal serum
indicated because although fluid balance potassium level
can be compromised, it usually isn't
Rationale: A client with HHNS has an He could develop atelectasis but it
overall body deficit of potassium resulting typically doesn’t produce progressive
from diuresis, which occurs secondary to hypoxia.
the hyperosmolar, hyperglycemic state 92. Answer: (D) Spontaneous pneumothorax
caused by the relative insulin deficiency. Rationale: A spontaneous pneumothorax
An elevated serum acetone level and occurs when the client’s lung collapses,
serum ketone bodies are characteristic of causing an acute decreased in the amount
diabetic ketoacidosis. Metabolic acidosis, of functional lung used in oxygenation.
not serum alkalosis, may occur in HHNS. The sudden collapse was the cause of his
87. Answer: (D) Maintaining room chest pain and shortness of breath. An
temperature in the low-normal range asthma attack would show wheezing
Rationale: Graves' disease causes signs breath sounds, and bronchitis would have
and symptoms of hypermetabolism, such rhonchi. Pneumonia would have bronchial
as heat intolerance, diaphoresis, excessive breath sounds over the area of
thirst and appetite, and weight loss. To consolidation.
reduce heat intolerance and diaphoresis, 93. Answer: (C) Pneumothorax
the nurse should keep the client's room Rationale: From the trauma the client
temperature in the low-normal range. To experienced, it’s unlikely he has
replace fluids lost via diaphoresis, the bronchitis, pneumonia, or TB; rhonchi
nurse should encourage, not restrict, with bronchitis, bronchial breath sounds
intake of oral fluids. Placing extra blankets with TB would be heard.
on the bed of a client with heat 94. Answer: (C) Serous fluids fills the space
intolerance would cause discomfort. To and consolidates the region
provide needed energy and calories, the Rationale: Serous fluid fills the space and
nurse should encourage the client to eat eventually consolidates, preventing
high-carbohydrate foods. extensive mediastinal shift of the heart
88. Answer: (A) Fracture of the distal radius and remaining lung. Air can’t be left in the
Rationale: Colles' fracture is a fracture of space. There’s no gel that can be placed in
the distal radius, such as from a fall on an the pleural space. The tissue from the
outstretched hand. It's most common in other lung can’t cross the mediastinum,
women. Colles' fracture doesn't refer to a although a temporary mediastinal shift
fracture of the olecranon, humerus, or exits until the space is filled.
carpal scaphoid. 95. Answer: (A) Alveolar damage in the
89. Answer: (B) Calcium and phosphorous infracted area
Rationale: In osteoporosis, bones lose Rationale: The infracted area produces
calcium and phosphate salts, becoming alveolar damage that can lead to the
porous, brittle, and abnormally vulnerable production of bloody sputum, sometimes
to fracture. Sodium and potassium aren't in massive amounts. Clot formation
involved in the development of usually occurs in the legs. There’s a loss of
steoporosis. lung parenchyma and subsequent scar
90. Answer: (A) Adult respiratory distress tissue formation.
syndrome (ARDS) 96. Answer: (D) Respiratory alkalosis
Rationale: Severe hypoxia after smoke Rationale: A client with massive
inhalation is typically related to ARDS. The pulmonary embolism will have a large
other conditions listed aren’t typically region and blow off large amount of
associated with smoke inhalation and carbon dioxide, which crosses the
severe hypoxia. unaffected alveolar-capillary membrane
91. Answer: (D) Fat embolism more readily than does oxygen and results
Rationale: Long bone fractures are in respiratory alkalosis.
correlated with fat emboli, which cause 97. Answer: (A) Air leak
shortness of breath and hypoxia. It’s Rationale: Bubbling in the water seal
unlikely the client has developed asthma chamber of a chest drainage system stems
or bronchitis without a previous history. from an air leak. In pneumothorax an air

33
33
leak can occur as air is pulled from the
pleural space. Bubbling doesn’t normally
occur with either adequate or inadequate
suction or any preexisting bubbling in the
water seal chamber.
98. Answer: (B) 21
Rationale: 3000 x 10 divided by 24 x 60.
99. Answer: (B) 2.4 ml
Rationale: .05 mg/ 1 ml = .12mg/ x ml,
.05x = .12, x = 2.4 ml.
100. Answer: (D) “I should put on the stockings
before getting out of bed in the morning.
Rationale: Promote venous return by
applying external pressure on veins.
TEST V - Care of Clients with Physiologic and d. Suggest that the father and son work
Psychosocial Alterations things out.

1. Mr. Marquez reports of losing his job, not being 5. What is Nurse John likely to note in a male client
able to sleep at night, and feeling upset with his being admitted for alcohol withdrawal?
wife. Nurse John responds to the client, “You a. Perceptual disorders.
may want to talk about your employment b. Impending coma.
situation in group today.” The Nurse is using c. Recent alcohol intake.
which therapeutic technique? d. Depression with mutism.
a. Observations
b. Restating 6. Aira has taken amitriptyline HCL (Elavil) for 3
c. Exploring days, but now complains that it “doesn’t help”
d. Focusing and refuses to take it. What should the nurse say
or do?
2. Tony refuses his evening dose of Haloperidol a. Withhold the drug.
(Haldol), then becomes extremely agitated in the b. Record the client’s response.
dayroom while other clients are watching c. Encourage the client to tell the doctor.
television. He begins cursing and throwing d. Suggest that it takes a while before
furniture. Nurse Oliver first action is to: seeing the results.
a. Check the client’s medical record for an
order for an as-needed I.M. dose of 7. Dervid, an adolescent has a history of truancy
medication for agitation. from school, running away from home and
b. Place the client in full leather restraints. “barrowing” other people’s things without their
c. Call the attending physician and report permission. The adolescent denies stealing,
the behavior. rationalizing instead that as long as no one was
d. Remove all other clients from the using the items, it was all right to borrow them.
dayroom. It is important for the nurse to understand the
psychodynamically, this behavior may be largely
3. Tina who is manic, but not yet on medication, attributed to a developmental defect related to
comes to the drug treatment center. The nurse the:
would not let this client join the group session a. Id
because: b. Ego
a. The client is disruptive. c. Superego
b. The client is harmful to self. d. Oedipal complex
c. The client is harmful to others.
d. The client needs to be on medication 8. In preparing a female client for electroconvulsive
first. therapy (ECT), Nurse Michelle knows that
succinylcoline (Anectine) will be administered
4. Dervid, an adolescent boy was admitted for for which therapeutic effect?
substance abuse and hallucinations. The client’s a. Short-acting anesthesia
mother asks Nurse Armando to talk with his b. Decreased oral and respiratory
husband when he arrives at the hospital. The secretions.
mother says that she is afraid of what the father c. Skeletal muscle paralysis.
might say to the boy. The most appropriate d. Analgesia.
nursing intervention would be to:
a. Inform the mother that she and the 9. Nurse Gina is aware that the dietary implications
father can work through this problem for a client in manic phase of bipolar disorder is:
themselves. a. Serve the client a bowl of soup, buttered
b. Refer the mother to the hospital social French bread, and apple slices.
worker. b. Increase calories, decrease fat, and
c. Agree to talk with the mother and the decrease protein.
father together. c. Give the client pieces of cut-up steak,
carrots, and an apple.

33
33
d. Increase calories, carbohydrates, and deal with this conflict if you want to walk
protein. again."
b. "It must be awful not to be able to move
10. What parental behavior toward a child during an your legs. You may feel better if you
admission procedure should cause Nurse Ron to realize the problem is psychological, not
suspect child abuse? physical."
a. Flat affect c. "Your problem is real but there is no
b. Expressing guilt physical basis for it. We'll work on what
c. Acting overly solicitous toward the child. is going on in your life to find out why
d. Ignoring the child. it's happened."
d. "It isn't uncommon for someone with
11. Nurse Lynnette notices that a female client with your personality to develop a conversion
obsessive-compulsive disorder washes her hands disorder during times of stress."
for long periods each day. How should the nurse
respond to this compulsive behavior? 14. Nurse Krina knows that the following drugs have
a. By designating times during which the been known to be effective in treating
client can focus on the behavior. obsessive-compulsive disorder (OCD):
b. By urging the client to reduce the a. benztropine (Cogentin) and
frequency of the behavior as rapidly as diphenhydramine (Benadryl).
possible. b. chlordiazepoxide (Librium) and
c. By calling attention to or attempting to diazepam (Valium)
prevent the behavior. c. fluvoxamine (Luvox) and clomipramine
d. By discouraging the client from (Anafranil)
verbalizing anxieties. d. divalproex (Depakote) and lithium
(Lithobid)
12. After seeking help at an outpatient mental
health clinic, Ruby who was raped while walking 15. Alfred was newly diagnosed with anxiety
her dog is diagnosed with posttraumatic stress disorder. The physician prescribed buspirone
disorder (PTSD). Three months later, Ruby (BuSpar). The nurse is aware that the teaching
returns to the clinic, complaining of fear, loss of instructions for newly prescribed buspirone
control, and helpless feelings. Which nursing should include which of the following?
intervention is most appropriate for Ruby? a. A warning about the drugs delayed
a. Recommending a high-protein, low-fat therapeutic effect, which is from 14 to
diet. 30 days.
b. Giving sleep medication, as prescribed, b. A warning about the incidence of
to restore a normal sleep- wake cycle. neuroleptic malignant syndrome (NMS).
c. Allowing the client time to heal. c. A reminder of the need to schedule
d. Exploring the meaning of the traumatic blood work in 1 week to check blood
event with the client. levels of the drug.
d. A warning that immediate sedation can
13. Meryl, age 19, is highly dependent on her occur with a resultant drop in pulse.
parents and fears leaving home to go away to
college. Shortly before the semester starts, she 16. Richard with agoraphobia has been symptom-
complains that her legs are paralyzed and is free for 4 months. Classic signs and symptoms
rushed to the emergency department. When of phobias include:
physical examination rules out a physical cause a. Insomnia and an inability to concentrate.
for her paralysis, the physician admits her to the b. Severe anxiety and fear.
psychiatric unit where she is diagnosed with c. Depression and weight loss.
conversion disorder. Meryl asks the nurse, "Why d. Withdrawal and failure to distinguish
has this happened to me?" What is the nurse's reality from fantasy.
best response?
a. "You've developed this paralysis so you 17. Which medications have been found to help
can stay with your parents. You must reduce or eliminate panic attacks?
a. Antidepressants d. A low tolerance for frustration
b. Anticholinergics
c. Antipsychotics 22. Nurse Amy is providing care for a male client
d. Mood stabilizers undergoing opiate withdrawal. Opiate
withdrawal causes severe physical discomfort
18. A client seeks care because she feels depressed and can be life-threatening. To minimize these
and has gained weight. To treat her atypical effects, opiate users are commonly detoxified
depression, the physician prescribes with:
tranylcypromine sulfate (Parnate), 10 mg by a. Barbiturates
mouth twice per day. When this drug is used to b. Amphetamines
treat atypical depression, what is its onset of c. Methadone
action? d. Benzodiazepines
a. 1 to 2 days
b. 3 to 5 days 23. Nurse Cristina is caring for a client who
c. 6 to 8 days experiences false sensory perceptions with no
d. 10 to 14 days basis in reality. These perceptions are known as:
a. Delusions
19. A 65 years old client is in the first stage of b. Hallucinations
Alzheimer's disease. Nurse Patricia should plan c. Loose associations
to focus this client's care on: d. Neologisms
a. Offering nourishing finger foods to help
maintain the client's nutritional status. 24. Nurse Marco is developing a plan of care for a
b. Providing emotional support and client with anorexia nervosa. Which action
individual counseling. should the nurse include in the plan?
c. Monitoring the client to prevent minor a. Restricts visits with the family and
illnesses from turning into major friends until the client begins to eat.
problems. b. Provide privacy during meals.
d. Suggesting new activities for the client c. Set up a strict eating plan for the client.
and family to do together. d. Encourage the client to exercise, which
will reduce her anxiety.
20. The nurse is assessing a client who has just been
admitted to the emergency department. Which 25. Tim is admitted with a diagnosis of delusions of
signs would suggest an overdose of an grandeur. The nurse is aware that this diagnosis
antianxiety agent? reflects a belief that one is:
a. Combativeness, sweating, and confusion a. Highly important or famous.
b. Agitation, hyperactivity, and grandiose b. Being persecuted
ideation c. Connected to events unrelated to
c. Emotional lability, euphoria, and oneself
impaired memory d. Responsible for the evil in the world.
d. Suspiciousness, dilated pupils, and
increased blood pressure 26. Nurse Jen is caring for a male client with manic
depression. The plan of care for a client in a
21. The nurse is caring for a client diagnosed with manic state would include:
antisocial personality disorder. The client has a a. Offering a high-calorie meals and
history of fighting, cruelty to animals, and strongly encouraging the client to finish
stealing. Which of the following traits would the all food.
nurse be most likely to uncover during b. Insisting that the client remain active
assessment? through the day so that he’ll sleep at
a. History of gainful employment night.
b. Frequent expression of guilt regarding c. Allowing the client to exhibit
antisocial behavior hyperactive, demanding, manipulative
c. Demonstrated ability to maintain close, behavior without setting limits.
stable relationships

33
33
d. Listening attentively with a neutral 32. The nurse is aware that the side effect of
attitude and avoiding power struggles. electroconvulsive therapy that a client may
experience:
27. Ramon is admitted for detoxification after a a. Loss of appetite
cocaine overdose. The client tells the nurse that b. Postural hypotension
he frequently uses cocaine but that he can c. Confusion for a time after treatment
control his use if he chooses. Which coping d. Complete loss of memory for a time
mechanism is he using? 33. A dying male client gradually moves toward
a. Withdrawal resolution of feelings regarding impending
b. Logical thinking death. Basing care on the theory of Kubler-Ross,
c. Repression Nurse Trish plans to use nonverbal interventions
d. Denial when assessment reveals that the client is in the:
a. Anger stage
28. Richard is admitted with a diagnosis of b. Denial stage
schizotypal personality disorder. hich signs c. Bargaining stage
would this client exhibit during social situations? d. Acceptance stage
a. Aggressive behavior
b. Paranoid thoughts 34. The outcome that is unrelated to a crisis state is:
c. Emotional affect a. Learning more constructive coping skills
d. Independence needs b. Decompensation to a lower level of
functioning.
29. Nurse Mickey is caring for a client diagnosed c. Adaptation and a return to a prior level
with bulimia. The most appropriate initial goal of functioning.
for a client diagnosed with bulimia is to: d. A higher level of anxiety continuing for
a. Avoid shopping for large amounts of more than 3 months.
food.
b. Control eating impulses. 35. Miranda a psychiatric client is to be discharged
c. Identify anxiety-causing situations with orders for haloperidol (haldol) therapy.
d. Eat only three meals per day. When developing a teaching plan for discharge,
the nurse should include cautioning the client
30. Rudolf is admitted for an overdose of against:
amphetamines. When assessing the client, the a. Driving at night
nurse should expect to see: b. Staying in the sun
a. Tension and irritability c. Ingesting wines and cheeses
b. Slow pulse d. Taking medications containing aspirin
c. Hypotension
d. Constipation 36. Jen a nursing student is anxious about the
upcoming board examination but is able to study
31. Nicolas is experiencing hallucinations tells the intently and does not become distracted by a
nurse, “The voices are telling me I’m no good.” roommate’s talking and loud music. The
The client asks if the nurse hears the voices. The student’s ability to ignore distractions and to
most appropriate response by the nurse would focus on studying demonstrates:
be: a. Mild-level anxiety
a. “It is the voice of your conscience, which b. Panic-level anxiety
only you can control.” c. Severe-level anxiety
b. “No, I do not hear your voices, but I d. Moderate-level anxiety
believe you can hear them”.
c. “The voices are coming from within you 37. When assessing a premorbid personality
and only you can hear them.” characteristic of a client with a major
d. “Oh, the voices are a symptom of your depression, it would be unusual for the nurse to
illness; don’t pay any attention to them.” find that this client demonstrated:
a. Rigidity
b. Stubbornness
c. Diverse interest
d. Over meticulousness 43. When establishing an initial nurse-client
relationship, Nurse Hazel should explore with
38. Nurse Krina recognizes that the suicidal risk for the client the:
depressed client is greatest: a. Client’s perception of the presenting
a. As their depression begins to improve problem.
b. When their depression is most severe b. Occurrence of fantasies the client may
c. Before any type of treatment is started experience.
d. As they lose interest in the environment c. Details of any ritualistic acts carried out
by the client
39. Nurse Kate would expect that a client with d. Client’s feelings when external; controls
vascular dementis would experience: are instituted.
a. Loss of remote memory related
to anoxia 44. Tranylcypromine sulfate (Parnate) is prescribed
b. Loss of abstract thinking related to for a depressed client who has not responded to
emotional state the tricyclic antidepressants. After teaching the
c. Inability to concentrate related to client about the medication, Nurse Marian
decreased stimuli evaluates that learning has occurred when the
d. Disturbance in recalling recent events client states, “I will avoid:
related to cerebral hypoxia. a. Citrus fruit, tuna, and yellow
vegetables.”
40. Josefina is to be discharged on a regimen of b. Chocolate milk, aged cheese, and
lithium carbonate. In the teaching plan for yogurt’”
discharge the nurse should include: c. Green leafy vegetables, chicken, and
a. Advising the client to watch the diet milk.”
carefully d. Whole grains, red meats, and
b. Suggesting that the client take the pills carbonated soda.”
with milk
c. Reminding the client that a CBC must be 45. Nurse John is a aware that most crisis situations
done once a month. should resolve in about:
d. Encouraging the client to have blood a. 1 to 2 weeks
levels checked as ordered. b. 4 to 6 weeks
c. 4 to 6 months
41. The psychiatrist orders lithium carbonate 600 d. 6 to 12 months
mg p.o t.i.d for a female client. Nurse Katrina
would be aware that the teachings about the 46. Nurse Judy knows that statistics show that in
side effects of this drug were understood when adolescent suicide behavior:
the client state, “I will call my doctor a. Females use more dramatic methods
immediately if I notice any: than males
a. Sensitivity to bright light or sun b. Males account for more attempts than
b. Fine hand tremors or slurred speech do females
c. Sexual dysfunction or breast c. Females talk more about suicide before
enlargement attempting it
d. Inability to urinate or difficulty when d. Males are more likely to use lethal
urinating methods than are females

42. Nurse Mylene recognizes that the most 47. Dervid with paranoid schizophrenia repeatedly
important factor necessary for the establishment uses profanity during an activity therapy session.
of trust in a critical care area is: Which response by the nurse would be most
a. Privacy appropriate?
b. Respect a. "Your behavior won't be tolerated. Go to
c. Empathy your room immediately."
d. Presence

33
33
b. "You're just doing this to get back at me 52. Mr. Cruz visits the physician's office to seek
for making you come to therapy." treatment for depression, feelings of
c. "Your cursing is interrupting the activity. hopelessness, poor appetite, insomnia, fatigue,
Take time out in your room for 10 low self- esteem, poor concentration, and
minutes." difficulty making decisions. The client states that
d. "I'm disappointed in you. You can't these symptoms began at least 2 years ago.
control yourself even for a few minutes." Based on this report, the nurse Tyfany suspects:
a. Cyclothymic disorder.
48. Nurse Maureen knows that the nonantipsychotic b. Atypical affective disorder.
medication used to treat some clients with c. Major depression.
schizoaffective disorder is: d. Dysthymic disorder.
a. phenelzine (Nardil)
b. chlordiazepoxide (Librium) 53. After taking an overdose of phenobarbital
c. lithium carbonate (Lithane) (Barbita), Mario is admitted to the emergency
d. imipramine (Tofranil) department. Dr. Trinidad prescribes activated
charcoal (Charcocaps) to be administered by
49. Which information is most important for the mouth immediately. Before administering the
nurse Trinity to include in a teaching plan for a dose, the nurse verifies the dosage ordered.
male schizophrenic client taking clozapine What is the usual minimum dose of activated
(Clozaril)? charcoal?
a. Monthly blood tests will be necessary. a. 5 g mixed in 250 ml of water
b. Report a sore throat or fever to the b. 15 g mixed in 500 ml of water
physician immediately. c. 30 g mixed in 250 ml of water
c. Blood pressure must be monitored for d. 60 g mixed in 500 ml of water
hypertension.
d. Stop the medication when symptoms 54. What herbal medication for depression, widely
subside. used in Europe, is now being prescribed in the
United States?
50. Ricky with chronic schizophrenia takes a. Ginkgo biloba
neuroleptic medication is admitted to the b. Echinacea
psychiatric unit. Nursing assessment reveals c. St. John's wort
rigidity, fever, hypertension, and diaphoresis. d. Ephedra
These findings suggest which life- threatening
reaction: 55. Cely with manic episodes is taking lithium.
a. Tardive dyskinesia. Which electrolyte level should the nurse check
b. Dystonia. before administering this medication?
c. Neuroleptic malignant syndrome. a. Clcium
d. Akathisia. b. Sodium
c. Chloride
51. Which nursing intervention would be most d. Potassium
appropriate if a male client develop orthostatic
hypotension while taking amitriptyline (Elavil)? 56. Nurse Josefina is caring for a client who has been
a. Consulting with the physician about diagnosed with delirium. Which statement about
substituting a different type of delirium is true?
antidepressant. a. It's characterized by an acute onset and
b. Advising the client to sit up for 1 minute lasts about 1 month.
before getting out of bed. b. It's characterized by a slowly evolving
c. Instructing the client to double the onset and lasts about 1 week.
dosage until the problem resolves. c. It's characterized by a slowly evolving
d. Informing the client that this adverse onset and lasts about 1 month.
reaction should disappear within 1 d. It's characterized by an acute onset and
week. lasts hours to a number of days.
57. Edward, a 66 year old client with slight memory
impairment and poor concentration is diagnosed 61. Mr. Garcia, an attorney who throws books and
with primary degenerative dementia of the furniture around the office after losing a case is
Alzheimer's type. Early signs of this dementia referred to the psychiatric nurse in the law firm's
include subtle personality changes and employee assistance program. Nurse Beatriz
withdrawal from social interactions. To assess knows that the client's behavior most likely
for progression to the middle stage of represents the use of which defense
Alzheimer's disease, the nurse should observe mechanism?
the client for: a. Regression
a. Occasional irritable outbursts. b. Projection
b. Impaired communication. c. Reaction-formation
c. Lack of spontaneity. d. Intellectualization
d. Inability to perform self-care activities.
62. Nurse Anne is caring for a client who has been
58. Isabel with a diagnosis of depression is started treated long term with antipsychotic medication.
on imipramine (Tofranil), 75 mg by mouth at During the assessment, Nurse Anne checks the
bedtime. The nurse should tell the client that: client for tardive dyskinesia. If tardive dyskinesia
a. This medication may be habit forming is present, Nurse Anne would most likely
and will be discontinued as soon as the observe:
client feels better. a. Abnormal movements and involuntary
b. This medication has no serious adverse movements of the mouth, tongue, and
effects. face.
c. The client should avoid eating such b. Abnormal breathing through the nostrils
foods as aged cheeses, yogurt, and accompanied by a “thrill.”
chicken livers while taking the c. Severe headache, flushing, tremors, and
medication. ataxia.
d. This medication may initially cause d. Severe hypertension, migraine
tiredness, which should become less headache,
bothersome over time.
63. Dennis has a lithium level of 2.4 mEq/L. The
59. Kathleen is admitted to the psychiatric clinic for nurse immediately would assess the client for
treatment of anorexia nervosa. To promote the which of the following signs or symptoms?
client's physical health, the nurse should plan a. Weakness
to: b. Diarrhea
a. Severely restrict the client's physical c. Blurred vision
activities. d. Fecal incontinence
b. Weigh the client daily, after the evening
meal. 64. Nurse Jannah is monitoring a male client who
c. Monitor vital signs, serum electrolyte has been placed inrestraints because of violent
levels, and acid-base balance. behavior. Nurse determines that it will be safe to
d. Instruct the client to keep an accurate remove the restraints when:
record of food and fluid intake. a. The client verbalizes the reasons for the
violent behavior.
60. Celia with a history of polysubstance abuse is b. The client apologizes and tells the nurse
admitted to the facility. She complains of nausea that it will never happen again.
and vomiting 24 hours after admission. The c. No acts of aggression have been
nurse assesses the client and notes piloerection, observed within 1 hour after the release
pupillary dilation, and lacrimation. The nurse of two of the extremity restraints.
suspects that the client is going through which of d. The administered medication has taken
the following withdrawals? effect.
a. Alcohol withdrawal
b. Cannibis withdrawal
c. Cocaine withdrawal
d. Opioid withdrawal
33
33
65. Nurse Irish is aware that Ritalin is the drug of a. Revealing personal information to the
choice for a child with ADHD. The side effects of client
the following may be noted by the nurse: b. Focusing on the feelings of the client.
a. Increased attention span and c. Confronting the client about
concentration discrepancies in verbal or non-verbal
b. Increase in appetite behavior
c. Sleepiness and lethargy d. The client feels angry towards the nurse
d. Bradycardia and diarrhea who resembles his mother.

66. Kitty, a 9 year old child has very limited 72. Tristan is on Lithium has suffered from diarrhea
vocabulary and interaction skills. She has an I.Q. and vomiting. What should the nurse in-charge
of 45. She is diagnosed to have Mental do first:
retardation of this classification: a. Recognize this as a drug interaction
a. Profound b. Give the client Cogentin
b. Mild c. Reassure the client that these are
c. Moderate common side effects of lithium therapy
d. Severe d. Hold the next dose and obtain an order
for a stat serum lithium level
67. The therapeutic approach in the care of Armand
an autistic child include the following EXCEPT: 73. Nurse Sarah ensures a therapeutic environment
a. Engage in diversionary activities when for all the client. Which of the following best
acting -out describes a therapeutic milieu?
b. Provide an atmosphere of acceptance a. A therapy that rewards adaptive
c. Provide safety measures behavior
d. Rearrange the environment to activate b. A cognitive approach to change behavior
the child c. A living, learning or working
environment.
68. Jeremy is brought to the emergency room by d. A permissive and congenial environment
friends who state that he took something an
hour ago. He is actively hallucinating, agitated, 74. Anthony is very hostile toward one of the staff
with irritated nasal septum. for no apparent reason. He is manifesting:
a. Heroin a. Splitting
b. Cocaine b. Transference
c. LSD c. Countertransference
d. Marijuana d. Resistance

69. Nurse Pauline is aware that Dementia unlike 75. Marielle, 17 years old was sexually attacked
delirium is characterized by: while on her way home from school. She is
a. Slurred speech brought to the hospital by her mother. Rape is
b. Insidious onset an example of which type of crisis:
c. Clouding of consciousness a. Situational
d. Sensory perceptual change b. Adventitious
c. Developmental
70. A 35 year old female has intense fear of riding an d. Internal
elevator. She claims “ As if I will die inside.” The
client is suffering from: 76. Nurse Greta is aware that the following is
a. Agoraphobia classified as an Axis I disorder by the Diagnosis
b. Social phobia and Statistical Manual of Mental Disorders, Text
c. Claustrophobia Revision (DSM-IV-TR) is:
d. Xenophobia a. Obesity
b. Borderline personality disorder
71. Nurse Myrna develops a counter-transference c. Major depression
reaction. This is evidenced by: d. Hypertension
d. It promotes emotional support or
77. Katrina, a newly admitted is extremely hostile attention for the client
toward a staff member she has just met, without
apparent reason. According to Freudian theory, 82. Dervid is diagnosed with panic disorder with
the nurse should suspect that the client is agoraphobia is talking with the nurse in-charge
experiencing which of the following about the progress made in treatment. Which of
phenomena? the following statements indicates a positive
a. Intellectualization client response?
b. Transference a. “I went to the mall with my friends last
c. Triangulation Saturday”
d. Splitting b. “I’m hyperventilating only when I have a
panic attack”
78. An 83year-old male client is in extended care c. “Today I decided that I can stop taking
facility is anxious most of the time and my medication”
frequently complains of a number of vague d. “Last night I decided to eat more than a
symptoms that interfere with his ability to eat. bowl of cereal”
These symptoms indicate which of the following
disorders? 83. The effectiveness of monoamine oxidase (MAO)
a. Conversion disorder inhibitor drug therapy in a client with
b. Hypochondriasis posttraumatic stress disorder can be
c. Severe anxiety demonstrated by which of the following client
d. Sublimation self –reports?
a. “I’m sleeping better and don’t have
79. Charina, a college student who frequently visited nightmares”
the health center during the past year with b. “I’m not losing my temper as much”
multiple vague complaints of GI symptoms c. “I’ve lost my craving for alcohol”
before course examinations. Although physical d. I’ve lost my phobia for water”
causes have been eliminated, the student
continues to express her belief that she has a 84. Mark, with a diagnosis of generalized anxiety
serious illness. These symptoms are typically of disorder wants to stop taking his lorazepam
which of the following disorders? (Ativan). Which of the following important facts
a. Conversion disorder should nurse Betty discuss with the client about
b. Depersonalization discontinuing the medication?
c. Hypochondriasis a. Stopping the drug may cause depression
d. Somatization disorder b. Stopping the drug increases cognitive
abilities
80. Nurse Daisy is aware that the following c. Stopping the drug decreases sleeping
pharmacologic agents are sedative- hypnotic difficulties
medication is used to induce sleep for a client d. Stopping the drug can cause withdrawal
experiencing a sleep disorder is: symptoms
a. Triazolam (Halcion)
b. Paroxetine (Paxil)\ 85. Jennifer, an adolescent who is depressed and
c. Fluoxetine (Prozac) reported by his parents as having difficulty in
d. Risperidone (Risperdal) school is brought to the community mental
health center to be evaluated. Which of the
81. Aldo, with a somatoform pain disorder may following other health problems would the nurse
obtain secondary gain. Which of the following suspect?
statement refers to a secondary gain? a. Anxiety disorder
a. It brings some stability to the family b. Behavioral difficulties
b. It decreases the preoccupation with the c. Cognitive impairment
physical illness d. Labile moods
c. It enables the client to avoid some
unpleasant activity

34
34
86. Ricardo, an outpatient in psychiatric facility is c. The client becomes anxious whenever
diagnosed with dysthymic disorder. Which of the the nurse leaves the bedside
following statement about dysthymic disorder is d. The client looks at the shadow on a wall
true? and tells the nurse she sees frightening
a. It involves a mood range from moderate faces on the wall.
depression to hypomania
b. It involves a single manic depression 91. During conversation of Nurse John with a client,
c. It’s a form of depression that occurs in he observes that the client shift from one topic
the fall and winter to the next on a regular basis. Which of the
d. It’s a mood disorder similar to major following terms describes this disorder?
depression but of mild to moderate a. Flight of ideas
severity b. Concrete thinking
c. Ideas of reference
87. The nurse is aware that the following ways in d. Loose association
vascular dementia different from Alzheimer’s
disease is: 92. Francis tells the nurse that her coworkers are
a. Vascular dementia has more abrupt sabotaging the computer. When the nurse asks
onset questions, the client becomes argumentative.
b. The duration of vascular dementia is This behavior shows personality traits associated
usually brief with which of the following personality disorder?
c. Personality change is common in a. Antisocial
vascular dementia b. Histrionic
d. The inability to perform motor activities c. Paranoid
occurs in vascular dementia d. Schizotypal

88. Loretta, a newly admitted client was diagnosed 93. Which of the following interventions is
with delirium and has history of hypertension important for a Cely experiencing with paranoid
and anxiety. She had been taking digoxin, personality disorder taking olanzapine
furosemide (Lasix), and diazepam (Valium) for (Zyprexa)?
anxiety. This client’s impairment may be related a. Explain effects of serotonin syndrome
to which of the following conditions? b. Teach the client to watch for
a. Infection extrapyramidal adverse reaction
b. Metabolic acidosis c. Explain that the drug is less affective if
c. Drug intoxication the client smokes
d. Hepatic encephalopathy d. Discuss the need to report paradoxical
effects such as euphoria
89. Nurse Ron enters a client’s room, the client says,
“They’re crawling on my sheets! Get them off 94. Nurse Alexandra notices other clients on the unit
my bed!” Which of the following assessment is avoiding a client diagnosed with antisocial
the most accurate? personality disorder. When discussing
a. The client is experiencing aphasia appropriate behavior in group therapy, which of
b. The client is experiencing dysarthria the following comments is expected about this
c. The client is experiencing a flight of ideas client by his peers?
d. The client is experiencing visual a. Lack of honesty
hallucination b. Belief in superstition
c. Show of temper tantrums
90. Which of the following descriptions of a client’s d. Constant need for attention
experience and behavior can be assessed as an
illusion? 95. Tommy, with dependent personality disorder is
a. The client tries to hit the nurse when working to increase his self- esteem. Which of
vital signs must be taken the following statements by the Tommy shows
b. The client says, “I keep hearing a voice teaching was successful?
telling me to run away”
a. “I’m not going to look just at the 100. Rocky has started taking haloperidol (Haldol).
negative things about myself” Which of the following instructions is most
b. “I’m most concerned about my level of appropriate for Ricky before taking
competence and progress” haloperidol?
c. “I’m not as envious of the things other a. Should report feelings of restlessness or
people have as I used to be” agitation at once
d. “I find I can’t stop myself from taking b. Use a sunscreen outdoors on a year-
over things other should be doing” round basis
c. Be aware you’ll feel increased energy
96. Norma, a 42-year-old client with a diagnosis of taking this drug
chronic undifferentiated schizophrenia lives in a d. This drug will indirectly control essential
rooming house that has a weekly nursing clinic. hypertension
She scratches while she tells the nurse she feels
creatures eating away at her skin. Which of the
following interventions should be done first?
a. Talk about his hallucinations and fears
b. Refer him for anticholinergic adverse
reactions
c. Assess for possible physical problems
such as rash
d. Call his physician to get his medication
increased to control his psychosis

97. Ivy, who is on the psychiatric unit is copying and


imitating the movements of her primary nurse.
During recovery, she says, “I thought the nurse
was my mirror. I felt connected only when I saw
my nurse.” This behavior is known by which of
the following terms?
a. Modeling
b. Echopraxia
c. Ego-syntonicity
d. Ritualism

98. Jun approaches the nurse and tells that he hears


a voice telling him that he’s evil and deserves to
die. Which of the following terms describes the
client’s perception?
a. Delusion
b. Disorganized speech
c. Hallucination
d. Idea of reference

99. Mike is admitted to a psychiatric unit with a


diagnosis of undifferentiated schizophrenia.
Which of the following defense mechanisms is
probably used by mike?
a. Projection
b. Rationalization
c. Regression
d. Repression

34
34
Answers and Rationale – Care of Clients with Rationale: This behavior is an example of
Physiologic and Psychosocial Alterations reaction formation, a coping mechanism.
11. Answer: (A) By designating times during which
1. Answer: (D) Focusing the client can focus on the behavior.
Rationale: The nurse is using focusing by Rationale: The nurse should designate times
suggesting that the client discuss a specific issue. during which the client can focus on the
The nurse didn’t restate the question, make compulsive behavior or obsessive thoughts. The
observation, or ask further question (exploring). nurse should urge the client to reduce the
2. Answer: (D) Remove all other clients from the frequency of the compulsive behavior gradually,
dayroom. not rapidly. She shouldn't call attention to or try
Rationale: The nurse’s first priority is to consider to prevent the behavior. Trying to prevent the
the safety of the clients in the therapeutic behavior may cause pain and terror in the client.
setting. The other actions are appropriate The nurse should encourage the client to
responses after ensuring the safety of other verbalize anxieties to help distract attention
clients. from the compulsive behavior.
3. Answer: (A) The client is disruptive. 12. Answer: (D) Exploring the meaning of the
Rationale: Group activity provides too much traumatic event with the client.
stimulation, which the client will not be able to Rationale: The client with PTSD needs
handle (harmful to self) and as a result will be encouragement to examine and understand the
disruptive to others. meaning of the traumatic event and consequent
4. Answer: (C) Agree to talk with the mother and losses. Otherwise, symptoms may worsen and
the father together. the client may become depressed or engage in
Rationale: By agreeing to talk with both parents, self-destructive behavior such as substance
the nurse can provide emotional support and abuse. The client must explore the meaning of
further assess and validate the family’s needs. the event and won't heal without this, no matter
5. Answer: (A) Perceptual disorders. how much time passes. Behavioral techniques,
Rationale: Frightening visual hallucinations are such as relaxation therapy, may help decrease
especially common in clients experiencing the client's anxiety and induce sleep. The
alcohol withdrawal. physician may prescribe antianxiety agents or
6. Answer: (D) Suggest that it takes a while before antidepressants cautiously to avoid dependence;
seeing the results. sleep medication is rarely appropriate. A special
Rationale: The client needs a specific response; diet isn't indicated unless the client also has an
that it takes 2 to 3 weeks (a delayed effect) until eating disorder or a nutritional problem.
the therapeutic blood level is reached. 13. Answer: (C) "Your problem is real but there is no
7. Answer: (C) Superego physical basis for it. We'll work on what is going
Rationale: This behavior shows a weak sense of on in your life to find out why it's happened."
moral consciousness. According to Freudian Rationale: The nurse must be honest with the
theory, personality disorders stem from a weak client by telling her that the paralysis has no
superego. physiologic cause while also conveying empathy
8. Answer: (C) Skeletal muscle paralysis. and acknowledging that her symptoms are real.
Rationale: Anectine is a depolarizing muscle The client will benefit from psychiatric
relaxant causing paralysis. It is used to reduce treatment, which will help her understand the
the intensity of muscle contractions during the underlying cause of her symptoms. After the
convulsive stage, thereby reducing the risk of psychological conflict is resolved, her symptoms
bone fractures or dislocation. will disappear. Saying that it must be awful not
9. Answer: (D) Increase calories, carbohydrates, to be able to move her legs wouldn't answer the
and protein. client's question; knowing that the cause is
Rationale: This client increased protein for tissue psychological wouldn't necessarily make her feel
building and increased calories to replace what is better. Telling her that she has developed
burned up (usually via carbohydrates). paralysis to avoid leaving her parents or that her
10. Answer: (C) Acting overly solicitous toward the personality caused her disorder wouldn't help
child. her understand and resolve the underlying
conflict.
14. Answer: (C) fluvoxamine (Luvox) and Rationale: Clients in the first stage of Alzheimer's
clomipramine (Anafranil) disease are aware that something is happening
Rationale: The antidepressants fluvoxamine and to them and may become overwhelmed and
clomipramine have been effective in the frightened. Therefore, nursing care typically
treatment of OCD. Librium and Valium may be focuses on providing emotional support and
helpful in treating anxiety related to OCD but individual counseling. The other options are
aren't drugs of choice to treat the illness. The appropriate during the second stage of
other medications mentioned aren't effective in Alzheimer's disease, when the client needs
the treatment of OCD. continuous monitoring to prevent minor
15. Answer: (A) A warning about the drugs delayed illnesses from progressing into major problems
therapeutic effect, which is from 14 to 30 days. and when maintaining adequate nutrition may
Rationale: The client should be informed that become a challenge. During this stage, offering
the drug's therapeutic effect might not be nourishing finger foods helps clients to feed
reached for 14 to 30 days. The client must be themselves and maintain adequate nutrition.
instructed to continue taking the drug as 20. Answer: (C) Emotional lability, euphoria, and
directed. Blood level checks aren't necessary. impaired memory
NMS hasn't been reported with this drug, but Rationale: Signs of antianxiety agent overdose
tachycardia is frequently reported. include emotional lability, euphoria, and
16. Answer: (B) Severe anxiety and fear. impaired memory. Phencyclidine overdose can
Rationale: Phobias cause severe anxiety (such as cause combativeness, sweating, and confusion.
a panic attack) that is out of proportion to the Amphetamine overdose can result in agitation,
threat of the feared object or situation. Physical hyperactivity, and grandiose ideation.
signs and symptoms of phobias include profuse Hallucinogen overdose can produce
sweating, poor motor control, tachycardia, and suspiciousness, dilated pupils, and increased
elevated blood pressure. Insomnia, an inability blood pressure.
to concentrate, and weight loss are common in 21. Answer: (D) A low tolerance for frustration
depression. Withdrawal and failure to Rationale: Clients with an antisocial personality
distinguish reality from fantasy occur in disorder exhibit a low tolerance for frustration,
schizophrenia. emotional immaturity, and a lack of impulse
17. Answer: (A) Antidepressants control. They commonly have a history of
Rationale: Tricyclic and monoamine oxidase unemployment, miss work repeatedly, and quit
(MAO) inhibitor antidepressants have been work without other plans for employment. They
found to be effective in treating clients with don't feel guilt about their behavior and
panic attacks. Why these drugs help control commonly perceive themselves as victims. They
panic attacks isn't clearly understood. also display a lack of responsibility for the
Anticholinergic agents, which are smooth- outcome of their actions. Because of a lack of
muscle relaxants, relieve physical symptoms of trust in others, clients with antisocial personality
anxiety but don't relieve the anxiety itself. disorder commonly have difficulty developing
Antipsychotic drugs are inappropriate because stable, close relationships.
clients who experience panic attacks aren't 22. Answer: (C) Methadone
psychotic. Mood stabilizers aren't indicated Rationale: Methadone is used to detoxify opiate
because panic attacks are rarely associated with users because it binds with opioid receptors at
mood changes. many sites in the central nervous system but
18. Answer: (B) 3 to 5 days doesn’t have the same deterious effects as other
Rationale: Monoamine oxidase inhibitors, such opiates, such as cocaine, heroin, and morphine.
as tranylcypromine, have an onset of action of Barbiturates, amphetamines, and
approximately 3 to 5 days. A full clinical benzodiazepines are highly addictive and would
response may be delayed for 3 to 4 weeks. The require detoxification treatment.
therapeutic effects may continue for 1 to 2 23. Answer: (B) Hallucinations
weeks after discontinuation. Rationale: Hallucinations are visual, auditory,
19. Answer: (B) Providing emotional support and gustatory, tactile, or olfactory perceptions that
individual counseling. have no basis in reality. Delusions are false
beliefs, rather than perceptions, that the client

34
34
accepts as real. Loose associations are rapid behavior is uncommon, although these clients
shifts among unrelated ideas. Neologisms are may experience agitation with anxiety. Their
bizarre words that have meaning only to the behavior is emotionally cold with a flattened
client. affect, regardless of the situation. These clients
24. Answer: (C) Set up a strict eating plan for the demonstrate a reduced capacity for close or
client. dependent relationships.
Rationale: Establishing a consistent eating plan 29. Answer: (C) Identify anxiety-causing situations
and monitoring the client’s weight are very Rationale: Bulimic behavior is generally a
important in this disorder. The family and friends maladaptive coping response to stress and
should be included in the client’s care. The client underlying issues. The client must identify
should be monitored during meals-not given anxiety-causing situations that stimulate the
privacy. Exercise must be limited and supervised. bulimic behavior and then learn new ways of
25. Answer: (A) Highly important or famous. coping with the anxiety.
Rationale: A delusion of grandeur is a false belief 30. Answer: (A) Tension and irritability
that one is highly important or famous. A Rationale: An amphetamine is a nervous system
delusion of persecution is a false belief that one stimulant that is subject to abuse because of its
is being persecuted. A delusion of reference is a ability to produce wakefulness and euphoria. An
false belief that one is connected to events overdose increases tension and irritability.
unrelated to oneself or a belief that one is Options B and C are incorrect because
responsible for the evil in the world. amphetamines stimulate norepinephrine, which
26. Answer: (D) Listening attentively with a neutral increase the heart rate and blood flow. Diarrhea
attitude and avoiding power struggles. is a common adverse effect so option D is
Rationale: The nurse should listen to the client’s incorrect.
requests, express willingness to seriously 31. Answer: (B) “No, I do not hear your voices, but I
consider the request, and respond later. The believe you can hear them”.
nurse should encourage the client to take short Rationale: The nurse, demonstrating knowledge
daytime naps because he expends so much and understanding, accepts the client’s
energy. The nurse shouldn’t try to restrain the perceptions even though they are hallucinatory.
client when he feels the need to move around as 32. Answer: (C) Confusion for a time after treatment
long as his activity isn’t harmful. High calorie Rationale: The electrical energy passing through
finger foods should be offered to supplement the cerebral cortex during ECT results in a
the client’s diet, if he can’t remain seated long temporary state of confusion after treatment.
enough to eat a complete meal. The nurse 33. Answer: (D) Acceptance stage
shouldn’t be forced to stay seated at the table to Rationale: Communication and intervention
finid=sh a meal. The nurse should set limits in a during this stage are mainly nonverbal, as when
calm, clear, and self-confident tone of voice. the client gestures to hold the nurse’s hand.
27. Answer: (D) Denial 34. Answer: (D) A higher level of anxiety continuing
Rationale: Denial is unconscious defense for more than 3 months.
mechanism in which emotional conflict and Rationale: This is not an expected outcome of a
anxiety is avoided by refusing to acknowledge crisis because by definition a crisis would be
feelings, desires, impulses, or external facts that resolved in 6 weeks.
are consciously intolerable. Withdrawal is a 35. Answer: (B) Staying in the sun
common response to stress, characterized by Rationale: Haldol causes photosensitivity. Severe
apathy. Logical thinking is the ability to think sunburn can occur on exposure to the sun.
rationally and make responsible decisions, which 36. Answer: (D) Moderate-level anxiety
would lead the client admitting the problem and Rationale: A moderately anxious person can
seeking help. Repression is suppressing past ignore peripheral events and focuses on central
events from the consciousness because of guilty concerns.
association. 37. Answer: (C) Diverse interest
28. Answer: (B) Paranoid thoughts Rationale: Before onset of depression, these
Rationale: Clients with schizotypal personality clients usually have very narrow, limited
disorder experience excessive social anxiety that interest.
can lead to paranoid thoughts. Aggressive
38. Answer: (A) As their depression begins to option A. Option B is incorrect because it implies
improve that the client’s actions reflect feelings toward
Rationale: At this point the client may have the staff instead of the client's own misery.
enough energy to plan and execute an attempt. Judgmental remarks, such as option D, may
39. Answer: (D) Disturbance in recalling recent decrease the client's self-esteem.
events related to cerebral hypoxia. 48. Answer: (C) lithium carbonate (Lithane)
Rationale: Cell damage seems to interfere with Rationale: Lithium carbonate, an antimania drug,
registering input stimuli, which affects the ability is used to treat clients with cyclical
to register and recall recent events; vascular schizoaffective disorder, a psychotic disorder
dementia is related to multiple vascular lesions once classified under schizophrenia that causes
of the cerebral cortex and subcortical structure. affective symptoms, including maniclike activity.
40. Answer: (D) Encouraging the client to have blood Lithium helps control the affective component of
levels checked as ordered. this disorder. Phenelzine is a monoamine
Rationale: Blood levels must be checked monthly oxidase inhibitor prescribed for clients who don't
or bimonthly when the client is on maintenance respond to other antidepressant drugs such as
therapy because there is only a small range imipramine. Chlordiazepoxide, an antianxiety
between therapeutic and toxic levels. agent, generally is contraindicated in psychotic
41. Answer: (B) Fine hand tremors or slurred speech clients. Imipramine, primarily considered an
Rationale: These are common side effects of antidepressant agent, is also used to treat clients
lithium carbonate. with agoraphobia and that undergoing cocaine
42. Answer: (D) Presence detoxification.
Rationale: The constant presence of a nurse 49. Answer: (B) Report a sore throat or fever to the
provides emotional support because the client physician immediately.
knows that someone is attentive and available in Rationale: A sore throat and fever are
case of an emergency. indications of an infection caused by
43. Answer: (A) Client’s perception of the presenting agranulocytosis, a potentially life-threatening
problem. complication of clozapine. Because of the risk of
Rationale: The nurse can be most therapeutic by agranulocytosis, white blood cell (WBC) counts
starting where the client is, because it is the are necessary weekly, not monthly. If the WBC
client’s concept of the problem that serves as count drops below 3,000/μl, the medication
the starting point of the relationship. must be stopped. Hypotension may occur in
44. Answer: (B) Chocolate milk, aged cheese, and clients taking this medication. Warn the client to
yogurt’” stand up slowly to avoid dizziness from
Rationale: These high-tyramine foods, when orthostatic hypotension. The medication should
ingested in the presence of an MAO inhibitor, be continued, even when symptoms have been
cause a severe hypertensive response. controlled. If the medication must be stopped, it
45. Answer: (B) 4 to 6 weeks should be slowly tapered over 1 to 2 weeks and
Rationale: Crisis is self-limiting and lasts from 4 only under the supervision of a physician.
to 6 weeks. 50. Answer: (C) Neuroleptic malignant syndrome.
46. Answer: (D) Males are more likely to use lethal Rationale: The client's signs and symptoms
methods than are females suggest neuroleptic malignant syndrome, a life-
Rationale: This finding is supported by research; threatening reaction to neuroleptic medication
females account for 90% of suicide attempts but that requires immediate treatment. Tardive
males are three times more successful because dyskinesia causes involuntary movements of the
of methods used. tongue, mouth, facial muscles, and arm and leg
47. Answer: (C) "Your cursing is interrupting the muscles. Dystonia is characterized by cramps
activity. Take time out in your room for 10 and rigidity of the tongue, face, neck, and back
minutes." muscles. Akathisia causes restlessness, anxiety,
Rationale: The nurse should set limits on client and jitteriness.
behavior to ensure a comfortable environment 51. Answer: (B) Advising the client to sit up for 1
for all clients. The nurse should accept hostile or minute before getting out of bed.
quarrelsome client outbursts within limits Rationale: To minimize the effects of
without becoming personally offended, as in amitriptyline-induced orthostatic hypotension,

34
34
the nurse should advise the client to sit up for 1 functions but sodium is most important to the
minute before getting out of bed. Orthostatic absorption of lithium.
hypotension commonly occurs with tricyclic 56. Answer: (D) It's characterized by an acute onset
antidepressant therapy. In these cases, the and lasts hours to a number of days
dosage may be reduced or the physician may Rationale: Delirium has an acute onset and
prescribe nortriptyline, another tricyclic typically can last from several hours to several
antidepressant. Orthostatic hypotension days.
disappears only when the drug is discontinued. 57. Answer: (B) Impaired communication.
52. Answer: (D) Dysthymic disorder. Rationale: Initially, memory impairment may be
Rationale: Dysthymic disorder is marked by the only cognitive deficit in a client with
feelings of depression lasting at least 2 years, Alzheimer's disease. During the early stage of
accompanied by at least two of the following this disease, subtle personality changes may also
symptoms: sleep disturbance, appetite be present. However, other than occasional
disturbance, low energy or fatigue, low self- irritable outbursts and lack of spontaneity, the
esteem, poor concentration, difficulty making client is usually cooperative and exhibits socially
decisions, and hopelessness. These symptoms appropriate behavior. Signs of advancement to
may be relatively continuous or separated by the middle stage of Alzheimer's disease include
intervening periods of normal mood that last a exacerbated cognitive impairment with obvious
few days to a few weeks. Cyclothymic disorder is personality changes and impaired
a chronic mood disturbance of at least 2 years' communication, such as inappropriate
duration marked by numerous periods of conversation, actions, and responses. During the
depression and hypomania. Atypical affective late stage, the client can't perform self-care
disorder is characterized by manic signs and activities and may become mute.
symptoms. Major depression is a recurring, 58. Answer: (D) This medication may initially cause
persistent sadness or loss of interest or pleasure tiredness, which should become less
in almost all activities, with signs and symptoms bothersome over time.
recurring for at least 2 weeks. Rationale: Sedation is a common early adverse
53. Answer: (C) 30 g mixed in 250 ml of water effect of imipramine, a tricyclic antidepressant,
Rationale: The usual adult dosage of activated and usually decreases as tolerance develops.
charcoal is 5 to 10 times the estimated weight of Antidepressants aren't habit forming and don't
the drug or chemical ingested, or a minimum cause physical or psychological dependence.
dose of 30 g, mixed in 250 ml of water. Doses However, after a long course of high-dose
less than this will be ineffective; doses greater therapy, the dosage should be decreased
than this can increase the risk of adverse gradually to avoid mild withdrawal symptoms.
reactions, although toxicity doesn't occur with Serious adverse effects, although rare, include
activated charcoal, even at the maximum dose. myocardial infarction, heart failure, and
54. Answer: (C) St. John's wort tachycardia. Dietary restrictions, such as
Rationale: St. John's wort has been found to avoiding aged cheeses, yogurt, and chicken
have serotonin-elevating properties, similar to livers, are necessary for a client taking a
prescription antidepressants. Ginkgo biloba is monoamine oxidase inhibitor, not a tricyclic
prescribed to enhance mental acuity. Echinacea antidepressant.
has immune-stimulating properties. Ephedra is a 59. Answer: (C) Monitor vital signs, serum
naturally occurring stimulant that is similar to electrolyte levels, and acid-base balance.
ephedrine. Rationale: An anorexic client who requires
55. Answer: (B) Sodium hospitalization is in poor physical condition from
Rationale: Lithium is chemically similar to starvation and may die as a result of
sodium. If sodium levels are reduced, such as arrhythmias, hypothermia, malnutrition,
from sweating or diuresis, lithium will be infection, or cardiac abnormalities secondary to
reabsorbed by the kidneys, increasing the risk of electrolyte imbalances. Therefore, monitoring
toxicity. Clients taking lithium shouldn't restrict the client's vital signs, serum electrolyte level,
their intake of sodium and should drink and acid base balance is crucial. Option A may
adequate amounts of fluid each day. The other worsen anxiety. Option B is incorrect because a
electrolytes are important for normal body weight obtained after breakfast is more accurate
than one obtained after the evening meal. 65. Answer: (A) increased attention span and
Option D would reward the client with attention concentration
for not eating and reinforce the control issues Rationale: The medication has a paradoxic effect
that are central to the underlying psychological that decreases hyperactivity and impulsivity
problem; also, the client may record food and among children with ADHD. B, C, D. Side effects
fluid intake inaccurately. of Ritalin include anorexia, insomnia, diarrhea
60. Answer: (D) Opioid withdrawal and irritability.
Rationale: The symptoms listed are specific to 66. Answer: (C) Moderate
opioid withdrawal. Alcohol withdrawal would Rationale: The child with moderate mental
show elevated vital signs. There is no real retardation has an I.Q. of 35- 50 Profound
withdrawal from cannibis. Symptoms of cocaine Mental retardation has an I.Q. of below 20; Mild
withdrawal include depression, anxiety, and mental retardation 50-70 and Severe mental
agitation. retardation has an I.Q. of 20-35.
61. Answer: (A) Regression 67. Answer: (D) Rearrange the environment to
Rationale: An adult who throws temper activate the child
tantrums, such as this one, is displaying Rationale: The child with autistic disorder does
regressive behavior, or behavior that is not want change. Maintaining a consistent
appropriate at a younger age. In projection, the environment is therapeutic. A. Angry outburst
client blames someone or something other than can be re-channeling through safe activities. B.
the source. In reaction formation, the client acts Acceptance enhances a trusting relationship. C.
in opposition to his feelings. In Ensure safety from self-destructive behaviors
intellectualization, the client overuses rational like head banging and hair pulling.
explanations or abstract thinking to decrease the 68. Answer: (B) cocaine
significance of a feeling or event. Rationale: The manifestations indicate
62. Answer: (A) Abnormal movements and intoxication with cocaine, a CNS stimulant. A.
involuntary movements of the mouth, tongue, Intoxication with heroine is manifested by
and face. euphoria then impairment in judgment,
Rationale: Tardive dyskinesia is a severe reaction attention and the presence of papillary
associated with long term use of antipsychotic constriction. C. Intoxication with hallucinogen
medication. The clinical manifestations include like LSD is manifested by grandiosity,
abnormal movements (dyskinesia) and hallucinations, synesthesia and increase in vital
involuntary movements of the mouth, tongue signs D. Intoxication with Marijuana, a
(fly catcher tongue), and face. cannabinoid is manifested by sensation of
63. Answer: (C) Blurred vision slowed time, conjunctival redness, social
Rationale: At lithium levels of 2 to 2.5 mEq/L the withdrawal, impaired judgment and
client will experienced blurred vision, muscle hallucinations.
twitching, severe hypotension, and persistent 69. Answer: (B) insidious onset
nausea and vomiting. With levels between 1.5 Rationale: Dementia has a gradual onset and
and 2 mEq/L the client experiencing vomiting, progressive deterioration. It causes pronounced
diarrhea, muscle weakness, ataxia, dizziness, memory and cognitive disturbances. A,C and D
slurred speech, and confusion. At lithium levels are all characteristics of delirium.
of 2.5 to 3 mEq/L or higher, urinary and fecal 70. Answer: (C) Claustrophobia
incontinence occurs, as well as seizures, cardiac Rationale: Claustrophobia is fear of closed space.
dysrythmias, peripheral vascular collapse, and A. Agoraphobia is fear of open space or being a
death. situation where escape is difficult. B. Social
64. Answer: (C) No acts of aggression have been phobia is fear of performing in the presence of
observed within 1 hour after the release of two others in a way that will be humiliating or
of the extremity restraints. embarrassing. D. Xenophobia is fear of
Rationale: The best indicator that the behavior is strangers.
controlled, if the client exhibits no signs of 71. Answer: (A) Revealing personal information to
aggression after partial release of restraints. the client
Options , B, and D do not ensure that the client Rationale: Counter-transference is an emotional
has controlled the behavior. reaction of the nurse on the client based on her

34
34
unconscious needs and conflicts. B and C. These Rationale: The DSM-IV-TR classifies major
are therapeutic approaches. D. This is depression as an Axis I disorder. Borderline
transference reaction where a client has an personality disorder as an Axis II; obesity and
emotional reaction towards the nurse based on hypertension, Axis III.
her past. 77. Answer: (B) Transference
72. Answer: (D) Hold the next dose and obtain an Rationale: Transference is the unconscious
order for a stat serum lithium level assignment of negative or positive feelings
Rationale: Diarrhea and vomiting are evoked by a significant person in the client’s past
manifestations of Lithium toxicity. The next dose to another person. Intellectualization is a
of lithium should be withheld and test is done to defense mechanism in which the client avoids
validate the observation. A. The manifestations dealing with emotions by focusing on facts.
are not due to drug interaction. B. Cogentin is Triangulation refers to conflicts involving three
used to manage the extra pyramidal symptom family members. Splitting is a defense
side effects of antipsychotics. C. The common mechanism commonly seen in clients with
side effects of Lithium are fine hand tremors, personality disorder in which the world is
nausea, polyuria and polydipsia. perceived as all good or all bad.
73. Answer: (C) A living, learning or working 78. Answer: (B) Hypochondriasis
environment. Rationale: Complains of vague physical
Rationale: A therapeutic milieu refers to a broad symptoms that have no apparent medical causes
conceptual approach in which all aspects of the are characteristic of clients with
environment are channeled to provide a hypochondriasis. In many cases, the GI system is
therapeutic environment for the client. The six affected. Conversion disorders are characterized
environmental elements include structure, by one or more neurologic symptoms. The
safety, norms; limit setting, balance and unit client’s symptoms don’t suggest severe anxiety.
modification. A. Behavioral approach in A client experiencing sublimation channels
psychiatric care is based on the premise that maladaptive feelings or impulses into socially
behavior can be learned or unlearned through acceptable behavior
the use of reward and punishment. B. Cognitive 79. Answer: (C) Hypochondriasis
approach to change behavior is done by Rationale: Hypochodriasis in this case is shown
correcting distorted perceptions and irrational by the client’s belief that she has a serious
beliefs to correct maladaptive behaviors. D. This illness, although pathologic causes have been
is not congruent with therapeutic milieu. eliminated. The disturbance usually lasts at least
74. Answer: (B) Transference 6 with identifiable life stressor such as, in this
Rationale: Transference is a positive or negative case, course examinations. Conversion disorders
feeling associated with a significant person in are characterized by one or more neurologic
the client’s past that are unconsciously assigned symptoms. Depersonalization refers to
to another A. Splitting is a defense mechanism persistent recurrent episodes of feeling
commonly seen in a client with personality detached from one’s self or body. Somatoform
disorder in which the world is perceived as all disorders generally have a chronic course with
good or all bad C. Countert-transference is a few remissions.
phenomenon where the nurse shifts feelings 80. Answer: (A) Triazolam (Halcion)
assigned to someone in her past to the patient Rationale: Triazolam is one of a group of
D. Resistance is the client’s refusal to submit sedative hypnotic medication that can be used
himself to the care of the nurse for a limited time because of the risk of
75. Answer: (B) Adventitious dependence. Paroxetine is a scrotonin-specific
Rationale: Adventitious crisis is a crisis involving reutake inhibitor used for treatment of
a traumatic event. It is not part of everyday life. depression panic disorder, and obsessive-
A. Situational crisis is from an external source compulsive disorder. Fluoxetine is a scrotonin-
that upset ones psychological equilibrium C and specific reuptake inhibitor used for depressive
D. are the same. They are transitional or disorders and obsessive-compulsive disorders.
developmental periods in life Risperidome is indicated for psychotic disorders.
76. Answer: (C) Major depression 81. Answer: (D) It promotes emotional support or
attention for the client
Rationale: Secondary gain refers to the benefits from moderate depression to hypomania.
of the illness that allow the client to receive Bipolar I disorder is characterized by a single
emotional support or attention. Primary gain manic episode with no past major depressive
enables the client to avoid some unpleasant episodes. Seasonal- affective disorder is a form
activity. A dysfunctional family may disregard of depression occurring in the fall and winter.
the real issue, although some conflict is relieved. 87. Answer: (A) Vascular dementia has more abrupt
Somatoform pain disorder is a preoccupation onset
with pain in the absence of physical disease. Rationale: Vascular dementia differs from
82. Answer: (A) “I went to the mall with my friends Alzheimer’s disease in that it has a more abrupt
last Saturday” onset and runs a highly variable course.
Rationale: Clients with panic disorder tent to be Personally change is common in Alzheimer’s
socially withdrawn. Going to the mall is a sign of disease. The duration of delirium is usually brief.
working on avoidance behaviors. The inability to carry out motor activities is
Hyperventilating is a key symptom of panic common in Alzheimer’s disease.
disorder. Teaching breathing control is a major 88. Answer: (C) Drug intoxication
intervention for clients with panic disorder. The Rationale: This client was taking several
client taking medications for panic disorder; such medications that have a propensity for
as tricylic antidepressants and benzodiazepines producing delirium; digoxin (a digitalis
must be weaned off these drugs. Most clients glycoxide), furosemide (a thiazide diuretic), and
with panic disorder with agoraphobia don’t have diazepam (a benzodiazepine). Sufficient
nutritional problems. supporting data don’t exist to suspect the other
83. Answer: (A) “I’m sleeping better and don’t have options as causes.
nightmares” 89. Answer: (D) The client is experiencing visual
Rationale: MAO inhibitors are used to treat sleep hallucination
problems, nightmares, and intrusive daytime Rationale: The presence of a sensory stimulus
thoughts in individual with posttraumatic stress correlates with the definition of a hallucination,
disorder. MAO inhibitors aren’t used to help which is a false sensory perception. Aphasia
control flashbacks or phobias or to decrease the refers to a communication problem. Dysarthria is
craving for alcohol. difficulty in speech production. Flight of ideas is
84. Answer: (D) Stopping the drug can cause rapid shifting from one topic to another.
withdrawal symptoms 90. Answer: (D) The client looks at the shadow on a
Rationale: Stopping antianxiety drugs such as wall and tells the nurse she sees frightening
benzodiazepines can cause the client to have faces on the wall.
withdrawal symptoms. Stopping a Rationale: Minor memory problems are
benzodiazepine doesn’t tend to cause distinguished from dementia by their minor
depression, increase cognitive abilities, or severity and their lack of significant interference
decrease sleeping difficulties. with the client’s social or occupational lifestyle.
85. Answer: (B) Behavioral difficulties Other options would be included in the history
Rationale: Adolescents tend to demonstrate data but don’t directly correlate with the client’s
severe irritability and behavioral problems lifestyle.
rather than simply a depressed mood. Anxiety 91. Answer: (D) Loose association
disorder is more commonly associated with Rationale: Loose associations are conversations
small children rather than with adolescents. that constantly shift in topic. Concrete thinking
Cognitive impairment is typically associated with implies highly definitive thought processes.
delirium or dementia. Labile mood is more Flight of ideas is characterized by conversation
characteristic of a client with cognitive that’s disorganized from the onset. Loose
impairment or bipolar disorder. associations don’t necessarily start in a cogently,
86. Answer: (D) It’s a mood disorder similar to major then becomes loose.
depression but of mild to moderate severity 92. Answer: (C) Paranoid
Rationale: Dysthymic disorder is a mood disorder Rationale: Because of their suspiciousness,
similar to major depression but it remains mild paranoid personalities ascribe malevolent
to moderate in severity. Cyclothymic disorder is activities to others and tent to be defensive,
a mood disorder characterized by a mood range becoming quarrelsome and argumentative.

35
35
Clients with antisocial personality disorder can 97. Answer: (B) Echopraxia
also be antagonistic and argumentative but are Rationale: Echopraxia is the copying of another’s
less suspicious than paranoid personalities. behaviors and is the result of the loss of ego
Clients with histrionic personality disorder are boundaries. Modeling is the conscious copying
dramatic, not suspicious and argumentative. of someone’s behaviors. Ego-syntonicity refers
Clients with schizoid personality disorder are to behaviors that correspond with the
usually detached from other and tend to have individual’s sense of self. Ritualism behaviors are
eccentric behavior. repetitive and compulsive.
93. Answer: (C) Explain that the drug is less affective 98. Answer: (C) Hallucination
if the client smokes Rationale: Hallucinations are sensory
Rationale: Olanzapine (Zyprexa) is less effective experiences that are misrepresentations of
for clients who smoke cigarettes. Serotonin reality or have no basis in reality. Delusions are
syndrome occurs with clients who take a beliefs not based in reality. Disorganized speech
combination of antidepressant medications. is characterized by jumping from one topic to
Olanzapine doesn’t cause euphoria, and the next or using unrelated words. An idea of
extrapyramidal adverse reactions aren’t a reference is a belief that an unrelated situation
problem. However, the client should be aware of holds special meaning for the client.
adverse effects such as tardive dyskinesia. 99. Answer: (C) Regression
94. Answer: (A) Lack of honesty Rationale: Regression, a return to earlier
Rationale: Clients with antisocial personality behavior to reduce anxiety, is the basic defense
disorder tent to engage in acts of dishonesty, mechanism in schizophrenia. Projection is a
shown by lying. Clients with schizotypal defense mechanism in which one blames others
personality disorder tend to be superstitious. and attempts to justify actions; it’s used
Clients with histrionic personality disorders tend primarily by people with paranoid schizophrenia
to overreact to frustrations and and delusional disorder. Rationalization is a
disappointments, have temper tantrums, and defense mechanism used to justify one’s action.
seek attention. Repression is the basic defense mechanism in
95. Answer: (A) “I’m not going to look just at the the neuroses; it’s an involuntary exclusion of
negative things about myself” painful thoughts, feelings, or experiences from
Rationale: As the client makes progress on awareness.
improving self-esteem, self- blame and negative 100. Answer: (A) Should report feelings of
self-evaluation will decrease. Clients with restlessness or agitation at once
dependent personality disorder tend to feel Rationale: Agitation and restlessness are adverse
fragile and inadequate and would be extremely effect of haloperidol and can be treated with
unlikely to discuss their level of competence and antocholinergic drugs. Haloperidol isn’t likely to
progress. These clients focus on self and aren’t cause photosensitivity or control essential
envious or jealous. Individuals with dependent hypertension. Although the client may
personality disorders don’t take over situations experience increased concentration and activity,
because they see themselves as inept and these effects are due to a decreased in
inadequate. symptoms, not the drug itself.
96. Answer: (C) Assess for possible physical
problems such as rash
Rationale: Clients with schizophrenia generally
have poor visceral recognition because they live
so fully in their fantasy world. They need to have
as in-depth assessment of physical complaints
that may spill over into their delusional
symptoms. Talking with the client won’t provide
as assessment of his itching, and itching isn’t as
adverse reaction of antipsychotic drugs, calling
the physician to get the client’s medication
increased doesn’t address his physical
complaints.
PART III d. Colostomy irrigation

7. Sterile technique is used whenever:


PRACTICE TEST I FOUNDATION OF NURSING a. Strict isolation is required
b. Terminal disinfection is performed
1. Which element in the circular chain of infection c. Invasive procedures are performed
can be eliminated by preserving skin integrity? d. Protective isolation is necessary
a. Host
b. Reservoir 8. Which of the following constitutes a break in
c. Mode of transmission sterile technique while preparing a sterile field
d. Portal of entry for a dressing change?
a. Using sterile forceps, rather than sterile
2. Which of the following will probably result in a gloves, to handle a sterile item
break in sterile technique for respiratory b. Touching the outside wrapper of
isolation? sterilized material without sterile gloves
a. Opening the patient’s window to the c. Placing a sterile object on the edge of
outside environment the sterile field
b. Turning on the patient’s room ventilator d. Pouring out a small amount of solution
c. Opening the door of the patient’s room (15 to 30 ml) before pouring the solution
leading into the hospital corridor into a sterile container
d. Failing to wear gloves when 9. A natural body defense that plays an active role
administering a bed bath in preventing infection is:
a. Yawning
3. Which of the following patients is at greater risk b. Body hair
for contracting an infection? c. Hiccupping
a. A patient with leukopenia d. Rapid eye movements
b. A patient receiving broad-spectrum
antibiotics 10. All of the following statement are true about
c. A postoperative patient who has donning sterile gloves except:
undergone orthopedic surgery a. The first glove should be picked up by
d. A newly diagnosed diabetic patient grasping the inside of the cuff.
b. The second glove should be picked up by
4. Effective hand washing requires the use of: inserting the gloved fingers under the
a. Soap or detergent to promote cuff outside the glove.
emulsification c. The gloves should be adjusted by sliding
b. Hot water to destroy bacteria the gloved fingers under the sterile cuff
c. A disinfectant to increase surface and pulling the glove over the wrist
tension d. The inside of the glove is considered
d. All of the above sterile

5. After routine patient contact, hand washing 11. When removing a contaminated gown, the nurse
should last at least: should be careful that the first thing she touches
a. 30 seconds is the:
b. 1 minute a. Waist tie and neck tie at the back of the
c. 2 minute gown
d. 3 minutes b. Waist tie in front of the gown
c. Cuffs of the gown
6. Which of the following procedures always d. Inside of the gown
requires surgical asepsis?
a. Vaginal instillation of conjugated 12. Which of the following nursing interventions is
estrogen considered the most effective form or universal
b. Urinary catheterization precautions?
c. Nasogastric tube insertion

35
35
a. Cap all used needles before removing b. Before the procedure, the patient should
them from their syringes remove all jewelry, metallic objects, and
b. Discard all used uncapped needles and buttons above the waist
syringes in an impenetrable protective c. A signed consent is not required
container d. Eating, drinking, and medications are
c. Wear gloves when administering IM allowed before this test
injections
d. Follow enteric precautions 19. The most appropriate time for the nurse to
obtain a sputum specimen for culture is:
13. All of the following measures are recommended a. Early in the morning
to prevent pressure ulcers except: b. After the patient eats a light breakfast
a. Massaging the reddened are with lotion c. After aerosol therapy
b. Using a water or air mattress d. After chest physiotherapy
c. Adhering to a schedule for positioning
and turning 20. A patient with no known allergies is to receive
d. Providing meticulous skin care penicillin every 6 hours. When administering the
medication, the nurse observes a fine rash on
14. Which of the following blood tests should be the patient’s skin. The most appropriate nursing
performed before a blood transfusion? action would be to:
a. Prothrombin and coagulation time a. Withhold the moderation and notify the
b. Blood typing and cross-matching physician
c. Bleeding and clotting time b. Administer the medication and notify
d. Complete blood count (CBC) and the physician
electrolyte levels. c. Administer the medication with an
antihistamine
15. The primary purpose of a platelet count is to d. Apply corn starch soaks to the rash
evaluate the:
a. Potential for clot formation 21. All of the following nursing interventions are
b. Potential for bleeding correct when using the Z- track method of drug
c. Presence of an antigen-antibody injection except:
response a. Prepare the injection site with alcohol
d. Presence of cardiac enzymes b. Use a needle that’s a least 1” long
c. Aspirate for blood before injection
16. Which of the following white blood cell (WBC) d. Rub the site vigorously after the
counts clearly indicates leukocytosis? injection to promote absorption
a. 4,500/mm³
b. 7,000/mm³ 22. The correct method for determining the vastus
c. 10,000/mm³ lateralis site for I.M. injection is to:
d. 25,000/mm³ a. Locate the upper aspect of the upper
outer quadrant of the buttock about 5 to
17. After 5 days of diuretic therapy with 20mg of 8 cm below the iliac crest
furosemide (Lasix) daily, a patient begins to b. Palpate the lower edge of the acromion
exhibit fatigue, muscle cramping and muscle process and the midpoint lateral aspect
weakness. These symptoms probably indicate of the arm
that the patient is experiencing: c. Palpate a 1” circular area anterior to the
a. Hypokalemia umbilicus
b. Hyperkalemia d. Divide the area between the greater
c. Anorexia femoral trochanter and the lateral
d. Dysphagia femoral condyle into thirds, and select
the middle third on the anterior of the
18. Which of the following statements about chest thigh
X-ray is false?
a. No contradictions exist for this test
23. The mid-deltoid injection site is seldom used for a. Fever
I.M. injections because it: b. Chronic Obstructive Pulmonary Disease
a. Can accommodate only 1 ml or less of c. Renal Failure
medication d. Dehydration
b. Bruises too easily
c. Can be used only when the patient is 31. All of the following are common signs and
lying down symptoms of phlebitis except:
d. Does not readily parenteral medication a. Pain or discomfort at the IV insertion site
b. Edema and warmth at the IV insertion
24. The appropriate needle size for insulin injection site
is: c. A red streak exiting the IV insertion site
a. 18G, 1 ½” long d. Frank bleeding at the insertion site
b. 22G, 1” long
c. 22G, 1 ½” long 32. The best way of determining whether a patient
d. 25G, 5/8” long has learned to instill ear medication properly is
for the nurse to:
25. The appropriate needle gauge for intradermal a. Ask the patient if he/she has used ear
injection is: drops before
a. 20G b. Have the patient repeat the nurse’s
b. 22G instructions using her own words
c. 25G c. Demonstrate the procedure to the
d. 26G patient and encourage to ask questions
d. Ask the patient to demonstrate the
26. Parenteral penicillin can be administered as an: procedure
a. IM injection or an IV solution
b. IV or an intradermal injection 33. Which of the following types of medications can
c. Intradermal or subcutaneous injection be administered via gastrostomy tube?
d. IM or a subcutaneous injection a. Any oral medications
b. Capsules whole contents are dissolve in
27. The physician orders gr 10 of aspirin for a water
patient. The equivalent dose in milligrams is: c. Enteric-coated tablets that are
a. 0.6 mg thoroughly dissolved in water
b. 10 mg d. Most tablets designed for oral use,
c. 60 mg except for extended-duration
d. 600 mg compounds

28. The physician orders an IV solution of dextrose 34. A patient who develops hives after receiving an
5% in water at 100ml/hour. What would the antibiotic is exhibiting drug:
flow rate be if the drop factor is 15 gtt = 1 ml? a. Tolerance
a. 5 gtt/minute b. Idiosyncrasy
b. 13 gtt/minute c. Synergism
c. 25 gtt/minute d. Allergy
d. 50 gtt/minute
35. A patient has returned to his room after femoral
29. Which of the following is a sign or symptom of a arteriography. All of the following are
hemolytic reaction to blood transfusion? appropriate nursing interventions except:
a. Hemoglobinuria a. Assess femoral, popliteal, and pedal
b. Chest pain pulses every 15 minutes for 2 hours
c. Urticaria b. Check the pressure dressing for
d. Distended neck veins sanguineous drainage
c. Assess vital signs every 15 minutes for 2
30. Which of the following conditions may require hours
fluid restriction?

35
35
d. Order a hemoglobin and hematocrit 42. All of the following are good sources of vitamin A
count 1 hour after the arteriography except:
a. White potatoes
36. The nurse explains to a patient that a cough: b. Carrots
a. Is a protective response to clear the c. Apricots
respiratory tract of irritants d. Egg yolks
b. Is primarily a voluntary action
c. Is induced by the administration of an 43. Which of the following is a primary nursing
antitussive drug intervention necessary for all patients with a
d. Can be inhibited by “splinting” the Foley Catheter in place?
abdomen a. Maintain the drainage tubing and
collection bag level with the patient’s
37. An infected patient has chills and begins bladder
shivering. The best nursing intervention is to: b. Irrigate the patient with 1% Neosporin
a. Apply iced alcohol sponges solution three times a daily
b. Provide increased cool liquids c. Clamp the catheter for 1 hour every 4
c. Provide additional bedclothes hours to maintain the bladder’s elasticity
d. Provide increased ventilation d. Maintain the drainage tubing and
collection bag below bladder level to
38. A clinical nurse specialist is a nurse who has: facilitate drainage by gravity
a. Been certified by the National League for
Nursing 44. The ELISA test is used to:
b. Received credentials from the Philippine a. Screen blood donors for antibodies to
Nurses’ Association human immunodeficiency virus (HIV)
c. Graduated from an associate degree b. Test blood to be used for transfusion for
program and is a registered professional HIV antibodies
nurse c. Aid in diagnosing a patient with AIDS
d. Completed a master’s degree in the d. All of the above
prescribed clinical area and is a
registered professional nurse. 45. The two blood vessels most commonly used for
TPN infusion are the:
39. The purpose of increasing urine acidity through a. Subclavian and jugular veins
dietary means is to: b. Brachial and subclavian veins
a. Decrease burning sensations c. Femoral and subclavian veins
b. Change the urine’s color d. Brachial and femoral veins
c. Change the urine’s concentration
d. Inhibit the growth of microorganisms 46. Effective skin disinfection before a surgical
procedure includes which of the following
40. Clay colored stools indicate: methods?
a. Upper GI bleeding a. Shaving the site on the day
b. Impending constipation before surgery
c. An effect of medication b. Applying a topical antiseptic to the skin
d. Bile obstruction on the evening before surgery
c. Having the patient take a tub bath on
41. In which step of the nursing process would the the morning of surgery
nurse ask a patient if the medication she d. Having the patient shower with an
administered relieved his pain? antiseptic soap on the evening v=before
a. Assessment and the morning of surgery
b. Analysis
c. Planning 47. When transferring a patient from a bed to a
d. Evaluation chair, the nurse should use which muscles to
avoid back injury?
a. Abdominal muscles
b. Back muscles
c. Leg muscles
d. Upper arm muscles

48. Thrombophlebitis typically develops in patients


with which of the following conditions?
a. Increases partial thromboplastin time
b. Acute pulsus paradoxus
c. An impaired or traumatized blood vessel
wall
d. Chronic Obstructive Pulmonary Disease
(COPD)

49. In a recumbent, immobilized patient, lung


ventilation can become altered, leading to such
respiratory complications as:
a. Respiratory acidosis, ateclectasis, and
hypostatic pneumonia
b. Appneustic breathing, atypical
pneumonia and respiratory alkalosis
c. Cheyne-Strokes respirations and
spontaneous pneumothorax
d. Kussmail’s respirations and
hypoventilation

50. Immobility impairs bladder elimination, resulting


in such disorders as
a. Increased urine acidity and relaxation of
the perineal muscles, causing
incontinence
b. Urine retention, bladder distention, and
infection
c. Diuresis, natriuresis, and decreased
urine specific gravity
d. Decreased calcium and phosphate levels
in the urine

35
35
ANSWERS AND RATIONALE – FOUNDATION OF to prepare them for reuse by another patient.
NURSING The purpose of protective (reverse) isolation is
to prevent a person with seriously impaired
1. D. In the circular chain of infection, pathogens resistance from coming into contact who
must be able to leave their reservoir and be potentially pathogenic organisms.
transmitted to a susceptible host through a 8. C. The edges of a sterile field are considered
portal of entry, such as broken skin. contaminated. When sterile items are allowed to
2. C. Respiratory isolation, like strict isolation, come in contact with the edges of the field, the
requires that the door to the door patient’s sterile items also become contaminated.
room remain closed. However, the patient’s 9. B. Hair on or within body areas, such as the
room should be well ventilated, so opening the nose, traps and holds particles that contain
window or turning on the ventricular is microorganisms. Yawning and hiccupping do not
desirable. The nurse does not need to wear prevent microorganisms from entering or
gloves for respiratory isolation, but good hand leaving the body. Rapid eye movement marks
washing is important for all types of isolation. the stage of sleep during which dreaming occurs.
3. A. Leukopenia is a decreased number of 10. D. The inside of the glove is always considered to
leukocytes (white blood cells), which are be clean, but not sterile.
important in resisting infection. None of the 11. A. The back of the gown is considered clean, the
other situations would put the patient at risk for front is contaminated. So, after removing gloves
contracting an infection; taking broad- spectrum and washing hands, the nurse should untie the
antibiotics might actually reduce the infection back of the gown; slowly move backward away
risk. from the gown, holding the inside of the gown
4. A. Soaps and detergents are used to help and keeping the edges off the floor; turn and
remove bacteria because of their ability to lower fold the gown inside out; discard it in a
the surface tension of water and act as contaminated linen container; then wash her
emulsifying agents. Hot water may lead to skin hands again.
irritation or burns. 12. B. According to the Centers for Disease Control
5. A. Depending on the degree of exposure to (CDC), blood-to-blood contact occurs most
pathogens, hand washing may last from 10 commonly when a health care worker attempts
seconds to 4 minutes. After routine patient to cap a used needle. Therefore, used needles
contact, hand washing for 30 seconds effectively should never be recapped; instead they should
minimizes the risk of pathogen transmission. be inserted in a specially designed puncture
6. B. The urinary system is normally free of resistant, labeled container. Wearing gloves is
microorganisms except at the urinary meatus. not always necessary when administering an I.M.
Any procedure that involves entering this system injection. Enteric precautions prevent the
must use surgically aseptic measures to maintain transfer of pathogens via feces.
a bacteria-free state. 13. A. Nurses and other health care professionals
7. C. All invasive procedures, including surgery, previously believed that massaging a reddened
catheter insertion, and administration of area with lotion would promote venous return
parenteral therapy, require sterile technique to and reduce edema to the area. However,
maintain a sterile environment. All equipment research has shown that massage only increases
must be sterile, and the nurse and the physician the likelihood of cellular ischemia and necrosis
must wear sterile gloves and maintain surgical to the area.
asepsis. In the operating room, the nurse and 14. B. Before a blood transfusion is performed, the
physician are required to wear sterile gowns, blood of the donor and recipient must be
gloves, masks, hair covers, and shoe covers for checked for compatibility. This is done by blood
all invasive procedures. Strict isolation requires typing (a test that determines a person’s blood
the use of clean gloves, masks, gowns and type) and cross-matching (a procedure that
equipment to prevent the transmission of highly determines the compatibility of the donor’s and
communicable diseases by contact or by recipient’s blood after the blood types has been
airborne routes. Terminal disinfection is the matched). If the blood specimens are
disinfection of all contaminated supplies and incompatible, hemolysis and antigen-antibody
equipment after a patient has been discharged reactions will occur.
15. A. Platelets are disk-shaped cells that are 21. D. The Z-track method is an I.M. injection
essential for blood coagulation. A platelet count technique in which the patient’s skin is pulled in
determines the number of thrombocytes in such a way that the needle track is sealed off
blood available for promoting hemostasis and after the injection. This procedure seals
assisting with blood coagulation after injury. It medication deep into the muscle, thereby
also is used to evaluate the patient’s potential minimizing skin staining and irritation. Rubbing
for bleeding; however, this is not its primary the injection site is contraindicated because it
purpose. The normal count ranges from 150,000 may cause the medication to extravasate into
to 350,000/mm3. A count of 100,000/mm3 or the skin.
less indicates a potential for bleeding; count of 22. D. The vastus lateralis, a long, thick muscle that
less than 20,000/mm3 is associated with extends the full length of the thigh, is viewed by
spontaneous bleeding. many clinicians as the site of choice for I.M.
16. D. Leukocytosis is any transient increase in the injections because it has relatively few major
number of white blood cells (leukocytes) in the nerves and blood vessels. The middle third of the
blood. Normal WBC counts range from 5,000 to muscle is recommended as the injection site.
100,000/mm3. Thus, a count of 25,000/mm3 The patient can be in a supine or sitting position
indicates leukocytosis. for an injection into this site.
17. A. Fatigue, muscle cramping, and muscle 23. A. The mid-deltoid injection site can
weaknesses are symptoms of hypokalemia (an accommodate only 1 ml or less of medication
inadequate potassium level), which is a potential because of its size and location (on the deltoid
side effect of diuretic therapy. The physician muscle of the arm, close to the brachial artery
usually orders supplemental potassium to and radial nerve).
prevent hypokalemia in patients receiving 24. D. A 25G, 5/8” needle is the recommended size
diuretics. Anorexia is another symptom of for insulin injection because insulin is
hypokalemia. Dysphagia means difficulty administered by the subcutaneous route. An
swallowing. 18G, 1 ½” needle is usually used for I.M.
18. A. Pregnancy or suspected pregnancy is the only injections in children, typically in the vastus
contraindication for a chest X-ray. However, if a lateralis. A 22G, 1 ½” needle is usually used for
chest X-ray is necessary, the patient can wear a adult I.M. injections, which are typically
lead apron to protect the pelvic region from administered in the vastus lateralis or
radiation. Jewelry, metallic objects, and buttons ventrogluteal site.
would interfere with the X-ray and thus should 25. D. Because an intradermal injection does not
not be worn above the waist. A signed consent is penetrate deeply into the skin, a small-bore 25G
not required because a chest X-ray is not an needle is recommended. This type of injection is
invasive examination. Eating, drinking and used primarily to administer antigens to
medications are allowed because the X-ray is of evaluate reactions for allergy or sensitivity
the chest, not the abdominal region. studies. A 20G needle is usually used for I.M.
19. A. Obtaining a sputum specimen early in this injections of oil- based medications; a 22G
morning ensures an adequate supply of bacteria needle for I.M. injections; and a 25G needle, for
for culturing and decreases the risk of I.M. injections; and a 25G needle, for
contamination from food or medication. subcutaneous insulin injections.
20. A. Initial sensitivity to penicillin is commonly 26. A. Parenteral penicillin can be administered I.M.
manifested by a skin rash, even in individuals or added to a solution and given I.V. It cannot be
who have not been allergic to it previously. administered subcutaneously or intradermally.
Because of the danger of anaphylactic shock, he 27. D. gr 10 x 60mg/gr 1 = 600 mg
nurse should withhold the drug and notify the 28. C. 100ml/60 min X 15 gtt/ 1 ml = 25 gtt/minute
physician, who may choose to substitute 29. A. Hemoglobinuria, the abnormal presence of
another drug. Administering an antihistamine is hemoglobin in the urine, indicates a hemolytic
a dependent nursing intervention that requires a reaction (incompatibility of the donor’s and
written physician’s order. Although applying recipient’s blood). In this reaction, antibodies in
corn starch to the rash may relieve discomfort, it the recipient’s plasma combine rapidly with
is not the nurse’s top priority in such a donor RBC’s; the cells are hemolyzed in either
potentially life-threatening situation. circulatory or reticuloendothelial system.

35
35
Hemolysis occurs more rapidly in ABO 36. A. Coughing, a protective response that clears
incompatibilities than in Rh incompatibilities. the respiratory tract of irritants, usually is
Chest pain and urticarial may be symptoms of involuntary; however it can be voluntary, as
impending anaphylaxis. Distended neck veins are when a patient is taught to perform coughing
an indication of hypervolemia. exercises. An antitussive drug inhibits coughing.
30. C. In real failure, the kidney loses their ability to Splinting the abdomen supports the abdominal
effectively eliminate wastes and fluids. Because muscles when a patient coughs.
of this, limiting the patient’s intake of oral and 37. C. In an infected patient, shivering results from
I.V. fluids may be necessary. Fever, chronic the body’s attempt to increase heat production
obstructive pulmonary disease, and dehydration and the production of neutrophils and
are conditions for which fluids should be phagocytotic action through increased skeletal
encouraged. muscle tension and contractions. Initial
31. D. Phlebitis, the inflammation of a vein, can be vasoconstriction may cause skin to feel cold to
caused by chemical irritants (I.V. solutions or the touch. Applying additional bed clothes helps
medications), mechanical irritants (the needle or to equalize the body temperature and stop the
catheter used during venipuncture or chills. Attempts to cool the body result in further
cannulation), or a localized allergic reaction to shivering, increased metabloism, and thus
the needle or catheter. Signs and symptoms of increased heat production.
phlebitis include pain or discomfort, edema and 38. D. A clinical nurse specialist must have
heat at the I.V. insertion site, and a red streak completed a master’s degree in a clinical
going up the arm or leg from the I.V. insertion specialty and be a registered professional nurse.
site. The National League of Nursing accredits
32. D. Return demonstration provides the most educational programs in nursing and provides a
certain evidence for evaluating the effectiveness testing service to evaluate student nursing
of patient teaching. competence but it does not certify nurses. The
33. D. Capsules, enteric-coated tablets, and most American Nurses Association identifies
extended duration or sustained release products requirements for certification and offers
should not be dissolved for use in a gastrostomy examinations for certification in many areas of
tube. They are pharmaceutically manufactured nursing, such as medical surgical nursing. These
in these forms for valid reasons, and altering certification (credentialing) demonstrates that
them destroys their purpose. The nurse should the nurse has the knowledge and the ability to
seek an alternate physician’s order when an provide high quality nursing care in the area of
ordered medication is inappropriate for delivery her certification. A graduate of an associate
by tube. degree program is not a clinical nurse specialist:
34. D. A drug-allergy is an adverse reaction resulting however, she is prepared to provide bed side
from an immunologic response following a nursing with a high degree of knowledge and
previous sensitizing exposure to the drug. The skill. She must successfully complete the
reaction can range from a rash or hives to licensing examination to become a registered
anaphylactic shock. Tolerance to a drug means professional nurse.
that the patient experiences a decreasing 39. D. Microorganisms usually do not grow in an
physiologic response to repeated administration acidic environment.
of the drug in the same dosage. Idiosyncrasy is 40. D. Bile colors the stool brown. Any inflammation
an individual’s unique hypersensitivity to a drug, or obstruction that impairs bile flow will affect
food, or other substance; it appears to be the stool pigment, yielding light, clay-colored
genetically determined. Synergism, is a drug stool. Upper GI bleeding results in black or tarry
interaction in which the sum of the drug’s stool. Constipation is characterized by small,
combined effects is greater than that of their hard masses. Many medications and foods will
separate effects. discolor stool – for example, drugs containing
35. D. A hemoglobin and hematocrit count would be iron turn stool black.; beets turn stool red.
ordered by the physician if bleeding were 41. D. In the evaluation step of the nursing process,
suspected. The other answers are appropriate the nurse must decide whether the patient has
nursing interventions for a patient who has achieved the expected outcome that was
undergone femoral arteriography. identified in the planning phase.
42. A. The main sources of vitamin A are yellow and not necessarily impede venous return of injure
green vegetables (such as carrots, sweet vessel walls.
potatoes, squash, spinach, collard greens, 49. A. Because of restricted respiratory movement, a
broccoli, and cabbage) and yellow fruits (such as recumbent, immobilize patient is at particular
apricots, and cantaloupe). Animal sources risk for respiratory acidosis from poor gas
include liver, kidneys, cream, butter, and egg exchange; atelectasis from reduced surfactant
yolks. and accumulated mucus in the bronchioles, and
43. D. Maintaing the drainage tubing and collection hypostatic pneumonia from bacterial growth
bag level with the patient’s bladder could result caused by stasis of mucus secretions.
in reflux of urine into the kidney. Irrigating the 50. B. The immobilized patient commonly suffers
bladder with Neosporin and clamping the from urine retention caused by decreased
catheter for 1 hour every 4 hours must be muscle tone in the perineum. This leads to
prescribed by a physician. bladder distention and urine stagnation, which
44. D. The ELISA test of venous blood is used to provide an excellent medium for bacterial
assess blood and potential blood donors to growth leading to infection. Immobility also
human immunodeficiency virus (HIV). A positive results in more alkaline urine with excessive
ELISA test combined with various signs and amounts of calcium, sodium and phosphate, a
symptoms helps to diagnose acquired gradual decrease in urine production, and an
immunodeficiency syndrome (AIDS) increased specific gravity.
45. D. Tachypnea (an abnormally rapid rate of
breathing) would indicate that the patient was
still hypoxic (deficient in oxygen).The partial
pressures of arterial oxygen and carbon dioxide
listed are within the normal range. Eupnea refers
to normal respiration.
46. D. Studies have shown that showering with an
antiseptic soap before surgery is the most
effective method of removing microorganisms
from the skin. Shaving the site of the intended
surgery might cause breaks in the skin, thereby
increasing the risk of infection; however, if
indicated, shaving, should be done immediately
before surgery, not the day before. A topical
antiseptic would not remove microorganisms
and would be beneficial only after proper
cleaning and rinsing. Tub bathing might transfer
organisms to another body site rather than rinse
them away.
47. C. The leg muscles are the strongest muscles in
the body and should bear the greatest stress
when lifting. Muscles of the abdomen, back, and
upper arms may be easily injured.
48. C. The factors, known as Virchow’s triad,
collectively predispose a patient to
thromboplebitis; impaired venous return to the
heart, blood hypercoagulability, and injury to a
blood vessel wall. Increased partial
thromboplastin time indicates a prolonged
bleeding time during fibrin clot formation,
commonly the result of anticoagulant (heparin)
therapy. Arterial blood disorders (such as pulsus
paradoxus) and lung diseases (such as COPD) do

36
36
PRACTICE TEST II Maternal and Child Health
7. The client tells the nurse that her last menstrual
1. For the client who is using oral contraceptives, period started on January 14 and ended on
the nurse informs the client about the need to January 20. Using Nagele’s rule, the nurse
take the pill at the same time each day to determines her EDD to be which of the
accomplish which of the following? following?
a. Decrease the incidence of nausea a. September 27
b. Maintain hormonal levels b. October 21
c. Reduce side effects c. November 7
d. Prevent drug interactions d. December 27

2. When teaching a client about contraception. 8. When taking an obstetrical history on a pregnant
Which of the following would the nurse include client who states, “I had a son born at 38 weeks
as the most effective method for preventing gestation, a daughter born at 30 weeks gestation
sexually transmitted infections? and I lost a baby at about 8 weeks,” the nurse
a. Spermicides should record her obstetrical history as which of
b. Diaphragm the following?
c. Condoms a. G2 T2 P0 A0 L2
d. Vasectomy b. G3 T1 P1 A0 L2
c. G3 T2 P0 A0 L2
3. When preparing a woman who is 2 days d. G4 T1 P1 A1 L2
postpartum for discharge, recommendations for
which of the following contraceptive methods 9. When preparing to listen to the fetal heart rate
would be avoided? at 12 weeks’ gestation, the nurse would use
a. Diaphragm which of the following?
b. Female condom a. Stethoscope placed midline at the
c. Oral contraceptives umbilicus
d. Rhythm method b. Doppler placed midline at the
suprapubic region
4. For which of the following clients would the c. Fetoscope placed midway between the
nurse expect that an intrauterine device would umbilicus and the xiphoid process
not be recommended? d. External electronic fetal monitor placed
a. Woman over age 35 at the umbilicus
b. Nulliparous woman
c. Promiscuous young adult 10. When developing a plan of care for a client
d. Postpartum client newly diagnosed with gestational diabetes,
which of the following instructions would be the
5. A client in her third trimester tells the nurse, priority?
“I’m constipated all the time!” Which of the a. Dietary intake
following should the nurse recommend? b. Medication
a. Daily enemas c. Exercise
b. Laxatives d. Glucose monitoring
c. Increased fiber intake
d. Decreased fluid intake 11. A client at 24 weeks gestation has gained 6
pounds in 4 weeks. Which of the following would
6. Which of the following would the nurse use as be the priority when assessing the client?
the basis for the teaching plan when caring for a a. Glucosuria
pregnant teenager concerned about gaining too b. Depression
much weight during pregnancy? c. Hand/face edema
a. 10 pounds per trimester d. Dietary intake
b. 1 pound per week for 40 weeks
c. ½ pound per week for 40 weeks 12. A client 12 weeks’ pregnant come to the
d. A total gain of 25 to 30 pounds emergency department with abdominal
cramping and moderate vaginal bleeding. a. A dark red discharge on a 2-day
Speculum examination reveals 2 to 3 cms postpartum client
cervical dilation. The nurse would document b. A pink to brownish discharge on a client
these findings as which of the following? who is 5 days postpartum
a. Threatened abortion c. Almost colorless to creamy discharge on
b. Imminent abortion a client 2 weeks after delivery
c. Complete abortion d. A bright red discharge 5 days after
d. Missed abortion delivery

13. Which of the following would be the priority 18. A postpartum client has a temperature of
nursing diagnosis for a client with an ectopic 101.4ºF, with a uterus that is tender when
pregnancy? palpated, remains unusually large, and not
a. Risk for infection descending as normally expected. Which of the
b. Pain following should the nurse assess next?
c. Knowledge Deficit a. Lochia
d. Anticipatory Grieving b. Breasts
c. Incision
14. Before assessing the postpartum client’s uterus d. Urine
for firmness and position in relation to the
umbilicus and midline, which of the following 19. Which of the following is the priority focus of
should the nurse do first? nursing practice with the current early
a. Assess the vital signs postpartum discharge?
b. Administer analgesia a. Promoting comfort and restoration of
c. Ambulate her in the hall health
d. Assist her to urinate b. Exploring the emotional status of the
family
15. Which of the following should the nurse do c. Facilitating safe and effective self-and
when a primipara who is lactating tells the nurse newborn care
that she has sore nipples? d. Teaching about the importance of family
a. Tell her to breast feed more frequently planning
b. Administer a narcotic before breast
feeding 20. Which of the following actions would be least
c. Encourage her to wear a effective in maintaining a neutral thermal
nursing brassiere environment for the newborn?
d. Use soap and water to clean the nipples a. Placing infant under radiant warmer
after bathing
16. The nurse assesses the vital signs of a client, 4 b. Covering the scale with a warmed
hours’ postpartum that are as follows: BP 90/60; blanket prior to weighing
temperature 100.4ºF; pulse 100 weak, thready; c. Placing crib close to nursery window for
R 20 per minute. Which of the following should family viewing
the nurse do first? d. Covering the infant’s head with a knit
a. Report the temperature to the physician stockinette
b. Recheck the blood pressure with
another cuff 21. A newborn who has an asymmetrical Moro
c. Assess the uterus for firmness and reflex response should be further assessed for
position which of the following?
d. Determine the amount of lochia a. Talipes equinovarus
b. Fractured clavicle
17. The nurse assesses the postpartum vaginal c. Congenital hypothyroidism
discharge (lochia) on four clients. Which of the d. Increased intracranial pressure
following assessments would warrant
notification of the physician?

36
36
22. During the first 4 hours after a male b. 3 ounces
circumcision, assessing for which of the c. 4 ounces
following is the priority? d. 6 ounces
a. Infection
b. Hemorrhage 27. The postterm neonate with meconium-stained
c. Discomfort amniotic fluid needs care designed to especially
d. Dehydration monitor for which of the following?
a. Respiratory problems
23. The mother asks the nurse. “What’s wrong with b. Gastrointestinal problems
my son’s breasts? Why are they so enlarged?” c. Integumentary problems
Whish of the following would be the best d. Elimination problems
response by the nurse?
a. “The breast tissue is inflamed from the 28. When measuring a client’s fundal height, which
trauma experienced with birth” of the following techniques denotes the correct
b. “A decrease in material hormones method of measurement used by the nurse?
present before birth causes a. From the xiphoid process to the
enlargement,” umbilicus
c. “You should discuss this with your b. From the symphysis pubis to the xiphoid
doctor. It could be a malignancy” process
d. “The tissue has hypertrophied while the c. From the symphysis pubis to the fundus
baby was in the uterus” d. From the fundus to the umbilicus

24. Immediately after birth the nurse notes the 29. A client with severe preeclampsia is admitted
following on a male newborn: respirations 78; with of BP 160/110, proteinuria, and severe
apical hearth rate 160 BPM, nostril flaring; mild pitting edema. Which of the following would be
intercostal retractions; and grunting at the end most important to include in the client’s plan of
of expiration. Which of the following should the care?
nurse do? a. Daily weights
a. Call the assessment data to the b. Seizure precautions
physician’s attention c. Right lateral positioning
b. Start oxygen per nasal cannula at 2 d. Stress reduction
L/min.
c. Suction the infant’s mouth and nares 30. A postpartum primipara asks the nurse, “When
d. Recognize this as normal first period of can we have sexual intercourse again?” Which of
reactivity the following would be the nurse’s best
response?
25. The nurse hears a mother telling a friend on the a. “Anytime you both want to.”
telephone about umbilical cord care. Which of b. “As soon as choose a contraceptive
the following statements by the mother method.”
indicates effective teaching? c. “When the discharge has stopped and
a. “Daily soap and water cleansing is best” the incision is healed.”
b. ‘Alcohol helps it dry and kills germs” d. “After your 6 weeks examination.”
c. “An antibiotic ointment applied daily
prevents infection” 31. When preparing to administer the vitamin K
d. “He can have a tub bath each day” injection to a neonate, the nurse would select
which of the following sites as appropriate for
26. A newborn weighing 3000 grams and feeding the injection?
every 4 hours needs 120 calories/kg of body a. Deltoid muscle
weight every 24 hours for proper growth and b. Anterior femoris muscle
development. How many ounces of 20 cal/oz c. Vastus lateralis muscle
formula should this newborn receive at each d. Gluteus maximus muscle
feeding to meet nutritional needs?
a. 2 ounces
32. When performing a pelvic examination, the understanding that breathing techniques are
nurse observes a red swollen area on the right most important in achieving which of the
side of the vaginal orifice. The nurse would following?
document this as enlargement of which of the a. Eliminate pain and give the expectant
following? parents something to do
a. Clitoris b. Reduce the risk of fetal distress by
b. Parotid gland increasing uteroplacental perfusion
c. Skene’s gland c. Facilitate relaxation, possibly reducing
d. Bartholin’s gland the perception of pain
d. Eliminate pain so that less analgesia and
33. To differentiate as a female, the hormonal anesthesia are needed
stimulation of the embryo that must occur
involves which of the following? 38. After 4 hours of active labor, the nurse notes
a. Increase in maternal estrogen secretion that the contractions of a primigravida client are
b. Decrease in maternal androgen not strong enough to dilate the cervix. Which of
secretion the following would the nurse anticipate doing?
c. Secretion of androgen by the fetal gonad a. Obtaining an order to begin IV oxytocin
d. Secretion of estrogen by the fetal gonad infusion
b. Administering a light sedative to allow
34. A client at 8 weeks’ gestation calls complaining the patient to rest for several hour
of slight nausea in the morning hours. Which of c. Preparing for a cesarean section for
the following client interventions should the failure to progress
nurse question? d. Increasing the encouragement to the
a. Taking 1 teaspoon of bicarbonate of patient when pushing begins
soda in an 8-ounce glass of water
b. Eating a few low-sodium crackers before 39. A multigravida at 38 weeks’ gestation is
getting out of bed admitted with painless, bright red bleeding and
c. Avoiding the intake of liquids in the mild contractions every 7 to 10 minutes. Which
morning hours of the following assessments should be avoided?
d. Eating six small meals a day instead of a. Maternal vital sign
thee large meals b. Fetal heart rate
c. Contraction monitoring
35. The nurse documents positive ballottement in d. Cervical dilation
the client’s prenatal record. The nurse
understands that this indicates which of the 40. Which of the following would be the nurse’s
following? most appropriate response to a client who asks
a. Palpable contractions on the abdomen why she must have a cesarean delivery if she has
b. Passive movement of the unengaged a complete placenta previa?
fetus a. “You will have to ask your physician
c. Fetal kicking felt by the client when he returns.”
d. Enlargement and softening of the uterus b. “You need a cesarean to prevent
hemorrhage.”
36. During a pelvic exam the nurse notes a purple- c. “The placenta is covering most of your
blue tinge of the cervix. The nurse documents cervix.”
this as which of the following? d. “The placenta is covering the opening of
a. Braxton-Hicks sign the uterus and blocking your baby.”
b. Chadwick’s sign
c. Goodell’s sign 41. The nurse understands that the fetal head is in
d. McDonald’s sign which of the following positions with a face
presentation?
37. During a prenatal class, the nurse explains the a. Completely flexed
rationale for breathing techniques during b. Completely extended
preparation for labor based on the c. Partially extended

36
36
d. Partially flexed b. Nurse-midwifery
c. Clinical nurse specialist
42. With a fetus in the left-anterior breech d. Prepared childbirth
presentation, the nurse would expect the fetal
heart rate would be most audible in which of the 48. A client has a midpelvic contracture from a
following areas? previous pelvic injury due to a motor vehicle
a. Above the maternal umbilicus and to the accident as a teenager. The nurse is aware that
right of midline this could prevent a fetus from passing through
b. In the lower-left maternal abdominal or around which structure during childbirth?
quadrant a. Symphysis pubis
c. In the lower-right maternal abdominal b. Sacral promontory
quadrant c. Ischial spines
d. Above the maternal umbilicus and to the d. Pubic arch
left of midline
49. When teaching a group of adolescents about
43. The amniotic fluid of a client has a greenish tint. variations in the length of the menstrual cycle,
The nurse interprets this to be the result of the nurse understands that the underlying
which of the following? mechanism is due to variations in which of the
a. Lanugo following phases?
b. Hydramnio a. Menstrual phase
c. Meconium b. Proliferative phase
d. Vernix c. Secretory phase
d. Ischemic phase
44. A patient is in labor and has just been told she
has a breech presentation. The nurse should be 50. When teaching a group of adolescents about
particularly alert for which of the following? male hormone production, which of the
a. Quickening following would the nurse include as being
b. Ophthalmia neonatorum produced by the Leydig cells?
c. Pica a. Follicle-stimulating hormone
d. Prolapsed umbilical cord b. Testosterone
c. Leuteinizing hormone
45. When describing dizygotic twins to a couple, on d. Gonadotropin releasing hormone
which of the following would the nurse base the
explanation?
a. Two ova fertilized by separate sperm
b. Sharing of a common placenta
c. Each ova with the same genotype
d. Sharing of a common chorion

46. Which of the following refers to the single cell


that reproduces itself after conception?
a. Chromosome
b. Blastocyst
c. Zygote
d. Trophoblast

47. In the late 1950s, consumers and health care


professionals began challenging the routine use
of analgesics and anesthetics during childbirth.
Which of the following was an outgrowth of this
concept?
a. Labor, delivery, recovery, postpartum
(LDRP)
ANSWERS AND RATIONALE – MATERNAL AND 4. C. An IUD may increase the risk of pelvic
CHILD HEALTH inflammatory disease, especially in women with
more than one sexual partner, because of the
increased risk of sexually transmitted infections.
1. B. Regular timely ingestion of oral contraceptives An UID should not be used if the woman has an
is necessary to maintain hormonal levels of the active or chronic pelvic infection, postpartum
drugs to suppress the action of the infection, endometrial hyperplasia or carcinoma,
hypothalamus and anterior pituitary leading to or uterine abnormalities. Age is not a factor in
inappropriate secretion of FSH and LH. determining the risks associated with IUD use.
Therefore, follicles do not mature, ovulation is Most IUD users are over the age of 30. Although
inhibited, and pregnancy is prevented. The there is a slightly higher risk for infertility in
estrogen content of the oral site contraceptive women who have never been pregnant, the IUD
may cause the nausea, regardless of when the is an acceptable option as long as the risk-
pill is taken. Side effects and drug interactions benefit ratio is discussed. IUDs may be inserted
may occur with oral contraceptives regardless of immediately after delivery, but this is not
the time the pill is taken. recommended because of the increased risk and
2. C. Condoms, when used correctly and rate of expulsion at this time.
consistently, are the most effective 5. C. During the third trimester, the enlarging
contraceptive method or barrier against uterus places pressure on the intestines. This
bacterial and viral sexually transmitted coupled with the effect of hormones on smooth
infections. Although spermicides kill sperm, they muscle relaxation causes decreased intestinal
do not provide reliable protection against the motility (peristalsis). Increasing fiber in the diet
spread of sexually transmitted infections, will help fecal matter pass more quickly through
especially intracellular organisms such as HIV. the intestinal tract, thus decreasing the amount
Insertion and removal of the diaphragm along of water that is absorbed. As a result, stool is
with the use of the spermicides may cause softer and easier to pass. Enemas could
vaginal irritations, which could place the client at precipitate preterm labor and/or electrolyte loss
risk for infection transmission. Male sterilization and should be avoided. Laxatives may cause
eliminates spermatozoa from the ejaculate, but preterm labor by stimulating peristalsis and may
it does not eliminate bacterial and/or viral interfere with the absorption of nutrients. Use
microorganisms that can cause sexually for more than 1 week can also lead to laxative
transmitted infections. dependency. Liquid in the diet helps provide a
3. A. The diaphragm must be fitted individually to semisolid, soft consistency to the stool. Eight to
ensure effectiveness. Because of the changes to ten glasses of fluid per day are essential to
the reproductive structures during pregnancy maintain hydration and promote stool
and following delivery, the diaphragm must be evacuation.
refitted, usually at the 6 weeks’ examination 6. D. To ensure adequate fetal growth and
following childbirth or after a weight loss of 15 development during the 40 weeks of a
lbs or more. In addition, for maximum pregnancy, a total weight gain 25 to 30 pounds is
effectiveness, spermicidal jelly should be placed recommended: 1.5 pounds in the first 10 weeks;
in the dome and around the rim. However, 9 pounds by 30 weeks; and 27.5 pounds by 40
spermicidal jelly should not be inserted into the weeks. The pregnant woman should gain less
vagina until involution is completed at weight in the first and second trimester than in
approximately 6 weeks. Use of a female condom the third. During the first trimester, the client
protects the reproductive system from the should only gain 1.5 pounds in the first 10
introduction of semen or spermicides into the weeks, not 1 pound per week. A weight gain of
vagina and may be used after childbirth. Oral ½ pound per week would be 20 pounds for the
contraceptives may be started within the first total pregnancy, less than the recommended
postpartum week to ensure suppression of amount.
ovulation. For the couple who has determined 7. B. To calculate the EDD by Nagele’s rule, add 7
the female’s fertile period, using the rhythm days to the first day of the last menstrual period
method, avoidance of intercourse during this and count back 3 months, changing the year
period, is safe and effective. appropriately. To obtain a date of September 27,

36
36
7 days have been added to the last day of the suspected, which may be caused by fluid
LMP (rather than the first day of the LMP), plus 4 retention manifested by edema, especially of the
months (instead of 3 months) were counted hands and face. The three classic signs of
back. To obtain the date of November 7, 7 days preeclampsia are hypertension, edema, and
have been subtracted (instead of added) from proteinuria. Although urine is checked for
the first day of LMP plus November indicates glucose at each clinic visit, this is not the priority.
counting back 2 months (instead of 3 months) Depression may cause either anorexia or
from January. To obtain the date of December excessive food intake, leading to excessive
27, 7 days were added to the last day of the LMP weight gain or loss. This is not, however, the
(rather than the first day of the LMP) and priority consideration at this time. Weight gain
December indicates counting back only 1 month thought to be caused by excessive food intake
(instead of 3 months) from January. would require a 24-hour diet recall. However,
8. D. The client has been pregnant four times, excessive intake would not be the primary
including current pregnancy (G). Birth at 38 consideration for this client at this time.
weeks’ gestation is considered full term (T), 12. B. Cramping and vaginal bleeding coupled with
while birth form 20 weeks to 38 weeks is cervical dilation signifies that termination of the
considered preterm (P). A spontaneous abortion pregnancy is inevitable and cannot be
occurred at 8 weeks (A). She has two living prevented. Thus, the nurse would document an
children (L). imminent abortion. In a threatened abortion,
9. B. At 12 weeks gestation, the uterus rises out of cramping and vaginal bleeding are present, but
the pelvis and is palpable above the symphysis there is no cervical dilation. The symptoms may
pubis. The Doppler intensifies the sound of the subside or progress to abortion. In a complete
fetal pulse rate so it is audible. The uterus has abortion all the products of conception are
merely risen out of the pelvis into the abdominal expelled. A missed abortion is early fetal
cavity and is not at the level of the umbilicus. intrauterine death without expulsion of the
The fetal heart rate at this age is not audible products of conception.
with a stethoscope. The uterus at 12 weeks is 13. B. For the client with an ectopic pregnancy,
just above the symphysis pubis in the abdominal lower abdominal pain, usually unilateral, is the
cavity, not midway between the umbilicus and primary symptom. Thus, pain is the priority.
the xiphoid process. At 12 weeks the FHR would Although the potential for infection is always
be difficult to auscultate with a fetoscope. present, the risk is low in ectopic pregnancy
Although the external electronic fetal monitor because pathogenic microorganisms have not
would project the FHR, the uterus has not risen been introduced from external sources. The
to the umbilicus at 12 weeks. client may have a limited knowledge of the
10. A. Although all of the choices are important in pathology and treatment of the condition and
the management of diabetes, diet therapy is the will most likely experience grieving, but this is
mainstay of the treatment plan and should not the priority at this time.
always be the priority. Women diagnosed with 14. D. Before uterine assessment is performed, it is
gestational diabetes generally need only diet essential that the woman empty her bladder. A
therapy without medication to control their full bladder will interfere with the accuracy of
blood sugar levels. Exercise, is important for all the assessment by elevating the uterus and
pregnant women and especially for diabetic displacing to the side of the midline. Vital sign
women, because it burns up glucose, thus assessment is not necessary unless an
decreasing blood sugar. However, dietary intake, abnormality in uterine assessment is identified.
not exercise, is the priority. All pregnant women Uterine assessment should not cause acute pain
with diabetes should have periodic monitoring that requires administration of analgesia.
of serum glucose. However, those with Ambulating the client is an essential component
gestational diabetes generally do not need daily of postpartum care, but is not necessary prior to
glucose monitoring. The standard of care assessment of the uterus.
recommends a fasting and 2- hour postprandial 15. A. Feeding more frequently, about every 2
blood sugar level every 2 weeks. hours, will decrease the infant’s frantic, vigorous
11. C. After 20 weeks’ gestation, when there is a sucking from hunger and will decrease breast
rapid weight gain, preeclampsia should be engorgement, soften the breast, and promote
ease of correct latching-on for feeding. Narcotics 18. A. The data suggests an infection of the
administered prior to breast feeding are passed endometrial lining of the uterus. The lochia may
through the breast milk to the infant, causing be decreased or copious, dark brown in
excessive sleepiness. Nipple soreness is not appearance, and foul smelling, providing further
severe enough to warrant narcotic analgesia. All evidence of a possible infection. All the client’s
postpartum clients, especially lactating mothers, data indicate a uterine problem, not a breast
should wear a supportive brassiere with wide problem. Typically, transient fever, usually
cotton straps. This does not, however, prevent 101ºF, may be present with breast
or reduce nipple soreness. Soaps are drying to engorgement. Symptoms of mastitis include
the skin of the nipples and should not be used influenza-like manifestations. Localized infection
on the breasts of lactating mothers. Dry nipple of an episiotomy or C-section incision rarely
skin predisposes to cracks and fissures, which causes systemic symptoms, and uterine
can become sore and painful. involution would not be affected. The client data
16. D. A weak, thready pulse elevated to 100 BPM do not include dysuria, frequency, or urgency,
may indicate impending hemorrhagic shock. An symptoms of urinary tract infections, which
increased pulse is a compensatory mechanism of would necessitate assessing the client’s urine.
the body in response to decreased fluid volume. 19. C. Because of early postpartum discharge and
Thus, the nurse should check the amount of limited time for teaching, the nurse’s priority is
lochia present. Temperatures up to 100.48F in to facilitate the safe and effective care of the
the first 24 hours after birth are related to the client and newborn. Although promoting
dehydrating effects of labor and are considered comfort and restoration of health, exploring the
normal. Although rechecking the blood pressure family’s emotional status, and teaching about
may be a correct choice of action, it is not the family planning are important in
first action that should be implemented in light postpartum/newborn nursing care, they are not
of the other data. The data indicate a potential the priority focus in the limited time presented
impending hemorrhage. Assessing the uterus for by early post-partum discharge.
firmness and position in relation to the umbilicus 20. C. Heat loss by radiation occurs when the
and midline is important, but the nurse should infant’s crib is placed too near cold walls or
check the extent of vaginal bleeding first. Then it windows. Thus placing the newborn’s crib close
would be appropriate to check the uterus, which to the viewing window would be least effective.
may be a possible cause of the hemorrhage. Body heat is lost through evaporation during
17. D. Any bright red vaginal discharge would be bathing. Placing the infant under the radiant
considered abnormal, but especially 5 days after warmer after bathing will assist the infant to be
delivery, when the lochia is typically pink to rewarmed. Covering the scale with a warmed
brownish. Lochia rubra, a dark red discharge, is blanket prior to weighing prevents heat loss
present for 2 to 3 days after delivery. Bright red through conduction. A knit cap prevents heat
vaginal bleeding at this time suggests late loss from the head a large head, a large body
postpartum hemorrhage, which occurs after the surface area of the newborn’s body.
first 24 hours following delivery and is generally 21. B. A fractured clavicle would prevent the normal
caused by retained placental fragments or Moro response of symmetrical sequential
bleeding disorders. Lochia rubra is the normal extension and abduction of the arms followed by
dark red discharge occurring in the first 2 to 3 flexion and adduction. In talipes equinovarus
days after delivery, containing epithelial cells, (clubfoot) the foot is turned medially, and in
erythrocyes, leukocytes and decidua. Lochia plantar flexion, with the heel elevated. The feet
serosa is a pink to brownish serosanguineous are not involved with the Moro reflex.
discharge occurring from 3 to 10 days after Hypothyroiddism has no effect on the primitive
delivery that contains decidua, erythrocytes, reflexes. Absence of the Moror reflex is the most
leukocytes, cervical mucus, and microorganisms. significant single indicator of central nervous
Lochia alba is an almost colorless to yellowish system status, but it is not a sign of increased
discharge occurring from 10 days to 3 weeks intracranial pressure.
after delivery and containing leukocytes, 22. B. Hemorrhage is a potential risk following any
decidua, epithelial cells, fat, cervical mucus, surgical procedure. Although the infant has been
cholesterol crystals, and bacteria. given vitamin K to facilitate clotting, the

36
36
prophylactic dose is often not sufficient to for gastrointestinal problems. Even though the
prevent bleeding. Although infection is a skin is stained with meconium, it is noninfectious
possibility, signs will not appear within 4 hours (sterile) and nonirritating. The postterm
after the surgical procedure. The primary meconium- stained infant is not at additional risk
discomfort of circumcision occurs during the for bowel or urinary problems.
surgical procedure, not afterward. Although 28. C. The nurse should use a nonelastic, flexible,
feedings are withheld prior to the circumcision, paper measuring tape, placing the zero point on
the chances of dehydration are minimal. the superior border of the symphysis pubis and
23. B. The presence of excessive estrogen and stretching the tape across the abdomen at the
progesterone in the maternal- fetal blood midline to the top of the fundus. The xiphoid and
followed by prompt withdrawal at birth umbilicus are not appropriate landmarks to use
precipitates breast engorgement, which will when measuring the height of the fundus
spontaneously resolve in 4 to 5 days after birth. (McDonald’s measurement).
The trauma of the birth process does not cause 29. B. Women hospitalized with severe
inflammation of the newborn’s breast tissue. preeclampsia need decreased CNS stimulation to
Newborns do not have breast malignancy. This prevent a seizure. Seizure precautions provide
reply by the nurse would cause the mother to environmental safety should a seizure occur.
have undue anxiety. Breast tissue does not Because of edema, daily weight is important but
hypertrophy in the fetus or newborns. not the priority. Preclampsia causes vasospasm
24. D. The first 15 minutes to 1 hour after birth is and therefore can reduce utero-placental
the first period of reactivity involving respiratory perfusion. The client should be placed on her left
and circulatory adaptation to extrauterine life. side to maximize blood flow, reduce blood
The data given reflect the normal changes during pressure, and promote diuresis. Interventions to
this time period. The infant’s assessment data reduce stress and anxiety are very important to
reflect normal adaptation. Thus, the physician facilitate coping and a sense of control, but
does not need to be notified and oxygen is not seizure precautions are the priority.
needed. The data do not indicate the presence 30. C. Cessation of the lochial discharge signifies
of choking, gagging or coughing, which are signs healing of the endometrium. Risk of hemorrhage
of excessive secretions. Suctioning is not and infection are minimal 3 weeks after a
necessary. normal vaginal delivery. Telling the client
25. B. Application of 70% isopropyl alcohol to the anytime is inappropriate because this response
cord minimizes microorganisms (germicidal) and does not provide the client with the specific
promotes drying. The cord should be kept dry information she is requesting. Choice of a
until it falls off and the stump has healed. contraceptive method is important, but not the
Antibiotic ointment should only be used to treat specific criteria for safe resumption of sexual
an infection, not as a prophylaxis. Infants should activity. Culturally, the 6- weeks’ examination
not be submerged in a tub of water until the has been used as the time frame for resuming
cord falls off and the stump has completely sexual activity, but it may be resumed earlier.
healed. 31. C. The middle third of the vastus lateralis is the
26. B. To determine the amount of formula needed, preferred injection site for vitamin K
do the following mathematical calculation. 3 kg x administration because it is free of blood vessels
120 cal/kg per day = 360 calories/day feeding q 4 and nerves and is large enough to absorb the
hours = 6 feedings per day = 60 calories per medication. The deltoid muscle of a newborn is
feeding: 60 calories per feeding; 60 calories per not large enough for a newborn IM injection.
feeding with formula 20 cal/oz = 3 ounces per Injections into this muscle in a small child might
feeding. Based on the calculation. 2, 4 or 6 cause damage to the radial nerve. The anterior
ounces are incorrect. femoris muscle is the next safest muscle to use
27. A. Intrauterine anoxia may cause relaxation of in a newborn but is not the safest. Because of
the anal sphincter and emptying of meconium the proximity of the sciatic nerve, the gluteus
into the amniotic fluid. At birth some of the maximus muscle should not be until the child
meconium fluid may be aspirated, causing has been walking 2 years.
mechanical obstruction or chemical 32. D. Bartholin’s glands are the glands on either
pneumonitis. The infant is not at increased risk side of the vaginal orifice. The clitoris is female
erectile tissue found in the perineal area above 39. D. The signs indicate placenta previa and vaginal
the urethra. The parotid glands are open into the exam to determine cervical dilation would not
mouth. Skene’s glands open into the posterior be done because it could cause hemorrhage.
wall of the female urinary meatus. Assessing maternal vital signs can help
33. D. The fetal gonad must secrete estrogen for the determine maternal physiologic status. Fetal
embryo to differentiate as a female. An increase heart rate is important to assess fetal well-being
in maternal estrogen secretion does not affect and should be done. Monitoring the contractions
differentiation of the embryo, and maternal will help evaluate the progress of labor.
estrogen secretion occurs in every pregnancy. 40. D. A complete placenta previa occurs when the
Maternal androgen secretion remains the same placenta covers the opening of the uterus, thus
as before pregnancy and does not affect blocking the passageway for the baby. This
differentiation. Secretion of androgen by the response explains what a complete previa is and
fetal gonad would produce a male fetus. the reason the baby cannot come out except by
34. A. Using bicarbonate would increase the amount cesarean delivery. Telling the client to ask the
of sodium ingested, which can cause physician is a poor response and would increase
complications. Eating low-sodium crackers the patient’s anxiety. Although a cesarean would
would be appropriate. Since liquids can increase help to prevent hemorrhage, the statement does
nausea avoiding them in the morning hours not explain why the hemorrhage could occur.
when nausea is usually the strongest is With a complete previa, the placenta is covering
appropriate. Eating six small meals a day would the entire cervix, not just most of it.
keep the stomach full, which often decrease 41. B. With a face presentation, the head is
nausea. completely extended. With a vertex
35. B. Ballottement indicates passive movement of presentation, the head is completely or partially
the unengaged fetus. Ballottement is not a flexed. With a brow (forehead) presentation, the
contraction. Fetal kicking felt by the client head would be partially extended.
represents quickening. Enlargement and 42. D. With this presentation, the fetal upper torso
softening of the uterus is known as Piskacek’s and back face the left upper maternal abdominal
sign. wall. The fetal heart rate would be most audible
36. B. Chadwick’s sign refers to the purple-blue tinge above the maternal umbilicus and to the left of
of the cervix. Braxton Hicks contractions are the middle. The other positions would be
painless contractions beginning around the 4 th incorrect.
month. Goodell’s sign indicates softening of the 43. C. The greenish tint is due to the presence of
cervix. Flexibility of the uterus against the cervix meconium. Lanugo is the soft, downy hair on the
is known as McDonald’s sign. shoulders and back of the fetus. Hydramnios
37. C. Breathing techniques can raise the pain represents excessive amniotic fluid. Vernix is the
threshold and reduce the perception of pain. white, cheesy substance covering the fetus.
They also promote relaxation. Breathing 44. D. In a breech position, because of the space
techniques do not eliminate pain, but they can between the presenting part and the cervix,
reduce it. Positioning, not breathing, increases prolapse of the umbilical cord is common.
uteroplacental perfusion. Quickening is the woman’s first perception of
38. A. The client’s labor is hypotonic. The nurse fetal movement. Ophthalmia neonatorum
should call the physical and obtain an order for usually results from maternal gonorrhea and is
an infusion of oxytocin, which will assist the conjunctivitis. Pica refers to the oral intake of
uterus to contact more forcefully in an attempt nonfood substances.
to dilate the cervix. Administering light sedative 45. A. Dizygotic (fraternal) twins involve two ova
would be done for hypertonic uterine fertilized by separate sperm. Monozygotic
contractions. Preparing for cesarean section is (identical) twins involve a common placenta,
unnecessary at this time. Oxytocin would same genotype, and common chorion.
increase the uterine contractions and hopefully 46. C. The zygote is the single cell that reproduces
progress labor before a cesarean would be itself after conception. The chromosome is the
necessary. It is too early to anticipate client material that makes up the cell and is gained
pushing with contractions. from each parent. Blastocyst and trophoblast are
later terms for the embryo after zygote.

37
37
47. D. Prepared childbirth was the direct result of
the 1950’s challenging of the routine use of
analgesic and anesthetics during childbirth. The
LDRP was a much later concept and was not a
direct result of the challenging of routine use of
analgesics and anesthetics during childbirth.
Roles for nurse midwives and clinical nurse
specialists did not develop from this challenge.
48. C. The ischial spines are located in the mid-pelvic
region and could be narrowed due to the
previous pelvic injury. The symphysis pubis,
sacral promontory, and pubic arch are not part
of the mid-pelvis.
49. B. Variations in the length of the menstrual cycle
are due to variations in the proliferative phase.
The menstrual, secretory and ischemic phases
do not contribute to this variation.
50. B. Testosterone is produced by the Leyding cells
in the seminiferous tubules. Follicle-stimulating
hormone and leuteinzing hormone are released
by the anterior pituitary gland. The
hypothalamus is responsible for releasing
gonadotropin-releasing hormone.
MEDICAL SURGICAL NURSING c. Perineal edema
d. Urethral discharge
1. Marco who was diagnosed with brain tumor was
scheduled for craniotomy. In preventing the 7. A client has undergone with penile implant.
development of cerebral edema after surgery, After 24 hrs of surgery, the client’s scrotum was
the nurse should expect the use of: edematous and painful. The nurse should:
a. Diuretics a. Assist the client with sitz bath
b. Antihypertensive b. Apply war soaks in the scrotum
c. Steroids c. Elevate the scrotum using a soft support
d. Anticonvulsants d. Prepare for a possible incision and
drainage.
2. Halfway through the administration of blood,
the female client complains of lumbar pain. After 8. Nurse hazel receives emergency laboratory
stopping the infusion Nurse Hazel should: results for a client with chest pain and
a. Increase the flow of normal saline immediately informs the physician. An increased
b. Assess the pain further myoglobin level suggests which of the following?
c. Notify the blood bank a. Liver disease
d. Obtain vital signs. b. Myocardial damage
c. Hypertension
3. Nurse Maureen knows that the positive d. Cancer
diagnosis for HIV infection is made based on
which of the following: 9. Nurse Maureen would expect the client with
a. A history of high risk sexual behaviors. mitral stenosis would demonstrate symptoms
b. Positive ELISA and western blot tests associated with congestion in the:
c. Identification of an associated a. Right atrium
opportunistic infection b. Superior vena cava
d. Evidence of extreme weight loss and c. Aorta
high fever d. Pulmonary

4. Nurse Maureen is aware that a client who has 10. A client has been diagnosed with hypertension.
been diagnosed with chronic renal failure The nurse priority nursing diagnosis would be:
recognizes an adequate amount of high-biologic- a. Ineffective health maintenance
value protein when the food the client selected b. Impaired skin integrity
from the menu was: c. Deficient fluid volume
a. Raw carrots d. Pain
b. Apple juice
c. Whole wheat bread 11. Nurse Hazel teaches the client with angina about
d. Cottage cheese common expected side effects of nitroglycerin
including:
5. Kenneth who has diagnosed with uremic a. high blood pressure
syndrome has the potential to develop b. stomach cramps
complications. Which among the following c. headache
complications should the nurse anticipates: d. shortness of breath
a. Flapping hand tremors
b. An elevated hematocrit level 12. The following are lipid abnormalities. Which of
c. Hypotension the following is a risk factor for the development
d. Hypokalemia of atherosclerosis and PVD?
a. High levels of low density lipid (LDL)
6. A client is admitted to the hospital with benign cholesterol
prostatic hyperplasia, the nurse most relevant b. High levels of high density lipid (HDL)
assessment would be: cholesterol
a. Flank pain radiating in the groin c. Low concentration triglycerides
b. Distention of the lower abdomen d. Low levels of LDL cholesterol.

37
37

b. Administering Coumadin
13. Which of the following represents a significant c. Treating the underlying cause
risk immediately after surgery for repair of aortic d. Replacing depleted blood products
aneurysm?
a. Potential wound infection 20. Which of the following findings is the best
b. Potential ineffective coping indication that fluid replacement for the client
c. Potential electrolyte balance with hypovolemic shock is adequate?
d. Potential alteration in renal perfusion a. Urine output greater than 30ml/hr
b. Respiratory rate of 21 breaths/minute
14. Nurse Josie should instruct the client to eat c. Diastolic blood pressure greater than 90
which of the following foods to obtain the best mmhg
supply of Vitamin B12? d. Systolic blood pressure greater than 110
a. dairy products mmhg
b. vegetables
c. Grains 21. Which of the following signs and symptoms
d. Broccoli would Nurse Maureen include in teaching plan
as an early manifestation of laryngeal cancer?
15. Karen has been diagnosed with aplastic anemia. a. Stomatitis
The nurse monitors for changes in which of the b. Airway obstruction
following physiologic functions? c. Hoarseness
a. Bowel function d. Dysphagia
b. Peripheral sensation
c. Bleeding tendencies 22. Karina a client with myasthenia gravis is to
d. Intake and out put receive immunosuppressive therapy. The nurse
understands that this therapy is effective
16. Lydia is scheduled for elective splenectomy. because it:
Before the clients goes to surgery, the nurse in a. Promotes the removal of antibodies that
charge final assessment would be: impair the transmission of impulses
a. signed consent b. Stimulates the production of
b. vital signs acetylcholine at the neuromuscular
c. name band junction.
d. empty bladder c. Decreases the production of
autoantibodies that attack the
17. What is the peak age range in acquiring acute acetylcholine receptors.
lymphocytic leukemia (ALL)? d. Inhibits the breakdown of acetylcholine
a. 4 to 12 years. at the neuromuscular junction.
b. 20 to 30 years
c. 40 to 50 years 23. A female client is receiving IV Mannitol. An
d. 60 60 70 years assessment specific to safe administration of the
said drug is:
18. Marie with acute lymphocytic leukemia suffers a. Vital signs q4h
from nausea and headache. These clinical b. Weighing daily
manifestations may indicate all of the following c. Urine output hourly
except d. Level of consciousness q4h
a. effects of radiation
b. chemotherapy side effects 24. Patricia a 20 year old college student with
c. meningeal irritation diabetes mellitus requests additional
d. gastric distension information about the advantages of using a pen
like insulin delivery devices. The nurse explains
19. A client has been diagnosed with Disseminated that the advantages of these devices over
Intravascular Coagulation (DIC). Which of the syringes include:
following is contraindicated with the client? a. Accurate dose delivery
a. Administering Heparin b. Shorter injection time
c. Lower cost with reusable insulin
cartridges 30. A male client has undergone spinal surgery, the
d. Use of smaller gauge needle. nurse should:
a. Observe the client’s bowel movement
25. A male client’s left tibia is fractures in an and voiding patterns
automobile accident, and a cast is applied. To b. Log-roll the client to prone position
assess for damage to major blood vessels from c. Assess the client’s feet for sensation and
the fracture tibia, the nurse in charge should circulation
monitor the client for: d. Encourage client to drink plenty of fluids
a. Swelling of the left thigh
b. Increased skin temperature of the foot 31. Marina with acute renal failure moves into the
c. Prolonged reperfusion of the toes after diuretic phase after one week of therapy. During
blanching this phase the client must be assessed for signs
d. Increased blood pressure of developing:
a. Hypovolemia
26. After a long leg cast is removed, the male client b. renal failure
should: c. metabolic acidosis
a. Cleanse the leg by scrubbing with a brisk d. hyperkalemia
motion
b. Put leg through full range of motion 32. Nurse Judith obtains a specimen of clear nasal
twice daily drainage from a client with a head injury. Which
c. Report any discomfort or stiffness to the of the following tests differentiates mucus from
physician cerebrospinal fluid (CSF)?
d. Elevate the leg when sitting for long a. Protein
periods of time. b. Specific gravity
c. Glucose
27. While performing a physical assessment of a d. Microorganism
male client with gout of the great toe,
NurseVivian should assess for additional tophi 33. A 22 year old client suffered from his first tonic-
(urate deposits) on the: clonic seizure. Upon awakening the client asks
a. Buttocks the nurse, “What caused me to have a seizure?
b. Ears Which of the following would the nurse include
c. Face in the primary cause of tonic-clonic seizures in
d. Abdomen adults more the 20 years?
a. Electrolyte imbalance
28. Nurse Katrina would recognize that the b. Head trauma
demonstration of crutch walking with tripod gait c. Epilepsy
was understood when the client places weight d. Congenital defect
on the:
a. Palms of the hands and axillary regions 34. What is the priority nursing assessment in the
b. Palms of the hand first 24 hours after admission of the client with
c. Axillary regions thrombotic CVA?
d. Feet, which are set apart a. Pupil size and papillary response
b. cholesterol level
29. Mang Jose with rheumatoid arthritis states, “the c. Echocardiogram
only time I am without pain is when I lie in bed d. Bowel sounds
perfectly still”. During the convalescent stage,
the nurse in charge with Mang Jose should 35. Nurse Linda is preparing a client with multiple
encourage: sclerosis for discharge from the hospital to
a. Active joint flexion and extension home. Which of the following instruction is most
b. Continued immobility until pain subsides appropriate?
c. Range of motion exercises twice daily
d. Flexion exercises three times daily

37
37
a. “Practice using the mechanical aids that d. A client with U.T.I
you will need when future disabilities
arise”. 42. Among the following clients, which among them
b. “Follow good health habits to change is high risk for potential hazards from the
the course of the disease”. surgical experience?
c. “Keep active, use stress reduction a. 67-year-old client
strategies, and avoid fatigue. b. 49-year-old client
d. “You will need to accept the necessity c. 33-year-old client
for a quiet and inactive lifestyle”. d. 15-year-old client

36. The nurse is aware the early indicator of hypoxia 43. Nurse Jon assesses vital signs on a client
in the unconscious client is: undergone epidural anesthesia.
a. Cyanosis 44. Which of the following would the nurse assess
b. Increased respirations next?
c. Hypertension a. Headache
d. Restlessness b. Bladder distension
c. Dizziness
37. A client is experiencing spinal shock. Nurse d. Ability to move legs
Myrna should expect the function of the bladder
to be which of the following? 45. Nurse Katrina should anticipate that all of the
a. Normal following drugs may be used in the attempt to
b. Atonic control the symptoms of Meniere's disease
c. Spastic except:
d. Uncontrolled a. Antiemetics
b. Diuretics
38. Which of the following stage the carcinogen is c. Antihistamines
irreversible? d. Glucocorticoids
a. Progression stage
b. Initiation stage 46. Which of the following complications associated
c. Regression stage with tracheostomy tube?
d. Promotion stage a. Increased cardiac output
b. Acute respiratory distress syndrome
39. Among the following components thorough pain (ARDS)
assessment, which is the most significant? c. Increased blood pressure
a. Effect d. Damage to laryngeal nerves
b. Cause
c. Causing factors 47. Nurse Faith should recognize that fluid shift in a
d. Intensity client with burn injury results from increase in
the:
40. A 65 year old female is experiencing flare up of a. Total volume of circulating whole blood
pruritus. Which of the client’s action could b. Total volume of intravascular plasma
aggravate the cause of flare ups? c. Permeability of capillary walls
a. Sleeping in cool and humidified d. Permeability of kidney tubules
environment
b. Daily baths with fragrant soap 48. An 83-year-old woman has several ecchymotic
c. Using clothes made from 100% cotton areas on her right arm. The bruises are probably
d. Increasing fluid intake caused by:
a. increased capillary fragility and
41. Atropine sulfate (Atropine) is contraindicated in permeability
all but one of the following client? b. increased blood supply to the skin
a. A client with high blood c. self-inflicted injury
b. A client with bowel obstruction d. elder abuse
c. A client with glaucoma
49. Nurse Anna is aware that early adaptation of
client with renal carcinoma is:
a. Nausea and vomiting
b. flank pain
c. weight gain
d. intermittent hematuria

50. A male client with tuberculosis asks Nurse Brian


how long the chemotherapy must be continued.
Nurse Brian’s accurate reply would be:
a. 1 to 3 weeks
b. 6 to 12 months
c. 3 to 5 months
d. 3 years and more

51. A client has undergone laryngectomy. The


immediate nursing priority would be:
a. Keep trachea free of secretions
b. Monitor for signs of infection
c. Provide emotional support
d. Promote means of communication

37
37
ANSWERS AND RATIONALE – MEDICAL SURGICAL 14. A. Good source of vitamin B12 are dairy
NURSING products and meats.
15. C. Aplastic anemia decreases the bone marrow
1. C. Glucocorticoids (steroids) are used for their production of RBC’s, white blood cells, and
anti-inflammatory action, which decreases the platelets. The client is at risk for bruising and
development of edema. bleeding tendencies.
2. A. The blood must be stopped at once, and then 16. B. An elective procedure is scheduled in advance
normal saline should be infused to keep the line so that all preparations can be completed ahead
patent and maintain blood volume. of time. The vital signs are the final check that
3. B. These tests confirm the presence of HIV must be completed before the client leaves the
antibodies that occur in response to the room so that continuity of care and assessment
presence of the human immunodeficiency virus is provided for.
(HIV). 17. A. The peak incidence of Acute Lymphocytic
4. D. One cup of cottage cheese contains Leukemia (ALL) is 4 years of age. It is uncommon
approximately 225 calories, 27g of protein, 9g of after 15 years of age.
fat, 30mg cholesterol, and 6g of carbohydrate. 18. D. Acute Lymphocytic Leukemia (ALL) does not
Proteins of high biologic value (HBV) contain cause gastric distention. It does invade the
optimal levels of amino acids essential for life. central nervous system, and clients experience
5. A. Elevation of uremic waste products causes headaches and vomiting from meningeal
irritation of the nerves, resulting in flapping irritation.
hand tremors. 19. B. Disseminated Intravascular Coagulation (DIC)
6. B. This indicates that the bladder is distended has not been found to respond to oral
with urine, therefore palpable. anticoagulants such as Coumadin.
7. C. Elevation increases lymphatic drainage, 20. A. Urine output provides the most sensitive
reducing edema and pain. indication of the client’s response to therapy for
8. B. Detection of myoglobin is a diagnostic tool to hypovolemic shock. Urine output should be
determine whether myocardial damage has consistently greater than 30 to 35 mL/hr.
occurred. 21. C. Early warning signs of laryngeal cancer can
9. D. When mitral stenosis is present, the left vary depending on tumor location. Hoarseness
atrium has difficulty emptying its contents into lasting 2 weeks should be evaluated because it is
the left ventricle because there is no valve to one of the most common warning signs.
prevent back ward flow into the pulmonary vein, 22. C. Steroids decrease the body’s immune
the pulmonary circulation is under pressure. response thus decreasing the production of
10. A. Managing hypertension is the priority for the antibodies that attack the acetylcholine
client with hypertension. Clients with receptors at the neuromuscular junction
hypertension frequently do not experience pain, 23. C. The osmotic diuretic mannitol is
deficient volume, or impaired skin integrity. It is contraindicated in the presence of inadequate
the asymptomatic nature of hypertension that renal function or heart failure because it
makes it so difficult to treat. increases the intravascular volume that must be
11. C. Because of its widespread vasodilating effects, filtered and excreted by the kidney.
nitroglycerin often produces side effects such as 24. A. These devices are more accurate because
headache, hypotension and dizziness. they are easily to used and have improved
12. A. An increased in LDL cholesterol concentration adherence in insulin regimens by young people
has been documented at risk factor for the because the medication can be administered
development of atherosclerosis. LDL cholesterol discreetly.
is not broken down into the liver but is 25. C. Damage to blood vessels may decrease the
deposited into the wall of the blood vessels. circulatory perfusion of the toes, this would
13. D. There is a potential alteration in renal indicate the lack of blood supply to the
perfusion manifested by decreased urine output. extremity.
The altered renal perfusion may be related to 26. D. Elevation will help control the edema that
renal artery embolism, prolonged hypotension, usually occurs.
or prolonged aortic cross-clamping during the 27. B. Uric acid has a low solubility, it tends to
surgery. precipitate and form deposits at various sites
where blood flow is least active, including 43. B. The last area to return sensation is in the
cartilaginous tissue such as the ears. perineal area, and the nurse in charge should
28. B. The palms should bear the client’s weight monitor the client for distended bladder.
to avoid damage to the nerves in the axilla. 44. D. Glucocorticoids play no significant role in
29. A. Active exercises, alternating extension, disease treatment.
flexion, abduction, and adduction, mobilize 45. D. Tracheostomy tube has several potential
exudates in the joints relieves stiffness and pain. complications including bleeding, infection and
30. C. Alteration in sensation and circulation laryngeal nerve damage.
indicates damage to the spinal cord, if these 46. C. In burn, the capillaries and small vessels
occurs notify physician immediately. dilate, and cell damage cause the release of a
31. A. In the diuretic phase fluid retained during the histamine-like substance. The substance causes
oliguric phase is excreted and may reach 3 to 5 the capillary walls to become more permeable
liters daily, hypovolemia may occur and fluids and significant quantities of fluid are lost.
should be replaced. 47. A. Aging process involves increased capillary
32. C. The constituents of CSF are similar to those of fragility and permeability. Older adults have a
blood plasma. An examination for glucose decreased amount of subcutaneous fat and
content is done to determine whether a body cause an increased incidence of bruise like
fluid is a mucus or a CSF. A CSF normally contains lesions caused by collection of extravascular
glucose. blood in loosely structured dermis.
33. B. Trauma is one of the primary causes of brain 48. D. Intermittent pain is the classic sign of renal
damage and seizure activity in adults. Other carcinoma. It is primarily due to capillary erosion
common causes of seizure activity in adults by the cancerous growth.
include neoplasms, withdrawal from drugs and 49. B. Tubercle bacillus is a drug resistant organism
alcohol, and vascular disease. and takes a long time to be eradicated. Usually a
34. A. It is crucial to monitor the pupil size and combination of three drugs is used for minimum
papillary response to indicate changes around of 6 months and at least six months beyond
the cranial nerves. culture conversion.
35. C. The nurse most positive approach is to 50. A. Patent airway is the most priority; therefore
encourage the client with multiple sclerosis to removal of secretions is necessary
stay active, use stress reduction techniques and
avoid fatigue because it is important to support
the immune system while remaining active.
36. D. Restlessness is an early indicator of hypoxia.
The nurse should suspect hypoxia in unconscious
client who suddenly becomes restless.
37. B. In spinal shock, the bladder becomes
completely atonic and will continue to fill unless
the client is catheterized.
38. A. Progression stage is the change of tumor from
the preneoplastic state or low degree of
malignancy to a fast growing tumor that cannot
be reversed.
39. D. Intensity is the major indicative of severity of
pain and it is important for the evaluation of the
treatment.
40. B. The use of fragrant soap is very drying to skin
hence causing the pruritus.
41. C. Atropine sulfate is contraindicated with
glaucoma patients because it increases
intraocular pressure.
42. A. A 67 year old client is greater risk because the
older adult client is more likely to have a less-
effective immune system.

37
37
PSYCHIATRIC NURSING 7. A 20 year old client was diagnosed with
dependent personality disorder. Which behavior
1. Marco approached Nurse Trish asking for advice is not likely to be evidence of ineffective
on how to deal with his alcohol addiction. Nurse individual coping?
Trish should tell the client that the only effective a. Recurrent self-destructive behavior
treatment for alcoholism is: b. Avoiding relationship
a. Psychotherapy c. Showing interest in solitary activities
b. Alcoholics anonymous (A.A.) d. Inability to make choices and decision
c. Total abstinence without advise
d. Aversion Therapy
8. A male client is diagnosed with schizotypal
2. Nurse Hazel is caring for a male client who personality disorder. Which signs would this
experience false sensory perceptions with no client exhibit during social situation?
basis in reality. This perception is known as: a. Paranoid thoughts
a. Hallucinations b. Emotional affect
b. Delusions c. Independence need
c. Loose associations d. Aggressive behavior
d. Neologisms
9. Nurse Claire is caring for a client diagnosed with
3. Nurse Monet is caring for a female client who bulimia. The most appropriate initial goal for a
has suicidal tendency. When accompanying the client diagnosed with bulimia is?
client to the restroom, Nurse Monet should… a. Encourage to avoid foods
a. Give her privacy b. Identify anxiety causing situations
b. Allow her to urinate c. Eat only three meals a day
c. Open the window and allow her to get d. Avoid shopping plenty of groceries
some fresh air
d. Observe her 10. Nurse Tony was caring for a 41 year old female
client. Which behavior by the client indicates
4. Nurse Maureen is developing a plan of care for a adult cognitive development?
female client with anorexia nervosa. Which a. Generates new levels of awareness
action should the nurse include in the plan? b. Assumes responsibility for her actions
a. Provide privacy during meals c. Has maximum ability to solve problems
b. Set-up a strict eating plan for the client and learn new skills
c. Encourage client to exercise to reduce d. Her perception are based on reality
anxiety
d. Restrict visits with the family 11. A neuromuscular blocking agent is administered
to a client before ECT therapy. The Nurse should
5. A client is experiencing anxiety attack. The most carefully observe the client for?
appropriate nursing intervention should include? a. Respiratory difficulties
a. Turning on the television b. Nausea and vomiting
b. Leaving the client alone c. Dizziness
c. Staying with the client and speaking in d. Seizures
short sentences
d. Ask the client to play with other clients 12. A 75 year old client is admitted to the hospital
with the diagnosis of dementia of the
6. A female client is admitted with a diagnosis of Alzheimer’s type and depression. The symptom
delusions of GRANDEUR. This diagnosis reflects a that is unrelated to depression would be?
belief that one is: a. Apathetic response to the environment
a. Being Killed b. “I don’t know” answer to questions
b. Highly famous and important c. Shallow of labile effect
c. Responsible for evil world d. Neglect of personal hygiene
d. Connected to client unrelated to oneself
13. Nurse Trish is working in a mental health facility; following actions by the nurse would be most
the nurse priority nursing intervention for a important?
newly admitted client with bulimia nervosa a. Ask a family member to stay with the
would be to? client at home temporarily
a. Teach client to measure I & O b. Discuss the meaning of the client’s
b. Involve client in planning daily meal statement with her
c. Observe client during meals c. Request an immediate extension for the
d. Monitor client continuously client
d. Ignore the clients statement because it’s
14. Nurse Patricia is aware that the major health a sign of manipulation
complication associated with intractable
anorexia nervosa would be? 19. Joey a client with antisocial personality disorder
a. Cardiac dysrhythmias resulting to belches loudly. A staff member asks Joey, “Do
cardiac arrest you know why people find you repulsive?” this
b. Glucose intolerance resulting in statement most likely would elicit which of the
protracted hypoglycemia following client reaction?
c. Endocrine imbalance causing cold a. Depensiveness
amenorrhea b. Embarrassment
d. Decreased metabolism causing cold c. Shame
intolerance d. Remorsefulness

15. Nurse Anna can minimize agitation in a 20. Which of the following approaches would be
disturbed client by? most appropriate to use with a client suffering
a. Increasing stimulation from narcissistic personality disorder when
b. limiting unnecessary interaction discrepancies exist between what the client
c. increasing appropriate sensory states and what actually exist?
perception a. Rationalization
d. ensuring constant client and staff b. Supportive confrontation
contact c. Limit setting
d. Consistency
16. A 39 year old mother with obsessive-compulsive
disorder has become immobilized by her 21. Cely is experiencing alcohol withdrawal exhibits
elaborate hand washing and walking rituals. tremors, diaphoresis and hyperactivity. Blood
Nurse Trish recognizes that the basis of O.C. pressure is 190/87 mmhg and pulse is 92 bpm.
disorder is often: Which of the medications would the nurse
a. Problems with being too conscientious expect to administer?
b. Problems with anger and remorse a. Naloxone (Narcan)
c. Feelings of guilt and inadequacy b. Benzlropine (Cogentin)
d. Feeling of unworthiness and c. Lorazepam (Ativan)
hopelessness d. Haloperidol (Haldol)

17. Mario is complaining to other clients about not 22. Which of the following foods would the nurse
being allowed by staff to keep food in his room. Trish eliminate from the diet of a client in
Which of the following interventions would be alcohol withdrawal?
most appropriate? a. Milk
a. Allowing a snack to be kept in his room b. Orange Juice
b. Reprimanding the client c. Soda
c. Ignoring the clients behavior d. Regular Coffee
d. Setting limits on the behavior
23. Which of the following would Nurse Hazel
18. Conney with borderline personality disorder who expect to assess for a client who is exhibiting
is to be discharge soon threatens to “do late signs of heroin withdrawal?
something” to herself if discharged. Which of the a. Yawning & diaphoresis

38
38
b. Restlessness & Irritability my best friend. The nurse recognizes that the
c. Constipation & steatorrhea client is using the defense mechanism known as?
d. Vomiting and Diarrhea a. Displacement
b. Projection
24. To establish open and trusting relationship with c. Sublimation
a female client who has been hospitalized with d. Denial
severe anxiety, the nurse in charge should?
a. Encourage the staff to have frequent 30. When working with a male client suffering
interaction with the client phobia about black cats, Nurse Trish should
b. Share an activity with the client anticipate that a problem for this client would
c. Give client feedback about behavior be?
d. Respect client’s need for personal space a. Anxiety when discussing phobia
b. Anger toward the feared object
25. Nurse Monette recognizes that the focus of c. Denying that the phobia exist
environmental (MILIEU) therapy is to: d. Distortion of reality when completing
a. Manipulate the environment to bring daily routines
about positive changes in behavior
b. Allow the client’s freedom to determine 31. Linda is pacing the floor and appears extremely
whether or not they will be involved in anxious. The duty nurse approaches in an
activities attempt to alleviate Linda’s anxiety. The most
c. Role play life events to meet individual therapeutic question by the nurse would be?
needs a. Would you like to watch TV?
d. Use natural remedies rather than drugs b. Would you like me to talk with you?
to control behavior c. Are you feeling upset now?
d. Ignore the client
26. Nurse Trish would expect a child with a diagnosis
of reactive attachment disorder to: 32. Nurse Penny is aware that the symptoms that
a. Have more positive relation with the distinguish post-traumatic stress disorder from
father than the mother other anxiety disorder would be:
b. Cling to mother & cry on separation a. Avoidance of situation & certain
c. Be able to develop only superficial activities that resemble the stress
relation with the others b. Depression and a blunted affect when
d. Have been physically abuse discussing the traumatic situation
c. Lack of interest in family & others
27. When teaching parents about childhood d. Re-experiencing the trauma in dreams or
depression Nurse Trina should say? flashback
a. It may appear acting out behavior
b. Does not respond to conventional 33. Nurse Benjie is communicating with a male client
treatment with substance-induced persisting dementia; the
c. Is short in duration & resolves easily client cannot remember facts and fills in the
d. Looks almost identical to adult gaps with imaginary information. Nurse Benjie is
depression aware that this is typical of?
a. Flight of ideas
28. Nurse Perry is aware that language development b. Associative looseness
in autistic child resembles: c. Confabulation
a. Scanning speech d. Concretism
b. Speech lag
c. Shuttering 34. Nurse Joey is aware that the signs & symptoms
d. Echolalia that would be most specific for diagnosis
anorexia are?
29. A 60 year old female client who lives alone tells a. Excessive weight loss, amenorrhea &
the nurse at the community health center “I abdominal distension
really don’t need anyone to talk to”. The TV is b. Slow pulse, 10% weight loss & alopecia
c. Compulsive behavior, excessive fears & detailed assessment, a diagnosis of
nausea schizophrenia is made. It is unlikely that the
d. Excessive activity, memory lapses & an client will demonstrate:
increased pulse a. Low self esteem
b. Concrete thinking
35. A characteristic that would suggest to Nurse c. Effective self-boundaries
Anne that an adolescent may have bulimia d. Weak ego
would be:
a. Frequent regurgitation & re-swallowing 41. A 23 year old client has been admitted with a
of food diagnosis of schizophrenia says to the nurse
b. Previous history of gastritis “Yes, its march, March is little woman”. That’s
c. Badly stained teeth literal you know”. These statement illustrate:
d. Positive body image a. Neologisms
b. Echolalia
36. Nurse Monette is aware that extremely c. Flight of ideas
depressed clients seem to do best in settings d. Loosening of association
where they have:
a. Multiple stimuli 42. A long term goal for a paranoid male client who
b. Routine Activities has unjustifiably accused his wife of having many
c. Minimal decision making extramarital affairs would be to help the client
d. Varied Activities develop:
a. Insight into his behavior
37. To further assess a client’s suicidal potential. b. Better self-control
Nurse Katrina should be especially alert to the c. Feeling of self-worth
client expression of: d. Faith in his wife
a. Frustration & fear of death
b. Anger & resentment 43. A male client who is experiencing disordered
c. Anxiety & loneliness thinking about food being poisoned is admitted
d. Helplessness & hopelessness to the mental health unit. The nurse uses which
communication technique to encourage the
38. A nursing care plan for a male client with bipolar client to eat dinner?
I disorder should include: a. Focusing on self-disclosure of own food
a. Providing a structured environment preference
b. Designing activities that will require the b. Using open ended question and silence
client to maintain contact with reality c. Offering opinion about the need to eat
c. Engaging the client in conversing about d. Verbalizing reasons that the client may
current affairs not choose to eat
d. Touching the client provide assurance
44. Nurse Nina is assigned to care for a client
39. When planning care for a female client using diagnosed with Catatonic Stupor. When Nurse
ritualistic behavior, Nurse Gina must recognize Nina enters the client’s room, the client is found
that the ritual: lying on the bed with a body pulled into a fetal
a. Helps the client focus on the inability to position. Nurse Nina should?
deal with reality a. Ask the client direct questions to
b. Helps the client control the anxiety encourage talking
c. Is under the client’s conscious control b. Rake the client into the dayroom to be
d. Is used by the client primarily for with other clients
secondary gains c. Sit beside the client in silence and
occasionally ask open-ended question
40. A 32 year old male graduate student, who has d. Leave the client alone and continue with
become increasingly withdrawn and neglectful providing care to the other clients
of his work and personal hygiene, is brought to
the psychiatric hospital by his parents. After

38
38
45. Nurse Tina is caring for a client with delirium and 49. Nurse Tina is caring for a client with depression
states that “look at the spiders on the wall”. who has not responded to antidepressant
What should the nurse respond to the client? medication. The nurse anticipates that what
a. “You’re having hallucination, there are treatment procedure may be prescribed.
no spiders in this room at all” a. Neuroleptic medication
b. “I can see the spiders on the wall, but b. Short term seclusion
they are not going to hurt you” c. Psychosurgery
c. “Would you like me to kill the spiders” d. Electroconvulsive therapy
d. “I know you are frightened, but I do not
see spiders on the wall” 50. Mario is admitted to the emergency room with
drug-included anxiety related to over ingestion
46. Nurse Jonel is providing information to a of prescribed antipsychotic medication. The
community group about violence in the family. most important piece of information the nurse
Which statement by a group member would in charge should obtain initially is the:
indicate a need to provide additional a. Length of time on the med.
information? b. Name of the ingested medication & the
a. “Abuse occurs more in low-income amount ingested
families” c. Reason for the suicide attempt
b. “Abuser Are often jealous or self- d. Name of the nearest relative & their
centered” phone number
c. “Abuser use fear and intimidation”
d. “Abuser usually have poor self-esteem”

47. During electroconvulsive therapy (ECT) the client


receives oxygen by mask via positive pressure
ventilation. The nurse assisting with this
procedure knows that positive pressure
ventilation is necessary because?
a. Anesthesia is administered during the
procedure
b. Decrease oxygen to the brain increases
confusion and disorientation
c. Grand mal seizure activity depresses
respirations
d. Muscle relaxations given to prevent
injury during seizure activity depress
respirations.

48. When planning the discharge of a client with


chronic anxiety, Nurse Chris evaluates
achievement of the discharge maintenance
goals. Which goal would be most appropriately
having been included in the plan of care
requiring evaluation?
a. The client eliminates all anxiety from
daily situations
b. The client ignores feelings of anxiety
c. The client identifies anxiety producing
situations
d. The client maintains contact with a crisis
counselor
ANSWERS AND RATIONALE – PSYCHIATRIC Rationale: With depression, there is little or no
NURSING emotional involvement therefore little alteration
in affect.
1. Answer: C 13. Answer: D
Rationale: Total abstinence is the only effective Rationale: These clients often hide food or force
treatment for alcoholism vomiting; therefore they must be carefully
2. Answer: A monitored.
Rationale: Hallucinations are visual, auditory, 14. Answer: A
gustatory, tactile or olfactory perceptions that Rationale: These clients have severely depleted
have no basis in reality. levels of sodium and potassium because of their
3. Answer: D starvation diet and energy expenditure, these
Rationale: The Nurse has a responsibility to electrolytes are necessary for cardiac
observe continuously the acutely suicidal client. functioning.
The Nurse should watch for clues, such as 15. Answer: B
communicating suicidal thoughts, and messages; Rationale: Limiting unnecessary interaction will
hoarding medications and talking about death. decrease stimulation and agitation.
4. Answer: B 16. Answer: C
Rationale: Establishing a consistent eating plan Rationale: Ritualistic behavior seen in this
and monitoring client’s weight are important to disorder is aimed at controlling guilt and
this disorder. inadequacy by maintaining an absolute set
5. Answer: C pattern of behavior.
Rationale: Appropriate nursing interventions for 17. Answer: D
an anxiety attack include using short sentences, Rationale: The nurse needs to set limits in the
staying with the client, decreasing stimuli, client’s manipulative behavior to help the client
remaining calm and medicating as needed. control dysfunctional behavior. A consistent
6. Answer:B approach by the staff is necessary to decrease
Rationale: Delusion of grandeur is a false belief manipulation.
that one is highly famous and important. 18. Answer: B
7. Answer: D Rationale: Any suicidal statement must be
Rationale: Individual with dependent personality assessed by the nurse. The nurse should discuss
disorder typically shows indecisiveness the client’s statement with her to determine its
submissiveness and clinging behavior so that meaning in terms of suicide.
others will make decisions with them. 19. Answer: A
8. Answer: A Rationale: When the staff member ask the client
Rationale: Clients with schizotypal personality if he wonders why others find him repulsive, the
disorder experience excessive social anxiety that client is likely to feel defensive because the
can lead to paranoid thoughts question is belittling. The natural tendency is to
9. Answer: B counterattack the threat to self-image.
Rationale: Bulimia disorder generally is a 20. Answer: B
maladaptive coping response to stress and Rationale: The nurse would specifically use
underlying issues. The client should identify supportive confrontation with the client to point
anxiety causing situation that stimulate the out discrepancies between what the client states
bulimic behavior and then learn new ways of and what actually exists to increase
coping with the anxiety. responsibility for self.
10. Answer: A 21. Answer: C
Rationale: An adult age 31 to 45 generates new Rationale: The nurse would most likely
level of awareness. administer benzodiazepine, such as lorazepan
11. Answer: A (ativan) to the client who is experiencing
Rationale: Neuromuscular Blocker, such as symptom: The client’s experiences symptoms of
SUCCINYLCHOLINE (Anectine) produces withdrawal because of the rebound
respiratory depression because it inhibits phenomenon when the sedation of the CNS
contractions of respiratory muscles. from alcohol begins to decrease.
12. Answer: C 22. Answer: D

38
38
Rationale: Regular coffee contains caffeine Rationale: These are the major signs of anorexia
which acts as psychomotor stimulants and leads nervosa. Weight loss is excessive (15% of
to feelings of anxiety and agitation. Serving expected weight)
coffee top the client may add to tremors or 35. Answer: C
wakefulness. Rationale: Dental enamel erosion occurs from
23. Answer: D repeated self-induced vomiting.
Rationale: Vomiting and diarrhea are usually the 36. Answer: B
late signs of heroin withdrawal, along with Rationale: Depression usually is both emotional
muscle spasm, fever, nausea, repetitive, & physical. A simple daily routine is the best,
abdominal cramps and backache. least stressful and least anxiety producing.
24. Answer: D 37. Answer: D
Rationale: Moving to a client’s personal space Rationale: The expression of these feeling may
increases the feeling of threat, which increases indicate that this client is unable to continue the
anxiety. struggle of life.
25. Answer: A 38. Answer: A
Rationale: Environmental (MILIEU) therapy aims Rationale: Structure tends to decrease agitation
at having everything in the client’s surrounding and anxiety and to increase the client’s feeling of
area toward helping the client. security.
26. Answer: C 39. Answer: B
Rationale: Children who have experienced Rationale: The rituals used by a client with
attachment difficulties with primary caregiver obsessive compulsive disorder help control the
are not able to trust others and therefore relate anxiety level by maintaining a set pattern of
superficially action.
27. Answer: A 40. Answer: C
Rationale: Children have difficulty verbally Rationale: A person with this disorder would not
expressing their feelings, acting out behavior, have adequate self-boundaries
such as temper tantrums, may indicate 41. Answer: D
underlying depression. Rationale: Loose associations are thoughts that
28. Answer: D are presented without the logical connections
Rationale: The autistic child repeats sounds or usually necessary for the listening to interpret
words spoken by others. the message.
29. Answer: D 42. Answer: C
Rationale: The client statement is an example of Rationale: Helping the client to develop feeling
the use of denial, a defense that blocks problem of self-worth would reduce the client’s need to
by unconscious refusing to admit they exist use pathologic defenses.
30. Answer: A 43. Answer: B
Rationale: Discussion of the feared object Rationale: Open ended questions and silence are
triggers an emotional response to the object. strategies used to encourage clients to discuss
31. Answer: B their problem in descriptive manner.
Rationale: The nurse presence may provide the 44. Answer: C
client with support & feeling of control. Rationale: Clients who are withdrawn may be
32. Answer: D immobile and mute, and require consistent,
Rationale: Experiencing the actual trauma in repeated interventions. Communication with
dreams or flashback is the major symptom that withdrawn clients requires much patience from
distinguishes post-traumatic stress disorder from the nurse. The nurse facilitates communication
other anxiety disorder. with the client by sitting in silence, asking open-
33. Answer: C ended question and pausing to provide
Rationale: Confabulation or the filling in of opportunities for the client to respond.
memory gaps with imaginary facts is a defense 45. Answer: D
mechanism used by people experiencing Rationale: When hallucination is present, the
memory deficits. nurse should reinforce reality with the client.
34. Answer: A 46. Answer: A
Rationale: Personal characteristics of abuser
include low self-esteem, immaturity,
dependence, insecurity and jealousy.
47. Answer: D
Rationale: A short acting skeletal muscle relaxant
such as succinylcholine (Anectine) is
administered during this procedure to prevent
injuries during seizure.
48. Answer: C
Rationale: Recognizing situations that produce
anxiety allows the client to prepare to cope with
anxiety or avoid specific stimulus.
49. Answer: D
Rationale: Electroconvulsive therapy is an
effective treatment for depression that has not
responded to medication
50. Answer: B
Rationale: In an emergency, lives saving facts are
obtained first. The name and the amount of
medication ingested are of outmost important in
treating this potentially life threatening
situation.

38
38
FUNDAMENTALS OF NURSING PART 1 a. Keep unnecessary furniture out of the
way
1. Using the principles of standard precautions, the b. Keep the lights on at all time
nurse would wear gloves in what nursing c. Keep side rails up at all time
interventions? d. Keep all equipment out of view
a. Providing a back massage 8. A walk-in client enters into the clinic with a chief
b. Feeding a client complaint of abdominal pain and diarrhea. The
c. Providing hair care nurse takes the client’s vital sign hereafter. What
d. Providing oral hygiene phrase of nursing process is being implemented
2. The nurse is preparing to take vital sign in an here by the nurse?
alert client admitted to the hospital with a. Assessment
dehydration secondary to vomiting and diarrhea. b. Diagnosis
What is the best method used to assess the c. Planning
client’s temperature? d. Implementation
a. Oral 9. It is best describe as a systematic, rational
b. Axillary method of planning and providing nursing care
c. Radial for individual, families, group and community
d. Heat sensitive tape a. Assessment
3. A nurse obtained a client’s pulse and found the b. Nursing Process
rate to be above normal. The nurse document c. Diagnosis
this findings as: d. Implementation
a. Tachypnea 10. Exchange of gases takes place in which of the
b. Hyper pyrexia following organ?
c. Arrythmia a. Kidney
d. Tachycardia b. Lungs
4. Which of the following actions should the nurse c. Liver
take to use a wide base support when assisting a d. Heart
client to get up in a chair? 11. The Chamber of the heart that receives
a. Bend at the waist and place arms under oxygenated blood from the lungs is the?
the client’s arms and lift a. Left atrium
b. Face the client, bend knees and place b. Right atrium
hands on client’s forearm and lift c. Left ventricle
c. Spread his or her feet apart d. Right ventricle
d. Tighten his or her pelvic muscles 12. A muscular enlarge pouch or sac that lies slightly
5. A client had oral surgery following a motor to the left which is used for temporary storage
vehicle accident. The nurse assessing the client of food…
finds the skin flushed and warm. Which of the a. Gallbladder
following would be the best method to take the b. Urinary bladder
client’s body temperature? c. Stomach
a. Oral d. Lungs
b. Axillary 13. The ability of the body to defend itself against
c. Arterial line scientific invading agent such as baceria, toxin,
d. Rectal viruses and foreign body
6. A client who is unconscious needs frequent a. Hormones
mouth care. When performing a mouth care, the b. Secretion
best position of a client is: c. Immunity
a. Fowler’s position d. Glands
b. Side lying 14. Hormones secreted by Islets of Langerhans
c. Supine a. Progesterone
d. Trendelenburg b. Testosterone
7. A client is hospitalized for the first time, which of c. Insulin
the following actions ensure the safety of the d. Hemoglobin
client?
15. It is a transparent membrane that focuses the d. Dr.
light that enters the eyes to the retina. 24. The abbreviation for micro drop is…
a. Lens a. µgtt
b. Sclera b. gtt
c. Cornea c. mdr
d. Pupils d. mgts
16. Which of the following is included in Orem’s 25. Which of the following is the meaning of PRN?
theory? a. When advice
a. Maintenance of a sufficient intake of air b. Immediately
b. Self perception c. When necessary
c. Love and belonging d. Now
d. Physiologic needs 26. Which of the following is the appropriate
17. Which of the following cluster of data belong to meaning of CBR?
Maslow’s hierarchy of needs a. Cardiac Board Room
a. Love and belonging b. Complete Bathroom
b. Physiologic needs c. Complete Bed Rest
c. Self actualization d. Complete Board Room
d. All of the above 27. 1 tsp is equals to how many drops?
18. This is characterized by severe symptoms a. 15
relatively of short duration. b. 60
a. Chronic Illness c. 10
b. Acute Illness d. 30
c. Pain 28. 20 cc is equal to how many ml?
d. Syndrome a. 2
19. Which of the following is the nurse’s role in the b. 20
health promotion c. 2000
a. Health risk appraisal d. 20000
b. Teach client to be effective health 29. 1 cup is equal to how many ounces?
consumer a. 8
c. Worksite wellness b. 80
d. None of the above c. 800
20. It is describe as a collection of people who share d. 8000
some attributes of their lives. 30. The nurse must verify the client’s identity before
a. Family administration of medication. Which of the
b. Illness following is the safest way to identify the client?
c. Community a. Ask the client his name
d. Nursing b. Check the client’s identification band
21. Five teaspoon is equivalent to how many c. State the client’s name aloud and have
milliliters (ml)? the client repeat it
a. 30 ml d. Check the room number
b. 25 ml 31. The nurse prepares to administer buccal
c. 12 ml medication. The medicine should be placed…
d. 75 ml a. On the client’s skin
22. 1800 ml is equal to how many liters? b. Between the client’s cheeks and gums
a. 1.8 c. Under the client’s tongue
b. 18000 d. On the client’s conjuctiva
c. 180 32. The nurse administers cleansing enema. The
d. 2800 common position for this procedure is…
23. Which of the following is the abbreviation of a. Sims left lateral
drops? b. Dorsal Recumbent
a. Gtt. c. Supine
b. Gtts. d. Prone
c. Dp.

38
38
33. A client complains of difficulty of swallowing, a. To cleanse, refresh and give comfort to
when the nurse try to administer capsule the client who must remain in bed
medication. Which of the following measures b. To expose the necessary parts of the
the nurse should do? body
a. Dissolve the capsule in a glass of water c. To develop skills in bed bath
b. Break the capsule and give the content d. To check the body temperature of the
with an applesauce client in bed
c. Check the availability of a liquid 41. Which of the following technique involves the
preparation sense of sight?
d. Crash the capsule and place it under the a. Inspection
tongue b. Palpation
34. Which of the following is the appropriate route c. Percussion
of administration for insulin? d. Auscultation
a. Intramuscular 42. The first techniques used examining the
b. Intradermal abdomen of a client is:
c. Subcutaneous a. Palpation
d. Intravenous b. Auscultation
35. The nurse is ordered to administer ampicillin c. Percussion
capsule TIP p.o. The nurse shoud give the d. Inspection
medication… 43. A technique in physical examination that is use
a. Three times a day orally to assess the movement of air through the
b. Three times a day after meals tracheobronchial tree:
c. Two time a day by mouth a. Palpation
d. Two times a day before meals b. Auscultation
36. Back Care is best describe as: c. Inspection
a. Caring for the back by means of massage d. Percussion
b. Washing of the back 44. An instrument used for auscultation is:
c. Application of cold compress at the back a. Percussion-hammer
d. Application of hot compress at the back b. Audiometer
37. It refers to the preparation of the bed with a c. Stethoscope
new set of linens d. Sphygmomanometer
a. Bed bath 45. Resonance is best describe as:
b. Bed making a. Sounds created by air filled lungs
c. Bed shampoo b. Short, high pitch and thudding
d. Bed lining c. Moderately loud with musical quality
38. Which of the following is the most important d. Drum-like
purpose of handwashing 46. The best position for examining the rectum is:
a. To promote hand circulation a. Prone
b. To prevent the transfer of b. Sim’s
microorganism c. Knee-chest
c. To avoid touching the client with a dirty d. Lithotomy
hand 47. It refers to the manner of walking
d. To provide comfort a. Gait
39. What should be done in order to prevent b. Range of motion
contaminating of the environment in bed c. Flexion and extension
making? d. Hopping
a. Avoid funning soiled linens 48. The nurse asked the client to read the Snellen
b. Strip all linens at the same time chart. Which of the following is tested:
c. Finished both sides at the time a. Optic
d. Embrace soiled linen b. Olfactory
40. The most important purpose of cleansing bed c. Oculomotor
bath is: d. Troclear
49. Another name for knee-chest position is:
a. Genu-dorsal
b. Genu-pectoral
c. Lithotomy
d. Sim’s
50. The nurse prepares IM injection that is irritating
to the subcutaneous tissue. Which of the
following is the best action in order to prevent
tracking of the medication
a. Use a small gauge needle
b. Apply ice on the injection site
c. Administer at a 45° angle
d. Use the Z-track technique

39
391

1.d 11.a 21.b 31.b 41.a


2.b 12.c 22.a 32.a 42.d
3.d 13.c 23.b 33.c 43.b
4b 14.c 24.a 34.c 44.c
5.b 15.c 25.c 35.a 45.a
6.b 16.a 26.c 36.a 46.c
7.c 17.d 27.b 37.b 47.a
8.a 18.b 28.b 38.b 48.a
9.b 19.b 29.a 39.a 49.b
10.b 20.c 30.a 40.a 50.d
6. The four main concepts common to nursing that
appear in each of the current conceptual models
FUNDAMENTALS OF NURSING PART 2 are:
a. Person, nursing, environment, medicine
1. The most appropriate nursing order for a patient b. Person, health, nursing, support systems
who develops dyspnea and shortness of breath c. Person, health, psychology, nursing
would be… d. Person, environment, health, nursing
a. Maintain the patient on strict bed rest at 7. In Maslow’s hierarchy of physiologic needs, the
all times human need of greatest priority is:
b. Maintain the patient in an orthopneic a. Love
position as needed b. Elimination
c. Administer oxygen by Venturi mask at c. Nutrition
24%, as needed d. Oxygen
d. Allow a 1 hour rest period between 8. The family of an accident victim who has been
activities declared brain-dead seems amenable to organ
2. The nurse observes that Mr. Adams begins to donation. What should the nurse do?
have increased difficulty breathing. She elevates a. Discourage them from making a decision
the head of the bed to the high Fowler position, until their grief has eased
which decreases his respiratory distress. The b. Listen to their concerns and answer their
nurse documents this breathing as: questions honestly
a. Tachypnea c. Encourage them to sign the consent
b. Eupnca form right away
c. Orthopnea d. Tell them the body will not be available
d. Hyperventilation for a wake or funeral
3. The physician orders a platelet count to be 9. A new head nurse on a unit is distressed about
performed on Mrs. Smith after breakfast. The the poor staffing on the 11 p.m. to 7 a.m. shift.
nurse is responsible for: What should she do?
a. Instructing the patient about this a. Complain to her fellow nurses
diagnostic test b. Wait until she knows more about the
b. Writing the order for this test unit
c. Giving the patient breakfast c. Discuss the problem with her supervisor
d. All of the above d. Inform the staff that they must
4. Mrs. Mitchell has been given a copy of her diet. volunteer to rotate
The nurse discusses the foods allowed on a 500- 10. Which of the following principles of primary
mg low sodium diet. These include: nursing has proven the most satisfying to the
a. A ham and Swiss cheese sandwich on patient and nurse?
whole wheat bread a. Continuity of patient care promotes
b. Mashed potatoes and broiled chicken efficient, cost-effective nursing care
c. A tossed salad with oil and vinegar and b. Autonomy and authority for planning
olives are best delegated to a nurse who
d. Chicken bouillon knows the patient well
5. The physician orders a maintenance dose of c. Accountability is clearest when one
5,000 units of subcutaneous heparin (an nurse is responsible for the overall plan
anticoagulant) daily. Nursing responsibilities for and its implementation.
Mrs. Mitchell now include: d. The holistic approach provides for a
a. Reviewing daily activated partial therapeutic relationship, continuity, and
thromboplastin time (APTT) and efficient nursing care.
prothrombin time. 11. If nurse administers an injection to a patient
b. Reporting an APTT above 45 seconds to who refuses that injection, she has committed:
the physician a. Assault and battery
c. Assessing the patient for signs and b. Negligence
symptoms of frank and occult bleeding c. Malpractice
d. All of the above
39
39
d. None of the above would immediately alert the nurse that the
12. If patient asks the nurse her opinion about a patient has bleeding from the GI tract?
particular physicians and the nurse replies that a. Complete blood count
the physician is incompetent, the nurse could be b. Guaiac test
held liable for: c. Vital signs
a. Slander d. Abdominal girth
b. Libel 17. The correct sequence for assessing the abdomen
c. Assault is:
d. Respondent superior a. Tympanic percussion, measurement of
13. A registered nurse reaches to answer the abdominal girth, and inspection
telephone on a busy pediatric unit, momentarily b. Assessment for distention, tenderness,
turning away from a 3 month-old infant she has and discoloration around the umbilicus.
been weighing. The infant falls off the scale, c. Percussions, palpation, and auscultation
suffering a skull fracture. The nurse could be d. Auscultation, percussion, and palpation
charged with: 18. High-pitched gurgles head over the right lower
a. Defamation quadrant are:
b. Assault a. A sign of increased bowel motility
c. Battery b. A sign of decreased bowel motility
d. Malpractice c. Normal bowel sounds
14. Which of the following is an example of nursing d. A sign of abdominal cramping
malpractice? 19. A patient about to undergo abdominal
a. The nurse administers penicillin to a inspection is best placed in which of the
patient with a documented history of following positions?
allergy to the drug. The patient a. Prone
experiences an allergic reaction and has b. Trendelenburg
cerebral damage resulting from anoxia. c. Supine
b. The nurse applies a hot water bottle or a d. Side-lying
heating pad to the abdomen of a patient 20. For a rectal examination, the patient can be
with abdominal cramping. directed to assume which of the following
c. The nurse assists a patient out of bed positions?
with the bed locked in position; the a. Genupecterol
patient slips and fractures his right b. Sims
humerus. c. Horizontal recumbent
d. The nurse administers the wrong d. All of the above
medication to a patient and the patient 21. During a Romberg test, the nurse asks the
vomits. This information is documented patient to assume which position?
and reported to the physician and the a. Sitting
nursing supervisor. b. Standing
15. Which of the following signs and symptoms c. Genupectoral
would the nurse expect to find when assessing d. Trendelenburg
an Asian patient for postoperative pain following 22. If a patient’s blood pressure is 150/96, his pulse
abdominal surgery? pressure is:
a. Decreased blood pressure and heart a. 54
rate and shallow respirations b. 96
b. Quiet crying c. 150
c. Immobility, diaphoresis, and avoidance d. 246
of deep breathing or coughing 23. A patient is kept off food and fluids for 10 hours
d. Changing position every 2 hours before surgery. His oral temperature at 8 a.m. is
16. A patient is admitted to the hospital with 99.8 F (37.7 C) This temperature reading
complaints of nausea, vomiting, diarrhea, and probably indicates:
severe abdominal pain. Which of the following a. Infection
b. Hypothermia

39
39
c. Anxiety following nursing diagnosis: Impaired gas
d. Dehydration exchange related to increased secretions. Which
24. Which of the following parameters should be of the following nursing interventions has the
checked when assessing respirations? greatest potential for improving this situation?
a. Rate a. Encourage the patient to increase her
b. Rhythm fluid intake to 200 ml every 2 hours
c. Symmetry b. Place a humidifier in the patient’s room.
d. All of the above c. Continue administering oxygen by high
25. A 38-year old patient’s vital signs at 8 a.m. are humidity face mask
axillary temperature 99.6 F (37.6 C); pulse rate, d. Perform chest physiotheraphy on a
88; respiratory rate, 30. Which findings should regular schedule
be reported? 31. The most common deficiency seen in alcoholics
a. Respiratory rate only is:
b. Temperature only a. Thiamine
c. Pulse rate and temperature b. Riboflavin
d. Temperature and respiratory rate c. Pyridoxine
26. All of the following can cause tachycardia d. Pantothenic acid
except: 32. Which of the following statement is incorrect
a. Fever about a patient with dysphagia?
b. Exercise a. The patient will find pureed or soft
c. Sympathetic nervous system stimulation foods, such as custards, easier to
d. Parasympathetic nervous system swallow than water
stimulation b. Fowler’s or semi Fowler’s position
27. Palpating the midclavicular line is the correct reduces the risk of aspiration during
technique for assessing swallowing
a. Baseline vital signs c. The patient should always feed himself
b. Systolic blood pressure d. The nurse should perform oral hygiene
c. Respiratory rate before assisting with feeding.
d. Apical pulse 33. To assess the kidney function of a patient with
28. The absence of which pulse may not be a an indwelling urinary (Foley) catheter, the nurse
significant finding when a patient is admitted to measures his hourly urine output. She should
the hospital? notify the physician if the urine output is:
a. Apical a. Less than 30 ml/hour
b. Radial b. 64 ml in 2 hours
c. Pedal c. 90 ml in 3 hours
d. Femoral d. 125 ml in 4 hours
29. Which of the following patients is at greatest risk 34. Certain substances increase the amount of urine
for developing pressure ulcers? produced. These include:
a. An alert, chronic arthritic patient treated a. Caffeine-containing drinks, such as
with steroids and aspirin coffee and cola.
b. An 88-year old incontinent patient with b. Beets
gastric cancer who is confined to his bed c. Urinary analgesics
at home d. Kaolin with pectin (Kaopectate)
c. An apathetic 63-year old COPD patient 35. A male patient who had surgery 2 days ago for
receiving nasal oxygen via cannula head and neck cancer is about to make his first
d. A confused 78-year old patient with attempt to ambulate outside his room. The
congestive heart failure (CHF) who nurse notes that he is steady on his feet and
requires assistance to get out of bed. that his vision was unaffected by the surgery.
30. The physician orders the administration of high- Which of the following nursing interventions
humidity oxygen by face mask and placement of would be appropriate?
the patient in a high Fowler’s position. After a. Encourage the patient to walk in the hall
assessing Mrs. Paul, the nurse writes the alone

39
39
b. Discourage the patient from walking in a. Asses the patient’s ability to ambulate
the hall for a few more days and transfer from a bed to a chair
c. Accompany the patient for his walk. b. Demonstrate the signal system to the
d. Consuit a physical therapist before patient
allowing the patient to ambulate c. Check to see that the patient is wearing
36. A patient has exacerbation of chronic his identification band
obstructive pulmonary disease (COPD) d. All of the above
manifested by shortness of breath; orthopnea: 42. Studies have shown that about 40% of patients
thick, tenacious secretions; and a dry hacking fall out of bed despite the use of side rails; this
cough. An appropriate nursing diagnosis would has led to which of the following conclusions?
be: a. Side rails are ineffective
a. Ineffective airway clearance related to b. Side rails should not be used
thick, tenacious secretions. c. Side rails are a deterrent that prevent a
b. Ineffective airway clearance related to patient from falling out of bed.
dry, hacking cough. d. Side rails are a reminder to a patient not
c. Ineffective individual coping to COPD. to get out of bed
d. Pain related to immobilization of 43. Examples of patients suffering from impaired
affected leg. awareness include all of the following
37. Mrs. Lim begins to cry as the nurse discusses hair except:
loss. The best response would be: a. A semiconscious or over fatigued patient
a. “Don’t worry. It’s only temporary” b. A disoriented or confused patient
b. “Why are you crying? I didn’t get to the c. A patient who cannot care for himself at
bad news yet” home
c. “Your hair is really pretty” d. A patient demonstrating symptoms of
d. “I know this will be difficult for you, but drugs or alcohol withdrawal
your hair will grow back after the 44. The most common injury among elderly persons
completion of chemotheraphy” is:
38. An additional Vitamin C is required during all of a. Atheroscleotic changes in the blood
the following periods except: vessels
a. Infancy b. Increased incidence of gallbladder
b. Young adulthood disease
c. Childhood c. Urinary Tract Infection
d. Pregnancy d. Hip fracture
39. A prescribed amount of oxygen s needed for a 45. The most common psychogenic disorder among
patient with COPD to prevent: elderly person is:
a. Cardiac arrest related to increased a. Depression
partial pressure of carbon dioxide in b. Sleep disturbances (such as bizarre
arterial blood (PaCO2) dreams)
b. Circulatory overload due to c. Inability to concentrate
hypervolemia d. Decreased appetite
c. Respiratory excitement 46. Which of the following vascular system changes
d. Inhibition of the respiratory hypoxic results from aging?
stimulus a. Increased peripheral resistance of the
40. After 1 week of hospitalization, Mr. Gray blood vessels
develops hypokalemia. Which of the following is b. Decreased blood flow
the most significant symptom of his disorder? c. Increased work load of the left ventricle
a. Lethargy d. All of the above
b. Increased pulse rate and blood pressure 47. Which of the following is the most common
c. Muscle weakness cause of dementia among elderly persons?
d. Muscle irritability a. Parkinson’s disease
41. Which of the following nursing interventions b. Multiple sclerosis
promotes patient safety? c. Amyotrophic lateral sclerosis (Lou
Gerhig’s disease)
39
39
d. Alzheimer’s disease
48. The nurse’s most important legal responsibility
after a patient’s death in a hospital is:
a. Obtaining a consent of an autopsy
b. Notifying the coroner or medical
examiner
c. Labeling the corpse appropriately
d. Ensuring that the attending physician
issues the death certification
49. Before rigor mortis occurs, the nurse is
responsible for:
a. Providing a complete bath and dressing
change
b. Placing one pillow under the body’s
head and shoulders
c. Removing the body’s clothing and
wrapping the body in a shroud
d. Allowing the body to relax normally
50. When a patient in the terminal stages of lung
cancer begins to exhibit loss of consciousness, a
major nursing priority is to:
a. Protect the patient from injury
b. Insert an airway
c. Elevate the head of the bed
d. Withdraw all pain medications

39
39
ANSWERS and RATIONALES for FUNDAMENTALS OF continuum, and the nursing actions necessary to
NURSING PART 2 meet his needs.
7. D. Maslow, who defined a need as a satisfaction
1. B. When a patient develops dyspnea and whose absence causes illness, considered
shortness of breath, the orthopneic position oxygen to be the most important physiologic
encourages maximum chest expansion and need; without it, human life could not exist.
keeps the abdominal organs from pressing According to this theory, other physiologic needs
against the diaphragm, thus improving (including food, water, elimination, shelter, rest
ventilation. Bed rest and oxygen by Venturi mask and sleep, activity and temperature regulation)
at 24% would improve oxygenation of the must be met before proceeding to the next
tissues and cells but must be ordered by a hierarchical levels on psychosocial needs.
physician. Allowing for rest periods decreases 8. B. The brain-dead patient’s family needs support
the possibility of hypoxia. and reassurance in making a decision about
2. C. Orthopnea is difficulty of breathing except in organ donation. Because transplants are done
the upright position. Tachypnea is rapid within hours of death, decisions about organ
respiration characterized by quick, shallow donation must be made as soon as possible.
breaths. Eupnea is normal respiration – quiet, However, the family’s concerns must be
rhythmic, and without effort. addressed before members are asked to sign a
3. C. A platelet count evaluates the number of consent form. The body of an organ donor is
platelets in the circulating blood volume. The available for burial.
nurse is responsible for giving the patient 9. C. Although a new head nurse should initially
breakfast at the scheduled time. The physician is spend time observing the unit for its strengths
responsible for instructing the patient about the and weakness, she should take action if a
test and for writing the order for the test. problem threatens patient safety. In this case,
4. B. Mashed potatoes and broiled chicken are low the supervisor is the resource person to
in natural sodium chloride. Ham, olives, and approach.
chicken bouillon contain large amounts of 10. D. Studies have shown that patients and nurses
sodium and are contraindicated on a low sodium both respond well to primary nursing care units.
diet. Patients feel less anxious and isolated and more
5. D. All of the identified nursing responsibilities secure because they are allowed to participate
are pertinent when a patient is receiving in planning their own care. Nurses feel personal
heparin. The normal activated partial satisfaction, much of it related to positive
thromboplastin time is 16 to 25 seconds and the feedback from the patients. They also seem to
normal prothrombin time is 12 to 15 seconds; gain a greater sense of achievement and esprit
these levels must remain within two to two and de corps.
one half the normal levels. All patients receiving 11. A. Assault is the unjustifiable attempt or threat
anticoagulant therapy must be observed for to touch or injure another person. Battery is the
signs and symptoms of frank and occult bleeding unlawful touching of another person or the
(including hemorrhage, hypotension, carrying out of threatened physical harm. Thus,
tachycardia, tachypnea, restlessness, pallor, cold any act that a nurse performs on the patient
and clammy skin, thirst and confusion); blood against his will is considered assault and battery.
pressure should be measured every 4 hours and 12. A. Oral communication that injures an
the patient should be instructed to report individual’s reputation is considered slander.
promptly any bleeding that occurs with tooth Written communication that does the same is
brushing, bowel movements, urination or heavy considered libel.
prolonged menstruation. 13. D. Malpractice is defined as injurious or
6. D. The focus concepts that have been accepted unprofessional actions that harm another. It
by all theorists as the focus of nursing practice involves professional misconduct, such as
from the time of Florence Nightingale include omission or commission of an act that a
the person receiving nursing care, his reasonable and prudent nurse would or would
environment, his health on the health illness not do. In this example, the standard of care was

39
39
breached; a 3-month-old infant should never be cramping with hyperactive, high pitched tinkling
left unattended on a scale. bowel sounds can indicate a bowel obstruction.
14. A. The three elements necessary to establish a 19. C. The supine position (also called the dorsal
nursing malpractice are nursing error position), in which the patient lies on his back
(administering penicillin to a patient with a with his face upward, allows for easy access to
documented allergy to the drug), injury (cerebral the abdomen. In the prone position, the patient
damage), and proximal cause (administering the lies on his abdomen with his face turned to the
penicillin caused the cerebral damage). Applying side. In the Trendelenburg position, the head of
a hot water bottle or heating pad to a patient the bed is tilted downward to 30 to 40 degrees
without a physician’s order does not include the so that the upper body is lower than the legs. In
three required components. Assisting a patient the lateral position, the patient lies on his side.
out of bed with the bed locked in position is the 20. D. All of these positions are appropriate for a
correct nursing practice; therefore, the fracture rectal examination. In the genupectoral (knee-
was not the result of malpractice. Administering chest) position, the patient kneels and rests his
an incorrect medication is a nursing error; chest on the table, forming a 90 degree angle
however, if such action resulted in a serious between the torso and upper legs. In Sims’
illness or chronic problem, the nurse could be position, the patient lies on his left side with the
sued for malpractice. left arm behind the body and his right leg flexed.
15. C. An Asian patient is likely to hide his pain. In the horizontal recumbent position, the patient
Consequently, the nurse must observe for lies on his back with legs extended and hips
objective signs. In an abdominal surgery patient, rotated outward.
these might include immobility, diaphoresis, and 21. B. During a Romberg test, which evaluates for
avoidance of deep breathing or coughing, as well sensory or cerebellar ataxia, the patient must
as increased heart rate, shallow respirations stand with feet together and arms resting at the
(stemming from pain upon moving the sides—first with eyes open, then with eyes
diaphragm and respiratory muscles), and closed. The need to move the feet apart to
guarding or rigidity of the abdominal wall. Such a maintain this stance is an abnormal finding.
patient is unlikely to display emotion, such as 22. A. The pulse pressure is the difference between
crying. the systolic and diastolic blood pressure readings
16. B. To assess for GI tract bleeding when frank – in this case, 54.
blood is absent, the nurse has two options: She 23. D. A slightly elevated temperature in the
can test for occult blood in vomitus, if present, immediate preoperative or post operative
or in stool – through guaiac (Hemoccult) test. A period may result from the lack of fluids before
complete blood count does not provide surgery rather than from infection. Anxiety will
immediate results and does not always not cause an elevated temperature.
immediately reflect blood loss. Changes in vital Hypothermia is an abnormally low body
signs may be cause by factors other than blood temperature.
loss. Abdominal girth is unrelated to blood loss. 24. D. The quality and efficiency of the respiratory
17. D. Because percussion and palpation can affect process can be determined by appraising the
bowel motility and thus bowel sounds, they rate, rhythm, depth, ease, sound, and symmetry
should follow auscultation in abdominal of respirations.
assessment. Tympanic percussion, measurement 25. D. Under normal conditions, a healthy adult
of abdominal girth, and inspection are methods breathes in a smooth uninterrupted pattern 12
of assessing the abdomen. Assessing for to 20 times a minute. Thus, a respiratory rate of
distention, tenderness and discoloration around 30 would be abnormal. A normal adult body
the umbilicus can indicate various bowel-related temperature, as measured on an oral
conditions, such as cholecystitis, appendicitis thermometer, ranges between 97° and 100°F
and peritonitis. (36.1° and 37.8°C); an axillary temperature is
18. C. Hyperactive sounds indicate increased bowel approximately one degree lower and a rectal
motility; two or three sounds per minute temperature, one degree higher. Thus, an
indicate decreased bowel motility. Abdominal axillary temperature of 99.6°F (37.6°C) would be
considered abnormal. The resting pulse rate in

39
39
an adult ranges from 60 to 100 beats/minute, so 34. A. Fluids containing caffeine have a diuretic
a rate of 88 is normal. effect. Beets and urinary analgesics, such as
26. D. Parasympathetic nervous system stimulation pyridium, can color urine red. Kaopectate is an
of the heart decreases the heart rate as well as anti diarrheal medication.
the force of contraction, rate of impulse 35. C. A hospitalized surgical patient leaving his
conduction and blood flow through the coronary room for the first time fears rejection and others
vessels. Fever, exercise, and sympathetic staring at him, so he should not walk alone.
stimulation all increase the heart rate. Accompanying him will offer moral support,
27. D. The apical pulse (the pulse at the apex of the enabling him to face the rest of the world.
heart) is located on the midclavicular line at the Patients should begin ambulation as soon as
fourth, fifth, or sixth intercostal space. Base line possible after surgery to decrease complications
vital signs include pulse rate, temperature, and to regain strength and confidence. Waiting
respiratory rate, and blood pressure. Blood to consult a physical therapist is unnecessary.
pressure is typically assessed at the antecubital 36. A. Thick, tenacious secretions, a dry, hacking
fossa, and respiratory rate is assessed best by cough, orthopnea, and shortness of breath are
observing chest movement with each inspiration signs of ineffective airway clearance. Ineffective
and expiration. airway clearance related to dry, hacking cough is
28. C. Because the pedal pulse cannot be detected incorrect because the cough is not the reason for
in 10% to 20% of the population, its absence is the ineffective airway clearance. Ineffective
not necessarily a significant finding. However, individual coping related to COPD is wrong
the presence or absence of the pedal pulse because the etiology for a nursing diagnosis
should be documented upon admission so that should not be a medical diagnosis (COPD) and
changes can be identified during the hospital because no data indicate that the patient is
stay. Absence of the apical, radial, or femoral coping ineffectively. Pain related to
pulse is abnormal and should be investigated. immobilization of affected leg would be an
29. B. Pressure ulcers are most likely to develop in appropriate nursing diagnosis for a patient with
patients with impaired mental status, mobility, a leg fracture.
activity level, nutrition, circulation and bladder 37. D. “I know this will be difficult” acknowledges
or bowel control. Age is also a factor. Thus, the the problem and suggests a resolution to it.
88-year old incontinent patient who has “Don’t worry..” offers some relief but doesn’t
impaired nutrition (from gastric cancer) and is recognize the patient’s feelings. “..I didn’t get to
confined to bed is at greater risk. the bad news yet” would be inappropriate at any
30. A. Adequate hydration thins and loosens time. “Your hair is really pretty” offers no
pulmonary secretions and also helps to replace consolation or alternatives to the patient.
fluids lost from elevated temperature, 38. B. Additional Vitamin C is needed in growth
diaphoresis, dehydration and dyspnea. High- periods, such as infancy and childhood, and
humidity air and chest physiotherapy help during pregnancy to supply demands for fetal
liquefy and mobilize secretions. growth and maternal tissues. Other conditions
31. A. Chronic alcoholism commonly results in requiring extra vitamin C include wound healing,
thiamine deficiency and other symptoms of fever, infection and stress.
malnutrition. 39. D. Delivery of more than 2 liters of oxygen per
32. C. A patient with dysphagia (difficulty minute to a patient with chronic obstructive
swallowing) requires assistance with feeding. pulmonary disease (COPD), who is usually in a
Feeding himself is a long-range expected state of compensated respiratory acidosis
outcome. Soft foods, Fowler’s or semi-Fowler’s (retaining carbon dioxide (CO2)), can inhibit the
position, and oral hygiene before eating should hypoxic stimulus for respiration. An increased
be part of the feeding regimen. partial pressure of carbon dioxide in arterial
33. A. A urine output of less than 30ml/hour blood (PACO2) would not initially result in
indicates hypovolemia or oliguria, which is cardiac arrest. Circulatory overload and
related to kidney function and inadequate fluid respiratory excitement have no relevance to the
intake. question.

39
40
40. C. Presenting symptoms of hypokalemia ( a unknown. Parkinson’s disease is a neurologic
serum potassium level below 3.5 mEq/liter) disorder caused by lesions in the extrapyramidial
include muscle weakness, chronic fatigue, and system and manifested by tremors, muscle
cardiac dysrhythmias. The combined effects of rigidity, hypokinesis, dysphagia, and dysphonia.
inadequate food intake and prolonged diarrhea Multiple sclerosis, a progressive, degenerative
can deplete the potassium stores of a patient disease involving demyelination of the nerve
with GI problems. fibers, usually begins in young adulthood and is
41. D. Assisting a patient with ambulation and marked by periods of remission and
transfer from a bed to a chair allows the nurse to exacerbation. Amyotrophic lateral sclerosis, a
evaluate the patient’s ability to carry out these disease marked by progressive degeneration of
functions safely. Demonstrating the signal the neurons, eventually results in atrophy of all
system and providing an opportunity for a return the muscles; including those necessary for
demonstration ensures that the patient knows respiration.
how to operate the equipment and encourages 48. C. The nurse is legally responsible for labeling
him to call for assistance when needed. Checking the corpse when death occurs in the hospital.
the patient’s identification band verifies the She may be involved in obtaining consent for an
patient’s identity and prevents identification autopsy or notifying the coroner or medical
mistakes in drug administration. examiner of a patient’s death; however, she is
42. D. Since about 40% of patients fall out of bed not legally responsible for performing these
despite the use of side rails, side rails cannot be functions. The attending physician may need
said to prevent falls; however, they do serve as a information from the nurse to complete the
reminder that the patient should not get out of death certificate, but he is responsible for issuing
bed. The other answers are incorrect it.
interpretations of the statistical data. 49. B. The nurse must place a pillow under the
43. C. A patient who cannot care for himself at decreased person’s head and shoulders to
home does not necessarily have impaired prevent blood from settling in the face and
awareness; he may simply have some degree of discoloring it. She is required to bathe only
immobility. soiled areas of the body since the mortician will
44. D. Hip fracture, the most common injury among wash the entire body. Before wrapping the body
elderly persons, usually results from in a shroud, the nurse places a clean gown on
osteoporosis. The other answers are diseases the body and closes the eyes and mouth.
that can occur in the elderly from physiologic 50. A. Ensuring the patient’s safety is the most
changes. essential action at this time. The other nursing
45. A. Sleep disturbances, inability to concentrate actions may be necessary but are not a major
and decreased appetite are symptoms of priority.
depression, the most common psychogenic
disorder among elderly persons. Other
symptoms include diminished memory, apathy,
disinterest in appearance, withdrawal, and
irritability. Depression typically begins before the
onset of old age and usually is caused by
psychosocial, genetic, or biochemical factors
46. D. Aging decreases elasticity of the blood
vessels, which leads to increased peripheral
resistance and decreased blood flow. These
changes, in turn, increase the work load of the
left ventricle.
47. D. Alzheimer;s disease, sometimes known as
senile dementia of the Alzheimer’s type or
primary degenerative dementia, is an insidious;
progressive, irreversible, and degenerative
disease of the brain whose etiology is still

40
40
FUNDAMENTALS OF NURSING PART 3 8. Which of the following constitutes a break in
sterile technique while preparing a sterile field
1. Which element in the circular chain of infection for a dressing change?
can be eliminated by preserving skin integrity? a. Using sterile forceps, rather than sterile
a. Host gloves, to handle a sterile item
b. Reservoir b. Touching the outside wrapper of
c. Mode of transmission sterilized material without sterile gloves
d. Portal of entry c. Placing a sterile object on the edge of
2. Which of the following will probably result in a the sterile field
break in sterile technique for respiratory d. Pouring out a small amount of solution
isolation? (15 to 30 ml) before pouring the solution
a. Opening the patient’s window to the into a sterile container
outside environment 9. A natural body defense that plays an active role
b. Turning on the patient’s room ventilator in preventing infection is:
c. Opening the door of the patient’s room a. Yawning
leading into the hospital corridor b. Body hair
d. Failing to wear gloves when c. Hiccupping
administering a bed bath d. Rapid eye movements
3. Which of the following patients is at greater risk 10. All of the following statement are true about
for contracting an infection? donning sterile gloves except:
a. A patient with leukopenia a. The first glove should be picked up by
b. A patient receiving broad-spectrum grasping the inside of the cuff.
antibiotics b. The second glove should be picked up by
c. A postoperative patient who has inserting the gloved fingers under the
undergone orthopedic surgery cuff outside the glove.
d. A newly diagnosed diabetic patient c. The gloves should be adjusted by sliding
4. Effective hand washing requires the use of: the gloved fingers under the sterile cuff
a. Soap or detergent to promote and pulling the glove over the wrist
emulsification d. The inside of the glove is considered
b. Hot water to destroy bacteria sterile
c. A disinfectant to increase surface 11. When removing a contaminated gown, the nurse
tension should be careful that the first thing she touches
d. All of the above is the:
5. After routine patient contact, hand washing a. Waist tie and neck tie at the back of the
should last at least: gown
a. 30 seconds b. Waist tie in front of the gown
b. 1 minute c. Cuffs of the gown
c. 2 minute d. Inside of the gown
d. 3 minutes 12. Which of the following nursing interventions is
6. Which of the following procedures always considered the most effective form or universal
requires surgical asepsis? precautions?
a. Vaginal instillation of conjugated a. Cap all used needles before removing
estrogen them from their syringes
b. Urinary catheterization b. Discard all used uncapped needles and
c. Nasogastric tube insertion syringes in an impenetrable protective
d. Colostomy irrigation container
7. Sterile technique is used whenever: c. Wear gloves when administering IM
a. Strict isolation is required injections
b. Terminal disinfection is performed d. Follow enteric precautions
c. Invasive procedures are performed 13. All of the following measures are recommended
d. Protective isolation is necessary to prevent pressure ulcers except:
a. Massaging the reddened are with lotion

40
40
b. Using a water or air mattress the patient’s skin. The most appropriate nursing
c. Adhering to a schedule for positioning action would be to:
and turning a. Withhold the moderation and notify the
d. Providing meticulous skin care physician
14. Which of the following blood tests should be b. Administer the medication and notify
performed before a blood transfusion? the physician
a. Prothrombin and coagulation time c. Administer the medication with an
b. Blood typing and cross-matching antihistamine
c. Bleeding and clotting time d. Apply corn starch soaks to the rash
d. Complete blood count (CBC) and 21. All of the following nursing interventions are
electrolyte levels. correct when using the Z-track method of drug
15. The primary purpose of a platelet count is to injection except:
evaluate the: a. Prepare the injection site with alcohol
a. Potential for clot formation b. Use a needle that’s a least 1” long
b. Potential for bleeding c. Aspirate for blood before injection
c. Presence of an antigen-antibody d. Rub the site vigorously after the
response injection to promote absorption
d. Presence of cardiac enzymes 22. The correct method for determining the vastus
16. Which of the following white blood cell (WBC) lateralis site for I.M. injection is to:
counts clearly indicates leukocytosis? a. Locate the upper aspect of the upper
a. 4,500/mm³ outer quadrant of the buttock about 5 to
b. 7,000/mm³ 8 cm below the iliac crest
c. 10,000/mm³ b. Palpate the lower edge of the acromion
d. 25,000/mm³ process and the midpoint lateral aspect
17. After 5 days of diuretic therapy with 20mg of of the arm
furosemide (Lasix) daily, a patient begins to c. Palpate a 1” circular area anterior to the
exhibit fatigue, muscle cramping and muscle umbilicus
weakness. These symptoms probably indicate d. Divide the area between the greater
that the patient is experiencing: femoral trochanter and the lateral
a. Hypokalemia femoral condyle into thirds, and select
b. Hyperkalemia the middle third on the anterior of the
c. Anorexia thigh
d. Dysphagia 23. The mid-deltoid injection site is seldom used for
18. Which of the following statements about chest I.M. injections because it:
X-ray is false? a. Can accommodate only 1 ml or less of
a. No contradictions exist for this test medication
b. Before the procedure, the patient should b. Bruises too easily
remove all jewelry, metallic objects, and c. Can be used only when the patient is
buttons above the waist lying down
c. A signed consent is not required d. Does not readily parenteral medication
d. Eating, drinking, and medications are 24. The appropriate needle size for insulin injection
allowed before this test is:
19. The most appropriate time for the nurse to a. 18G, 1 ½” long
obtain a sputum specimen for culture is: b. 22G, 1” long
a. Early in the morning c. 22G, 1 ½” long
b. After the patient eats a light breakfast d. 25G, 5/8” long
c. After aerosol therapy 25. The appropriate needle gauge for intradermal
d. After chest physiotherapy injection is:
20. A patient with no known allergies is to receive a. 20G
penicillin every 6 hours. When administering the b. 22G
medication, the nurse observes a fine rash on c. 25G
d. 26G

40
40
26. Parenteral penicillin can be administered as an: b. Capsules whole contents are dissolve in
a. IM injection or an IV solution water
b. IV or an intradermal injection c. Enteric-coated tablets that are
c. Intradermal or subcutaneous injection thoroughly dissolved in water
d. IM or a subcutaneous injection d. Most tablets designed for oral use,
27. The physician orders gr 10 of aspirin for a except for extended-duration
patient. The equivalent dose in milligrams is: compounds
a. 0.6 mg 34. A patient who develops hives after receiving an
b. 10 mg antibiotic is exhibiting drug:
c. 60 mg a. Tolerance
d. 600 mg b. Idiosyncrasy
28. The physician orders an IV solution of dextrose c. Synergism
5% in water at 100ml/hour. What would the d. Allergy
flow rate be if the drop factor is 15 gtt = 1 ml? 35. A patient has returned to his room after femoral
a. 5 gtt/minute arteriography. All of the following are
b. 13 gtt/minute appropriate nursing interventions except:
c. 25 gtt/minute a. Assess femoral, popliteal, and pedal
d. 50 gtt/minute pulses every 15 minutes for 2 hours
29. Which of the following is a sign or symptom of a b. Check the pressure dressing for
hemolytic reaction to blood transfusion? sanguineous drainage
a. Hemoglobinuria c. Assess a vital signs every 15 minutes for
b. Chest pain 2 hours
c. Urticaria d. Order a hemoglobin and hematocrit
d. Distended neck veins count 1 hour after the arteriography
30. Which of the following conditions may require 36. The nurse explains to a patient that a cough:
fluid restriction? a. Is a protective response to clear the
a. Fever respiratory tract of irritants
b. Chronic Obstructive Pulmonary Disease b. Is primarily a voluntary action
c. Renal Failure c. Is induced by the administration of an
d. Dehydration antitussive drug
31. All of the following are common signs and d. Can be inhibited by “splinting” the
symptoms of phlebitis except: abdomen
a. Pain or discomfort at the IV insertion site 37. An infected patient has chills and begins
b. Edema and warmth at the IV insertion shivering. The best nursing intervention is to:
site a. Apply iced alcohol sponges
c. A red streak exiting the IV insertion site b. Provide increased cool liquids
d. Frank bleeding at the insertion site c. Provide additional bedclothes
32. The best way of determining whether a patient d. Provide increased ventilation
has learned to instill ear medication properly is 38. A clinical nurse specialist is a nurse who has:
for the nurse to: a. Been certified by the National League for
a. Ask the patient if he/she has used ear Nursing
drops before b. Received credentials from the Philippine
b. Have the patient repeat the nurse’s Nurses’ Association
instructions using her own words c. Graduated from an associate degree
c. Demonstrate the procedure to the program and is a registered professional
patient and encourage to ask questions nurse
d. Ask the patient to demonstrate the d. Completed a master’s degree in the
procedure prescribed clinical area and is a
33. Which of the following types of medications can registered professional nurse.
be administered via gastrostomy tube? 39. The purpose of increasing urine acidity through
a. Any oral medications dietary means is to:
a. Decrease burning sensations

40
40
b. Change the urine’s color b. Applying a topical antiseptic to the skin
c. Change the urine’s concentration on the evening before surgery
d. Inhibit the growth of microorganisms c. Having the patient take a tub bath on
40. Clay colored stools indicate: the morning of surgery
a. Upper GI bleeding d. Having the patient shower with an
b. Impending constipation antiseptic soap on the evening v=before
c. An effect of medication and the morning of surgery
d. Bile obstruction 47. When transferring a patient from a bed to a
41. In which step of the nursing process would the chair, the nurse should use which muscles to
nurse ask a patient if the medication she avoid back injury?
administered relieved his pain? a. Abdominal muscles
a. Assessment b. Back muscles
b. Analysis c. Leg muscles
c. Planning d. Upper arm muscles
d. Evaluation 48. Thrombophlebitis typically develops in patients
42. All of the following are good sources of vitamin A with which of the following conditions?
except: a. Increases partial thromboplastin time
a. White potatoes b. Acute pulsus paradoxus
b. Carrots c. An impaired or traumatized blood vessel
c. Apricots wall
d. Egg yolks d. Chronic Obstructive Pulmonary Disease
43. Which of the following is a primary nursing (COPD)
intervention necessary for all patients with a 49. In a recumbent, immobilized patient, lung
Foley Catheter in place? ventilation can become altered, leading to such
a. Maintain the drainage tubing and respiratory complications as:
collection bag level with the patient’s a. Respiratory acidosis, ateclectasis, and
bladder hypostatic pneumonia
b. Irrigate the patient with 1% Neosporin b. Appneustic breathing, atypical
solution three times a daily pneumonia and respiratory alkalosis
c. Clamp the catheter for 1 hour every 4 c. Cheyne-Strokes respirations and
hours to maintain the bladder’s elasticity spontaneous pneumothorax
d. Maintain the drainage tubing and d. Kussmail’s respirations and
collection bag below bladder level to hypoventilation
facilitate drainage by gravity 50. Immobility impairs bladder elimination, resulting
44. The ELISA test is used to: in such disorders as
a. Screen blood donors for antibodies to a. Increased urine acidity and relaxation of
human immunodeficiency virus (HIV) the perineal muscles, causing
b. Test blood to be used for transfusion for incontinence
HIV antibodies b. Urine retention, bladder distention, and
c. Aid in diagnosing a patient with AIDS infection
d. All of the above c. Diuresis, natriuresis, and decreased
45. The two blood vessels most commonly used for urine specific gravity
TPN infusion are the: d. Decreased calcium and phosphate levels
a. Subclavian and jugular veins in the urine
b. Brachial and subclavian veins
c. Femoral and subclavian veins
d. Brachial and femoral veins
46. Effective skin disinfection before a surgical
procedure includes which of the following
methods?
a. Shaving the site on the day before
surgery

40
40
ANSWERS and RATIONALES for FUNDAMENTALS OF equipment after a patient has been discharged
NURSING PART 3 to prepare them for reuse by another patient.
The purpose of protective (reverse) isolation is
1. D. In the circular chain of infection, pathogens to prevent a person with seriously impaired
must be able to leave their reservoir and be resistance from coming into contact who
transmitted to a susceptible host through a potentially pathogenic organisms.
portal of entry, such as broken skin. 8. C. The edges of a sterile field are considered
2. C. Respiratory isolation, like strict isolation, contaminated. When sterile items are allowed to
requires that the door to the door patient’s come in contact with the edges of the field, the
room remain closed. However, the patient’s sterile items also become contaminated.
room should be well ventilated, so opening the 9. B. Hair on or within body areas, such as the
window or turning on the ventricular is nose, traps and holds particles that contain
desirable. The nurse does not need to wear microorganisms. Yawning and hiccupping do not
gloves for respiratory isolation, but good hand prevent microorganisms from entering or
washing is important for all types of isolation. leaving the body. Rapid eye movement marks
3. A. Leukopenia is a decreased number of the stage of sleep during which dreaming occurs.
leukocytes (white blood cells), which are 10. D. The inside of the glove is always considered to
important in resisting infection. None of the be clean, but not sterile.
other situations would put the patient at risk for 11. A. The back of the gown is considered clean, the
contracting an infection; taking broad-spectrum front is contaminated. So, after removing gloves
antibiotics might actually reduce the infection and washing hands, the nurse should untie the
risk. back of the gown; slowly move backward away
4. A. Soaps and detergents are used to help from the gown, holding the inside of the gown
remove bacteria because of their ability to lower and keeping the edges off the floor; turn and
the surface tension of water and act as fold the gown inside out; discard it in a
emulsifying agents. Hot water may lead to skin contaminated linen container; then wash her
irritation or burns. hands again.
5. A. Depending on the degree of exposure to 12. B. According to the Centers for Disease Control
pathogens, hand washing may last from 10 (CDC), blood-to-blood contact occurs most
seconds to 4 minutes. After routine patient commonly when a health care worker attempts
contact, hand washing for 30 seconds effectively to cap a used needle. Therefore, used needles
minimizes the risk of pathogen transmission. should never be recapped; instead they should
6. B. The urinary system is normally free of be inserted in a specially designed puncture
microorganisms except at the urinary meatus. resistant, labeled container. Wearing gloves is
Any procedure that involves entering this system not always necessary when administering an I.M.
must use surgically aseptic measures to maintain injection. Enteric precautions prevent the
a bacteria-free state. transfer of pathogens via feces.
7. C. All invasive procedures, including surgery, 13. A. Nurses and other health care professionals
catheter insertion, and administration of previously believed that massaging a reddened
parenteral therapy, require sterile technique to area with lotion would promote venous return
maintain a sterile environment. All equipment and reduce edema to the area. However,
must be sterile, and the nurse and the physician research has shown that massage only increases
must wear sterile gloves and maintain surgical the likelihood of cellular ischemia and necrosis
asepsis. In the operating room, the nurse and to the area.
physician are required to wear sterile gowns, 14. B. Before a blood transfusion is performed, the
gloves, masks, hair covers, and shoe covers for blood of the donor and recipient must be
all invasive procedures. Strict isolation requires checked for compatibility. This is done by blood
the use of clean gloves, masks, gowns and typing (a test that determines a person’s blood
equipment to prevent the transmission of highly type) and cross-matching (a procedure that
communicable diseases by contact or by determines the compatibility of the donor’s and
airborne routes. Terminal disinfection is the recipient’s blood after the blood types has been
disinfection of all contaminated supplies and matched). If the blood specimens are

40
40
incompatible, hemolysis and antigen-antibody corn starch to the rash may relieve discomfort, it
reactions will occur. is not the nurse’s top priority in such a
15. A. Platelets are disk-shaped cells that are potentially life-threatening situation.
essential for blood coagulation. A platelet count 21. D. The Z-track method is an I.M. injection
determines the number of thrombocytes in technique in which the patient’s skin is pulled in
blood available for promoting hemostasis and such a way that the needle track is sealed off
assisting with blood coagulation after injury. It after the injection. This procedure seals
also is used to evaluate the patient’s potential medication deep into the muscle, thereby
for bleeding; however, this is not its primary minimizing skin staining and irritation. Rubbing
purpose. The normal count ranges from 150,000 the injection site is contraindicated because it
to 350,000/mm3. A count of 100,000/mm3 or may cause the medication to extravasate into
less indicates a potential for bleeding; count of the skin.
less than 20,000/mm3 is associated with 22. D. The vastus lateralis, a long, thick muscle that
spontaneous bleeding. extends the full length of the thigh, is viewed by
16. D. Leukocytosis is any transient increase in the many clinicians as the site of choice for I.M.
number of white blood cells (leukocytes) in the injections because it has relatively few major
blood. Normal WBC counts range from 5,000 to nerves and blood vessels. The middle third of the
100,000/mm3. Thus, a count of 25,000/mm3 muscle is recommended as the injection site.
indicates leukocytosis. The patient can be in a supine or sitting position
17. A. Fatigue, muscle cramping, and muscle for an injection into this site.
weaknesses are symptoms of hypokalemia (an 23. A. The mid-deltoid injection site can
inadequate potassium level), which is a potential accommodate only 1 ml or less of medication
side effect of diuretic therapy. The physician because of its size and location (on the deltoid
usually orders supplemental potassium to muscle of the arm, close to the brachial artery
prevent hypokalemia in patients receiving and radial nerve).
diuretics. Anorexia is another symptom of 24. D. A 25G, 5/8” needle is the recommended size
hypokalemia. Dysphagia means difficulty for insulin injection because insulin is
swallowing. administered by the subcutaneous route. An
18. A. Pregnancy or suspected pregnancy is the only 18G, 1 ½” needle is usually used for I.M.
contraindication for a chest X-ray. However, if a injections in children, typically in the vastus
chest X-ray is necessary, the patient can wear a lateralis. A 22G, 1 ½” needle is usually used for
lead apron to protect the pelvic region from adult I.M. injections, which are typically
radiation. Jewelry, metallic objects, and buttons administered in the vastus lateralis or
would interfere with the X-ray and thus should ventrogluteal site.
not be worn above the waist. A signed consent is 25. D. Because an intradermal injection does not
not required because a chest X-ray is not an penetrate deeply into the skin, a small-bore 25G
invasive examination. Eating, drinking and needle is recommended. This type of injection is
medications are allowed because the X-ray is of used primarily to administer antigens to
the chest, not the abdominal region. evaluate reactions for allergy or sensitivity
19. A. Obtaining a sputum specimen early in this studies. A 20G needle is usually used for I.M.
morning ensures an adequate supply of bacteria injections of oil-based medications; a 22G needle
for culturing and decreases the risk of for I.M. injections; and a 25G needle, for I.M.
contamination from food or medication. injections; and a 25G needle, for subcutaneous
20. A. Initial sensitivity to penicillin is commonly insulin injections.
manifested by a skin rash, even in individuals 26. A. Parenteral penicillin can be administered I.M.
who have not been allergic to it previously. or added to a solution and given I.V. It cannot be
Because of the danger of anaphylactic shock, he administered subcutaneously or intradermally.
nurse should withhold the drug and notify the 27. D. gr 10 x 60mg/gr 1 = 600 mg
physician, who may choose to substitute 28. C. 100ml/60 min X 15 gtt/ 1 ml = 25 gtt/minute
another drug. Administering an antihistamine is 29. A. Hemoglobinuria, the abnormal presence of
a dependent nursing intervention that requires a hemoglobin in the urine, indicates a hemolytic
written physician’s order. Although applying reaction (incompatibility of the donor’s and

40
40
recipient’s blood). In this reaction, antibodies in 35. D. A hemoglobin and hematocrit count would be
the recipient’s plasma combine rapidly with ordered by the physician if bleeding were
donor RBC’s; the cells are hemolyzed in either suspected. The other answers are appropriate
circulatory or reticuloendothelial system. nursing interventions for a patient who has
Hemolysis occurs more rapidly in ABO undergone femoral arteriography.
incompatibilities than in Rh incompatibilities. 36. A. Coughing, a protective response that clears
Chest pain and urticaria may be symptoms of the respiratory tract of irritants, usually is
impending anaphylaxis. Distended neck veins are involuntary; however it can be voluntary, as
an indication of hypervolemia. when a patient is taught to perform coughing
30. C. In real failure, the kidney loses their ability to exercises. An antitussive drug inhibits coughing.
effectively eliminate wastes and fluids. Because Splinting the abdomen supports the abdominal
of this, limiting the patient’s intake of oral and muscles when a patient coughs.
I.V. fluids may be necessary. Fever, chronic 37. C. In an infected patient, shivering results from
obstructive pulmonary disease, and dehydration the body’s attempt to increase heat production
are conditions for which fluids should be and the production of neutrophils and
encouraged. phagocytotic action through increased skeletal
31. D. Phlebitis, the inflammation of a vein, can be muscle tension and contractions. Initial
caused by chemical irritants (I.V. solutions or vasoconstriction may cause skin to feel cold to
medications), mechanical irritants (the needle or the touch. Applying additional bed clothes helps
catheter used during venipuncture or to equalize the body temperature and stop the
cannulation), or a localized allergic reaction to chills. Attempts to cool the body result in further
the needle or catheter. Signs and symptoms of shivering, increased metabloism, and thus
phlebitis include pain or discomfort, edema and increased heat production.
heat at the I.V. insertion site, and a red streak 38. D. A clinical nurse specialist must have
going up the arm or leg from the I.V. insertion completed a master’s degree in a clinical
site. specialty and be a registered professional nurse.
32. D. Return demonstration provides the most The National League of Nursing accredits
certain evidence for evaluating the effectiveness educational programs in nursing and provides a
of patient teaching. testing service to evaluate student nursing
33. D. Capsules, enteric-coated tablets, and most competence but it does not certify nurses. The
extended duration or sustained release products American Nurses Association identifies
should not be dissolved for use in a gastrostomy requirements for certification and offers
tube. They are pharmaceutically manufactured examinations for certification in many areas of
in these forms for valid reasons, and altering nursing., such as medical surgical nursing. These
them destroys their purpose. The nurse should certification (credentialing) demonstrates that
seek an alternate physician’s order when an the nurse has the knowledge and the ability to
ordered medication is inappropriate for delivery provide high quality nursing care in the area of
by tube. her certification. A graduate of an associate
34. D. A drug-allergy is an adverse reaction resulting degree program is not a clinical nurse specialist:
from an immunologic response following a however, she is prepared to provide bed side
previous sensitizing exposure to the drug. The nursing with a high degree of knowledge and
reaction can range from a rash or hives to skill. She must successfully complete the
anaphylactic shock. Tolerance to a drug means licensing examination to become a registered
that the patient experiences a decreasing professional nurse.
physiologic response to repeated administration 39. D. Microorganisms usually do not grow in an
of the drug in the same dosage. Idiosyncrasy is acidic environment.
an individual’s unique hypersensitivity to a drug, 40. D. Bile colors the stool brown. Any inflammation
food, or other substance; it appears to be or obstruction that impairs bile flow will affect
genetically determined. Synergism, is a drug the stool pigment, yielding light, clay-colored
interaction in which the sum of the drug’s stool. Upper GI bleeding results in black or tarry
combined effects is greater than that of their stool. Constipation is characterized by small,
separate effects. hard masses. Many medications and foods will

40
40
discolor stool – for example, drugs containing blood vessel wall. Increased partial
iron turn stool black.; beets turn stool red. thromboplastin time indicates a prolonged
41. D. In the evaluation step of the nursing process, bleeding time during fibrin clot formation,
the nurse must decide whether the patient has commonly the result of anticoagulant (heparin)
achieved the expected outcome that was therapy. Arterial blood disorders (such as pulsus
identified in the planning phase. paradoxus) and lung diseases (such as COPD) do
42. A. The main sources of vitamin A are yellow and not necessarily impede venous return of injure
green vegetables (such as carrots, sweet vessel walls.
potatoes, squash, spinach, collard greens, 49. A. Because of restricted respiratory movement,
broccoli, and cabbage) and yellow fruits (such as a recumbent, immobilize patient is at particular
apricots, and cantaloupe). Animal sources risk for respiratory acidosis from poor gas
include liver, kidneys, cream, butter, and egg exchange; atelectasis from reduced surfactant
yolks. and accumulated mucus in the bronchioles, and
43. D. Maintaing the drainage tubing and collection hypostatic pneumonia from bacterial growth
bag level with the patient’s bladder could result caused by stasis of mucus secretions.
in reflux of urine into the kidney. Irrigating the 50. B. The immobilized patient commonly suffers
bladder with Neosporin and clamping the from urine retention caused by decreased
catheter for 1 hour every 4 hours must be muscle tone in the perineum. This leads to
prescribed by a physician. bladder distention and urine stagnation, which
44. D. The ELISA test of venous blood is used to provide an excellent medium for bacterial
assess blood and potential blood donors to growth leading to infection. Immobility also
human immunodeficiency virus (HIV). A positive results in more alkaline urine with excessive
ELISA test combined with various signs and amounts of calcium, sodium and phosphate, a
symptoms helps to diagnose acquired gradual decrease in urine production, and an
immunodeficiency syndrome (AIDS) increased specific gravity
45. D. Tachypnea (an abnormally rapid rate of
breathing) would indicate that the patient was
still hypoxic (deficient in oxygen).The partial
pressures of arterial oxygen and carbon dioxide
listed are within the normal range. Eupnea refers
to normal respiration.
46. D. Studies have shown that showering with an
antiseptic soap before surgery is the most
effective method of removing microorganisms
from the skin. Shaving the site of the intended
surgery might cause breaks in the skin, thereby
increasing the risk of infection; however, if
indicated, shaving, should be done immediately
before surgery, not the day before. A topical
antiseptic would not remove microorganisms
and would be beneficial only after proper
cleaning and rinsing. Tub bathing might transfer
organisms to another body site rather than rinse
them away.
47. C. The leg muscles are the strongest muscles in
the body and should bear the greatest stress
when lifting. Muscles of the abdomen, back, and
upper arms may be easily injured.
48. C. The factors, known as Virchow’s triad,
collectively predispose a patient to
thromboplebitis; impaired venous return to the
heart, blood hypercoagulability, and injury to a

40
40
MATERNITY NURSING Part 1 7. Which of the following represents the average
amount of weight gained during pregnancy?
1. When assessing the adequacy of sperm for a. 12 to 22 lb
conception to occur, which of the following is b. 15 to 25 lb
the most useful criterion? c. 24 to 30 lb
a. Sperm count d. 25 to 40 lb
b. Sperm motility 8. When talking with a pregnant client who is
c. Sperm maturity experiencing aching swollen, leg veins, the nurse
d. Semen volume would explain that this is most probably the
2. A couple who wants to conceive but has been result of which of the following?
unsuccessful during the last 2 years has a. Thrombophlebitis
undergone many diagnostic procedures. When b. Pregnancy-induced hypertension
discussing the situation with the nurse, one c. Pressure on blood vessels from the
partner states, “We know several friends in our enlarging uterus
age group and all of them have their own child d. The force of gravity pulling down on the
already, Why can’t we have one?”. Which of the uterus
following would be the most pertinent nursing 9. Cervical softening and uterine souffle are
diagnosis for this couple? classified as which of the following?
a. Fear related to the unknown a. Diagnostic signs
b. Pain related to numerous procedures. b. Presumptive signs
c. Ineffective family coping related to c. Probable signs
infertility. d. Positive signs
d. Self-esteem disturbance related to 10. Which of the following would the nurse identify
infertility. as a presumptive sign of pregnancy?
3. Which of the following urinary symptoms does a. Hegar sign
the pregnant woman most frequently b. Nausea and vomiting
experience during the first trimester? c. Skin pigmentation changes
a. Dysuria d. Positive serum pregnancy test
b. Frequency 11. Which of the following common emotional
c. Incontinence reactions to pregnancy would the nurse expect
d. Burning to occur during the first trimester?
4. Heartburn and flatulence, common in the a. Introversion, egocentrism, narcissism
second trimester, are most likely the result of b. Awkwardness, clumsiness, and
which of the following? unattractiveness
a. Increased plasma HCG levels c. Anxiety, passivity, extroversion
b. Decreased intestinal motility d. Ambivalence, fear, fantasies
c. Decreased gastric acidity 12. During which of the following would the focus of
d. Elevated estrogen levels classes be mainly on physiologic changes, fetal
5. On which of the following areas would the nurse development, sexuality, during pregnancy, and
expect to observe chloasma? nutrition?
a. Breast, areola, and nipples a. Prepregnant period
b. Chest, neck, arms, and legs b. First trimester
c. Abdomen, breast, and thighs c. Second trimester
d. Cheeks, forehead, and nose d. Third trimester
6. A pregnant client states that she “waddles” 13. Which of the following would be disadvantage of
when she walks. The nurse’s explanation is breast feeding?
based on which of the following as the cause? a. Involution occurs more rapidly
a. The large size of the newborn b. The incidence of allergies increases due
b. Pressure on the pelvic muscles to maternal antibodies
c. Relaxation of the pelvic joints c. The father may resent the infant’s
d. Excessive weight gain demands on the mother’s body

40
41
d. There is a greater chance for error 21. During which of the following stages of labor
during preparation would the nurse assess “crowning”?
14. Which of the following would cause a false- a. First stage
positive result on a pregnancy test? b. Second stage
a. The test was performed less than 10 c. Third stage
days after an abortion d. Fourth stage
b. The test was performed too early or too 22. Barbiturates are usually not given for pain relief
late in the pregnancy during active labor for which of the following
c. The urine sample was stored too long at reasons?
room temperature a. The neonatal effects include hypotonia,
d. A spontaneous abortion or a missed hypothermia, generalized drowsiness,
abortion is impending and reluctance to feed for the first few
15. FHR can be auscultated with a fetoscope as early days.
as which of the following? b. These drugs readily cross the placental
a. 5 weeks gestation barrier, causing depressive effects in the
b. 10 weeks gestation newborn 2 to 3 hours after
c. 15 weeks gestation intramuscular injection.
d. 20 weeks gestation c. They rapidly transfer across the
16. A client LMP began July 5. Her EDD should be placenta, and lack of an antagonist make
which of the following? them generally inappropriate during
a. January 2 labor.
b. March 28 d. Adverse reactions may include maternal
c. April 12 hypotension, allergic or toxic reaction or
d. October 12 partial or total respiratory failure
17. Which of the following fundal heights indicates 23. Which of the following nursing interventions
less than 12 weeks’ gestation when the date of would the nurse perform during the third stage
the LMP is unknown? of labor?
a. Uterus in the pelvis a. Obtain a urine specimen and other
b. Uterus at the xiphoid laboratory tests.
c. Uterus in the abdomen b. Assess uterine contractions every 30
d. Uterus at the umbilicus minutes.
18. Which of the following danger signs should be c. Coach for effective client pushing
reported promptly during the antepartum d. Promote parent-newborn interaction.
period? 24. Which of the following actions demonstrates the
a. Constipation nurse’s understanding about the newborn’s
b. Breast tenderness thermoregulatory ability?
c. Nasal stuffiness a. Placing the newborn under a radiant
d. Leaking amniotic fluid warmer.
19. Which of the following prenatal laboratory test b. Suctioning with a bulb syringe
values would the nurse consider as significant? c. Obtaining an Apgar score
a. Hematocrit 33.5% d. Inspecting the newborn’s umbilical cord
b. Rubella titer less than 1:8 25. Immediately before expulsion, which of the
c. White blood cells 8,000/mm3 following cardinal movements occur?
d. One hour glucose challenge test 110 a. Descent
g/dL b. Flexion
20. Which of the following characteristics of c. Extension
contractions would the nurse expect to find in a d. External rotation
client experiencing true labor? 26. Before birth, which of the following structures
a. Occurring at irregular intervals connects the right and left auricles of the heart?
b. Starting mainly in the abdomen a. Umbilical vein
c. Gradually increasing intervals b. Foramen ovale
d. Increasing intensity with walking c. Ductus arteriosus

41
41
d. Ductus venosus 32. Which of the following statements best
27. Which of the following when present in the describes hyperemesis gravidarum?
urine may cause a reddish stain on the diaper of a. Severe anemia leading to electrolyte,
a newborn? metabolic, and nutritional imbalances in
a. Mucus the absence of other medical problems.
b. Uric acid crystals b. Severe nausea and vomiting leading to
c. Bilirubin electrolyte, metabolic, and nutritional
d. Excess iron imbalances in the absence of other
28. When assessing the newborn’s heart rate, which medical problems.
of the following ranges would be considered c. Loss of appetite and continuous
normal if the newborn were sleeping? vomiting that commonly results in
a. 80 beats per minute dehydration and ultimately decreasing
b. 100 beats per minute maternal nutrients
c. 120 beats per minute d. Severe nausea and diarrhea that can
d. 140 beats per minute cause gastrointestinal irritation and
29. Which of the following is true regarding the possibly internal bleeding
fontanels of the newborn? 33. Which of the following would the nurse identify
a. The anterior is triangular shaped; the as a classic sign of PIH?
posterior is diamond shaped. a. Edema of the feet and ankles
b. The posterior closes at 18 months; the b. Edema of the hands and face
anterior closes at 8 to 12 weeks. c. Weight gain of 1 lb/week
c. The anterior is large in size when d. Early morning headache
compared to the posterior fontanel. 34. In which of the following types of spontaneous
d. The anterior is bulging; the posterior abortions would the nurse assess dark brown
appears sunken. vaginal discharge and a negative pregnancy
30. Which of the following groups of newborn tests?
reflexes below are present at birth and remain a. Threatened
unchanged through adulthood? b. Imminent
a. Blink, cough, rooting, and gag c. Missed
b. Blink, cough, sneeze, gag d. Incomplete
c. Rooting, sneeze, swallowing, and cough 35. Which of the following factors would the nurse
d. Stepping, blink, cough, and sneeze suspect as predisposing a client to placenta
31. Which of the following describes the Babinski previa?
reflex? a. Multiple gestation
a. The newborn’s toes will hyperextend b. Uterine anomalies
and fan apart from dorsiflexion of the c. Abdominal trauma
big toe when one side of foot is stroked d. Renal or vascular disease
upward from the ball of the heel and 36. Which of the following would the nurse assess in
across the ball of the foot. a client experiencing abruptio placenta?
b. The newborn abducts and flexes all a. Bright red, painless vaginal bleeding
extremities and may begin to cry when b. Concealed or external dark red bleeding
exposed to sudden movement or loud c. Palpable fetal outline
noise. d. Soft and nontender abdomen
c. The newborn turns the head in the 37. Which of the following is described as premature
direction of stimulus, opens the mouth, separation of a normally implanted placenta
and begins to suck when cheek, lip, or during the second half of pregnancy, usually with
corner of mouth is touched. severe hemorrhage?
d. The newborn will attempt to crawl a. Placenta previa
forward with both arms and legs when b. Ectopic pregnancy
he is placed on his abdomen on a flat c. Incompetent cervix
surface d. Abruptio placentae

41
41
38. Which of the following may happen if the uterus 43. When uterine rupture occurs, which of the
becomes overstimulated by oxytocin during the following would be the priority?
induction of labor? a. Limiting hypovolemic shock
a. Weak contraction prolonged to more b. Obtaining blood specimens
than 70 seconds c. Instituting complete bed rest
b. Tetanic contractions prolonged to more d. Inserting a urinary catheter
than 90 seconds 44. Which of the following is the nurse’s initial
c. Increased pain with bright red vaginal action when umbilical cord prolapse occurs?
bleeding a. Begin monitoring maternal vital signs
d. Increased restlessness and anxiety and FHR
39. When preparing a client for cesarean delivery, b. Place the client in a knee-chest position
which of the following key concepts should be in bed
considered when implementing nursing care? c. Notify the physician and prepare the
a. Instruct the mother’s support person to client for delivery
remain in the family lounge until after d. Apply a sterile warm saline dressing to
the delivery the exposed cord
b. Arrange for a staff member of the 45. Which of the following amounts of blood loss
anesthesia department to explain what following birth marks the criterion for describing
to expect postoperatively postpartum hemorrhage?
c. Modify preoperative teaching to meet a. More than 200 ml
the needs of either a planned or b. More than 300 ml
emergency cesarean birth c. More than 400 ml
d. Explain the surgery, expected outcome, d. More than 500 ml
and kind of anesthetics 46. Which of the following is the primary
40. Which of the following best describes preterm predisposing factor related to mastitis?
labor? a. Epidemic infection from nosocomial
a. Labor that begins after 20 weeks sources localizing in the lactiferous
gestation and before 37 weeks gestation glands and ducts
b. Labor that begins after 15 weeks b. Endemic infection occurring randomly
gestation and before 37 weeks gestation and localizing in the periglandular
c. Labor that begins after 24 weeks connective tissue
gestation and before 28 weeks gestation c. Temporary urinary retention due to
d. Labor that begins after 28 weeks decreased perception of the urge to
gestation and before 40 weeks gestation avoid
41. When PROM occurs, which of the following d. Breast injury caused by overdistention,
provides evidence of the nurse’s understanding stasis, and cracking of the nipples
of the client’s immediate needs? 47. Which of the following best describes
a. The chorion and amnion rupture 4 hours thrombophlebitis?
before the onset of labor. a. Inflammation and clot formation that
b. PROM removes the fetus most effective result when blood components combine
defense against infection to form an aggregate body
c. Nursing care is based on fetal viability b. Inflammation and blood clots that
and gestational age. eventually become lodged within the
d. PROM is associated with pulmonary blood vessels
malpresentation and possibly c. Inflammation and blood clots that
incompetent cervix eventually become lodged within the
42. Which of the following factors is the underlying femoral vein
cause of dystocia? d. Inflammation of the vascular
a. Nurtional endothelium with clot formation on the
b. Mechanical vessel wall
c. Environmental
d. Medical

41
41
48. Which of the following assessment findings 53. When preparing a woman who is 2 days
would the nurse expect if the client develops postpartum for discharge, recommendations for
DVT? which of the following contraceptive methods
a. Midcalf pain, tenderness and redness would be avoided?
along the vein a. Diaphragm
b. Chills, fever, malaise, occurring 2 weeks b. Female condom
after delivery c. Oral contraceptives
c. Muscle pain the presence of Homans d. Rhythm method
sign, and swelling in the affected limb 54. For which of the following clients would the
d. Chills, fever, stiffness, and pain occurring nurse expect that an intrauterine device would
10 to 14 days after delivery not be recommended?
49. Which of the following are the most commonly a. Woman over age 35
assessed findings in cystitis? b. Nulliparous woman
a. Frequency, urgency, dehydration, c. Promiscuous young adult
nausea, chills, and flank pain d. Postpartum client
b. Nocturia, frequency, urgency dysuria, 55. A client in her third trimester tells the nurse,
hematuria, fever and suprapubic pain “I’m constipated all the time!” Which of the
c. Dehydration, hypertension, dysuria, following should the nurse recommend?
suprapubic pain, chills, and fever a. Daily enemas
d. High fever, chills, flank pain nausea, b. Laxatives
vomiting, dysuria, and frequency c. Increased fiber intake
50. Which of the following best reflects the d. Decreased fluid intake
frequency of reported postpartum “blues”? 56. Which of the following would the nurse use as
a. Between 10% and 40% of all new the basis for the teaching plan when caring for a
mothers report some form of pregnant teenager concerned about gaining too
postpartum blues much weight during pregnancy?
b. Between 30% and 50% of all new a. 10 pounds per trimester
mothers report some form of b. 1 pound per week for 40 weeks
postpartum blues c. ½ pound per week for 40 weeks
c. Between 50% and 80% of all new d. A total gain of 25 to 30 pounds
mothers report some form of 57. The client tells the nurse that her last menstrual
postpartum blues period started on January 14 and ended on
d. Between 25% and 70% of all new January 20. Using Nagele’s rule, the nurse
mothers report some form of determines her EDD to be which of the
postpartum blues following?
51. For the client who is using oral contraceptives, a. September 27
the nurse informs the client about the need to b. October 21
take the pill at the same time each day to c. November 7
accomplish which of the following? d. December 27
a. Decrease the incidence of nausea 58. When taking an obstetrical history on a pregnant
b. Maintain hormonal levels client who states, “I had a son born at 38 weeks
c. Reduce side effects gestation, a daughter born at 30 weeks gestation
d. Prevent drug interactions and I lost a baby at about 8 weeks,” the nurse
52. When teaching a client about contraception. should record her obstetrical history as which of
Which of the following would the nurse include the following?
as the most effective method for preventing a. G2 T2 P0 A0 L2
sexually transmitted infections? b. G3 T1 P1 A0 L2
a. Spermicides c. G3 T2 P0 A0 L2
b. Diaphragm d. G4 T2 P1 A1 L2
c. Condoms 59. When preparing to listen to the fetal heart rate
d. Vasectomy at 12 weeks’ gestation, the nurse would use
which of the following?

41
41
a. Stethoscope placed midline at the b. Administer a narcotic before breast
umbilicus feeding
b. Doppler placed midline at the c. Encourage her to wear a nursing
suprapubic region brassiere
c. Fetoscope placed midway between the d. Use soap and water to clean the nipples
umbilicus and the xiphoid process 66. The nurse assesses the vital signs of a client, 4
d. External electronic fetal monitor placed hours’ postpartum that are as follows: BP 90/60;
at the umbilicus temperature 100.4ºF; pulse 100 weak, thready;
60. When developing a plan of care for a client R 20 per minute. Which of the following should
newly diagnosed with gestational diabetes, the nurse do first?
which of the following instructions would be the a. Report the temperature to the physician
priority? b. Recheck the blood pressure with
a. Dietary intake another cuff
b. Medication c. Assess the uterus for firmness and
c. Exercise position
d. Glucose monitoring d. Determine the amount of lochia
61. A client at 24 weeks gestation has gained 6 67. The nurse assesses the postpartum vaginal
pounds in 4 weeks. Which of the following would discharge (lochia) on four clients. Which of the
be the priority when assessing the client? following assessments would warrant
a. Glucosuria notification of the physician?
b. Depression a. A dark red discharge on a 2-day
c. Hand/face edema postpartum client
d. Dietary intake b. A pink to brownish discharge on a client
62. A client 12 weeks’ pregnant come to the who is 5 days postpartum
emergency department with abdominal c. Almost colorless to creamy discharge on
cramping and moderate vaginal bleeding. a client 2 weeks after delivery
Speculum examination reveals 2 to 3 cms d. A bright red discharge 5 days after
cervical dilation. The nurse would document delivery
these findings as which of the following? 68. A postpartum client has a temperature of
a. Threatened abortion 101.4ºF, with a uterus that is tender when
b. Imminent abortion palpated, remains unusually large, and not
c. Complete abortion descending as normally expected. Which of the
d. Missed abortion following should the nurse assess next?
63. Which of the following would be the priority a. Lochia
nursing diagnosis for a client with an ectopic b. Breasts
pregnancy? c. Incision
a. Risk for infection d. Urine
b. Pain 69. Which of the following is the priority focus of
c. Knowledge Deficit nursing practice with the current early
d. Anticipatory Grieving postpartum discharge?
64. Before assessing the postpartum client’s uterus a. Promoting comfort and restoration of
for firmness and position in relation to the health
umbilicus and midline, which of the following b. Exploring the emotional status of the
should the nurse do first? family
a. Assess the vital signs c. Facilitating safe and effective self-and
b. Administer analgesia newborn care
c. Ambulate her in the hall d. Teaching about the importance of family
d. Assist her to urinate planning
65. Which of the following should the nurse do 70. Which of the following actions would be least
when a primipara who is lactating tells the nurse effective in maintaining a neutral thermal
that she has sore nipples? environment for the newborn?
a. Tell her to breast feed more frequently

41
41
a. Placing infant under radiant warmer a. “Daily soap and water cleansing is best”
after bathing b. ‘Alcohol helps it dry and kills germs”
b. Covering the scale with a warmed c. “An antibiotic ointment applied daily
blanket prior to weighing prevents infection”
c. Placing crib close to nursery window for d. “He can have a tub bath each day”
family viewing 76. A newborn weighing 3000 grams and feeding
d. Covering the infant’s head with a knit every 4 hours needs 120 calories/kg of body
stockinette weight every 24 hours for proper growth and
71. A newborn who has an asymmetrical Moro development. How many ounces of 20 cal/oz
reflex response should be further assessed for formula should this newborn receive at each
which of the following? feeding to meet nutritional needs?
a. Talipes equinovarus a. 2 ounces
b. Fractured clavicle b. 3 ounces
c. Congenital hypothyroidism c. 4 ounces
d. Increased intracranial pressure d. 6 ounces
72. During the first 4 hours after a male 77. The postterm neonate with meconium-stained
circumcision, assessing for which of the amniotic fluid needs care designed to especially
following is the priority? monitor for which of the following?
a. Infection a. Respiratory problems
b. Hemorrhage b. Gastrointestinal problems
c. Discomfort c. Integumentary problems
d. Dehydration d. Elimination problems
73. The mother asks the nurse. “What’s wrong with 78. When measuring a client’s fundal height, which
my son’s breasts? Why are they so enlarged?” of the following techniques denotes the correct
Whish of the following would be the best method of measurement used by the nurse?
response by the nurse? a. From the xiphoid process to the
a. “The breast tissue is inflamed from the umbilicus
trauma experienced with birth” b. From the symphysis pubis to the xiphoid
b. “A decrease in material hormones process
present before birth causes c. From the symphysis pubis to the fundus
enlargement,” d. From the fundus to the umbilicus
c. “You should discuss this with your 79. A client with severe preeclampsia is admitted
doctor. It could be a malignancy” with of BP 160/110, proteinuria, and severe
d. “The tissue has hypertrophied while the pitting edema. Which of the following would be
baby was in the uterus” most important to include in the client’s plan of
74. Immediately after birth the nurse notes the care?
following on a male newborn: respirations 78; a. Daily weights
apical hearth rate 160 BPM, nostril flaring; mild b. Seizure precautions
intercostal retractions; and grunting at the end c. Right lateral positioning
of expiration. Which of the following should the d. Stress reduction
nurse do? 80. A postpartum primipara asks the nurse, “When
a. Call the assessment data to the can we have sexual intercourse again?” Which of
physician’s attention the following would be the nurse’s best
b. Start oxygen per nasal cannula at 2 response?
L/min. a. “Anytime you both want to.”
c. Suction the infant’s mouth and nares b. “As soon as choose a contraceptive
d. Recognize this as normal first period of method.”
reactivity c. “When the discharge has stopped and
75. The nurse hears a mother telling a friend on the the incision is healed.”
telephone about umbilical cord care. Which of d. “After your 6 weeks examination.”
the following statements by the mother 81. When preparing to administer the vitamin K
indicates effective teaching? injection to a neonate, the nurse would select

41
41
which of the following sites as appropriate for 87. During a prenatal class, the nurse explains the
the injection? rationale for breathing techniques during
a. Deltoid muscle preparation for labor based on the
b. Anterior femoris muscle understanding that breathing techniques are
c. Vastus lateralis muscle most important in achieving which of the
d. Gluteus maximus muscle following?
82. When performing a pelvic examination, the a. Eliminate pain and give the expectant
nurse observes a red swollen area on the right parents something to do
side of the vaginal orifice. The nurse would b. Reduce the risk of fetal distress by
document this as enlargement of which of the increasing uteroplacental perfusion
following? c. Facilitate relaxation, possibly reducing
a. Clitoris the perception of pain
b. Parotid gland d. Eliminate pain so that less analgesia and
c. Skene’s gland anesthesia are needed
d. Bartholin’s gland 88. After 4 hours of active labor, the nurse notes
83. To differentiate as a female, the hormonal that the contractions of a primigravida client are
stimulation of the embryo that must occur not strong enough to dilate the cervix. Which of
involves which of the following? the following would the nurse anticipate doing?
a. Increase in maternal estrogen secretion a. Obtaining an order to begin IV oxytocin
b. Decrease in maternal androgen infusion
secretion b. Administering a light sedative to allow
c. Secretion of androgen by the fetal gonad the patient to rest for several hour
d. Secretion of estrogen by the fetal gonad c. Preparing for a cesarean section for
84. A client at 8 weeks’ gestation calls complaining failure to progress
of slight nausea in the morning hours. Which of d. Increasing the encouragement to the
the following client interventions should the patient when pushing begins
nurse question? 89. A multigravida at 38 weeks’ gestation is
a. Taking 1 teaspoon of bicarbonate of admitted with painless, bright red bleeding and
soda in an 8-ounce glass of water mild contractions every 7 to 10 minutes. Which
b. Eating a few low-sodium crackers before of the following assessments should be avoided?
getting out of bed a. Maternal vital sign
c. Avoiding the intake of liquids in the b. Fetal heart rate
morning hours c. Contraction monitoring
d. Eating six small meals a day instead of d. Cervical dilation
thee large meals 90. Which of the following would be the nurse’s
85. The nurse documents positive ballottement in most appropriate response to a client who asks
the client’s prenatal record. The nurse why she must have a cesarean delivery if she has
understands that this indicates which of the a complete placenta previa?
following? a. “You will have to ask your physician
a. Palpable contractions on the abdomen when he returns.”
b. Passive movement of the unengaged b. “You need a cesarean to prevent
fetus hemorrhage.”
c. Fetal kicking felt by the client c. “The placenta is covering most of your
d. Enlargement and softening of the uterus cervix.”
86. During a pelvic exam the nurse notes a purple- d. “The placenta is covering the opening of
blue tinge of the cervix. The nurse documents the uterus and blocking your baby.”
this as which of the following? 91. The nurse understands that the fetal head is in
a. Braxton-Hicks sign which of the following positions with a face
b. Chadwick’s sign presentation?
c. Goodell’s sign a. Completely flexed
d. McDonald’s sign b. Completely extended
c. Partially extended

41
41
d. Partially flexed accident as a teenager. The nurse is aware that
92. With a fetus in the left-anterior breech this could prevent a fetus from passing through
presentation, the nurse would expect the fetal or around which structure during childbirth?
heart rate would be most audible in which of the a. Symphysis pubis
following areas? b. Sacral promontory
a. Above the maternal umbilicus and to the c. Ischial spines
right of midline d. Pubic arch
b. In the lower-left maternal abdominal 99. When teaching a group of adolescents about
quadrant variations in the length of the menstrual cycle,
c. In the lower-right maternal abdominal the nurse understands that the underlying
quadrant mechanism is due to variations in which of the
d. Above the maternal umbilicus and to the following phases?
left of midline a. Menstrual phase
93. The amniotic fluid of a client has a greenish tint. b. Proliferative phase
The nurse interprets this to be the result of c. Secretory phase
which of the following? d. Ischemic phase
a. Lanugo 100. When teaching a group of adolescents
b. Hydramnio about male hormone production, which of the
c. Meconium following would the nurse include as being
d. Vernix produced by the Leydig cells?
94. A patient is in labor and has just been told she a. Follicle-stimulating hormone
has a breech presentation. The nurse should be b. Testosterone
particularly alert for which of the following? c. Leuteinizing hormone
a. Quickening d. Gonadotropin releasing hormone
b. Ophthalmia neonatorum
c. Pica
d. Prolapsed umbilical cord
95. When describing dizygotic twins to a couple, on
which of the following would the nurse base the
explanation?
a. Two ova fertilized by separate sperm
b. Sharing of a common placenta
c. Each ova with the same genotype
d. Sharing of a common chorion
96. Which of the following refers to the single cell
that reproduces itself after conception?
a. Chromosome
b. Blastocyst
c. Zygote
d. Trophoblast
97. In the late 1950s, consumers and health care
professionals began challenging the routine use
of analgesics and anesthetics during childbirth.
Which of the following was an outgrowth of this
concept?
a. Labor, delivery, recovery, postpartum
(LDRP)
b. Nurse-midwifery
c. Clinical nurse specialist
d. Prepared childbirth
98. A client has a midpelvic contracture from a
previous pelvic injury due to a motor vehicle

41
41
ANSWERS and RATIONALES for MATERNITY A weight gain of 25 to 40 lb is considered
NURSING Part 1 excessive.
8. C. Pressure of the growing uterus on blood
1. B. Although all of the factors listed are vessels results in an increased risk for venous
important, sperm motility is the most significant stasis in the lower extremities. Subsequently,
criterion when assessing male infertility. Sperm edema and varicose vein formation may occur.
count, sperm maturity, and semen volume are Thrombophlebitis is an inflammation of the veins
all significant, but they are not as significant due to thrombus formation. Pregnancy-induced
sperm motility. hypertension is not associated with these
2. D. Based on the partner’s statement, the couple symptoms. Gravity plays only a minor role with
is verbalizing feelings of inadequacy and these symptoms.
negative feelings about themselves and their 9. C. Cervical softening (Goodell sign) and uterine
capabilities. Thus, the nursing diagnosis of self- soufflé are two probable signs of pregnancy.
esteem disturbance is most appropriate. Fear, Probable signs are objective findings that
pain, and ineffective family coping also may be strongly suggest pregnancy. Other probable
present but as secondary nursing diagnoses. signs include Hegar sign, which is softening of
3. B. Pressure and irritation of the bladder by the the lower uterine segment; Piskacek sign, which
growing uterus during the first trimester is is enlargement and softening of the uterus;
responsible for causing urinary frequency. serum laboratory tests; changes in skin
Dysuria, incontinence, and burning are pigmentation; and ultrasonic evidence of a
symptoms associated with urinary tract gestational sac. Presumptive signs are subjective
infections. signs and include amenorrhea; nausea and
4. C. During the second trimester, the reduction in vomiting; urinary frequency; breast tenderness
gastric acidity in conjunction with pressure from and changes; excessive fatigue; uterine
the growing uterus and smooth muscle enlargement; and quickening.
relaxation, can cause heartburn and flatulence. 10. B. Presumptive signs of pregnancy are subjective
HCG levels increase in the first, not the second, signs. Of the signs listed, only nausea and
trimester. Decrease intestinal motility would vomiting are presumptive signs. Hegar sign, skin
most likely be the cause of constipation and pigmentation changes, and a positive serum
bloating. Estrogen levels decrease in the second pregnancy test are considered probably signs,
trimester. which are strongly suggestive of pregnancy.
5. D. Chloasma, also called the mask of pregnancy, 11. D. During the first trimester, common emotional
is an irregular hyperpigmented area found on reactions include ambivalence, fear, fantasies, or
the face. It is not seen on the breasts, areola, anxiety. The second trimester is a period of well-
nipples, chest, neck, arms, legs, abdomen, or being accompanied by the increased need to
thighs. learn about fetal growth and development.
6. C. During pregnancy, hormonal changes cause Common emotional reactions during this
relaxation of the pelvic joints, resulting in the trimester include narcissism, passivity, or
typical “waddling” gait. Changes in posture are introversion. At times the woman may seem
related to the growing fetus. Pressure on the egocentric and self-centered. During the third
surrounding muscles causing discomfort is due trimester, the woman typically feels awkward,
to the growing uterus. Weight gain has no effect clumsy, and unattractive, often becoming more
on gait. introverted or reflective of her own childhood.
7. C. The average amount of weight gained during 12. B. First-trimester classes commonly focus on
pregnancy is 24 to 30 lb. This weight gain such issues as early physiologic changes, fetal
consists of the following: fetus – 7.5 lb; placenta development, sexuality during pregnancy, and
and membrane – 1.5 lb; amniotic fluid – 2 lb; nutrition. Some early classes may include
uterus – 2.5 lb; breasts – 3 lb; and increased pregnant couples. Second and third trimester
blood volume – 2 to 4 lb; extravascular fluid and classes may focus on preparation for birth,
fat – 4 to 9 lb. A gain of 12 to 22 lb is insufficient, parenting, and newborn care.
whereas a weight gain of 15 to 25 lb is marginal. 13. C. With breast feeding, the father’s body is not
capable of providing the milk for the newborn,

41
41
which may interfere with feeding the newborn, 20. D. With true labor, contractions increase in
providing fewer chances for bonding, or he may intensity with walking. In addition, true labor
be jealous of the infant’s demands on his wife’s contractions occur at regular intervals, usually
time and body. Breast feeding is advantageous starting in the back and sweeping around to the
because uterine involution occurs more rapidly, abdomen. The interval of true labor contractions
thus minimizing blood loss. The presence of gradually shortens.
maternal antibodies in breast milk helps 21. B. Crowing, which occurs when the newborn’s
decrease the incidence of allergies in the head or presenting part appears at the vaginal
newborn. A greater chance for error is opening, occurs during the second stage of
associated with bottle feeding. No preparation is labor. During the first stage of labor, cervical
required for breast feeding. dilation and effacement occur. During the third
14. A. A false-positive reaction can occur if the stage of labor, the newborn and placenta are
pregnancy test is performed less than 10 days delivered. The fourth stage of labor lasts from 1
after an abortion. Performing the tests too early to 4 hours after birth, during which time the
or too late in the pregnancy, storing the urine mother and newborn recover from the physical
sample too long at room temperature, or having process of birth and the mother’s organs
a spontaneous or missed abortion impending undergo the initial readjustment to the
can all produce false-negative results. nonpregnant state.
15. D. The FHR can be auscultated with a fetoscope 22. C. Barbiturates are rapidly transferred across the
at about 20 week’s gestation. FHR usually is placental barrier, and lack of an antagonist
ausculatated at the midline suprapubic region makes them generally inappropriate during
with Doppler ultrasound transducer at 10 to 12 active labor. Neonatal side effects of
week’s gestation. FHR, cannot be heard any barbiturates include central nervous system
earlier than 10 weeks’ gestation. depression, prolonged drowsiness, delayed
16. C. To determine the EDD when the date of the establishment of feeding (e.g. due to poor
client’s LMP is known use Nagele rule. To the sucking reflex or poor sucking pressure).
first day of the LMP, add 7 days, subtract 3 Tranquilizers are associated with neonatal
months, and add 1 year (if applicable) to arrive effects such as hypotonia, hypothermia,
at the EDD as follows: 5 + 7 = 12 (July) minus 3 = generalized drowsiness, and reluctance to feed
4 (April). Therefore, the client’s EDD is April 12. for the first few days. Narcotic analgesic readily
17. A. When the LMP is unknown, the gestational cross the placental barrier, causing depressive
age of the fetus is estimated by uterine size or effects in the newborn 2 to 3 hours after
position (fundal height). The presence of the intramuscular injection. Regional anesthesia is
uterus in the pelvis indicates less than 12 weeks’ associated with adverse reactions such as
gestation. At approximately 12 to 14 weeks, the maternal hypotension, allergic or toxic reaction,
fundus is out of the pelvis above the symphysis or partial or total respiratory failure.
pubis. The fundus is at the level of the umbilicus 23. D. During the third stage of labor, which begins
at approximately 20 weeks’ gestation and with the delivery of the newborn, the nurse
reaches the xiphoid at term or 40 weeks. would promote parent-newborn interaction by
18. D. Danger signs that require prompt reporting placing the newborn on the mother’s abdomen
leaking of amniotic fluid, vaginal bleeding, and encouraging the parents to touch the
blurred vision, rapid weight gain, and elevated newborn. Collecting a urine specimen and other
blood pressure. Constipation, breast tenderness, laboratory tests is done on admission during the
and nasal stuffiness are common discomforts first stage of labor. Assessing uterine
associated with pregnancy. contractions every 30 minutes is performed
19. B. A rubella titer should be 1:8 or greater. Thurs, during the latent phase of the first stage of
a finding of a titer less than 1:8 is significant, labor. Coaching the client to push effectively is
indicating that the client may not possess appropriate during the second stage of labor.
immunity to rubella. A hematocrit of 33.5% a 24. A. The newborn’s ability to regulate body
white blood cell count of 8,000/mm3, and a 1 temperature is poor. Therefore, placing the
hour glucose challenge test of 110 g/dl are with newborn under a radiant warmer aids in
normal parameters. maintaining his or her body temperature.

41
42
Suctioning with a bulb syringe helps maintain a legs when he is placed on his abdomen on a flat
patent airway. Obtaining an Apgar score surface.
measures the newborn’s immediate adjustment 32. B. The description of hyperemesis gravidarum
to extrauterine life. Inspecting the umbilical cord includes severe nausea and vomiting, leading to
aids in detecting cord anomalies. electrolyte, metabolic, and nutritional
25. D. Immediately before expulsion or birth of the imbalances in the absence of other medical
rest of the body, the cardinal movement of problems. Hyperemesis is not a form of anemia.
external rotation occurs. Descent flexion, Loss of appetite may occur secondary to the
internal rotation, extension, and restitution (in nausea and vomiting of hyperemesis, which, if it
this order) occur before external rotation. continues, can deplete the nutrients transported
26. B. The foramen ovale is an opening between the to the fetus. Diarrhea does not occur with
right and left auricles (atria) that should close hyperemesis.
shortly after birth so the newborn will not have a 33. B. Edema of the hands and face is a classic sign
murmur or mixed blood traveling through the of PIH. Many healthy pregnant woman
vascular system. The umbilical vein, ductus experience foot and ankle edema. A weight gain
arteriosus, and ductus venosus are obliterated at of 2 lb or more per week indicates a problem.
birth. Early morning headache is not a classic sign of
27. B. Uric acid crystals in the urine may produce the PIH.
reddish “brick dust” stain on the diaper. Mucus 34. C. In a missed abortion, there is early fetal
would not produce a stain. Bilirubin and iron are intrauterine death, and products of conception
from hepatic adaptation. are not expelled. The cervix remains closed;
28. B. The normal heart rate for a newborn that is there may be a dark brown vaginal discharge,
sleeping is approximately 100 beats per minute. negative pregnancy test, and cessation of
If the newborn was awake, the normal heart rate uterine growth and breast tenderness. A
would range from 120 to 160 beats per minute. threatened abortion is evidenced with cramping
29. C. The anterior fontanel is larger in size than the and vaginal bleeding in early pregnancy, with no
posterior fontanel. Additionally, the anterior cervical dilation. An incomplete abortion
fontanel, which is diamond shaped, closes at 18 presents with bleeding, cramping, and cervical
months, whereas the posterior fontanel, which dilation. An incomplete abortion involves only
is triangular shaped, closes at 8 to 12 weeks. expulsion of part of the products of conception
Neither fontanel should appear bulging, which and bleeding occurs with cervical dilation.
may indicate increased intracranial pressure, or 35. A. Multiple gestation is one of the predisposing
sunken, which may indicate dehydration. factors that may cause placenta previa. Uterine
30. B. Blink, cough, sneeze, swallowing and gag anomalies abdominal trauma, and renal or
reflexes are all present at birth and remain vascular disease may predispose a client to
unchanged through adulthood. Reflexes such as abruptio placentae.
rooting and stepping subside within the first 36. B. A client with abruptio placentae may exhibit
year. concealed or dark red bleeding, possibly
31. A. With the babinski reflex, the newborn’s toes reporting sudden intense localized uterine pain.
hyperextend and fan apart from dorsiflexion of The uterus is typically firm to boardlike, and the
the big toe when one side of foot is stroked fetal presenting part may be engaged. Bright
upward form the heel and across the ball of the red, painless vaginal bleeding, a palpable fetal
foot. With the startle reflex, the newborn outline and a soft nontender abdomen are
abducts and flexes all extremities and may begin manifestations of placenta previa.
to cry when exposed to sudden movement of 37. D. Abruptio placentae is described as premature
loud noise. With the rooting and sucking reflex, separation of a normally implanted placenta
the newborn turns his head in the direction of during the second half of pregnancy, usually with
stimulus, opens the mouth, and begins to suck severe hemorrhage. Placenta previa refers to
when the cheeks, lip, or corner of mouth is implantation of the placenta in the lower uterine
touched. With the crawl reflex, the newborn will segment, causing painless bleeding in the third
attempt to crawl forward with both arms and trimester of pregnancy. Ectopic pregnancy refers
to the implantation of the products of

42
42
conception in a site other than the factors may contribute to the mechanical factors
endometrium. Incompetent cervix is a that cause dystocia.
conduction characterized by painful dilation of 43. A. With uterine rupture, the client is at risk for
the cervical os without uterine contractions. hypovolemic shock. Therefore, the priority is to
38. B. Hyperstimulation of the uterus such as with prevent and limit hypovolemic shock. Immediate
oxytocin during the induction of labor may result steps should include giving oxygen, replacing lost
in tetanic contractions prolonged to more than fluids, providing drug therapy as needed,
90seconds, which could lead to such evaluating fetal responses and preparing for
complications as fetal distress, abruptio surgery. Obtaining blood specimens, instituting
placentae, amniotic fluid embolism, laceration of complete bed rest, and inserting a urinary
the cervix, and uterine rupture. Weak catheter are necessary in preparation for surgery
contractions would not occur. Pain, bright red to remedy the rupture.
vaginal bleeding, and increased restlessness and 44. B. The immediate priority is to minimize
anxiety are not associated with pressure on the cord. Thus the nurse’s initial
hyperstimulation. action involves placing the client on bed rest and
39. C. A key point to consider when preparing the then placing the client in a knee-chest position
client for a cesarean delivery is to modify the or lowering the head of the bed, and elevating
preoperative teaching to meet the needs of the maternal hips on a pillow to minimize the
either a planned or emergency cesarean birth, pressure on the cord. Monitoring maternal vital
the depth and breadth of instruction will depend signs and FHR, notifying the physician and
on circumstances and time available. Allowing preparing the client for delivery, and wrapping
the mother’s support person to remain with her the cord with sterile saline soaked warm gauze
as much as possible is an important concept, are important. But these actions have no effect
although doing so depends on many variables. on minimizing the pressure on the cord.
Arranging for necessary explanations by various 45. D. Postpartum hemorrhage is defined as blood
staff members to be involved with the client’s loss of more than 500 ml following birth. Any
care is a nursing responsibility. The nurse is amount less than this not considered
responsible for reinforcing the explanations postpartum hemorrhage.
about the surgery, expected outcome, and type 46. D. With mastitis, injury to the breast, such as
of anesthetic to be used. The obstetrician is overdistention, stasis, and cracking of the
responsible for explaining about the surgery and nipples, is the primary predisposing factor.
outcome and the anesthesiology staff is Epidemic and endemic infections are probable
responsible for explanations about the type of sources of infection for mastitis. Temporary
anesthesia to be used. urinary retention due to decreased perception of
40. A. Preterm labor is best described as labor that the urge to void is a contributory factor to the
begins after 20 weeks’ gestation and before 37 development of urinary tract infection, not
weeks’ gestation. The other time periods are mastitis.
inaccurate. 47. D. Thrombophlebitis refers to an inflammation
41. B. PROM can precipitate many potential and of the vascular endothelium with clot formation
actual problems; one of the most serious is the on the wall of the vessel. Blood components
fetus loss of an effective defense against combining to form an aggregate body describe a
infection. This is the client’s most immediate thrombus or thrombosis. Clots lodging in the
need at this time. Typically, PROM occurs about pulmonary vasculature refers to pulmonary
1 hour, not 4 hours, before labor begins. Fetal embolism; in the femoral vein, femoral
viability and gestational age are less immediate thrombophlebitis.
considerations that affect the plan of care. 48. C. Classic symptoms of DVT include muscle pain,
Malpresentation and an incompetent cervix may the presence of Homans sign, and swelling of the
be causes of PROM. affected limb. Midcalf pain, tenderness, and
42. B. Dystocia is difficult, painful, prolonged labor redness, along the vein reflect superficial
due to mechanical factors involving the fetus thrombophlebitis. Chills, fever and malaise
(passenger), uterus (powers), pelvis (passage), or occurring 2 weeks after delivery reflect pelvic
psyche. Nutritional, environment, and medical thrombophlebitis. Chills, fever, stiffness and

42
42
pain occurring 10 to 14 days after delivery approximately 6 weeks. Use of a female condom
suggest femoral thrombophlebitis. protects the reproductive system from the
49. B. Manifestations of cystitis include, frequency, introduction of semen or spermicides into the
urgency, dysuria, hematuria nocturia, fever, and vagina and may be used after childbirth. Oral
suprapubic pain. Dehydration, hypertension, and contraceptives may be started within the first
chills are not typically associated with cystitis. postpartum week to ensure suppression of
High fever chills, flank pain, nausea, vomiting, ovulation. For the couple who has determined
dysuria, and frequency are associated with the female’s fertile period, using the rhythm
pvelonephritis. method, avoidance of intercourse during this
50. C. According to statistical reports, between 50% period, is safe and effective.
and 80% of all new mothers report some form of 54. C. An IUD may increase the risk of pelvic
postpartum blues. The ranges of 10% to 40%, inflammatory disease, especially in women with
30% to 50%, and 25% to 70% are incorrect. more than one sexual partner, because of the
51. B. Regular timely ingestion of oral contraceptives increased risk of sexually transmitted infections.
is necessary to maintain hormonal levels of the An UID should not be used if the woman has an
drugs to suppress the action of the active or chronic pelvic infection, postpartum
hypothalamus and anterior pituitary leading to infection, endometrial hyperplasia or carcinoma,
inappropriate secretion of FSH and LH. or uterine abnormalities. Age is not a factor in
Therefore, follicles do not mature, ovulation is determining the risks associated with IUD use.
inhibited, and pregnancy is prevented. The Most IUD users are over the age of 30. Although
estrogen content of the oral site contraceptive there is a slightly higher risk for infertility in
may cause the nausea, regardless of when the women who have never been pregnant, the IUD
pill is taken. Side effects and drug interactions is an acceptable option as long as the risk-
may occur with oral contraceptives regardless of benefit ratio is discussed. IUDs may be inserted
the time the pill is taken. immediately after delivery, but this is not
52. C. Condoms, when used correctly and recommended because of the increased risk and
consistently, are the most effective rate of expulsion at this time.
contraceptive method or barrier against 55. C. During the third trimester, the enlarging
bacterial and viral sexually transmitted uterus places pressure on the intestines. This
infections. Although spermicides kill sperm, they coupled with the effect of hormones on smooth
do not provide reliable protection against the muscle relaxation causes decreased intestinal
spread of sexually transmitted infections, motility (peristalsis). Increasing fiber in the diet
especially intracellular organisms such as HIV. will help fecal matter pass more quickly through
Insertion and removal of the diaphragm along the intestinal tract, thus decreasing the amount
with the use of the spermicides may cause of water that is absorbed. As a result, stool is
vaginal irritations, which could place the client at softer and easier to pass. Enemas could
risk for infection transmission. Male sterilization precipitate preterm labor and/or electrolyte loss
eliminates spermatozoa from the ejaculate, but and should be avoided. Laxatives may cause
it does not eliminate bacterial and/or viral preterm labor by stimulating peristalsis and may
microorganisms that can cause sexually interfere with the absorption of nutrients. Use
transmitted infections. for more than 1 week can also lead to laxative
53. A. The diaphragm must be fitted individually to dependency. Liquid in the diet helps provide a
ensure effectiveness. Because of the changes to semisolid, soft consistency to the stool. Eight to
the reproductive structures during pregnancy ten glasses of fluid per day are essential to
and following delivery, the diaphragm must be maintain hydration and promote stool
refitted, usually at the 6 weeks’ examination evacuation.
following childbirth or after a weight loss of 15 56. D. To ensure adequate fetal growth and
lbs or more. In addition, for maximum development during the 40 weeks of a
effectiveness, spermicidal jelly should be placed pregnancy, a total weight gain 25 to 30 pounds is
in the dome and around the rim. However, recommended: 1.5 pounds in the first 10 weeks;
spermicidal jelly should not be inserted into the 9 pounds by 30 weeks; and 27.5 pounds by 40
vagina until involution is completed at weeks. The pregnant woman should gain less

42
42
weight in the first and second trimester than in pregnant women and especially for diabetic
the third. During the first trimester, the client women, because it burns up glucose, thus
should only gain 1.5 pounds in the first 10 decreasing blood sugar. However, dietary intake,
weeks, not 1 pound per week. A weight gain of not exercise, is the priority. All pregnant women
½ pound per week would be 20 pounds for the with diabetes should have periodic monitoring
total pregnancy, less than the recommended of serum glucose. However, those with
amount. gestational diabetes generally do not need daily
57. B. To calculate the EDD by Nagele’s rule, add 7 glucose monitoring. The standard of care
days to the first day of the last menstrual period recommends a fasting and 2-hour postprandial
and count back 3 months, changing the year blood sugar level every 2 weeks.
appropriately. To obtain a date of September 27, 61. C. After 20 weeks’ gestation, when there is a
7 days have been added to the last day of the rapid weight gain, preeclampsia should be
LMP (rather than the first day of the LMP), plus suspected, which may be caused by fluid
4 months (instead of 3 months) were counted retention manifested by edema, especially of the
back. To obtain the date of November 7, 7 days hands and face. The three classic signs of
have been subtracted (instead of added) from preeclampsia are hypertension, edema, and
the first day of LMP plus November indicates proteinuria. Although urine is checked for
counting back 2 months (instead of 3 months) glucose at each clinic visit, this is not the priority.
from January. To obtain the date of December Depression may cause either anorexia or
27, 7 days were added to the last day of the LMP excessive food intake, leading to excessive
(rather than the first day of the LMP) and weight gain or loss. This is not, however, the
December indicates counting back only 1 month priority consideration at this time. Weight gain
(instead of 3 months) from January. thought to be caused by excessive food intake
58. D. The client has been pregnant four times, would require a 24-hour diet recall. However,
including current pregnancy (G). Birth at 38 excessive intake would not be the primary
weeks’ gestation is considered full term (T), consideration for this client at this time.
while birth form 20 weeks to 38 weeks is 62. B. Cramping and vaginal bleeding coupled with
considered preterm (P). A spontaneous abortion cervical dilation signifies that termination of the
occurred at 8 weeks (A). She has two living pregnancy is inevitable and cannot be
children (L). prevented. Thus, the nurse would document an
59. B. At 12 weeks gestation, the uterus rises out of imminent abortion. In a threatened abortion,
the pelvis and is palpable above the symphysis cramping and vaginal bleeding are present, but
pubis. The Doppler intensifies the sound of the there is no cervical dilation. The symptoms may
fetal pulse rate so it is audible. The uterus has subside or progress to abortion. In a complete
merely risen out of the pelvis into the abdominal abortion all the products of conception are
cavity and is not at the level of the umbilicus. expelled. A missed abortion is early fetal
The fetal heart rate at this age is not audible intrauterine death without expulsion of the
with a stethoscope. The uterus at 12 weeks is products of conception.
just above the symphysis pubis in the abdominal 63. B. For the client with an ectopic pregnancy,
cavity, not midway between the umbilicus and lower abdominal pain, usually unilateral, is the
the xiphoid process. At 12 weeks the FHR would primary symptom. Thus, pain is the priority.
be difficult to auscultate with a fetoscope. Although the potential for infection is always
Although the external electronic fetal monitor present, the risk is low in ectopic pregnancy
would project the FHR, the uterus has not risen because pathogenic microorganisms have not
to the umbilicus at 12 weeks. been introduced from external sources. The
60. A. Although all of the choices are important in client may have a limited knowledge of the
the management of diabetes, diet therapy is the pathology and treatment of the condition and
mainstay of the treatment plan and should will most likely experience grieving, but this is
always be the priority. Women diagnosed with not the priority at this time.
gestational diabetes generally need only diet 64. D. Before uterine assessment is performed, it is
therapy without medication to control their essential that the woman empty her bladder. A
blood sugar levels. Exercise, is important for all full bladder will interfere with the accuracy of

42
42
the assessment by elevating the uterus and caused by retained placental fragments or
displacing to the side of the midline. Vital sign bleeding disorders. Lochia rubra is the normal
assessment is not necessary unless an dark red discharge occurring in the first 2 to 3
abnormality in uterine assessment is identified. days after delivery, containing epithelial cells,
Uterine assessment should not cause acute pain erythrocyes, leukocytes and decidua. Lochia
that requires administration of analgesia. serosa is a pink to brownish serosanguineous
Ambulating the client is an essential component discharge occurring from 3 to 10 days after
of postpartum care, but is not necessary prior to delivery that contains decidua, erythrocytes,
assessment of the uterus. leukocytes, cervical mucus, and microorganisms.
65. A. Feeding more frequently, about every 2 Lochia alba is an almost colorless to yellowish
hours, will decrease the infant’s frantic, vigorous discharge occurring from 10 days to 3 weeks
sucking from hunger and will decrease breast after delivery and containing leukocytes,
engorgement, soften the breast, and promote decidua, epithelial cells, fat, cervical mucus,
ease of correct latching-on for feeding. Narcotics cholesterol crystals, and bacteria.
administered prior to breast feeding are passed 68. A. The data suggests an infection of the
through the breast milk to the infant, causing endometrial lining of the uterus. The lochia may
excessive sleepiness. Nipple soreness is not be decreased or copious, dark brown in
severe enough to warrant narcotic analgesia. All appearance, and foul smelling, providing further
postpartum clients, especially lactating mothers, evidence of a possible infection. All the client’s
should wear a supportive brassiere with wide data indicate a uterine problem, not a breast
cotton straps. This does not, however, prevent problem. Typically, transient fever, usually
or reduce nipple soreness. Soaps are drying to 101ºF, may be present with breast
the skin of the nipples and should not be used engorgement. Symptoms of mastitis include
on the breasts of lactating mothers. Dry nipple influenza-like manifestations. Localized infection
skin predisposes to cracks and fissures, which of an episiotomy or C-section incision rarely
can become sore and painful. causes systemic symptoms, and uterine
66. D. A weak, thready pulse elevated to 100 BPM involution would not be affected. The client data
may indicate impending hemorrhagic shock. An do not include dysuria, frequency, or urgency,
increased pulse is a compensatory mechanism of symptoms of urinary tract infections, which
the body in response to decreased fluid volume. would necessitate assessing the client’s urine.
Thus, the nurse should check the amount of 69. C. Because of early postpartum discharge and
lochia present. Temperatures up to 100.48F in limited time for teaching, the nurse’s priority is
the first 24 hours after birth are related to the to facilitate the safe and effective care of the
dehydrating effects of labor and are considered client and newborn. Although promoting
normal. Although rechecking the blood pressure comfort and restoration of health, exploring the
may be a correct choice of action, it is not the family’s emotional status, and teaching about
first action that should be implemented in light family planning are important in
of the other data. The data indicate a potential postpartum/newborn nursing care, they are not
impending hemorrhage. Assessing the uterus for the priority focus in the limited time presented
firmness and position in relation to the umbilicus by early post-partum discharge.
and midline is important, but the nurse should 70. C. Heat loss by radiation occurs when the
check the extent of vaginal bleeding first. Then it infant’s crib is placed too near cold walls or
would be appropriate to check the uterus, which windows. Thus placing the newborn’s crib close
may be a possible cause of the hemorrhage. to the viewing window would be least effective.
67. D. Any bright red vaginal discharge would be Body heat is lost through evaporation during
considered abnormal, but especially 5 days after bathing. Placing the infant under the radiant
delivery, when the lochia is typically pink to warmer after bathing will assist the infant to be
brownish. Lochia rubra, a dark red discharge, is rewarmed. Covering the scale with a warmed
present for 2 to 3 days after delivery. Bright red blanket prior to weighing prevents heat loss
vaginal bleeding at this time suggests late through conduction. A knit cap prevents heat
postpartum hemorrhage, which occurs after the loss from the head a large head, a large body
first 24 hours following delivery and is generally surface area of the newborn’s body.

42
42
71. B. A fractured clavicle would prevent the normal cord falls off and the stump has completely
Moro response of symmetrical sequential healed.
extension and abduction of the arms followed by 76. B. To determine the amount of formula needed,
flexion and adduction. In talipes equinovarus do the following mathematical calculation. 3 kg x
(clubfoot) the foot is turned medially, and in 120 cal/kg per day = 360 calories/day feeding q
plantar flexion, with the heel elevated. The feet 4 hours = 6 feedings per day = 60 calories per
are not involved with the Moro reflex. feeding: 60 calories per feeding; 60 calories per
Hypothyroiddism has no effect on the primitive feeding with formula 20 cal/oz = 3 ounces per
reflexes. Absence of the Moror reflex is the most feeding. Based on the calculation. 2, 4 or 6
significant single indicator of central nervous ounces are incorrect.
system status, but it is not a sign of increased 77. A. Intrauterine anoxia may cause relaxation of
intracranial pressure. the anal sphincter and emptying of meconium
72. B. Hemorrhage is a potential risk following any into the amniotic fluid. At birth some of the
surgical procedure. Although the infant has been meconium fluid may be aspirated, causing
given vitamin K to facilitate clotting, the mechanical obstruction or chemical
prophylactic dose is often not sufficient to pneumonitis. The infant is not at increased risk
prevent bleeding. Although infection is a for gastrointestinal problems. Even though the
possibility, signs will not appear within 4 hours skin is stained with meconium, it is noninfectious
after the surgical procedure. The primary (sterile) and nonirritating. The postterm
discomfort of circumcision occurs during the meconium-stained infant is not at additional risk
surgical procedure, not afterward. Although for bowel or urinary problems.
feedings are withheld prior to the circumcision, 78. C. The nurse should use a nonelastic, flexible,
the chances of dehydration are minimal. paper measuring tape, placing the zero point on
73. B. The presence of excessive estrogen and the superior border of the symphysis pubis and
progesterone in the maternal-fetal blood stretching the tape across the abdomen at the
followed by prompt withdrawal at birth midline to the top of the fundus. The xiphoid and
precipitates breast engorgement, which will umbilicus are not appropriate landmarks to use
spontaneously resolve in 4 to 5 days after birth. when measuring the height of the fundus
The trauma of the birth process does not cause (McDonald’s measurement).
inflammation of the newborn’s breast tissue. 79. B. Women hospitalized with severe
Newborns do not have breast malignancy. This preeclampsia need decreased CNS stimulation to
reply by the nurse would cause the mother to prevent a seizure. Seizure precautions provide
have undue anxiety. Breast tissue does not environmental safety should a seizure occur.
hypertrophy in the fetus or newborns. Because of edema, daily weight is important but
74. D. The first 15 minutes to 1 hour after birth is not the priority. Preclampsia causes vasospasm
the first period of reactivity involving respiratory and therefore can reduce utero-placental
and circulatory adaptation to extrauterine life. perfusion. The client should be placed on her left
The data given reflect the normal changes during side to maximize blood flow, reduce blood
this time period. The infant’s assessment data pressure, and promote diuresis. Interventions to
reflect normal adaptation. Thus, the physician reduce stress and anxiety are very important to
does not need to be notified and oxygen is not facilitate coping and a sense of control, but
needed. The data do not indicate the presence seizure precautions are the priority.
of choking, gagging or coughing, which are signs 80. C. Cessation of the lochial discharge signifies
of excessive secretions. Suctioning is not healing of the endometrium. Risk of hemorrhage
necessary. and infection are minimal 3 weeks after a
75. B. Application of 70% isopropyl alcohol to the normal vaginal delivery. Telling the client
cord minimizes microorganisms (germicidal) and anytime is inappropriate because this response
promotes drying. The cord should be kept dry does not provide the client with the specific
until it falls off and the stump has healed. information she is requesting. Choice of a
Antibiotic ointment should only be used to treat contraceptive method is important, but not the
an infection, not as a prophylaxis. Infants should specific criteria for safe resumption of sexual
not be submerged in a tub of water until the activity. Culturally, the 6-weeks’ examination has

42
42
been used as the time frame for resuming sexual 87. C. Breathing techniques can raise the pain
activity, but it may be resumed earlier. threshold and reduce the perception of pain.
81. C. The middle third of the vastus lateralis is the They also promote relaxation. Breathing
preferred injection site for vitamin K techniques do not eliminate pain, but they can
administration because it is free of blood vessels reduce it. Positioning, not breathing, increases
and nerves and is large enough to absorb the uteroplacental perfusion.
medication. The deltoid muscle of a newborn is 88. A. The client’s labor is hypotonic. The nurse
not large enough for a newborn IM injection. should call the physical and obtain an order for
Injections into this muscle in a small child might an infusion of oxytocin, which will assist the
cause damage to the radial nerve. The anterior uterus to contact more forcefully in an attempt
femoris muscle is the next safest muscle to use to dilate the cervix. Administering light sedative
in a newborn but is not the safest. Because of would be done for hypertonic uterine
the proximity of the sciatic nerve, the gluteus contractions. Preparing for cesarean section is
maximus muscle should not be until the child unnecessary at this time. Oxytocin would
has been walking 2 years. increase the uterine contractions and hopefully
82. D. Bartholin’s glands are the glands on either progress labor before a cesarean would be
side of the vaginal orifice. The clitoris is female necessary. It is too early to anticipate client
erectile tissue found in the perineal area above pushing with contractions.
the urethra. The parotid glands are open into the 89. D. The signs indicate placenta previa and vaginal
mouth. Skene’s glands open into the posterior exam to determine cervical dilation would not
wall of the female urinary meatus. be done because it could cause hemorrhage.
83. D. The fetal gonad must secrete estrogen for the Assessing maternal vital signs can help
embryo to differentiate as a female. An increase determine maternal physiologic status. Fetal
in maternal estrogen secretion does not effect heart rate is important to assess fetal well-being
differentiation of the embryo, and maternal and should be done. Monitoring the contractions
estrogen secretion occurs in every pregnancy. will help evaluate the progress of labor.
Maternal androgen secretion remains the same 90. D. A complete placenta previa occurs when the
as before pregnancy and does not effect placenta covers the opening of the uterus, thus
differentiation. Secretion of androgen by the blocking the passageway for the baby. This
fetal gonad would produce a male fetus. response explains what a complete previa is and
84. A. Using bicarbonate would increase the amount the reason the baby cannot come out except by
of sodium ingested, which can cause cesarean delivery. Telling the client to ask the
complications. Eating low-sodium crackers physician is a poor response and would increase
would be appropriate. Since liquids can increase the patient’s anxiety. Although a cesarean would
nausea avoiding them in the morning hours help to prevent hemorrhage, the statement does
when nausea is usually the strongest is not explain why the hemorrhage could occur.
appropriate. Eating six small meals a day would With a complete previa, the placenta is covering
keep the stomach full, which often decrease all the cervix, not just most of it.
nausea. 91. B. With a face presentation, the head is
85. B. Ballottement indicates passive movement of completely extended. With a vertex
the unengaged fetus. Ballottement is not a presentation, the head is completely or partially
contraction. Fetal kicking felt by the client flexed. With a brow (forehead) presentation, the
represents quickening. Enlargement and head would be partially extended.
softening of the uterus is known as Piskacek’s 92. D. With this presentation, the fetal upper torso
sign. and back face the left upper maternal abdominal
86. B. Chadwick’s sign refers to the purple-blue tinge wall. The fetal heart rate would be most audible
of the cervix. Braxton Hicks contractions are above the maternal umbilicus and to the left of
painless contractions beginning around the 4 th the middle. The other positions would be
month. Goodell’s sign indicates softening of the incorrect.
cervix. Flexibility of the uterus against the cervix 93. C. The greenish tint is due to the presence of
is known as McDonald’s sign. meconium. Lanugo is the soft, downy hair on the
shoulders and back of the fetus. Hydramnios

42
42
represents excessive amniotic fluid. Vernix is the
white, cheesy substance covering the fetus.
94. D. In a breech position, because of the space
between the presenting part and the cervix,
prolapse of the umbilical cord is common.
Quickening is the woman’s first perception of
fetal movement. Ophthalmia neonatorum
usually results from maternal gonorrhea and is
conjunctivitis. Pica refers to the oral intake of
nonfood substances.
95. A. Dizygotic (fraternal) twins involve two ova
fertilized by separate sperm. Monozygotic
(identical) twins involve a common placenta,
same genotype, and common chorion.
96. C. The zygote is the single cell that reproduces
itself after conception. The chromosome is the
material that makes up the cell and is gained
from each parent. Blastocyst and trophoblast are
later terms for the embryo after zygote.
97. D. Prepared childbirth was the direct result of
the 1950’s challenging of the routine use of
analgesic and anesthetics during childbirth. The
LDRP was a much later concept and was not a
direct result of the challenging of routine use of
analgesics and anesthetics during childbirth.
Roles for nurse midwives and clinical nurse
specialists did not develop from this challenge.
98. C. The ischial spines are located in the mid-pelvic
region and could be narrowed due to the
previous pelvic injury. The symphysis pubis,
sacral promontory, and pubic arch are not part
of the mid-pelvis.
99. B. Variations in the length of the menstrual cycle
are due to variations in the proliferative phase.
The menstrual, secretory and ischemic phases
do not contribute to this variation.
100. B. Testosterone is produced by the
Leyding cells in the seminiferous tubules.
Follicle-stimulating hormone and leuteinzing
hormone are released by the anterior pituitary
gland. The hypothalamus is responsible for
releasing gonadotropin-releasing hormone.

42
42
MATERNITY NURSING Part 2 months pregnant. Which of Duvall’s family life
stages is the family currently experiencing?
1. Suppose Melissa Chung asks you whether a. Pregnancy stage
maternal child health nursing is a profession. b. Preschool stage
What qualifies an activity as a profession? c. School-age stage
a. Members supervise other people d. Launching stage
b. Members use a distinct body of knowledge
c. Members enjoy good working conditions 7. While she is in the hospital, Carmela makes the
d. Members receive relatively high pay following statements. Which is the best example
of stereotyping?
2. Nursing is changing because social change a. My doctor is funny; he tells jokes and makes me
affects care. Which of the following is a trend laugh.
that is occurring in nursing because of social b. I’m glad I’m Batangueño because all
change? Batangueños are smart.
a. So many children are treated in ambulatory c. I’m sure my leg will heal quickly; I’m overall
units that nurses are hardly needed healthy.
b. Immunizations are no longer needed for d. I like foods in Batangas, although not if it tastes
infectious diseases too spicy.
c. The use of skilled technology has made
nursing care more complex 8. Monet Rivera tells you she used to wrry
d. Pregnant women are so healthy today that because she developed breasts later than most
they rarely need prenatal care of her friends. Breast development is termed:
a. Adrenarche
3. The best description if the family nurse b. Mamarche
practitioner role is c. Thelarche
a. To give bedside care to critically ill family d. Menarche
members
b. To supervise the health of children up to age 9. Suppose Jaypee Manalo tells you that he is
18 years considering a vasectomy after the birth of his
c. To provide health supervision for families new child. Vasectomy is the incision of which
d. To supervise women during pregnancy organ?
a. Testes
4. The Delos Reyes family was a single-parent one b. Vas deferens
before Mrs. Delos Reyes remarried. What is a c. Fallopian tube
common concern of single-parent families? d. Epididymis
a. Too many people give advice
b. Finances are inadequate 10. On physical examination, Monet Rivera is found
c. Children miss many days of school to have cystocele. A cystocele is:
d. Children don’t know any other family like theirs a. A sebaceous cyst arising from a vulvar fold
b. Protrusion of the intestine into the vagina
5. Mrs. Delos Reyes serves many roles in her c. Prolapse of the uterus and cervix into the vagina
family. If, when you talk to Veronica, her d. Herniation of the bladder into the vaginal wall
daughter, she interrupts to say, “Don’t tell our
family secrets,” she is fulfilling what family role? 11. Monet Rivera typically has a menstrual cycle of
a. Decision-maker 34 days. She tells you she had coitus on days 8,
b. Gatekeeper 10, 15, and 20 of her last cycle. Which is the day
c. Problem-solver on which she most likely conceived?
d. Bread-earner a. The 8th day
b. The 10th day
6. The Delos Reyes family consists of two parents; c. Day 15
Veronica, 12; and Paolo, 2. Mrs. Delos Reyes is 5 d. Day 20

42
42
12. The Manalo’s neighbor Cahrell is a woman who 17. Roseann, 17 years old, asks you how a tubal
has sex with women. Another term for this ligation prevents pregnancy. Which would be the
sexual orientation is best answer?
a. Lesbian a. Sperm can no longer reach the ova because
b. Celibate fallopian tubes are blocked
c. Gay b. Sperm can not enter the uterus because the
d. Voyeur cervical entrance is blocked
c. Prostaglandins released from the cut fallopian
13. Suppose Roseann, 17 years old, tells you that tubes can kill sperm
she wants to use fertility awareness method of d. The ovary no longer releases ova as there is no
contraception. How will she determine her where for them to go
fertile days?
a. She will notice that she feels hot, as if she has an 18. The Atienzas are a couple undergoing testing for
elevated temperature infertility. Infertility is said to exist when:
b. She should assess whether her cervical mucus is a. A couple has been trying to conceive for 1 year
thin and watery b. A woman has no children
c. She should monitor her emotions for sudden c. A woman has no uterus
anger or crying d. A couple has wanted a child for 6 months
d. She should assess whether her breasts feels
sensitive to cool air 19. Guadalupe Atienza is diagnosed as having
endometriosis. This condition interferes with
14. Suppose Roseann, 17 years old, chooses to use a fertility because:
combination oral contraceptive (COC) as her a. The ovaries stop producing adequate estrogen
family planning method. What is a danger sign of b. The uterine cervix becomes inflamed and
COCs you would ask her to report? swollen
a. A stuffy or runny nose c. Pressure on the pituitary leads to decreased FSH
b. Arthritis-like symptoms levels
c. Slight weight gain d. Endometrial implants can block the fallopian
d. Migraine headache tubes

15. Suppose Roseann, 17 years old, chooses 20. Guadalupe Atienza is scheduled to have a
subcutaneous implants (Norplant) as her hysterosalpingogram. Which of the following
method of reproductive life planning. How long instructions would you give her regarding this
will these implants be effective? procedure?
a. One month a. She may feel some mild cramping when the dye
b. 12 months is inserted
c. Five years b. The sonogram of the uterus will reveal any
d. 10 years tumors present
c. She will not be able to conceive for three
16. Roseann, 17 years old, wants to try female months after the procedure
condoms as her reproductive life planning d. May women experience mild bleeding as an
method. Which instruction would you give her? aftereffect
a. The hormone the condom releases may cause
mild weight gain. 21. Ruel Marasigan asks you what artificial
b. She should insert the condom before any penile insemination by donor entails. Which would be
penetration your best answer?
c. She should coat the condom with a spermicide a. Artificial sperm are injected vaginally to test
before use tubal patency
d. Female condoms, unlike male condoms, can be b. Donor sperm are introduced vaginally into the
reused. uterus of the cervix
c. The husband’s sperm is administered
intravenously weekly

42
43
d. Donor sperm are injected intraabdominally into d. Wrinkles on soles of the feet
each ovary
27. Rizalyn asks how much longer her doctor will
22. Guadalupe Atienza is having a gamete refer to the baby inside her as an embryo. What
intrafallopian transfer (GIFT) procedure. What would be your best explanation?
makes her a good candidate for this procedure? a. This term is used during the time before
a. She has patent fallopian tubes, so fertilized ova fertilization
can be implanted into them b. Her baby will be a fetus as soon as the placenta
b. She is Rh negative, a necessary stipulation to forms
rule out Rh incompatibility c. After the 20th week of pregnancy, the baby is
c. She has a normal uterus, so sperm can be called zygote
injected through the cervix into it d. From the time of implantation until 5 to 8
d. Her husband is taking sildenafil (Viagra), so all weeks, the baby is an embryo
his perm will be motile
28. Rizalyn is worried that her baby will be born with
23. Jean Suarez is pregnant with her first child. Her congenital heart disease. What assessment of a
phenotype refers to: fetus at birth is important to help detect
a. Her concept of herself as male or female congenital heart defects?
b. Whether she has 46 chromosomes or not a. Assessing whether the Wharton’s jelly if the cord
c. Her actual genetic composition has a pH higher than 7.2
d. Her outward appearance b. Assessing whether the umbilical cord has two
arteries and one vein
24. Jean Suarez is a balanced translocation carrier c. Measuring the length of the cord to be certain
for Down syndrome. This term means that: that it is longer than three feet
a. All of her children will be born with some d. Determining that the color of the umbilical cord
aspects of Down syndrome is not green
b. All of her female and none of her male children
will have Down syndrome 29. Rizalyn asks you why her doctor is concerned
c. She has a greater than average chance a child about whether her fetus us producing surfactant
will have Down syndrome or not. Your best answer would be:
d. It is impossible for any of her children to be born a. Surfactant keeps lungs from collapsing on
with Down syndrome expiration, and thus aids newborn breathing
b. Surfactant is produced by the fetal liver, so its
25. Jean Suarez was told at a genetic counseling precursor reveals liver maturity
session tat she is a balanced translocation carrier c. Surfactant is the precursor to IgM antibody
for Down syndrome. What would be your best production, so it prevents infection
action regarding this information? d. Surfactant reveals mature kidney function, as it
a. Be certain all of her family understand what this is produced by kidney glomeruli
means
b. Discuss the cost of various abortion techniques 30. Rizalyn is scheduled to have an ultrasound
with Jean examination. What instruction would you give
c. Be sure Jean knows she should not have any her before her examination?
more children a. Void immediately before the procedure to
d. Ask Jean is she has any questions that you could reduce your bladder size
answer for her b. The intravenous fluid infused to dilate your
uterus does not hurt the fetus
26. Jean Suarez’s child is born with Down Syndrome. c. You will need to drink at least 3 glasses of fluid
What is a common physical feature of newborn before the procedure
with this disorder? d. You can have medicine for pain for any
a. Spastic and stiff muscles contractions caused by the test
b. Loose skin at back of neck
c. A white lock of forehead hair

43
43
31. Rizalyn is scheduled to have an amniocentesis to how decreased insulin effectiveness safeguards
test for fetal maturity. What instruction would the fetus?
you give her before this procedure? a. Decreased effectiveness prevents the fetus from
a. Void immediately before the procedure to being hypoglycemic
reduce your bladder size b. If insulin is ineffective it cannot cross the
b. The x-ray used to reveal your fetus’ position has placenta and harm the fetus
no long-term effects c. The lessened action prevents the fetus from
c. The intravenous fluid infused to dilate your gaining too much weight
uterus does not hurt the fetus d. The mother, not the fetus, is guarded by this
d. No more amniotic fluid forms afterward, which decreased insulin action
is why only a small amount is removed
37. Riza Cua feels well. She asks you why she needs
32. Bernadette sometimes feels ambivalent about to come for prenatal care The best reason for
being pregnant. What is the psychological task her to receive regular care is:
you’d like to see her complete during the first a. Discovering allergies can help eliminate early
trimester of pregnancy? birth
a. View morning sickness as tolerable b. It helps document how many pregnancies occur
b. Accept the fact that she’s pregnant each year
c. Accept the fact that a baby is growing inside her c. It provides time for education about pregnancy
d. Choose a name for the baby and birth
d. It determines whether pregnancies today are
33. Bernadette is aware that she’s been showing planned or not
some narcissism since becoming pregnant.
Which of her actions best describes narcissism? 38. Why is it important to ask Riza about past
a. Her skin feels “pulled thin” across her abdomen surgery on a pregnancy health history?
b. Her thoughts tend to be mainly about herself a. To test her recent and long-term memory
c. She feels a need to sleep a lot more than usual b. Adhesions from surgery could limit uterine
d. She often feels “numb” or as if she’s taken a growth
narcotic c. To assess she could be allergic to any medication
d. To determine if she has effective heath
34. Bernadette did a urine pregnancy test but was insurance
surprised to learn that a positive result is not a
sure sign if pregnancy. She asks you what would 39. Riza reports that the palms of her hands are
be a positive sign. You tell her would be if: always itchy. You notice scratches on them when
a. She is having consistent uterine growth you do a physical exam. What is the most likely
b. She can feel the fetus move inside her cause of this finding during pregnancy?
c. hCG can be found in her bloodstream a. She must have become allergic to dishwashing
d. The fetal heart can be seen on ultrasound soap
b. She has an allergy to her fetus and will probably
35. Bernadette’s doctor told her she had a positive abort
Chadwick’s sign. She asks you what this means, c. Her weight gain has stretched the skin over her
and you tell her that: hands
a. Her abdomen is soft and tender d. This is a common reaction to increasing estrogen
b. Her uterus has tipped forward levels.
c. Cervical mucus is clear and sticky
d. Her vagina has darkened in color 40. Riza has not had a pelvic exam since she was in
highschool. What advice would you give her to
36. Bernadette overheard her doctor say that insulin help her relax during her first prenatal pelvic
is not as effective during pregnancy as usual. exam?
That made her worry that she is developing a. Have her take a deep breath and hold it during
diabetes, like her aunt. How would you explain the exam

43
43
b. Tell her to bear down slightly as the speculum is d. Witch hazel pads feel cool against swollen
inserted hemorrhoids
c. Singing out loud helps, because this pushes
down the diaphragm 45. Vanna has ankle edema by the end of each day.
d. She should breathe slowly and evenly during the Which statement by her would reveal that she
exam understands what causes this?
a. “I know this is a beginning complication; I’ll call
41. Riza has pelvic measurements taken. What size my doctor tonight.”
should the ischial tuberosity diameter be to be b. “I understand this is from eating too much salt;
considered adequate? I’ll restrict that more.”
a. 6 cm c. “I’ll rest in a Sims’ position to take pressure off
b. Twice the width of the conjugate diameter lower extremity veins.”
c. 11 cm d. “I’ll walk for half an hour every day to relieve
d. Half the width of the symphysis pubis this; I’ll try walking more.”

Situation: One of the nursing roles in caring for the


pregnant family is promoting fetal and maternal health

42. Which statement by Vanna Delgado would alert


you that she needs more teaching about safe
practices during pregnancy?
a. “I take either a shower or tub bath, because I
know both are safe.”
b. “I wash my breasts with clear water, not with
soap daily.”
c. “I’m glad I don’t have to ask my boyfriend to use
condoms anymore.”
d. “I’m wearing low-heeled shoes to try and avoid
backache.”

43. Vanna describes her typical day to you. What


would alert you that she may need further
pregnancy advice?
a. “I jog rather than walk every time I can for
exercise.”
b. “I always go to sleep on my side, not on my
back.”
c. “I pack my lunch in the morning when I’m not so
tired.”
d. “I walk around my desk every hour to prevent
varicosities.”

44. Vanna tells you that she is developing painful


hemorrhoids. Advice you would give her would
be:
a. Take a tablespoon of mineral oil with each of
your meals
b. Omit fiber from your diet. This will prevent
constipation
c. Lie on your stomach daily to drain blood from
the rectal veins

43
43
Answer for maternity part 2

BCCBB
CBCBD
DABDC
BAADA
BADCD
BDBAC
ABBDD
ACBDD
CCADC

43
43
PEDIATRIC NURSING a. Increased food intake owing to age
b. Underdeveloped abdominal muscles
1. While performing physical assessment of a 12 c. Bowlegged posture
month-old, the nurse notes that the infant’s d. Linear growth curve
anterior fontanelle is still slightly open. Which of 7. If parents keep a toddler dependent in areas
the following is the nurse’s most appropriate where he is capable of using skills, the toddle
action? will develop a sense of which of the following?
a. Notify the physician immediately a. Mistrust
because there is a problem. b. Shame
b. Perform an intensive neurologic c. Guilt
examination. d. Inferiority
c. Perform an intensive developmental 8. Which of the following is an appropriate toy for
examination. an 18-month-old?
d. Do nothing because this is a normal a. Multiple-piece puzzle
finding for the age. b. Miniature cars
2. When teaching a mother about introducing solid c. Finger paints
foods to her child, which of the following d. Comic book
indicates the earliest age at which this should be 9. When teaching parents about the child’s
done? readiness for toilet training, which of the
a. 1 month following signs should the nurse instruct them to
b. 2 months watch for in the toddler?
c. 3 months a. Demonstrates dryness for 4 hours
d. 4 months b. Demonstrates ability to sit and walk
3. The infant of a substance-abusing mother is at c. Has a new sibling for stimulation
risk for developing a sense of which of the d. Verbalizes desire to go to the bathroom
following? 10. When teaching parents about typical toddler
a. Mistrust eating patterns, which of the following should be
b. Shame included?
c. Guilt a. Food “jags”
d. Inferiority b. Preference to eat alone
4. Which of the following toys should the nurse c. Consistent table manners
recommend for a 5-month-old? d. Increase in appetite
a. A big red balloon 11. Which of the following suggestions should the
b. A teddy bear with button eyes nurse offer the parents of a 4-year-old boy who
c. A push-pull wooden truck resists going to bed at night?
d. A colorful busy box a. “Allow him to fall asleep in your room,
5. The mother of a 2-month-old is concerned that then move him to his own bed.”
she may be spoiling her baby by picking her up b. “Tell him that you will lock him in his
when she cries. Which of the following would be room if he gets out of bed one more
the nurse’s best response? time.”
a. “ Let her cry for a while before picking c. “Encourage active play at bedtime to tire
her up, so you don’t spoil her” him out so he will fall asleep faster.”
b. “Babies need to be held and cuddled; d. “Read him a story and allow him to play
you won’t spoil her this way” quietly in his bed until he falls asleep.”
c. “Crying at this age means the baby is 12. When providing therapeutic play, which of the
hungry; give her a bottle” following toys would best promote imaginative
d. “If you leave her alone she will learn play in a 4-year-old?
how to cry herself to sleep” a. Large blocks
6. When assessing an 18-month-old, the nurse b. Dress-up clothes
notes a characteristic protruding abdomen. c. Wooden puzzle
Which of the following would explain the d. Big wheels
rationale for this finding?

43
43
13. Which of the following activities, when voiced by c. Inferiority
the parents following a teaching session about d. Role diffusion
the characteristics of school-age cognitive 19. Which of the following would be most
development would indicate the need for appropriate for a nurse to use when describing
additional teaching? menarche to a 13-year-old?
a. Collecting baseball cards and marbles a. A female’s first menstruation or
b. Ordering dolls according to size menstrual “periods”
c. Considering simple problem-solving b. The first year of menstruation or
options “period”
d. Developing plans for the future c. The entire menstrual cycle or from one
14. A hospitalized schoolager states: “I’m not afraid “period” to another
of this place, I’m not afraid of anything.” This d. The onset of uterine maturation or peak
statement is most likely an example of which of growth
the following? 20. A 14-year-old boy has acne and according to his
a. Regression parents, dominates the bathroom by using the
b. Repression mirror all the time. Which of the following
c. Reaction formation remarks by the nurse would be least helpful in
d. Rationalization talking to the boy and his parents?
15. After teaching a group of parents about accident a. “This is probably the only concern he has
prevention for schoolagers, which of the about his body. So don’t worry about it
following statements by the group would or the time he spends on it.”
indicate the need for more teaching? b. “Teenagers are anxious about how their
a. “Schoolagers are more active and peers perceive them. So they spend a lot
adventurous than are younger children.” of time grooming.”
b. “Schoolagers are more susceptible to c. “A teen may develop a poor self-image
home hazards than are younger when experiencing acne. Do you feel this
children.” way sometimes?”
c. “Schoolagers are unable to understand d. “You appear to be keeping your face
potential dangers around them.” well washed. Would you feel
d. “Schoolargers are less subject to comfortable discussing your cleansing
parental control than are younger method?”
children.” 21. Which of the following should the nurse suspect
16. Which of the following skills is the most when noting that a 3-year-old is engaging in
significant one learned during the schoolage explicit sexual behavior during doll play?
period? a. The child is exhibiting normal pre-school
a. Collecting curiosity
b. Ordering b. The child is acting out personal
c. Reading experiences
d. Sorting c. The child does not know how to play
17. A child age 7 was unable to receive the measles, with dolls
mumps, and rubella (MMR) vaccine at the d. The child is probably developmentally
recommended scheduled time. When would the delayed.
nurse expect to administer MMR vaccine? 22. Which of the following statements by the
a. In a month from now parents of a child with school phobia would
b. In a year from now indicate the need for further teaching?
c. At age 10 a. “We’ll keep him at home until phobia
d. At age 13 subsides.”
18. The adolescent’s inability to develop a sense of b. “We’ll work with his teachers and
who he is and what he can become results in a counselors at school.”
sense of which of the following? c. “We’ll try to encourage him to talk
a. Shame about his problem.”
b. Guilt

43
43
d. “We’ll discuss possible solutions with b. Encouraging adequate intake of iron-rich
him and his counselor.” foods
23. When developing a teaching plan for a group of c. Assisting with coping with chronic illness
high school students about teenage pregnancy, d. Administering medications via IM
the nurse would keep in mind which of the injections
following? 29. Which of the following information, when voiced
a. The incidence of teenage pregnancies is by the mother, would indicate to the nurse that
increasing. she understands home care instructions
b. Most teenage pregnancies are planned. following the administration of a diphtheria,
c. Denial of the pregnancy is common early tetanus, and pertussis injection?
on. a. Measures to reduce fever
d. The risk for complications during b. Need for dietary restrictions
pregnancy is rare. c. Reasons for subsequent rash
24. When assessing a child with a cleft palate, the d. Measures to control subsequent
nurse is aware that the child is at risk for more diarrhea
frequent episodes of otitis media due to which 30. Which of the following actions by a community
of the following? health nurse is most appropriate when noting
a. Lowered resistance from malnutrition multiple bruises and burns on the posterior
b. Ineffective functioning of the Eustachian trunk of an 18-month-old child during a home
tubes visit?
c. Plugging of the Eustachian tubes with a. Report the child’s condition to
food particles Protective Services immediately.
d. Associated congenital defects of the b. Schedule a follow-up visit to check for
middle ear. more bruises.
25. While performing a neurodevelopmental c. Notify the child’s physician immediately.
assessment on a 3-month-old infant, which of d. Don nothing because this is a normal
the following characteristics would be expected? finding in a toddler.
a. A strong Moro reflex 31. Which of the following is being used when the
b. A strong parachute reflex mother of a hospitalized child calls the student
c. Rolling from front to back nurse and states, “You idiot, you have no idea
d. Lifting of head and chest when prone how to care for my sick child”?
26. By the end of which of the following would the a. Displacement
nurse most commonly expect a child’s birth b. Projection
weight to triple? c. Repression
a. 4 months d. Psychosis
b. 7 months 32. Which of the following should the nurse expect
c. 9 months to note as a frequent complication for a child
d. 12 months with congenital heart disease?
27. Which of the following best describes parallel a. Susceptibility to respiratory infection
play between two toddlers? b. Bleeding tendencies
a. Sharing crayons to color separate c. Frequent vomiting and diarrhea
pictures d. Seizure disorder
b. Playing a board game with a nurse 33. Which of the following would the nurse do first
c. Sitting near each other while playing for a 3-year-old boy who arrives in the
with separate dolls emergency room with a temperature of 105
d. Sharing their dolls with two different degrees, inspiratory stridor, and restlessness,
nurses who is learning forward and drooling?
28. Which of the following would the nurse identify a. Auscultate his lungs and place him in a
as the initial priority for a child with acute mist tent.
lymphocytic leukemia? b. Have him lie down and rest after
a. Instituting infection control precautions encouraging fluids.

43
43
c. Examine his throat and perform a throat c. Marked motor delays
culture d. Gait disability
d. Notify the physician immediately and 40. Which of the following assessment findings
prepare for intubation. would lead the nurse to suspect Down syndrome
34. Which of the following would the nurse need to in an infant?
keep in mind as a predisposing factor when a. Small tongue
formulating a teaching plan for child with a b. Transverse palmar crease
urinary tract infection? c. Large nose
a. A shorter urethra in females d. Restricted joint movement
b. Frequent emptying of the bladder 41. While assessing a newborn with cleft lip, the
c. Increased fluid intake nurse would be alert that which of the following
d. Ingestion of acidic juices will most likely be compromised?
35. Which of the following should the nurse do first a. Sucking ability
for a 15-year-old boy with a full leg cast who is b. Respiratory status
screaming in unrelenting pain and exhibiting c. Locomotion
right foot pallor signifying compartment d. GI function
syndrome? 42. When providing postoperative care for the child
a. Medicate him with acetaminophen. with a cleft palate, the nurse should position the
b. Notify the physician immediately child in which of the following positions?
c. Release the traction a. Supine
d. Monitor him every 5 minutes b. Prone
36. At which of the following ages would the nurse c. In an infant seat
expect to administer the varicella zoster vaccine d. On the side
to child? 43. While assessing a child with pyloric stenosis, the
a. At birth nurse is likely to note which of the following?
b. 2 months a. Regurgitation
c. 6 months b. Steatorrhea
d. 12 months c. Projectile vomiting
37. When discussing normal infant growth and d. “Currant jelly” stools
development with parents, which of the 44. Which of the following nursing diagnoses would
following toys would the nurse suggest as most be inappropriate for the infant with
appropriate for an 8-month-old? gastroesophageal reflux (GER)?
a. Push-pull toys a. Fluid volume deficit
b. Rattle b. Risk for aspiration
c. Large blocks c. Altered nutrition: less than body
d. Mobile requirements
38. Which of the following aspects of psychosocial d. Altered oral mucous membranes
development is necessary for the nurse to keep 45. Which of the following parameters would the
in mind when providing care for the preschool nurse monitor to evaluate the effectiveness of
child? thickened feedings for an infant with
a. The child can use complex reasoning to gastroesophageal reflux (GER)?
think out situations. a. Vomiting
b. Fear of body mutilation is a common b. Stools
preschool fear c. Uterine
c. The child engages in competitive types d. Weight
of play 46. Discharge teaching for a child with celiac disease
d. Immediate gratification is necessary to would include instructions about avoiding which
develop initiative. of the following?
39. Which of the following is characteristic of a a. Rice
preschooler with mid mental retardation? b. Milk
a. Slow to feed self c. Wheat
b. Lack of speech d. Chicken

43
43
47. Which of the following would the nurse expect
to assess in a child with celiac disease having a
celiac crisis secondary to an upper respiratory
infection?
a. Respiratory distress
b. Lethargy
c. Watery diarrhea
d. Weight gain
48. Which of the following should the nurse do first
after noting that a child with Hirschsprung
disease has a fever and watery explosive
diarrhea?
a. Notify the physician immediately
b. Administer antidiarrheal medications
c. Monitor child ever 30 minutes
d. Nothing, this is characteristic of
Hirschsprung disease
49. A newborn’s failure to pass meconium within
the first 24 hours after birth may indicate which
of the following?
a. Hirschsprung disease
b. Celiac disease
c. Intussusception
d. Abdominal wall defect
50. When assessing a child for possible
intussusception, which of the following would be
least likely to provide valuable information?
a. Stool inspection
b. Pain pattern
c. Family history
d. Abdominal palpation

43
43
ANSWERS and RATIONALES for PEDIATRIC NURSING decreases, not increases. Toddlers are
characteristically bowlegged because the leg
1. D. The anterior fontanelle typically closes muscles must bear the weight of the relatively
anywhere between 12 to 18 months of age. large trunk. Toddler growth patterns occur in a
Thus, assessing the anterior fontanelle as still steplike, not linear pattern.
being slightly open is a normal finding requiring 7. B. According to Erikson, toddlers experience a
no further action. Because it is normal finding for sense of shame when they are not allowed to
this age, notifying he physician or performing develop appropriate independence and
additional examinations are inappropriate. autonomy. Infants develop mistrust when their
2. D. Solid foods are not recommended before age needs are not consistently gratified.
4 to 6 months because of the sucking reflex and Preschoolers develop guilt when their initiative
the immaturity of the gastrointestinal tract and needs are not met while schoolagers develop a
immune system. Therefore, the earliest age at sense of inferiority when their industry needs
which to introduce foods is 4 months. Any time are not met.
earlier would be inappropriate. 8. C. Young toddlers are still sensorimotor learners
3. A. According to Erikson, infants need to have and they enjoy the experience of feeling
their needs met consistently and effectively to different textures. Thus, finger paints would be
develop a sense of trust. An infant whose needs an appropriate toy choice. Multiple-piece toys,
are consistently unmet or who experiences such as puzzle, are too difficult to manipulate
significant delays in having them met, such as in and may be hazardous if the pieces are small
the case of the infant of a substance-abusing enough to be aspirated. Miniature cars also have
mother, will develop a sense of uncertainty, a high potential for aspiration. Comic books are
leading to mistrust of caregivers and the on too high a level for toddlers. Although they
environment. Toddlers develop a sense of may enjoy looking at some of the pictures,
shame when their autonomy needs are not met toddlers are more likely to rip a comic book
consistently. Preschoolers develop a sense of apart.
guilt when their sense of initiative is thwarted. 9. D. The child must be able to sate the need to go
Schoolagers develop a sense of inferiority when to the bathroom to initiate toilet training.
they do not develop a sense of industry. Usually, a child needs to be dry for only 2 hours,
4. D. A busy box facilitates the fine motor not 4 hours. The child also must be able to sit,
development that occurs between 4 and 6 walk, and squat. A new sibling would most likely
months. Balloons are contraindicated because hinder toilet training.
small children may aspirate balloons. Because 10. A. Toddlers become picky eaters, experiencing
the button eyes of a teddy bear may detach and food jags and eating large amounts one day and
be aspirated, this toy is unsafe for children very little the next. A toddler’s food gags express
younger than 3 years. A 5-month-old is too a preference for the ritualism of eating one type
young to use a push-pull toy. of food for several days at a time. Toddlers
5. B. Infants need to have their security needs met typically enjoy socialization and limiting others
by being held and cuddled. At 2 months of age, at meal time. Toddlers prefer to feed themselves
they are unable to make the connection and thus are too young to have table manners. A
between crying and attention. This association toddler’s appetite and need for calories, protein,
does not occur until late infancy or early and fluid decrease due to the dramatic slowing
toddlerhood. Letting the infant cry for a time of growth rate.
before picking up the infant or leaving the infant 11. D. Preschoolers commonly have fears of the
alone to cry herself to sleep interferes with dark, being left alone especially at bedtime, and
meeting the infant’s need for security at this ghosts, which may affect the child’s going to bed
very young age. Infants cry for many reasons. at night. Quiet play and time with parents is a
Assuming that the child s hungry may cause positive bedtime routine that provides security
overfeeding problems such as obesity. and also readies the child for sleep. The child
6. B. Underdeveloped abdominal musculature should sleep in his own bed. Telling the child
gives the toddler a characteristically protruding about locking him in his room will viewed by the
abdomen. During toddlerhood, food intake child as a threat. Additionally, a locked door is

43
44
frightening and potentially hazardous. Vigorous 16. C. The most significant skill learned during the
activity at bedtime stirs up the child and makes school-age period is reading. During this time
more difficult to fall asleep. the child develops formal adult articulation
12. B. Dress-up clothes enhance imaginative play patterns and learns that words can be arranged
and imagination, allowing preschoolers to in structure. Collective, ordering, and sorting,
engage in rich fantasy play. Building blocks and although important, are not most significant
wooden puzzles are appropriate for encouraging skills learned.
fine motor development. Big wheels and 17. C. Based on the recommendations of the
tricycles encourage gross motor development. American Academy of Family Physicians and the
13. D. The school-aged child is in the stage of American Academy of Pediatrics, the MMR
concrete operations, marked by inductive vaccine should be given at the age of 10 if the
reasoning, logical operations, and reversible child did not receive it between the ages of 4 to
concrete thought. The ability to consider the 6 years as recommended. Immunization for
future requires formal thought operations, diphtheria and tetanus is required at age 13.
which are not developed until adolescence. 18. D. According to Erikson, role diffusion develops
Collecting baseball cards and marbles, ordering when the adolescent does not develop a sense
dolls by size, and simple problem-solving options of identity and a sense or where he fits in.
are examples of the concrete operational Toddlers develop a sense of shame when they
thinking of the schoolager. do not achieve autonomy. Preschoolers develop
14. C. Reaction formation is the schoolager’s typical a sense of guilt when they do not develop a
defensive response when hospitalized. In sense of initiative. School-age children develop a
reaction formation, expression of unacceptable sense of inferiority when they do not develop a
thoughts or behaviors is prevented (or sense of industry.
overridden) by the exaggerated expression of 19. A. Menarche refers to the onset of the first
opposite thoughts or types of behaviors. menstruation or menstrual period and refers
Regression is seen in toddlers and preshcoolers only to the first cycle. Uterine growth and
when they retreat or return to an earlier level of broadening of the pelvic girdle occurs before
development. Repression refers to the menarche.
involuntary blocking of unpleasant feelings and 20. A. Stating that this is probably the only concern
experiences from one’s awareness. the adolescent has and telling the parents not to
Rationalization is the attempt to make excuses worry about it or the time her spends on it shuts
to justify unacceptable feelings or behaviors. off further investigation and is likely to make the
15. C. The schoolager’s cognitive level is sufficiently adolescent and his parents feel defensive. The
developed to enable good understanding of and statement about peer acceptance and time
adherence to rules. Thus, schoolagers should be spent in front of the mirror for the development
able to understand the potential dangers around of self image provides information about the
them. With growth comes greater freedom and adolescent’s needs to the parents and may help
children become more adventurous and daring. to gain trust with the adolescent. Asking the
The school-aged child is also still prone to adolescent how he feels about the acne will
accidents and home hazards, especially because encourage the adolescent to share his feelings.
of increased motor abilities and independence. Discussing the cleansing method shows interest
Plus the home hazards differ from other age and concern for the adolescent and also can help
groups. These hazards, which are potentially to identify any patient-teaching needs for the
lethal but tempting, may include firearms, adolescent regarding cleansing.
alcohol, and medications. School-age children 21. B. Preschoolers should be developmentally
begin to internalize their own controls and need incapable of demonstrating explicit sexual
less outside direction. Plus the child is away from behavior. If a child does so, the child has been
home more often. Some parental or caregiver exposed to such behavior, and sexual abuse
assistance is still needed to answer questions should be suspected. Explicit sexual behavior
and provide guidance for decisions and during doll play is not a characteristic of
responsibilities. preschool development nor symptomatic of

44
44
developmental delay. Whether or nor the child play. Sharing crayons, playing a board game with
knows how to play with dolls is irrelevant. a nurse, or sharing dolls with two different
22. A. The parents need more teaching if they state nurses are all examples of cooperative play.
that they will keep the child home until the 28. A. Acute lymphocytic leukemia (ALL) causes
phobia subsides. Doing so reinforces the child’s leukopenia, resulting in immunosuppression and
feelings of worthlessness and dependency. The increasing the risk of infection, a leading cause
child should attend school even during of death in children with ALL. Therefore, the
resolution of the problem. Allowing the child to initial priority nursing intervention would be to
verbalize helps the child to ventilate feelings and institute infection control precautions to
may help to uncover causes and solutions. decrease the risk of infection. Iron-rich foods
Collaboration with the teachers and counselors help with anemia, but dietary iron is not an
at school may lead to uncovering the cause of initial intervention. The prognosis of ALL usually
the phobia and to the development of solutions. is good. However, later on, the nurse may need
The child should participate and play an active to assist the child and family with coping since
role in developing possible solutions. death and dying may still be an issue in need of
23. C. The adolescent who becomes pregnant discussion. Injections should be discouraged,
typically denies the pregnancy early on. Early owing to increased risk from bleeding due to
recognition by a parent or health care provider thrombocytopenia.
may be crucial to timely initiation of prenatal 29. A. The pertusis component may result in fever
care. The incidence of adolescent pregnancy has and the tetanus component may result in
declined since 1991, yet morbidity remains high. injection soreness. Therefore, the mother’s
Most teenage pregnancies are unplanned and verbalization of information about measures to
occur out of wedlock. The pregnant adolescent is reduce fever indicates understanding. No dietary
at high risk for physical complications including restrictions are necessary after this injection is
premature labor and low-birth-weight infants, given. A subsequent rash is more likely to be
high neonatal mortality, iron deficiency anemia, seen 5 to 10 days after receiving the MMR
prolonged labor, and fetopelvic disproportion as vaccine, not the diphtheria, pertussis, and
well as numerous psychological crises. tetanus vaccine. Diarrhea is not associated with
24. B. Because of the structural defect, children with this vaccine.
cleft palate may have ineffective functioning of 30. A. Multiple bruises and burns on a toddler are
their Eustachian tubes creating frequent bouts signs child abuse. Therefore, the nurse is
of otitis media. Most children with cleft palate responsible for reporting the case to Protective
remain well-nourished and maintain adequate Services immediately to protect the child from
nutrition through the use of proper feeding further harm. Scheduling a follow-up visit is
techniques. Food particles do not pass through inappropriate because additional harm may
the cleft and into the Eustachian tubes. There is come to the child if the nurse waits for further
no association between cleft palate and assessment data. Although the nurse should
congenial ear deformities. notify the physician, the goal is to initiate
25. D. A 3-month-old infant should be able to lift the measures to protect the child’s safety. Notifying
head and chest when prone. The Moro reflex the physician immediately does not initiate the
typically diminishes or subsides by 3 months. removal of the child from harm nor does it
The parachute reflex appears at 9 months. absolve the nurse from responsibility. Multiple
Rolling from front to back usually is bruises and burns are not normal toddler
accomplished at about 5 months. injuries.
26. D. A child’s birth weight usually triples by 12 31. B. The mother is using projection, the defense
months and doubles by 4 months. No specific mechanism used when a person attributes his or
birth weight parameters are established for 7 or her own undesirable traits to another.
9 months. Displacement is the transfer of emotion onto an
27. C. Toddlers engaging in parallel play will play unrelated object, such as when the mother
near each other, but not with each other. Thus, would kick a chair or bang the door shut.
when two toddlers sit near each other but play Repression is the submerging of painful ideas
with separate dolls, they are exhibiting parallel

44
44
into the unconscious. Psychosis is a state of capable of making hand-to-hand transfers, large
being out of touch with reality. blocks would be the most appropriate toy
32. A. Children with congenital heart disease are selection. Push-pull toys would be more
more prone to respiratory infections. Bleeding appropriate for the 10 to 12-month-old as he or
tendencies, frequent vomiting, and diarrhea and she begins to cruise the environment. Rattles
seizure disorders are not associated with and mobiles are more appropriate for infants in
congenital heart disease. the 1 to 3 month age range. Mobiles pose a
33. D. The child is exhibiting classic signs of danger to older infants because of possible
epiglottitis, always a pediatric emergency. The strangulation.
physician must be notified immediately and the 38. B. During the preschool period, the child has
nurse must be prepared for an emergency mastered a sense of autonomy and goes on to
intubation or tracheostomy. Further assessment master a sense of initiative. During this period,
with auscultating lungs and placing the child in a the child commonly experiences more fears than
mist tent wastes valuable time. The situation is a at any other time. One common fear is fear of
possible life-threatening emergency. Having the the body mutilation, especially associated with
child lie down would cause additional distress painful experiences. The preschool child uses
and may result in respiratory arrest. Throat simple, not complex, reasoning, engages in
examination may result in laryngospasm that associative, not competitive, play (interactive
could be fatal. and cooperative play with sharing), and is able to
34. A. In females, the urethra is shorter than in tolerate longer periods of delayed gratification.
males. This decreases the distance for organisms 39. A. Mild mental retardation refers to
to travel, thereby increasing the chance of the development disability involving an IQ 50 to 70.
child developing a urinary tract infection. Typically, the child is not noted as being
Frequent emptying of the bladder would help to retarded, but exhibits slowness in performing
decrease urinary tract infections by avoiding tasks, such as self-feeding, walking, and taking.
sphincter stress. Increased fluid intake enables Little or no speech, marked motor delays, and
the bladder to be cleared more frequently, thus gait disabilities would be seen in more severe
helping to prevent urinary tract infections. The forms mental retardation.
intake of acidic juices helps to keep the urine pH 40. B. Down syndrome is characterized by the
acidic and thus decrease the chance of flora following a transverse palmar crease (simian
development. crease), separated sagittal suture, oblique
35. B. Compartment syndrome is an emergent palpebral fissures, small nose, depressed nasal
situation and the physician needs to be notified bridge, high-arched palate, excess and lax skin,
immediately so that interventions can be wide spacing and plantar crease between the
initiated to relieve the increasing pressure and second and big toes, hyperextensible and lax
restore circulation. Acetaminophen (Tylenol) will joints, large protruding tongue, and muscle
be ineffective since the pain is related to the weakness.
increasing pressure and tissue ischemia. The 41. A. Because of the defect, the child will be unable
cast, not traction, is being used in this situation to from the mouth adequately around nipple,
for immobilization, so releasing the traction thereby requiring special devices to allow for
would be inappropriate. In this situation, specific feeding and sucking gratification. Respiratory
action not continued monitoring is indicated. status may be compromised if the child is fed
36. D. The varicella zoster vaccine (VZV) is a live improperly or during postoperative period,
vaccine given after age 12 months. The first dose Locomotion would be a problem for the older
of hepatitis B vaccine is given at birth to 2 infant because of the use of restraints. GI
months, then at 1 to 4 months, and then again at functioning is not compromised in the child with
6 to 18 months. DtaP is routinely given at 2, 4, 6, a cleft lip.
and 15 to 18 months and a booster at 4 to 6 42. B. Postoperatively children with cleft palate
years. should be placed on their abdomens to facilitate
37. C. Because the 8-month-old is refining his gross drainage. If the child is placed in the supine
motor skills, being able to sit unsupported and position, he or she may aspirate. Using an infant
also improving his fine motor skills, probably seat does not facilitate drainage. Side-lying does

44
44
not facilitate drainage as well as the prone inadequate motility in an intestinal segment.
position. Failure to pass meconium is not associated with
43. C. Projectile vomiting is a key symptom of pyloric celiac disease, intussusception, or abdominal
stenosis. Regurgitation is seen more commonly wall defect.
with GER. Steatorrhea occurs in malabsorption 50. C. Because intussusception is not believed to
disorders such as celiac disease. “Currant jelly” have a familial tendency, obtaining a family
stools are characteristic of intussusception. history would provide the least amount of
44. D. GER is the backflow of gastric contents into information. Stool inspection, pain pattern, and
the esophagus resulting from relaxation or abdominal palpation would reveal possible
incompetence of the lower esophageal (cardiac) indicators of intussusception. Current, jelly-like
sphincter. No alteration in the oral mucous stools containing blood and mucus are an
membranes occurs with this disorder. Fluid indication of intussusception. Acute, episodic
volume deficit, risk for aspiration, and altered abdominal pain is characteristics of
nutrition are appropriate nursing diagnoses. intussusception. A sausage-shaped mass may be
45. A. Thickened feedings are used with GER to stop palpated in the right upper quadrant.
the vomiting. Therefore, the nurse would
monitor the child’s vomiting to evaluate the
effectiveness of using the thickened feedings. No
relationship exists between feedings and
characteristics of stools and uterine. If feedings
are ineffective, this should be noted before
there is any change in the child’s weight.
46. C. Children with celiac disease cannot tolerate or
digest gluten. Therefore, because of its gluten
content, wheat and wheat-containing products
must be avoided. Rice, milk, and chicken do not
contain gluten and need not be avoided.
47. C. Episodes of celiac crises are precipitated by
infections, ingestion of gluten, prolonged fasting,
or exposure to anticholinergic drugs. Celiac crisis
is typically characterized by severe watery
diarrhea. Respiratory distress is unlikely in a
routine upper respiratory infection. Irritability,
rather than lethargy, is more likely. Because of
the fluid loss associated with the severe watery
diarrhea, the child’s weight is more likely to be
decreased.
48. A. For the child with Hirschsprung disease, fever
and explosive diarrhea indicate enterocolitis, a
life-threatening situation. Therefore, the
physician should be notified immediately.
Generally, because of the intestinal obstruction
and inadequate propulsive intestinal movement,
antidiarrheals are not used to treat Hirschsprung
disease. The child is acutely ill and requires
intervention, with monitoring more frequently
than every 30 minutes. Hirschsprung disease
typically presents with chronic constipation.
49. A. Failure to pass meconium within the first 24
hours after birth may be an indication of
Hirschsprung disease, a congenital anomaly
resulting in mechanical obstruction due to

44
44
COMMUNITY HEALTH NURSING Part 1 disease causation
C. Immunize nearby communities with Measles
SITUATION : Epidemiology and Vital statistics is a very D. Educate the community in future prevention of
important tool that a nurse could use in controlling the similar outbreaks
spread of disease in the community and at the same
time, surveying the impact of the disease on the 6. The main concern of a public health nurse is the
population and prevent it’s future occurrence. prevention of disease, prolonging of life and promoting
physical health and efficiency through which of the
1. It is concerned with the study of factors that influence following?
the occurrence and distribution of diseases, defects,
disability or death which occurs in groups or aggregation A. Use of epidemiological tools and vital health statistics
of individuals. B. Determine the spread and occurrence of the disease
C. Political empowerment and Socio Economic
A. Epidemiology Assistance
B. Demographics D. Organized Community Efforts
C. Vital Statistics
D. Health Statistics 7. In order to control a disease effectively, which of the
following must first be known?
2. Which of the following is the backbone in disease
prevention? 1. The conditions surrounding its occurrence
2. Factors that do not favor its development
A. Epidemiology 3. The condition that do not surround its occurrence
B. Demographics 4. Factors that favors its development
C. Vital Statistics
D. Health Statistics A. 1 and 3
B. 1 and 4
3. Which of the following type of research could show C. 2 and 3
how community expectations can result in the actual D. 2 and 4
provision of services?
8. All of the following are uses of epidemiology except:
A. Basic Research
B. Operational Research A. To study the history of health population and the rise
C. Action Research and fall of disease
D. Applied Research B. To diagnose the health of the community and the
condition of the people
4. An outbreak of measles has been reported in C. To provide summary data on health service delivery
Community A. As a nurse, which of the following is your D. To identify groups needing special attention
first action for an Epidemiological investigation?
9. Before reporting the fact of presence of an epidemic,
A. Classify if the outbreak of measles is epidemic or which of the following is of most importance to
just sporadic determine?
B. Report the incidence into the RHU
C. Determine the first day when the outbreak occurred A. Are the facts complete?
D. Identify if it is the disease which it is reported to be B. Is the disease real?
C. Is the disease tangible?
5. After the epidemiological investigation produced final D. Is it epidemic or endemic?
conclusions, which of the following is your initial step in
your operational procedure during disease outbreak? 10. An unknown epidemic has just been reported in
Barangay Dekbudekbu. People said that affected person
A. Coordinate personnel from Municipal to the demonstrates hemorrhagic type of fever. You are
National level designated now to plan for epidemiological
B. Collect pertinent laboratory specimen to confirm investigation. Arrange the sequence of events in

44
44
accordance with the correct outline plan for
epidemiological investigation. 13. All of the following are function of Nurse Budek in
epidemiology except
1. Report the presence of dengue
2. Summarize data and conclude the final picture of A. Laboratory Diagnosis
epidemic B. Surveillance of disease occurrence
3. Relate the occurrence to the population group, C. Follow up cases and contacts
facilities, food supply and carriers D. Refer cases to hospitals if necessary
4. Determine if the disease is factual or real E. Isolate cases of communicable disease
5. Determine any unusual prevalence of the disease
and its nature; is it epidemic, sporadic, endemic or 14. All of the following are performed in
pandemic? team organization except
6. Determine onset and the geographical limitation of
the disease. A. Orientation and demonstration of methodology to be
employed
A. 4,1,3,5,2,6 B. Area assignments of team members
B. 4,1,5,6,3,2 C. Check team’s equipments and paraphernalia
C. 5,4,6,2,1,3 D. Active case finding and Surveillance
D. 5,4,6,1,2,3
E. 1,2,3,4,5,6 15. Which of the following is the final output of data
reporting in epidemiological operational procedure?
11. In the occurrence of SARS and other pandemics,
which of the following is the most vital role of a nurse in A. Recommendation
epidemiology? B. Evaluation
C. Final Report
A. Health promotion D. Preliminary report
B. Disease prevention
C. Surveillance 16. The office in charge with registering vital facts in the
D. Casefinding Philippines is none other than the

12. Measles outbreak has been reported in Barangay A. PCSO


Bahay Toro, After conducting an epidemiological B PAGCOR
investigation you have confirmed that the outbreak is C. DOH
factual. You are tasked to lead a team of medical D. NSO
workers for operational procedure in disease outbreak.
Arrange the correct sequence of events that you must 17. The following are possible sources of Data except:
do to effectively contain the disease
A. Experience
1. Create a final report and recommendation B. Census
2. Perform nasopharyngeal swabbing to infected C. Surveys
individuals D. Research
3. Perform mass measles immunization to vulnerable
groups 18. This refers to systematic study of vital events such as
4. Perform an environmental sanitation survey on the births, illnesses, marriages, divorces and deaths
immediate environment
5. Organize your team and Coordinate the personnels A. Epidemiology
6. Educate the community on disease transmission B. Demographics
C. Vital Statistics
A. 1,2,3,4,5,6 D. Health Statistics
B. 6,5,4,3,2,1
C. 5,6,4,2,3,1 19. In case of clerical errors in your birth certificate,
D. 5,2,3,4,6,1 Where should you go to have it corrected?

44
44

A. 651
A. NSO B. 541
B. Court of Appeals C. 996
C. Municipal Trial Court D. 825
D. Local Civil Registrar
25. These rates are referred to the total living
20. Acasia just gave birth to Lestat, A healthy baby boy. population, It must be presumed that the total
Who are going to report the birth of Baby Lestat? population was exposed to the risk of occurrence of the
event.
A. Nurse
B. Midwife A. Rate
C. OB Gyne B. Ratio
D. Birth Attendant C. Crude/General Rates
D. Specific Rate
21. In reporting the birth of Baby Lestat, where will he
be registered? 26. These are used to describe the relationship between
two numerical quantities or measures of events without
A. At the Local Civil Registrar taking particular considerations to the time or place.
B. In the National Statistics Office
C. In the City Health Department A. Rate
D. In the Field Health Services and Information B. Ratios
System Main Office C. Crude/General Rate
D. Specific Rate
22. Deejay, The birth attendant noticed that Lestat has
low set of ears, Micrognathia, Microcephaly and a typical 27. This is the most sensitive index in determining the
cat like cry. What should Deejay do? general health condition of a community since it reflects
the changes in the environment and medical conditions
A. Bring Lestat immediately to the nearest hospital of a community
B. Ask his assistant to call the nearby pediatrician
C. Bring Lestat to the nearest pediatric clinic A. Crude death rate
D. Call a Taxi and together with Acasia, Bring Lestat B. Infant mortality rate
to the nearest hospital C. Maternal mortality rate
D. Fetal death rate
23. Deejay would suspect which disorder?
28. According to the WHO, which of the following is
A. Trisomy 21 the most frequent cause of death in children underfive
B. Turners Syndrome worldwide in the 2003 WHO Survey?
C. Cri Du Chat
D. Klinefelters Syndrome A. Neonatal
B. Pneumonia
24. Deejay could expect which of the following C. Diarrhea
congenital anomaly that would accompany this D. HIV/AIDS
disorder?
29. In the Philippines, what is the most common cause of
A. AVSD death of infants according to the latest survey?
B. PDA
C. TOF A. Pneumonia
D. TOGV B. Diarrhea
C. Other perinatal condition
26. Which presidential decree orders reporting of births D. Respiratory condition of fetus and newborn
within 30 days after its occurrence?
30. The major cause of mortality from 1999 up to 2002

44
44
in the Philippines are 4. Death from CVD : 3,029
5. Deaths under 1 year of age : 23
A. Diseases of the heart 6. Fetal deaths : 8
B. Diseases of the vascular system 7. Deaths under 28 days : 8
C. Pneumonias 8. Death due to rabies : 45
D. Tuberculosis 9. Registered cases of rabies : 45
10. People with pneumonia : 79
31. Alicia, a 9 year old child asked you “ What is the 11. People exposed with pneumonia : 2,593
common cause of death in my age group here in the 12. Total number of deaths from all causes : 10,998
Philippines? “ The nurse is correct if he will answer
The following questions refer to these data
A. Pneumonia is the top leading cause of death in
children age 5 to 9 35. What is the crude birth rate of Barangay PinoyBSN?
B. Malignant neoplasm if common in your age group
C. Probability wise, You might die due to accidents A. 90/100,000
D. Diseases of the respiratory system is the most B. 9/100
common cause of death in children C. 90/1000
D. 9/1000
32. In children 1 to 4 years old, which is the most
common cause of death? 36. What is the cause specific death rate from
cardiovascular diseases?
A. Diarrhea
B. Accidents A. 27/100
C. Pneumonia B. 1191/100,000
D. Diseases of the heart C. 27/100,000
D. 1.1/1000
33. Working in the community as a PHN for almost 10
years, Aida knew the fluctuation in vital statistics. She 37. What is the Maternal Mortality rate of this
knew that the most common cause of morbidity among barangay?
the Filipinos is
A. 6.55/1000
A. Diseases of the heart B. 5.89/1000
B. Diarrhea C. 1.36/1000
C. Pneumonia D. 3.67/1000
D. Vascular system diseases
38. What is the fetal death rate?
34. Nurse Aida also knew that most maternal deaths
are caused by A. 3.49/1000
B. 10.04/1000
A. Hemorrhage C. 3.14/1000
B. Other Complications related to pregnancy D. 3.14/100,000
occurring in the course of labor, delivery and
puerperium 39. What is the attack rate of pneumonia?
C. Hypertension complicating pregnancy, childbirth and
puerperium A. 3.04/1000
D. Abortion B. 7.18/1000
C. 32.82/100
SITUATION : Barangay PinoyBSN has the following data D. 3.04/100
in year 2006
40. Determine the Case fatality ratio of rabies in this
1. July 1 population : 254,316 Barangay
2. Livebirths : 2,289
3. Deaths from maternal cause : 15
44
44
A. 1/100
B. 100% 47. Which of the following is a POINT SOURCE epidemic?
C. 1%
D. 100/1000 A. Dengue H.F
B. Malaria
41. The following are all functions of the nurse in vital C. Contaminated Water Source
statistics, which of the following is not? D. Tuberculosis

A. Consolidate Data 48. All but one is a characteristic of a point source


B. Collects Data epidemic, which one is not?
C. Analyze Data
D. Tabulate Data A. The spread of the disease is caused by a common
vehicle
42. The following are Notifiable diseases that needs B. The disease is usually caused by contaminated food
to have a tally sheet in data reporting, Which one is C. There is a gradual increase of cases
not? D. Epidemic is usually sudden

A. Hypertension 49. The only Microorganism monitored in cases of


B. Bronchiolitis contaminated water is
C. Chemical Poisoning
D. Accidents A. Vibrio Cholera
B. Escherichia Coli
43. Which of the following requires reporting within 24 C. Entamoeba Histolytica
hours? D. Coliform Test

A. Neonatal tetanus 50. Dengue increase in number during June, July and
B. Measles August. This pattern is called
C. Hypertension
D. Tetanus A. Epidemic
B. Endemic
44. Which Act declared that all communicable disease C. Cyclical
be reported to the nearest health station? D. Secular

A. 1082 SITUATION : Field health services and information


B. 1891 system provides summary data on health service
C. 3573 delivery and selected program from the barangay level
D. 6675 up to the national level. As a nurse, you should know the
process on how these information became processed
45. In the RHU Team, Which professional is directly and consolidated.
responsible in caring a sick person who is homebound?
51. All of the following are objectives of FHSIS Except
A. Midwife
B. Nurse A. To complete the clinical picture of chronic disease
C. BHW and describe their natural history
D. Physician B. To provide standardized, facility level data base which
can be accessed for more in depth studies
46. During epidemics, which of the following C. To minimize recording and reporting burden allowing
epidemiological function will you have to perform first? more time for patient care and promotive activities
D. To ensure that data reported are useful and accurate
A. Teaching the community on disease prevention and are disseminated in a timely and easy to use fashion
B. Assessment on suspected cases
C. Monitor the condition of people affected 52. What is the fundamental block or foundation of the
D. Determining the source and nature of the epidemic

44
44
field health service information system? A. Leprosy cases
B. TB cases
A. Family treatment record C. Prenatal care
B. Target Client list D. Diarrhea cases
C. Reporting forms
D. Output record 59. This is the only mechanism through which data are
routinely transmitted from once facility to another
53. What is the primary advantage of having a target
client list? A. Family treatment record
B. Target Client list
A. Nurses need not to go back to FTR to monitor C. Reporting forms
treatment and services to beneficiaries thus saving D. Output record
time and effort
B. Help monitor service rendered to clients in general 60. FHSIS/Q-3 Or the report for environmental health
C. Facilitate monitoring and supervision of services activities is prepared how frequently?
D. Facilitates easier reporting
A. Daily
54. Which of the following is used to monitor particular B. Weekly
groups that are qualified as eligible to a certain C. Quarterly
program of the DOH? D. Yearly

A. Family treatment record 61. Nurse Budek is preparing the reporting form for
B. Target Client list weekly notifiable diseases. He knew that he will code the
C. Reporting forms report form as
D. Output record
A. FHSIS/E-1
55. In using the tally sheet, what is the recommended B. FHSIS/E-2
frequency in tallying activities and services? C. FHSIS/E-3
D. FHSIS/M-1
A. Daily
B. Weekly 62. In preparing the maternal death report, which of
C. Monthly the following correctly codes this occurrence?
D. Quarterly
A. FHSIS/E-1
56. When is the counting of the tally sheet done? B. FHSIS/E-2
C. FHSIS/E-3
A. At the end of the day D. FHSIS/M-1
B. At the end of the week
C. At the end of the month 63. Where should Nurse Budek bring the reporting forms
D. At the end of the year if he is in the BHU Facility?

57. Target client list will be transmitted to the next A. Rural health office
facility in the form of B. FHSIS Main office
C. Provincial health office
A. Family treatment record D. Regional health office
B. Target Client list
C. Reporting forms 64. After bringing the reporting forms in the right facility
D. Output record for processing, Nurse Budek knew that the output
reports are solely produced by what office?
58. All but one of the following are eligible target
client list A. Rural health office
B. FHSIS Main office

44
45
C. Provincial health office
D. Regional health office 70. Data submitted to the PHO is processed using what
type of technology?
65. Mang Raul entered the health center complaining of
fatigue and frequent syncope. You assessed Mang Raul A. Internet
and found out that he is severely malnourished and B. Microcomputer
anemic. What record should you get first to document C. Supercomputer
these findings? D. Server Interlink Connections

A. Family treatment record SITUATION : Community organizing is a process by which


B. Target Client list people, health services and agencies of the community
C. Reporting forms are brought together to act and solve their own
D. Output record problems.

66. The information about Mang Raul’s address, full 71. Mang Ambo approaches you for counseling. You are
name, age, symptoms and diagnosis is recorded in an effective counselor if you

A. Family treatment record A. Give good advice to Mang Ambo


B. Target Client list B. Identify Mang Ambo’s problems
C. Reporting forms C. Convince Mang Ambo to follow your advice
D. Output record D. Help Mang Ambo identify his problems

67. Another entry is to be made for Mang Raul because 72. As a newly appointed PHN instructed to organize
he is in the target client’s list, In what TCL should Mang Barangay Baritan, Which of the following is your initial
Raul’s entry be documented? step in organizing the community for initial action?

A. TCL Eligible Population A. Study the Barangay Health statistics and records
B. TCL Family Planning B. Make a courtesy call to the Barangay Captain
C. TCL Nutrition C. Meet with the Barangay Captain to make plans
D. TCL Pre Natal D. Make a courtesy call to the Municipal Mayor

68. The nurse uses the FHSIS Record system incorrectly 73. Preparatory phase is the first phase in organizing the
when she found out that community. Which of the following is the initial step in
the preparatory phase?
A. She go to the individual or FTR for entry confirmation
in the Tally/Report Summary A. Area selection
B. She refer to other sources for completing B. Community profiling
monthly and quarterly reports C. Entry in the community
C. She records diarrhea in the Tally sheet/Report form D. Integration with the people
with a code FHSIS/M-1
D. She records a Child who have frequent diarrhea in TCL 74. the most important factor in determining the
: Under Five proper area for community organizing is that this area
should
69. The BHS Is the lowest level of reporting unit in FHSIS.
A BHS can be considered a reporting unit if all of the A. Be already adopted by another organization
following are met except B. Be able to finance the projects
C. Have problems and needs assistance
A. It renders service to 3 barangays D. Have people with expertise to be developed as
B. There is a midwife the regularly renders service to the leaders
area
C. The BHS Have no mother BHS 75. Which of the following dwelling place should the
D. It should be a satellite BHS Nurse choose when integrating with the people?

45
45
A. A simple house in the border of Barangay Baritan and Nurse must first
San Pablo
B. A simple house with fencing and gate located in the A. Make a lesson plan
center of Barangay Baritan B. Set learning goals and objective
C. A modest dwelling place where people will not C. Assess their learning needs
hesitate to enter D. Review materials needed for training
D. A modest dwelling place where people will
not hesitate to enter located in the center of the 81. Nurse Budek wrote a letter to PCSO asking them for
community assistance in their feeding programs for the community’s
nutrition and health projects. PCSO then approved the
76. In choosing a leader in the community during the request and gave Budek 50,000 Pesos and a truckload of
Organizational phase, Which among these people will rice, fruits and vegetables. Which phase of COPAR did
you choose? Budek utilized?

A. Miguel Zobel, 50 years old, Rich and Famous A. Preparatory


B. Rustom, 27 years old, Actor B. Organizational
C. Mang Ambo, 70, Willing to work for the desired C. Education and Training
change D. Intersectoral Collaboration
D. Ricky, 30 years old, Influential and Willing to E. Phase out
work for the desired change
82. Ideally, How many years should the Nurse stay in the
77. Which type of leadership style should the leaders of community before he can phase out and be assured of a
the community practice? Self Reliant community?

A. Autocratic A. 5 years
B. Democratic B. 10 years
C. Laissez Faire C. 1 year
D. Consultative D. 6 months

78. Setting up Committee on Education and Training is in 83. Major discussion in community organization are
what phase of COPAR? made by

A. Preparatory A. The nurse


B. Organizational B. The leaders of each committee
C. Education and Training C. The entire group
D. Intersectoral Collaboration D. Collaborating Agencies
E. Phase out
84. The nurse should know that Organizational plan best
79. Community diagnosis is done to come up with a succeeds when
profile of local health situation that will serve as basis of
health programs and services. This is done in what phase 1. People sees its values
of COPAR? 2. People think its antagonistic professionally
3. It is incompatible with their personal beliefs
A. Preparatory 4. It is compatible with their personal beliefs
B. Organizational
C. Education and Training A. 1 and 3
D. Intersectoral Collaboration B. 2 and 4
E. Phase out C. 1 and 2
D. 1 and 4
80. The people named the community health workers
based on the collective decision in accordance with the 85. Nurse Budek made a proposal that people should
set criteria. Before they can be trained by the Nurse, The turn their backyard into small farming lots to plant

45
45
vegetables and fruits. He specified that the objective is community health problems and needs.
to save money in buying vegetables and fruits that tend
to have a fluctuating and cyclical price. Which step in A. Residing in the area of assignment
Community organizing process did he utilized? B. Listing down the name of person to contact for
courtesy call
A. Fact finding C. Gathering initial information about the community
B. Determination of needs D. Preparing Agenda for the first meeting
C. Program formation
D. Education and Interpretation SITUATION : Health education is the process whereby
knowledge, attitude and practice of people are changed
86. One of the critical steps in COPAR is becoming one to improve individual, family and community health.
with the people and understanding their culture and
lifestyle. Which critical step in COPAR will the Nurse try 91. Which of the following is the correct sequence in
to immerse himself in the community? health education?

A. Integration 1. Information
B. Social Mobilization 2. Communication
C. Ground Work 3. Education
D. Mobilization
A. 1,2,3
87. The Actual exercise of people power occurs B. 3,2,1
during when? C. 1,3,2
D. 3,1,2
A. Integration
B. Social Mobilization 92. The health status of the people is greatly affected
C. Ground Work and determined by which of the following?
D. Mobilization
A. Behavioral factors
88. Which steps in COPAR trains indigenous and informal B. Socioeconomic factors
leaders? C. Political factors
D. Psychological factors
A. Ground Work
B. Mobilization 93. Nurse Budek is conducting a health teaching to
C. Core Group formation Agnesia, 50 year old breast cancer survivor needing
D. Integration rehabilitative measures. He knows that health education
is effective when
89. As a PHN, One of your role is to organize the
community. Nurse Budek knows that the purposes of A. Agnesia recites the procedure and instructions
community organizing are perfectly
B. Agnesia’s behavior and outlook in life was
1. Move the community to act on their own problems changed positively
2. Make people aware of their own problems C. Agnesia gave feedback to Budek saying that she
3. Enable the nurse to solve the community problems understood the instruction
4. Offer people means of solving their own problems D. Agnesia requested a written instruction from Budek

A. 1,2,3 94. Which of the following is true about health


B. 1,2,3,4 education?
C. 1,2
D. 1,2,4 A. It helps people attain their health through the nurse’s
sole efforts
90. This is considered the first act of integrating with the B. It should not be flexible
people. This gives an in depth participation in C. It is a fast and mushroom like process

45
45
D. It is a slow and continuous process D. Muscle Built

95. Which of the following factors least influence the 100. Appearance and disposition of clients are best
learning readiness of an adult learner? observed initially during which of the following
situation?
A. The individuals stage of development
B. Ability to concentrate on information to be learned A. Taking V/S
C. The individual’s psychosocial adaptation to his illness B. Interview
D. The internal impulses that drive the person to C. Implementation of the initial care
take action D. Actual Physical examination

96. Which of the following is the most important


condition for diabetic patients to learn how to control
their diet?

A. Use of pamphlets and other materials during


instructions
B. Motivation to be symptom free
C. Ability of the patient to understand teaching
instruction
D. Language used by the nurse

97. An important skill that a primigravida has to acquire


is the ability to bathe her newborn baby and clean her
breast if she decides to breastfeed her baby, Which of
the following learning domain will you classify the above
goals?

A. Psychomotor
B. Cognitive
C. Affective
D. Attitudinal

98. When you prepare your teaching plan for a group of


hypertensive patients, you first formulate your learning
objectives. Which of the following steps in the nursing
process corresponds to the writing of the learning
objectives?

A. Planning
B. Implementing
C. Evaluation
C. Assessment

99. Rose, 50 years old and newly diagnosed diabetic


patient must learn how to inject insulin. Which of the
following physical attribute is not in anyway related to
her ability to administer insulin?

A. Strength
B. Coordination
C. Dexterity

45
45
COMMUNITY HEALTH NURSING Part 2
4. R.A. 1054 is also known as the Occupational Health
1. Which is the primary goal of community health Act. Aside from number of employees, what other factor
nursing? must be considered in determining the occupational
A. To support and supplement the efforts of the medical health privileges to which the workers will be entitled?
profession in the promotion of health and prevention of A. Type of occupation: agricultural, commercial,
illness industrial
B. To enhance the capacity of individuals, families and B. Location of the workplace in relation to health
communities to cope with their health needs facilities
C. To increase the productivity of the people by C. Classification of the business enterprise based on net
providing them with services that will increase their level profit
of health D. Sex and age composition of employees
D. To contribute to national development through
promotion of family welfare, focusing particularly on Answer: (B) Location of the workplace in relation to
mothers and children. health facilities
Based on R.A. 1054, an occupational nurse must be
Answer: (B) To enhance the capacity of individuals, employed when there are 30 to 100 employees and the
families and communities to cope with their health workplace is more than 1 km. away from the nearest
needs health center.
To contribute to national development through
promotion of family welfare, focusing particularly on 5. A business firm must employ an occupational health
mothers and children. nurse when it has at least how many employees?
A. 21
2. CHN is a community-based practice. Which best B. 101
explains this statement? C. 201
A. The service is provided in the natural environment of D. 301
people.
B. The nurse has to conduct community diagnosis to Answer: (B) 101
determine nursing needs and problems. Again, this is based on R.A. 1054.
C. The services are based on the available resources
within the community. 6. When the occupational health nurse employs
D. Priority setting is based on the magnitude of the ergonomic principles, she is performing which of her
health problems identified. roles?
A. Health care provider
Answer: A. The service is provided in the natural B. Health educator
environment of people. C. Health care coordinator
Community-based practice means providing care to D. Environmental manager
people in their own natural environments: the home,
school and workplace, for example. Answer: (D) Environmental manager
Ergonomics is improving efficiency of workers by
3. Population-focused nursing practice requires which of improving the worker’s environment through
the following processes? appropriately designed furniture, for example.
A. Community organizing
B. Nursing process 7. A garment factory does not have an occupational
C. Community diagnosis nurse. Who shall provide the occupational health needs
D. Epidemiologic process of the factory workers?
A. Occupational health nurse at the Provincial Health
Answer: (C) Community diagnosis Office
Population-focused nursing care means providing care B. Physician employed by the factory
based on the greater need of the majority of the C. Public health nurse of the RHU of their municipality
population. The greater need is identified through D. Rural sanitary inspector of the RHU of their
community diagnosis. municipality

45
45

A. It involves providing home care to sick people who are


Answer: (C) Public health nurse of the RHU of their not confined in the hospital.
municipality B. Services are provided free of charge to people
You’re right! This question is based on R.A.1054. within the catchment area.
C. The public health nurse functions as part of a team
8. “Public health services are given free of charge.” Is providing a public health nursing services.
this statement true or false? D. Public health nursing focuses on preventive,
A. The statement is true; it is the responsibility of not curative, services.
government to provide basic services.
B. The statement is false; people pay indirectly for public Answer: (D) Public health nursing focuses
health services. on preventive, not curative, services.
C. The statement may be true or false, depending on The catchment area in PHN consists of a residential
the specific service required. community, many of whom are well individuals who
D. The statement may be true or false, depending on have greater need for preventive rather than curative
policies of the government concerned. services.

Answer: (B) The statement is false; people pay 12. According to Margaret Shetland, the philosophy of
indirectly for public health services. public health nursing is based on which of the following?
Community health services, including public health A. Health and longevity as birthrights
services, are pre-paid services, though taxation, for B. The mandate of the state to protect the birthrights of
example. its citizens
C. Public health nursing as a specialized field of nursing
9. According to C.E.Winslow, which of the following is D. The worth and dignity of man
the goal of Public Health?
A. For people to attain their birthrights of health and Answer: (D) The worth and dignity of man
longevity This is a direct quote from Dr. Margaret Shetland’s
B. For promotion of health and prevention of disease statements on Public Health Nursing.
C. For people to have access to basic health services
D. For people to be organized in their health efforts 13. Which of the following is the mission of the
Department of Health?
Answer: (A) For people to attain their birthrights of A. Health for all Filipinos
health and longevity B. Ensure the accessibility and quality of health care
According to Winslow, all public health efforts are for C. Improve the general health status of the population
people to realize their birthrights of health and D. Health in the hands of the Filipino people by the year
longevity. 2020

10. We say that a Filipino has attained longevity when he Answer: (B) Ensure the accessibility and quality of
is able to reach the average lifespan of Filipinos. What health care
other statistic may be used to determine attainment of (none)
longevity?
A. Age-specific mortality rate 14. Region IV Hospital is classified as what level of
B. Proportionate mortality rate facility?
C. Swaroop’s index A. Primary
D. Case fatality rate B. Secondary
C. Intermediate
Answer: (C) Swaroop’s index D. Tertiary
Swaroop’s index is the percentage of the deaths aged 50
years or older. Its inverse represents the percentage of Answer: (D) Tertiary
untimely deaths (those who died younger than 50 years). Regional hospitals are tertiary facilities because they
serve as training hospitals for the region.
11. Which of the following is the most prominent feature
of public health nursing?

45
45
15. Which is true of primary facilities?
A. They are usually government-run. 19. R.A. 7160 mandates devolution of basic services
B. Their services are provided on an out-patient basis. from the national government to local government units.
C. They are training facilities for health professionals. Which of the following is the major goal of devolution?
D. A community hospital is an example of this level of A. To strengthen local government units
health facilities. B. To allow greater autonomy to local government units
C. To empower the people and promote their self-
Answer: (B) Their services are provided on an out- reliance
patient basis. D. To make basic services more accessible to the people
Primary facilities government and non-government
facilities that provide basic out-patient services. Answer: (C) To empower the people and promote their
self-reliance
16. Which is an example of the school nurse’s health People empowerment is the basic motivation behind
care provider functions? devolution of basic services to LGU’s.
A. Requesting for BCG from the RHU for school entrant
immunization 20. Who is the Chairman of the Municipal Health Board?
B. Conducting random classroom inspection during a A. Mayor
measles epidemic B. Municipal Health Officer
C. Taking remedial action on an accident hazard in the C. Public Health Nurse
school playground D. Any qualified physician
D. Observing places in the school where pupils spend
their free time Answer: (A) Mayor
The local executive serves as the chairman of the
Answer: (B) Conducting random classroom inspection Municipal Health Board.
during a measles epidemic
Random classroom inspection is assessment of 21. Which level of health facility is the usual point of
pupils/students and teachers for signs of a health entry of a client into the health care delivery system?
problem prevalent in the community. A. Primary
B. Secondary
17. When the nurse determines whether resources were C. Intermediate
maximized in implementing Ligtas Tigdas, she is D. Tertiary
evaluating
A. Effectiveness Answer: (A) Primary
B. Efficiency The entry of a person into the health care delivery
C. Adequacy system is usually through a consultation in out-patient
D. Appropriateness services.

Answer: (B) Efficiency 22. The public health nurse is the supervisor of rural
Efficiency is determining whether the goals were health midwives. Which of the following is a supervisory
attained at the least possible cost. function of the public health nurse?
A. Referring cases or patients to the midwife
18. You are a new B.S.N. graduate. You want to become B. Providing technical guidance to the midwife
a Public Health Nurse. Where will you apply? C. Providing nursing care to cases referred by the
A. Department of Health midwife
B. Provincial Health Office D. Formulating and implementing training programs for
C. Regional Health Office midwives
D. Rural Health Unit
Answer: (B) Providing technical guidance to the
Answer: (D) Rural Health Unit midwife
R.A. 7160 devolved basic health services to local The nurse provides technical guidance to the midwife in
government units (LGU’s ). The public health nurse is an the care of clients, particularly in the implementation of
employee of the LGU.

45
45
management guidelines, as in Integrated Management Diseases, enacted in 1929, mandated the reporting of
of Childhood Illness. diseases listed in the law to the nearest health station.

23. One of the participants in a hilot training class asked 27. According to Freeman and Heinrich, community
you to whom she should refer a patient in labor who health nursing is a developmental service. Which of the
develops a complication. You will answer, to the following best illustrates this statement?
A. Public Health Nurse A. The community health nurse continuously develops
B. Rural Health Midwife himself personally and professionally.
C. Municipal Health Officer B. Health education and community organizing are
D. Any of these health professionals necessary in providing community health services.
C. Community health nursing is intended primarily for
Answer: (C) Municipal Health Officer health promotion and prevention and treatment of
A public health nurse and rural health midwife can disease.
provide care during normal childbirth. A physician should D. The goal of community health nursing is to provide
attend to a woman with a complication during labor. nursing services to people in their own places of
residence.
24. You are the public health nurse in a municipality with
a total population of about 20,000. There are 3 rural Answer: (B) Health education and community
health midwives among the RHU personnel. How many organizing are necessary in providing community health
more midwife items will the RHU need? services.
A. 1 The community health nurse develops the health
B. 2 capability of people through health education and
C. 3 community organizing activities.
D. The RHU does not need any more midwife item.
28. Which disease was declared through Presidential
Answer: (A) 1 Proclamation No. 4 as a target for eradication in the
Each rural health midwife is given a population Philippines?
assignment of about 5,000. A. Poliomyelitis
B. Measles
25. If the RHU needs additional midwife items, you C. Rabies
will submit the request for additional midwife items D. Neonatal tetanus
for approval to the
A. Rural Health Unit Answer: (B) Measles
B. District Health Office Presidential Proclamation No. 4 is on the Ligtas Tigdas
C. Provincial Health Office Program.
D. Municipal Health Board
29. The public health nurse is responsible for presenting
Answer: (D) Municipal Health Board the municipal health statistics using graphs and tables.
As mandated by R.A. 7160, basic health services have To compare the frequency of the leading causes of
been devolved from the national government to local mortality in the municipality, which graph will you
government units. prepare?
A. Line
26. As an epidemiologist, the nurse is responsible for B. Bar
reporting cases of notifiable diseases. What law C. Pie
mandates reporting of cases of notifiable diseases? D. Scatter diagram
A. Act 3573
B. R.A. 3753 Answer: (B) Bar
C. R.A. 1054 A bar graph is used to present comparison of values, a
D. R.A. 1082 line graph for trends over time or age, a pie graph for
population composition or distribution, and a scatter
Answer: (A) Act 3573 diagram for correlation of two variables.
Act 3573, the Law on Reporting of Communicable

45
45
30. Which step in community organizing involves problem
training of potential leaders in the community? D. Identify the health problem as a common concern
A. Integration
B. Community organization Answer: (A) Participate in community activities for the
C. Community study solution of a community problem
D. Core group formation Participation in community activities in resolving a
community problem may be in any of the processes
Answer: (D) Core group formation mentioned in the other choices.
In core group formation, the nurse is able to transfer the
technology of community organizing to the potential or 34. Tertiary prevention is needed in which stage of the
informal community leaders through a training program. natural history of disease?
A. Pre-pathogenesis
31. In which step are plans formulated for B. Pathogenesis
solving community problems? C. Prodromal
A. Mobilization D. Terminal
B. Community organization
C. Follow-up/extension Answer: (D) Terminal
D. Core group formation Tertiary prevention involves rehabilitation, prevention of
permanent disability and disability limitation appropriate
Answer: (B) Community organization for convalescents, the disabled, complicated cases and
Community organization is the step when community the terminally ill (those in the terminal stage of a
assemblies take place. During the community assembly, disease)
the people may opt to formalize the community
organization and make plans for community action to 35. Isolation of a child with measles belongs to what
resolve a community health problem. level of prevention?
A. Primary
32. The public health nurse takes an active role in B. Secondary
community participation. What is the primary goal of C. Intermediate
community organizing? D. Tertiary
A. To educate the people regarding community health
problems Answer: (A) Primary
B. To mobilize the people to resolve community health The purpose of isolating a client with a communicable
problems disease is to protect those who are not sick (specific
C. To maximize the community’s resources in dealing disease prevention).
with health problems
D. To maximize the community’s resources in dealing 36. On the other hand, Operation Timbang is
with health problems prevention.
A. Primary
Answer: (D) To maximize the community’s resources in B. Secondary
dealing with health problems C. Intermediate
Community organizing is a developmental service, with D. Tertiary
the goal of developing the people’s self-reliance in
dealing with community health problems. A, B and C are Answer: (B) Secondary
objectives of contributory objectives to this goal. Operation Timbang is done to identify members of the
susceptible population who are malnourished. Its
33. An indicator of success in community organizing is purpose is early diagnosis and, subsequently, prompt
when people are able to treatment.
A. Participate in community activities for the solution of
a community problem 37. Which type of family-nurse contact will provide you
B. Implement activities for the solution of the with the best opportunity to observe family dynamics?
community problem A. Clinic consultation
C. Plan activities for the solution of the community B. Group conference

45
45
C. Home visit 41. Which is CONTRARY to the principles in planning a
D. Written communication home visit?
A. A home visit should have a purpose or objective.
Answer: (C) Home visit B. The plan should revolve around family health needs.
Dynamics of family relationships can best be observed in C. A home visit should be conducted in the manner
the family’s natural environment, which is the home. prescribed by the RHU.
D. Planning of continuing care should involve
38. The typology of family nursing problems is used in a responsible family member.
the statement of nursing diagnosis in the care of
families. The youngest child of the de los Reyes family Answer: (C) A home visit should be conducted in the
has been diagnosed as mentally retarded. This is manner prescribed by the RHU.
classified as a The home visit plan should be flexible and practical,
A. Health threat depending on factors, such as the family’s needs and
B. Health deficit the resources available to the nurse and the family.
C. Foreseeable crisis
D. Stress point 42. The PHN bag is an important tool in providing
nursing care during a home visit. The most important
Answer: (B) Health deficit principle of bag technique states that it
Failure of a family member to develop according to what A. Should save time and effort.
is expected, as in mental retardation, is a health deficit. B. Should minimize if not totally prevent the spread of
infection.
39. The de los Reyes couple have a 6-year old child C. Should not overshadow concern for the patient
entering school for the first time. The de los Reyes family and his family.
has a D. May be done in a variety of ways depending on
A. Health threat the home situation, etc.
B. Health deficit
C. Foreseeable crisis Answer: (B) Should minimize if not totally prevent the
D. Stress point spread of infection.
Bag technique is performed before and after handling a
Answer: (C) Foreseeable crisis client in the home to prevent transmission of infection
Entry of the 6-year old into school is an anticipated to and from the client.
period of unusual demand on the family.
43. To maintain the cleanliness of the bag and its
40. Which of the following is an advantage of a home contents, which of the following must the nurse do?
visit? A. Wash his/her hands before and after providing
A. It allows the nurse to provide nursing care to a greater nursing care to the family members.
number of people. B. In the care of family members, as much as possible,
B. It provides an opportunity to do first hand appraisal of use only articles taken from the bag.
the home situation. C. Put on an apron to protect her uniform and fold it
C. It allows sharing of experiences among people with with the right side out before putting it back into the
similar health problems. bag.
D. It develops the family’s initiative in providing for D. At the end of the visit, fold the lining on which the
health needs of its members. bag was placed, ensuring that the contaminated side is
on the outside.
Answer: (B) It provides an opportunity to do first hand
appraisal of the home situation. Answer: (A) Wash his/her hands before and after
Choice A is not correct since a home visit requires that providing nursing care to the family members.
the nurse spend so much time with the family. Choice C Choice B goes against the idea of utilizing the family’s
is an advantage of a group conference, while choice D is resources, which is encouraged in CHN. Choices C and D
true of a clinic consultation. goes against the principle of asepsis of confining the
contaminated surface of objects.

45
46
44. The public health nurse conducts a study on the 47. The primary purpose of conducting an epidemiologic
factors contributing to the high mortality rate due to investigation is to
heart disease in the municipality where she works. A. Delineate the etiology of the epidemic
Which branch of epidemiology does the nurse practice in B. Encourage cooperation and support of the community
this situation? C. Identify groups who are at risk of contracting the
A. Descriptive disease
B. Analytical D. Identify geographical location of cases of the disease
C. Therapeutic in the community
D. Evaluation
Answer: (A) Delineate the etiology of the epidemic
Answer: (B) Analytical Delineating the etiology of an epidemic is identifying its
Analytical epidemiology is the study of factors or source.
determinants affecting the patterns of occurrence and
distribution of disease in a community. 48. Which is a characteristic of person-to-person
propagated epidemics?
45. Which of the following is a function of epidemiology? A. There are more cases of the disease than expected.
A. Identifying the disease condition based on B. The disease must necessarily be transmitted through a
manifestations presented by a client vector.
B. Determining factors that contributed to the C. The spread of the disease can be attributed to a
occurrence of pneumonia in a 3 year old common vehicle.
C. Determining the efficacy of the antibiotic used in the D. There is a gradual build up of cases before the
treatment of the 3 year old client with pneumonia epidemic becomes easily noticeable.
D. Evaluating the effectiveness of the implementation of
the Integrated Management of Childhood Illness Answer: (D) There is a gradual build up of cases before
the epidemic becomes easily noticeable.
Answer: (D) Evaluating the effectiveness of the A gradual or insidious onset of the epidemic is usually
implementation of the Integrated Management of observable in person-to-person propagated epidemics.
Childhood Illness
Epidemiology is used in the assessment of a community 49. In the investigation of an epidemic, you compare the
or evaluation of interventions in community health present frequency of the disease with the usual
practice. frequency at this time of the year in this community. This
is done during which stage of the investigation?
46. Which of the following is an epidemiologic function A. Establishing the epidemic
of the nurse during an epidemic? B. Testing the hypothesis
A. Conducting assessment of suspected cases to detect C. Formulation of the hypothesis
the communicable disease D. Appraisal of facts
B. Monitoring the condition of the cases affected by the
communicable disease Answer: (A) Establishing the epidemic
C. Participating in the investigation to determine the Establishing the epidemic is determining whether there
source of the epidemic is an epidemic or not. This is done by comparing the
D. Teaching the community on preventive measures present number of cases with the usual number of cases
against the disease of the disease at the same time of the year, as well as
establishing the relatedness of the cases of the disease.
Answer: (C) Participating in the investigation to
determine the source of the epidemic 50. The number of cases of Dengue fever usually
Epidemiology is the study of patterns of occurrence and increases towards the end of the rainy season. This
distribution of disease in the community, as well as the pattern of occurrence of Dengue fever is best described
factors that affect disease patterns. The purpose of an as
epidemiologic investigation is to identify the source of A. Epidemic occurrence
an epidemic, i.e., what brought about the epidemic. B. Cyclical variation
C. Sporadic occurrence
D. Secular variation

46
46
of a
Answer: (B) Cyclical variation
A cyclical variation is a periodic fluctuation in the
number of cases of a disease in the community.

51. In the year 1980, the World Health Organization


declared the Philippines, together with some other
countries in the Western Pacific Region, “free” of which
disease?
A. Pneumonic plague
B. Poliomyelitis
C. Small pox
D. Anthrax

Answer: (C) Small pox


The last documented case of Small pox was in 1977 at
Somalia.

52. In the census of the Philippines in 1995, there were


about 35,299,000 males and about 34,968,000 females.
What is the sex ratio?
A. 99.06:100
B. 100.94:100
C. 50.23%
D. 49.76%

Answer: (B) 100.94:100


Sex ratio is the number of males for every 100 females in
the population.

53. Primary health care is a total approach to


community development. Which of the following is an
indicator of success in the use of the primary health care
approach?
A. Health services are provided free of charge to
individuals and families.
B. Local officials are empowered as the major decision
makers in matters of health.
C. Health workers are able to provide care based on
identified health needs of the people.
D. Health programs are sustained according to the
level of development of the community.

Answer: (D) Health programs are sustained according to


the level of development of the community.
Primary health care is essential health care that can be
sustained in all stages of development of the
community.

54. Sputum examination is the major screening tool


for pulmonary tuberculosis. Clients would sometimes
get false negative results in this exam. This means that
the test is not perfect in terms of which characteristic
46
46
diagnostic examination?
A. Effectiveness
B. Efficacy
C. Specificity
D. Sensitivity

Answer: (D) Sensitivity


Sensitivity is the capacity of a diagnostic examination
to detect cases of the disease. If a test is 100%
sensitive, all the cases tested will have a positive result,
i.e., there will be no false negative results.

55. Use of appropriate technology requires knowledge


of indigenous technology. Which medicinal herb is
given for fever, headache and cough?
A. Sambong
B. Tsaang gubat
C. Akapulko
D. Lagundi

Answer: (D) Lagundi


Sambong is used as a diuretic. Tsaang gubat is used
to relieve diarrhea. Akapulko is used for its
antifungal property.

56. What law created the Philippine Institute


of Traditional and Alternative Health Care?
A. R.A. 8423
B. R.A. 4823
C. R.A. 2483
D. R.A. 3482

Answer: (A) R.A. 8423


(none)

57. In traditional Chinese medicine, the


yielding, negative and feminine force is
termed
A. Yin
B. Yang
C. Qi
D. Chai

Answer: (A) Yin


Yang is the male dominating, positive and
masculine force.

58. What is the legal basis for Primary Health


Care approach in the Philippines?
A. Alma Ata Declaration on PHC
B. Letter of Instruction No. 949
C. Presidential Decree No. 147
D. Presidential Decree 996

46
46

C. Population pyramid
Answer: (B) Letter of Instruction No. 949 D. Any of these may be used.
Letter of Instruction 949 was issued by then President
Ferdinand Marcos, directing the formerly called Ministry Answer: (D) Any of these may be used.
of Health, now the Department of Health, to utilize Sex ratio and sex proportion are used to determine the
Primary Health Care approach in planning and sex composition of a population. A population pyramid is
implementing health programs. used to present the composition of a population by age
and sex.
59. Which of the following demonstrates intersectoral
linkages? 63. Which of the following is a natality rate?
A. Two-way referral system A. Crude birth rate
B. Team approach B. Neonatal mortality rate
C. Endorsement done by a midwife to another midwife C. Infant mortality rate
D. Cooperation between the PHN and public D. General fertility rate
school teacher
Answer: (A) Crude birth rate
Answer: (D) Cooperation between the PHN and public Natality means birth. A natality rate is a birth rate.
school teacher
Intersectoral linkages refer to working relationships 64. You are computing the crude death rate of your
between the health sector and other sectors involved in municipality, with a total population of about 18,000, for
community development. last year. There were 94 deaths. Among those who died,
20 died because of diseases of the heart and 32 were
60. The municipality assigned to you has a population of aged 50 years or older. What is the crude death rate?
about 20,000. Estimate the number of 1-4 year old A. 4.2/1,000
children who will be given Retinol capsule 200,000 I.U. B. 5.2/1,000
every 6 months. C. 6.3/1,000
A. 1,500 D. 7.3/1,000
B. 1,800
C. 2,000 Answer: (B) 5.2/1,000
D. 2,300 To compute crude death rate divide total number of
deaths (94) by total population (18,000) and multiply by
Answer: (D) 2,300 1,000.
Based on the Philippine population composition, to
estimate the number of 1-4 year old children, multiply 65. Knowing that malnutrition is a frequent community
total population by 11.5%. health problem, you decided to conduct nutritional
assessment. What population is particularly susceptible
61. Estimate the number of pregnant women who will to protein energy malnutrition (PEM)?
be given tetanus toxoid during an immunization A. Pregnant women and the elderly
outreach activity in a barangay with a population of B. Under-5 year old children
about 1,500. A. 265 C. 1-4 year old children
B. 300 D. School age children
C. 375
D. 400 Answer: (C) 1-4 year old children
Preschoolers are the most susceptible to PEM because
Answer: (A) 265 they have generally been weaned. Also, this is the
To estimate the number of pregnant women, multiply population who, unable to feed themselves, are often
the total population by 3.5%. the victims of poor intrafamilial food distribution.

62. To describe the sex composition of the population, 66. Which statistic can give the most accurate reflection
which demographic tool may be used? of the health status of a community?
A. Sex ratio A. 1-4 year old age-specific mortality rate
B. Sex proportion B. Infant mortality rate

46
46
C. Swaroop’s index 70. You will gather data for nutritional assessment of a
D. Crude death rate purok. You will gather information only from families
with members who belong to the target population for
Answer: (C) Swaroop’s index PEM. What method of data gathering is best for this
Swaroop’s index is the proportion of deaths aged 50 purpose?
years and above. The higher the Swaroop’s index of a A. Census
population, the greater the proportion of the deaths B. Survey
who were able to reach the age of at least 50 years, i.e., C. Record review
more people grew old before they died. D. Review of civil registry
67. In the past year, Barangay A had an average
population of 1655. 46 babies were born in that year, 2 Answer: (B) Survey
of whom died less than 4 weeks after they were born. A survey, also called sample survey, is data gathering
There were 4 recorded stillbirths. What is the neonatal about a sample of the population.
mortality rate?
A. 27.8/1,000 71. In the conduct of a census, the method of population
B. 43.5/1,000 assignment based on the actual physical location of the
C. 86.9/1,000 people is termed
D. 130.4/1,000 A. De jure
B. De locus
Answer: (B) 43.5/1,000 C. De facto
To compute for neonatal mortality rate, divide the D. De novo
number of babies who died before reaching the age of
28 days by the total number of live births, then multiply Answer: (C) De facto
by 1,000. The other method of population assignment, de jure, is
based on the usual place of residence of the people.
68. Which statistic best reflects the nutritional status of a
population? 72. The Field Health Services and Information System
A. 1-4 year old age-specific mortality rate (FHSIS) is the recording and reporting system in public
B. Proportionate mortality rate health care in the Philippines. The Monthly Field Health
C. Infant mortality rate Service Activity Report is a form used in which of the
D. Swaroop’s index components of the FHSIS?
A. Tally report
Answer: (A) 1-4 year old age-specific mortality rate B. Output report
Since preschoolers are the most susceptible to the C. Target/client list
effects of malnutrition, a population with poor D. Individual health record
nutritional status will most likely have a high 1-4 year old
age-specific mortality rate, also known as child mortality Answer: (A) Tally report
rate. A tally report is prepared monthly or quarterly by the
RHU personnel and transmitted to the Provincial Health
69. What numerator is used in computing general Office.
fertility rate?
A. Estimated midyear population 73. To monitor clients registered in long-term regimens,
B. Number of registered live births such as the Multi-Drug Therapy, which component will
C. Number of pregnancies in the year be most useful?
D. Number of females of reproductive age A. Tally report
B. Output report
Answer: (B) Number of registered live births C. Target/client list
To compute for general or total fertility rate, divide the D. Individual health record
number of registered live births by the number of
females of reproductive age (15-45 years), then multiply Answer: (C) Target/client list
by 1,000. The MDT Client List is a record of clients enrolled in MDT

46
46
and other relevant data, such as dates when clients centers able to comply with standards.
collected their monthly supply of drugs. Sentrong Sigla Movement is a joint project of the DOH
and local government units. Its main strategy is
74. Civil registries are important sources of data. certification of health centers that are able to comply
Which law requires registration of births within 30 days with standards set by the DOH.
from the occurrence of the birth?
A. P.D. 651 78. Which of the following women should be considered
B. Act 3573 as special targets for family planning?
C. R.A. 3753 A. Those who have two children or more
D. R.A. 3375 B. Those with medical conditions such as anemia
C. Those younger than 20 years and older than 35 years
Answer: (A) P.D. 651 D. Those who just had a delivery within the past
P.D. 651 amended R.A. 3753, requiring the registry of 15 months
births within 30 days from their occurrence.
Answer: (D) Those who just had a delivery within the
75. Which of the following professionals can sign the past 15 months
birth certificate? The ideal birth spacing is at least two years. 15 months
A. Public health nurse plus 9 months of pregnancy = 2 years.
B. Rural health midwife
C. Municipal health officer 79. Freedom of choice is one of the policies of the
D. Any of these health professionals Family Planning Program of the Philippines. Which of the
following illustrates this principle?
Answer: (D) Any of these health professionals A. Information dissemination about the need for family
D. R.A. 3753 states that any birth attendant may sign the planning
certificate of live birth. B. Support of research and development in family
planning methods
76. Which criterion in priority setting of health problems C. Adequate information for couples regarding the
is used only in community health care? different methods
A. Modifiability of the problem D. Encouragement of couples to take family planning
B. Nature of the problem presented as a joint responsibility
C. Magnitude of the health problem
D. Preventive potential of the health problem Answer: (C) Adequate information for couples
regarding the different methods
Answer: (C) Magnitude of the health problem To enable the couple to choose freely among different
Magnitude of the problem refers to the percentage of methods of family planning, they must be given full
the population affected by a health problem. The other information regarding the different methods that are
choices are criteria considered in both family and available to them, considering the availability of quality
community health care. services that can support their choice.

77. The Sentrong Sigla Movement has been launched to 80. A woman, 6 months pregnant, came to the center
improve health service delivery. Which of the following for consultation. Which of the following substances is
is/are true of this movement? contraindicated?
A. This is a project spearheaded by local government A. Tetanus toxoid
units. B. Retinol 200,000 IU
B. It is a basis for increasing funding from local C. Ferrous sulfate 200 mg
government units. D. Potassium iodate 200 mg. capsule
C. It encourages health centers to focus on disease
prevention and control. Answer: (B) Retinol 200,000 IU
D. Its main strategy is certification of health centers able Retinol 200,000 IU is a form of megadose Vitamin A. This
to comply with standards. may have a teratogenic effect.

Answer: (D) Its main strategy is certification of health

46
46
81. During prenatal consultation, a client asked you if Answer: (D) Explain to her that putting the baby to
she can have her delivery at home. After history taking breast will lessen blood loss after delivery.
and physical examination, you advised her against a Suckling of the nipple stimulates the release of oxytocin
home delivery. Which of the following findings by the posterior pituitary gland, which causes uterine
disqualifies her for a home delivery? contraction. Lactation begins 1 to 3 days after delivery.
A. Her OB score is G5P3. Nipple stretching exercises are done when the nipples
B. She has some palmar pallor. are flat or inverted. Frequent washing dries up the
C. Her blood pressure is 130/80. nipples, making them prone to the formation of fissures.
D. Her baby is in cephalic presentation.
85. A primigravida is instructed to offer her breast to the
Answer: (A) Her OB score is G5P3. baby for the first time within 30 minutes after delivery.
Only women with less than 5 pregnancies are qualified What is the purpose of offering the breast this early?
for a home delivery. It is also advisable for a primigravida A. To initiate the occurrence of milk letdown
to have delivery at a childbirth facility. B. To stimulate milk production by the mammary acini
C. To make sure that the baby is able to get the
82. Inadequate intake by the pregnant woman of colostrum
which vitamin may cause neural tube defects? D. To allow the woman to practice breastfeeding in the
A. Niacin presence of the health worker
B. Riboflavin
C. Folic acid Answer: (B) To stimulate milk production by the
D. Thiamine mammary acini
Suckling of the nipple stimulates prolactin reflex (the
Answer: (C) Folic acid release of prolactin by the anterior pituitary gland),
It is estimated that the incidence of neural tube defects which initiates lactation.
can be reduced drastically if pregnant women have an
adequate intake of folic acid. 86. In a mothers’ class, you discuss proper breastfeeding
technique. Which is of these is a sign that the baby has
83. You are in a client’s home to attend to a delivery. “latched on” to the breast properly?
Which of the following will you do first? A. The baby takes shallow, rapid sucks.
A. Set up the sterile area. B. The mother does not feel nipple pain.
B. Put on a clean gown or apron. C. The baby’s mouth is only partly open.
C. Cleanse the client’s vulva with soap and water. D. Only the mother’s nipple is inside the baby’s mouth.
D. Note the interval, duration and intensity of labor
contractions. Answer: (B) The mother does not feel nipple pain.
When the baby has properly latched on to the breast, he
Answer: (D) Note the interval, duration and intensity of takes deep, slow sucks; his mouth is wide open; and
labor contractions. much of the areola is inside his mouth. And, you’re right!
Assessment of the woman should be done first to The mother does not feel nipple pain.
determine whether she is having true labor and, if so,
what stage of labor she is in. 87. You explain to a breastfeeding mother that breast
milk is sufficient for all of the baby’s nutrient needs only
84. In preparing a primigravida for breastfeeding, which up to .
of the following will you do? A. 3 months
A. Tell her that lactation begins within a day after B. 6 months
delivery. C. 1 year
B. Teach her nipple stretching exercises if her nipples are D. 2 years
everted.
C. Instruct her to wash her nipples before and after each Answer: (B) 6 months
breastfeeding. After 6 months, the baby’s nutrient needs, especially the
D. Explain to her that putting the baby to breast will baby’s iron requirement, can no longer be provided by
lessen blood loss after delivery. mother’s milk alone.

46
46
88. What is given to a woman within a month after the
delivery of a baby? 92. Which immunization produces a permanent scar?
A. Malunggay capsule A. DPT
B. Ferrous sulfate 100 mg. OD B. BCG
C. Retinol 200,000 I.U., 1 capsule C. Measles vaccination
D. Potassium iodate 200 mg, 1 capsule D. Hepatitis B vaccination

Answer: (C) Retinol 200,000 I.U., 1 capsule Answer: (B) BCG


A capsule of Retinol 200,000 IU is given within 1 month BCG causes the formation of a superficial abscess, which
after delivery. Potassium iodate is given during begins 2 weeks after immunization. The abscess heals
pregnancy; malunggay capsule is not routinely without treatment, with the formation of a permanent
administered after delivery; and ferrous sulfate is taken scar.
for two months after delivery.
93. A 4-week old baby was brought to the health
89. Which biological used in Expanded Program on center for his first immunization. Which can be given to
Immunization (EPI) is stored in the freezer? him?
A. DPT A. DPT1
B. Tetanus toxoid B. OPV1
C. Measles vaccine C. Infant BCG
D. Hepatitis B vaccine D. Hepatitis B vaccine 1

Answer: (C) Measles vaccine Answer: (C) Infant BCG


Among the biologicals used in the Expanded Program on Infant BCG may be given at birth. All the other
Immunization, measles vaccine and OPV are highly immunizations mentioned can be given at 6 weeks of
sensitive to heat, requiring storage in the freezer. age.

90. Unused BCG should be discarded how many hours 94. You will not give DPT 2 if the mother says that the
after reconstitution? infant had
A. 2 A. Seizures a day after DPT 1.
B. 4 B. Fever for 3 days after DPT 1.
C. 6 C. Abscess formation after DPT 1.
D. At the end of the day D. Local tenderness for 3 days after DPT 1.

Answer: (B) 4 Answer: (A) Seizures a day after DPT 1.


While the unused portion of other biologicals in EPI may Seizures within 3 days after administration of DPT is an
be given until the end of the day, only BCG is discarded 4 indication of hypersensitivity to pertussis vaccine, a
hours after reconstitution. This is why BCG immunization component of DPT. This is considered a specific
is scheduled only in the morning. contraindication to subsequent doses of DPT.

91. In immunizing school entrants with BCG, you are 95. A 2-month old infant was brought to the health
not obliged to secure parental consent. This is because center for immunization. During assessment, the infant’s
of which legal document? temperature registered at 38.1°C. Which is the best
A. P.D. 996 course of action that you will take?
B. R.A. 7846 A. Go on with the infant’s immunizations.
C. Presidential Proclamation No. 6 B. Give Paracetamol and wait for his fever to subside.
D. Presidential Proclamation No. 46 C. Refer the infant to the physician for further
assessment.
Answer: (A) P.D. 996 D. Advise the infant’s mother to bring him back for
Presidential Decree 996, enacted in 1976, made immunization when he is well.
immunization in the EPI compulsory for children under 8
years of age. Hepatitis B vaccination was made Answer: (A) Go on with the infant’s immunizations.
compulsory for the same age group by R.A. 7846. In the EPI, fever up to 38.5°C is not a contraindication to
immunization. Mild acute respiratory tract infection,
46
46
simple diarrhea and malnutrition are not Answers A, C and D are done for a client classified as
contraindications either. having pneumonia.

96. A pregnant woman had just received her 4th dose 100. A 5-month old infant was brought by his mother
of tetanus toxoid. Subsequently, her baby will have to the health center because of diarrhea occurring 4 to
protection against tetanus for how long? 5 times a day. His skin goes back slowly after a skin
A. 1 year pinch and his eyes are sunken. Using the IMCI
B. 3 years guidelines, you will classify this infant in which
C. 10 years category?
D. Lifetime A. No signs of dehydration
B. Some dehydration
Answer: (A) 1 year C. Severe dehydration
The baby will have passive natural immunity by placental D. The data is insufficient.
transfer of antibodies. The mother will have active
artificial immunity lasting for about 10 years. 5 doses will Answer: (B) Some dehydration
give the mother lifetime protection. Using the assessment guidelines of IMCI, a child (2
months to 5 years old) with diarrhea is classified as
97. A 4-month old infant was brought to the health having SOME DEHYDRATION if he shows 2 or more of
center because of cough. Her respiratory rate is the following signs: restless or irritable, sunken eyes,
42/minute. Using the Integrated Management of Child the skin goes back slow after a skin pinch.
Illness (IMCI) guidelines of assessment, her breathing is
considered 101. Based on assessment, you classified a 3-month old
A. Fast infant with the chief complaint of diarrhea in the
B. Slow category of SOME DEHYDRATION. Based on IMCI
C. Normal management guidelines, which of the following will you
D. Insignificant do?
A. Bring the infant to the nearest facility where IV fluids
Answer: (C) Normal can be given.
In IMCI, a respiratory rate of 50/minute or more is fast B. Supervise the mother in giving 200 to 400 ml. of
breathing for an infant aged 2 to 12 months. Oresol in 4 hours.
C. Give the infant’s mother instructions on home
98. Which of the following signs will indicate that a management.
young child is suffering from severe pneumonia? D. Keep the infant in your health center for close
A. Dyspnea observation.
B. Wheezing Answer: (B) Supervise the mother in giving 200 to 400
C. Fast breathing ml. of Oresol in 4 hours.
D. Chest indrawing In the IMCI management guidelines, SOME
DEHYDRATION is treated with the administration of
Answer: (D) Chest indrawing Oresol within a period of 4 hours. The amount of Oresol
In IMCI, chest indrawing is used as the positive sign of is best computed on the basis of the child’s weight (75
dyspnea, indicating severe pneumonia. ml/kg body weight). If the weight is unknown, the
amount of Oresol is based on the child’s age.
99. Using IMCI guidelines, you classify a child as having
severe pneumonia. What is the best management for 102. A mother is using Oresol in the management of
the child? diarrhea of her 3-year old child. She asked you what to
A. Prescribe an antibiotic. do if her child vomits. You will tell her to
B. Refer him urgently to the hospital. A. Bring the child to the nearest hospital for further
C. Instruct the mother to increase fluid intake. assessment.
D. Instruct the mother to continue breastfeeding. B. Bring the child to the health center for intravenous
fluid therapy.
Answer: (B) Refer him urgently to the hospital. C. Bring the child to the health center for assessment
Severe pneumonia requires urgent referral to a hospital. by the physician.
D. Let the child rest for 10 minutes then continue giving
46
46
Oresol more slowly. which is not observable during physical examination.The
earliest visible lesion is conjunctival xerosis or dullness of
Answer: (D) Let the child rest for 10 minutes then the conjunctiva due to inadequate tear production.
continue giving Oresol more slowly.
If the child vomits persistently, that is, he vomits 106. To prevent xerophthalmia, young children are given
everything that he takes in, he has to be referred Retinol capsule every 6 months. What is the dose given
urgently to a hospital. Otherwise, vomiting is managed to preschoolers?
by letting the child rest for 10 minutes and then A. 10,000 IU
continuing with Oresol administration. Teach the mother B. 20,000 IU
to give Oresol more slowly. C. 100,000 IU
D. 200,000 IU
103. A 1 ½ year old child was classified as having 3rd
degree protein energy malnutrition, kwashiorkor. Which Answer: (D) 200,000 IU
of the following signs will be most apparent in this Preschoolers are given Retinol 200,000 IU every 6
child? months. 100,000 IU is given once to infants aged 6 to 12
A. Voracious appetite months. The dose for pregnant women is 10,000 IU.
B. Wasting
C. Apathy 107. The major sign of iron deficiency anemia is pallor.
D. Edema What part is best examined for pallor?
A. Palms
Answer: (D) Edema B. Nailbeds
Edema, a major sign of kwashiorkor, is caused by C. Around the lips
decreased colloidal osmotic pressure of the blood D. Lower conjunctival sac
brought about by hypoalbuminemia. Decreased blood
albumin level is due a protein-deficient diet. Answer: (A) Palms
The anatomic characteristics of the palms allow a
104. Assessment of a 2-year old child revealed “baggy reliable and convenient basis for examination for pallor.
pants”. Using the IMCI guidelines, how will you manage 108. Food fortification is one of the strategies to prevent
this child? micronutrient deficiency conditions. R.A. 8976 mandates
A. Refer the child urgently to a hospital for confinement. fortification of certain food items. Which of the following
B. Coordinate with the social worker to enroll the child is among these food items?
in a feeding program. A. Sugar
C. Make a teaching plan for the mother, focusing on B. Bread
menu planning for her child. C. Margarine
D. Assess and treat the child for health problems like D. Filled milk
infections and intestinal parasitism.
Answer: (A) Sugar
Answer: (A) Refer the child urgently to a hospital for R.A. 8976 mandates fortification of rice, wheat flour,
confinement. sugar and cooking oil with Vitamin A, iron and/or iodine.
“Baggy pants” is a sign of severe marasmus. The best
management is urgent referral to a hospital. 109. What is the best course of action when there is a
measles epidemic in a nearby municipality?
105. During the physical examination of a young child, A. Give measles vaccine to babies aged 6 to 8 months.
what is the earliest sign of xerophthalmia that you may B. Give babies aged 6 to 11 months one dose of 100,000
observe? I.U. of Retinol
A. Keratomalacia C. Instruct mothers to keep their babies at home
B. Corneal opacity to prevent disease transmission.
C. Night blindness D. Instruct mothers to feed their babies adequately to
D. Conjunctival xerosis enhance their babies’ resistance.

Answer: (D) Conjunctival xerosis Answer: (A) Give measles vaccine to babies aged 6 to 8
The earliest sign of Vitamin A deficiency (xerophthalmia) months.
is night blindness. However, this is a functional change,
46
47
Ordinarily, measles vaccine is given at 9 months of age.
During an impending epidemic, however, one dose may 113. The following are strategies implemented by the
be given to babies aged 6 to 8 months. The mother is Department of Health to prevent mosquito-borne
instructed that the baby needs another dose when the diseases. Which of these is most effective in the control
baby is 9 months old. of Dengue fever?
A. Stream seeding with larva-eating fish
110. A mother brought her daughter, 4 years old, to the B. Destroying breeding places of mosquitoes
RHU because of cough and colds. Following the IMCI C. Chemoprophylaxis of non-immune persons going
assessment guide, which of the following is a danger sign to endemic areas
that indicates the need for urgent referral to a hospital? D. Teaching people in endemic areas to use chemically
A. Inability to drink treated mosquito nets
B. High grade fever
C. Signs of severe dehydration Answer: (B) Destroying breeding places of mosquitoes
D. Cough for more than 30 days Aedes aegypti, the vector of Dengue fever, breeds in
stagnant, clear water. Its feeding time is usually during
Answer: (A) Inability to drink the daytime. It has a cyclical pattern of occurrence,
A sick child aged 2 months to 5 years must be referred unlike malaria which is endemic in certain parts of the
urgently to a hospital if he/she has one or more of the country.
following signs: not able to feed or drink, vomits
everything, convulsions, abnormally sleepy or difficult to 114. Secondary prevention for malaria includes
awaken. A. Planting of neem or eucalyptus trees
B. Residual spraying of insecticides at night
111. Management of a child with measles includes the C. Determining whether a place is endemic or not
administration of which of the following? D. Growing larva-eating fish in mosquito breeding places
A. Gentian violet on mouth lesions
B. Antibiotics to prevent pneumonia Answer: (C) Determining whether a place is endemic or
C. Tetracycline eye ointment for corneal opacity not
D. Retinol capsule regardless of when the last dose This is diagnostic and therefore secondary level
was given prevention. The other choices are for primary
prevention.
Answer: (D) Retinol capsule regardless of when the last
dose was given 115. Scotch tape swab is done to check for which
An infant 6 to 12 months classified as a case of measles intestinal parasite?
is given Retinol 100,000 IU; a child is given 200,000 IU A. Ascaris
regardless of when the last dose was given. B. Pinworm
C. Hookworm
112. A mother brought her 10 month old infant for D. Schistosoma
consultation because of fever, which started 4 days prior
to consultation. To determine malaria risk, what will you Answer: (B) Pinworm
do? Pinworm ova are deposited around the anal orifice.
A. Do a tourniquet test.
B. Ask where the family resides. 116. Which of the following signs indicates the need for
C. Get a specimen for blood smear. sputum examination for AFB?
D. Ask if the fever is present everyday. A. Hematemesis
B. Fever for 1 week
Answer: (B) Ask where the family resides. C. Cough for 3 weeks
Because malaria is endemic, the first question to D. Chest pain for 1 week
determine malaria risk is where the client’s family
resides. If the area of residence is not a known endemic Answer: (C) Cough for 3 weeks
area, ask if the child had traveled within the past 6 A client is considered a PTB suspect when he has cough
months, where he/she was brought and whether he/she for 2 weeks or more, plus one or more of the following
stayed overnight in that area. signs: fever for 1 month or more; chest pain lasting for
2
47
47
weeks or more not attributed to other conditions; 120. Which of the following clients should be
progressive, unexplained weight loss; night sweats; and classified as a case of multibacillary leprosy?
hemoptysis. A. 3 skin lesions, negative slit skin smear
B. 3 skin lesions, positive slit skin smear
117. Which clients are considered targets for DOTS C. 5 skin lesions, negative slit skin smear
Category I? D. 5 skin lesions, positive slit skin smear
A. Sputum negative cavitary cases
B. Clients returning after a default Answer: (D) 5 skin lesions, positive slit skin smear
C. Relapses and failures of previous PTB treatment A multibacillary leprosy case is one who has a positive
regimens slit skin smear and at least 5 skin lesions.
D. Clients diagnosed for the first time through a positive
sputum exam 121. In the Philippines, which condition is the most
frequent cause of death associated with schistosomiasis?
Answer: (D) Clients diagnosed for the first time through A. Liver cancer
a positive sputum exam B. Liver cirrhosis
Category I is for new clients diagnosed by sputum C. Bladder cancer
examination and clients diagnosed to have a serious D. Intestinal perforation
form of extrapulmonary tuberculosis, such as TB
osteomyelitis. Answer: (B) Liver cirrhosis
The etiologic agent of schistosomiasis in the Philippines
118. To improve compliance to treatment, what is Schistosoma japonicum, which affects the small
innovation is being implemented in DOTS? intestine and the liver. Liver damage is a consequence of
A. Having the health worker follow up the client at home fibrotic reactions to schistosoma eggs in the liver.
B. Having the health worker or a responsible family
member monitor drug intake 122. What is the most effective way of controlling
C. Having the patient come to the health center every schistosomiasis in an endemic area?
month to get his medications A. Use of molluscicides
D. Having a target list to check on whether the B. Building of foot bridges
patient has collected his monthly supply of drugs C. Proper use of sanitary toilets
D. Use of protective footwear, such as rubber boots
Answer: (B) Having the health worker or a responsible
family member monitor drug intake Answer: (C) Proper use of sanitary toilets
Directly Observed Treatment Short Course is so-called The ova of the parasite get out of the human body
because a treatment partner, preferably a health worker together with feces. Cutting the cycle at this stage is the
accessible to the client, monitors the client’s compliance most effective way of preventing the spread of the
to the treatment. disease to susceptible hosts.

119. Diagnosis of leprosy is highly dependent on 123. When residents obtain water from an artesian
recognition of symptoms. Which of the following is an well in the neighborhood, the level of this approved
early sign of leprosy? type of water facility is
A. Macular lesions A. I
B. Inability to close eyelids B. II
C. Thickened painful nerves C. III
D. Sinking of the nosebridge D. IV

Answer: (C) Thickened painful nerves Answer: (B) II


The lesion of leprosy is not macular. It is characterized by A communal faucet or water standpost is classified as
a change in skin color (either reddish or whitish) and loss Level II.
of sensation, sweating and hair growth over the lesion.
Inability to close the eyelids (lagophthalmos) and sinking 124. For prevention of hepatitis A, you decided to
of the nosebridge are late symptoms. conduct health education activities. Which of the
following is IRRELEVANT?

47
47
A. Use of sterile syringes and needles Oresol/orem. When the foregoing measures are not
B. Safe food preparation and food handling by vendors possible or effective, tehn urgent referral to the hospital
C. Proper disposal of human excreta and personal is done.
hygiene
D. Immediate reporting of water pipe leaks and illegal 129. A client was diagnosed as having Dengue fever.
water connections You will say that there is slow capillary refill when the
color of the nailbed that you pressed does not return
Answer: (A) Use of sterile syringes and needles within how many seconds?
Hepatitis A is transmitted through the fecal oral route. A. 3
Hepatitis B is transmitted through infected body B. 5
secretions like blood and semen. C. 8
D. 10
126. Which biological used in Expanded Program on
Immunization (EPI) should NOT be stored in the freezer? Answer: (A) 3
A. DPT Adequate blood supply to the area allows the return of
B. Oral polio vaccine the color of the nailbed within 3 seconds.
C. Measles vaccine
D. MMR 130. A 3-year old child was brought by his mother to the
health center because of fever of 4-day duration. The
Answer: (A) DPT child had a positive tourniquet test result. In the absence
DPT is sensitive to freezing. The appropriate storage of other signs, which is the most appropriate measure
temperature of DPT is 2 to 8° C only. OPV and measles that the PHN may carry out to prevent Dengue shock
vaccine are highly sensitive to heat and require freezing. syndrome?
MMR is not an immunization in the Expanded Program A. Insert an NGT and give fluids per NGT.
on Immunization. B. Instruct the mother to give the child Oresol.
C. Start the patient on intravenous fluids STAT.
127. You will conduct outreach immunization in a D. Refer the client to the physician for
barangay with a population of about 1500. Estimate the appropriate management.
number of infants in the barangay.
A. 45 Answer: (B) Instruct the mother to give the child
B. 50 Oresol.
C. 55 Since the child does not manifest any other danger sign,
D. 60 maintenance of fluid balance and replacement of fluid
loss may be done by giving the client Oresol.
Answer: (A) 45
To estimate the number of infants, multiply total 131. The pathognomonic sign of measles is Koplik’s spot.
population by 3%. You may see Koplik’s spot by inspecting the .
A. Nasal mucosa
128. In Integrated Management of Childhood Illness, B. Buccal mucosa
severe conditions generally require urgent referral to a C. Skin on the abdomen
hospital. Which of the following severe conditions DOES D. Skin on the antecubital surface
NOT always require urgent referral to a hospital?
A. Mastoiditis Answer: (B) Buccal mucosa
B. Severe dehydration Koplik’s spot may be seen on the mucosa of the mouth
C. Severe pneumonia or the throat.
D. Severe febrile disease
132. Among the following diseases, which is airborne?
Answer: (B) Severe dehydration A. Viral conjunctivitis
The order of priority in the management of severe B. Acute poliomyelitis
dehydration is as follows: intravenous fluid therapy, C. Diphtheria
referral to a facility where IV fluids can be initiated D. Measles
within 30 minutes, Oresol/nasogastric tube,

47
47
Answer: (D) Measles 136. Mosquito-borne diseases are prevented mostly
Viral conjunctivitis is transmitted by direct or indirect with the use of mosquito control measures. Which of the
contact with discharges from infected eyes. Acute following is NOT appropriate for malaria control?
poliomyelitis is spread through the fecal-oral route and A. Use of chemically treated mosquito nets
contact with throat secretions, whereas diphtheria is B. Seeding of breeding places with larva-eating fish
through direct and indirect contact with respiratory C. Destruction of breeding places of the mosquito vector
secretions. D. Use of mosquito-repelling soaps, such as those with
basil or citronella
133. Among children aged 2 months to 3 years, the most
prevalent form of meningitis is caused by which Answer: (C) Destruction of breeding places of the
microorganism? mosquito vector
A. Hemophilus influenzae Anopheles mosquitoes breed in slow-moving, clear
B. Morbillivirus water, such as mountain streams.
C. Steptococcus pneumoniae
D. Neisseria meningitidis 137. A 4-year old client was brought to the health center
with the chief complaint of severe diarrhea and the
Answer: (A) Hemophilus influenzae passage of “rice water” stools. The client is most
Hemophilus meningitis is unusual over the age of 5 probably suffering from which condition?
years. In developing countries, the peak incidence is in A. Giardiasis
children less than 6 months of age. Morbillivirus is the B. Cholera
etiology of measles. Streptococcus pneumoniae and C. Amebiasis
Neisseria meningitidis may cause meningitis, but age D. Dysentery
distribution is not specific in young children.
Answer: (B) Cholera
134. Human beings are the major reservoir of malaria. Passage of profuse watery stools is the major symptom
Which of the following strategies in malaria control is of cholera. Both amebic and bacillary dysentery are
based on this fact? characterized by the presence of blood and/or mucus in
A. Stream seeding the stools. Giardiasis is characterized by fat
B. Stream clearing malabsorption and, therefore, steatorrhea.
C. Destruction of breeding places
D. Zooprophylaxis 138. In the Philippines, which specie of schistosoma is
endemic in certain regions?
Answer: (D) Zooprophylaxis A. S. mansoni
Zooprophylaxis is done by putting animals like cattle or B. S. japonicum
dogs close to windows or doorways just before nightfall. C. S. malayensis
The Anopheles mosquito takes his blood meal from the D. S. haematobium
animal and goes back to its breeding place, thereby
preventing infection of humans. Answer: (B) S. japonicum
S. mansoni is found mostly in Africa and South America;
135. The use of larvivorous fish in malaria control is S. haematobium in Africa and the Middle East; and S.
the basis for which strategy of malaria control? malayensis only in peninsular Malaysia.
A. Stream seeding
B. Stream clearing 139. A 32-year old client came for consultation at the
C. Destruction of breeding places health center with the chief complaint of fever for a
D. Zooprophylaxis week. Accompanying symptoms were muscle pains and
body malaise. A week after the start of fever, the client
Answer: (A) Stream seeding noted yellowish discoloration of his sclera. History
Stream seeding is done by putting tilapia fry in streams showed that he waded in flood waters about 2 weeks
or other bodies of water identified as breeding places of before the onset of symptoms. Based on his history,
the Anopheles mosquito which disease condition will you suspect?
A. Hepatitis A
B. Hepatitis B

47
47
C. Tetanus Transmission occurs mostly through sexual intercourse
D. Leptospirosis and exposure to blood or tissues.

Answer: (D) Leptospirosis 143. The most frequent causes of death among
Leptospirosis is transmitted through contact with the clients with AIDS are opportunistic diseases. Which of
skin or mucous membrane with water or moist soil the following opportunistic infections is characterized
contaminated with urine of infected animals, like rats. by tonsillopharyngitis?
A. Respiratory candidiasis
140. MWSS provides water to Manila and other cities in B. Infectious mononucleosis
Metro Manila. This is an example of which level of water C. Cytomegalovirus disease
facility? D. Pneumocystis carinii pneumonia
A. I
B. II Answer: (B) Infectious mononucleosis
C. III Cytomegalovirus disease is an acute viral disease
D. IV characterized by fever, sore throat and
lymphadenopathy.
Answer: (C) III
Waterworks systems, such as MWSS, are classified as 144. To determine possible sources of sexually
level III. transmitted infections, which is the BEST method that
may be undertaken by the public health nurse?
141. You are the PHN in the city health center. A client A. Contact tracing
underwent screening for AIDS using ELISA. His result was B. Community survey
positive. What is the best course of action that you may C. Mass screening tests
take? D. Interview of suspects
A. Get a thorough history of the client, focusing on the
practice of high risk behaviors. Answer: (A) Contact tracing
B. Ask the client to be accompanied by a significant Contact tracing is the most practical and reliable method
person before revealing the result. of finding possible sources of person-to-person
C. Refer the client to the physician since he is the transmitted infections, such as sexually transmitted
best person to reveal the result to the client. diseases.
D. Refer the client for a supplementary test, such as
Western blot, since the ELISA result may be false. 145. Antiretroviral agents, such as AZT, are used in the
management of AIDS. Which of the following is NOT an
Answer: (D) Refer the client for a supplementary test, action expected of these drugs.
such as Western blot, since the ELISA result may be A. They prolong the life of the client with AIDS.
false. B. They reduce the risk of opportunistic infections
A client having a reactive ELISA result must undergo a C. They shorten the period of communicability of the
more specific test, such as Western blot. A negative disease.
supplementary test result means that the ELISA result D. They are able to bring about a cure of the
was false and that, most probably, the client is not disease condition.
infected.
Answer: (D) They are able to bring about a cure of
142. Which is the BEST control measure for AIDS? the disease condition.
A. Being faithful to a single sexual partner There is no known treatment for AIDS. Antiretroviral
B. Using a condom during each sexual contact agents reduce the risk of opportunistic infections and
C. Avoiding sexual contact with commercial sex workers prolong life, but does not cure the underlying
D. Making sure that one’s sexual partner does not immunodeficiency.
have signs of AIDS
146. A barangay had an outbreak of German measles.
Answer: (A) Being faithful to a single sexual partner To prevent congenital rubella, what is the BEST advice
Sexual fidelity rules out the possibility of getting the that you can give to women in the first trimester of
disease by sexual contact with another infected person. pregnancy in the barangay?

47
47
A. Advice them on the signs of German measles. children. Complications, such as pneumonia, are higher
B. Avoid crowded places, such as markets and in incidence in adults.
moviehouses.
C. Consult at the health center where rubella vaccine 149. Complications to infectious parotitis (mumps) may
may be given. be serious in which type of clients?
D. Consult a physician who may give them rubella A. Pregnant women
immunoglobulin. B. Elderly clients
C. Young adult males
Answer: (D) Consult a physician who may give them D. Young infants
rubella immunoglobulin.
Rubella vaccine is made up of attenuated German Answer: (C) Young adult males
measles viruses. This is contraindicated in pregnancy. Epididymitis and orchitis are possible complications of
Immune globulin, a specific prophylactic against German mumps. In post-adolescent males, bilateral inflammation
measles, may be given to pregnant women. of the testes and epididymis may cause sterility.

147. You were invited to be the resource person in a


training class for food handlers. Which of the following
would you emphasize regarding prevention of
staphylococcal food poisoning?
A. All cooking and eating utensils must be thoroughly
washed.
B. Food must be cooked properly to
destroy staphylococcal microorganisms.
C. Food handlers and food servers must have a negative
stool examination result.
D. Proper handwashing during food preparation is the
best way of preventing the condition.

Answer: (D) Proper handwashing during food


preparation is the best way of preventing the
condition.
Symptoms of this food poisoning are due to
staphylococcal enterotoxin, not the microorganisms
themselves. Contamination is by food handling by
persons with staphylococcal skin or eye infections.

148. In a mothers’ class, you discussed childhood


diseases such as chicken pox. Which of the following
statements about chicken pox is correct?
A. The older one gets, the more susceptible he becomes
to the complications of chicken pox.
B. A single attack of chicken pox will prevent future
episodes, including conditions such as shingles.
C. To prevent an outbreak in the community, quarantine
may be imposed by health authorities.
D. Chicken pox vaccine is best given when there is an
impending outbreak in the community.

Answer: (A) The older one gets, the more susceptible


he becomes to the complications of chicken pox.
Chicken pox is usually more severe in adults than in

47
47
MEDICAL SURGICAL NURSING Part 1 8. Dr. Marquez orders a continuous intravenous
nitroglycerin infusion for the client suffering from
1. Mrs. Chua a 78 year old client is admitted with the myocardial infarction. Which of the following is the most
diagnosis of mild chronic heart failure. The nurse expects essential nursing action?
to hear when listening to client’s lungs indicative of a. Monitoring urine output frequently
chronic heart failure would be: b. Monitoring blood pressure every 4 hours
a. Stridor c. Obtaining serum potassium levels daily
b. Crackles d. Obtaining infusion pump for the medication
c. Wheezes 9. During the second day of hospitalization of the client
d. Friction rubs after a Myocardial Infarction. Which of the following is
2. Patrick who is hospitalized following a myocardial an expected outcome?
infarction asks the nurse why he is taking morphine. The a. Able to perform self-care activities without pain
nurse explains that morphine: b. Severe chest pain
a. Decrease anxiety and restlessness c. Can recognize the risk factors of Myocardial Infarction
b. Prevents shock and relieves pain d. Can Participate in cardiac rehabilitation walking
c. Dilates coronary blood vessels program
d. Helps prevent fibrillation of the heart 10. A 68 year old client is diagnosed with a right-sided
3. Which of the following should the nurse teach the brain attack and is admitted to the hospital. In caring for
client about the signs of digitalis toxicity? this client, the nurse should plan to:
a. Increased appetite a. Application of elastic stockings to prevent flaccid by
b. Elevated blood pressure muscle
c. Skin rash over the chest and back b. Use hand roll and extend the left upper extremity on a
d. Visual disturbances such as seeing yellow spots pillow to prevent contractions
4. Nurse Trisha teaches a client with heart failure to take c. Use a bed cradle to prevent dorsiflexion of feet
oral Furosemide in the morning. The reason for this is to d. Do passive range of motion exercise
help… 11. Nurse Liza is assigned to care for a client who has
a. Retard rapid drug absorption returned to the nursing unit after left nephrectomy.
b. Excrete excessive fluids accumulated at night Nurse Liza’s highest priority would be…
c. Prevents sleep disturbances during night a. Hourly urine output
d. Prevention of electrolyte imbalance b. Temperature
5. What would be the primary goal of therapy for a c. Able to turn side to side
client with pulmonary edema and heart failure? d. Able to sips clear liquid
a. Enhance comfort 12. A 64 year old male client with a long history of
b. Increase cardiac output cardiovascular problem including hypertension and
c. Improve respiratory status angina is to be scheduled for cardiac catheterization.
d. Peripheral edema decreased During pre cardiac catheterization teaching, Nurse
6. Nurse Linda is caring for a client with head injury and Cherry should inform the client that the primary purpose
monitoring the client with decerebrate posturing. of the procedure is…..
Which of the following is a characteristic of this type of a. To determine the existence of CHD
posturing? b. To visualize the disease process in the coronary
a. Upper extremity flexion with lower extremity flexion arteries
b. Upper extremity flexion with lower extremity c. To obtain the heart chambers pressure
extension d. To measure oxygen content of different heart
c. Extension of the extremities after a stimulus chambers
d. Flexion of the extremities after stimulus 13. During the first several hours after a cardiac
7. A female client is taking Cascara Sagrada. Nurse Betty catheterization, it would be most essential for nurse
informs the client that the following maybe experienced Cherry to…
as side effects of this medication: a. Elevate clients bed at 45°
a. GI bleeding b. Instruct the client to cough and deep breathe every
b. Peptic ulcer disease 2 hours
c. Abdominal cramps c. Frequently monitor client’s apical pulse and blood
d. Partial bowel obstruction pressure

47
47
d. Monitor clients temperature every hour male client will go home with a prescription for which
14. Kate who has undergone mitral valve replacement medication?
suddenly experiences continuous bleeding from the a. Paracetamol
surgical incision during postoperative period. Which of b. Ibuprofen
the following pharmaceutical agents should Nurse Aiza c. Nitroglycerin
prepare to administer to Kate? d. Nicotine (Nicotrol)
a. Protamine Sulfate 21. Nurse Lilly has been assigned to a client with
b. Quinidine Sulfate Raynaud’s disease. Nurse Lilly realizes that the etiology
c. Vitamin C of the disease is unknown but it is characterized by:
d. Coumadin a. Episodic vasospastic disorder of capillaries
15. In reducing the risk of endocarditis, good dental care b. Episodic vasospastic disorder of small veins
is an important measure. To promote good dental care c. Episodic vasospastic disorder of the aorta
in client with mitral stenosis in teaching plan should d. Episodic vasospastic disorder of the small arteries
include proper use of… 22. Nurse Jamie should explain to male client with
a. Dental floss diabetes that self-monitoring of blood glucose is
b. Electric toothbrush preferred to urine glucose testing because…
c. Manual toothbrush a. More accurate
d. Irrigation device b. Can be done by the client
16. Among the following signs and symptoms, which c. It is easy to perform
would most likely be present in a client with mitral d. It is not influenced by drugs
gurgitation? 23. Jessie weighed 210 pounds on admission to the
a. Altered level of consciousness hospital. After 2 days of diuretic therapy, Jessie weighs
b. Exceptional Dyspnea 205.5 pounds. The nurse could estimate the amount of
c. Increase creatine phospholinase concentration fluid Jessie has lost…
d. Chest pain a. 0.3 L
17. Kris with a history of chronic infection of the urinary b. 1.5 L
system complains of urinary frequency and burning c. 2.0 L
sensation. To figure out whether the current problem is d. 3.5 L
in renal origin, the nurse should assess whether the 24. Nurse Donna is aware that the shift of body fluids
client has discomfort or pain in the… associated with Intravenous administration of albumin
a. Urinary meatus occurs in the process of:
b. Pain in the Labium a. Osmosis
c. Suprapubic area b. Diffusion
d. Right or left costovertebral angle c. Active transport
18. Nurse Perry is evaluating the renal function of a d. Filtration
male client. After documenting urine volume and 25. Myrna a 52 year old client with a fractured left tibia
characteristics, Nurse Perry assesses which signs as the has a long leg cast and she is using crutches to ambulate.
best indicator of renal function. Nurse Joy assesses for which sign and symptom that
a. Blood pressure indicates complication associated with crutch walking?
b. Consciousness a. Left leg discomfort
c. Distension of the bladder b. Weak biceps brachii
d. Pulse rate c. Triceps muscle spasm
19. John suddenly experiences a seizure, and Nurse Gina d. Forearm weakness
notice that John exhibits uncontrollable jerking 26. Which of the following statements should the nurse
movements. Nurse Gina documents that John teach the neutropenic client and his family to avoid?
experienced which type of seizure? a. Performing oral hygiene after every meal
a. Tonic seizure b. Using suppositories or enemas
b. Absence seizure c. Performing perineal hygiene after each bowel
c. Myoclonic seizure movement
d. Clonic seizure d. Using a filter mask
20. Smoking cessation is critical strategy for the client 27. A female client is experiencing painful and rigid
with Burgher’s disease, Nurse Jasmin anticipates that the abdomen and is diagnosed with perforated peptic ulcer.

47
47
A surgery has been scheduled and a nasogastric tube is 34. Nurse Lucy is planning to give pre operative
inserted. The nurse should place the client before teaching to a client who will be undergoing rhinoplasty.
surgery in Which of the following should be included?
a. Sims position a. Results of the surgery will be immediately noticeable
b. Supine position postoperatively
c. Semi-fowlers position b. Normal saline nose drops will need to be administered
d. Dorsal recumbent position preoperatively
28. Which nursing intervention ensures adequate c. After surgery, nasal packing will be in place 8 to 10
ventilating exchange after surgery? days
a. Remove the airway only when client is fully conscious d. Aspirin containing medications should not be taken 14
b. Assess for hypoventilation by auscultating the lungs days before surgery
c. Position client laterally with the neck extended 35. Paul is admitted to the hospital due to metabolic
d. Maintain humidified oxygen via nasal canula acidosis caused by Diabetic ketoacidosis (DKA). The
29. George who has undergone thoracic surgery has nurse prepares which of the following medications as an
chest tube connected to a water-seal drainage system initial treatment for this problem?
attached to suction. Presence of excessive bubbling is a. Regular insulin
identified in water-seal chamber, the nurse should… b. Potassium
a. “Strip” the chest tube catheter c. Sodium bicarbonate
b. Check the system for air leaks d. Calcium gluconate
c. Recognize the system is functioning correctly 36. Dr. Marquez tells a client that an increase intake of
d. Decrease the amount of suction pressure foods that are rich in Vitamin E and beta-carotene are
30. A client who has been diagnosed of hypertension is important for healthier skin. The nurse teaches the
being taught to restrict intake of sodium. The nurse client that excellent food sources of both of these
would know that the teachings are effective if the client substances are:
states that… a. Fish and fruit jam
a. I can eat celery sticks and carrots b. Oranges and grapefruit
b. I can eat broiled scallops c. Carrots and potatoes
c. I can eat shredded wheat cereal d. Spinach and mangoes
d. I can eat spaghetti on rye bread 37. A client has Gastroesophageal Reflux Disease (GERD).
31. A male client with a history of cirrhosis and The nurse should teach the client that after every meals,
alcoholism is admitted with severe dyspnea resulted to the client should…
ascites. The nurse should be aware that the ascites is a. Rest in sitting position
most likely the result of increased… b. Take a short walk
a. Pressure in the portal vein c. Drink plenty of water
b. Production of serum albumin d. Lie down at least 30 minutes
c. Secretion of bile salts 38. After gastroscopy, an adaptation that indicates major
d. Interstitial osmotic pressure complication would be:
32. A newly admitted client is diagnosed with Hodgkin’s a. Nausea and vomiting
disease undergoes an excisional cervical lymph node b. Abdominal distention
biopsy under local anesthesia. What does the nurse c. Increased GI motility
assess first after the procedure? d. Difficulty in swallowing
a. Vital signs 39. A client who has undergone a cholecystectomy asks
b. Incision site the nurse whether there are any dietary restrictions that
c. Airway must be followed. Nurse Hilary would recognize that the
d. Level of consciousness dietary teaching was well understood when the client
33. A client has 15% blood loss. Which of the following tells a family member that:
nursing assessment findings indicates hypovolemic a. “Most people need to eat a high protein diet for
shock? 12 months after surgery”
a. Systolic blood pressure less than 90mm Hg b. “I should not eat those foods that upset me before the
b. Pupils unequally dilated surgery”
c. Respiratory rate of 4 breath/min c. “I should avoid fatty foods as long as I live”
d. Pulse rate less than 60bpm

47
47
d. “Most people can tolerate regular diet after this type 46. Nurse Jenny is instilling an otic solution into an adult
of surgery” male client left ear. Nurse Jenny avoids doing which of
40. Nurse Rachel teaches a client who has been recently the following as part of the procedure
diagnosed with hepatitis A about untoward signs and a. Pulling the auricle backward and upward
symptoms related to Hepatitis that may develop. The b. Warming the solution to room temperature
one that should be reported immediately to the c. Pacing the tip of the dropper on the edge of ear canal
physician is: d. Placing client in side lying position
a. Restlessness 47. Nurse Bea should instruct the male client with an
b. Yellow urine ileostomy to report immediately which of the following
c. Nausea symptom?
d. Clay- colored stools a. Absence of drainage from the ileostomy for 6 or more
41. Which of the following antituberculosis drugs can hours
damage the 8th cranial nerve? b. Passage of liquid stool in the stoma
a. Isoniazid (INH) c. Occasional presence of undigested food
b. Paraoaminosalicylic acid (PAS) d. A temperature of 37.6 °C
c. Ethambutol hydrochloride (myambutol) 48. Jerry has diagnosed with appendicitis. He develops a
d. Streptomycin fever, hypotension and tachycardia. The nurse suspects
42. The client asks Nurse Annie the causes of peptic which of the following complications?
ulcer. Nurse Annie responds that recent research a. Intestinal obstruction
indicates that peptic ulcers are the result of which of the b. Peritonitis
following: c. Bowel ischemia
a. Genetic defect in gastric mucosa d. Deficient fluid volume
b. Stress 49. Which of the following compilations should the nurse
c. Diet high in fat carefully monitors a client with acute pancreatitis.
d. Helicobacter pylori infection a. Myocardial Infarction
43. Ryan has undergone subtotal gastrectomy. The nurse b. Cirrhosis
should expect that nasogastric tube drainage will be c. Peptic ulcer
what color for about 12 to 24 hours after surgery? d. Pneumonia
a. Bile green 50. Which of the following symptoms during the
b. Bright red icteric phase of viral hepatitis should the nurse expect
c. Cloudy white the client to inhibit?
d. Dark brown a. Watery stool
44. Nurse Joan is assigned to come for client who has b. Yellow sclera
just undergone eye surgery. Nurse Joan plans to teach c. Tarry stool
the client activities that are permitted during the post d. Shortness of breath
operative period. Which of the following is best
recommended for the client?
a. Watching circus
b. Bending over
c. Watching TV
d. Lifting objects
45. A client suffered from a lower leg injury and seeks
treatment in the emergency room. There is a prominent
deformity to the lower aspect of the leg, and the injured
leg appears shorter that the other leg. The affected leg is
painful, swollen and beginning to become ecchymotic.
The nurse interprets that the client is experiencing:
a. Fracture
b. Strain
c. Sprain
d. Contusion

47
48
ANSWERS and RATIONALES for MEDICAL gums, allowing bacteria to enter and increasing
SURGICAL NURSING Part 1 the risk of endocarditis.
16. B. Weight gain due to retention of fluids and
1. B. Left sided heart failure causes fluid worsening heart failure causes exertional
accumulation in the capillary network of the dyspnea in clients with mitral regurgitation.
lung. Fluid eventually enters alveolar spaces and 17. D. Discomfort or pain is a problem that
causes crackling sounds at the end of inspiration. originates in the kidney. It is felt at the
2. B. Morphine is a central nervous system costovertebral angle on the affected side.
depressant used to relieve the pain associated 18. A. Perfusion can be best estimated by blood
with myocardial infarction, it also decreases pressure, which is an indirect reflection of the
apprehension and prevents cardiogenic shock. adequacy of cardiac output.
3. D. Seeing yellow spots and colored vision are 19. C. Myoclonic seizure is characterized by sudden
common symptoms of digitalis toxicity uncontrollable jerking movements of a single or
4. C. When diuretics are taken in the morning, multiple muscle group.
client will void frequently during daytime and 20. D. Nicotine (Nicotrol) is given in controlled and
will not need to void frequently at night. decreasing doses for the management of
5. B. The primary goal of therapy for the client with nicotine withdrawal syndrome.
pulmonary edema or heart failure is increasing 21. D. Raynaud’s disease is characterized by
cardiac output. Pulmonary edema is an acute vasospasms of the small cutaneous arteries that
medical emergency requiring immediate involves fingers and toes.
intervention. 22. A. Urine testing provides an indirect measure
6. C. Decerebrate posturing is the extension of the that maybe influenced by kidney function while
extremities after a stimulus, which may occur blood glucose testing is a more direct and
with upper brain stem injury. accurate measure.
7. C. The most frequent side effects of Cascara 23. C. One liter of fluid approximately weighs 2.2
Sagrada (Laxative) is abdominal cramps and pounds. A 4.5 pound weight loss equals to
nausea. approximately 2L.
8. D. Administration of Intravenous Nitroglycerin 24. A. Osmosis is the movement of fluid from an
infusion requires pump for accurate control of area of lesser solute concentration to an area of
medication. greater solute concentration.
9. A. By the 2nd day of hospitalization after 25. D. Forearm muscle weakness is a probable sign
suffering a Myocardial Infarction, Clients are of radial nerve injury caused by crutch pressure
able to perform care without chest pain on the axillae.
10. B. The left side of the body will be affected in a 26. B. Neutropenic client is at risk for infection
right-sided brain attack. especially bacterial infection of the
11. A. After nephrectomy, it is necessary to measure gastrointestinal and respiratory tract.
urine output hourly. This is done to assess the 27. C. Semi-fowlers position will localize the spilled
effectiveness of the remaining kidney also to stomach contents in the lower part of the
detect renal failure early. abdominal cavity.
12. B. The lumen of the arteries can be assessed by 28. C. Positioning the client laterally with the neck
cardiac catheterization. Angina is usually caused extended does not obstruct the airway so that
by narrowing of the coronary arteries. drainage of secretions and oxygen and carbon
13. C. Blood pressure is monitored to detect dioxide exchange can occur.
hypotension which may indicate shock or 29. B. Excessive bubbling indicates an air leak which
hemorrhage. Apical pulse is taken to detect must be eliminated to permit lung expansion.
dysrhythmias related to cardiac irritability. 30. C. Wheat cereal has a low sodium content.
14. A. Protamine Sulfate is used to prevent 31. A. Enlarged cirrhotic liver impinges the portal
continuous bleeding in client who has system causing increased hydrostatic pressure
undergone open heart surgery. resulting to ascites.
15. C. The use of electronic toothbrush, irrigation 32. C. Assessing for an open airway is the priority.
device or dental floss may cause bleeding of The procedure involves the neck, the anesthesia
may have affected the swallowing reflex or the

48
48
inflammation may have closed in on the airway 49. D. A client with acute pancreatitis is prone to
leading to ineffective air exchange. complications associated with respiratory
33. A. Typical signs and symptoms of hypovolemic system.
shock includes systolic blood pressure of less 50. B. Liver inflammation and obstruction block the
than 90 mm Hg. normal flow of bile. Excess bilirubin turns the
34. D. Aspirin containing medications should not be skin and sclera yellow and the urine dark and
taken 14 days before surgery to decrease the frothy.
risk of bleeding.
35. A. Metabolic acidosis is anaerobic metabolism
caused by lack of ability of the body to use
circulating glucose. Administration of insulin
corrects this problem.
36. D. Beta-carotene and Vitamin E are antioxidants
which help to inhibit oxidation. Vitamin E is
found in the following foods: wheat germ, corn,
nuts, seeds, olives, spinach, asparagus and other
green leafy vegetables. Food sources of beta-
carotene include dark green vegetables, carrots,
mangoes and tomatoes.
37. A. Gravity speeds up digestion and prevents
reflux of stomach contents into the esophagus.
38. B. Abdominal distension may be associated with
pain, may indicate perforation, a complication
that could lead to peritonitis.
39. D. It may take 4 to 6 months to eat anything, but
most people can eat anything they want.
40. D. Clay colored stools are indicative of hepatic
obstruction
41. D. Streptomycin is an aminoglycoside and
damage on the 8th cranial nerve (ototoxicity) is a
common side effect of aminoglycosides.
42. D. Most peptic ulcer is caused by Helicopter
pylori which is a gram negative bacterium.
43. D. 12 to 24 hours after subtotal gastrectomy
gastric drainage is normally brown, which
indicates digested food.
44. C. Watching TV is permissible because the eye
does not need to move rapidly with this activity,
and it does not increase intraocular pressure.
45. A. Common signs and symptoms of fracture
include pain, deformity, shortening of the
extremity, crepitus and swelling.
46. C. The dropper should not touch any object or
any part of the client’s ear.
47. A. Sudden decrease in drainage or onset of
severe abdominal pain should be reported
immediately to the physician because it could
mean that obstruction has been developed.
48. B. Complications of acute appendicitis are
peritonitis, perforation and abscess
development.

48
48
MEDICAL SURGICAL NURSING Part 2 c. Protects the client’s head from injury
d. Attempt to insert a tongue depressor between the
1. A client is scheduled for insertion of an inferior vena client’s teeth
cava (IVC) filter. Nurse Patricia consults the physician 8. A client has undergone right pneumonectomy. When
about withholding which regularly scheduled medication turning the client, the nurse should plan to position the
on the day before the surgery? client either:
a. Potassium Chloride a. Right side-lying position or supine
b. Warfarin Sodium b. High fowlers
c. Furosemide c. Right or left side lying position
d. Docusate d. Low fowler’s position
2. A nurse is planning to assess the corneal reflex on 9. Nurse Jenny should caution a female client who is
unconscious client. Which of the following is the safest sexually active in taking Isoniazid (INH) because the drug
stimulus to touch the client’s cornea? has which of the following side effects?
a. Cotton buds a. Prevents ovulation
b. Sterile glove b. Has a mutagenic effect on ova
c. Sterile tongue depressor c. Decreases the effectiveness of oral contraceptives
d. Wisp of cotton d. Increases the risk of vaginal infection
3. A female client develops an infection at the catheter 10. A client has undergone gastrectomy. Nurse Jovy
insertion site. The nurse in charge uses the term is aware that the best position for the client is:
“iatrogenic” when describing the infection because it a. Left side lying
resulted from: b. Low fowler’s
a. Client’s developmental level c. Prone
b. Therapeutic procedure d. Supine
c. Poor hygiene 11. During the initial postoperative period of the client’s
d. Inadequate dietary patterns stoma. The nurse evaluates which of the following
4. Nurse Carol is assessing a client with Parkinson’s observations should be reported immediately to the
disease. The nurse recognize bradykinesia when the physician?
client exhibits: a. Stoma is dark red to purple
a. Intentional tremor b. Stoma is oozes a small amount of blood
b. Paralysis of limbs c. Stoma is lightly edematous
c. Muscle spasm d. Stoma does not expel stool
d. Lack of spontaneous movement 12. Kate which has diagnosed with ulcerative colitis is
5. A client who suffered from automobile accident following physician’s order for bed rest with bathroom
complains of seeing frequent flashes of light. The nurse privileges. What is the rationale for this activity
should expect: restriction?
a. Myopia a. Prevent injury
b. Detached retina b. Promote rest and comfort
c. Glaucoma c. Reduce intestinal peristalsis
d. Scleroderma d. Conserve energy
6. Kate with severe head injury is being monitored by the 13. Nurse KC should regularly assess the client’s ability to
nurse for increasing intracranial pressure (ICP). Which metabolize the total parenteral nutrition (TPN) solution
finding should be most indicative sign of increasing adequately by monitoring the client for which of the
intracranial pressure? following signs:
a. Intermittent tachycardia a. Hyperglycemia
b. Polydipsia b. Hypoglycemia
c. Tachypnea c. Hypertension
d. Increased restlessness d. Elevate blood urea nitrogen concentration
7. A hospitalized client had a tonic-clonic seizure while 14. A female client has an acute pancreatitis. Which of
walking in the hall. During the seizure the nurse priority the following signs and symptoms the nurse would
should be: expect to see?
a. Hold the clients arms and leg firmly a. Constipation
b. Place the client immediately to soft surface b. Hypertension

48
48
c. Ascites 22. The nurse is assessing a client with pleural effusion.
d. Jaundice The nurse expect to find:
15. A client is suspected to develop tetany after a a. Deviation of the trachea towards the involved side
subtotal thyroidectomy. Which of the following b. Reduced or absent of breath sounds at the base of
symptoms might indicate tetany? the lung
a. Tingling in the fingers c. Moist crackles at the posterior of the lungs
b. Pain in hands and feet d. Increased resonance with percussion of the
c. Tension on the suture lines involved area
d. Bleeding on the back of the dressing 23. A client admitted with newly diagnosed with
16. A 58 year old woman has newly diagnosed with Hodgkin’s disease. Which of the following would the
hypothyroidism. The nurse is aware that the signs and nurse expect the client to report?
symptoms of hypothyroidism include: a. Lymph node pain
a. Diarrhea b. Weight gain
b. Vomiting c. Night sweats
c. Tachycardia d. Headache
d. Weight gain 24. A client has suffered from fall and sustained a leg
17. A client has undergone for an ileal conduit, the nurse injury. Which appropriate question would the nurse ask
in charge should closely monitor the client for the client to help determine if the injury caused
occurrence of which of the following complications fracture?
related to pelvic surgery? a. “Is the pain sharp and continuous?”
a. Ascites b. “Is the pain dull ache?”
b. Thrombophlebitis c. “Does the discomfort feel like a cramp?”
c. Inguinal hernia d. “Does the pain feel like the muscle was stretched?”
d. Peritonitis 25. The Nurse is assessing the client’s casted extremity
18. Dr. Marquez is about to defibrillate a client in for signs of infection. Which of the following findings is
ventricular fibrillation and says in a loud voice “clear”. indicative of infection?
What should be the action of the nurse? a. Edema
a. Places conductive gel pads for defibrillation on the b. Weak distal pulse
client’s chest c. Coolness of the skin
b. Turn off the mechanical ventilator d. Presence of “hot spot” on the cast
c. Shuts off the client’s IV infusion 26. Nurse Rhia is performing an otoscopic examination
d. Steps away from the bed and make sure all others on a female client with a suspected diagnosis of
have done the same mastoiditis. Nurse Rhia would expect to note which of
19. A client has been diagnosed with the following if this disorder is present?
glomerulonephritis complains of thirst. The nurse a. Transparent tympanic membrane
should offer: b. Thick and immobile tympanic membrane
a. Juice c. Pearly colored tympanic membrane
b. Ginger ale d. Mobile tympanic membrane
c. Milk shake 27. Nurse Jocelyn is caring for a client with
d. Hard candy nasogastric tube that is attached to low suction. Nurse
20. A client with acute renal failure is aware that the Jocelyn assesses the client for symptoms of which
most serious complication of this condition is: acid-base disorder?
a. Constipation a. Respiratory alkalosis
b. Anemia b. Respiratory acidosis
c. Infection c. Metabolic acidosis
d. Platelet dysfunction d. Metabolic alkalosis
21. Nurse Karen is caring for clients in the OR. The nurse 28. A male adult client has undergone a lumbar puncture
is aware that the last physiologic function that the client to obtain cerebrospinal fluid (CSF) for analysis. Which of
loss during the induction of anesthesia is: the following values should be negative if the CSF is
a. Consciousness normal?
b. Gag reflex a. Red blood cells
c. Respiratory movement b. White blood cells
d. Corneal reflex
48
48
c. Insulin 36. A client with peptic ulcer is being assessed by the
d. Protein nurse for gastrointestinal perforation. The nurse should
29. A client is suspected of developing diabetes monitor for:
insipidus. Which of the following is the most effective a. (+) guaiac stool test
assessment? b. Slow, strong pulse
a. Taking vital signs every 4 hours c. Sudden, severe abdominal pain
b. Monitoring blood glucose d. Increased bowel sounds
c. Assessing ABG values every other day 37. A client has undergone surgery for retinal
d. Measuring urine output hourly detachment. Which of the following goal should be
30. A 58 year old client is suffering from acute phase of prioritized?
rheumatoid arthritis. Which of the following would the a. Prevent an increase intraocular pressure
nurse in charge identify as the lowest priority of the plan b. Alleviate pain
of care? c. Maintain darkened room
a. Prevent joint deformity d. Promote low-sodium diet
b. Maintaining usual ways of accomplishing task 38. A Client with glaucoma has been prescribed
c. Relieving pain with miotics. The nurse is aware that miotics is for:
d. Preserving joint function a. Constricting pupil
31. Among the following, which client is autotransfusion b. Relaxing ciliary muscle
possible? c. Constricting intraocular vessel
a. Client with AIDS d. Paralyzing ciliary muscle
b. Client with ruptured bowel 39. When suctioning an unconscious client, which
c. Client who is in danger of cardiac arrest nursing intervention should the nurse prioritize in
d. Client with wound infection maintaining cerebral perfusion?
32. Which of the following is not a sign of a. Administer diuretics
thromboembolism? b. Administer analgesics
a. Edema c. Provide hygiene
b. Swelling d. Hyperoxygenate before and after suctioning
c. Redness 40. When discussing breathing exercises with a
d. Coolness postoperative client, Nurse Hazel should include which
33. Nurse Becky is caring for client who begins to of the following teaching?
experience seizure while in bed. Which action should the a. Short frequent breaths
nurse implement to prevent aspiration? b. Exhale with mouth open
a. Position the client on the side with head flexed c. Exercise twice a day
forward d. Place hand on the abdomen and feel it rise
b. Elevate the head 41. Louie, with burns over 35% of the body, complains of
c. Use tongue depressor between teeth chilling. In promoting the client’s comfort, the nurse
d. Loosen restrictive clothing should:
34. A client has undergone bone biopsy. Which a. Maintain room humidity below 40%
nursing action should the nurse provide after the b. Place top sheet on the client
procedure? c. Limit the occurrence of drafts
a. Administer analgesics via IM d. Keep room temperature at 80 degrees
b. Monitor vital signs 42. Nurse Trish is aware that temporary heterograft (pig
c. Monitor the site for bleeding, swelling and hematoma skin) is used to treat burns because this graft will:
formation a. Relieve pain and promote rapid epithelialization
d. Keep area in neutral position b. Be sutured in place for better adherence
35. A client is suffering from low back pain. Which of the c. Debride necrotic epithelium
following exercises will strengthen the lower back d. Concurrently used with topical antimicrobials
muscle of the client? 43. Mark has multiple abrasions and a laceration to the
a. Tennis trunk and all four extremities says, “I can’t eat all this
b. Basketball food”. The food that the nurse should suggest to be
c. Diving eaten first should be:
d. Swimming a. Meat loaf and coffee

48
48
b. Meat loaf and strawberries
c. Tomato soup and apple pie
d. Tomato soup and buttered bread
44. Tony returns form surgery with permanent
colostomy. During the first 24 hours the colostomy
does not drain. The nurse should be aware that:
a. Proper functioning of nasogastric suction
b. Presurgical decrease in fluid intake
c. Absence of gastrointestinal motility
d. Intestinal edema following surgery
45. When teaching a client about the signs of colorectal
cancer, Nurse Trish stresses that the most common
complaint of persons with colorectal cancer is:
a. Abdominal pain
b. Hemorrhoids
c. Change in caliber of stools
d. Change in bowel habits
46. Louis develops peritonitis and sepsis after surgical
repair of ruptures diverticulum. The nurse in charge
should expect an assessment of the client to reveal:
a. Tachycardia
b. Abdominal rigidity
c. Bradycardia
d. Increased bowel sounds
47. Immediately after liver biopsy, the client is placed on
the right side, the nurse is aware that that this position
should be maintained because it will:
a. Help stop bleeding if any occurs
b. Reduce the fluid trapped in the biliary ducts
c. Position with greatest comfort
d. Promote circulating blood volume
48. Tony has diagnosed with hepatitis A. The information
from the health history that is most likely linked to
hepatitis A is:
a. Exposed with arsenic compounds at work
b. Working as local plumber
c. Working at hemodialysis clinic
d. Dish washer in restaurants
49. Nurse Trish is aware that the laboratory test result
that most likely would indicate acute pancreatitis is an
elevated:
a. Serum bilirubin level
b. Serum amylase level
c. Potassium level
d. Sodium level
50. Dr. Marquez orders serum electrolytes. To
determine the effect of persistent vomiting, Nurse Trish
should be most concerned with monitoring the:
a. Chloride and sodium levels
b. Phosphate and calcium levels
c. Protein and magnesium levels
d. Sulfate and bicarbonate levels

48
48
MEDICAL SURGICAL NURSING Part 2 c. Protects the client’s head from injury
d. Attempt to insert a tongue depressor between
1. A client is scheduled for insertion of an inferior vena the client’s teeth
cava (IVC) filter. Nurse Patricia consults the physician 8. A client has undergone right pneumonectomy. When
about withholding which regularly scheduled turning the client, the nurse should plan to position
medication on the day before the surgery? the client either:
a. Potassium Chloride a. Right side-lying position or supine
b. Warfarin Sodium b. High fowlers
c. Furosemide c. Right or left side lying position
d. Docusate d. Low fowler’s position
2. A nurse is planning to assess the corneal reflex on 9. Nurse Jenny should caution a female client who is
unconscious client. Which of the following is the sexually active in taking Isoniazid (INH) because the
safest stimulus to touch the client’s cornea? drug has which of the following side effects?
a. Cotton buds a. Prevents ovulation
b. Sterile glove b. Has a mutagenic effect on ova
c. Sterile tongue depressor c. Decreases the effectiveness of oral
d. Wisp of cotton contraceptives
3. A female client develops an infection at the catheter d. Increases the risk of vaginal infection
insertion site. The nurse in charge uses the term 10. A client has undergone gastrectomy. Nurse Jovy is
“iatrogenic” when describing the infection because it aware that the best position for the client is:
resulted from: a. Left side lying
a. Client’s developmental level b. Low fowler’s
b. Therapeutic procedure c. Prone
c. Poor hygiene d. Supine
d. Inadequate dietary patterns 11. During the initial postoperative period of the client’s
4. Nurse Carol is assessing a client with Parkinson’s stoma. The nurse evaluates which of the following
disease. The nurse recognize bradykinesia when the observations should be reported immediately to the
client exhibits: physician?
a. Intentional tremor a. Stoma is dark red to purple
b. Paralysis of limbs b. Stoma is oozes a small amount of blood
c. Muscle spasm c. Stoma is lightly edematous
d. Lack of spontaneous movement d. Stoma does not expel stool
5. A client who suffered from automobile accident 12. Kate which has diagnosed with ulcerative colitis is
complains of seeing frequent flashes of light. The following physician’s order for bed rest with
nurse should expect: bathroom privileges. What is the rationale for this
a. Myopia activity restriction?
b. Detached retina a. Prevent injury
c. Glaucoma b. Promote rest and comfort
d. Scleroderma c. Reduce intestinal peristalsis
6. Kate with severe head injury is being monitored by the d. Conserve energy
nurse for increasing intracranial pressure (ICP). 13. Nurse KC should regularly assess the client’s ability to
Which finding should be most indicative sign of metabolize the total parenteral nutrition (TPN)
increasing intracranial pressure? solution adequately by monitoring the client for
a. Intermittent tachycardia which of the following signs:
b. Polydipsia a. Hyperglycemia
c. Tachypnea b. Hypoglycemia
d. Increased restlessness c. Hypertension
7. A hospitalized client had a tonic-clonic seizure while d. Elevate blood urea nitrogen concentration
walking in the hall. During the seizure the nurse 14. A female client has an acute pancreatitis. Which of
priority should be: the following signs and symptoms the nurse would
a. Hold the clients arms and leg firmly expect to see?
b. Place the client immediately to soft surface a. Constipation

48
48
b. Hypertension d. Corneal reflex
c. Ascites 22. The nurse is assessing a client with pleural effusion.
d. Jaundice The nurse expect to find:
15. A client is suspected to develop tetany after a a. Deviation of the trachea towards the involved
subtotal thyroidectomy. Which of the following side
symptoms might indicate tetany? b. Reduced or absent of breath sounds at the
a. Tingling in the fingers base of the lung
b. Pain in hands and feet c. Moist crackles at the posterior of the lungs
c. Tension on the suture lines d. Increased resonance with percussion of the
d. Bleeding on the back of the dressing involved area
16. A 58 year old woman has newly diagnosed with 23. A client admitted with newly diagnosed with
hypothyroidism. The nurse is aware that the signs Hodgkin’s disease. Which of the following would the
and symptoms of hypothyroidism include: nurse expect the client to report?
a. Diarrhea a. Lymph node pain
b. Vomiting b. Weight gain
c. Tachycardia c. Night sweats
d. Weight gain d. Headache
17. A client has undergone for an ileal conduit, the nurse 24. A client has suffered from fall and sustained a leg
in charge should closely monitor the client for injury. Which appropriate question would the nurse
occurrence of which of the following complications ask the client to help determine if the injury caused
related to pelvic surgery? fracture?
a. Ascites a. “Is the pain sharp and continuous?”
b. Thrombophlebitis b. “Is the pain dull ache?”
c. Inguinal hernia c. “Does the discomfort feel like a cramp?”
d. Peritonitis d. “Does the pain feel like the muscle was
18. Dr. Marquez is about to defibrillate a client in stretched?”
ventricular fibrillation and says in a loud voice 25. The Nurse is assessing the client’s casted extremity
“clear”. What should be the action of the nurse? for signs of infection. Which of the following findings
a. Places conductive gel pads for defibrillation on is indicative of infection?
the client’s chest a. Edema
b. Turn off the mechanical ventilator b. Weak distal pulse
c. Shuts off the client’s IV infusion c. Coolness of the skin
d. Steps away from the bed and make sure all d. Presence of “hot spot” on the cast
others have done the same 26. Nurse Rhia is performing an otoscopic examination
19. A client has been diagnosed with glomerulonephritis on a female client with a suspected diagnosis of
complains of thirst. The nurse should offer: mastoiditis. Nurse Rhia would expect to note which
a. Juice of the following if this disorder is present?
b. Ginger ale a. Transparent tympanic membrane
c. Milk shake b. Thick and immobile tympanic membrane
d. Hard candy c. Pearly colored tympanic membrane
20. A client with acute renal failure is aware that the d. Mobile tympanic membrane
most serious complication of this condition is: 27. Nurse Jocelyn is caring for a client with nasogastric
a. Constipation tube that is attached to low suction. Nurse Jocelyn
b. Anemia assesses the client for symptoms of which acid-base
c. Infection disorder?
d. Platelet dysfunction a. Respiratory alkalosis
21. Nurse Karen is caring for clients in the OR. The nurse b. Respiratory acidosis
is aware that the last physiologic function that the c. Metabolic acidosis
client loss during the induction of anesthesia is: d. Metabolic alkalosis
a. Consciousness 28. A male adult client has undergone a lumbar puncture
b. Gag reflex to obtain cerebrospinal fluid (CSF) for analysis.
c. Respiratory movement

48
48
Which of the following values should be negative if b. Basketball
the CSF is normal? c. Diving
a. Red blood cells d. Swimming
b. White blood cells 36. A client with peptic ulcer is being assessed by the
c. Insulin nurse for gastrointestinal perforation. The nurse
d. Protein should monitor for:
29. A client is suspected of developing diabetes a. (+) guaiac stool test
insipidus. Which of the following is the most b. Slow, strong pulse
effective assessment? c. Sudden, severe abdominal pain
a. Taking vital signs every 4 hours d. Increased bowel sounds
b. Monitoring blood glucose 37. A client has undergone surgery for retinal
c. Assessing ABG values every other day detachment. Which of the following goal should be
d. Measuring urine output hourly prioritized?
30. A 58 year old client is suffering from acute phase of a. Prevent an increase intraocular pressure
rheumatoid arthritis. Which of the following would b. Alleviate pain
the nurse in charge identify as the lowest priority of c. Maintain darkened room
the plan of care? d. Promote low-sodium diet
a. Prevent joint deformity 38. A Client with glaucoma has been prescribed with
b. Maintaining usual ways of accomplishing task miotics. The nurse is aware that miotics is for:
c. Relieving pain a. Constricting pupil
d. Preserving joint function b. Relaxing ciliary muscle
31. Among the following, which client is autotransfusion c. Constricting intraocular vessel
possible? d. Paralyzing ciliary muscle
a. Client with AIDS 39. When suctioning an unconscious client, which
b. Client with ruptured bowel nursing intervention should the nurse prioritize in
c. Client who is in danger of cardiac arrest maintaining cerebral perfusion?
d. Client with wound infection a. Administer diuretics
32. Which of the following is not a sign of b. Administer analgesics
thromboembolism? c. Provide hygiene
a. Edema d. Hyperoxygenate before and after suctioning
b. Swelling 40. When discussing breathing exercises with a
c. Redness postoperative client, Nurse Hazel should include
d. Coolness which of the following teaching?
33. Nurse Becky is caring for client who begins to a. Short frequent breaths
experience seizure while in bed. Which action should b. Exhale with mouth open
the nurse implement to prevent aspiration? c. Exercise twice a day
a. Position the client on the side with head d. Place hand on the abdomen and feel it rise
flexed forward 41. Louie, with burns over 35% of the body, complains of
b. Elevate the head chilling. In promoting the client’s comfort, the nurse
c. Use tongue depressor between teeth should:
d. Loosen restrictive clothing a. Maintain room humidity below 40%
34. A client has undergone bone biopsy. Which nursing b. Place top sheet on the client
action should the nurse provide after the procedure? c. Limit the occurrence of drafts
a. Administer analgesics via IM d. Keep room temperature at 80 degrees
b. Monitor vital signs 42. Nurse Trish is aware that temporary heterograft (pig
c. Monitor the site for bleeding, swelling and skin) is used to treat burns because this graft will:
hematoma formation a. Relieve pain and promote rapid
d. Keep area in neutral position epithelialization
35. A client is suffering from low back pain. Which of the b. Be sutured in place for better adherence
following exercises will strengthen the lower back c. Debride necrotic epithelium
muscle of the client? d. Concurrently used with topical antimicrobials
a. Tennis

48
48
43. Mark has multiple abrasions and a laceration to the 50. Dr. Marquez orders serum electrolytes. To determine
trunk and all four extremities says, “I can’t eat all this the effect of persistent vomiting, Nurse Trish should
food”. The food that the nurse should suggest to be be most concerned with monitoring the:
eaten first should be: a. Chloride and sodium levels
a. Meat loaf and coffee b. Phosphate and calcium levels
b. Meat loaf and strawberries c. Protein and magnesium levels
c. Tomato soup and apple pie d. Sulfate and bicarbonate levels
d. Tomato soup and buttered bread
44. Tony returns form surgery with permanent
colostomy. During the first 24 hours the colostomy
does not drain. The nurse should be aware that:
a. Proper functioning of nasogastric suction
b. Presurgical decrease in fluid intake
c. Absence of gastrointestinal motility
d. Intestinal edema following surgery
45. When teaching a client about the signs of colorectal
cancer, Nurse Trish stresses that the most common
complaint of persons with colorectal cancer is:
a. Abdominal pain
b. Hemorrhoids
c. Change in caliber of stools
d. Change in bowel habits
46. Louis develops peritonitis and sepsis after surgical
repair of ruptures diverticulum. The nurse in charge
should expect an assessment of the client to reveal:
a. Tachycardia
b. Abdominal rigidity
c. Bradycardia
d. Increased bowel sounds
47. Immediately after liver biopsy, the client is placed on
the right side, the nurse is aware that that this
position should be maintained because it will:
a. Help stop bleeding if any occurs
b. Reduce the fluid trapped in the biliary ducts
c. Position with greatest comfort
d. Promote circulating blood volume
48. Tony has diagnosed with hepatitis A. The information
from the health history that is most likely linked to
hepatitis A is:
a. Exposed with arsenic compounds at work
b. Working as local plumber
c. Working at hemodialysis clinic
d. Dish washer in restaurants
49. Nurse Trish is aware that the laboratory test result
that most likely would indicate acute pancreatitis is
an elevated:
a. Serum bilirubin level
b. Serum amylase level
c. Potassium level
d. Sodium level

48
49
ANSWERS and RATIONALES for MEDICAL SURGICAL manipulation that can interfere with circulation and
NURSING Part 2 promote venous stasis.
18. D. For the safety of all personnel, if the defibrillator
1. B. In preoperative period, the nurse should consult paddles are being discharged, all personnel must
with the physician about withholding Warfarin stand back and be clear of all the contact with the
Sodium to avoid occurrence of hemorrhage. client or the client’s bed.
2. D. A client who is unconscious is at greater risk for 19. D. Hard candy will relieve thirst and increase
corneal abrasion. For this reason, the safest way to carbohydrates but does not supply extra fluid.
test the cornel reflex is by touching the cornea 20. C. Infection is responsible for one third of the
lightly with a wisp of cotton. traumatic or surgically induced death of clients with
3. B. Iatrogenic infection is caused by the heath care renal failure as well as medical induced acute renal
provider or is induced inadvertently by medical failure (ARF)
treatment or procedures. 21. C. There is no respiratory movement in stage 4 of
4. D. Bradykinesia is slowing down from the initiation anesthesia, prior to this stage, respiration is
and execution of movement. depressed but present.
5. B. This symptom is caused by stimulation of retinal 22. B. Compression of the lung by fluid that accumulates
cells by ocular movement. at the base of the lungs reduces expansion and air
6. D. Restlessness indicates a lack of oxygen to the brain exchange.
stem which impairs the reticular activating system. 23. C. Assessment of a client with Hodgkin’s disease
7. C. Rhythmic contraction and relaxation associated most often reveals enlarged, painless lymph node,
with tonic-clonic seizure can cause repeated banging fever, malaise and night sweats.
of head. 24. A. Fractured pain is generally described as sharp,
8. A. Right side lying position or supine position permits continuous, and increasing in frequency.
ventilation of the remaining lung and prevent fluid 25. D. Signs and symptoms of infection under a casted
from draining into sutured bronchial stump. area include odor or purulent drainage and the
9. C. Isoniazid (INH) interferes in the effectiveness of oral presence of “hot spot” which are areas on the cast
contraceptives and clients of childbearing age should that are warmer than the others.
be counseled to use an alternative form of birth 26. B. Otoscopic examnation in a client with mastoiditis
control while taking this drug. reveals a dull, red, thick and immobile tymphanic
10. B. A client who has had abdominal surgery is best membrane with or without perforation.
placed in a low fowler’s position. This relaxes 27. D. Loss of gastric fluid via nasogastric suction or
abdominal muscles and provides maximum vomiting causes metabolic alkalosis because of the
respiratory and cardiovascular function. loss of hydrochloric acid which is a potent acid in the
11. A. Dark red to purple stoma indicates inadequate body.
blood supply. 28. A. The adult with normal cerebrospinal fluid has no
12. C. The rationale for activity restriction is to help red blood cells.
reduce the hypermotility of the colon. 29. D. Measuring the urine output to detect excess
13. A. During Total Parenteral Nutrition (TPN) amount and checking the specific gravity of urine
administration, the client should be monitored samples to determine urine concentration are
regularly for hyperglycemia. appropriate measures to determine the onset of
14. D. Jaundice may be present in acute pancreatitis diabetes insipidus.
owing to obstruction of the biliary duct. 30. B. The nurse should focus more on developing less
15. A. Tetany may occur after thyroidectomy if the stressful ways of accomplishing routine task.
parathyroid glands are accidentally injured or 31. C. Autotransfusion is acceptable for the client who is
removed. in danger of cardiac arrest.
16. D. Typical signs of hypothyroidism includes weight 32. D. The client with thromboembolism does not have
gain, fatigue, decreased energy, apathy, brittle nails, coolness.
dry skin, cold intolerance, constipation and 33. A. Positioning the client on one side with head flexed
numbness. forward allows the tongue to fall forward and
17. B. After a pelvic surgery, there is an increased chance facilitates drainage secretions therefore prevents
of thrombophlebitits owing to the pelvic aspiration.

49
49
34. C. Nursing care after bone biopsy includes close
monitoring of the punctured site for bleeding,
swelling and hematoma formation.
35. D. Walking and swimming are very helpful in
strengthening back muscles for the client suffering
from lower back pain.
36. C. Sudden, severe abdominal pain is the most
indicative sign of perforation. When perforation of
an ulcer occurs, the nurse maybe unable to hear
bowel sounds at all.
37. A. After surgery to correct a detached retina,
prevention of increased intraocular pressure is the
priority goal.
38. A. Miotic agent constricts the pupil and contracts
ciliary muscle. These effects widen the filtration
angle and permit increased out flow of aqueous
humor.
39. D. It is a priority to hyperoxygenate the client before
and after suctioning to prevent hypoxia and to
maintain cerebral perfusion.
40. D. Abdominal breathing improves lungs expansion
41. C. A Client with burns is very sensitive to
temperature changes because heat is loss in the
burn areas.
42. A. The graft covers the nerve endings, which reduces
pain and provides framework for granulation
43. B. Meat provides proteins and the fruit proteins
vitamin C that both promote wound healing.
44. C. This is primarily caused by the trauma of intestinal
manipulation and the depressive effects anesthetics
and analgesics.
45. D. Constipation, diarrhea, and/or constipation
alternating with diarrhea are the most common
symptoms of colorectal cancer.
46. B. With increased intraabdominal pressure, the
abdominal wall will become tender and rigid.
47. A. Pressure applied in the puncture site indicates
that a biliary vessel was puncture which is a common
complication after liver biopsy.
48. B. Hepatitis A is primarily spread via fecal-oral route.
Sewage polluted water may harbor the virus.
49. B. Amylase concentration is high in the pancreas and
is elevated in the serum when the pancreas becomes
acutely inflamed and also it distinguishes
pancreatitis from other acute abdominal problems.
50. A. Sodium, which is concerned with the regulation of
extracellular fluid volume, it is lost with vomiting.
Chloride, which balances cations in the extracellular
compartments, is also lost with vomiting, because
sodium and chloride are parallel electrolytes,
hyponatremia will accompany.

49
49
MEDICAL SURGICAL NURSING Part 3 7. A client has undergone with penile implant. After 24
hrs of surgery, the client’s scrotum was edematous
1. Marco who was diagnosed with brain tumor was and painful. The nurse should:
scheduled for craniotomy. In preventing the a. Assist the client with sitz bath
development of cerebral edema after surgery, the b. Apply war soaks in the scrotum
nurse should expect the use of: c. Elevate the scrotum using a soft support
a. Diuretics d. Prepare for a possible incision and drainage.
b. Antihypertensive 8. Nurse hazel receives emergency laboratory results for
c. Steroids a client with chest pain and immediately informs the
d. Anticonvulsants physician. An increased myoglobin level suggests
2. Halfway through the administration of blood, the which of the following?
female client complains of lumbar pain. After a. Liver disease
stopping the infusion Nurse Hazel should: b. Myocardial damage
a. Increase the flow of normal saline c. Hypertension
b. Assess the pain further d. Cancer
c. Notify the blood bank 9. Nurse Maureen would expect the a client with mitral
d. Obtain vital signs. stenosis would demonstrate symptoms associated
3. Nurse Maureen knows that the positive diagnosis for with congestion in the:
HIV infection is made based on which of the a. Right atrium
following: b. Superior vena cava
a. A history of high risk sexual behaviors. c. Aorta
b. Positive ELISA and western blot tests d. Pulmonary
c. Identification of an associated opportunistic 10. A client has been diagnosed with hypertension. The
infection nurse priority nursing diagnosis would be:
d. Evidence of extreme weight loss and high a. Ineffective health maintenance
fever b. Impaired skin integrity
4. Nurse Maureen is aware that a client who has been c. Deficient fluid volume
diagnosed with chronic renal failure recognizes an d. Pain
adequate amount of high-biologic-value protein 11. Nurse Hazel teaches the client with angina about
when the food the client selected from the menu common expected side effects of nitroglycerin
was: including:
a. Raw carrots a. high blood pressure
b. Apple juice b. stomach cramps
c. Whole wheat bread c. headache
d. Cottage cheese d. shortness of breath
5. Kenneth who has diagnosed with uremic syndrome 12. The following are lipid abnormalities. Which of the
has the potential to develop complications. Which following is a risk factor for the development of
among the following complications should the nurse atherosclerosis and PVD?
anticipates: a. High levels of low density lipid (LDL)
a. Flapping hand tremors cholesterol
b. An elevated hematocrit level b. High levels of high density lipid (HDL)
c. Hypotension cholesterol
d. Hypokalemia c. Low concentration triglycerides
6. A client is admitted to the hospital with benign d. Low levels of LDL cholesterol.
prostatic hyperplasia, the nurse most relevant 13. Which of the following represents a significant risk
assessment would be: immediately after surgery for repair of aortic
a. Flank pain radiating in the groin aneurysm?
b. Distention of the lower abdomen a. Potential wound infection
c. Perineal edema b. Potential ineffective coping
d. Urethral discharge c. Potential electrolyte balance
d. Potential alteration in renal perfusion

49
49
14. Nurse Josie should instruct the client to eat which of 21. Which of the following signs and symptoms would
the following foods to obtain the best supply of Nurse Maureen include in teaching plan as an early
Vitamin B12? manifestation of laryngeal cancer?
a. dairy products a. Stomatitis
b. vegetables b. Airway obstruction
c. Grains c. Hoarseness
d. Broccoli d. Dysphagia
15. Karen has been diagnosed with aplastic anemia. The 22. Karina a client with myasthenia gravis is to receive
nurse monitors for changes in which of the following immunosuppressive therapy. The nurse understands
physiologic functions? that this therapy is effective because it:
a. Bowel function a. Promotes the removal of antibodies that
b. Peripheral sensation impair the transmission of impulses
c. Bleeding tendencies b. Stimulates the production of acetylcholine at
d. Intake and out put the neuromuscular junction.
16. Lydia is scheduled for elective splenectomy. Before c. Decreases the production of autoantibodies
the clients goes to surgery, the nurse in charge final that attack the acetylcholine receptors.
assessment would be: d. Inhibits the breakdown of acetylcholine at the
a. signed consent neuromuscular junction.
b. vital signs 23. A female client is receiving IV Mannitol. An
c. name band assessment specific to safe administration of the
d. empty bladder said drug is:
17. What is the peak age range in acquiring acute a. Vital signs q4h
lymphocytic leukemia (ALL)? b. Weighing daily
a. 4 to 12 years. c. Urine output hourly
b. 20 to 30 years d. Level of consciousness q4h
c. 40 to 50 years 24. Patricia a 20 year old college student with diabetes
d. 60 60 70 years mellitus requests additional information about the
18. Marie with acute lymphocytic leukemia suffers from advantages of using a pen like insulin delivery
nausea and headache. These clinical manifestations devices. The nurse explains that the advantages of
may indicate all of the following except these devices over syringes includes:
a. effects of radiation a. Accurate dose delivery
b. chemotherapy side effects b. Shorter injection time
c. meningeal irritation c. Lower cost with reusable insulin cartridges
d. gastric distension d. Use of smaller gauge needle.
19. A client has been diagnosed with Disseminated 25. A male client’s left tibia was fractured in an
Intravascular Coagulation (DIC). Which of the automobile accident, and a cast is applied. To assess
following is contraindicated with the client? for damage to major blood vessels from the fracture
a. Administering Heparin tibia, the nurse in charge should monitor the client
b. Administering Coumadin for:
c. Treating the underlying cause a. Swelling of the left thigh
d. Replacing depleted blood products b. Increased skin temperature of the foot
20. Which of the following findings is the best indication c. Prolonged reperfusion of the toes after
that fluid replacement for the client with blanching
hypovolemic shock is adequate? d. Increased blood pressure
a. Urine output greater than 30ml/hr 26. After a long leg cast is removed, the male client
b. Respiratory rate of 21 breaths/minute should:
c. Diastolic blood pressure greater than 90 a. Cleanse the leg by scrubbing with a brisk
mmhg motion
d. Systolic blood pressure greater than 110 b. Put leg through full range of motion twice
mmhg daily
c. Report any discomfort or stiffness to the
physician

49
49
d. Elevate the leg when sitting for long periods of cause of tonic clonic seizures in adults more the 20
time. years?
27. While performing a physical assessment of a male a. Electrolyte imbalance
client with gout of the great toe, Nurse Vivian should b. Head trauma
assess for additional tophi (urate deposits) on the: c. Epilepsy
a. Buttocks d. Congenital defect
b. Ears 34. What is the priority nursing assessment in the first 24
c. Face hours after admission of the client with thrombotic
d. Abdomen CVA?
28. Nurse Katrina would recognize that the a. Pupil size and papillary response
demonstration of crutch walking with tripod gait was b. cholesterol level
understood when the client places weight on the: c. Echocardiogram
a. Palms of the hands and axillary regions d. Bowel sounds
b. Palms of the hand 35. Nurse Linda is preparing a client with multiple
c. Axillary regions sclerosis for discharge from the hospital to home.
d. Feet, which are set apart Which of the following instruction is most
29. Mang Jose with rheumatoid arthritis states, “the only appropriate?
time I am without pain is when I lie in bed perfectly a. “Practice using the mechanical aids that you
still”. During the convalescent stage, the nurse in will need when future disabilities arise”.
charge with Mang Jose should encourage: b. “Follow good health habits to change the
a. Active joint flexion and extension course of the disease”.
b. Continued immobility until pain subsides c. “Keep active, use stress reduction strategies,
c. Range of motion exercises twice daily and avoid fatigue.
d. Flexion exercises three times daily d. “You will need to accept the necessity for a
30. A male client has undergone spinal surgery, the quiet and inactive lifestyle”.
nurse should: 36. The nurse is aware the early indicator of hypoxia in
a. Observe the client’s bowel movement and the unconscious client is:
voiding patterns a. Cyanosis
b. Log-roll the client to prone position b. Increased respirations
c. Assess the client’s feet for sensation and c. Hypertension
circulation d. Restlessness
d. Encourage client to drink plenty of fluids 37. A client is experiencing spinal shock. Nurse Myrna
31. Marina with acute renal failure moves into the should expect the function of the bladder to be
diuretic phase after one week of therapy. During this which of the following?
phase the client must be assessed for signs of a. Normal
developing: b. Atonic
a. Hypovolemia c. Spastic
b. renal failure d. Uncontrolled
c. metabolic acidosis 38. Which of the following stage the carcinogen is
d. hyperkalemia irreversible?
32. Nurse Judith obtains a specimen of clear nasal a. Progression stage
drainage from a client with a head injury. Which of b. Initiation stage
the following tests differentiates mucus from c. Regression stage
cerebrospinal fluid (CSF)? d. Promotion stage
a. Protein 39. Among the following components thorough pain
b. Specific gravity assessment, which is the most significant?
c. Glucose a. Effect
d. Microorganism b. Cause
33. A 22 year old client suffered from his first tonic- c. Causing factors
clonic seizure. Upon awakening the client asks the d. Intensity
nurse, “What caused me to have a seizure? Which of
the following would the nurse include in the primary

49
49
40. A 65 year old female is experiencing flare up of d. elder abuse
pruritus. Which of the client’s action could aggravate 48. Nurse Anna is aware that early adaptation of client
the cause of flare ups? with renal carcinoma is:
a. Sleeping in cool and humidified environment a. Nausea and vomiting
b. Daily baths with fragrant soap b. flank pain
c. Using clothes made from 100% cotton c. weight gain
d. Increasing fluid intake d. intermittent hematuria
41. Atropine sulfate (Atropine) is contraindicated in all 49. A male client with tuberculosis asks Nurse Brian how
but one of the following client? long the chemotherapy must be continued. Nurse
a. A client with high blood Brian’s accurate reply would be:
b. A client with bowel obstruction a. 1 to 3 weeks
c. A client with glaucoma b. 6 to 12 months
d. A client with U.T.I c. 3 to 5 months
42. Among the following clients, which among them is d. 3 years and more
high risk for potential hazards from the surgical 50. A client has undergone laryngectomy. The immediate
experience? nursing priority would be:
a. 67-year-old client a. Keep trachea free of secretions
b. 49-year-old client b. Monitor for signs of infection
c. 33-year-old client c. Provide emotional support
d. 15-year-old client d. Promote means of communication
43. Nurse Jon assesses vital signs on a client undergone
epidural anesthesia. Which of the following would
the nurse assess next?
a. Headache
b. Bladder distension
c. Dizziness
d. Ability to move legs
44. Nurse Katrina should anticipate that all of the
following drugs may be used in the attempt to
control the symptoms of Meniere’s disease except:
a. Antiemetics
b. Diuretics
c. Antihistamines
d. Glucocorticoids
45. Which of the following complications associated with
tracheostomy tube?
a. Increased cardiac output
b. Acute respiratory distress syndrome (ARDS)
c. Increased blood pressure
d. Damage to laryngeal nerves
46. Nurse Faith should recognize that fluid shift in an
client with burn injury results from increase in the:
a. Total volume of circulating whole blood
b. Total volume of intravascular plasma
c. Permeability of capillary walls
d. Permeability of kidney tubules
47. An 83-year-old woman has several ecchymotic areas
on her right arm. The bruises are probably caused
by:
a. increased capillary fragility and permeability
b. increased blood supply to the skin
c. self inflicted injury

49
49
ANSWERS and RATIONALES for MEDICAL SURGICAL 15. . Aplastic anemia decreases the bone marrow
NURSING Part 3 production of RBC’s, white blood cells, and platelets.
The client is at risk for bruising and bleeding
1. C. Glucocorticoids (steroids) are used for their anti- tendencies.
inflammatory action, which decreases the 16. B. An elective procedure is scheduled in advance so
development of edema. that all preparations can be completed ahead of
2. A. The blood must be stopped at once, and then time. The vital signs are the final check that must be
normal saline should be infused to keep the line completed before the client leaves the room so that
patent and maintain blood volume. continuity of care and assessment is provided for.
3. B. These tests confirm the presence of HIV antibodies 17. A. The peak incidence of Acute Lymphocytic
that occur in response to the presence of the human Leukemia (ALL) is 4 years of age. It is uncommon
immunodeficiency virus (HIV). after 15 years of age.
4. D. One cup of cottage cheese contains approximately 18. D. Acute Lymphocytic Leukemia (ALL) does not cause
225 calories, 27 g of protein, 9 g of fat, 30 mg gastric distention. It does invade the central nervous
cholesterol, and 6 g of carbohydrate. Proteins of system, and clients experience headaches and
high biologic value (HBV) contain optimal levels of vomiting from meningeal irritation.
amino acids essential for life. 19. B. Disseminated Intravascular Coagulation (DIC) has
5. A. Elevation of uremic waste products causes irritation not been found to respond to oral anticoagulants
of the nerves, resulting in flapping hand tremors. such as Coumadin.
6. B. This indicates that the bladder is distended with 20. A. Urine output provides the most sensitive
urine, therefore palpable. indication of the client’s response to therapy for
7. C. Elevation increases lymphatic drainage, reducing hypovolemic shock. Urine output should be
edema and pain. consistently greater than 30 to 35 mL/hr.
8. B. Detection of myoglobin is a diagnostic tool to 21. C. Early warning signs of laryngeal cancer can vary
determine whether myocardial damage has depending on tumor location. Hoarseness lasting 2
occurred. weeks should be evaluated because it is one of the
9. D. When mitral stenosis is present, the left atrium has most common warning signs.
difficulty emptying its contents into the left ventricle 22. C. Steroids decrease the body’s immune response
because there is no valve to prevent back ward flow thus decreasing the production of antibodies that
into the pulmonary vein, the pulmonary circulation is attack the acetylcholine receptors at the
under pressure. neuromuscular junction
10. A. Managing hypertension is the priority for the 23. C. The osmotic diuretic mannitol is contraindicated in
client with hypertension. Clients with hypertension the presence of inadequate renal function or heart
frequently do not experience pain, deficient volume, failure because it increases the intravascular volume
or impaired skin integrity. It is the asymptomatic that must be filtered and excreted by the kidney.
nature of hypertension that makes it so difficult to 24. A. These devices are more accurate because they are
treat. easily to used and have improved adherence in
11. C. Because of its widespread vasodilating effects, insulin regimens by young people because the
nitroglycerin often produces side effects such as medication can be administered discreetly.
headache, hypotension and dizziness. 25. C. Damage to blood vessels may decrease the
12. A. An increased in LDL cholesterol concentration has circulatory perfusion of the toes, this would indicate
been documented at risk factor for the development the lack of blood supply to the extremity.
of atherosclerosis. LDL cholesterol is not broken 26. D. Elevation will help control the edema that usually
down into the liver but is deposited into the wall of occurs.
the blood vessels. 27. B. Uric acid has a low solubility, it tends to
13. D. There is a potential alteration in renal perfusion precipitate and form deposits at various sites where
manifested by decreased urine output. The altered blood flow is least active, including cartilaginous
renal perfusion may be related to renal artery tissue such as the ears.
embolism, prolonged hypotension, or prolonged 28. B. The palms should bear the client’s weight to avoid
aortic cross-clamping during the surgery. damage to the nerves in the axilla.
14. A. Good source of vitamin B12 are dairy products
and meats.

49
49
29. A. Active exercises, alternating extension, flexion, 45. D. Tracheostomy tube has several potential
abduction, and adduction, mobilize exudates in the complications including bleeding, infection and
joints relieves stiffness and pain. laryngeal nerve damage.
30. C. Alteration in sensation and circulation indicates 46. C. In burn, the capillaries and small vessels dilate,
damage to the spinal cord, if these occurs notify and cell damage cause the release of a histamine-
physician immediately. like substance. The substance causes the capillary
31. A. In the diuretic phase fluid retained during the walls to become more permeable and significant
oliguric phase is excreted and may reach 3 to 5 liters quantities of fluid are lost.
daily, hypovolemia may occur and fluids should be 47. A. Aging process involves increased capillary fragility
replaced. and permeability. Older adults have a decreased
32. C. The constituents of CSF are similar to those of amount of subcutaneous fat and cause an increased
blood plasma. An examination for glucose content is incidence of bruise like lesions caused by collection
done to determine whether a body fluid is a mucus of extravascular blood in loosely structured dermis.
or a CSF. A CSF normally contains glucose. 48. D. Intermittent pain is the classic sign of renal
33. B. Trauma is one of the primary cause of brain carcinoma. It is primarily due to capillary erosion by
damage and seizure activity in adults. Other the cancerous growth.
common causes of seizure activity in adults include 49. B. Tubercle bacillus is a drug resistant organism and
neoplasms, withdrawal from drugs and alcohol, and takes a long time to be eradicated. Usually a
vascular disease. combination of three drugs is used for minimum of 6
34. A. It is crucial to monitor the pupil size and papillary months and at least six months beyond culture
response to indicate changes around the cranial conversion.
nerves. 50. A. Patent airway is the most priority; therefore
35. C. The nurse most positive approach is to encourage removal of secretions is necessary.
the client with multiple sclerosis to stay active, use
stress reduction techniques and avoid fatigue
because it is important to support the immune
system while remaining active.
36. D. Restlessness is an early indicator of hypoxia. The
nurse should suspect hypoxia in unconscious client
who suddenly becomes restless.
37. B. In spinal shock, the bladder becomes completely
atonic and will continue to fill unless the client is
catheterized.
38. A. Progression stage is the change of tumor from the
preneoplastic state or low degree of malignancy to a
fast growing tumor that cannot be reversed.
39. D. Intensity is the major indicative of severity of pain
and it is important for the evaluation of the
treatment.
40. B. The use of fragrant soap is very drying to skin
hence causing the pruritus.
41. C. Atropine sulfate is contraindicated with glaucoma
patients because it increases intraocular pressure.
42. A. A 67 year old client is greater risk because the
older adult client is more likely to have a less-
effective immune system.
43. B. The last area to return sensation is in the perineal
area, and the nurse in charge should monitor the
client for distended bladder.
44. D. Glucocorticoids play no significant role in disease
treatment.

49
49
PSYCHIATRIC NURSING Part 1 b. Avoiding relationship
c. Showing interest in solitary activities
1. Marco approached Nurse Trish asking for advice d. Inability to make choices and decision
on how to deal with his alcohol addiction. Nurse without advise
Trish should tell the client that the only effective 8. A male client is diagnosed with schizotypal
treatment for alcoholism is: personality disorder. Which signs would this client
a. Psychotherapy exhibit during social situation?
b. Alcoholics anonymous (A.A.) a. Paranoid thoughts
c. Total abstinence b. Emotional affect
d. Aversion Therapy c. Independence need
2. Nurse Hazel is caring for a male client who d. Aggressive behavior
experience false sensory perceptions with no basis in 9. Nurse Claire is caring for a client diagnosed with
reality. This perception is known as: bulimia. The most appropriate initial goal for a client
a. Hallucinations diagnosed with bulimia is?
b. Delusions a. Encourage to avoid foods
c. Loose associations b. Identify anxiety causing situations
d. Neologisms c. Eat only three meals a day
3. Nurse Monet is caring for a female client who d. Avoid shopping plenty of groceries
has suicidal tendency. When accompanying the 10. Nurse Tony was caring for a 41 year old female
client to the restroom, Nurse Monet should… client. Which behavior by the client indicates adult
a. Give her privacy cognitive development?
b. Allow her to urinate a. Generates new levels of awareness
c. Open the window and allow her to get b. Assumes responsibility for her actions
some fresh air c. Has maximum ability to solve problems
d. Observe her and learn new skills
4. Nurse Maureen is developing a plan of care for a d. Her perception are based on reality
female client with anorexia nervosa. Which action 11. A neuromuscular blocking agent is administered
should the nurse include in the plan? to a client before ECT therapy. The Nurse should
a. Provide privacy during meals carefully observe the client for?
b. Set-up a strict eating plan for the client a. Respiratory difficulties
c. Encourage client to exercise to reduce b. Nausea and vomiting
anxiety c. Dizziness
d. Restrict visits with the family d. Seizures
5. A client is experiencing anxiety attack. The most 12. A 75 year old client is admitted to the hospital
appropriate nursing intervention should include? with the diagnosis of dementia of the Alzheimer’s
a. Turning on the television type and depression. The symptom that is unrelated
b. Leaving the client alone to depression would be?
c. Staying with the client and speaking in a. Apathetic response to the environment
short sentences b. “I don’t know” answer to questions
d. Ask the client to play with other clients c. Shallow of labile effect
6. A female client is admitted with a diagnosis of d. Neglect of personal hygiene
delusions of GRANDEUR. This diagnosis reflects a 13. Nurse Trish is working in a mental health facility;
belief that one is: the nurse priority nursing intervention for a newly
a. Being Killed admitted client with bulimia nervosa would be to?
b. Highly famous and important a. Teach client to measure I & O
c. Responsible for evil world b. Involve client in planning daily meal
d. Connected to client unrelated to oneself c. Observe client during meals
7. A 20 year old client was diagnosed with d. Monitor client continuously
dependent personality disorder. Which behavior is 14. Nurse Patricia is aware that the major health
not most likely to be evidence of ineffective complication associated with intractable anorexia
individual coping? nervosa would be?
a. Recurrent self-destructive behavior

49
49
a. Cardiac dysrhythmias resulting to c. Shame
cardiac arrest d. Remorsefulness
b. Glucose intolerance resulting in 20. Which of the following approaches would be
protracted hypoglycemia most appropriate to use with a client suffering from
c. Endocrine imbalance causing cold narcissistic personality disorder when discrepancies
amenorrhea exist between what the client states and what
d. Decreased metabolism causing cold actually exist?
intolerance a. Rationalization
15. Nurse Anna can minimize agitation in a b. Supportive confrontation
disturbed client by? c. Limit setting
a. Increasing stimulation d. Consistency
b. limiting unnecessary interaction 21. Cely is experiencing alcohol withdrawal exhibits
c. increasing appropriate sensory tremors, diaphoresis and hyperactivity. Blood
perception pressure is 190/87 mmhg and pulse is 92 bpm.
d. ensuring constant client and staff Which of the medications would the nurse expect to
contact administer?
16. A 39 year old mother with obsessive-compulsive a. Naloxone (Narcan)
disorder has become immobilized by her elaborate b. Benzlropine (Cogentin)
hand washing and walking rituals. Nurse Trish c. Lorazepam (Ativan)
recognizes that the basis of O.C. disorder is often: d. Haloperidol (Haldol)
a. Problems with being too conscientious 22. Which of the following foods would the nurse
b. Problems with anger and remorse Trish eliminate from the diet of a client in alcohol
c. Feelings of guilt and inadequacy withdrawal?
d. Feeling of unworthiness and a. Milk
hopelessness b. Orange Juice
17. Mario is complaining to other clients about not c. Soda
being allowed by staff to keep food in his room. d. Regular Coffee
Which of the following interventions would be most 23. Which of the following would Nurse Hazel
appropriate? expect to assess for a client who is exhibiting late
a. Allowing a snack to be kept in his room signs of heroin withdrawal?
b. Reprimanding the client a. Yawning & diaphoresis
c. Ignoring the clients behavior b. Restlessness & Irritability
d. Setting limits on the behavior c. Constipation & steatorrhea
18. Conney with borderline personality disorder who d. Vomiting and Diarrhea
is to be discharge soon threatens to “do something” 24. To establish open and trusting relationship with
to herself if discharged. Which of the following a female client who has been hospitalized with
actions by the nurse would be most important? severe anxiety, the nurse in charge should?
a. Ask a family member to stay with the a. Encourage the staff to have frequent
client at home temporarily interaction with the client
b. Discuss the meaning of the client’s b. Share an activity with the client
statement with her c. Give client feedback about behavior
c. Request an immediate extension for the d. Respect client’s need for personal space
client 25. Nurse Monette recognizes that the focus of
d. Ignore the clients statement because it’s environmental (MILIEU) therapy is to:
a sign of manipulation a. Manipulate the environment to bring
19. Joey a client with antisocial personality disorder about positive changes in behavior
belches loudly. A staff member asks Joey, “Do you b. Allow the client’s freedom to determine
know why people find you repulsive?” this whether or not they will be involved in activities
statement most likely would elicit which of the c. Role play life events to meet individual
following client reaction? needs
a. Depensiveness d. Use natural remedies rather than drugs
b. Embarrassment to control behavior

49
50
26. Nurse Trish would expect a child with a diagnosis b. Depression and a blunted affect when
of reactive attachment disorder to: discussing the traumatic situation
a. Have more positive relation with the c. Lack of interest in family & others
father than the mother d. Re-experiencing the trauma in dreams or
b. Cling to mother & cry on separation flashback
c. Be able to develop only superficial 33. Nurse Benjie is communicating with a male client
relation with the others with substance-induced persisting dementia; the
d. Have been physically abuse client cannot remember facts and fills in the gaps
27. When teaching parents about childhood with imaginary information. Nurse Benjie is aware
depression Nurse Trina should say? that this is typical of?
a. It may appear acting out behavior a. Flight of ideas
b. Does not respond to conventional b. Associative looseness
treatment c. Confabulation
c. Is short in duration & resolves easily d. Concretism
d. Looks almost identical to adult 34. Nurse Joey is aware that the signs & symptoms
depression that would be most specific for diagnosis anorexia
28. Nurse Perry is aware that language development are?
in autistic child resembles: a. Excessive weight loss, amenorrhea &
a. Scanning speech abdominal distension
b. Speech lag b. Slow pulse, 10% weight loss & alopecia
c. Shuttering c. Compulsive behavior, excessive fears &
d. Echolalia nausea
29. A 60 year old female client who lives alone tells d. Excessive activity, memory lapses & an
the nurse at the community health center “I really increased pulse
don’t need anyone to talk to”. The TV is my best 35. A characteristic that would suggest to Nurse
friend. The nurse recognizes that the client is using Anne that an adolescent may have bulimia would be:
the defense mechanism known as? a. Frequent regurgitation & re-swallowing
a. Displacement of food
b. Projection b. Previous history of gastritis
c. Sublimation c. Badly stained teeth
d. Denial d. Positive body image
30. When working with a male client suffering 36. Nurse Monette is aware that extremely
phobia about black cats, Nurse Trish should depressed clients seem to do best in settings where
anticipate that a problem for this client would be? they have:
a. Anxiety when discussing phobia a. Multiple stimuli
b. Anger toward the feared object b. Routine Activities
c. Denying that the phobia exist c. Minimal decision making
d. Distortion of reality when completing d. Varied Activities
daily routines 37. To further assess a client’s suicidal potential.
31. Linda is pacing the floor and appears extremely Nurse Katrina should be especially alert to the client
anxious. The duty nurse approaches in an attempt to expression of:
alleviate Linda’s anxiety. The most therapeutic a. Frustration & fear of death
question by the nurse would be? b. Anger & resentment
a. Would you like to watch TV? c. Anxiety & loneliness
b. Would you like me to talk with you? d. Helplessness & hopelessness
c. Are you feeling upset now? 38. A nursing care plan for a male client with bipolar
d. Ignore the client I disorder should include:
32. Nurse Penny is aware that the symptoms that a. Providing a structured environment
distinguish post traumatic stress disorder from other b. Designing activities that will require the
anxiety disorder would be: client to maintain contact with reality
a. Avoidance of situation & certain c. Engaging the client in conversing about
activities that resemble the stress current affairs

50
50
d. Touching the client provide assurance a. Ask the client direct questions to
39. When planning care for a female client using encourage talking
ritualistic behavior, Nurse Gina must recognize that b. Rake the client into the dayroom to be
the ritual: with other clients
a. Helps the client focus on the inability to c. Sit beside the client in silence and
deal with reality occasionally ask open-ended question
b. Helps the client control the anxiety d. Leave the client alone and continue with
c. Is under the client’s conscious control providing care to the other clients
d. Is used by the client primarily for 45. Nurse Tina is caring for a client with delirium and
secondary gains states that “look at the spiders on the wall”. What
40. A 32 year old male graduate student, who has should the nurse respond to the client?
become increasingly withdrawn and neglectful of his a. “You’re having hallucination, there are
work and personal hygiene, is brought to the no spiders in this room at all”
psychiatric hospital by his parents. After detailed b. “I can see the spiders on the wall, but
assessment, a diagnosis of schizophrenia is made. It they are not going to hurt you”
is unlikely that the client will demonstrate: c. “Would you like me to kill the spiders”
a. Low self esteem d. “I know you are frightened, but I do not
b. Concrete thinking see spiders on the wall”
c. Effective self boundaries 46. Nurse Jonel is providing information to a
d. Weak ego community group about violence in the family.
41. A 23 year old client has been admitted with a Which statement by a group member would indicate
diagnosis of schizophrenia says to the nurse “Yes, its a need to provide additional information?
march, March is little woman”. That’s literal you a. “Abuse occurs more in low-income
know”. These statement illustrate: families”
a. Neologisms b. “Abuser Are often jealous or self-
b. Echolalia centered”
c. Flight of ideas c. “Abuser use fear and intimidation”
d. Loosening of association d. “Abuser usually have poor self-esteem”
42. A long term goal for a paranoid male client who 47. During electroconvulsive therapy (ECT) the client
has unjustifiably accused his wife of having many receives oxygen by mask via positive pressure
extramarital affairs would be to help the client ventilation. The nurse assisting with this procedure
develop: knows that positive pressure ventilation is necessary
a. Insight into his behavior because?
b. Better self control a. Anesthesia is administered during the
c. Feeling of self worth procedure
d. Faith in his wife b. Decrease oxygen to the brain increases
43. A male client who is experiencing disordered confusion and disorientation
thinking about food being poisoned is admitted to c. Grand mal seizure activity depresses
the mental health unit. The nurse uses which respirations
communication technique to encourage the client to d. Muscle relaxations given to prevent
eat dinner? injury during seizure activity depress respirations.
a. Focusing on self-disclosure of own food 48. When planning the discharge of a client with
preference chronic anxiety, Nurse Chris evaluates achievement
b. Using open ended question and silence of the discharge maintenance goals. Which goal
c. Offering opinion about the need to eat would be most appropriately having been included
d. Verbalizing reasons that the client may in the plan of care requiring evaluation?
not choose to eat a. The client eliminates all anxiety from
44. Nurse Nina is assigned to care for a client daily situations
diagnosed with Catatonic Stupor. When Nurse Nina b. The client ignores feelings of anxiety
enters the client’s room, the client is found lying on c. The client identifies anxiety producing
the bed with a body pulled into a fetal position. situations
Nurse Nina should?

50
50
d. The client maintains contact with a crisis
counselor
49. Nurse Tina is caring for a client with depression
who has not responded to antidepressant
medication. The nurse anticipates that what
treatment procedure may be prescribed?
a. Neuroleptic medication
b. Short term seclusion
c. Psychosurgery
d. Electroconvulsive therapy
50. Mario is admitted to the emergency room with
drug-included anxiety related to over ingestion of
prescribed antipsychotic medication. The most
important piece of information the nurse in charge
should obtain initially is the:
a. Length of time on the med.
b. Name of the ingested medication & the
amount ingested
c. Reason for the suicide attempt
d. Name of the nearest relative & their phone
number

50
50
ANSWERS and RATIONALES for PSYCHIATRIC 17. D. The nurse needs to set limits in the client’s
NURSING Part 1 manipulative behavior to help the client control
dysfunctional behavior. A consistent approach by the
1. C. Total abstinence is the only effective treatment for staff is necessary to decrease manipulation.
alcoholism. 18. B. Any suicidal statement must be assessed by the
2. A. Hallucinations are visual, auditory, gustatory, tactile nurse. The nurse should discuss the client’s
or olfactory perceptions that have no basis in reality. statement with her to determine its meaning in
3. D. The Nurse has a responsibility to observe terms of suicide.
continuously the acutely suicidal client. The Nurse 19. A. When the staff member ask the client if he
should watch for clues, such as communicating wonders why others find him repulsive, the client is
suicidal thoughts, and messages; hoarding likely to feel defensive because the question is
medications and talking about death. belittling. The natural tendency is to counterattack
4. B. Establishing a consistent eating plan and the threat to self image.
monitoring client’s weight are important to this 20. B. The nurse would specifically use supportive
disorder. confrontation with the client to point out
5. C. Appropriate nursing interventions for an anxiety discrepancies between what the client states and
attack include using short sentences, staying with what actually exists to increase responsibility for
the client, decreasing stimuli, remaining calm and self.
medicating as needed. 21. C. The nurse would most likely administer
6. B. Delusion of grandeur is a false belief that one is benzodiazepine, such as lorazepan (ativan) to the
highly famous and important. client who is experiencing symptom: The client’s
7. D. Individual with dependent personality disorder experiences symptoms of withdrawal because of the
typically shows indecisiveness submissiveness and rebound phenomenon when the sedation of the CNS
clinging behavior so that others will make decisions from alcohol begins to decrease.
with them. 22. D. Regular coffee contains caffeine which acts as
8. A. Clients with schizotypal personality disorder psychomotor stimulants and leads to feelings of
experience excessive social anxiety that can lead anxiety and agitation. Serving coffee top the client
to paranoid thoughts. may add to tremors or wakefulness.
9. B. Bulimia disorder generally is a maladaptive coping 23. D. Vomiting and diarrhea are usually the late signs of
response to stress and underlying issues. The client heroin withdrawal, along with muscle spasm, fever,
should identify anxiety causing situation that nausea, repetitive, abdominal cramps and backache.
stimulate the bulimic behavior and then learn new 24. D. Moving to a client’s personal space increases
ways of coping with the anxiety. the feeling of threat, which increases anxiety.
10. A. An adult age 31 to 45 generates new level of 25. A. Environmental (MILIEU) therapy aims at having
awareness. everything in the client’s surrounding area toward
11. A. Neuromuscular Blocker, such as helping the client.
SUCCINYLCHOLINE (Anectine) produces respiratory 26. C. Children who have experienced attachment
depression because it inhibits contractions of difficulties with primary caregiver are not able to
respiratory muscles. trust others and therefore relate superficially
12. C. With depression, there is little or no emotional 27. A. Children have difficulty verbally expressing their
involvement therefore little alteration in affect. feelings, acting out behavior, such as temper
13. D. These clients often hide food or force vomiting; tantrums, may indicate underlying depression.
therefore they must be carefully monitored. 28. D. The autistic child repeat sounds or words spoken
14. A. These clients have severely depleted levels of by others.
sodium and potassium because of their starvation 29. D. The client statement is an example of the use
diet and energy expenditure, these electrolytes are of denial, a defense that blocks problem by
necessary for cardiac functioning. unconscious refusing to admit they exist.
15. B. Limiting unnecessary interaction will decrease 30. A. Discussion of the feared object triggers an
stimulation and agitation. emotional response to the object.
16. C. Ritualistic behavior seen in this disorder is aimed 31. B. The nurse presence may provide the client with
at controlling guilt and inadequacy by maintaining an support & feeling of control.
absolute set pattern of behavior.

50
50
32. D. Experiencing the actual trauma in dreams or 49. D. Electroconvulsive therapy is an effective
flashback is the major symptom that distinguishes treatment for depression that has not responded to
post traumatic stress disorder from other anxiety medication.
disorder. 50. B. In an emergency, lives saving facts are obtained
33. C. Confabulation or the filling in of memory gaps with first. The name and the amount of medication
imaginary facts is a defense mechanism used by ingested are of outmost important in treating this
people experiencing memory deficits. potentially life threatening situation.
34. A. These are the major signs of anorexia nervosa.
Weight loss is excessive (15% of expected
weight).
35. C. Dental enamel erosion occurs from repeated self-
induced vomiting.
36. B. Depression usually is both emotional & physical. A
simple daily routine is the best, least stressful and
least anxiety producing.
37. D. The expression of these feeling may indicate
that this client is unable to continue the struggle of
life.
38. A. Structure tends to decrease agitation and anxiety
and to increase the client’s feeling of security.
39. B. The rituals used by a client with obsessive
compulsive disorder help control the anxiety level by
maintaining a set pattern of action.
40. C. A person with this disorder would not have
adequate self-boundaries.
41. D. Loose associations are thoughts that are
presented without the logical connections usually
necessary for the listening to interpret the message.
42. C. Helping the client to develop feeling of self worth
would reduce the client’s need to use pathologic
defenses.
43. B. Open ended questions and silence are strategies
used to encourage clients to discuss their problem in
descriptive manner.
44. C. Clients who are withdrawn may be immobile and
mute, and require consistent, repeated
interventions. Communication with withdrawn
clients requires much patience from the nurse. The
nurse facilitates communication with the client by
sitting in silence, asking open-ended question and
pausing to provide opportunities for the client to
respond.
45. D. When hallucination is present, the nurse should
reinforce reality with the client.
46. A. Personal characteristics of abuser include low self-
esteem, immaturity, dependence, insecurity and
jealousy.
47. D. A short acting skeletal muscle relaxant such as
succinylcholine (Anectine) is administered during
this procedure to prevent injuries during seizure.
48. C. Recognizing situations that produce anxiety
allows the client to prepare to cope with anxiety or
avoid specific stimulus.
50
50
PSYCHIATRIC NURSING Part 2 c. Psychoanalysis
d. Antidepressant therapy
1. Nurse Tony should first discuss terminating the nurse- 7. Danny who is diagnosed with bipolar disorder and
client relationship with a client during the: acute mania, states the nurse, “Where is my
a. Termination phase when discharge plans are daughter? I love Louis. Rain, rain go away. Dogs eat
being made. dirt.” The nurse interprets these statements as
b. Working phase when the client shows some indicating which of the following?
progress. a. Echolalia
c. Orientation phase when a contract is b. Neologism
established. c. Clang associations
d. Working phase when the client brings it up. d. Flight of ideas
2. Malou is diagnosed with major depression spends 8. Terry with mania is skipping up and down the hallway
majority of the day lying in bed with the sheet pulled practically running into other clients. Which of the
over his head. Which of the following approaches by following activities would the nurse in charge expect
the nurse would be the most therapeutic? to include in Terry’s plan of care?
a. Question the client until he responds a. Watching TV
b. Initiate contact with the client frequently b. Cleaning dayroom tables
c. Sit outside the clients room c. Leading group activity
d. Wait for the client to begin the conversation d. Reading a book
3. Joe who is very depressed exhibits psychomotor 9. When assessing a male client for suicidal risk, which of
retardation, a flat affect and apathy. The nurse in the following methods of suicide would the nurse
charge observes Joe to be in need of grooming and identify as most lethal?
hygiene. Which of the following nursing actions a. Wrist cutting
would be most appropriate? b. Head banging
a. Waiting until the client’s family can participate c. Use of gun
in the client’s care d. Aspirin overdose
b. Asking the client if he is ready to take shower 10. Jun has been hospitalized for major depression and
c. Explaining the importance of hygiene to the suicidal ideation. Which of the following statements
client indicates to the nurse that the client is improving?
d. Stating to the client that it’s time for him to take a. “I’m of no use to anyone anymore.”
a shower b. “I know my kids don’t need me anymore since
4. When teaching Mario with a typical depression about they’re grown.”
foods to avoid while taking phenelzine(Nardil), which c. “I couldn’t kill myself because I don’t want to go
of the following would the nurse in charge include? to hell.”
a. Roasted chicken d. “I don’t think about killing myself as much as I
b. Fresh fish used to.”
c. Salami 11. Which of the following activities would Nurse Trish
d. Hamburger recommend to the client who becomes very anxious
5. When assessing a female client who is receiving when thoughts of suicide occur?
tricyclic antidepressant therapy, which of the following a. Using exercise bicycle
would b. Meditating
alert the nurse to the possibility that the client is c. Watching TV
experiencing anticholinergic effects? d. Reading comics
a. Urine retention and blurred vision 12. When developing the plan of care for a client
b. Respiratory depression and convulsion receiving haloperidol, which of the following
c. Delirium and Sedation medications would nurse Monet anticipate
d. Tremors and cardiac arrhythmias administering if the client developed extra pyramidal
6. For a male client with dysthymic disorder, which of side effects?
the following approaches would the nurse expect to a. Olanzapine (Zyprexa)
implement? b. Paroxetine (Paxil)
a. ECT c. Benztropine mesylate (Cogentin)
b. Psychotherapeutic approach d. Lorazepam (Ativan)

50
50
13. Jon a suspicious client states that “I know you socialize during activities without being seductive.
nurses are spraying my food with poison as you take Nurse John would focus the discussion on which of
it out of the cart.” Which of the following would be the following areas?
the best response of the nurse? a. Discussing his relationship with his mother
a. Giving the client canned supplements until the b. Asking him to explain reasons for his seductive
delusion subsides behavior
b. Asking what kind of poison the client suspects is c. Suggesting to apologize to others for his
being used behavior
c. Serving foods that come in sealed packages d. Explaining the negative reactions of others
d. Allowing the client to be the first to open the toward his behavior
cart and get a tray 19. Tina with a histrionic personality disorder is
14. A client is suffering from catatonic behaviors. Which melodramatic and responds to others and situations
of the following would the nurse use to determine in an exaggerated manner. Nurse Trish would
that the medication administered PRN have been recommend which of the following activities for
most effective? Tina?
a. The client responds to verbal directions to eat a. Baking class
b. The client initiates simple activities without b. Role playing
direction c. Scrap book making
c. The client walks with the nurse to her room d. Music group
d. The client is able to move all extremities 20. Joy has entered the chemical dependency unit for
occasionally treatment of alcohol dependency. Which of the
15. Nurse Hazel invites new client’s parents to attend the following client’s possession will the nurse most
psycho educational program for families of the likely place in a locked area?
chronically mentally ill. The program would be most a. Toothpaste
likely to help the family with which of the following b. Shampoo
issues? c. Antiseptic mouthwash
a. Developing a support network with other d. Moisturizer
families 21. Which of the following assessment would provide
b. Feeling more guilty about the client’s illness the best information about the client’s physiologic
c. Recognizing the client’s weakness response and the effectiveness of the medication
d. Managing their financial concern and problems prescribed specifically for alcohol withdrawal?
16. When planning care for Dory with schizotypal a. Sleeping pattern
personality disorder, which of the following would b. Mental alertness
help the client become involved with others? c. Nutritional status
a. Attending an activity with the nurse d. Vital signs
b. Leading a sing a long in the afternoon 22. After administering naloxone (Narcan), an opioid
c. Participating solely in group activities antagonist, Nurse Ronald should monitor the female
d. Being involved with primarily one to one client carefully for which of the following?
activities a. Respiratory depression
17. Which statement about an individual with a b. Epilepsy
personality disorder is true? c. Kidney failure
a. Psychotic behavior is common during acute d. Cerebral edema
episodes 23. Which of the following would nurse Ronald use as
b. Prognosis for recovery is good with therapeutic the best measure to determine a client’s progress in
intervention rehabilitation?
c. The individual typically remains in the a. The way he gets along with his parents
mainstream of society, although he has problems in b. The number of drug-free days he has
social and occupational roles c. The kinds of friends he makes
d. The individual usually seeks treatment willingly d. The amount of responsibility his job entails
for symptoms that are personally distressful. 24. A female client is brought by ambulance to the
18. Nurse John is talking with a client who has been hospital emergency room after taking an overdose
diagnosed with antisocial personality about how to

50
50
of barbiturates is comatose. Nurse Trish would be d. Take the client a lunch tray and let the client eat
especially alert for which of the following? in his room
a. Epilepsy 30. The initial nursing intervention for the significant-
b. Myocardial Infarction others during shock phase of a grief reaction should
c. Renal failure be focused on:
d. Respiratory failure a. Presenting full reality of the loss of the
25. Joey who has a chronic user of cocaine reports that individuals
he feels like he has cockroaches crawling under his b. Directing the individual’s activities at this time
skin. His arms are red because of scratching. The c. Staying with the individuals involved
nurse in charge interprets these findings as possibly d. Mobilizing the individual’s support system
indicating which of the following? 31. Joy’s stream of consciousness is occupied
a. Delusion exclusively with thoughts of her father’s death.
b. Formication Nurse Ronald should plan to help Joy through this
c. Flash back stage of grieving, which is known as:
d. Confusion a. Shock and disbelief
26. Jose is diagnosed with amphetamine psychosis and b. Developing awareness
was admitted in the emergency room. Nurse Ronald c. Resolving the loss
would most likely prepare to administer which of the d. Restitution
following medication? 32. When taking a health history from a female client
a. Librium who has a moderate level of cognitive impairment
b. Valium due to dementia, the nurse would expect to note
c. Ativan the presence of:
d. Haldol a. Accentuated premorbid traits
27. Which of the following liquids would nurse Leng b. Enhance intelligence
administer to a female client who is intoxicated with c. Increased inhibitions
phencyclidine (PCP) to hasten excretion of the d. Hyper vigilance
chemical? 33. What is the priority care for a client with a dementia
a. Shake resulting from AIDS?
b. Tea a. Planning for remotivational therapy
c. Cranberry Juice b. Arranging for long term custodial care
d. Grape juice c. Providing basic intellectual stimulation
28. When developing a plan of care for a female client d. Assessing pain frequently
with acute stress disorder who lost her sister in a car 34. Jerome who has eating disorder often exhibits similar
accident. Which of the following would the nurse symptoms. Nurse Lhey would expect an adolescent
expect to initiate? client with anorexia to exhibit:
a. Facilitating progressive review of the accident a. Affective instability
and its consequences b. Dishered, unkempt physical appearance
b. Postponing discussion of the accident until the c. Depersonalization and derealization
client brings it up d. Repetitive motor mechanisms
c. Telling the client to avoid details of the accident 35. The primary nursing diagnosis for a female client
d. Helping the client to evaluate her sister’s with a medical diagnosis of major depression would
behavior be:
29. The nursing assistant tells nurse Ronald that the a. Situational low self-esteem related to altered
client is not in the dining room for lunch. Nurse role
Ronald would direct the nursing assistant to do b. Powerlessness related to the loss of idealized
which of the following? self
a. Tell the client he’ll need to wait until supper to c. Spiritual distress related to depression
eat if he misses lunch d. Impaired verbal communication related to
b. Invite the client to lunch and accompany him to depression
the dining room 36. When developing an initial nursing care plan for a
c. Inform the client that he has 10 minutes to get male client with a Bipolar I disorder (manic episode)
to the dining room for lunch nurse Ron should plan to?

50
50
a. Isolate his gym time have turned to glass!” Nurse Ron is aware that this is
b. Encourage his active participation in unit an example of:
programs a. Somatic delusions
c. Provide foods, fluids and rest b. Depersonalization
d. Encourage his participation in programs c. Hypochondriasis
37. Grace is exhibiting withdrawn patterns of behavior. d. Echolalia
Nurse Johnny is aware that this type of behavior 43. In recognizing common behaviors exhibited by
eventually produces feeling of: male client who has a diagnosis of schizophrenia,
a. Repression nurse Josie can anticipate:
b. Loneliness a. Slumped posture, pessimistic out look and flight
c. Anger of ideas
d. Paranoia b. Grandiosity, arrogance and distractibility
38. One morning a female client on the inpatient c. Withdrawal, regressed behavior and lack of
psychiatric service complains to nurse Hazel that she social skills
has been waiting for over an hour for someone to d. Disorientation, forgetfulness and anxiety
accompany her to activities. Nurse Hazel replies to 44. One morning, nurse Diane finds a disturbed client
the client “We’re doing the best we can. There are a curled up in the fetal position in the corner of the
lot of other people on the unit who needs attention dayroom. The most accurate initial evaluation of the
too.” This statement shows that the nurse’s use of: behavior would be that the client is:
a. Defensive behavior a. Physically ill and experiencing abdominal
b. Reality reinforcement discomfort
c. Limit-setting behavior b. Tired and probably did not sleep well last night
d. Impulse control c. Attempting to hide from the nurse
39. A nursing diagnosis for a male client with a d. Feeling more anxious today
diagnosed multiple personality disorder is chronic 45. Nurse Bea notices a female client sitting alone in the
low self-esteem probably related to childhood corner smiling and talking to herself. Realizing that
abuse. The most appropriate short term client the client is hallucinating. Nurse Bea should:
outcome would be: a. Invite the client to help decorate the dayroom
a. Verbalizing the need for anxiety medications b. Leave the client alone until he stops talking
b. Recognizing each existing personality c. Ask the client why he is smiling and talking
c. Engaging in object-oriented activities d. Tell the client it is not good for him to talk
d. Eliminating defense mechanisms and phobia to himself
40. A 25 year old male is admitted to a mental health 46. When being admitted to a mental health facility, a
facility because of inappropriate behavior. The client young female adult tells Nurse Mylene that the
has been hearing voices, responding to imaginary voices she hears frighten her. Nurse Mylene
companions and withdrawing to his room for several understands that the client tends to hallucinate
days at a time. Nurse Monette understands that the more vividly:
withdrawal is a defense against the client’s fear of: a. While watching TV
a. Phobia b. During meal time
b. Powerlessness c. During group activities
c. Punishment d. After going to bed
d. Rejection 47. Nurse John recognizes that paranoid delusions
41. When asking the parents about the onset of usually are related to the defense mechanism of:
problems in young client with the diagnosis of a. Projection
schizophrenia, Nurse Linda would expect that they b. Identification
would relate the client’s difficulties began in: c. Repression
a. Early childhood d. Regression
b. Late childhood 48. When planning care for a male client using paranoid
c. Adolescence ideation, nurse Jasmin should realize the importance
d. Puberty of:
42. Jose who has been hospitalized with schizophrenia a. Giving the client difficult tasks to provide
tells Nurse Ron, “My heart has stopped and my veins stimulation

50
50
b. Providing the client with activities in which
success can be achieved
c. Removing stress so that the client can relax
d. Not placing any demands on the client
49. Nurse Gerry is aware that the defense
mechanism commonly used by clients who are
alcoholics is:
a. Displacement
b. Denial
c. Projection
d. Compensation
50. Within a few hours of alcohol withdrawal, nurse John
should assess the male client for the presence of:
a. Disorientation, paranoia, tachycardia
b. Tremors, fever, profuse diaphoresis
c. Irritability, heightened alertness, jerky
movements
d. Yawning, anxiety, convulsions

50
51
ANSWERS and RATIONALES for PSYCHIATRIC (Haldol) is benztropine mesylate (cogentin)
NURSING Part 2 because of its anti cholinergic properties.
13. D. Allowing the client to be the first to open the
1. C. When the nurse and client agree to work cart & take a tray presents the client with the
together, a contract should be established, the reality that the nurses are not touching the food
length of the relationship should be discussed in & tray, thereby dispelling the delusion.
terms of its ultimate termination. 14. B. Although all the actions indicate
2. B. The nurse should initiate brief, frequent improvement, the ability to initiate simple
contacts throughout the day to let the client activities without directions indicates the most
know that he is important to the nurse. This will improvement in the catatonic behaviors.
positively affect the client’s self-esteem. 15. A. Psychoeducational groups for families
3. D. The client with depression is preoccupied, has develop a support network. They provide
decreased energy, and is unable to make education about the biochemical etiology of
decisions. The nurse presents the situation, “It’s psychiatric disease to reduce, not increase family
time for a shower”, and assists the client with guilt.
personal hygiene to preserve his dignity and self- 16. C. Attending activity with the nurse assists the
esteem. client to become involved with others slowly.
4. C. Foods high in tyramine, those that are The client with schizotypal personality disorder
fermented, pickled, aged, or smoked must be needs support, kindness & gentle suggestion to
avoided because when they are ingested in improve social skills & interpersonal relationship.
combination with MAOIs a hypertensive crisis 17. C. An individual with personality disorder usually
will occur. is not hospitalized unless a coexisting Axis I
5. A. Anticholinergic effects, which result from psychiatric disorder is present. Generally, these
blockage of the parasympathetic (craniosacral) individuals make marginal adjustments and
nervous system including urine retention, remain in society, although they typically
blurred vision, dry mouth & constipation. experience relationship and occupational
6. B. Dysthymia is a less severe, chronic depression problems related to their inflexible behaviors.
diagnosed when a client has had a depressed Personality disorders are chronic lifelong
mood for more days than not over a period of at patterns of behavior; acute episodes do not
least 2 years. Client with dysthymic disorder occur. Psychotic behavior is usually not common,
benefit from psychotherapeutic approaches that although it can occur in either schizotypal
assist the client in reversing the negative self personality disorder or borderline personality
image, negative feelings about the future. disorder. Because these disorders are enduring
7. D. Flight of ideas is speech pattern of rapid and evasive and the individual is inflexible,
transition from topic to topic, often without prognosis for recovery is unfavorable. Generally,
finishing one idea. It is common in mania. the individual does not seek treatment because
8. B. The client with mania is very active & needs to he does not perceive problems with his own
have this energy channeled in a constructive task behavior. Distress can occur based on other
such as cleaning or tidying the room. people’s reaction to the individual’s behavior.
9. C. A crucial factor is determining the lethality of 18. D. The nurse would explain the negative
a method is the amount of time that occurs reactions of others towards the client’s
between initiating the method & the delivery of behaviors to make the clients aware of the
the lethal impact of the method. impact of his seductive behaviors on others.
10. D. The statement “I don’t think about killing 19. B. The nurse would use role-playing to teach the
myself as much as I used to.” Indicates a client appropriate responses to others and in
lessening of suicidal ideation and improvement various situations. This client dramatizes events,
in the client’s condition. drawn attention to self, and is unaware of and
11. A. Using exercise bicycle is appropriate for the does not deal with feelings. The nurse works to
client who becomes very anxious when thoughts help the client clarify true feelings & learn to
of suicidal occur. express them appropriately.
12. C. The drug of choice for a client experiencing 20. C. Antiseptic mouthwash often contains alcohol
extra pyramidal side effects from haloperidol & should be kept in locked area, unless labeling

51
51
clearly indicates that the product does not dependence on environment & social structure
contain alcohol. and by increasing psychologic rigidity with
21. D. Monitoring of vital signs provides the best accentuated previous traits & behaviors.
information about the client’s overall physiologic 33. C. This action maintains for as long as possible,
status during alcohol withdrawal & the the clients intellectual functions by providing an
physiologic response to the medication used. opportunity to use them.
22. A. After administering naloxone (Narcan) the 34. A. Individuals with anorexia often display
nurse should monitor the client’s respiratory irritability, hospitality, and a depressed mood.
status carefully, because the drug is short acting 35. D. Depressed clients demonstrate decreased
& respiratory depression may recur after its communication because of lack of psychic or
effects wear off. physical energy.
23. B. The best measure to determine a client’s 36. C. The client in a manic episode of the illness
progress in rehabilitation is the number of drug- often neglects basic needs, these needs are a
free days he has. The longer the client is free of priority to ensure adequate nutrition, fluid, and
drugs, the better the prognosis is. rest.
24. D. Barbiturates are CNS depressants; the nurse 37. B. The withdrawn pattern of behavior presents
would be especially alert for the possibility of the individual from reaching out to others for
respiratory failure. Respiratory failure is the sharing the isolation produces feeling of
most likely cause of death from barbiturate over loneliness.
dose. 38. A. The nurse’s response is not therapeutic
25. B. The feeling of bugs crawling under the skin is because it does not recognize the client’s needs
termed as formication, and is associated with but tries to make the client feel guilty for being
cocaine use. demanding.
26. D. The nurse would prepare to administer an 39. B. The client must recognize the existence of
antipsychotic medication such as Haldol to a the sub personalities so that interpretation can
client experiencing amphetamine psychosis to occur.
decrease agitation & psychotic symptoms, 40. D. An aloof, detached, withdrawn posture is a
including delusions, hallucinations & cognitive means of protecting the self by withdrawing and
impairment. maintaining a safe, emotional distance.
27. C. An acid environment aids in the excretion of 41. C. The usual age of onset of schizophrenia is
PCP. The nurse will definitely give the client with adolescence or early childhood.
PCP intoxication cranberry juice to acidify the 42. A. Somatic delusion is a fixed false belief about
urine to a ph of 5.5 & accelerate excretion. one’s body.
28. A. The nurse would facilitate progressive review 43. C. These are the classic behaviors exhibited by
of the accident and its consequence to help the clients with a diagnosis of schizophrenia.
client integrate feelings & memories and to 44. D. The fetal position represents regressed
begin the grieving process. behavior. Regression is a way of responding to
29. B. The nurse instructs the nursing assistant to overwhelming anxiety.
invite the client to lunch & accompany him to 45. B. This provides a stimulus that competes with
the dinning room to decrease manipulation, and reduces hallucination.
secondary gain, dependency and reinforcement 46. D. Auditory hallucinations are most troublesome
of negative behavior while maintaining the when environmental stimuli are diminished and
client’s worth. there are few competing distractions.
30. C. This provides support until the individuals 47. A. Projection is a mechanism in which inner
coping mechanisms and personal support thoughts and feelings are projected onto the
systems can be immobilized. environment, seeming to come from outside the
31. C. Resolving a loss is a slow, painful, continuous self rather than from within.
process until a mental image of the dead person, 48. B. This will help the client develop self-esteem
almost devoid of negative or undesirable and reduce the use of paranoid ideation.
features emerges. 49. B. Denial is a method of resolving conflict or
32. A. A moderate level of cognitive impairment due escaping unpleasant realities by ignoring their
to dementia is characterized by increasing existence.

51
51
50. C. Alcohol is a central nervous system
depressant. These symptoms are the body’s
neurologic adaptation to the withdrawal of
alcohol.

51
51
PSYCHIATRIC NURSING Part 3 a. Projection
b. Displacement
1. Francis who is addicted to cocaine withdraws from the c. Denial
drug. Nurse Ron should expect to observe: d. Reaction formation
a. Hyperactivity 8. The most critical factor for nurse Linda to
b. Depression determine during crisis intervention would be the
c. Suspicion client’s:
d. Delirium a. Available situational supports
2. Nurse John is aware that a serious effect of inhaling b. Willingness to restructure the personality
cocaine is? c. Developmental theory
a. Deterioration of nasal septum d. Underlying unconscious conflict
b. Acute fluid and electrolyte imbalances 9. Nurse Trish suggests a crisis intervention group to a
c. Extra pyramidal tract symptoms client experiencing a developmental crisis. These
d. Esophageal varices groups are successful because the:
3. A tentative diagnosis of opiate addiction, Nurse Candy a. Crisis intervention worker is a psychologist
should assess a recently hospitalized client for signs and understands behavior patterns
of opiate withdrawal. These signs would include: b. Crisis group supplies a workable solution to
a. Rhinorrhea, convulsions, subnormal the client’s problem
temperature c. Client is encouraged to talk about
b. Nausea, dilated pupils, constipation personal problems
c. Lacrimation, vomiting, drowsiness d. Client is assisted to investigate alternative
d. Muscle aches, papillary constriction, yawning approaches to solving the identified problem
4. A 48 year old male client is brought to the psychiatric 10. Nurse Ronald could evaluate that the staff’s
emergency room after attempting to jump off a approach to setting limits for a demanding, angry
bridge. The client’s wife states that he lost his job client was effective if the client:
several months ago and has been unable to find a. Apologizes for disrupting the unit’s routine
another job. The primary nursing intervention at this when something is needed
time would be to assess for: b. Understands the reason why frequent calls to
a. A past history of depression the staff were made
b. Current plans to commit suicide c. Discuss concerns regarding the emotional
c. The presence of marital difficulties condition that required hospitalizations
d. Feelings of excessive failure d. No longer calls the nursing staff for assistance
5. Before helping a male client who has been sexually 11. Nurse John is aware that the therapy that has the
assaulted, nurse Maureen should recognize that the highest success rate for people with phobias would
rapist is motivated by feelings of: be:
a. Hostility a. Psychotherapy aimed at rearranging maladaptive
b. Inadequacy thought process
c. Incompetence b. Psychoanalytical exploration of repressed conflicts
d. Passion of an earlier development phase
6. When working with children who have been sexually c. Systematic desensitization using relaxation
abused by a family member it is important for the technique
nurse to understand that these victims usually are d. Insight therapy to determine the origin of the
overwhelmed with feelings of: anxiety and fear
a. Humiliation 12. When nurse Hazel considers a client’s placement on
b. Confusion the continuum of anxiety, a key in determining the
c. Self blame degree of anxiety being experienced is the client’s:
d. Hatred a. Perceptual field
7. Joy who has just experienced her second spontaneous b. Delusional system
abortion expresses anger towards her physician, the c. Memory state
hospital and the “rotten nursing care”. When d. Creativity level
assessing the situation, the nurse recognizes that the 13. In the diagnosis of a possible pervasive
client may be using the coping mechanism of: developmental autistic disorder. The nurse would

51
51
find it most unusual for a 3 year old child to d. Decreased respiratory rate
demonstrate: 20. Initial interventions for Marco with acute anxiety
a. An interest in music include all except which of the following?
b. An attachment to odd objects a. Touching the client in an attempt to comfort him
c. Ritualistic behavior b. Approaching the client in calm, confident manner
d. Responsiveness to the parents c. Encouraging the client to verbalize feelings and
14. Malou with schizophrenia tells Nurse Melinda, “My concerns
intestines are rotted from worms chewing on them.” d. Providing the client with a safe, quiet and private
This statement indicates a: place
a. Jealous delusion 21. Nurse Jessie is assessing a client suffering from stress
b. Somatic delusion and anxiety. A common physiological response to
c. Delusion of grandeur stress and anxiety is:
d. Delusion of persecution a. Uticaria
15. Andy is admitted to the psychiatric unit with a b. Vertigo
diagnosis of borderline personality disorder. Nurse c. Sedation
Hilary should expects the assessment to reveal: d. Diarrhea
a. Coldness, detachment and lack of tender feelings 22. When performing a physical examination on a female
b. Somatic symptoms anxious client, nurse Nelli would expect to find
c. Inability to function as responsible parent which of the following effects produced by the
d. Unpredictable behavior and intense interpersonal parasympathetic system?
relationships a. Muscle tension
16. PROPRANOLOL (Inderal) is used in the mental health b. Hyperactive bowel sounds
setting to manage which of the following conditions? c. Decreased urine output
a. Antipsychotic – induced akathisia and anxiety d. Constipation
b. Obsessive – compulsive disorder (OCD) to reduce 23. Which of the following drugs have been known to be
ritualistic behavior effective in treating obsessive-compulsive disorder
c. Delusions for clients suffering from schizophrenia (OCD)?
d. The manic phase of bipolar illness as a mood a. Divalproex (depakote) and Lithium (lithobid)
stabilizer b. Chlordiazepoxide (Librium) and diazepam (valium)
17. Which medication can control the extra pyramidal c. Fluvoxamine (Luvox) and clomipramine (anafranil)
effects associated with antipsychotic agents? d. Benztropine (Cogentin) and diphenhydramine
a. Clorazepate (Tranxene) (benadryl)
b. Amantadine (Symmetrel) 24. Tony with agoraphobia has been symptom-free for 4
c. Doxepin (Sinequan) months. Classic signs and symptoms of phobia
d. Perphenazine (Trilafon) include:
18. Which of the following statements should be a. Severe anxiety and fear
included when teaching clients about monoamine b. Withdrawal and failure to distinguish reality from
oxidase inhibitor (MAOI) antidepressants? fantasy
a. Don’t take aspirin or nonsteroidal anti- c. Depression and weight loss
inflammatory drugs (NSAIDs) d. Insomnia and inability to concentrate
b. Have blood levels screened weekly for leucopenia 25. Which nursing action is most appropriate when
c. Avoid strenuous activity because of the cardiac trying to diffuse a client’s impending violent
effects of the drug behavior?
d. Don’t take prescribed or over the counter a. Place the client in seclusion
medications without consulting the physician b. Leaving the client alone until he can talk about his
19. Kris periodically has acute panic attacks. These feelings
attacks are unpredictable and have no apparent c. Involving the client in a quiet activity to divert
association with a specific object or situation. During attention
an acute panic attack, Kris may experience: d. Helping the client identify and express feelings of
a. Heightened concentration anxiety and anger
b. Decreased perceptual field
c. Decreased cardiac rate

51
51
26. Rosana is in the second stage of Alzheimer’s disease b. Don’t consume alcohol
who appears to be in pain. Which question by Nurse c. Discontinue if dry mouth and blurred vision occur
Jenny would best elicit information about the pain? d. Restrict fluid and sodium intake
a. “Where is your pain located?” 33. Important teaching for women in their
b. “Do you hurt? (pause) “Do you hurt?” childbearing years who are receiving antipsychotic
c. “Can you describe your pain?” medications includes which of the following?
d. “Where do you hurt?” a. Increased incidence of dysmenorrhea while taking
27. Nursing preparation for a client undergoing the drug
electroconvulsive therapy (ECT) resemble those used b. Occurrence of incomplete libido due to
for: medication adverse effects
a. General anesthesia c. Continuing previous use of contraception during
b. Cardiac stress testing periods of amenorrhea
c. Neurologic examination d. Instruction that amenorrhea is irreversible
d. Physical therapy 34. A client refuses to remain on psychotropic
28. Jose who is receiving monoamine oxidase inhibitor medications after discharge from an inpatient
antidepressant should avoid tyramine, a compound psychiatric unit. Which information should the
found in which of the following foods? community health nurse assess first during the initial
a. Figs and cream cheese follow-up with this client?
b. Fruits and yellow vegetables a. Income level and living arrangements
c. Aged cheese and Chianti wine b. Involvement of family and support systems
d. Green leafy vegetables c. Reason for inpatient admission
29. Erlinda, age 85, with major depression d. Reason for refusal to take medications
undergoes a sixth electroconvulsive therapy (ECT) 35. The nurse understands that the therapeutic effects
treatment. When assessing the client immediately of typical antipsychotic medications are associated
after ECT, the nurse expects to find: with which neurotransmitter change?
a. Permanent short-term memory loss and a. Decreased dopamine level
hypertension b. Increased acetylcholine level
b. Permanent long-term memory loss and c. Stabilization of serotonin
hypomania d. Stimulation of GABA
c. Transitory short-term memory loss and permanent 36. Which of the following best explains why tricyclic
long-term memory loss antidepressants are used with caution in elderly
d. Transitory short and long term memory loss and patients?
confusion a. Central Nervous System effects
30. Barbara with bipolar disorder is being treated with b. Cardiovascular system effects
lithium for the first time. Nurse Clint should observe c. Gastrointestinal system effects
the client for which common adverse effect of d. Serotonin syndrome effects
lithium? 37. A client with depressive symptoms is given
a. Polyuria prescribed medications and talks with his therapist
b. Seizures about his belief that he is worthless and unable to
c. Constipation cope with life. Psychiatric care in this treatment plan
d. Sexual dysfunction is based on which framework?
31. Nurse Fred is assessing a client who has just been a. Behavioral framework
admitted to the ER department. Which signs would b. Cognitive framework
suggest an overdose of an antianxiety agent? c. Interpersonal framework
a. Suspiciousness, dilated pupils and incomplete BP d. Psychodynamic framework
b. Agitation, hyperactivity and grandiose ideation 38. A nurse who explains that a client’s psychotic
c. Combativeness, sweating and confusion behavior is unconsciously motivated understands
d. Emotional lability, euphoria and impaired memory that the client’s disordered behavior arises from
32. Discharge instructions for a male client receiving which of the following?
tricyclic antidepressants include which of the a. Abnormal thinking
following information? b. Altered neurotransmitters
a. Restrict fluids and sodium intake c. Internal needs

51
51
d. Response to stimuli 45. Which nursing diagnosis is most appropriate for a
39. A client with depression has been hospitalized for client with anorexia nervosa who expresses feelings
treatment after taking a leave of absence from work. of guilt about not meeting family expectations?
The client’s employer expects the client to return to a. Anxiety
work following inpatient treatment. The client tells b. Disturbed body image
the nurse, “I’m no good. I’m a failure”. According to c. Defensive coping
cognitive theory, these statements reflect: d. Powerlessness
a. Learned behavior 46. A nurse is evaluating therapy with the family of a
b. Punitive superego and decreased self-esteem client with anorexia nervosa. Which of the following
c. Faulty thought processes that govern behavior would indicate that the therapy was successful?
d. Evidence of difficult relationships in the work a. The parents reinforced increased decision
environment making by the client
40. The nurse describes a client as anxious. Which of the b. The parents clearly verbalize their expectations
following statement about anxiety is true? for the client
a. Anxiety is usually pathological c. The client verbalizes that family meals are now
b. Anxiety is directly observable enjoyable
c. Anxiety is usually harmful d. The client tells her parents about feelings of low-
d. Anxiety is a response to a threat self esteem
41. A client with a phobic disorder is treated by 47. A client with dysthymic disorder reports to a nurse
systematic desensitization. The nurse understands that his life is hopeless and will never improve in
that this approach will do which of the following? the future. How can the nurse best respond using a
a. Help the client execute actions that are feared cognitive approach?
b. Help the client develop insight into irrational fears a. Agree with the client’s painful feelings
c. Help the client substitutes one fear for another b. Challenge the accuracy of the client’s belief
d. Help the client decrease anxiety c. Deny that the situation is hopeless
42. Which client outcome would best indicate successful d. Present a cheerful attitude
treatment for a client with an antisocial personality 48. A client with major depression has not verbalized
disorder? problem areas to staff or peers since admission to a
a. The client exhibits charming behavior when psychiatric unit. Which activity should the nurse
around authority figures recommend to help this client express himself?
b. The client has decreased episodes of impulsive a. Art therapy in a small group
behaviors b. Basketball game with peers on the unit
c. The client makes statements of self-satisfaction c. Reading a self-help book on depression
d. The client’s statements indicate no remorse for d. Watching movie with the peer group
behaviors 49. The home health psychiatric nurse visits a client with
43. The nurse is caring for a client with an autoimmune chronic schizophrenia who was recently discharged
disorder at a medical clinic, where alternative after a prolong stay in a state hospital. The client
medicine is used as an adjunct to traditional lives in a boarding home, reports no family
therapies. Which information should the nurse teach involvement, and has little social interaction. The
the client to help foster a sense of control over his nurse plan to refer the client to a day treatment
symptoms? program in order to help him with:
a. Pathophysiology of disease process a. Managing his hallucinations
b. Principles of good nutrition b. Medication teaching
c. Side effects of medications c. Social skills training
d. Stress management techniques d. Vocational training
44. Which of the following is the most distinguishing 50. Which activity would be most appropriate for a
feature of a client with an antisocial personality severely withdrawn client?
disorder? a. Art activity with a staff member
a. Attention to detail and order b. Board game with a small group of clients
b. Bizarre mannerisms and thoughts c. Team sport in the gym
c. Submissive and dependent behavior d. Watching TV in the dayroom
d. Disregard for social and legal norms

51
51
ANSWERS and RATIONALES for PSYCHIATRIC 16. A. Propranolol is a potent beta adrenergic blocker
NURSING Part 3 and producing a sedating effect, therefore it is used
to treat antipsychotic induced akathisia and anxiety.
1. B. There is no set of symptoms associated with 17. B. Amantadine is an anticholinergic drug used to
cocaine withdrawal, only the depression that follows relive drug-induced extra pyramidal adverse effects
the high caused by the drug. such as muscle weakness, involuntary muscle
2. A. Cocaine is a chemical that when inhaled, causes movements, pseudoparkinsonism and tar dive
destruction of the mucous membranes of the nose. dyskinesia.
3. D. These adaptations are associated with opiate 18. C. MAOI antidepressants when combined with a
withdrawal which occurs after cessation or reduction number of drugs can cause life-threatening
of prolonged moderate or heavy use of opiates. hypertensive crisis. It’s imperative that a client
4. B. Whether there is a suicide plan is a criterion when checks with his physician and pharmacist before
assessing the client’s determination to make another taking any other medications.
attempt. 19. B. Panic is the most severe level of anxiety. During
5. A. Rapists are believed to harbor and act out hostile panic attack, the client experiences a decrease in the
feelings toward all women through the act of rape. perceptual field, becoming more focused on self,
6. C. These children often have nonsexual needs met less aware of surroundings and unable to process
by individual and are powerless to refuse. information from the environment. The decreased
Ambivalence results in self-blame and also guilt. perceptual field contributes to impaired attention
7. B. The client’s anger over the abortion is shifted to and inability to concentrate.
the staff and the hospital because she is unable to 20. A. The emergency nurse must establish rapport and
deal with the abortion at this time. trust with the anxious client before using therapeutic
8. A. Personal internal strength and supportive touch. Touching an anxious client may actually
individuals are critical factors that can be employed increase anxiety.
to assist the individual to cope with a crisis. 21. D. Diarrhea is a common physiological response to
9. D. Crisis intervention group helps client reestablish stress and anxiety.
psychologic equilibrium by assisting them to explore 22. B. The parasympathetic nervous system would
new alternatives for coping. It considers realistic produce incomplete G.I. motility resulting in
situations using rational and flexible problem solving hyperactive bowel sounds, possibly leading to
methods. diarrhea.
10. C. This would document that the client feels 23. C. The antidepressants fluvoxamine and
comfortable enough to discuss the problems that clomipramine have been effective in the treatment
have motivated the behavior. of OCD.
11. C. The most successful therapy for people with 24. A. Phobias cause severe anxiety (such as panic
phobias involves behavior modification techniques attack) that is out of proportion to the threat of the
using desensitization. feared object or situation. Physical signs and
12. A. Perceptual field is a key indicator of anxiety level symptoms of phobias include profuse sweating, poor
because the perceptual fields narrow as anxiety motor control, tachycardia and elevated B.P.
increases. 25. D. In many instances, the nurse can diffuse
13. D. One of the symptoms of autistic child displays a impending violence by helping the client identify and
lack of responsiveness to others. There is little or no express feelings of anger and anxiety. Such
extension to the external environment. statement as “What happened to get you this
14. B. Somatic delusions focus on bodily functions or angry?” may help the client verbalizes feelings rather
systems and commonly include delusion about foul than act on them.
odor emissions, insect manifestations, internal 26. B. When speaking to a client with Alzheimer’s
parasites and misshapen parts. disease, the nurse should use close-ended questions.
15. D. A client with borderline personality displays a Those that the client can answer with “yes” or “no”
pervasive pattern of unpredictable behavior, mood whenever possible and avoid questions that require
and self image. Interpersonal relationships may be the client to make choices. Repeating the question
intense and unstable and behavior may be aids comprehension.
inappropriate and impulsive. 27. A. The nurse should prepare a client for ECT in a
manner similar to that for general anesthesia.

51
51
28. C. Aged cheese and Chianti wine contain high Using medications to alter neurotransmitter activity
concentrations of tyramine. is a psychobiologic approach to treatment. The other
29. D. ECT commonly causes transitory short and long answer choices are frameworks for care, but hey are
term memory loss and confusion, especially in not applicable to this situation.
geriatric clients. It rarely results in permanent short 38. C. The concept that behavior is motivated and has
and long term memory loss. meaning comes from the psychodynamic
30. A. Polyuria commonly occurs early in the treatment framework. According to this perspective, behavior
with lithium and could result in fluid volume deficit. arises from internal wishes or needs. Much of what
31. D. Signs of anxiety agent overdose include emotional motivates behavior comes from the unconscious.
lability, euphoria and impaired memory. The remaining responses do not address the internal
32. B. Drinking alcohol can potentiate the sedating forces thought to motivate behavior.
action of tricyclic antidepressants. Dry mouth and 39. C. The client is demonstrating faulty thought
blurred vision are normal adverse effects of tricyclic processes that are negative and that govern his
antidepressants. behavior in his work situation – issues that are
33. C. Women may experience amenorrhea, which is typically examined using a cognitive theory
reversible, while taking antipsychotics. Amenorrhea approach. Issues involving learned behavior are best
doesn’t indicate cessation of ovulation thus, the explored through behavior theory, not cognitive
client can still be pregnant. theory. Issues involving ego development are the
34. D. The first are for assessment would be the client’s focus of psychoanalytic theory. Option 4 is incorrect
reason for refusing medication. The client may not because there is no evidence in this situation that
understand the purpose for the medication, may be the client has conflictual relationships in the work
experiencing distressing side effects, or may be environment.
concerned about the cost of medicine. In any case, 40. D. Anxiety is a response to a threat arising from
the nurse cannot provide appropriate intervention internal or external stimuli.
before assessing the client’s problem with the 41. A. Systematic desensitization is a behavioral therapy
medication. The patient’s income level, living technique that helps clients with irrational fears and
arrangements, and involvement of family and avoidance behavior to face the thing they fear,
support systems are relevant issues following without experiencing anxiety. There is no attempt to
determination of the client’s reason for refusing promote insight with this procedure, and the client
medication. The nurse providing follow-up care will not be taught to substitute one fear for another.
would have access to the client’s medical record and Although the client’s anxiety may decrease with
should already know the reason for inpatient successful confrontation of irrational fears, the
admission. purpose of the procedure is specifically related to
35. A. Excess dopamine is thought to be the chemical performing activities that typically are avoided as
cause for psychotic thinking. The typical part of the phobic response.
antipsychotics act to block dopamine receptors and 42. B. A client with antisocial personality disorder
therefore decrease the amount of neurotransmitter typically has frequent episodes of acting impulsively
at the synapses. The typical antipsychotics do not with poor ability to delay self-gratification.
increase acetylcholine, stabilize serotonin, stimulate Therefore, decreased frequency of impulsive
GABA. behaviors would be evidence of improvement.
36. B. The TCAs affect norepinephrine as well as other Charming behavior when around authority figures
neurotransmitters, and thus have significant and statements indicating no remorse are examples
cardiovascular side effects. Therefore, they are used of symptoms typical of someone with this disorder
with caution in elderly clients who may have and would not indicate successful treatment. Self-
increased risk factors for cardiac problems because satisfaction would be viewed as a positive change if
of their age and other medical conditions. The the client expresses low self-esteem; however this is
remaining side effects would apply to any client not a characteristic of a client with antisocial
taking a TCA and are not particular to an elderly personality disorder.
person. 43. D. In autoimmune disorders, stress and the response
37. B. Cognitive thinking therapy focuses on the client’s to stress can exacerbate symptoms. Stress
misperceptions about self, others and the world that management techniques can help the client reduce
impact functioning and contribute to symptoms. the psychological response to stress, which in turn

51
51
will help reduce the physiologic stress response. This 49. C. Day treatment programs provide clients with
will afford the client an increased sense of control chronic, persistent mental illness training in social
over his symptoms. The nurse can address the skills, such as meeting and greeting people, asking
remaining answer choices in her teaching about the questions or directions, placing an order in a
client’s disease and treatment; however, knowledge restaurant, taking turns in a group setting activity.
alone will not help the client to manage his stress Although management of hallucinations and
effectively enough to control symptoms. medication teaching may also be part of the
44. D. Disregard for established rules of society is the program offered in a day treatment, the nurse is
most common characteristic of a client with referring the client in this situation because of his
antisocial personality disorder. Attention to detail need for socialization skills. Vocational training
and order is characteristic of someone with generally takes place in a rehabilitation facility; the
obsessive compulsive disorder. Bizarre mannerisms client described in this situation would not be a
and thoughts are characteristics of a client with candidate for this service.
schizoid or schizotypal disorder. Submissive and 50. A. The best approach with a withdrawn client is to
dependent behaviors are characteristic of someone initiate brief, nondemanding activities on a one-to-
with a dependent personality. one basis. This approach gives the nurse an
45. D. The client with anorexia typically feels powerless, opportunity to establish a trusting relationship with
with a sense of having little control over any aspect the client. A board game with a group clients or
of life besides eating behavior. Often, parental playing a team sport in the gym may overwhelm a
expectations and standards are quite high and lead severely withdrawn client. Watching TV is a solitary
to the clients’ sense of guilt over not measuring up. activity that will reinforce the client’s withdrawal
46. A. One of the core issues concerning the family of a from others.
client with anorexia is control. The family’s
acceptance of the client’s ability to make
independent decisions is key to successful family
intervention. Although the remaining options may
occur during the process of therapy, they would not
necessarily indicate a successful outcome; the
central family issues of dependence and
independence are not addresses on these responses.
47. B. Use of cognitive techniques allows the nurse to
help the client recognize that this negative beliefs
may be distortions and that, by changing his
thinking, he can adopt more positive beliefs that are
realistic and hopeful. Agreeing with the client’s
feelings and presenting a cheerful attitude are not
consistent with a cognitive approach and would not
be helpful in this situation. Denying the client’s
feelings is belittling and may convey that the nurse
does not understand the depth of the client’s
distress.
48. A. Art therapy provides a nonthreatening vehicle for
the expression of feelings, and use of a small group
will help the client become comfortable with peers
in a group setting. Basketball is a competitive game
that requires energy; the client with major
depression is not likely to participate in this activity.
Recommending that the client read a self-help book
may increase, not decrease his isolation. Watching
movie with a peer group does not guarantee that
interaction will occur; therefore, the client may
remain isolated.

51
52
PROFESSIONAL ADJUSTMENT in both government and private hospitals
D. Income tax which paid every March 15 and
1. A nurse who would like to practice nursing in the professional tax which is paid every January 31.
Philippines can obtain a license to practice by:
A. Paying the professional tax after taking the board Answer: (B) Income tax only since they are exempt from
exams paying professional tax
B. Passing the board exams and taking the oath of According to the Magna Carta for Public Health Workers,
professionals government nurses are exempted from paying
C. Paying the examination fee before taking the board professional tax. Hence, as an employee in the
exams government, s/he will pay only the income tax.
D. Undergoing the interview conducted by the Board
of Nursing and taking the board exams 4. According to RA 9173 Philippine Nursing Act of 2002, a
graduate nurse who wants to take must licensure
Answer: (B) Passing the board exams and taking the oath examination must comply with the following
of professionals qualifications:
For a nurse to obtain a license to practice nursing in the A. At least 21 years old, graduate of BSN from a
Philippines, s/he must pass the board examinations and recognized school, and of good moral character
then take the oath of professionals before the Board of B. At least 18 years old, graduate of BSN from
Nursing. a recognized school and of good moral
character
2. Reciprocity of license to practice requires that the C. At least 18 years old, provided that when s/he passes
country of origin of the interested foreign nurse the board exams, s/he must be at least 21 years old;
complies with the following conditions: BSN graduate of a recognized school, and of good moral
A. The country of origin has similar preparation for a character
nurse and has laws allowing Filipino nurses to practice in D. Filipino citizen or a citizen of a country where we have
their country. reciprocity; graduate of BSN from a recognized school
B. The Philippines is recognized by the country of origin and of good moral character
as one that has high quality of nursing education
C. The country of origin requires Filipinos to take their Answer: (D) Filipino citizen or a citizen of a country
own board examination where we have reciprocity; graduate of BSN from a
D. The country of origin exempts Filipinos from recognized school and of good moral character
passing their licensure examination RA 9173 section 13 states that the qualifications to take
the board exams are: Filipino citizen or citizen of a
Answer: (A) The country of origin has similar preparation country where the Philippines has reciprocity; of good
for a nurse and has laws allowing Filipino nurses to moral character and graduate of BSN from a recognized
practice in their country. school of nursing. There is no explicit provision about the
According to the Philippine Nurses Act of 2002, foreign age requirement in RA 9173 unlike in RA7164 (old law).
nurses wanting to practice in the Philippines must show
proof that his/her country of origin meets the two 5. Which of the following is TRUE about membership
essential conditions: a) the requirements for registration to the Philippine Nurses Association (PNA)?
between the two countries are substantially the same; A. Membership to PNA is mandatory and is stipulated in
and b) the country of origin of the foreign nurse has laws the Philippine Nursing Act of 2002
allowing the Filipino nurse to practice in his/her country B. Membership to PNA is compulsory for newly
just like its own citizens. registered nurses wanting to enter the practice of
nursing in the country
3. Nurses practicing the profession in the Philippines C. Membership to PNA is voluntary and is encouraged by
and are employed in government hospitals are required the PRC Code of Ethics for Nurses
to pay taxes such as: D. Membership to PNA is required by government
A. Both income tax and professional tax hospitals prior to employment
B. Income tax only since they are exempt from paying
professional tax Answer: (C) Membership to PNA is voluntary and is
C. Professional tax which is paid by all nurses employed encouraged by the PRC Code of Ethics for Nurses
Membership to any organization, including the PNA, is
52
52
only voluntary and this right to join any organization is Answer: (A) BSN with at least 9 units of post graduate
guaranteed in the 1987 constitution of the Philippines. studies in nursing administration
However, the PRC Code of Ethics states that one of the According to RA 9173 sec. 29, the educational
ethical obligations of the professional nurse towards the qualification to be a supervisor in a hospital is at least 9
profession is to be an active member of the accredited units of postgraduate studies in nursing administration.
professional organization. A masters degree in nursing is required for the chief
nurse of a secondary or tertiary hospital.
6. When the license of the nurse is revoked, it means
that the nurse: 9. The Board of Nursing has quasi-judicial power. An
A. Is no longer allowed to practice the profession for the example of this power is:
rest of her life A. The Board can issue rules and regulations that will
B. Will never have her/his license re-issued since it has govern the practice of nursing
been revoked B. The Board can investigate violations of the nursing law
C. May apply for re-issuance of his/her license based on and code of ethics
certain conditions stipulated in RA 9173 C. The Board can visit a school applying for a permit in
D. Will remain unable to practice professional nursing collaboration with CHED
D. The Board prepares the board examinations
Answer: (C) May apply for re-issuance of his/her license
based on certain conditions stipulated in RA 9173 Answer: (B) The Board can investigate violations of the
RA 9173 sec. 24 states that for equity and justice, a nursing law and code of ethics
revoked license maybe re-issued provided that the Quasi-judicial power means that the Board of Nursing
following conditions are met: a) the cause for revocation has the authority to investigate violations of the nursing
of license has already been corrected or removed; and, law and can issue summons, subpoena or subpoena
b) at least four years has elapsed since the license has duces tecum as needed.
been revoked.
10. When a nurse causes an injury to the patient and the
7. According to the current nursing law, the minimum injury caused becomes the proof of the negligent act,
educational qualification for a faculty member of a the presence of the injury is said to exemplify the
college of nursing is: principle of:
A. Only a Master of Arts in Nursing is acceptable A. Force majeure
B. Masters degree in Nursing or in the related fields B. Respondeat superior
C. At least a doctorate in nursing C. Res ipsa loquitur
D. At least 18 units in the Master of Arts in Nursing D. Holdover doctrine
Program
Answer: (C) Res ipsa loquitur
Answer: (B) Masters degree in Nursing or in the related Res ipsa loquitur literally means the thing speaks for
fields itself. This means in operational terms that the injury
According to RA 9173 sec. 27, the educational caused is the proof that there was a negligent act.
qualification of a faculty member teaching in a college of
nursing must be masters degree which maybe in nursing 11. Ensuring that there is an informed consent on the
or related fields like education, allied health professions, part of the patient before a surgery is done, illustrates
psychology. the bioethical principle of:
A. Beneficence
8. The educational qualification of a nurse to become a B. Autonomy
supervisor in a hospital is: C. Truth telling/veracity
A. BSN with at least 9 units of post graduate studies in D. Non-maleficence
nursing administration
B. Master of Arts in Nursing major in administration Answer: (B) Autonomy
C. At least 2 years experience as a headnurse Informed consent means that the patient fully
D. At least 18 units of post graduate studies in nursing understands what will be the surgery to be done, the
administration risks involved and the alternative solutions so that when
s/he give consent it is done with full knowledge and is

52
52
given freely. The action of allowing the patient to decide not required to give extraordinary measures but cannot
whether a surgery is to be done or not exemplifies the withhold the basic needs like food, water, and air. It also
bioethical principle of autonomy. means that the nurse is still duty bound to give the basic
nursing care to the terminally ill patient and ensure that
12. When a nurse is providing care to her/his patient, the spiritual needs of the patient is taken cared of.
s/he must remember that she is duty bound not to do
doing any action that will cause the patient harm. This 15. Which of the following statements is TRUE of
is the meaning of the bioethical principle: abortion in the Philippines?
A. Non-maleficence A. Induced abortion is allowed in cases of rape and
B. Beneficence incest
C. Justice B. Induced abortion is both a criminal act and an
D. Solidarity unethical act for the nurse
C. Abortion maybe considered acceptable if the
Answer: (A) Non-maleficence mother is unprepared for the pregnancy
Non-maleficence means do not cause harm or do any D. A nurse who performs induced abortion will have
action that will cause any harm to the patient/client. To no legal accountability if the mother requested that
do good is referred as beneficence. the abortion done on her.

13. When the patient is asked to testify in court, s/he Answer: (B) Induced abortion is both a criminal act and
must abide by the ethical principle of: an unethical act for the nurse
A. Privileged communication Induced abortion is considered a criminal act which is
B. Informed consent punishable by imprisonment which maybe up to a
C. Solidarity maximum of 12 years if the nurse gets paid for it. Also,
D. Autonomy the PRC Code of Ethics states that the nurse must
respect life and must not do any action that will destroy
Answer: (A) Privileged communication life. Abortion is an act that destroys life albeit at the
All confidential information that comes to the beginning of life.
knowledge of the nurse in the care of her/his patients is
considered privileged communications. Hence, s/he is
not allowed to just reveal the confidential information
arbitrarily. S/he may only be allowed to break the seal of
secrecy in certain conditions. One such condition is when
the court orders the nurse to testify in a criminal or
medico-legal case.

14. When the doctor orders “do not resuscitate”, this


means that
A. The nurse need not give due care to the patient
since s/he is terminally ill
B. The patient need not be given food and water after all
s/he is dying
C. The nurses and the attending physician should not do
any heroic or extraordinary measures for the patient
D. The patient need not be given ordinary care so
that her/his dying process is hastened

Answer: (C) The nurses and the attending physician


should not do any heroic or extraordinary measures for
the patient
Do not resuscitate” is a medical order which is written
on the chart after the doctor has consulted the family
and this means that the members of the health team are

52
52
LEADERSHIP and MANAGEMENT 4. Which of the following conclusions of Ms.
Castro about leadership characteristics is TRUE?
1. Ms. Castro is newly-promoted to a patient care A. There is a high correlation between the
manager position. She updates her knowledge on the communication skills of a leader and the ability to get
theories in management and leadership in order to the job done.
become effective in her new role. She learns that some B. A manager is effective when he has the ability to plan
managers have low concern for services and high well.
concern for staff. Which style of management refers to C. Assessment of personal traits is a reliable tool for
this? predicting a manager’s potential.
A. Organization Man D. There is good evidence that certain personal
B. Impoverished Management qualities favor success in managerial role.
C. Country Club Management
D. Team Management Answer: (C) Assessment of personal traits is a reliable
tool for predicting a manager’s potential.
Answer: (C) Country Club Management It is not conclusive that certain qualities of a person
Country club management style puts concern for the would make him become a good manager. It can only
staff as number one priority at the expense of the predict a manager’s potential of becoming a good one.
delivery of services. He/she runs the department just like
a country club where every one is happy including the 5. She reads about Path Goal theory. Which of the
manager. following behaviors is manifested by the leader who
uses this theory?
2. Her former manager demonstrated passion for A. Recognizes staff for going beyond expectations by
serving her staff rather than being served. She takes giving them citations
time to listen, prefers to be a teacher first before being a B. Challenges the staff to take individual accountability
leader, which is characteristic of for their own practice
A. Transformational leader C. Admonishes staff for being laggards.
B. Transactional leader D. Reminds staff about the sanctions for non
C. Servant leader performance.
D. Charismatic leader
Answer: (A) Recognizes staff for going beyond
Answer: (C) Servant leader expectations by giving them citations
Servant leaders are open-minded, listen deeply, try to Path Goal theory according to House and associates
fully understand others and not being judgmental rewards good performance so that others would do the
same
3. On the other hand, Ms. Castro notices that the Chief
Nurse Executive has charismatic leadership style. Which 6. One leadership theory states that “leaders are born
of the following behaviors best describes this style? and not made,” which refers to which of the following
A. Possesses inspirational quality that makes followers theories?
gets attracted of him and regards him with reverence A. Trait
B. Acts as he does because he expects that his B. Charismatic
behavior will yield positive results C. Great Man
C. Uses visioning as the core of his leadership D. Situational
D. Matches his leadership style to the situation at hand.
Answer: (C) Great Man
Answer: (A) Possesses inspirational quality that makes Leaders become leaders because of their birth right. This
followers gets attracted of him and regards him with is also called Genetic theory or the Aristotelian theory
reverence
Charismatic leaders make the followers feel at ease in 7. She came across a theory which states that the
their presence. They feel that they are in good hands leadership style is effective dependent on the situation.
whenever the leader is around. Which of the following styles best fits a situation when
the followers are self-directed, experts and arematured
individuals?

52
52
A. Democratic their subordinates
B. Authoritarian Benevolent-authoritative managers pretentiously show
C. Laissez faire their trust and confidence to their followers
D. Bureaucratic
11. Harry is a Unit Manager I the Medical Unit. He is not
Answer: (C) Laissez faire satisfied with the way things are going in his unit. Patient
Laissez faire leadership is preferred when the followers satisfaction rate is 60% for two consecutive months and
know what to do and are experts in the field. This staff morale is at its lowest. He decides to plan and
leadership style is relationship-oriented rather than task- initiate changes that will push for a turnaround in the
centered. condition of the unit. Which of the following actions is a
priority for Harry?
8. She surfs the internet for more information about A. Call for a staff meeting and take this up in the agenda.
leadership styles. She reads about shared leadership as a B. Seek help from her manager.
practice in some magnet hospitals. Which of the C. Develop a strategic action on how to deal with these
following describes this style of leadership? concerns.
A. Leadership behavior is generally determined by the D. Ignore the issues since these will be resolved
relationship between the leader’s personality and the naturally.
specific situation
B. Leaders believe that people are basically good and Answer: (A) Call for a staff meeting and take this up in
need not be closely controlled the agenda.
C. Leaders rely heavily on visioning and inspire members This will allow for the participation of every staff in the
to achieve results unit. If they contribute to the solutions of the problem,
D. Leadership is shared at the point of care. they will own the solutions; hence the chance for
compliance would be greater.
Answer: (D) Leadership is shared at the point of care.
Shared governance allows the staff nurses to have the 12. She knows that there are external forces that
authority, responsibility and accountability for their own influence changes in his unit. Which of the following is
practice. NOT an external force?
A. Memo from the CEO to cut down on electrical
9. Ms. Castro learns that some leaders are transactional consumption
leaders. Which of the following does NOT characterize a B. Demands of the labor sector to increase wages
transactional leader? C. Low morale of staff in her unit
A. Focuses on management tasks D. Exacting regulatory and accreditation standards
B. Is a caretaker
C. Uses trade-offs to meet goals Answer: (C) Low morale of staff in her unit
D. Inspires others with vision Low morale of staff is an internal factor that affects only
the unit. All the rest of the options emanate from the
Answer: (D) Inspires others with vision top executive or from outside the institution.
Inspires others with a vision is characteristic of a
transformational leader. He is focused more on the day- 13. After discussing the possible effects of the low
to-day operations of the department/unit. patient satisfaction rate, the staff started to list down
possible strategies to solve the problems head-on.
10. She finds out that some managers have benevolent- Should they decide to vote on the best change strategy,
authoritative style of management. Which of the which of the following strategies is referred to this?
following behaviors will she exhibit most likely? A. Collaboration
A. Have condescending trust and confidence in their B. Majority rule
subordinates C. Dominance
B. Gives economic or ego awards D. Compromise
C. Communicates downward to the staff
D. Allows decision making among subordinates Answer: (B) Majority rule
Majority rule involves dividing the house and the highest
Answer: (A) Have condescending trust and confidence in

52
52
vote wins.1/2 + 1 is a majority. at the same time applies the management process of
planning, organizing, directing and controlling
14. One staff suggests that they review the pattern of
nursing care that they are using, which is described as 18. Which of the following is the best guarantee that the
a patient’s priority needs are met?
A. job description A. Checking with the relative of the patient
B. system used to deliver care B. Preparing a nursing care plan in collaboration with the
C. manual of procedure patient
D. rules to be followed C. Consulting with the physician
D. Coordinating with other members of the team
Answer: (B) system used to deliver care
A system used to deliver care. In the 70’s it was termed Answer: (B) Preparing a nursing care plan in
as methods of patient assignment; in the early 80’s it collaboration with the patient
was called modalities of patient care then patterns of The best source of information about the priority needs
nursing care in the 90’s until recently authors called it of the patient is the patient himself. Hence using a
nursing care systems. nursing care plan based on his expressed priority needs
would ensure meeting his needs effectively.
15. Which of the following is TRUE about functional 19. When Harry uses team nursing as a care delivery
nursing? system, he and his team need to assess the priority of
A. Concentrates on tasks and activities care for a group of patients, which of the following
B. Emphasizes use of group collaboration should be a priority?
C. One-to-one nurse-patient ratio A. Each patient as listed on the worksheet
D. Provides continuous, coordinated and comprehensive B. Patients who needs least care
nursing services C. Medications and treatments required for all patients
D. Patients who need the most care
Answer: (A) Concentrates on tasks and activities
Functional nursing is focused on tasks and activities and Answer: (D) Patients who need the most care
not on the holistic care of the patients In setting priorities for a group of patients, those who
need the most care should be number-one priority to
16. Functional nursing has some advantages, which ensure that their critical needs are met adequately. The
one is an EXCEPTION? needs of other patients who need less care ca be
A. Psychological and sociological needs are emphasized. attended to later or even delegated to assistive
B. Great control of work activities. personnel according to rules on delegation.
C. Most economical way of delivering nursing services.
D. Workers feel secure in dependent role 20. She is hopeful that her unit will make a big
turnaround in the succeeding months. Which of the
Answer: (A) Psychological and sociological needs are following actions of Harry demonstrates that he has
emphasized. reached the third stage of change?
When the functional method is used, the psychological A. Wonders why things are not what it used to be
and sociological needs of the patients are neglected; the B. Finds solutions to the problems
patients are regarded as ‘tasks to be done ‘ C. Integrate the solutions to his day-to-day activities
D. Selects the best change strategy
17. He raised the issue on giving priority to patient
needs. Which of the following offers the best way for Answer: (C) Integrate the solutions to his day-to-day
setting priority? activities
A. Assessing nursing needs and problems Integrate the solutions to his day-to-day activities is a
B. Giving instructions on how nursing care needs are to expected to happen during the third stage of change
be met when the change agent incorporate the selected
C. Controlling and evaluating the delivery of nursing care solutions to his system and begins to create a change.
D. Assigning safe nurse: patient ratio
21. Julius is a newly-appointed nurse manager of The
Answer: (A) Assessing nursing needs and problems Good Shepherd Medical Center, a tertiary hospital
This option follows the framework of the nursing process
52
52
located within the heart of the metropolis. He thinks of Answer: (C) Broken line
scheduling planning workshop with his staff in order to This is a staff relationship hence it is depicted by a
ensure an effective and efficient management of the broken line in the organizational structure
department. Should he decide to conduct a strategic
planning workshop, which of the following is NOT a 25. He likewise stresses the need for all the employees
characteristic of this activity? to follow orders and instructions from him and not from
A. Long-term goal-setting anyone else. Which of the following principles does he
B. Extends to 3-5 years in the future refer to?
C. Focuses on routine tasks A. Scalar chain
D. Determines directions of the organization B. Discipline
C. Unity of command
Answer: (C) Focuses on routine tasks D. Order
Strategic planning involves options A, B and D except C
which is attributed to operational planning Answer: (C) Unity of command
22. Which of the following statements refer to the vision The principle of unity of command means that
of the hospital? employees should receive orders coming from only one
A. The Good Shepherd Medical Center is a trendsetter in manager and not from two managers. This averts the
tertiary health care in the Philippines in the next five possibility of sowing confusion among the members of
years the organization
B. The officers and staff of The Good Shepherd Medical
Center believe in the unique nature of the human person 26. Julius orients his staff on the patterns of reporting
C. All the nurses shall undergo continuing competency relationship throughout the organization. Which of the
training program. following principles refer to this?
D. The Good Shepherd Medical Center aims to provide a A. Span of control
patient-centered care in a total healing environment. B. Hierarchy
C. Esprit d’ corps
Answer: (A) The Good Shepherd Medical Center is a D. Unity of direction
trendsetter in tertiary health care in the Philippines in
the next five years Answer: (B) Hierarchy
A vision refers to what the institution wants to become Hierarchy refers to the pattern of reporting or the formal
within a particular period of time. line of authority in an organizational structure.

23. The statement, “The Good Shepherd Medical Center 27. He emphasizes to the team that they need to put
aims to provide patient-centered care in a total healing their efforts together towards the attainment of the
environment” refers to which of the following? goals of the program. Which of the following
A. Vision principles refers to this?
B. Goal A. Span of control
C. Philosophy B. Unity of direction
D. Mission C. Unity of command
D. Command responsibility
Answer: (B) Goal
Answer: (B) Unity of direction
24. Julius plans to revisit the organizational chart of Unity of direction means having one goal or one
the department. He plans to create a new position of a objective for the team to pursue; hence all members of
Patient Educator who has a coordinating relationship the organization should put their efforts together
with the head nurse in the unit. Which of the following towards the attainment of their common goal or
will likely depict this organizational relationship? objective.
A. Box
B. Solid line 28. Julius stresses the importance of promoting ‘esprit d
C. Broken line corps’ among the members of the unit. Which of the
D. Dotted line following remarks of the staff indicates that they
understand what he pointed out?

52
52
A. “Let’s work together in harmony; we need to be Answer: (A) Proactive and caring with one another
supportive of one another” Positive culture is based on humanism and affiliative
B. “In order that we achieve the same results; we must norms
all follow the directives of Julius and not from other
managers.” 32. Stephanie is a new Staff Educator of a private tertiary
C. “We will ensure that all the resources we need are hospital. She conducts orientation among new staff
available when needed.” nurses in her department. Joseph, one of the new staff
D. “We need to put our efforts together in order to raise nurses, wants to understand the channel of
the bar of excellence in the care we provide to all our communication, span of control and lines of
patients.” communication. Which of the following will provide this
information?
Answer: (A) “Let’s work together in harmony; we need A. Organizational structure
to be supportive of one another” B. Policy
The principle of ‘esprit d’ corps’ refers to promoting C. Job description
harmony in the workplace, which is essential in D. Manual of procedures
maintaining a climate conducive to work.
Answer: (A) Organizational structure
29. He discusses the goal of the department. Which of Organizational structure provides information on the
the following statements is a goal? channel of authority, i.e., who reports to whom and with
A. Increase the patient satisfaction rate what authority; the number of people who directly
B. Eliminate the incidence of delayed administration of reports to the various levels of hierarchy and the lines of
medications communication whether line or staff.
C. Establish rapport with patients.
D. Reduce response time to two minutes. 33. Stephanie is often seen interacting with the medical
intern during coffee breaks and after duty hours. What
Answer: (A) Increase the patient satisfaction rate type of organizational structure is this?
Goal is a desired result towards which efforts are A. Formal
directed. Options AB, C and D are all objectives which B. Informal
are aimed at specific end. C. Staff
D. Line
30. He wants to influence the customary way of
thinking and behaving that is shared by the members of Answer: (B) Informal
the department. Which of the following terms refer to This is usually not published and oftentimes concealed.
this?
A. Organizational chart 34. She takes pride in saying that the hospital has a
B. Cultural network decentralized structure. Which of the following is NOT
C. Organizational structure compatible with this type of model?
D. Organizational culture A. Flat organization
B. Participatory approach
Answer: (D) Organizational culture C. Shared governance
An organizational culture refers to the way the members D. Tall organization
of the organization think together and do things around
them together. It’s their way of life in that organization Answer: (D) Tall organization
Tall organizations are highly centralized organizations
31. He asserts the importance of promoting a positive where decision making is centered on one authority
organizational culture in their unit. Which of the level.
following behaviors indicate that this is attained by the
group? 35. Centralized organizations have some advantages.
A. Proactive and caring with one another Which of the following statements are TRUE?
B. Competitive and perfectionist 1. Highly cost-effective
C. Powerful and oppositional 2. Makes management easier
D. Obedient and uncomplaining 3. Reflects the interest of the worker

52
52
4. Allows quick decisions or actions. A. Uses visioning as the essence of leadership.
B. Serves the followers rather than being served.
A. 1 & 2 C. Maintains full trust and confidence in the
B. 2 & 4 subordinates
C. 2, 3& 4 D. Possesses innate charisma that makes others feel
D. 1, 2, & 4 good in his presence.

Answer: (A) 1 & 2 Answer: (A) Uses visioning as the essence of leadership.
Centralized organizations are needs only a few managers Transformational leadership relies heavily on visioning as
hence they are less expensive and easier to manage the core of leadership.

36. Stephanie delegates effectively if she has authority 40. As a manager, she focuses her energy on both the
to act, which is BEST defined as: quality of services rendered to the patients as well as the
A. having responsibility to direct others welfare of the staff of her unit. Which of the following
B. being accountable to the organization management styles does she adopt?
C. having legitimate right to act A. Country club management
D. telling others what to do B. Organization man management
C. Team management
Answer: (C) having legitimate right to act D. Authority-obedience management
Authority is a legitimate or official right to give
command. This is an officially sanctioned responsibility Answer: (C) Team management
Team management has a high concern for services and
37. Regardless of the size of a work group, enough staff high concern for staff.
must be available at all times to accomplish certain
purposes. Which of these purposes in NOT included? 41. Katherine is a young Unit Manager of the Pediatric
A. Meet the needs of patients Ward. Most of her staff nurses are senior to her, very
B. Provide a pair of hands to other units as needed articulate, confident and sometimes aggressive.
C. Cover all time periods adequately. Katherine feels uncomfortable believing that she is the
D. Allow for growth and development of nursing staff. scapegoat of everything that goes wrong in her
department. Which of the following is the best action
Answer: (B) Provide a pair of hands to other units as that she must take?
needed A. Identify the source of the conflict and understand the
Providing a pair of hands for other units is not a purpose points of friction
in doing an effective staffing process. This is a function of B. Disregard what she feels and continue to work
a staffing coordinator at a centralized model. independently
C. Seek help from the Director of Nursing
38. Which of the following guidelines should be least D. Quit her job and look for another employment.
considered in formulating objectives for nursing care?
A. Written nursing care plan Answer: (A) Identify the source of the conflict and
B. Holistic approach understand the points of friction
C. Prescribed standards This involves a problem solving approach, which
D. Staff preferences addresses the root cause of the problem.

Answer: (D) Staff preferences 42. As a young manager, she knows that conflict occurs
Staff preferences should be the least priority in in any organization. Which of the following statements
formulating objectives of nursing care. Individual regarding conflict is NOT true?
preferences should be subordinate to the interest of the A. Can be destructive if the level is too high
patients. B. Is not beneficial; hence it should be prevented at
all times
39. Stephanie considers shifting to transformational C. May result in poor performance
leadership. Which of the following statements best D. May create leaders
describes this type of leadership?

52
52
Answer: (B) Is not beneficial; hence it should be Performance appraisal deal with both positive and
prevented at all times negative performance; is not meant to be a fault-finding
Conflicts are beneficial because it surfaces out issues in activity
the open and can be solved right away. Likewise,
members of the team become more conscientious with 46. Which of the following statements is NOT true about
their work when they are aware that other members of performance appraisal?
the team are watching them. A. Informing the staff about the specific impressions of
their work help improve their performance.
43. Katherine tells one of the staff, “I don’t have time to B. A verbal appraisal is an acceptable substitute for a
discuss the matter with you now. See me in my office written report
later” when the latter asks if they can talk about an C. Patients are the best source of information regarding
issue. Which of the following conflict resolution personnel appraisal.
strategies did she use? D. The outcome of performance appraisal rests
A. Smoothing primarily with the staff.
B. Compromise
C. Avoidance Answer: (C) Patients are the best source of information
D. Restriction regarding personnel appraisal.
The patient can be a source of information about the
Answer: (C) Avoidance performance of the staff but it is never the best source.
This strategy shuns discussing the issue head-on and Directly observing the staff is the best source of
prefers to postpone it to a later time. In effect the information for personnel appraisal.
problem remains unsolved and both parties are in a lose-
lose situation. 47. There are times when Katherine evaluates her staff
as she makes her daily rounds. Which of the following is
44. Kathleen knows that one of her staff is experiencing NOT a benefit of conducting an informal appraisal?
burnout. Which of the following is the best thing for her A. The staff member is observed in natural setting.
to do? B. Incidental confrontation and collaboration is allowed.
A. Advise her staff to go on vacation. C. The evaluation is focused on objective data
B. Ignore her observations; it will be resolved even systematically.
without intervention D. The evaluation may provide valid information for
C. Remind her to show loyalty to the institution. compilation of a formal report.
D. Let the staff ventilate her feelings and ask how she
can be of help. Answer: (C) The evaluation is focused on objective data
systematically.
Answer: (D) Let the staff ventilate her feelings and ask Collecting objective data systematically can not be
how she can be of help. achieved in an informal appraisal. It is focused on what
Reaching out and helping the staff is the most effective actually happens in the natural work setting.
strategy in dealing with burn out. Knowing that
someone is ready to help makes the staff feel important; 48. She conducts a 6-month performance review
hence her self-worth is enhanced. session with a staff member. Which of the following
actions is appropriate?
45. She knows that performance appraisal consists of A. She asks another nurse to attest the session as a
all the following activities EXCEPT: witness.
A. Setting specific standards and activities for individual B. She informs the staff that she may ask another nurse
performance. to read the appraisal before the session is over.
B. Using agency standards as a guide. C. She tells the staff that the session is manager-
C. Determine areas of strength and weaknesses centered.
D. Focusing activity on the correction of identified D. The session is private between the two members.
behavior.
Answer: (D) The session is private between the two
Answer: (D) Focusing activity on the correction of members.
identified behavior. The session is private between the manager and the

52
53
staff and remains to be so when the two parties do not C. Membership to accredited professional organization
divulge the information to others. D. Professional identification card

49. Alexandra is tasked to organize the new wing of the Answer: (B) Record of related learning experience (RLE)
hospital. She was given the authority to do as she deems Record of RLE is not required for employment purposes
fit. She is aware that the director of nursing has but it is required for the nurse’s licensure examination.
substantial trust and confidence in her capabilities,
communicates through downward and upward channels 53. Which phase of the employment process includes
and usually uses the ideas and opinions of her staff. getting on the payroll and completing documentary
Which of the following is her style of management? requirements?
A. Benevolent –authoritative A. Orientation
B. Consultative B. Induction
C. Exploitive-authoritative C. Selection
D. Participative D. Recruitment

Answer: (B) Consultative Answer: (B) Induction


A consultative manager is almost like a participative This step in the recruitment process gives time for the
manager. The participative manager has complete trust staff to submit all the documentary requirements for
and confidence in the subordinate, always uses the employment.
opinions and ideas of subordinates and communicates in
all directions. 54. She tries to design an organizational structure that
allows communication to flow in all directions and
52. She decides to illustrate the organizational structure. involve workers in decision making. Which form of
Which of the following elements is NOT included? organizational structure is this?
A. Level of authority A. Centralized
B. Lines of communication B. Decentralized
C. Span of control C. Matrix
D. Unity of direction D. Informal

Answer: (D) Unity of direction Answer: (B) Decentralized


Unity of direction is a management principle, not an Decentralized structures allow the staff to make
element of an organizational structure. decisions on matters pertaining to their practice and
communicate in downward, upward, lateral and
51. She plans of assigning competent people to fill the diagonal flow.
roles designed in the hierarchy. Which process refers to
this? 55. In a horizontal chart, the lowest level worker is
A. Staffing located at the
B. Scheduling A. Leftmost box
C. Recruitment B. Middle
D. Induction C. Rightmost box
D. Bottom
Answer: (A) Staffing
Staffing is a management function involving putting the Answer: (C) Rightmost box
best people to accomplish tasks and activities to attain The leftmost box is occupied by the highest authority
the goals of the organization. while the lowest level worker occupies the rightmost
box.

52. She checks the documentary requirements for the 56. She decides to have a decentralized staffing system.
applicants for staff nurse position. Which one is NOT Which of the following is an advantage of this system of
necessary? staffing?
A. Certificate of previous employment A. greater control of activities
B. Record of related learning experience (RLE) B. Conserves time

53
53
C. Compatible with computerization D. Measure actual performances
D. Promotes better interpersonal relationship
Answer: (B) Identify the values of the department
Answer: (D) Promotes better interpersonal relationship Identify the values of the department will set the guiding
Decentralized structures allow the staff to solve principles within which the department will operate its
decisions by themselves, involve them in decision activities
making; hence they are always given opportunities to
interact with one another. 60. Ms. Valencia develops the standards to be followed.
Among the following standards, which is considered as a
57. Aubrey thinks about primary nursing as a system structure standard?
to deliver care. Which of the following activities is NOT A. The patients verbalized satisfaction of the nursing
done by a primary nurse? care received
A. Collaborates with the physician B. Rotation of duty will be done every four weeks for
B. Provides care to a group of patients together with a all patient care personnel.
group of nurses C. All patients shall have their weights taken recorded
C. Provides care for 5-6 patients during their hospital D. Patients shall answer the evaluation form before
stay. discharge
D. Performs comprehensive initial assessment
Answer: (B) Rotation of duty will be done every four
Answer: (B) Provides care to a group of patients together weeks for all patient care personnel.
with a group of nurses Structure standards include management system,
This function is done in team nursing where the nurse is facilities, equipment, materials needed to deliver care to
a member of a team that provides care for a group of patients. Rotation of duty is a management system.
patients.
61. When she presents the nursing procedures to be
58. Which pattern of nursing care involves the care given followed, she refers to what type of standards?
by a group of paraprofessional workers led by a A. Process
professional nurse who take care of patients with the B. Outcome
same disease conditions and are located geographically C. Structure
near each other? D. Criteria
A. Case method
B. Modular nursing Answer: (A) Process
C. Nursing case management Process standards include care plans, nursing procedure
D. Team nursing to be done to address the needs of the patients.

Answer: (B) Modular nursing 62. The following are basic steps in the controlling
Modular nursing is a variant of team nursing. The process of the department. Which of the following is
difference lies in the fact that the members in modular NOT included?
nursing are paraprofessional workers. A. Measure actual performance
B. Set nursing standards and criteria
59. St. Raphael Medical Center just opened its new C. Compare results of performance to standards and
Performance Improvement Department. Ms. Valencia is objectives
appointed as the Quality Control Officer. She commits D. Identify possible courses of action
herself to her new role and plans her strategies to realize
the goals and objectives of the department. Which of the Answer: (D) Identify possible courses of action
following is a primary task that they should perform to This is a step in a quality control process and not a basic
have an effective control system? step in the control process.
A. Make an interpretation about strengths and
weaknesses 63. Which of the following statements refers to criteria?
B. Identify the values of the department A. Agreed on level of nursing care
C. Identify structure, process, outcome standards & B. Characteristics used to measure the level of nursing
criteria care

53
53
C. Step-by-step guidelines B. Unity of command
D. Statement which guide the group in decision making C. Carrot and stick principle
and problem solving D. Esprit d’ corps

Answer: (B) Characteristics used to measure the level of Answer: (A) Span of control
nursing care Span of control refers to the number of workers who
Criteria are specific characteristics used to measure the report directly to a manager.
standard of care.
68. She notes that there is an increasing unrest of the
64. She wants to ensure that every task is carried out as staff due to fatigue brought about by shortage of staff.
planned. Which of the following tasks is NOT included in Which action is a priority?
the controlling process? A. Evaluate the overall result of the unrest
A. Instructing the members of the standards committee B. Initiate a group interaction
to prepare policies C. Develop a plan and implement it
B. Reviewing the existing policies of the hospital D. Identify external and internal forces.
C. Evaluating the credentials of all nursing staff
D. Checking if activities conform to schedule Answer: (B) Initiate a group interaction
Initiate a group interaction will be an opportunity to
Answer: (A) Instructing the members of the standards discuss the problem in the open.
committee to prepare policies
Instructing the members involves a directing function.

65. Ms. Valencia prepares the process standards. Which


of the following is NOT a process standard?
A. Initial assessment shall be done to all patients within
twenty four hours upon admission.
B. Informed consent shall be secured prior to any
invasive procedure
C. Patients’ reports 95% satisfaction rate prior to
discharge from the hospital.
D. Patient education about their illness and treatment
shall be provided for all patients and their families.

Answer: (C) Patients’ reports 95% satisfaction rate prior


to discharge from the hospital.
This refers to an outcome standard, which is a result of
the care that is rendered to the patient.

66. Which of the following is evidence that the


controlling process is effective?
A. The things that were planned are done
B. Physicians do not complain.
C. Employees are contended
D. There is an increase in customer satisfaction rate.

Answer: (A) The things that were planned are done


Controlling is defined as seeing to it that what is planned
is done.

67. Ms. Valencia is responsible to the number of


personnel reporting to her. This principle refers to:
A. Span of control

53
53
NURSING RESEARCH Part 1 D. “Environmental Manipulation and Client Outcomes”

1. Kevin is a member of the Nursing Research Council Answer: (B) “Turnaround Time in Emergency Rooms”
of the hospital. His first assignment is to determine the The article is for pediatric patients and may not be
level of patient satisfaction on the care they received relevant for adult patients.
from the hospital. He plans to include all adult patients
admitted from April to May, with average length of stay 5. Which of the following variables will he likely EXCLUDE
of 3-4 days, first admission, and with no complications. in his study?
Which of the following is an extraneous variable of the A. Competence of nurses
study? B. Caring attitude of nurses
A. Date of admission C. Salary of nurses
B. Length of stay D. Responsiveness of staff
C. Age of patients
D. Absence of complications Answer: (C) Salary of nurses
Salary of staff nurses is not an indicator of patient
Answer: (C) Age of patients satisfaction, hence need not be included as a variable in
An extraneous variable is not the primary concern of the the study.
researcher but has an effect on the results of the study.
Adult patients may be young, middle or late adult. 6. He plans to use a Likert Scale to determine
A. degree of agreement and disagreement
2. He thinks of an appropriate theoretical framework. B. compliance to expected standards
Whose theory addresses the four modes of adaptation? C. level of satisfaction
A. Martha Rogers D. degree of acceptance
B. Sr. Callista Roy
C. Florence Nightingale Answer: (A) degree of agreement and disagreement
D. Jean Watson Likert scale is a 5-point summated scale used to
determine the degree of agreement or disagreement of
Answer: (B) Sr. Callista Roy the respondents to a statement in a study.
Sr. Callista Roy developed the Adaptation Model which
involves the physiologic mode, self-concept mode, role 7. He checks if his instruments meet the criteria for
function mode and dependence mode evaluation. Which of the following criteria refers to the
consistency or the ability to yield the same response
3. He opts to use a self-report method. Which of the upon its repeated administration?
following is NOT TRUE about this method? A. Validity
A. Most direct means of gathering information B. Reliability
B. Versatile in terms of content coverage C. Sensitivity
C. Most accurate and valid method of data gathering D. Objectivity
D. Yields information that would be difficult to gather by
another method Answer: (B) Reliability
Reliability is repeatability of the instrument; it can elicit
Answer: (C) Most accurate and valid method of data the same responses even with varied administration of
gathering the instrument
The most serious disadvantage of this method is
accuracy and validity of information gathered 8. Which criteria refer to the ability of the instrument to
detect fine differences among the subjects being
4. Which of the following articles would Kevin least studied?
consider for his review of literature? A. Sensitivity
A. “Story-Telling and Anxiety Reduction Among Pediatric B. Reliability
Patients” C. Validity
B. “Turnaround Time in Emergency Rooms” D. Objectivity
C. “Outcome Standards in Tertiary Health Care
Institutions” Answer: (A) Sensitivity

53
53
Sensitivity is an attribute of the instrument that allow transcultural theory based on her observations on the
the respondents to distinguish differences of the options behavior of selected people within a culture
where to choose from
13. Which of the following statements best describes a
9. Which of the following terms refer to the degree to phenomenological study?
which an instrument measures what it is supposed to be A. Involves the description and interpretation of cultural
measure? behavior
A. Validity B. Focuses on the meaning of experiences as those
B. Reliability who experience it
C. Meaningfulness C. Involves an in-depth study of an individual or group
D. Sensitivity D. Involves collecting and analyzing data that aims to
develop theories grounded in real-world observations
Answer: (A) Validity
Validity is ensuring that the instrument contains Answer: (B) Focuses on the meaning of experiences as
appropriate questions about the research topic those who experience it
Phenomenological study involves understanding the
10. He plans for his sampling method. Which sampling meaning of experiences as those who experienced the
method gives equal chance to all units in the population phenomenon.
to get picked?
A. Random 14. He systematically plans his sampling plan. Should he
B. Accidental decides to include whoever patients are admitted during
C. Quota the study he uses what sampling method?
D. Judgment A. Judgment
B. Accidental
Answer: (A) Random C. Random
Random sampling gives equal chance for all the D. Quota
elements in the population to be picked as part of the
sample. Answer: (B) Accidental
Accidental sampling is a non-probability sampling
11. Raphael is interested to learn more about method which includes those who are at the site during
transcultural nursing because he is assigned at the data collection.
family suites where most patients come from different
cultures and countries. Which of the following designs is 15. He finally decides to use judgment sampling.
appropriate for this study? Which of the following actions of Raphael is correct?
A. Grounded theory A. Plans to include whoever is there during his study.
B. Ethnography B. Determines the different nationality of patients
C. Case study frequently admitted and decides to get representations
D. Phenomenology samples from each.
C. Assigns numbers for each of the patients, place these
Answer: (B) Ethnography in a fishbowl and draw 10 from it.
Ethnography is focused on patterns of behavior of D. Decides to get 20 samples from the admitted patients
selected people within a culture
Answer: (B) Determines the different nationality of
12. The nursing theorist who developed transcultural patients frequently admitted and decides to get
nursing theory is representations samples from each.
A. Dorothea Orem Judgment sampling involves including samples according
B. Madeleine Leininger to the knowledge of the investigator about the
C. Betty Newman participants in the study.
D. Sr. Callista Roy
16. He knows that certain patients who are in a
Answer: (B) Madeleine Leininger specialized research setting tend to respond
Madeleine Leininger developed the theory on psychologically to the conditions of the study. This is

53
53
referred to as 20. Which of the following titles of the study is
A. Bias appropriate for this study?
B. Hawthorne effect A. Lived Experiences of Terminally-Ill Cancer Patients
C. Halo effect B. Coping Skills of Terminally-Ill Cancer Patients in a
D. Horns effect Selected Hospital
C. Two Case Studies of Terminally-Ill Patients in Manila
Answer: (B) Hawthorne effect D. Beliefs & Practices of Patients with Terminal Cancer
Hawthorne effect is based on the study of Elton Mayo
and company about the effect of an intervention done to Answer: (B) Coping Skills of Terminally-Ill Cancer Patients
improve the working conditions of the workers on their in a Selected Hospital
productivity. It resulted to an increased productivity but The title has a specific phenomenon, sample and
not due to the intervention but due to the psychological research locale.
effects of being observed. They performed differently
because they were under observation. 21. Ms. Montana plans to conduct a research on the use
of a new method of pain assessment scale. Which of the
17. Which of the following items refer to the sense of following is the second step in the conceptualizing phase
closure that Raphael experiences when data collection of the research process?
ceases to yield any new information? A. Formulating the research hypothesis
A. Saturation B. Review related literature
B. Precision C. Formulating and delimiting the research problem
C. Limitation D. Design the theoretical and conceptual framework
D. Relevance
Answer: (B) Review related literature
Answer: (A) Saturation After formulating and delimiting the research problem,
Saturation is achieved when the investigator can not the researcher conducts a review of related literature to
extract new responses from the informants, but instead, determine the extent of what has been done on the
gets the same responses repeatedly. study by previous researchers.

18. In qualitative research the actual analysis of 22. Which of the following codes of research ethics
data begins with: requires informed consent in all cases governing human
A. search for themes subjects?
B. validation of thematic analysis A. Helsinki Declaration
C. weave the thematic strands together B. Nuremberg Code
D. quasi statistics C. Belmont Report
D. ICN Code of Ethics
Answer: (A) search for themes
The investigator starts data analysis by looking for Answer: (A) Helsinki Declaration
themes from the verbatim responses of the informants. Helsinki Declaration is the first international attempt to
set up ethical standards in research involving human
19. Raphael is also interested to know the coping research subjects.
abilities of patients who are newly diagnosed to have
terminal cancer. Which of the following types of 23. Which of the following ethical principles was NOT
research is appropriate? articulated in the Belmont Report?
A. Phenomenological A. Beneficence
B. Ethnographic B. Respect for human dignity
C. Grounded Theory C. Justice
D. Case Study D. Non-maleficence

Answer: (C) Grounded Theory Answer: (D) Non-maleficence


Grounded theory inductively develops a theory based on Non-maleficence is not articulated in the Belmont
the observed processes involving selected people Report. It only includes beneficence, respect for human

53
53
dignity and justice. rejected. Hypothesis is testable and is defined as a
statement that predicts the relationship between
24. Which one of the following criteria should be variables
considered as a top priority in nursing care?
A. Avoidance of destructive changes 28. Which of the following measures will best prevent
B. Preservation of life manipulation of vulnerable groups?
C. Assurance of safety A. Secure informed consent
D. Preservation of integrity B. Payment of stipends for subjects
C. Protect privacy of patient
Answer: (B) Preservation of life D. Ensure confidentiality of data
The preservation of life at all cost is a primary
responsibility of the nurse. This is embodied in the Code Answer: (A) Secure informed consent
of Ethics for registered nurses ( BON Resolution 220 s. Securing informed consent will free the researcher from
2004). being accused of manipulating the subjects because by
so doing he/she gives ample opportunity for the
25. Which of the following procedures ensures that subjects to weigh the advantages/disadvantages of
the investigator has fully described to prospective being included in the study prior to giving his consent.
subjects the nature of the study and the subject's This is done without any element of force, coercion,
rights? threat or even inducement.
A. Debriefing
B. Full disclosure 29. Which of the following procedures ensures that Ms.
C. Informed consent Montana has fully described to prospective subjects the
D. Cover data collection nature of the study and the subject’s rights?
A. Debriefing
Answer: (B) Full disclosure B. Full disclosure
Full disclosure is giving the subjects of the research C. Informed consent
information that they deserve to know prior to the D. Covert data collection
conduct of the study.
Answer: (B) Full disclosure
26. After the review session has been completed, Karen Full disclosure is giving the subjects of the research
and the staff signed the document. Which of the information that they deserve to know prior to the
following is the purpose of this? conduct of the study
A. Agree about the content of the evaluation.
B. Signify disagreement of the content of the evaluation. 30. This technique refers to the use of multiple referents
C. Document that Karen and the staff reviewed the to draw conclusions about what constitutes the truth
evaluation. A. Triangulation
D. Serve as basis for future evaluation. B. Experiment
C. Meta-analysis
Answer: (C) Document that Karen and the staff reviewed D. Delphi technique
the evaluation.
Signing the document is done to serve as a proof that Answer: (A) Triangulation
performance review was conducted during that date and Triangulation makes use of different sources of
time. information such as triangulation in design, researcher
and instrument.
27. Which of the following is NOT true about a
hypothesis? Hypothesis is: 31. The statement, “Ninety percent (90%) of the
A. testable respondents are female staff nurses validates previous
B. proven research findings (Santos, 2001; Reyes, 2005) that the
C. stated in a form that it can be accepted or rejected nursing profession is largely a female dominated
D. states a relationship between variables profession is an example of
A. implication
Answer: (B) proven B. interpretation
Hypothesis is not proven; it is either accepted or
53
53
C. analysis A. CINAHL
D. conclusion B. MEDLINE
C. HealthSTAR
Answer: (B) interpretation D. EMBASE
Interpretation includes the inferences of the researcher
about the findings of the study. Answer: (A) CINAHL
This refers to Cumulative Index to Nursing and Allied
32. The study is said to be completed when Ms. Health Literature which is a rich source for literature
Montana achieved which of the following activities? review for nurses. The rest of the sites are for
A. Published the results in a nursing journal. medicine, pharmacy and other health-related sites.
B. Presented the study in a research forum.
C. The results of the study is used by the nurses in the 36. While reviewing journal articles, Stephanie got
hospital interested in reading the brief summary of the article
D. Submitted the research report to the CEO. placed at the beginning of the journal report. Which of
the following refers to this?
Answer: (C) The results of the study is used by the nurses A. Introduction
in the hospital B. Preface
The last step in the research process is the utilization of C. Abstract
the research findings. D. Background

33. Situation: Stephanie is a nurse researcher of the Answer: (C) Abstract


Patient Care Services Division. She plans to conduct a Abstract contains concise description of the background
literature search for her study. of the study, research questions, research objectives,
methods, findings, implications to nursing practice as
Which of the following is the first step in selecting well as keywords used in the study.
appropriate materials for her review?
A. Track down most of the relevant resources 37. She notes down ideas that were derived from the
B. Copy relevant materials description of an investigation written by the person
C. Organize materials according to function who conducted it. Which type of reference source refers
D. Synthesize literature gathered. to this?
A. Footnote
Answer: (A) Track down most of the relevant resources B. Bibliography
The first step in the review of related literature is to C. Primary source
track down relevant sources before copying these. The D. Endnotes
last step is to synthesize the literature gathered.
Answer: (C) Primary source
34. She knows that the most important categories of This refers to a primary source which is a direct account
information in literature review is the: of the investigation done by the investigator. In contrast
A. research findings to this is a secondary source, which is written by
B. theoretical framework someone other than the original researcher.
C. methodology
D. opinions 38. She came across a study which is referred to as
meta-analysis. Which of the following statements best
Answer: (A) research findings defines this type of study?
The research findings is the most important category of A. Treats the findings from one study as a single piece of
information that the researcher should copy because data
this will give her valuable information as to what has B. Findings from multiple studies are combined to yield a
been discovered in past studies about the same topic. data set which is analyzed as individual data
C. Represents an application of statistical procedures to
35. She also considers accessing electronic data bases for findings from each report
her literature review. Which of the following is the most D. Technique for quantitatively combining and thus
useful electronic database for nurses? integrating the results of multiple studies on a given

53
53
topic. difference or no relationship between the variables in
the study
Answer: (D) Technique for quantitatively combining and
thus integrating the results of multiple studies on a given 42. She notes that the dependent variable in the
topic. hypothesis “Duration of sleep of cuddled infants is
Though all the options are correct, the best definition is longer than those infants who are not cuddled by
option D because it combines quantitatively the results mothers” is
and at the same time it integrates the results of the A. Cuddled infants
different studies as one finding. B. Duration of sleep
C. Infants
39. This kind of research gathers data in detail about a D. Absence of cuddling
individual or groups and presented in narrative form,
which is Answer: (B) Duration of sleep
A. Case study Duration of sleep is the ‘effect’ (dependent variable) of
B. Historical cuddling ‘cause’ (independent variable).
C. Analytical
D. Experimental 43. Situation: Aretha is a nurse researcher in a tertiary
hospital. She is tasked to conduct a research on the
Answer: (A) Case study effects of structured discharge plan for post-open heart
Case study focuses on in-depth investigations of single surgery patients.
entity or small number of entities. It attempts to analyze
and understand issues of importance to history, She states the significance of the research problem.
development or circumstances of the person or entity Which of the following statements is the MOST
under study. significant for this study?
A. Improvement in patient care
40. Stephanie is finished with the steps in the B. Development of a theoretical basis for nursing
conceptual phase when she has conducted the LAST C. Increase the accountability of nurses.
step, which is D. Improves the image of nursing
A. formulating and delimiting the problem.
B. review of related literature Answer: (A) Improvement in patient care
C. develop a theoretical framework The ultimate goal of conducting research is to improve
D. formulate a hypothesis patient care which is achieved by enhancing the practice
of nurses when they utilize research results in their
Answer: (D) formulate a hypothesis practice.
The last step in the conceptualizing phase of the
research process is formulating a hypothesis. The rest 44. Regardless of the significance of the study, the
are the first three steps in this phase. feasibility of the study needs to be considered. Which of
the following is considered a priority?
41. She states the hypothesis of the study. Which of A. Availability of research subjects
the following is a null hypothesis? B. Budgetary allocation
A. Infants who are breastfed have the same weight as C. Time frame
those who are bottle fed. D. Experience of the researcher
B. Bottle-fed infants have lower weight than breast-fed
infants Answer: (A) Availability of research subjects
C. Cuddled infants sleep longer than those who are left Availability is the most important criteria to be
by themselves to sleep. considered by the researcher in determining whether
D. Children of absentee parents are more prone to the study is feasible or not. No matter how significant
experience depression than those who live with both the study may be if there are no available
parents. subjects/respondents, the study can not push through.

Answer: (A) Infants who are breastfed have the same 46. Aretha knows that a good research problem exhibits
weight as those who are bottle fed. the following characteristics; which one is NOT included?
Null hypothesis predicts that there is no change, no
53
53
A. Clearly identified the variables/phenomenon under Answer: (C) Determine the budgetary allocation for the
consideration. study
B. Specifies the population being studied. Determining budgetary allocation for the study is not a
C. Implies the feasibility of empirical testing purpose of doing a pretesting of the instruments. This is
D. Indicates the hypothesis to be tested. done at an earlier stage of the design and planning
phase.
Answer: (D) Indicates the hypothesis to be tested.
Not all studies require a hypothesis such as qualitative 50. She tests the instrument whether it looks as
studies, which does not deal with variables but with though it is measuring appropriate constructs. Which
phenomenon or concepts. of the following refers to this?
A. Face validity
47. She states the purposes of the study. Which of B. Content validity
the following describe the purpose of a study? C. Construct Validity
D. Criterion-related validity
1. Establishes the general direction of a study
2. Captures the essence of the study Answer: (A) Face validity
3. Formally articulates the goals of the study Face validity measures whether the instrument appears
4. Sometimes worded as an intent to be measuring the appropriate construct. It is the
A. 1, 2, 3 easiest type of validity testing.
B. 2, 3, 4
C. 1, 3, 4 51. Which of the following questions would determine
D. 1, 2, 3, 4 the construct validity of the instrument?
A. “What is this instrument really measuring?”
Answer: (D) 1, 2, 3, 4 B. “How representative are the questions on this test of
The purposes of a research study covers all the options the universe of questions on this topic?”
indicated. C. “Does the question asked looks as though it is
measuring the appropriate construct?”
48. She opts to use interviews in data collection. In D. “Does the instrument correlate highly with an
addition to validity, what is the other MOST serious external criterion?
weakness of this method?
A. Accuracy Answer: (A) “What is this instrument really measuring?”
B. Sensitivity Construct validity aims to validate what the instrument is
C. Objectivity really measuring. The more abstract the concept, the
D. Reliability more difficult to measure the construct.

Answer: (A) Accuracy 52. Which of the following experimental research


Accuracy and validity are the most serious weaknesses designs would be appropriate for this study if she wants
of the self-report data. This is due to the fact that the to find out a cause and effect relationship between the
respondents sometimes do not want to tell the truth for structured discharge plan and compliance to home care
fear of being rejected or in order to please the regimen among the subjects?
interviewer. A. True experiment
B. Quasi experiment
49. She plans to subject her instrument to pretesting. C. Post-test only design
Which of the following is NOT achieved in doing D. Solomon four-group
pretesting?
A. Determines how much time it takes to administer Answer: (C) Post-test only design
the instrument package Post- Test only design is appropriate because it is
B. Identify parts that are difficult to read or understand impossible to measure the compliance to home care
C. Determine the budgetary allocation for the study regimen variable prior to the discharge of the patient
D. Determine if the measures yield data with sufficient from the hospital.
variability

53
54
53. One hypothesis that she formulated is “Compliance people refuse to be interviewed in person.
to home care regimen is greater among patients who
received the structured discharge plan than those who 57. Alyssa reads about exploratory research. Which
received verbal discharge instructions.’ Which is the of the following is the purpose of doing this type of
independent variable in this study? research?
A. Structured discharge plan A. Inductively develops a theory based on observations
B. Compliance to home care regimen about processes involving selected people
C. Post-open heart surgery patients B. Makes new knowledge useful and practical.
D. Greater compliance C. Identifies the variables in the study
D. Finds out the cause and effect relationship between
Answer: (A) Structured discharge plan variables
Structured discharge plan is the intervention or the
‘cause’ in the study that results to an ‘effect’, which is Answer: (C) Identifies the variables in the study
compliance to home care regimen or the dependent Exploratory research is the first level of investigation and
variable. it deals with identifying the variables in the study.

54. Situation : Alyssa plans to conduct a study about 58. She reviews qualitative design of research. Which of
nursing practice in the country. She decides to refresh the following is true about ethnographic study?
her knowledge about the different types of research in A. Develops theories that increase the knowledge about
order to choose the most appropriate design for her a certain phenomenon.
study. B. Focuses on the meanings of life experiences of people
C. Deals with patterns and experiences of a defined
55. She came across surveys, like the Social Weather cultural group in a holistic fashion
Station and Pulse Asia Survey. Which of the following is D. In-depth investigation of a single entity
the purpose of this kind of research?
A. Obtains information regarding the prevalence, Answer: (C) Deals with patterns and experiences of a
distribution and interrelationships of variables within a defined cultural group in a holistic fashion
population at a particular time Ethnographic research deals with the cultural patterns
B. Get an accurate and complete data about a and beliefs of certain culture groups.
phenomenon.
C. Develop a tool for data gathering. 59. She knows that the purpose of doing
D. Formulate a framework for the study ethnographic study is to:
A. Understand the worldview of a cultural group
Answer: (A) Obtains information regarding the B. Study the life experiences of people
prevalence, distribution and interrelationships of C. Determine the relationship between variables
variables within a population at a particular time D. Investigate intensively a single entity
Surveys are done to gather information on people’s
actions, knowledge, intentions, opinions and attitudes. Answer: (A) Understand the worldview of a cultural
group
56. She will likely use self-report method. Which of the The aim of ethnographers is to learn from the members
following self-report methods is the most respected of a cultural group by understanding their way of life as
method used in surveys? they perceive and live it.
A. Personal interviews
B. Questionnaires 60. Alyssa wants to learn more about
C. Telephone interviews experimental design. Which is the purpose of this
D. Rating Scale research?
A. Test the cause and effect relationship among the
Answer: (A) Personal interviews variable under a controlled situation
Personal interviews is the best method of collecting B. Identify the variables in the study
survey data because the quality of information they yield C. Predicts the future based on current intervention
is higher than other methods and because relatively few D. Describe the characteristics, opinions, attitudes or
behaviors of certain population about a current issue or
event
54
54

variables in the present which is linked to a variable that


Answer: (A) Test the cause and effect relationship occurred in the past?
among the variable under a controlled situation A. Prospective design
Experimental research is a Level III investigation which B. Retrospective design
determines the cause and effect relationship between C. Cross sectional study
variables. D. Longitudinal study

61. She knows that there are three elements of Answer: (B) Retrospective design
experimental research. Which is NOT included? Retrospective studies are done in order to establish a
A. Manipulation correlation between present variables and the
B. Randomization antecedent factors that have caused it.
C. Control
D. Trial 65. Situation : Harry a new research staff of the Research
and Development Department of a tertiary hospital is
Answer: (D) Trial tasked to conduct a research study about the increased
Trial is not an element of experimental research. incidence of nosocomial infection in the hospital.
Manipulation of variables, randomization and control are
the three elements of this type of research Which of the following ethical issues should he consider
in the conduct of his study?
62. Alyssa knows that there are times when only
manipulation of study variables is possible and the 1. Confidentiality of information given to him by the
elements of control or randomization are not attendant. subjects
Which type of research is referred to this? 2. Self-determination which includes the right to
A. Field study withdraw from the study group
B. Quasi-experiment 3. Privacy or the right not to be exposed publicly
C. Solomon-Four group design 4. Full disclosure about the study to be conducted
D. Post-test only design A. 1, 2, 3
B. 1, 3, 4
Answer: (B) Quasi-experiment C. 2, 3, 4
Quasi-experiment is done when randomization and D. 1, 2, 3, 4
control of the variables are not possible.
Answer: (D) 1, 2, 3, 4
63. One of the related studies that she reads is a This includes all the options as these are the four basic
phenomenological research. Which of the following rights of subjects for research.
questions is answered by this type of qualitative
research? 66. Which of the following is the best tool for data
A. ” What is the way of life of this cultural group?” gathering?
B. “What is the effect of the intervention to the A. Interview schedule
dependent variable?” B. Questionnaire
C. “What the essence of the phenomenon is as C. Use of laboratory data.
experienced by these people?” D. Observation
D. “What is the core category that is central in explaining
what is going on in that social scene?” Answer: (C) Use of laboratory data.
Incidence of nosocomial infection is best collected
Answer: (C) “What the essence of the phenomenon is as through the use of biophysiologic measures, particularly
experienced by these people?” in vitro measurements, hence laboratory data is
Phenomenological research deals with the meaning of essential.
experiences as those who experienced the phenomenon
understand it. 67. During data collection, Harry encounters a patient
who refuses to talk to him. Which of the following is a
64. Other studies are categorized according to the time limitation of the study?
frame. Which of the following refers to a study of A. Patient’s refusal to fully divulge information.

54
54
B. Patients with history of fever and cough A. MEDLINE
C. Patients admitted or who seeks consultation at the ER B. National Institute of Nursing Research
and doctors offices C. American Journal of Nursing
D. Contacts of patients with history of fever and cough D. International Council of Nurses

Answer: (A) Patient’s refusal to fully divulge information. Answer: (B) National Institute of Nursing Research
Patient’s refusal to divulge information is a limitation National Institute for Nursing Research is a useful source
because it is beyond the control of Harry. of information for nursing research. The rest of the
options may be helpful but NINR is the most useful site
68. What type of research is appropriate for this study? for nurses.
A. Descriptive- correlational
B. Experiment 72. He develops methods for data gathering. Which of
C. Quasi-experiment the following criteria of a good instrument refers to the
D. Historical ability of the instrument to yield the same results upon
its repeated administration?
Answer: (A) Descriptive- correlational A. Validity
Descriptive- correlational study is the most appropriate B. Specificity
for this study because it studies the variables that could C. Sensitivity
be the antecedents of the increased incidence of D. Reliability
nosocomial infection.
Answer: (D) Reliability
69. In the statement, “Frequent hand washing of health Reliability is consistency of the research instrument. It
workers decreases the incidence of nosocomial refers to the repeatability of the instrument in extracting
infections among post-surgery patients”, the dependent the same responses upon its repeated administration.
variable is
A. incidence of nosocomial infections 73. Harry is aware of the importance of controlling
B. decreases threats to internal validity for experimental research,
C. frequent hand washing which include the following examples EXCEPT:
D. post-surgery patients A. History
B. Maturation
Answer: (A) incidence of nosocomial infections C. Attrition
The dependent variable is the incidence of nosocomial D. Design
infection, which is the outcome or effect of the
independent variable, frequent hand washing. Answer: (D) Design
Design is not a threat to internal validity of the
70. Harry knows that he has to protect the rights of instrument just like the other options.
human research subjects. Which of the following actions
of Harry ensures anonymity? 74. His colleague asks about the external validity of
A. Keep the identities of the subject secret the research findings. Which of the responses of Harry
B. Obtain informed consent is appropriate? The research findings can be
C. Provide equal treatment to all the subjects of the A. generalized to other settings or samples
study. B. shown to result only from the effect of the
D. Release findings only to the participants of the study independent variable
C. reflected as results of extraneous variables
Answer: (A) Keep the identities of the subject secret D. free of selection biases
Keeping the identities of the research subject secret will
ensure anonymity because this will hinder providing link Answer: (A) generalized to other settings or samples
between the information given to whoever is its source. External validity refers to the generalizability of research
findings to other settings or samples. This is an issue of
71. He is oriented to the use of electronic databases for importance to evidence-based nursing practice.
nursing research. Which of the following will she likely
access?

54
54
NURSING RESEARCH Part 2 Situation 2: Survey and statistics are important part if
research that is necessary to explain the characteristics
Situation 1: You are fortunate to be chosen as part of of the population.
the research team in the hospital. A review of the
following IMPORTANT nursing concepts was made: 6. According to WHO statistics on the homeless
population around the world, which of the following
1. A professional nurse can do research for varied groups of people in the world disproportionately
reasons except: represents the homeless population?

a. Professional advancement through research a. Hispanics


participation b. Asians
b. To validate results of new nursing modalities c. African Americans
c. For financial gains d. Caucasians
d. To improve nursing care
7. All but one of the following in not a measure of
2. Each nurse participant was asked to identify a central tendency:
problem. After the identification of the research
problem, which of the following should be done? a. Mode
b. Variance
a. Methodology c. Standard deviation
b. Review of related literature d. Range
c. Acknowledgement
d. Formulate hypothesis 8. In the values: 87, 85, 88, 92, 90, what is the mean?

3. Which of the following communicate the results of a. 88.2


the research to the readers. They facilitate the b. 88.4
description of the data. c. 87
d. 90
a. Hypothesis
b. Statistics 9. In the values: 80, 80, 80, 82, 82, 90, 90, 100, what is
c. Research problem the mode?
d. Tables and graphs
a. 80
4. In quantitative data, which of the following is b. 82
described as the distance in the scoring units of the c. 90
variable from the highest to the lower? d. 85.5

a. Frequency 10. In the values: 80, 80, 10, 10, 25, 65, 100, 200, what is
b. Mean the median?
c. Median
d. Range a. 71.25
b. 22.5
5. This expresses the variability of the data in reference c. 10 and 25
to the mean. It provides as with a numerical d. 72.5
estimate of how far, on the average the separate
observation are from the mean: 11. Draw lots, lottery, table of random numbers or a
sampling that ensures that each element of the
a. Mode population has an equal and independent chance of
b. Standard deviation being chosen is called:
c. Median
d. Frequency a. Cluster
b. Simple

54
54
c. Stratified updated on the latest trends and issues affecting the
d. Systematic profession and the best practices arrived at by the
profession.
12. An investigator wants to determine some of the
problems experienced by diabetic clients when using 16. You are interested to study the effects of mediation
insulin pump. The investigator went to a clinic where and relaxation on the pain experienced by cancer
he personally knows several diabetic clients having patients. What type of variable is pain?
problem with insulin pump. The type of sampling
done by the investigator is called: a. Dependent
b. Correlational
a. Probability c. Independent
b. Purposive d. Demographic
c. Snowball
d. Incidental 17. You would like to compare the support system of
patient with chronic illness and those with acute
13. If the researcher implemented a new structured illness. How will you best state your problem?
counseling program with a randomized group of
subject and a routine counseling program with a. A descriptive study to compare the support
another randomized group of subject, the research is systems of patients with chronic illness and
utilizing which design? those with acute illness in terms of demographic
data and knowledge about intervention
a. Quasi experimental b. The effects of the types of support system of
b. Experimental patients with chronic illness and those with
c. Comparative acute illness
d. Methodological c. A comparative analysis of the support system of
patients with chronic illness and those with
14. Which of the following is not true about a pure acute illness
experimental research? d. A study to compare the support system of
patients with chronic illness and those with
a. There is a control group acute illness
b. There is an experimental group e. What are the differences of the support system
c. Selection of subjects in the control group is being received by patient with chronic illness
randomized and patients with acute illness?
d. There is a careful selection of subjects in the
experimental group 18. You would like to compare the support system of
patients with chronic illness to those with acute
15. The researcher implemented a medication regimen illness. Considering that the hypothesis was: “Clients
using a new type of combination drugs to manic with chronic illness have lesser support system than
patients while another group of manic patients clients with acute illness.” What type of research is
receives the routine drugs. The researcher however this?
handpicked the experimental group for they are the
clients with multiple episodes if bipolar disorder. The a. Descriptive
researcher utilized which research design? b. Correlational, non experimental
c. Experimental
a. Quasi experimental d. Quasi experimental
b. Pure experimental
c. Phenomenological 19. In any research study where individual persons are
d. Longitudinal involved, it is important that an informed consent of
the study is obtained. The following are essential
Situation 3: As a nurse, you are expected to participate information about the consent that you should
in initiating or participating in the conduct of research disclose to the prospective subjects except:
studies to improve nursing practice. You have to be

54
54
a. Consent to incomplete disclosure d. A study measuring differences in blood pressure
b. Description of benefits, risks, and discomforts before, during and after a procedure
c. Explanation of procedure
d. Assurance of anonymity and confidentiality 24. An 85 year old client in a nursing home tells a nurse,
“I signed the papers for that research study because
20. In the hypothesis: “The utilization of technology in the doctor was so insistent and I want him to
teaching improves the retention and attention of the continue taking care of me”. Which client right is
nursing students,” which is the dependent variable? being violated?

a. Utilization of technology a. Right of self determination


b. Improvement in the retention and attention b. Right to privacy and confidentiality
c. Nursing students c. Right to full disclosure
d. Teaching d. Right not to be harmed

Situation 4: You are an actively practicing nurse who has 25. “A supposition or system of ideas that is proposed to
just finished your graduate studies. You learned the explain a given phenomenon”, best defines:
value of research and would like to utilize the knowledge
and skills gained in the application of research to the a. A paradigm
nursing service. The following questions apply to b. A concept
research. c. A theory
d. A conceptual framework
21. Which type of research inquiry investigates the
issues of human complexity (e.g. understanding the Situation 5: Mastery of research design determination is
human expertise)? essential in passing the NLE.

a. Logical position 26. Monette wants to know if the length of time she will
b. Naturalistic inquiry study for the board examination is proportional to
c. Positivism her board rating. During the December 2007 board
d. Quantitative Research examination, she studied for six months and gained
68%. On June 2008 board exam, she studied for 6
22. Which of the following studies is based on months again for a total of one year and gained 74%.
quantitative research? On November 2008, she studied for 6 months for a
total of one and a half year and gained 82%. The
a. A study examining the bereavement process in research design she used is:
spouses of clients with terminal cancer
b. A study exploring factors influencing weight a. Comparative
control behavior. b. Correlational
c. A study measuring the effects of sleep c. Experimental
deprivation on wound healing d. Qualitative
d. A study examining client’s feeling before, during
and after a bone marrow aspiration 27. Rodrigo was always eating high fat diet. You want to
determine if what will be the effect of high
23. Which of the following studies is based on cholesterol food to Rodrigo in the next 10 years. You
qualitative research? will use:

a. A study examining clients reactions to stress a. Comparative


after open heart surgery b. Correlational
b. A study measuring nutrition and weight loss/gain c. Historical
in clients with cancer d. Longitudinal
c. A study examining oxygen levels after
endotracheal suctioning 28. Community A was selected randomly as well as
Community B, nurse Crystal conducted teaching to

54
54
Community A and assessed if Community A will have these people. They will best use which research
a better status than Community B. This is an design?
example of:
a. Historical
a. Comparative b. Case study
b. Correlational c. Phenomenological
c. Experimental d. Ethnographic
d. Qualitative
34. Jezza and Jenny researched about TB – its
29. Faye researched in the development of a new way to transmission, causative agent and factors,
measure intelligence by creating a 100-item treatment, signs and symptoms, as well as
questionnaire that will assess the cognitive skills of medication and all other in-depth information about
an individual. The design best suited for this study is: tuberculosis. This study is best suited for which
research design?
a. Historical
b. Methodological a. Historical
c. Survey b. Case study
d. Case study c. Phenomenological
d. Ethnographic
30. Jay Emmanuelle is conducting a research study on
how Ralph, an AIDS client lives his life. A design 35. Diana, Arlene, and Sally are to conduct a study about
suited for this is: relationship of the number of family members in the
household and the electricity bill, which of the
a. Historical following is the best research design suited for this
b. Case study study?
c. Phenomenological
d. Ethnographic 1. Descriptive
2. Exploratory
31. Maecee is to perform a study about how nurses 3. Explanatory
perform surgical asepsis during World War II. A 4. Correlational
design best for this study is: 5. Comparative
6. Experimental
a. Historical
b. Case study a. 1 and 4
c. Phenomenological b. 2 and 5
d. Ethnographic c. 3 and 6
d. 1 and 5
32. Medel conducts sampling at Barangay Maligaya. He e. 2 and 4
collected 100 random individuals and determine
who is their favorite actor. 50% said Piolo, 20% said Situation 6: As a nurse researcher, Vinz must have a very
John Lloyd, while some answered Sam, Dingdong, good understanding of the common terms of concept
Richard, and Derek. Medel conducted what type of used in research.
research study?
36. The information that an investigator like Vinz
a. Phenomenological collects from the subjects or participants in a
b. Case study research study is usually called:
c. Non experimental
d. Survey a. Hypothesis
b. Data
33. Mark and Toberts visited a tribe located somewhere c. Variable
in China, it is called Shin Jea tribe. They studied the d. Concept
way of life, tradition, and the societal structure of

54
54
37. Which of the following usually refers to the BBBBA
independent variables in doing research?

a. Result
b. Cause
c. Output
d. Effect

38. The recipients of experimental treatment in an


experimental design or the individuals to be
observed in a non-experimental design are called:

a. Setting
b. Subjects
c. Treatment
d. Sample

39. The device or techniques that Vinz employs to


collect data is called:

a. Sample
b. Instrument
c. Hypothesis
d. Concept

40. The use of another person’s ideas or wordings


without giving appropriate credit results from
inaccurate or incomplete attribution of materials to
its resources. Which of the following is referred to
when another person’s idea is inappropriately
credited as one’s own?

a. Plagiarism
b. Quotation
c. Assumption
d. Paraphrase

Nursing Research Suggested Answer Key

CBDDB
BABAD
BBBDA
AEAAB
BCAAC
BDCBC
ADDBD
54

You might also like